You are on page 1of 1089

Keep in mind the NAC OSCE score plays an important role in whether or not you match into a

program in Canada. You should aim for a score above 90 on the OSCE to be competitive for the
Canadian programs. Most successful applicants in Canada also have very high MCCEE scores that
is at least two SD above the mean. This means a competitive score is around 375-450 for the
Canadian programs.
All Canadian programs have predetermined score cut offs on the MCCEE and OSCE and have
filters in place so that if you don't have the right scores your application won't even be reviewed.

In its current state, the NAC examination consists of 12 stations (of 11 minutes each) based on
clinical scenarios. The clinical scenarios sample from problems in Medicine, Pediatrics, Obstetrics
and Gynecology, Psychiatry and Surgery. Performance on each scenario is assessed on up to nine
possible competencies, including: history-taking, physical examination, organizational skills,
communication skills, language fluency, diagnosis, data interpretation, investigations,
management, and therapeutics.

http://mcc.ca/examinations/nac-overview/preparation-resources/

http://mcc.ca/examinations/nac-overview/competency-descriptors/

The NAC OSCE is a typical osce style exam. Its based on clinical encounters with standardized
patients for which you will be scored by an examiner using present rating scales. If you are
familiar with step 2 CS it ll be relatively easy for you. Scoring well on this exam depends on what
your current skills are. I think the most important is language, most IMG's have trouble
communicating properly because they don't have complete command over English. The next is
being familiar with the western style of conducting a medical interview and looking at things like
involving the patient in decision making, listening, empathy etc. The last and least important is
your knowledge. Most of the cases are generic and its only testing how you approach a station
rather than your knowledge in getting to a specific diagnosis. If you have never done any exam
like this before try to practice with someone who has given the exam before or take the IMG
courses that are offered. The only people I'd recommend the course to are people who are older
graduates, ve language difficulties, people who have never attempted an OSCE style exam before.

http://nac-osce-mccqe2-usmle-step2cs-clinical-skills-review-course.thinkific.com/courses/a-
beginners-guide-to-the-nac-osce-exam-preparation

https://mdconsultants.ca/5-expert-tips-to-do-well-on-the-mccqe-2-and-nac-osce/

https://www.youtube.com/watch?v=iQAArBOLwGE

https://www.youtube.com/watch?v=Ce51P9gbjGU

http://www.crackthenac.com/
Books: (Also, some good notes written by various course offering Institutes are also available)

1. NAC OSCE comprehensive review


2. Dr. Basil Notes- Nac Osce Vip
3. OSCE and Clinical Skills Handbook
4. OSCE material (Ontario IMG school material)
5. First Aid for the USMLE Step 2 CS
6. Clinical Skills Review: Scenarios Based on Standardized Patients
7. Master the NAC

 Canada QBank for the USMLE Step 2 CS (Free)

Prior to the examination, if you are unfamiliar with disease management practices in North
America, you may wish to consult the latest edition of Therapeutic Choices edited by Jean
Gray, published by the Canadian Pharmacists Association, as a source of general information.

Free recourse for OSCE learning stations

https://www.medistudents.com/en/learning/osce-skills/

You tube:

https://www.youtube.com/user/geekymedics123/videos

https://www.youtube.com/channel/UC8Si5Q9ObZH9I0-LZLB6KiA

https://www.youtube.com/user/medquarterly88/videos

https://www.youtube.com/user/ABCMedicine2012/videos

Other Resources:

https://sites.google.com/site/nacoscemontreal/home/study-materials/clinical-examination-
videos

http://www.oscehome.com/
http://www.oscehome.com/A-step-by-step-guide-to-mastering-the-OSCEs.html

https://www.gacguidelines.ca/index.cfm?id=21080&form_submitted=1

Course: Medical Training Express course or other live online courses e.g. Ontario IMG School
course or Focused Education centre

Making Online Group Study: http://www.rxpgonline.com/forum372.htm


Before the OSCE:

1. Prepare a checklist for each major organ system history and examination you are required to
cover
2. Have a structure for how you approach histories (e.g. Identifying Information, Chief Complaint,
History of Presenting Illness, Associated Symptoms, Medications, Past Medical History, Allergies,
and a small section for Differential Diagnosis to guide your questioning).
3. Having a Focused History: Taking a good history is more than just having a checklist of questions.
This is really difficult to do as a first year student unless you’ve had a lot of clinical exposure and
practise with patients. This takes a lot of practice. The goal of the history is to rule in or rule out
what is in your differential diagnosis (a list of diagnosis that may explain the patient’s chief
complaint). Make a list of important Associated Symptoms and Differential Diagnosis that is
important to cover with certain major organ systems. For example: If your OSCE scenario is “68-
year-old male who presents with shortness of breath”, think of what organ systems cause
shortness of breath – is respiratory (ventilation/perfusion mismatch?), cardiovascular (congestive
heart failure), metabolic (metabolic acidosis), muscular (muscular dystropy), hematological
(anemia), neurologic (cancerous mass impinging on the brain’s central breathing centre”).
4. Physical Examination: Also have a systematic, and structured technique for each physical
examination you have prepared for the OSCE. For example, the cardiovascular exam, apart from
IPPA (inspection, palpation, percussion and auscultation), include Hepatojugular reflex, height of
JVP, presence of pitting edema, etc.
5. Practice with your Classmates: Google “Sample OSCE Scenarios” and print some sheets off. With
a few of your PA classmates (or family members and boyfriends or girlfriends, or even pets), re-
enact the OSCE evaluation environment. Give yourself 2 minutes to read the scenario and 8
minutes to perform it.
6. Counseling Patients: Mental health is no exception to the OSCE scenarios. Be prepared to
interview the depressed patient, the grieving patient, the seductive patient (yes, they have OSCE
scenarios where that does happen) or the angry/irritable patient. In cases of psych, always ask
about suicide, supports, hallucinations/delusions, mood. If you get stuck, it never hurts to ask
“How does that make you feel?”

Top 10 tips for CLINICAL CASE SCENARIO preparation: Duration of preparation: 3 months
1. Start preparation early. Practice daily for at least 2 hours, if you are working. Over the weekends
study for 4-6 hours atleast.
2. Do group study with 3-4 dedicated members. Smaller groups have better results.
3. Make a time table for the week & follow it strictly.
4. Communicate with each member of the group. Learn from each others mistakes & strengths. It is
better to make mistakes during preparation, than in the real exam.
5. Practice in front of the mirror at home or record your preparation, if you are studying alone.
6. Be professional & punctual while studying. Stick to the timing of 7 minutes per case every time you
practice from the beginning. You can use timers or stop watch for effective time management.
7. A typical case scenario should deal with one clinical topic only. Dont try to complicate a case by
adding multiple diagnosis.
example: Practice a case of bronchial asthma specifically. Dont complicate it with a past history of CCF or
domestic abuse.
8. Work on communication skills from day 1.
9. Minimize gossip time by concentrating on the goal ahead.
10. Practice! Practice! Practice! The more you practice, the better you will get.
NAC OSCE resources:

1. http://www.sickkids.ca/culturalcompetence/elearning-modules/eLearning-
modules.html - Important site developed by Sickkid hospital for culture and
communication skills – free online course
2. http://www.usmle.org/pdfs/step-2-ck/2012--13_FINAL_S2_GSI.pdf

3. An excellent source for Canadian Clinical Practice Guidelines


http://www.gacguidelines.ca/

4. A link for the Bio-ethics- http://www.cmaj.ca/content/178/1/116.full?sid=687bd4e6-


ce84-425f-a74f-eabad31d294f

5. www.usmleworld.com or www.nbme.org

6. http://www.ncbi.nlm.nih.gov/pmc/articles/PMC1235022/

7. http://www.osceskills.com/

8. Mastering the OSCE Case: Objective Structured Clinical Examination Clinical Skills
Assessment [Paperback] Jo-Ann Reteguiz, Beverly Cornel-Avendano

9. Fundamental Clinical Situations: A Practical OSCE Study Guide, 4e[Spiral-Bound]


Pieter J. Jugovic (Author), Richard Bitar, Laura C. McAdam

10. OSCE and Clinical Skills Handbook Katrina F. Hurley

(1) http://www.oscehome.com

OSCE home is designed by a group of medical students, residents, and graduate physicians
around the world who took and passed several OSCEs during their medical schools and
residencies in addition to licensure exams, specifically, USMLE Step 2 CS, LMCC MCCQE part
II, PLAB part 2, TRAS part 2, MRCGP, Australian Clinical, and NZREX.

(2) Book “NAC OSCE. A Comprehensive Review”

The book is a detailed study guide for the Canadian NAC OSCE. It contains therapeutic
guidelines with drug dosages, clinical case scenarios and clinical examinations. The book can be
used as a framework, complementing the clinical and therapeutic knowledge as one prepares for
the examination.

http://www.nacoscereview.com/
http://www.amazon.ca/Nac-OSCE-Comprehensive-Review-
Canadaprep/dp/146646416X/ref=sr_1_1?ie=UTF8&qid=1341501260&sr=8-1
Some free resources for NAC
OSCE exam preparation
(Some of the NAC OSCE Network Facebook Group posts)
Vahid Nilforushan M.D.
Contact information

© Vahid Nilforushan M.D. 1


Frequently asked questions about NAC OSCE
• If you are just starting preparation for the NAC OSCE exam, you may
have questions such as:
• 1. Is this the right time to register for the NAC OSCE?
• 2. When should I start preparation?
• 3. Where to start and what to do?
• 4. How to proceed?
• 5. Should I take a course?
• 6. When should I take a course?
• 7. Which course should I take?

© Vahid Nilforushan M.D. 2


FAQs about NAC OSCE (cont’d)
• If you are among those who ask these questions from themselves, you will
certainly benefit from using a beginner’s video guide for the NAC OSCE exam
preparation.
• It is a valuable resource which is currently offered as free. Although it is not
comprehensive, it contains a lot of practical and useful points that can help you
avoid major, costly and irreversible mistakes in your journey.
• Even those colleagues who had already taken paid courses or had passed the NAC
OSCE or the MCCQE2 exam have found new and valuable points in this course as
the points discussed in the course are hardly available elsewhere.
• It contains 8 videos, a guideline, a checklist and a survey. The videos are about 1
hour long in total.
• The successful applicants are those who are serious, are concerned about their
results and complete the guide early in their preparation and apply its
instructions. Unfortunately, those who take it lightly or do not complete it, do not
usually get good results.

© Vahid Nilforushan M.D. 3


Focused approach
Conducting a FOCUSED history or physical examination is one of the most
important factors that contributes to a high score in NAC OSCE. The
importance of being FOCUSED is explained on the last paragraph of the
following link from the Medical Council of Canada website. It needs to be
read very CAREFULLY.
http://mcc.ca/examinations/nac-overview/osce-station-description/

N.B: A FOCUSED approach could be regarded as an approach which helps


you in making a diagnosis and ruling out some of the probable DDx and
complications for the CURRENT OSCE station

© Vahid Nilforushan M.D. 4


Focused approach: Two resources for DDx
There are two useful resources that can help you with DDx in any clinical
(NAC OSCE, MCCQE2, USMLE step 2 CS) or even non-clinical examination as
well as the real-world clinical encounters:

• 1/ Objectives for the qualifying examination accessible


at: http://apps.mcc.ca/Objectives_Online/objectives.pl?lang=english&loc=c
ontents

• 2/ First aid for the USMLE step 2 CS (book). Review mainly the mini-cases
section of the book. You may want to review the patient note page in the
practice cases section as well. It does not cover all the sections in the
Objectives (first resource), so you may miss some points that may be tested
in the Canadian exams if you rely solely on this reference. However, in
general it is an acceptable resource for a quick review of DDx in a short
period of time.
© Vahid Nilforushan M.D. 5
Focused approach: How to recall DDx
Recalling DDx, is one the most important keys to success in any clinical skills
exam or real clinical encounter. Writing DDx along with diagnostic studies on
flash cards or excel spreadsheets (from resources mentioned in previous
posts) could be very helpful for quick review and practice. Preparing excel
spreadsheets are specially useful, since you can review them on your cell
phone wherever you are. However, it takes time and may interfere with your
plans if your exam is scheduled for the near future.

Despite using the above strategy, the stress of the exam may still pose a
problem in recalling some of the DDx on the big day! In that case, there are
still two other strategies that can help you in most clinical encounters:

© Vahid Nilforushan M.D. 6


Focused approach: How to recall DDx (cont’d)
• 1- Use an anatomical approach. Think of the closest systems to c.c
followed by the furthest ones. Try to think of diseases in each of those
systems that could cause the c.c. For example for chest pain think of
diseases in Cardiovascular, Pulmonary, GI, Musculoskeletal and Nervous
system that could cause chest pain.
• 2- Use VITAMINS C or any other useful mnemonics. V= vascular,
I=infectious, T=traumatic, A=allergic, autoimmune, M=metabolic,
I=iatrogenic, idiopathic, N=neoplastic, S=substance abuse, psychogenic,
C=Congenital. For example for chest pain we can think of vascular,
infectious, traumatic, substance abuse or psychogenic causes. Keep in mind
that some of the questions that we routinely ask the patients (for example
h/o recreational drug use) is included in VITAMINS C.
Although the above strategies are very helpful, they may not be useful in all
the clinical encounters or may not cover all the causes.
© Vahid Nilforushan M.D. 7
Physical exam: an online resource
One of the useful free online resources for preparation for the physical
examination component of clinical skills exam is Geekymedics:
http://geekymedics.com/category/osce/page/3/. There are a number of free
videos regarding different parts of the PE.
The videos are valuable for IMGs who are looking for English phrases that
could be useful in communication with SP and examiner in OSCEs. They are
also a good resource for reviewing the signs of the diseases, as the signs are
displayed throughout the videos.
However, they may not be a perfect resource. The videos are
comprehensive and demonstrate a very systematic approach for PE. For
example, a video regarding abdominal examination starts with the
examination of hands, arms and oral cavity for signs of diseases related to GI
tract (Cirrhosis, IBD, …). Since in most clinical skills exams, a FOCUSED
approach is expected, following the same pattern may not lead to a high
score. Although we should try to be as systematic as possible, a better
approach for PE in NAC OSCE might be:
© Vahid Nilforushan M.D. 8
Physical exam: an online resource (cont’d)
• 1/Considering the patient’s age, sex, the clinical setting (office, emergency
room,…), Hx (if available), to narrow down the DDx, and so the PE.
• 2/Quickly noting the general appearance of the patient.
• 3/Asking the examiner for the VS. (if not posted on the doorway
instructions)
• 4/Performing a systematic examination of the abdomen (inspection,
auscultation, percussion and palpation), including any special tests needed,
followed by the CVA.
• 5/Mentioning the need for DRE and/or inguinal, genital or pelvic exam to
the examiner if appropriate.
• 6/At the end, if there is any time left, performing the other and less
important parts of the exam for the current clinical encounter.

© Vahid Nilforushan M.D. 9


Physical exam: an online resource (cont’d)
Although it may not seem a totally systematic approach, you will cover all
the NECESSARY parts that you need to do and are assessed for. This
approach is especially important for the combined Hx and PE stations.
Indeed, a FOCUSED approach is an approach which helps you in making a
diagnosis and ruling out some of the probable DDx and complications for the
CURRENT OSCE station.
Regarding the videos, you may need to modify some of the phrases as well.
(for example, telling the patient that I am going to examine your cranial
nerves is a use of medical jargon). You may also find some other resources
better in performing some parts of the PE.
In brief, although geekymedics might not be the best resource for the PE
component of the NAC OSCE, with some modifications you could find it
useful. If you are looking for a perfect resource for PE for the exam, you may
consider taking this course (or only the PE component of that course by
contacting its instructor).
• N.B: If you find it difficult to change the position of the examination table
for a part of PE, IMMEDIATELY ask the examiner for help to save your
valuable time.
http://geekymedics.com/category/osce/page/3/ © Vahid Nilforushan M.D.
10
Physical exam: an online resource for MSK
British Journal of Sports Medicine contains a collection of useful videos
regarding physical examination of the musculoskeletal (MSK) system. In this
collection, the videos presented by Dr. Hutchinson are especially helpful in
demonstrating the routine and special tests of the shoulder, hip and knee.
The videos are accessible here:
https://www.youtube.com/user/BJSMVideos/playlists
• N.B: In the acute phase of traumatic injuries, performing some of the
routine or special tests of the MSK system may worsen the injuries or may
not be tolerable by the patient. Thus, if the clinical scenario represents an
acute traumatic injury, a thoughtful, selective and modified approach
should be used for the physical examination of the MSK system.
© Vahid Nilforushan M.D. 11
Communication skills: an online resource
Doccom is a high quality resource for development of communication and
interpersonal skills in clinical encounters. It contains 42 modules containing
texts and videos. Although most of the modules need paid subscription, four
of the modules are free of charge.
The modules range from core interviewing skills to advanced competencies.
Although the level of communication skills presented in Doccom modules is
generally higher than what expected from examinees in NAC OSCE, the
modules are useful and demonstrate a patient-centered approach in clinical
encounters.
The free modules consist of: Build a Relationship, Responding to Strong
Emotions, Giving Bad News and Effective Clinical Teaching are accessible
here:
https://webcampus.drexelmed.edu/doccom/user/individual_logon_1.asp#fr
eeModules
© Vahid Nilforushan M.D. 12
Assessment and Management: an online
resource
"BC Guidelines" is a very useful resource with over 50 evidence-based
clinical guidelines that provides recommendations to BC physicians on
delivering high quality care to patients.
It contains important information regarding evaluation and management of
different and common clinical conditions such as acute chest pain, febrile
seizure, osteoporosis and abnormal liver chemistry.
It would be helpful to review the table of contents and select a few
important guidelines and read them in more depth before any medical
examination.
You can browse the guidelines by topic area or download the entire
collection of BC guidelines at:
http://www2.gov.bc.ca/gov/content/health/practitioner-professional-
resources/bc-guidelines

© Vahid Nilforushan M.D. 13


Assessment and Management: an online
resource
There are 4 important Clinical Decision Rules regarding ordering an imaging
study in Canada known as: Canadian CT Head Rule, Canadian C-Spine Rule,
Ottawa Ankle Rules and Ottawa Knee Rule.
They help physicians to standardize care of the patients and to be much
more selective in the use of CT and radiography.
They are accessible at the following link:
http://www.emottawa.ca/eng/70-publications_presentations.html

© Vahid Nilforushan M.D. 14


A survey to identify the role of different
factors in success in NAC OSCE
As we know, there are a variety of factors that can contribute to obtaining a
high score in NAC OSCE exam. However, NO QUANTITATIVE DATA is
available that shows the IMPORTANCE AND ROLE OF EACH FACTOR in
success in the NAC OSCE. This leads to more stress and less predictability in
the outcome of NAC OSCE.
To address this issue, I have designed A SURVEY to identify the factors that
have historically been associated with better NAC OSCE results. This will
help the future applicants to have a better understanding of the importance
of each variable in their success, so that they can focus their efforts on the
more important factors and will have a better estimate of their success
before taking the NAC OSCE.

© Vahid Nilforushan M.D. 15


A survey to identify the role of different
factors in success in NAC OSCE (cont’d)
Please take the opportunity to complete this survey after you take your NAC
OSCE. It will take approximately 5 minutes of your time. Your contributions
to this effort are greatly appreciated.
Please also share the survey link with colleagues who might be interested in
this effort.
All responses are kept confidential and are anonymous. For further
information, please go to the survey link at:
https://goo.gl/forms/8hQ8g38AX6vBKv962

© Vahid Nilforushan M.D. 16


Good luck!

Vahid Nilforushan M.D.


Email: vnilforushan@gmail.com

© Vahid Nilforushan M.D. 17


CANADIAN IMGS
GUIDE TO OSCE AND Hanan Ahmed

PRACTICE
A practical guide for International Medical Graduates who wish to pursue
Medical Residency Training in Canada

1
2
Author and publisher; Hanan Ahmed, M.B.B.S
Internal Medicine resident, PGY3
University of Alberta

Canadian IMGs Guide to OSCE and Practice

ISBN 978-0-9947342-0-4

© 2015 Hanan Ahmed


All rights reserved. This book or any portion thereof
may not be reproduced or used in any manner
whatsoever without the express written permission of
the publisher except for the use of brief quotations for
scholarly purposes.

3
Disclaimer

This book is designed as a study guide for Immigrant and


Canadian International Medical Graduates to give them
an idea of the expectation of Canadian OSCE exams. It
was not written to provide advice on patient care or
management, and is not intended as a comprehensive
medical resource. Consultation with appropriate
resources is suggested when treating patients. The
author and editors are not responsible for any harm,
direct or indirect that results from the application of this
material to patient care.
Patient’s names used in this book are arbitrary. Any
similarity to reality is a complete coincidence.

4
Forward

To Doctors who move to Canada with a strong will


to succeed

5
Acknowledgment

My heartfelt gratitude to all people who supported this project. Many thanks to
my Program Director Dr. Darryl Rolfson for his encouragement, trust and support
throughout the whole process. Thanks also to Dr. Peter Hamilton for his help and
guidance. My appreciation to Dr. Liam Rourke for supporting the initial phases of
this work. Many thanks to the editors who volunteered their time and effort; Dr.
Sara Belga, Dr. David Ross, Dr. Shirley Schipper, Dr. Jay Shavadia, Dr. Subrata
Datta, Dr. Stephanie Keeling. Dr. Fraulin Morales, Dr. Anca Tapardel, Dr. Sadik
Salman, Dr. Abbeir Hussain, Dr. Erica Paras, Dr. Erin Toor, Dr. James Yeung, Dr.
Wasif Hussain, Dr. Abdullah Saleh.

My deepest thanks go as well to Cayti Beyer, who extended her passion to help
IMGs and volunteered to do the language editing.

Finally I would like to express my deepest appreciation to my dear husband,


Ayman, for his patience, support and help with website design.

6
Preface

When I moved to Canada, I was surprised to learn that there are very little
resources for International Medical Graduates (IMGs). There isn’t even a single
book to orient IMGs. Witnessing the struggle many IMGs face in preparing for
Canadian exams due to the lack of study guides targeting the special needs of this
group of physicians, I was motivated to execute this work. This book is an
overview of the Canadian Objective Structured Clinical Examinations (OSCEs). The
goal of this book is to introduce you to the special aspects and expectations of
Canadian exams and provide resources and links that will help you study while
preparing for these exams. While the book helps you prepare for exams step by
step, it is also a useful resource to understand what is expected from you when
you start your training.

Chapter one covers the type and content of OSCE exams IMGs commonly write. It
details the structure of the exams and pin points high yield scoring points.

Chapter two is history taking. History taking is the most common scenario in
Canadian OSCEs. This chapter details the various history taking situations based
on a general patient population and can be used as a question bank to choose
from for focused histories as needed on a case by case basis.

Chapter three is physical exam. It details most possible physical exam scenarios
and what is expected from examinees. The special aspects, expectations and
depth of physical exam pertinent to Canadian OSCEs are presented in this
chapter.

Chapter four is writing and counselling tips. An overview of counselling is


presented in this chapter along with any scenarios in which you may be asked to
document

Chapter five is clinical cases. Written in interactive format to facilitate practice,


this chapter covers 18 common clinical cases designed to reflect the exam format

7
and give you an idea of its flow. The cases presented are common clinical
scenarios. Each case has a variation of the case section at the end, to help you
practice a wide range of differentials when assessing common presentations. You
can use them as a guide as you design a study approach to more clinical cases.

Chapter six is ethical and legal principles. It goes over the basic ethical
foundations you need to observe during your practice and which could be tested
in an OSCE scenario

You can use this study guide to prepare for the NAC OSCE and MCCQE2.

The content was reviewed for the most recent updates; however, I strongly
encourage you to go over MCC website regularly for updates regarding the
exams.

The best way to use this book is as a study guide along with other resources. The
material presented here is meant to give you an idea of how things are done in
Canada and what is expected from you in an exam setting. It is not a
comprehensive medical resource. Every effort was made to ensure the accuracy
in this study guide. However, this book was not designed for direct patient care.
Consultation with appropriate resources is strongly encouraged when treating
patients.

8
Section Editors
Chapter 1
Sara Belga, MD
Third Year Internal Medicine resident
University of Alberta

Chapter 2
Shirley Schipper, MD, CCFP
Associate Professor
Department of Family Medicine
University of Alberta

David Ross, MD, CCFP


Associate Professor
Department of Family Medicine
University of Alberta

Chapter 3
Anca Tapardel, MD, FRCPC
Assistant Clinical Professor
Department of Internal Medicine
University of Alberta

Chapter 4
Cardiovascular Physical exam
Jay Shavadia, MD
Division of Cardiology, PGY6
University of Alberta

Gastrointestinal and Respiratory physical exam


Erin Toor, MD, FRCPC
Division of General Internal Medicine
Department of Medicine
University of Alberta

9
Neurology physical exam
M. Wasif Hussain, MBBS
Adult Neurology resident PGY4
Department of Neurology
University of Alberta

Musculoskeletal physical exam


Stephanie Keeling, MD, FRCPC
Associate Professor
Division of Rheumatology
Department of Medicine
University of Alberta

Breast physical exam and Fundoscopy


Sadik Salman, MD, ABIM, FRCPC
General Internist
Department of Internal Medicine
University of Alberta

Chapter 5

Case 1 and 2
Subrata Datta, MD,
Division of Internal Medicine, PGY4
University of Alberta

Cases 3 and 4
Jay Shavadia, MD
Division of Cardiology, PGY6
University of Alberta

Cases 5 and 6
James Yeung, MD, FRCPC
Rheumatologist
Division of Rheumatology
Department of Medicine
University of Alberta

Cases 7, 8, 10 and 15

10
Sadik Salman, MD, ABIM, FRCPC
General Internist
Department of Internal Medicine
University of Alberta

Case 9
Erin Toor, MD, FRCPC
Division of General Internal Medicine
Department of Medicine
University of Alberta

Cases 11, 12, 13 and 14


Erica Paras, MD
Obstetrics and Gynecology Resident, PGY4
University of Alberta

Cases 16 and 17
Abbeir Hussein, M.D
Pediatrics Resident, PGY3
University of Alberta

Case 18
Abdullah Saleh, MD
General Surgery Resident. PGY5
University of Alberta

Chapter 6
Fraulin Morales, MD, FRCPC
Associate Clinical Professor
General Internal Medicine
University of Alberta

English language editor of the whole book;

Cayti Beyer
IMG Program Developer/Coordinator & Career Coach
Directions/Bow Valley College
Calgary, Alberta

11
TABLE OF CONTENTS

ABOUT THE EXAM ..................................................................................................... 17


Overview of OSCE Exams ................................................................................................................................ 18

What Does the Exam Cover?........................................................................................................................... 18

Interaction with the Standardized Patient (SP) ................................................................................................ 18

Interaction with the examiner ........................................................................................................................ 19

Interaction with the nurse/QE2 ...................................................................................................................... 19

Cases with extra family member ..................................................................................................................... 20

Structure of MCCQE2 ..................................................................................................................................... 20

Structure of NAC ............................................................................................................................................ 21

Scoring .......................................................................................................................................................... 21

HISTORY TAKING....................................................................................................... 25
Introduction .................................................................................................................................................. 26

Basic Elements of History ............................................................................................................................... 28

Special Populations: History ........................................................................................................................... 36

PHYSICAL EXAMINATION ........................................................................................ 46


Introduction .................................................................................................................................................. 47

Cardiovascular System Examination (CVS P.E) ................................................................................................. 49

Respiratory System examination: ................................................................................................................... 64

Abdominal Examination ................................................................................................................................. 66

Neurological P.E ............................................................................................................................................. 72

Musculoskeletal System Examination (MSK) ................................................................................................... 84

Breast Physical Examination; ........................................................................................................................ 105

13
Fundoscopy; ................................................................................................................................................ 107

WRITING AND COUNSELING TIPS....................................................................... 111


Introduction; ............................................................................................................................................... 112

Sample writing scenarios .............................................................................................................................. 112

Counseling; .................................................................................................................................................. 120

SAMPLE CLINICAL CASES....................................................................................... 125


Case 1; Cough .............................................................................................................................................. 128

Case 2; Fatigue ............................................................................................................................................. 135

Case 3; Chest pain in the ER .......................................................................................................................... 142

Case 4; Chest pain in an out patient .............................................................................................................. 153

Case 5; Back pain ......................................................................................................................................... 161

Case 6; Knee pain ......................................................................................................................................... 171

Case 7; Diabetic Ketoacidosis (DKA) .............................................................................................................. 179

Case 8; DM history and counseling................................................................................................................ 191

Case 9; Diarrhea ........................................................................................................................................... 202

Case 10; Smoking counseling ........................................................................................................................ 210

Case 11; Preeclampsia (PET) ......................................................................................................................... 222

Case 12; Contraception ................................................................................................................................ 233

Case 13; Postmenopausal bleeding ............................................................................................................... 245

Case 14; Abortion......................................................................................................................................... 255

Case 15; Cancer patient refusing treatment ................................................................................................... 265

Case 18; Motor vehicle accident (MVA) ......................................................................................................... 291

ETHICAL AND LEGAL PRINCIPLES ...................................................................... 302


Introduction; ............................................................................................................................................... 303

14
Confidentiality; ............................................................................................................................................ 304

Autonomy, capacity, informed consent, substitute decision maker and power of attorney; ............................ 305

Beneficence and non-maleficence;................................................................................................................ 307

Justice; ........................................................................................................................................................ 308

Dealing with ones’ own and others errors; .................................................................................................... 309

Abbreviations .............................................................................................................................................. 312

INDEX ........................................................................................................................... 313

15
Chapter 1
About the exam
Edited by; Dr. Sara Belga

17
Overview of OSCE Exams
OSCE stands for Objective structured Clinical Examination. Its purpose is to expose examinees to the
same set of clinical scenarios in order to guarantee fair evaluation of all participants. IMGs seeking
residency positions in Canada are required to pass one or more OSCEs. This book will familiarize you
with the structure, environment and medical topics encountered in these exams. I aim at providing a
complete guide to successful performance in OSCEs from A-Z, including many practice cases.

The exams may be made up of both clinical and written components. The clinical part corresponds to a
number of different cases. Each one of them starts with a clinical scenario posted on the door, asking
you to perform a specific task. When you enter the room you will find 2 people; the examiner and a
standardized patient (SP). You have a certain amount of time to perform your task (history and/or
physical exam and/or manage). Then you will move to the next station repeating through all 12 stations.

IMGs are required to do one or more Canadian OSCEs depending on the requirements of the province
they wish to practice in. These include: Medical Council of Canada Qualifying Exam 2 (MCCQE2) and
National Assessment Collaboration (NAC). Overview of the structure of each of them is presented in this
chapter. I encourage you to review the MCC website regularly as the exams’ structure may change.

What Does the Exam Cover?


You will be tested on common diseases in the major areas of medicine: Internal Medicine, Pediatrics,
Surgery, Obstetrics and Gynecology, Psychiatry, Preventive Medicine and Community Health.

You could be asked to do one or more of the following:

History taking

Physical Exam

Manage an emergency situation

Counsel a patient or address concerns about health issues

There may be a nurse in the room in the management stations. You will be scored on your interaction
with him or her.

In MCCQE2 you may be asked to fill in admission orders, write a progress (SOAP) note, a letter to the
employer or a prescription. There is no longer a written component to the NAC OSCE.

Interaction with the Standardized Patient (SP)

18
Standardized patients are actors and/or actresses who are paid to perform as patients in OSCE exams.
They will give you history and if required mimic physical findings. They are trained to give you a clear
history if you ask the right questions. Be aware that some of them have real findings like a scar from
previous surgery or a skin rash. They may also mimic real findings like tenderness or rebound tenderness
on abdominal examination. Be attentive and gentle during physical exam and report what you find.
Respect the SP as you would your own patients; you will be scored on how you interact with them.
Make sure you introduce yourself and address the SP by the name given on the door sign. Explain why
you need to ask sensitive or personal questions, wash your hands and respectfully drape SP during
physical examination. Examples on the variety of scenarios and challenges that you may encounter
when dealing with SPs are presented as cases in chapter 5. The SPs undergo intense training for these
high stakes OSCEs to ensure standardization across all exam centers.

Interaction with the examiner


You will be provided a note book with bar codes when you sign in. You will give each examiner one or
more bar code to stick on your scoring sheet. The examiner fills in the sheet based on his/her
observation of your performance. The number of bar codes you give to the examiner will be indicated in
the question stem. In general, you will always give the examiner one bar code except in the couplets of
MCCQE2 (see below for examination structure). Beyond that your interaction with the examiner is
limited to the following:

The examiner may tell you some findings on physical exam.

The examiner may stop you to ask you to re-read the question. This may happen if you are not doing the
specific task you are asked to do; for example, you may be taking history where the question is about
physical exam. Don’t panic if that happens, simply read the question again, and take few seconds if
needed to organize your thoughts.

The examiner may also stop you to protect the SP if you did something that made the SP uncomfortable.
Make sure you work in advance on your professional manners and treat SPs with utmost respect. If you
feel your interaction with the SP has gone off track, apologize to the SP and tell the examiner you didn’t
mean to make the SP uncomfortable and ask if you may continue.

The examiner may ask you questions, which are usually related to the clinical presentation you
just saw; questions may include differential diagnoses, most likely diagnosis,
management, investigations or response to ethical issues.

Interaction with the nurse/QE2


Introduce yourself and get to know the people around you (e.g. ask the nurse his or her name if he/she
doesn’t volunteer that information). The nurse will only do the tasks you ask him or her to do. In

19
emergency stations, he or she will not give oxygen and put the patient on the monitor unless you ask
him or her to do so. Provide clear and direct instructions. Avoid overwhelming the nurse by giving
multiple requests at the same time, wait until he or she completes them all before you tell him or her
what to do next. An example of an interaction with a nurse is presented in chapter 5.

Cases with extra family member


In some cases, especially when the patient has delirium/dementia or sometimes in pediatrics, you will
be asked to interact with one or more family members. Do the task you are asked to perform. If it is
history then stick to it, but make sure to apply basic ethical principles of autonomy and confidentiality
and state that you would like to speak to the patient when possible. If the case involves a teenager and
parents, respectfully ask the parents to leave the room when asking sensitive personal questions. If the
patient is a young child (<10 years) try to involve him/her in the conversation as much as you can. If the
case involves both partners always ask your patient if it’s ok for partner to stay.

Structure of MCCQE2
Consists of two sessions over the course of two days:

The first session is held on a Saturday and is comprised of 8 stations with each being 10 minute long +
two rest stations. In all of them you are interacting with a SP. You have 2 minutes to read the
instructions on the door and jog down some notes. Then you have 10 minutes to interact with the SP. At
9 minutes, a buzzer will sound to give you a warning. At 10 minutes, a second buzzer indicates the end
of the station and you have to exit the room. The examiner may ask you one or two quick questions.
Please note that this will be indicated in the question stem posted on the door and that the examiner
will ask the questions at the 9 minute buzzer; therefore, you need to make sure you are finished with
the SP at 9 minutes. The second session is held on a Sunday and is comprised of:

4 stations with a mix of 6 + 6 couplet stations. The sessions alternate between patient encounter and
written stations. You have 2 minutes to read the instructions and jog down some notes, and then you
have 6 minutes to interact with the patient. A buzzer will sound at 5 minutes indicating that you need to
wrap up. At the second buzzer after 6 minutes, the examiner will hand you the post encounter probe
(written component) and you must exit the room to complete it. Keep in mind that you may be asked
questions at the 5 minute buzzer and this will be indicated on the question stem. The written
component is similar except that you will be writing (Examples are given in chapter 4).

One 14 minute station; focused on assessing more complex cases which could involve family members
or members of a health team. You have 2 minutes to read the question and jot down your notes. A
buzzer will sound at 13 minutes, and at 14 minutes at which you must exit the room.

20
Keep in mind that the staff members are there to help you. Don’t hesitate to ask for help or guidance. A
tour on the exam day is posted on the MCC website. I strongly encourage you to go over it;
http://mcc.ca/examinations/mccqe-part-ii/exam-preparation-resources/

Structure of NAC
The NAC OSCE is a one day exam. You have 2 minutes to read the instructions and 11 minutes with the
patient. A warning buzzer will sound at 8 minutes, another buzzer at 11 minutes to indicate the end of
the station. At 8 minutes the examiner may ask you questions and as in MCCQE2, this will be indicated
in the question stem. In other cases you have the full 11 minutes with the patient. The types of clinical
scenarios are very similar to MCCQE2, and this will be covered in chapter 5

I strongly encourage you to review the comprehensive guide to NAC examination on the MCC website:

http://mcc.ca/examinations/nac-overview/osce-station-therapeutics-descriptions/

http://mcc.ca/examinations/nac-overview/exam-day/

Scoring
There are some differences in scoring between MCCQE2 and NAC. Only IMGs write NAC while both
Canadian Medical Graduates and IMGs write MCCQE2. For example, you will be rated on language
proficiency in NAC but not in MCCQE2. In both exams there is a space on the scoring sheet for the
examiner to write any concerns about your conduct. Below is an overview of what you need to know
about scoring to successfully prepare for these exams. A preset passing score is determined by the
examination committee, and you are not compared to other candidates. Refer to the MCC website for
further details.

MCCQE2 Scoring;

In the OSCE component, the examiner will observe your interaction and fill in a checklist of items. The
checklist covers clinical skills as well as communication, professional and ethical expectations. For
example, when you take a history or perform a physical exam you maybe scored on your organizational
skills, meaning that covering everything but in a disorganized way will cost you credit. It is not enough to
have solid knowledge to pass Canadian exams; you need to demonstrate professional behavior. I
suggest that you keep the following points in mind as you study and practice:

Introduce yourself

21
Appropriate eye contact; this may differ among cultures. Do not stare at the patient, and don’t ignore
them. Find a medium where you are comfortable, showing respect and acknowledgement of your
patients. You can do that by looking them in the eyes when you introduce yourself, and when you ask
questions. It is OK to write some notes while the patient is talking; you may attempt, however, to always
keep some eye contact.

Be aware of your body language; preparing for the exam is the best opportunity to identify any lapses in
your professional conduct. Listen attentively; nod your head to show understanding. Offer to help the
patient if you ask them to stand up, and watch for any signs of discomfort. Smile appropriately. Doctors
are genuinely caring, and you need to make sure that your caring nature is evident in your interaction
with the SP.

Address the SP by name as given on the door sign.

Wash your hands before physical examination. You may use hand sanitizer (it is alcohol-based gel that
you rub your hands with and will be available in the room)

Drape your patient appropriately; more on this will follow in chapter 3 (physical exam)

Avoid interrupting patients; in some cases the SP may be talkative and may even start telling life stories
not related to their medical complaints. In this case you may gently say: “Sorry Mr Smith, I would love to
listen to your story but I really need to stop you here and ask more questions about your headache”.
Alternatively you may say: “This sound like an interesting story, how about you tell me about it when we
finish talking about your back pain?”

Listen attentively; some examinees think that the OSCE is about asking all the possible questions
without listening to the patient’s answers. They often end up asking about information the SP had
already volunteered. Try to avoid repeating questions that the SP has already answered unless you are
seeking clarification in which case you should frame your question appropriately ie: “I know you already
mentioned that you take headache but can you clarify for me where exactly you feel it”

Be organized; now is your opportunity to prepare yourself to the variety of clinical scenarios that you
may encounter, work on your organizational skills as you practice, and use the sample cases in this book
as a guide.

Show empathy and understanding.

Give advice in a respectful manner. Avoid lecturing or judgemental statements, listen to their views and
ideas and address their concerns, and involve them in the management plan.

Keep in mind the basic ethical and legal principles as you interact with patients. This will be covered in
chapter 6.

Be specific, this is particularly important when asked questions and in the written component of the
exam. For example, if you think a surgical intervention is warranted specify the type of surgery; it is not

22
enough to just write or respond: surgery, for example; say; appendectomy if you think the patient has
appendicitis.

There are 5 sample cases and checklists on the MCC website. I strongly encourage you to review them;

http://mcc.ca/examinations/mccqe-part-ii/scoring/

In my personal experience and opinion, you can cover more points than what is listed in the MCC
checklist.

The interaction rating scale used by examiners to evaluate your communication with SPs can be found in
the following link;

http://mcc.ca/wp-content/uploads/Exams-interaction-rating-scale-items.pdf

Note that in each given station you will be scored on a number of clinical competencies and some of the
interaction rating scale areas as well.

NAC Scoring

You are required to perform at the level of a graduating Canadian Medical student. You may be
wondering; and what does that look like? I never practiced in Canada before! I totally get your point
since that is exactly how I felt when I first came here! I’m writing this book to tell you all about it.
Medical knowledge is so advanced that you may know too much or too little about any particular
disease, but what exactly do you need to do in a 5 or 10 min stations for your performance to be
considered satisfactory or beyond? You need to demonstrate that you have the basic knowledge to sort
out a patient’s medical complaint. You also need to demonstrate ethical and professional behavior.
Language proficiency and communication skills are also taken into account along with your
organizational skills. The above tips in MCCQE2 scoring are useful. You will become more familiar with
the exam expectations and how much to cover when you navigate through the cases. The NAC guideline
rating scale can be found here;

http://mcc.ca/wp-content/uploads/Exams-NAC-Guideline-rating-scale.pdf

In summary, you will be scored on about 9 competencies; history taking, physical examination,
organization, communication skills, language proficiency, diagnoses, data interpretation, investigations,
therapeutics and management.

I strongly encourage you to review NAC scoring system;

http://mcc.ca/examinations/nac-overview/scoring/

23
Important note: A pass score on the NAC OSCE is 65 (score range 0-100).
In order to be competitive for residency positions it is important to score
as high as possible in the NAC as this exam cannot be repeated unless
you fail. Therefore, it is advisable to ensure that your clinical
knowledge, communication skills and language fluency are assets not
detractors. Do not rush to take this exam until you are well prepared.
In general your goal is to achieve a score over 75 but the higher your
score, the more likely you will be considered for a CaRMS interview.
Programs such as Family Medicine may filter on your NAC score ie:
Ontario Family Medicine is transparent in their CaRMS description and
state that in general those invited for interview have a score of 81 and
higher.

References
1- Medical Council of Canada, accessed March 15/ 2015, www.mcc.ca

2- NAC Overview, accessed March 15/2015, http://mcc.ca/examinations/nac-


overview/exam-preparation-resources/

3- MCCQE2 preparation resources, accessed March 15/2015,


http://mcc.ca/examinations/mccqe -part-ii/exam-preparation-resources/

4- NAC scoring, accessed March 15/2015, http://mcc.ca/examinations/nac-


overview/scoring/

5- MCCQE2 scoring, accessed March 15/2015,


http://mcc.ca/examinations/mccqe -part-ii/scoring/

24
Chapter 2
History Taking
Edited by: Dr. Shirley Schipper
Dr. David Ross

25
Introduction

History taking is probably the most important aspect of patient care. In addition to establishing
physician-patient relationship, asking the right questions in the right way determines the rest of
the management plan. One study estimated that the history alone led to the diagnoses in
78.58% of patients (1), while another study quotes the number 76% (2). It is the most common
element tested in OSCE exams, and is integral for patient care.

Stations in OSCE are 6-14 minutes long. During this limited time the examinee is trying to
achieve two goals; develop a trusting relationship with the patient, and get an accurate
comprehensive story. This may seem challenging, and some may feel overwhelmed. But it is
doable if you have a clear approach in mind, and approach the case from a solid clinical
reasoning foundation including a reasonable head to toe differential.

In most cases your history will be tailored to the case, and the job becomes easy if you have a
differential diagnoses. You need to analyze the chief complaint, make sure you are ruling out
fatal and incapacitating conditions, showing the examiner that you are thinking of more than
one possibility as a cause for the patient complaint. Then you need to go over the general
history including; past medical and surgical, family, medications, allergies and social histories.
All these sections are covered in details in this chapter. However, keep in mind that you don’t
need to ask every patient all the questions. You can think of this chapter as a questions bank
and use whatever is relevant to the case. This will be further illustrated as you go over specific
cases in chapter 5.

OSCEs are designed to measure competency of residents entering general practice, so it covers
a wide range of clinical encounters including; Internal Medicine, Pediatrics, Psychiatry,
Obstetrics and Gynecology and Surgery, Preventive Medicine and Community Health. Each
discipline has its unique essential history elements, which is covered in details in this chapter,
and expanded upon in case scenarios in chapter 5.

Practice is the key to master the science and art of history taking. A lot of IMGs are out of
practice for years before they write these exams, which makes their job a bit harder than
Canadian medical students and residents who take similar exams throughout medical school or
in the case of QE2, the same exam in year two of residency. This book is designed to help you
practice whether you choose to do it on your own or with a study group. I wrote my US Step 2
clinical skills exam at a different time than most of my colleagues. Left alone to practice I used a

26
pillow as my standardized patient (SP), and acted as if I was in a real exam; knocking the door,
and timing myself, etc... My sister made a lot of fun of me, but this was my way to succeed.
Bottom line; my advice to you; don’t just read this book, PRACTICE, PRACTICE, PRACTICE. It may
also help if you write scripts – word for word what you will say when presented with a common
case. You must practice for this exam by doing and saying not reading.

It is important to ask a mix of open and closed ended questions as you interview your patient.
Treat SPs as real patients; they are trained to respond to you, if you ask the right questions the
exam will move smoothly and you will be done on time. For example; in many cases just
starting with; how can I help you today? Will yield some information and save you some time.
Then you can ask direct questions to gather the rest of the story.

Key points;

1- Two Goals in mind when taking history; build a trusting relationship, and get accurate
comprehensive story

2- Be aware of time, always time yourself

3- Develop a clear differential diagnoses and your own organized approach to each
presentation

4- Use a mix of open and closed ended questions

5- Treat SP as you would treat your own patients in real life

6- Keep in mind the basic communication skills discussed in chapter 1

7- PRACTICE

27
Basic Elements of History

1. History of presenting illness (HPI)

Starts with analyzing the chief complaint. Common chief complaints include; pain, fever, cough,
fatigue, skin rash , trauma, etc…..

The first step in history is to have a differential diagnoses for all common chief complaints, then
analyze the chief complaint ruling in or out the most common, fatal and incapacitating
conditions. This is basically your HPI. Be precise and thorough; A systematic approach to data
collection leads to a logical differential diagnosis.

Apply your critical thinking skills to stay focused on the complaint so your history is relevant and
organized. Try to avoid low yield questions that are not relevant to ruling in/out from your DDX
as asking random questions will raise red flags in the examiner’s mind as to what you are
considering a DDX.

Common things to cover for each chief complaint include;

1- Onset; when did the problem start?


2- Frequency; How often do you get it?
3- Duration; How long does it last?
4- Timing; does it happen at a specific time of the day/night/month/year?
5- Progression; has two components depending on the nature of the problem. If it is a
one-time thing is it getting better or worse? If it comes in attacks for the last year or
so how is it progressing over time? And how does it start and progress for each
attack?
6- Characteristics; for pain it is the nature of pain (dull, sharp, stabbing, aching, etc…),
for other complaints it varies with the type of the complaint like the amount and
color of vaginal discharge. It will be illustrated for each complaint in detail in case
histories in chapter 5.
7- Aggravating and relieving factors: What makes it better? What makes it worse?
8- Pertinent positives and negatives; Appropriate review of systems (ROS) that rules in
or out differentials on your list. A comprehensive list of ROS is presented in this
chapter page 6.

28
9- Impact of the problem on patient’s life; How is it affecting your function, work?
Relationships? Sleep?
10- For pain remember to add; Location, radiation and severity. For the severity of pain
ask the patient; on a scale from 1-10, 10 being the worst pain ever, where do you
score your pain?

I will follow this layout for all common chief complaints detailed in chapter 5. Keep in mind that
some complaints; seizures for example, require that you develop a special HPI.

Common HPI questions for each chief complaint;

1- Onset

2- Frequency

3- Duration

4- Timing

5- Progression

6- Characteristics

7- Aggravating and relieving factors

8- Pertinent positives and negatives

9- Impact of the problem on patient’s life

10- For pain add; location, radiation and severity

2. Past Medical and Surgical History

In some cases it is enough to ask the patient if he/she has other medical problems, and any
surgeries or hospitalizations. In some cases you need to be more specific as patients may not
volunteer the information. For instance, in a patient presenting with chest pain you need to
specifically ask about the cardiovascular risk factors including; diabetes, hypertension, smoking,
dyslipidemia and previous heart attacks or heart disease. The patient will not understand what
dyslipidemia means so ask if he/she has high cholesterol. In some cases the patient may not
know and you will figure out what diseases they have from their medications history.

29
3. Family History

The same applies to family history. It may be enough in some cases to ask a generic question
like; what medical diseases run in the family? In other cases you need to be more specific; for
example; Is there a family history of inflammatory bowel disease in a patient with chronic
diarrhea. You may say; Does anyone in your family have chronic or bloody diarrhea? Has
anyone in your family ever been diagnosed with Crohn’s disease or Ulcerative Colitis?

4. Medication History

Includes;

1- Name of the medication


2- Dose
3- Frequency and time of ingestion
4- Route
5- Any recent change to medications
6- For how long has the patient been on the medication
7- In the elderly, you need to know who administers the medications and if they are
blister packed
8- Use of over the counter medications and herbal remedies
9- Compliance to medications
10- Significant side effects

In some cases the patient may have a list of his/her medications, or have the medication
bottles. Grab the list or bottles and quickly go over the medications with the patient to make
sure he/she is taking them. Don’t waste too much time trying to write all the details down.
Remember the sheet of paper they give you is for your own benefit and use, it will not be
scored.

5. Allergies and Medications Intolerance

30
Ask the patient if he/she is allergic to any medications, food and if they have environmental
allergies. If they say yes follow up what they are allergic to and what symptoms they get. In
some cases it is true allergy, in others it is medication intolerance like when the patient gets
headache from a drug. It is particularly important to document severe allergic reactions like
anaphylaxis.

6. Review of Systems

Next, I’m listing the most important questions to ask in each system. Note that you don’t need
to ask all the questions listed below when you go over the review of systems; ask questions that
are pertinent to the case to rule in/out your top differential diagnoses. You may choose to
include it in your HPI (which I recommend, as it shows you are organized and have a clear chain
of thoughts), or cover it as a separate section. If the patient has a positive symptom, you may
need to analyze it more depending on its significance.

Cardiovascular

1- Chest pain; Do you get chest pains?


2- Shortness of breath; including dyspnea at rest, exertional dyspnea, orthopnea and
paroxysmal nocturnal dyspnea; Do you feel short of breath? Do you feel out of
breath at rest or with activity? How far can you walk before you get short of breath?
On how many pillows do you sleep, or do you feel out of breath when you lie flat?
Does shortness of breath wake you up at night? Do you wake up at night gasping for
air?
3- Palpitations; Do you feel your heart is racing?
4- Dizziness; Do you feel dizzy or light headed?
5- Syncope and presyncope; have you ever lost consciousness or passed out? Do you
sometimes feel like passing out but you don’t?
6- Ankle edema; Do you have swelling of your ankles?
7- Fatigue; Do you feel tired more than usual? Or do you feel fatigued?
8- Intermittent claudication; Do you get pain in your calves when you walk?
9- Note that other complaints that are not primarily cardiac can be seen in some
cardiac diseases for example; cough and wheezing in heart failure.
Respiratory

1- Cough; Do you cough? What triggers your cough? Is your cough worse at a particular
time of the day/night?
2- Sputum; Do you bring up any phlegm? What’s the amount, color?

31
3- Hemoptysis; Do you cough up blood?
4- Shortness of breath; do you feel short of breath? Follow up with a functional
estimation, i.e.: how far the patient can walk?
5- Wheeze; do get noisy breathing or wheezing?
6- Chest pain
7- Snoring, night time apnea and excessive day time sleepiness to screen for
Obstructive sleep apnea (OSA); do you snore? Has your partner noticed that you
stop breathing at night? Do you feel sleepy during the day? Do you fall asleep behind
the wheel, reading a magazine, watching T.V?
Gastrointestinal

1- Abdominal pain; do you have tummy pain?


2- Nausea/Vomiting; Do you feel sick to your stomach? Do you throw up?
3- Hematemesis: Do you throw up blood? Do you throw up coffee-ground like vomit?
4- Diarrhea/Constipation; Do you have diarrhea? Are you constipated?
5- Acid reflux; Do you have a sour taste in your mouth? Do you have heart burn? Do
you have dry cough?
6- Jaundice; Have you noticed that your skin and eyes turned yellow?
7- Hematochezia; did you see any blood in your stool?
8- Tenesmus: do you feel the urge to poop and when you go to the washroom nothing
comes out?

Genitourinary

1- Dysuria; Does it hurt to pee?


2- Change in urine color or smell
3- Flank pain
4- Passing stones with urine; did you notice any small stones coming out with urine?
5- Vaginal or penile discharge
6- Groin masses, pain , itchiness

Neurological

1- Headache; Do you get headaches?


2- Loss of consciousness; Have you ever lost consciousness?
3- Seizures; Do you seize? Or do you have abnormal movements?
4- Visual changes
5- Hearing loss
6- Tinnitus; Do you feel ringing or any other noise in your ears?
7- Head injury

32
Musculoskeletal

1- Muscle pain; do you have pain in your muscles?


2- Joint pain or swelling; do you have pain or swelling of your joints?
3- Morning stiffness; do you feel stiff in the morning and need time to get going?
4- Muscle weakness (especially proximal muscles); do you find it hard to comb your
hair or get up from a chair?
5- Skin rash
6- Scleritis/episcleritis; do you have pain or redness of your eyes?
7- Jaw claudication; do you feel you get tired chewing and need to rest your jaw while
eating?
8- Headache
Constitutional

1- Fever; have you had a fever? Did you measure it?


2- Drenching night sweats; do you sweat at night so much that you have to change the
sheets?
3- Weight loss; have you lost weight recently? How much and over how long?

Endocrine

1- Heat or cold intolerance; do you feel hot/cold more than most people in the room?
2- Diarrhea/constipation
3- Fatigue and somnolence
4- Nervousness: do you feel more nervous or short tempered?
5- Palpitations
6- Sweating
7- Weight loss or gain; have you lost or gained weight recently?
8- Voice change; have you noticed any change to your voice?
9- Changes of vision; have you had loss of vision or double vision?
10- Skin/hair changes
11- Polydipsia/Polyuria; do you feel thirsty more than usual? Do you need to pee more
than usual?
Skin

1- Skin rash/itching; do you have skin rash or itching?


2- In diabetic patients, it is important to ask specifically about feet ulcers
3- In bedridden patients inquire about pressure ulcers

7. Social History

33
Social history is an important health determinant that can be modified. It includes the
following;

1- Occupation; which can be a risk factor for a disease, or affected by it, ask the patient;
what do you do for living? What did you do before?
2- Living conditions; with whom and where the patient lives, does he/she has easy
access to health care? One of the special things about the Canadian health care is
that you can consult a social worker and help patients out using means other than
drugs and surgery
3- Stress at work or relationships; do you have any stress in your life whether at work
or in your relationships?
4- Relationship status; do you have a partner?
5- Smoking; do you smoke? How much and for how long?
6- Alcohol intake; do you drink alcohol? What do you drink? And how often? I will
expand more on alcohol history next
7- Recreational drug use; do you use recreational drugs? What do you use? How often?
Will be further detailed next
8- Financial status, special cultural considerations and religion are appropriate in
certain cases

History of alcohol intake

Ask the patient what, how much and how often he/she drinks. If he/she drinks alcohol daily
over the guideline limits or binge drinks and if relevant to the case go over the CAGE
questionnaire;

1- C: Have tried to cut down on your drinking?


2- A: Do you feel annoyed by people criticizing your drinking?
3- G: Do you feel guilty about drinking?
4- E: Do you need alcohol first thing in the morning as an eye opener?
It is key that you gather this information without judgment. If the SP feels that your attitude is
judgmental they could resist giving you information.

According to JAMA; CAGE of 2 or more has a positive likelihood ratio of 6.9 for detecting alcohol
abuse or dependence in adults. (4)

If the CAGE is positive you can also inquire if the patient has had any legal problem because of
his/her drinking and if his/her work and relationship are affected by alcohol. This will help you

34
to identify any drinking problem and its extent. In cases where the patient has a drinking
problem, ask him/her if ready to change his/her behavior.

History of substance abuse

If positive for recreational drug use in addition to the general questions mentioned above, if
relevant to the case, the following should be covered;

1- Do you ever inject drugs?


2- Do you share needles?
3- In what circumstances do you use drugs? And with whom?
4- Were you tested for diseases that could be transmitted by injecting drugs?
5- Do you ever get admitted to the hospital because of intoxication/overdose?
6- Have you had any legal problems because of drug use?
7- Did drug use cause you any financial or relationship problems? Did it affect your
work?
8- How do you feel about your drug use?
9- Have you ever tried to quit? If yes; tell me more about it; when, for how long, if
medical help was provided and why did he/she fall off the wagon?
10- Are you ready to change your behavior regarding drug use?

8. Sexual history

If relevant to the case, a detailed non-judgmental sexual history is warranted. Assure the
patient that all the information he/she provides is completely confidential except if there was
an impending harm to the patient or others. Questions to cover;

1- Are you sexually active?


2- With men/women or both?
3- How many partners do you have now?
4- How many partners did you have in the last 3 months? 6 months? One year?
5- Do you have decreased libido?
6- Do you have problems with erection or ejaculation? ( for men obviously)
7- Are you satisfied in your sexual relationship with your partner?
8- Do you use contraception?
9- What type of contraception do you use? Do you use condoms? How often do you use
them?

35
10- Have you ever had any sexually transmitted disease? What, when and was it
treated?
11- Do you know if any of your partners ever had a sexually transmitted infection?
12- Do you have a vaginal/urethral discharge? Burning or itchiness?
13- History of sexual assault or abuse

The extent of questions to cover in history depends on the case. If, for example you are
interviewing a newly diagnosed HIV patient, you need to focus on questions related to sexually
transmitted infections. If the patient main issue was erectile dysfunction, then other aspects of
the sexual history in addition to detailed social and relationship history will be more relevant.
Cases detailing these points are covered in chapter 5.

Special Populations: History

When taking history of the following patient populations, you need to cover the general history
first, and then go over special areas pertaining to a particular subset of patients.

Obstetrics and Gynecology history

Gestational history

1- Total number of pregnancies


2- number of living children
3- number of abortions
4- number of term and premature deliveries
5- Any fertility problems and treatment.

History of current pregnancy

1- When was your last menstrual period?


2- Was it similar to your usual periods in terms of number of days and amount of
bleeding?
3- Were you on contraception when you got pregnant?

36
4- Do you have morning sickness? If vomiting ask how often and how much and if
she has lost weight also make sure she is keeping up her fluid intake
5- Do you have breast tenderness?
6- Do you have vaginal bleeding or discharge?
7- Do you have abdominal pain or dysuria?
8- Is there any change to your bowel habit?
9- Did you take folic acid before getting pregnant?
10- Are you taking prenatal vitamins?
11- Are you taking any medications or herbs?
12- Do you smoke, drink or do drugs?
13- What do you eat on a typical day? Do you exercise?
14- Do you have regular prenatal care?
15- Did you have any complications during this pregnancy such as diabetes,
hypertension or infection?
16- Is the baby moving (if GA is appropriate)?
17- What is your blood group? What’s the father’s blood group? (in many cases the
patient doesn’t know their blood group and you will order it anyway, but this is
to show the examiner that you are thinking about this very important point)

History of past pregnancies:

Including the date, mode of delivery and the outcome, gestational age at birth, birth
weight, need for resuscitation or neonatal intensive care admission and any
complications during past pregnancies or deliveries.

Bleeding history;

Includes history of menarche, menopause, regularity and heaviness of menstruation,


Painful cycles and severity of pain. Also includes abnormal bleeding including
prolonged heavy cycles, intermenstrual bleeding and post coital bleeding.

Premenstrual symptoms;

Including anxiety, nervousness, food cravings, bloating, headache, sleep


disturbances, breast tenderness and changes in libido.

37
Symptoms of menopause;

Includes hot flashes, sweating, sleep disturbances and vaginal dryness.

Previous use of contraception:

What was used, when and for how long? Were there any side effects?

Previous mammogram;

When and what was the result?

Previous Pap test:

When? What was the result? Any intervention needed?

History of infections

Previous sexually transmitted infection, urinary tract infections, vaginal infections


and their treatment.

Specific symptoms;

Vaginal dryness, itchiness, discharge, dyspareunia, urinary frequency, dysuria and


incontinence.

Sexual history

If relevant to the case obtain the sexual history (detailed in section 1.2.9)

Pediatrics history

In most pediatrics cases the history is obtained from the mother, father or another care giver.
Sometimes the history may be obtained over the phone (QE2). In some cases a child will
accompany his care giver, in this case make sure you manage time wisely and involve the child
as appropriate based on age and the complaint.

38
At the beginning of each SP encounter, establish who is telling the story and their relationship
with the child. Also ask who is the guardian, this is particularly important in cases of child
abuse.

Rourke record is a very good resource for age specific pediatric history, it is used by many
health care providers in Canada;

http://www.rourkebabyrecord.ca/

In addition to the general history, the following is a general outline of what you need to cover,
keep in mind your patient’s age and customize your history accordingly; ;

Perinatal history;

Especially for infants and young children. It is less significant in teenagers unless
related to the chief complaint. Includes;

1- Prenatal history;

 Mothers obstetric history including the number of pregnancies and abortions


 This pregnancy; was it planned? What was the gestational age at birth? Was it
single or multiple gestations? What is the age and blood group of the mother
and father? Was there a regular prenatal care? Was there any complication
during pregnancy (for example; infection, vaginal bleeding, diabetes,
hypertension, accidents and trauma)? Did the mother use any medications
during pregnancy including prescription and over the counter medications,
vitamins and herbs? Did one or both parents smoke, drink, or use recreational
drugs during pregnancy or after delivery? Was the mother exposed to X-ray
while pregnant?

2- Natal history (History of delivery);

At what gestational age was the baby delivered? Where did the delivery take
place? Was labor spontaneous or induced? What was the mode of delivery
(Normal vaginal, forceps, vacuum or cesarean)? What was the duration of
membranes rupture? Was the water clear or stained with meconium (baby’s
poop)? Did the mother have vaginal bleeding or fever? Was there fetal distress at
any point during labor?

3- Post natal history;

39
What was the birth weight? Did the baby cry immediately? What was the APGAR
score (some mothers would know, but frame your question carefully so the
mother doesn’t feel intimidated; there is a score that is calculated when the baby
is born called APGAR score, did your doctor tell you what was your child’s
score?)? Was the baby admitted to the neonatal intensive care unit? When was
the baby discharged from the hospital? Did he have breathing or feeding
difficulties? Did he have fever, jaundice or seizures?

Feeding history

Was the baby given breast or formula milk or both? If breast; how often and for how long?
Where there any problems with breast feeding? For formula; what is the type? How is it
prepared? How much and how often is the baby fed? Does the baby spit up or vomit often? If
yes what is the amount, color, frequency and content? When was solid food introduced? What
is the child currently eating? Is he/she given any vitamins or supplements? Also ask about the
child’s urine and stool frequency and amounts and if there are any problems (dysuria, bloody
stools or melena, bloody urine, etc...)

Vaccination History

What vaccines was the baby given? When? Did he/she have any reactions or side effects to
vaccines namely fever, rash, seizures or prolonged crying? If yes, then what was done?

Growth and development

40
What is the child’s weight and height? Then you need to go over the developmental milestones
by history, tailor your questions to the patient’s age, for example; inquire about sitting in a
seven months old, it’s too early to ask about walking.

Review RBR found on Bourke’s record for age appropriate questions;

http://www.rourkebabyrecord.ca/downloads.asp

Developmental milestones general inquiries;

Gross Motor; Holding the head up, rolling from side to side and from prone to supine and supine to
prone. Sitting with and without support, standing with and without support, walking, running,, jumping,
going up and down stairs and riding a tricycle or a bicycle.

Fine motor; Ability to hold the a crayon, scribble, draw a triangle, rectangle, circle and square

Speech and language; Does he point out his/her needs? How many words does he/she say? Can he/she
use full sentences? Can he/she tell a story?

Social; Does the baby make eye contact? Smile? Laugh? Recognize parents? Does he/she become overly
anxious around strangers? Can he/she wave bye bye? Play pee-ka-boo? Help with
buttoning/unbuttoning clothes, put on shoes, play with other children?

Family history

Ask about consanguinity and congenital anomalies in the family. Inquire about specific diseases
pertinent to the case

Social history

It differs in pediatrics from adults. Social history for adolescents will be detailed in the
adolescent history later in this chapter. Ask about;
1- Age of the building, space, occupants, pets and home environment.
2- Who cares for the child?
3- Any stress or violence at home?
4- Any major events like death, accidents or divorce?
5- What are the child’s interests and activities?
6- Does the child go to a day care?

41
7- How is the child’s school performance?
8- What is the parent’s occupation? Hours?
9- Are there any financial issues?
10- Does one or both parents smoke, drink or use recreational drugs?

Psychiatry history

There are usually one or two psychiatric cases in the exam. In some cases the patient maybe
depressed or angry which makes history taking challenging. The standardized patients are well
trained and the exam is designed with the purpose of testing how you would react in these
situations. Staying professional in these scenarios and asking questions in an organized non-
judgmental way will get you through.

Psychiatric history is comprised of the following;

 History of the four basic psychiatric illnesses; depression, mania, psychosis and
anxiety
 Assess suicidal and homicidal ideation (You may fail the station if you forget this
point). Ask a direct question: Have you ever thought about hurting yourself or
ending your life? If yes, then ask about details of previous attempts; when and what
did the patient do exactly, try to determine if it was a serious attempt or a cry for
help. The method used is useful in discerning seriousness; if the patient uses drugs
and sends a letter to someone at the same time; he/she is probably seeking
attention. If the patient tried to hang or shoot him/herself, this is more serious.
Attempting to commit suicide once is a predictor that the patient will do it again.
Next, ask the patient if he/she is suicidal now and if he/she has a plan. If yes, the
next step is to admit the patient to the hospital with or without his/her agreement
to make sure he/she is safe. The same goes for homicidal ideation, if the patient says
he/she wants to hurt a specific person, you have a duty to warn the victim if you can
, notify appropriate authorities and admit the patient to hospital (voluntarily or
involuntarily).
 Assess insight; does the patient think his mood or hallucinations for example are a
problem?
 Rule out secondary cause for the psychiatric problem like an organic disease or
drugs
 Look for a precipitating factor
 Take a proper social history exploring relationships/work problems, functional
decline , substance abuse
 Determine if the patient needs admission via Form 1
42
 You may need to obtain collateral history if the patient seems unreliable

Next are the specific questions you need to ask for each of the 4 basic psychiatric problems;

1- Depression;
Screen your patient by asking if he/she feels depressed or has suicidal thoughts, if yes
go into details. You can use the popular acronym;

M SIGE CAPS;
* Mood; Do you feel depressed? For how long?
*Sleep; Did your sleep patterns change?
*Interest: Have you lost interest in activities you used to enjoy?
*Guilt: Do you feel guilty or worthless?
*Energy: Do you feel you have less energy than usual?
*Concentration and memory: Are you able to concentrate? Do you feel more
forgetful?
*Psychomotor agitation or retardation: Do you feel you are moving more or less
than usual?
*Thoughts of death/suicide: Do you have thoughts of killing yourself? Do you have a
plan?

2- Mania;
*Do you feel you have more energy than what you can control?
* Did you engage in risky behaviors or investments lately?
* Do you need less sleep?
* Do you feel your self-esteem is inflated?

3- Anxiety:
*Do you feel you worry excessively?
*Is it general worry or do you have something specific in mind?
*Do you have fear or phobias?
* Are you obsessed about anything?
*Do you have any compulsive behavior? Ie:

4- Psychosis;
*Hallucinations: Do you see or hear things other people don’t see or hear?
* Delusions: Do you hold beliefs other people think are odd? If so, tell me about
them.
* Have you been taking care of yourself and look recently?

43
* Are you becoming more socially withdrawn?

Adolescent (teenager) history

Before even taking the history from a teenager, emphasize confidentiality. Tell them that
everything they share with you is strictly confidential, and that you are not going to tell their
parents or teachers. You need to let them know that the only time when you are going to tell
anybody is if someone else’s health or safety is affected as with planning homicide or in some
communicable diseases. This way you gain their trust and they are more likely to share
information with you. If the adolescent is accompanied by one or both parents, obtain some of
the history in the parents’ presence then ask them politely to leave the room.

Psychosocial history and identifying risky behaviors are the added areas of adolescent history.
There is a nice acronym commonly used in Canada to cover these areas;

HEEADSS

Home: With whom does he/she live? How is his/her relationship with parents/siblings?

Is there any stress or abuse at home? What does the home environment look like?

Education: which school does he/she attend? What grade? How is his/her school
performance? Is there any bullying/abuse at school?

Eating: here you screen for eating disorders which are more common in females. Ask first only
question number 1 to screen quickly. If negative – move on. If you hear a red flag in the
answer then continue with the other questions.

Eating disturbances can be a sign of depression as well. Ask about;

1- Number, content and frequency of meals


2- Do you snack?
3- Do you take any vitamins/supplements?
4- What is your current weight? Did you lose or gain weight? Was it intentional?
5- Are you happy with your weight and look?
6- Have you been on diet? How many diets over the last year?
7- Do you binge eating?
8- Do you try to make yourself sick and throw up to avoid gaining weight?
9- Do you take laxatives?
10- Do you exercise? How often and how much?

44
Activities: What activities do you do, including work, sports, exercise, art, etc…

Alcohol, Smoking and Drugs (if not previously asked in the social history) If positive
follow up with quantifying use. Then, was he/she ever in trouble because of substance abuse

Sex; Cover the sexual history and educate the teenager about safe sex practices.

Suicide and mood: Ask directly about any suicidal or homicidal thoughts and screen for
depression;

When obtaining a family history from an adolescent inquire about heart attacks, sudden death,
dyslipidemia, any genetic disorders and if there is a family history of substance or alcohol abuse
or a psychiatric illness.

References
1- Roshan M, Rao AP. "A study on relative contributions of the history, physical examination
and investigations in making medical diagnosis." J Assoc Physicians India. 48, no. 8
(2000): 771-5.

2- Peterson MC, Holbrook JH, Von Hales D, Smith NL, Staker LV. "Contributions of the history,
physical examination, and laboratory investigation in making medical diagnoses." West J Med.
156, no. 2 (1992): 163-5.

3- Simel, David L. The Rational Clinical Examination Evidence-based Clinical Diagnosis. New
York: McGraw-Hill Medical, 2009.

4- Macleod, John. Macleod's Clinical Examination. 12th ed. Edinburgh: Churchill


Livingstone/Elsevier, 2009.

5- Angela LaRossa, Developmental-behavioral surveillance and screening in primary care, In:


UpToDate, Topic 615 Version 20.0 , UpToDate, Waltham, MA, accessed August 20/2014,
http://www.uptodate.com/contents/developmental-behavioral-surveillance-and-screening-in-primary-
care?source=search_result&search=developmental+milestones&selectedTitle=1~92

45
Chapter 3
Physical Examination
Edited by; Dr. Jay Shavadia
Dr. Erin Toor
Dr. M. Wasif Hussain
Dr. Sadik Salman
Dr. Stephanie Keeling

46
Introduction

IMGs represent a diverse group of physicians; some of them are specialists in their respective
fields and know way more than what is needed to pass the Canadian entry level exams. This
particular group of IMGs may wonder how far should they go when doing physical examination;
should they state that every step is evidence based and quote the likelihood ratio? Should they
do every single maneuver? Other IMGs may be wondering about what organ systems are tested
in OSCE. This chapter will go over physical examination (P.E) in detail.

As mentioned earlier, Canadian OSCEs that IMGs are required to pass to become eligible to
apply for residency positions in Canada evaluate the knowledge of the generalist not the
specialist. For example; you should be able to do general respiratory physical exam, but not
necessarily elicit all the specific findings in COPD. You don’t need to back up your maneuvers
with evidence based data either. Unlike in some other countries, the exam covers all organ
systems and each one of them is detailed next.

One of the special aspects of physical examination in Canadian OSCEs is that you need to
verbalize it, i.e; think out load. For example when you are listening to the heart you need to say
what you are looking for while listening; I’m now listening to S1, S2, etc… Which means you
can’t really focus on eliciting findings, instead if you say: looking for murmurs, the examiner will
tell you if there is a murmur. However, in some cases, especially with inspection the patient
may have real findings and in this case describe what you see. Now comes the question; how
much should I say during P.E? There is no right or wrong. Ideally you should say what you are
doing, what you are looking for and your findings. This chapter includes suggested verbalizing of
PE. Nevertheless feel free to use your own words and say less or more as the time permits.

Be professional when performing physical exam; Ask for permission at the start and only once
then proceed, explain to the patient what you are doing and why in an easy to understand
language, don’t use medical terms. One of the IMGs I studied with mentioned to me that his
friend used to ask patients; can I examine your cardiovascular system? Instead simply say; can I
examine your heart? Wash your hands before and after examining the patient (It is enough to
rub your hands with sanitizer). Use respectful draping keeping in mind the patient comfort and
privacy; don’t repeat painful maneuvers, and make sure the patient is warm and your hands are
warm!

As with history, reading this chapter is not enough. You need to PRACTICE. It is even more
important for physical examination, as you need to complete most exams in five minutes or

47
less. I can’t stress enough the importance of working on your own organized approach to
physical examination. Following the traditional sequence of; Inspection, palpation, percussion
and auscultation is a good way. However, this sequence may be altered when doing special
exams, for example; auscultation before palpation and percussion in abdominal exam. Or may
be modified as in musculoskeletal exam where you would do; Inspection, palpation, range of
movement and special tests. Both way be organized, and take into account your patient’s
comfort – i.e.: do not have the patient lying down, then sitting up, then lying down again. Make
sure you master all maneuvers, the examiner will be looking on how well you do each step, and
you will get zero if you verbalize correctly but do the procedure in a wrong way.

You should tailor your P.E exam to sort out the patient’s problem, this is the focused part of the
PE. In any case involving ER always make sure the patient is stable before starting your P.E. by
asking for vitals if not given on the door sign. Comment on the vitals given i.e.: patient’s vitals
are normal or this patient is tachycardic, etc. This is especially relevant in management cases
where you will not be asked to do a particular physical exam. Solid knowledge and practicing
the cases in this book will help you perform high yield P.E maneuvers.

The objective of this book is not to teach you how to do P.E or the differential of positive and
negative finding , but rather to point out what aspects of the physical exam are more relevant
in Canadian OSCEs, and how you are expected to perform.

This chapter details the physical exam steps then outlines the suggested verbalizations in boxes.
Conversation directed to the patient is in italic. I’m assuming that there are no findings, make
sure to practice and master the procedures so that you are able to elicit findings if any and
report them, in many cases the examiner will tell you if there are findings.

Key Points
- Verbalize your physical exam including what you are doing, what you are looking for
and your findings
- You don’t need to talk about the evidence behind your maneuvers
- Make sure you do the maneuvers correctly
- Expect to be tested on any organ system
- Develop your own organized approach following the traditional sequence of;
Inspection, palpation, percussion and auscultation as appropriate
- Always take permission, wash your hands and be aware of patient comfort and privacy
- Drape your patient in a respectful way; Keep body parts you are not examining
covered.
- Communicate with the patient in an easy to understand language; Avoid medical
jargon

48
- Listen to the examiner, they will give you pertinent findings or ask you to move on – if
they ask you to move on, do so.
- Manage your time and priorities wisely
- Practice Practice Practice

Cardiovascular System Examination (CVS P.E)


Edited by; Dr. Jay Shavadia

Includes:

1- Peripheral and central pulse, Blood pressure

2- Jugular Venous Pressure (JVP)

3- Precordium examination

4- General Examination related to CVS exam

5- Peripheral Vascular exam

In most cases you need to do 1, 2, 3 and 4. The question will usually be specific about
peripheral vascular exam. Always make sure the patient has stable vitals before starting CVS
physical exam; Take a quick look at the patient looking for pain, pallor, increased work of
breathing, and mention if any medications / oxygen is present at the bedside. Go over ABCs
and ask for the vital signs if not provided in the question stem.

Peripheral and central pulse

Ask permission, wash your hands and tell the patient that you are going to take their pulse.
Count the rate and note the regularity. An irregular pulse could suggest atrial fibrillation, or
premature atrial / ventricular complexes. With the patients arm resting on your right arm, feel
the brachial pulse with your right thumb and the radial pulse with your left index finger for a
brachio-radial delay. Next feel the left or right carotid pulse using your thumb, and note the
volume and upstroke.

49
I will examine the CVS by starting with ABC; the patient looks comfortable, breathing
spontaneously and speaking in full sentences. I’m feeling the radial pulse and it is regular 80
beats/min. There is no brachio-radial delay. The vital signs were just reported by the nurse and
are normal. I would like to get them checked again in X minutes (Depends on the case; In
emergency situations check the V.S every 10-15 minutes or put the patient on the monitor; if
you see a stable patient in your clinic you don’t need to repeat the V.S). The central pulse is of
normal volume and upstroke.

Precordium physical exam

Ask permission, wash your hands, and respectfully drape the patient by lowering his/her gown
to his/her waist. Keep the bra on in female patients, and drape the chest once the inspection of
the precordium is complete. Do the examination with the patient lying down. Speak up and
explain what you are doing, what you are looking for and your findings

- Inspection; Look at the shape of the chest for any deformities or asymmetrical
movement during respiration. Note any dilated veins, visible pulsations, scars
(sternotomy, lateral thoracotomy, pacemaker) or other skin changes. Inspect the
infraclavicular areas for pacemakers / defibrillators. Also inspect the epigastrium for
pulsations suggestive of abdominal aortic aneurysm or right ventricular enlargement.

Can I examine your heart? The SP will say yes. Wash your hands and respectfully drape the
patient. I start the precordial examination by looking for any deformities or asymmetrical chest
expansion with respiration and I don’t appreciate any. There are no dilated veins, visible
pulsations, scars or any other skin changes (you need to list findings if any)

Note; if you only say I’m inspecting the epigastrium for any abnormalities, the examiner will
consider this unsatisfactory.

- Palpation; you are palpating for four things; First: The Apex beat (defined as the most
lateral and inferior impulse NOT the point of maximal impulse), normally located in

50
the fifth intercostal space, midclavicular line. Feel for the character of the apical
impulse. It could be normal or sustained. Second; palpate for heaves (Forceful
contractions, feels as if your hand is being lifted). Use the palm of your hand and
palpate over the right and left parasternal borders and over the apex. Third; thrills
(palpable murmurs) and palpable heart sounds. Use the tips of your fingers to feel for
thrills over the 4 major valvular areas; Aortic, Pulmonic, Tricuspid, and Mitral. A thrill
feels like a vibration. A palpable second heart sound may be appreciated over the left
second interspace in pulmonary hypertension. Fourth; palpate the epigastrium for
pulsations suggestive of either right ventricular hypertrophy or abdominal aortic
aneurysm. With your hand in the epigastrium, the pulsation of a AAA ‘comes from
below’ while that of the right ventricle ‘comes from the top, under the sternum’.

I will move now to palpation, Do you have any pain?. If the patient says yes start palpation
away from the painful area. I’m feeling for the apex beat, I notice it is located as expected in the
fifth intercostal space midclavicular line. The apical impulse is normal. Next, I’m feeling for any
heaves, thrills and palpable heart sounds and don’t appreciate any. Finally I’m palpating the
epigastrium for pulsations, and there isn’t any

- Percussion; Skip in precordial exam

- Auscultation; Auscultate over all four valvular areas, using the bell then the diaphragm
of the stethoscope in three positions; lying flat, leaning to the left and sitting up. Feel
the carotid pulse while listening (S1 heard before the pulse, S2 after). Start with the
patient flat; listen for S1, S2, note if they are normal, loud or soft and if S2 is split, then
listen for any added sounds (mainly S3, S4, rubs) or any murmurs. Specify if the
murmur you are hearing is systolic or diastolic. Then get the patient to lean towards
his/her left side and listen for murmurs of the mitral valve in end expiration. After
that, get the patient to sit up lean forwards, take in a deep breath then exhale and
listen for murmurs of the aortic valve. If the examiner says that there is a murmur or if
you hear a murmur then you need to fully describe it; location, type (systolic vs.
diastolic), radiation and intensity.

It doesn’t matter which order you listen in, as long as you have an organized approach. I listen
to the base first, starting with the aortic then pulmonary then tricuspid then mitral. Some
people start at the apex and move upwards.

51
I’m now auscultating the heart; I’m listening over the aortic, pulmonic, tricuspid and mitral
areas. Listening for S1, S2, there is no S3, S4, no rubs and no murmurs Can you lean to the left
side please? I’m listening again over the mitral area. Can you sit up for me, lean forward, take a
deep breath in- out and hold your breath. I’m listening for aortic valve murmurs.

Helpful info;

- Mitral regurgitant murmurs have a ‘blowing quality’ radiate to the axilla, and
accentuate on end expiration

- Mitral stenosis murmurs are low pitch, ‘rumbling’; may be accentuated by asking the
patient to exercise (lean to- and fro- a few times)

- Aortic regurgitant murmurs are high pitched, caused by turbulence of blood through
the incompetent aortic valve and radiate to the left lower sternal border. Best heard
with the patient leaning forward.

- Aortic stenosis murmur is harsh, and radiates to the root of the neck, and carotids.

- Systolic murmurs; Aortic and pulmonic stenosis (ejection systolic / crescendo-


decrescendo) mitral and tricuspid regurgitation (holosystolic i.e begin with the first
heart sound, hence S1 will be soft or absent)

- Diastolic murmurs; Aortic and pulmonic regurgitation (high-pitched), mitral and


tricuspid stenosis (low pitched)

Intensity (grading) of murmurs;

“I – is a murmur barely audible with stethoscope. It is soft, heard intermittently, always with
concentration and never immediately.

II – is a murmur that is low, but usually audible. It is soft, audible immediately and with every
beat.

III – is a murmur of medium intensity without a thrill. It is easily audible, and relatively loud.

52
IV – is a murmur, which is of medium intensity with a thrill, is relatively loud, and is associated
with a palpable thrill.

V – is the loudest murmur heard with stethoscope on chest. It is loud enough to be heard by
placing edge of the diaphragm of stethoscope over the patient’s chest. It is associated with a
palpable thrill.

VI – is a murmur heard with stethoscope off the chest. It is so loud it can be heard even when
the stethoscope is not in contact with the chest and is held slightly above the surface of the
chest. It is associated with a palpable thrill.” (1)

Jugular venous Pulse (JVP) examination;

The JVP is a very important P.E skill. JVP height reflects the pressure in the right atrium. Refer to
figure 3-2 for JVP anatomy.

Start your P.E by asking permission, then wash your hands and position the patient supine with
the head of the bed elevated at 45 degrees. Place a pillow under the patients head to relax the
neck muscles. Appropriately drape the patient by lowering the gown a little so that the root of
the neck is visible. Do the examination by standing to the right side of the bed. If the top of the
JVP is not visible at this 45-degree position, the head of the bed should be altered up or down
and the JVP reexamined.

- Inspection; look for the JVP between the two heads of the sternocleidomastoid
muscle, notice the highest point and measure its distance from the sternal angle (use
the ruler to draw a perpendicular line from the sternal angle, then your pen to mark
the intersection between the highest point of JVP and the ruler). According to
traditional teaching the normal jugular venous pressure is 6-9cmH₂O (3-4 cm from the
sternal angle + 5cm the distance between the sternal angle and the right atrium).
Traditionally JVP of 4 cm from sternal angle or more is considered elevated. However,
according to the most recent JAMA evidence based clinical diagnoses article JVP of 3
cm or more in any patient position is considered elevated, as most recent data suggest
that physicians often underestimate the JVP.

- Recognize the wave form; Normal JVP is double impulse. Follow the link below for
description of the waves;

http://www.medinterestgroup.com/portfolio-items/jvp-normal-wave-form/

Abnormalities of the wave form will be mentioned below.

53
- Differentiate the JVP from Carotid pulse;

1- Change with position; JVP decreases when the patient sits up and increase when he
lies flat, the carotid pulse doesn’t change with position.

2- Change with respiration; if there is no pathology JVP decrease with inspiration as the
intrathoracic pressure decreases. The carotid doesn’t change with respiration.

3- Waveform; normal JVP has double waves. The carotid has single wave.
Abnormalities of the JVP waveform are listed below.

4- Occlusion of the blood vessel; the JVP disappears with pressure at the base of the
neck, while the carotid pulsation remains persistent.

5- Palpability; the carotid pulse is always palpable; the JVP is not palpable in most
cases.

6- Abdominojugular reflux; formerly known as the Hepatojugular reflux. Has 2


purposes; differentiating the Carotid from the JVP, and is one of the signs of
elevated left ventricular filling pressures (suggestive of left heart failure). Ask the
patient if he/she has abdominal pain and tell him that you will apply pressure to his
abdomen. Inflate the blood pressure cuff and place it on the abdomen (You can
place it anywhere on the abdomen and not necessarily over the liver; thus the term
abdominojugular; my personal preference is to put over the Right upper quadrant or
epigastrium) with your right hand apply pressure over the cuff till the blood pressure
monitor goes up to 30mmHg. Sustain the pressure for 10 seconds while looking for
elevation in the JVP. Normal JVP increase a bit and goes down quickly. A positive
sign in heart failure is sustained elevation of at least 3 cm for 10 seconds. The
Carotid pulse is not affected by pressure over the abdomen.

Can I examine your neck veins? Wash your hands, I will lift the head of the bed a little bit, let me
know if you are uncomfortable. Lower the gown so that the root of the neck and both clavicles
are visible and ask the patient to turn the head a little to the left. I’m looking for the JVP
between the 2 heads of the sternocleidomastoid, and I see a double impulse here (point with
your finger then measure it). To make sure this is the JVP and not the carotid I will do some
maneuvers. Can you slowly take in a breath for me? I notice that it is decreasing with
inspiration as expected. Apply some pressure at the vein origin at the root of the neck, and I
notice that unlike the carotid it is occludable. Feel it, and it is not palpable in his case unlike the

54
carotid which is always palpable. Warn the SP; I’m going to lower the bed a little bit? And lower
the bed, I notice it went up, unlike the carotid which doesn’t change with position. And as I
mentioned earlier it is double impulse while the carotid is single. I also notice that the
waveform is normal, no Cannon waves or CV waves. Lastly I will do the abdominojugular reflux;
do you have pain in your tummy? I’m going to press a bit on your tummy let me know if it hurts.
Inflate the blood pressure cuff, put it over he epigastrium or right upper quadrant. Press on the
blood pressure cuff till the BP monitor goes up to 30mmHg. I’m doing the abdominojugular
reflux by applying 30mmHg pressure over the abdomen and looking at the JVP, an elevation of
at least 4 centimeters that is sustained for 10 seconds is seen in left heart failure. In this case
the JVP went up briefly then immediately went down, so the abdominojugular reflux is
negative. Of note is that the Carotid pulse is not affected by pressure on the abdomen

Figure 3-1; JVP anatomy (2) Note that you are examining the internal not external jugular

Helpful info

Abnormalities of the JVP

55
1- Kussmauls sign: Is the paradoxical increase of JVP with inspiration seen in restrictive
pericarditis, constrictive and restrictive cardiomyopathy

2- Single impulse; seen in atrial fibrillation as the a wave of atrial contraction is lost

3- Giant (cannon a wave); seen in complete heart block where the right atrium
contracts against a closed tricuspid valve

4- CV wave: seen in tricuspid regurgitation

General Exam related to CVS P.E

- Auscultate the bases of the lungs looking for crackles and wheezing seen in Heart
Failure

- Check the lower limbs for edema, and scars of venous graft harvest site for CABG
surgery

- Examine the hands for; nicotine stains, peripheral cyanosis, splinter hemorrhage, Osler
nodes, Janeway lesions and palmar erythema (The last four are seen in Infective
Endocarditis).Note have already checked the radial pulse at the beginning of the
examination? If not do it now, comment on the rate and regularity, compare both
sides

- Examine the face for pallor, central cyanosis, malar flush, corneal arcus, Xanthelasma

- Finally check the liver span and examine the abdomen for ascites (Ascites exam is
covered under abdominal exam)

Can you sit up please? I’m auscultating the lungs for any wheezes, and listening to the bases for
crackles, I don’t appreciate any. I’m looking for lower limbs edema, it’s negative. I’m examining
the hands for nicotine stains, Jane way lesions, splinter hemorrhage palmar erythema,
peripheral cyanosis, all negative. I have already checked the pulse and noticed its 70b/min and
regular. I’m now examining the face for pallor, can you open your mouth please? Central
cyanosis, malar flush, corneal arcus, xanthelasma, and don’t appreciate any. Can you lie down
please? Do you have any pain? I percuss the abdomen checking the liver span. And looking for
ascites (refer to abdominal examination below for the technique)

56
- Finish the exam by covering the patient (if not already done earlier), and washing your hands.

Peripheral Vascular Exam

Includes P.E of the peripheral arteries and veins. I will detail each one separately. The exam will
be specific; possible questions are; examine the lower limbs for arterial insufficiency, or
examine the lower limbs for deep vein thrombosis (DVT), prepare yourself for more generic
questions like for venous insufficiency. If asked to perform peripheral vascular exam then
merge both by looking for findings of both while doing the systematic approach; inspection,
palpation, auscultation and special maneuvers.

1- Peripheral Arterial System Examination

Ask permission, wash your hands and respectfully drape the patient by exposing the neck, both
upper arms and the legs keeping the sheets in between the legs (keep the gown on and expose
the areas you are examining). In this examination you need to examine the abdomen for
abdominal aortic aneurysm (AAA) and bruits, keep it covered till you reach it. Always compare
both sides.

Make sure you cover the classical 6 Ps;

1- Polar (cold)

2- Pain

3- Pallor

4- Paresthesia

5- Paralysis

6- Pulselessness

- Inspection; Look for muscle atrophy, masses, skin changes mainly pallor, shiny hairless skin,
rash, necrosis, scars and ulcers (arterial ulcers are sharply demarcated and found on the tips of

57
fingers, toes, on the heel and at the head of the fifth metatarsal). Make sure you inspect the
hands and legs closely looking for nicotine stains. Look between the toes. Notice any redness of
the eyes or joints swelling or deformity, which could suggest vasculitis as the underlying
pathology.

I notice that the patient is not in pain, no skin rash or redness of the eyes. I’m looking into the
patient’s mouth, no central cyanosis. I inspect both upper and lower limbs comparing sides; the
muscles are symmetrical with no atrophy, no pallor, no cyanosis, no necrosis, no ulcers, no
nicotine stains, no joint swelling or deformities, and no loss of hair.

- Palpation; Start by feeling the temperature, compare both sides as well as different spots on
the same limb. Palpate all pulses specifying the exact anatomical location of each (refer to the
box below). Comment on pulse regularity (regular, regularly irregular, irregularly irregular) and
strength. Compare both sides; for radial and brachial pulses you can feel the right and left at
the same time. For carotid pulse listen first, if you don’t hear bruits palpate. Listen for bruits of
the femoral pulse as well. Squeeze the calves for tenderness suggestive of critical ischemia.
Notice that for the sake of time, you can compare the right and left radial pulses, and then
examine one side only, in this case let the examiner know what you are doing by saying; I will
compare both sides but for the sake of time in an exam setting I will focus on the right side for
now.

I’m feeling the temperature of the upper limbs comparing both sides as well as proximal and
distal areas of the arms, and do the same for the lower limbs. I’m going now to feel the pulses;

1- Starting with the radial pulse lateral to Flexor Carpi Radials tendon, I notice
the pulse is regular and strong, I feel the pulse in both arms at the same
time and it is symmetrical

2- I move now to the brachial pulse medial to biceps tendon, I compare both
sides no abnormalities

3- I listen to the Carotids first making sure there are no bruits then I feel each
side at the lateral border of Sternocleidomastoid at the level of the thyroid
cartilage, noting that the pulse is normal. (P.S; Never feel both carotids at
the same time)

58
4- I move to the femoral pulses; I need to feel the pulse in the upper part of
your leg, this may feel uncomfortable, let me know if you need me to stop. I
feel the femoral pulse at the mid-point of the inguinal ligament. I feel the
other side and compare. I listen for bruits.

5- Then I move to the popliteal pulse, can you bend your knees for me? I feel
the popliteal pulse in the popliteal fossa

6- I’m feeling the posterior tibial pulse 2cm behind and below the medial
malleolus.

7- And finally the dorsalis pedis pulse on the dorsum of the foot lateral to
extensor tendon of the big toe.

- Neural exam; Both sensory and motor;

1- Sensory; test all the lower and upper limbs dermatomes for simple touch using a
cotton ball (can be found on the table in the room). Name which dermatome you are
testing and compare both sides

Follow the link below for dermatomal distribution;

https://www.pinterest.com/pin/487162884665368556/

2- Motor; you don’t have to do it all; for the upper limbs do resisted shoulder abduction
and adduction, elbow flexion and extension. For the lower limbs resisted hip flexion,
knee flexion and extension, ankle dorsiflexion and plantar flexion.

I will now do motor and sensory screening; I will start with fine touch, touch the patient lightly
with the cotton ball on his hand and ask; do you feel that? I want you now to close your eyes
and say yes every time you feel it, and let me know if it doesn’t feel the same on both sides. I will
start with the arms; C4, C5, C6, C7, C8, T1. Now the lower limbs; L1, L2, L3, L4, L5, S1. There is

59
normal symmetrical fine touch sensation. I want you now to resist me, please push down with
your shoulders, that’s adduction, push up; abduction. Please bend your elbow; elbow flexion
and extension, I will do the other side and compare, the power is 5/5 and symmetrical. Now the
lower limbs; hip abduction, can you bend your knees for me and kick out, knee extension, now
in; knee flexion. Press on my hand as you press on the gas; ankle plantar flexion, now the
opposite direction; dorsiflexion.

- Special tests;

1- Capillary refill; Apply pressure with your thumb and index finger to the distal end of
the thumb and big toe till it becomes pale then release, normal color should
reappear in 2-3 seconds. More than 5 seconds is considered abnormal.

2- Pallor on elevation and rubor (redness) on dependency test; Raise the leg about 60⁰
or until pallor develops, some pallor is normal, marked pallor is seen in arterial
insufficiency. Now ask the patient to sit up and dangle his legs to the side and notice
the color of the legs. In patients with arterial insufficiency the pallor persists for
about 10 seconds and then the legs become very red.

3- Ankle Brachial Index; you will not be able to actually perform this test, because you
need Doppler Ultrasound (US). Nevertheless, it’s a good idea to mention it. The
examiner may ask how you would do it. Measure the blood pressure of the arm,
then measure the blood pressure at the ankle by placing the blood pressure cuff
around the calves. Use the Doppler US instead of feeling the pulse, you can test
either the posterior tibial or dorsalis pedis pulses. Then simply divide the pressure of
the leg over that of the arm; 1 is normal, less than 0.9 is abnormal, and values below
0.5 suggest critical limb threatening ischemia.

4- Allen test; occlude both the radial and ulnar arteries with your thumbs, ask the
patient to open and close his hands till the palm blanches, then release the pressure

60
over the radial and look for the return of normal skin color. Repeat the steps but this
time release pressure over the ulnar artery. The color normally returns in 10
seconds, if it takes longer then it indicates insufficient collaterals and it is better to
avoid puncturing the radial artery.

I’m checking for capillary refill, in the upper and lower limbs and comparing both sides. I will
now do pallor on elevation rubor on dependency test. I’m going to raise your legs let me know
if it’s painful. I notice that the legs became pale, but not white. Can you sit up and dangle your
legs over the edge of the bed? I notice that the normal color of the limb returned almost
immediately with no excessive redness. The test is negative for arterial insufficiency. I would as
well like to get an ankle brachial pressure index. At this point the examiner may say: move onor
may ask you to describe it. In the latter case go over the steps mentioned above. I will now do
an Allen test, I’m going to press over the arteries, let me know if it’s uncomfortable. Can you
open and close your hand for me? I notice the hand turned pale, I release pressure over the
radial and notice the normal return of the color. I repeat the same steps occluding both
arteries; can you open and close your hand for me? I notice the pallor and release the ulnar side
noting return of normal color.

- Related abdominal exam; you can now cover the limbs, and expose the abdomen by
lifting the gown up and covering the patient with the sheets (only exposing the
abdomen). Inspect the abdomen for visible pulsations then feel the abdominal Aorta
which is normally palpable in thin people. Place your hand on either side of the
abdominal aorta to estimate its width, more than 2.5cm warrants further evaluation
with Ultrasound. Now listen to the abdomen for;

 Bruits of the abdominal aorta; listen in the epigastrium

 Renal arteries bruits; 5cm above the umbilicus and 3 cm to each side of the
midline.

2- Peripheral Venous System Examination

61
This section covers P.E for Deep Vein Thrombosis (DVT). I will go over lower limbs DVT P.E
because it is more common. The same principles apply for upper limbs DVT, you need however
to look specifically for venous puncture sites.

Ask permission, wash your hands, and respectfully drape the patient by exposing only the lower
limbs keeping the sheets in between the legs.

- Inspection; Look for redness, swelling, thickened skin, venous ulcers on the medial
side of the leg, dilated superficial veins or discoloration of the skin. Compare both
sides, asymmetrical swelling is a particularly useful sign of DVT, as well as entire leg
swelling. Notice the general appearance and comfort of the patient, check the
respiratory rate and Oxygen saturation to show the examiner you are thinking about
Pulmonary Embolism as a fatal complication of DVT.

Comparing both legs there is no swelling, both sides are symmetrical, no redness, no thickening
of the skin, no ulcers, no dilated superficial veins. The patient appears comfortable breathing
normally. I would like to know the respiratory rate and oxygen saturation please. At this point
the examiner may tell you the values or ask you to go ahead and check the respiratory rate.

- Palpation; compare the temperature of both legs with the dorsum of your hand.
Measure the width of the calf, more than 3cm difference between the two sides is
significant according to JAMA, some other resources quote 2cm (McGee). The width is
measured at a certain distance from a bony prominence; I use the tibial tuberosity, go
10cm down the leg and measure the width. Squeeze the calf for tenderness. Do
Homans sign; Dorsiflex the foot and ask the patient if this causes calf pain.

Check the legs for pitting edema; using your thumb start at the distal end of the leg, press over
the shin, if there is edema walk your way up until the level of edema becomes clear. Feel for
lower limbs pulses.

62
I’m checking the temperature of both legs and there is no difference between the two. I’m
measuring the width of both legs 10 cm below the tibial tuberosity comparing both sides; there
is 1cm difference which is normal. Do you have pain in the back of your legs? I will feel it, let me
know if it hurts. I check for lower limbs edema and don’t appreciate any. Next, I’m doing
Homan’s sign, dorsiflexing the foot? Do you have any pain? Finally I will check lower limbs
pulses;

1- Dorsalis Pedis lateral to extensor halluces longus tendon.

2- Posterior Tibial 2cm behind and below the medial malleoulus

3- Popliteal pulse in the popliteal fossa

4- Femoral pulse in the mid inguinal point

63
Respiratory System examination:
Edited by Dr. Erin Toor

Introduce yourself to the patient, ask permission and wash your hands. Do the general
examination that includes examination of hands, head and neck. Then respectfully drape the
patient by exposing the chest; lower the gown to the waist, keep the bra on in female patients.

Speak up and explain what you are doing, what you are looking for and your findings.

- General: Note if there are signs of respiratory distress (Tachypnea, inability to speak in
full sentences, use of accessory muscles of respiration, tripoding, pursed lip breathing,
nasal flaring, paradoxical indrawing of intercostal muscles and central cyanosis).
Examine the hands: look for nicotine staining, peripheral cyanosis, clubbing, feel the
pulse and check for flapping tremor.

- Head and neck: Look at lips and tip of the tongue for central cyanosis. Examine the
throat for congestion and palpate the sinuses for tenderness. Examine for tracheal
deviation, and then ask the patient to take a deep breath and examine for tracheal tug
that is a sign of hyperinflation. Examine the laryngeal height during expiration. Palpate
the lymph nodes of the head and neck.

The patient is speaking full sentences, which suggests open airway and
spontaneous breathing. I would like to check the vital signs (the examiner says
stable, or may give you values). There is no pursing of the lips, no nasal flaring, no
audible wheezes, and no use of accessory muscles of respiration. The patient is not
tripoding

I am examining the hands for nicotine staining, peripheral cyanosis and clubbing
and I don’t appreciate any.

Can you please open your mouth? Looking at the lips and tongue, no central
cyanosis. There is no throat congestion. Please let me know if it is sore, I’m

64
palpating the sinuses for tenderness which may suggest sinusitis. There is no
tracheal deviation; can you please take a deep breath? No tracheal tug. Can you
please breathe in then out and hold your breath? The laryngeal height is 5 cm. I am
palpating the lymph nodes in the head and neck. Starting with the occipital, post
auricular, pre auricular, submandibular, submental, anterior and posterior groups
of cervical lymph nodes and supraclavicular, infraclavicular lymph nodes. I don’t
feel any enlarged lymph nodes.

1. Examine the chest:

 Inspection: Look at the shape of the chest (kyphoscoliosis, pectus excavatum,


carinatum) and symmetry of chest expansion during respiration. Make a note of
scars, dilated veins, visible masses or skin changes.

 Palpation: Check for chest expansion (done by tape measurement at the level of
the nipples), palpate the chest for tenderness, deformities, subcutaneous
emphysema.

 Percussion: Percuss anteriorly and posteriorly, and over lungs apices. Compare
both sides and note any hyper resonance that may indicate pneumothorax.
Dullness on percussion may indicate consolidation, mass or effusion. Percuss for
diaphragmatic excursion.

 Auscultation: Use the bell of the stethoscope and compare both sides. Listen for
symmetry of breath sounds and note if breath sounds are bronchial or vesicular.
Also, note any crackles or wheezes. Ask the patient to say “eee” and listen for
egophony over the same areas. You expect to hear “aaa” over areas of
consolidation.

I am going to examine the chest starting with inspection. Looking anteriorly, posteriorly
and from the sides, there is no barrel chest, no pectus excavatum, no pectus carinatum,
no exaggerated kyphosis and scoliosis. The chest moves symmetrically with respiration.
There are no scars, no dilated veins, no visible masses and no skin changes. On palpation
of the chest, I will demonstrate chest wall expansion. By placing the measuring tape

65
around the chest at the level of nipples, I will ask the patient:, Can you please take a deep
breath in , hold for few seconds and then out? Chest expansion is about 5cm and
symmetrical. Next, I will do percussion over all lung spaces starting with the apex and
comparing both sides. Then proceeding to the upper, middle and lower lung zones. I do
not appreciate any dullness or hyper resonance. I am percussing now for diaphragmatic
excursion locating the level of the diaphragm, Can you please take a deep breath in and
hold? I’m marking the lower end of the diaphragm, can you please exhale and hold, I am
marking the upper level of the diaphragm. Normal diaphragmatic excursion is between 4-
5cm. I will now compare both sides. (The examiner may ask you to move on, if not you
can say I would do the other side but for the sake of time I will move on unless u want
me to demonstrate it again) I will now auscultate the chest. Can you please breathe in
and out through your mouth each time I place the stethoscope on your chest? I am
listening for breath sounds over the upper, middle and lower lung zones and comparing
side to side. Breath sounds are vesicular and symmetrical on both sides. I do not
appreciate any crackles, wheezes and pleural rubs. Can you please say “eeee”? I’m
listening over the same areas, there is no egophony.

Abdominal Examination
Edited by; Dr. Erin Toor

This section covers general comprehensive abdominal examination. Note that you need to
tailor your physical examination to the suspected pathology, and be even more specific in
emergency situations. This will be fully detailed in respective cases.

Abdominal exam includes;

1- Examination of the abdomen itself

2- General examination related to gastrointestinal pathology

Introduce yourself to the patient and explain what you are going to do. Then, take permission
and wash your hands. Before starting the examination, respectfully drape the patient by
covering him/her with the sheet. Lift the gown up and expose the abdomen from the nipples to
the pubic symphysis ensuring that the pubic area is covered. Do the examination in the
following order; Inspection, auscultation, percussion, palpation.

66
Inspection: Note the patient’s position; is he/she comfortable or in pain? Then inspect the
abdomen. Look for abdominal symmetry, movement of the abdominal wall with respiration,
scars, striae, dilated veins, Cullen’s sign (ecchymosis around the umbilicus seen in hemorrhagic
pancreatitis), Grey Turner’s sign (flanks ecchymosis seen in hemorrhagic pancreatitis, look at
the back as well), Caput medusa, masses, distention.

I am inspecting the abdomen noting that it is not distended and moving symmetrically with
respiration. There are no obvious masses, striae, surgical scars or dilated veins. There is no
obvious abdominal distention, caput medusa, Cullen’s sign or grey turner’s sign

Auscultation: Auscultate all 4 quadrants for bowel sounds. Listen for bruits in the following
locations:

1- Epigastrium: for Abdominal Aorta bruits

2- 2 cm above the umbilicus and 2 cm to either side of the midline for renal arteries
bruits

3- Over the liver in the right upper quadrant (Hepatoma, hepatic hemangioma,
arteriovenous malformations may have bruits)

I am listening for bowel sounds over the 4 abdominal quadrants and notice they are present
and within normal limits. I’m now listening for aortic bruits over the epigastric area. For renal
bruits, auscultating 2 cm above and to either side of the midline. There are no abdominal aortic
or renal bruits. Listening over the liver, there are no bruits.

Percussion:

1- General percussion over all the 4 quadrants looking for tenderness and dullness.
Note that percussion tenderness is a sign of peritoneal irritation (it is important to
show the examiner you are looking for signs of peritoneal irritation), always look at
the patient’s face for discomfort when you percuss.

67
2- Measure the liver span by percussing in the midclavicular line starting from the right
iliac fossa and going up; mark the border between dull and tympanic that represents
the lower border of the liver. Then start from the second intercostal space and go
down; the border between dull and resonant is the upper border of the liver.
Measure the distance between the upper and lower borders; that is the liver span.
Normal liver span is less than 13cm.

3- Percuss the spleen; percussion is more sensitive than palpation for splenic
enlargement. You need to do 2 maneuvers:

 Percussion over Traube’s space: marked by the 6th rib, mid-axillary line and
lower costal margin (on the left side of course). Percuss with the patient
breathing normally, dullness suggests splenomegaly. Other differential
includes food in the stomach, pleural effusion or lung consolidation.

 Percussion over Castell’s spot: Percuss the lower intercostal space in the
anterior axillary line with the patient breathing in expiration and full
inspiration. Dullness suggests splenomegaly, with the same differential as
Traube’s space dullness.

4- Ascites exam:

Shifting dullness: Percuss parallel to the midline. Start from the level of the umbilicus and go
down till you reach the border between dullness and tympani, take the patients’ permission
then mark the border. Ask the patient to lean to the opposite side and percuss starting from the
mark you made. Shifting dullness is noted to be present when the area of dullness becomes
resonant, continue to percuss and measure the new area of shifting dullness.

Do you have any pain? If the patient is in pain, start from the furthest point. I’m percussing all 4
quadrants looking for dullness or percussion tenderness, both negative. I will now measure the
liver span at the midclavicular line starting from the right iliac fossa, reaching the dullness point
marking the lower border of the liver. I will percuss now for the upper border of the liver
starting from the second intercostal space. Now marking the distance between the two, the
liver span is……cm. I will percuss for the spleen in Traube’s space marked by the lower costal
margin, the sixth rib and the mid-axillary line while the patient is breathing normally. There is
no dullness. I will check now Castell’s sign. This is percussion on the lowermost intercostal
space in the anterior axillary line while the patient is breathing in full inspiration and expiration.
There is no dullness over the Castell’s spot. I will now do shifting dullness to check for ascites.
With the patient lying supine I am percussing parallel to the midline starting at the umbilicus,

68
reaching the dullness-tympanic border; can I mark the spot on your skin? Can you lean to the
left side please? I start percussion at the mark moving toward the umbilicus I notice that the
dullness-tympani border didn’t shift so the test is negative for ascites.

Palpation:

1- Superficial palpation: Start palpation at a site away from pain. Feel all the 4
quadrants. Look for guarding; a sign of peritoneal irritation.

2- Deep palpation: For deep masses and tenderness. Feel all 4 quadrants. Look for
rebound tenderness; a sign of peritoneal irritation.

3- Palpate for the liver. Ask the patient to breathe deeply in and out. Start from the
right iliac fossa in the mid-clavicular line, feel during inspiration, and move up 1cm
each time during expiration. Also palpate in the epigastrium. If you feel the liver
describe it:

 How many cm below the costal margin does it extend?

 Edges: sharp or round?

 Surface: smooth or nodular

 Any masses?

 Is it soft, firm or hard?

 Is the liver tender?

 You have already measured the liver span when you did percussion.

4- Palpate for splenic enlargement: Start from the right iliac fossa and move obliquely
to the left upper quadrant. Then start from the left iliac fossa and move up. The
spleen may enlarge in either direction. Use the same technique you used for liver
palpation. If you don’t feel the spleen, ask the patient to lean to the right side and
try to feel for it. Percussion is more sensitive than palpation for splenic enlargement.
If it was negative it is unlikely that the spleen will be palpable. If you feel the spleen
then you can differentiate it from the kidney by the following:

 Splenic notch is felt in splenic enlargement not kidney

 The splenic surface feels smooth and regular unlike the kidney

69
 The spleen is always unilateral, while kidneys can be bilaterally enlarged (as
seen in polycystic kidneys)

 The spleen is always dull, the kidney can be dull or tympanic

 The kidneys are ballotable, the spleen is not

 The spleen moves diagonally (towards the left lower quadrant) with
respiration while the kidneys move vertically

5- Palpate for the kidneys: Place your left hand on the patient’s back between the
costal margin and iliac crest. Feel for the kidney with your right hand and use your
left hand to lift the kidney and see if it’s ballotable. Check both sides. Examine for
costophrenic angle tenderness with the patient sitting up (You may defer it to the
end to avoid moving the patient unnecessarily)

6- Palpate for Abdominal aortic aneurysm; described in CVS P.E (Cardiovascular


physical examination) above

7- Elicit signs of Appendicitis, include;

 Pin point tenderness at McBurney’s point: 1/3 the distance from the anterior
superior iliac spine to the umbilicus

 Rebound tenderness at McBurney’s point

 Rouvsing’s sign: Palpation of the right lower quadrant produces pain in the
left lower quadrant

 Psoas sign: Abdominal pain on active flexion of the hip

 Obturator sign: Abdominal pain on internal rotation of the flexed hip

8- Palpate for gallbladder pathology:

 Murphy’s sign: Ask the patient to breathe out, then place your hand on the
patient’s abdomen in the right midclavicular line below the costal margin.
Ask the patient to take a deep breath in. If the patient has cholecystitis,
he/she will have pain with inspiration (positive sign) due to tender
gallbladderand will hold his/her breath.

70
 Courvoisier’s sign: Is a palpable gallbladder. This can be seen in cases that
cause acute distension of the gallbladder. For example: pancreatic
malignancy

Do you have pain in your tummy right now? I’m starting with superficial palpation, feeling for
any masses or tenderness in all 4 quadrants. I notice there is no guarding. Looking at the
patients face, there is no tenderness. I will do deep palpation looking for deep masses or
tenderness. Both are negative. I will palpate now for the liver in the midclavicular line, can you
breathe in and out for me please? Feeling on inspiration and advancing with expiration, I don’t
feel the liver edge. I will use the same technique now to feel for the spleen, starting first from
the right iliac fossa and moving towards the left upper quadrant, I do not feel the splenic edge. I
will do the same to feel for the spleen starting from the left iliac fossa this time. I will now
palpate the kidneys, both are not palpable. I will check now for signs of appendicitis; looking for
pin point tenderness and rebound tenderness at McBurney’s point. Does it hurt when I let go?
Can you bring your right knee to your chest? Does this cause pain in your abdomen? The Psoas
sign is negative. I’m going now to move your leg, let me know if it hurts. Internally rotating the
right hip looking for obturator sign and it is negative. I will now demonstrate Murphy’s sign. I’m
pressing at the midclavicular line just below the right costal margin. Can you take a deep breath
and hold your breath? The patient didn’t catch their breath, so the Murphy’s sign is negative. I
don’t feel an enlarged gallbladder; hence Courvoisier’s sign is negative.

71
Neurological P.E
Edited by; Dr. M. Wasif Hussain

Includes six main examinations; mental status, cranial nerves, motor system, sensory system,
cerebellar exam and examining the gait. Mental status if directly asked should be done through
a Mini Mental status exam (MMSE) or Montreal cognitive assessment (MOCA) exam, but
otherwise can be conducted through a Glasgow coma scale, which will be discussed at the end
of this section. However, MMSE should be done in cases of delirium.

Cranial nerves

Include:

1- CN 1 (Olfactory); Test each nostril by asking the patient to close their eyes and one
nostril then try to identify a known smell like coffee. Repeat with the other side.
During the exam, look at the examination table, if there was coffee or a special smell
on it go ahead and examine the olfactory nerve. However, in most cases there will
be nothing in the room to test CN1 with. In this case just describe what would you
do.

2- CN2 (Optic); involves looking for five things;

 Visual acuity; using Snellen chart or near card examine each eye at a time. If
there isn’t a chart or card in the room, describe what you would do.

 Color vision; done with Ishihara chart. In most cases you would just mention
it and the examiner will ask you to move on

 Visual fields by confrontation; stand or sit at eye level with the patient. Test
each eye at a time. Ask the patient to cover one eye, and cover or close your
corresponding eye to compare your vision to that of the patient’s (i.e. cover
your right eye to examine the patient’s left eye). Use your pen or finger at a
distance that is approximately at the midpoint in between you and the
patient and ask the patient to let you know when he/she can see it,
alternatively you may do the counting method by asking the patient to count
how many fingers they see; test 4 quadrants for each eye; superior and

72
inferior nasal, and superior and inferior temporal. Provided you have normal
visual fields the normal patient can see the object or fingers when you can.
Test for extinction with the patient’s both eyes open, hold both of your
hands up and while the patient is staring at your nose, ask them to point to
the hand that is moving. Start by moving each hand individually and then
both simultaneously. A patient with extinction will neglect the contralateral
side.

 Pupillary reflex; shine a light into the patient’s eye (You should bring your
own to the exam), and notice the pupil’s direct (the eye to which you shine
the light at) and consensual reflex (constriction of the other pupil). Do the
swinging light test by moving the light in a swinging motion from one eye to
the other, holding for approximately 1-2 seconds on each eye to test for
RAPD (relative afferent pupillary defect). Note that the pupillary reflex
afferent limb is the optic nerve; the efferent is the Oculomotor nerve.

 Fundoscopy; Is detailed under Fundoscopy P.E

3- CN3 (Oculomotor), CN4 (Trochlear), CN6 (Abducens); Inspect the eyes for pupils
shape, size, position. Note if both sides are symmetrical and if the patient has
nystagmus at primary gaze. Then test the extra ocular movements; Ask the patient
to keep his/her head still, and follow your finger; draw an H and notice both eyes
moving in all direction. Look for any nystagmus and ask the patient to report any
diplopia.

4- CN5 (Trigeminal); Has two components;

 Motor; CN5 innervates the muscles of mastication. Ask the patient to clench
his/her teeth together and feel for the masseter muscle on both sides. Also
feel for temporalis muscles. Ask the patient to open his/her mouth against
resistance testing the power of Masseter and Temporalis muscles. Then
check the power of Ptyregoid muscle by asking the patient to move his/her
jaw right and left resisting your hand pushing in the opposite direction.

 Sensory; CN5 supplies the skin of the face. Use a pin to test pinprick over the
3 branches of CN5; frontal (V1), maxillary (V2) and mandibular (V3). Compare
both sides. Do the test with the patient’s eyes closed.

5- CN7 (Facial); Inspect the patient face for symmetry and ask the patient to make a
variety of facial expressions; Wrinkle his/her forehead or raise his/her eye brows to
differentiate an UMN facial palsy from a lower (UMN spares the frontalis) Close

73
his/her eyes tight and resist your try to open them, Smile; note symmetry of both
lower lips and Nasolabial folds, Close his/her mouth and resist you trying to open it.
Puff his/her cheeks and resist your try to pop them

6- CN8 (Vestibulocochlear); Test to cochlear component. Whisper into the patient’s ear
while distracting the other ear by rubbing your fingers together. Ask the patient to
identify what you have just whispered into his/her ear, repeat with the other ear.
You are not required to bring a 512 tuning fork to the examination. However, if
there was one in the room, do Weber and Rinne test; Weber; Place the tuning fork
on the patient’s forehead and ask him/her if it’s different between the two ears.
Rinne test; Put the tuning fork on the mastoid process and ask the patient to let you
know when the sound stops. Immediately place the fork in front of the patient ear.
Air conduction is normally stronger than bone conduction. You don’t need to test
the Vestibular component.

7- CN9 (Glossopharyngeal) and CN10 (Vagus); test them together; look into the
patient’s mouth and notice any deviation of the palate or any asymmetry of palatal
movement when the patient says: Ahh. Mention that you could examine the gag
reflex to be thorough, but this will rarely be asked for the sake of the actor’s
comfort. Note, a palatal deviation is due to weakness on the contralateral side. Test
articulation by asking the patient to say; Pa Ta Ka.

8- CN11 (Accessory); Ask the patient to shrug his/her shoulders up and resist your
hands pushing down. Test the power of Sternocleidomastoid muscle by asking the
patient to turn his/her head to either side and resist your hand pushing on the
lateral side of the chin.

9- CN12 (Hypoglossal); Ask the patient to open their mouth and notice any
fasciculations or atrophy. Then ask the patient to stick his/her tongue straight out
and notice any deviation. Ask the patient to move his/her tongue from side to side.
If there is a question of weakness, you can ask the patient to push their tongue into
their cheek and resist you pushing against it. Note, a tongue deviation is due to
weakness on the ipsilateral side.

The first cranial nerve is the Olfactory, which I test by asking the patient to identify a known
smell using one nostril at a time with his/her eyes closed. I examine the 5 components of CN2
starting with visual acuity using a Snellen chart, and color vision using Ishihara chart (the
examiner will most likely say move on). Then I test the visual fields by confrontation test; can

74
you please close your right eye and tell me how many fingers you see. Superior Temporal,
Superior Nasal, Inferior Temporal, Inferior Nasal. I will do the same on the right side; can you
please close your left eye? I will now test for extinction with the patient’s both eyes open; how
many fingers do you see? (point to the hand that is moving) I will shine a light into your eyes.
I’m doing the pupillary reflex noting the direct and consensual reflexes. Then I will do the
swinging light test looking for RAPD. All tests are normal so far. The final component of the
optic nerve exam is Fundoscopy; can you please focus on that point on the wall, I will dim the
light and look into patient’s eyes looking for papilledema. Next I will examine 3 cranial nerves
together; 3rd, 4th and 6th ; I’m inspecting the pupils I notice that both sides are symmetrical
2mm in diameter with normal shape and position and no nystagmus. Can you please keep your
head still and follow my finger? Please let me know if you have double vision at any point. The
patient has normal extraocular movements and no nystagmus. Now I will examine CN5 starting
with its sensory component; can you close your eyes and nod your head each time I touch your
face with the pin? Does it feel the same on both sides? I’m testing V1, V2 and V3. Now moving
to the motor component; can you please clench your teeth together? I’m palpating the
masseter muscle and it feels normal. I’m also palpating the Temporalis. Can you please open
your mouth and don’t let me close it? Testing the same two muscles. Can you move your jaw to
the left and resist me? Great now to the right. I’m testing Pterygoid. The trigeminal nerve is
intact. Next is the facial nerve, I notice that the face is symmetrical, Can you raise your
eyebrows? Forehead wrinkles are symmetrical. Can you close your eyes and don’t let me open
them? Can you smile please? No mouth drop and the Nasolabial folds are symmetrical. Now
close your mouth and don’t let me open it. Can you puff your cheeks and resist me trying to pop
them? The facial nerve is intact. Now cranial nerve 8, the Vestibulocochlear, I will test the
hearing component; I’m going to whisper in your right ear? Say anything like table, chair or a
number. What did I say? Will do the same on the left ear, what did I say? I will do Weber test to
see if there is any lateralization, placing the tuning fork on the patient’s forehead, do you hear it
in both ears? Does it sound the same? Next I will do Rinne test placing the tuning fork on the
mastoid process, let me know when the sound disappears? Putting the fork in front of the ear;
do you hear it now? Air conduction is normally stronger than bone conduction, CN8 is intact. I
will test now the Glossopharyngeal and Vagus nerves. Can you open your mouth and say Ahh?
There is no palatal deviation, the palatal movement is symmetrical. Can you say Pa Ta KA?
Articulation is intact. I will do the gag reflex. The examiner may say pass as the test is not
comfortable. Both nerves are intact. Next is the accessory nerve; Can you shrug your shoulders
up? Don’t let me bring them down. Move your face to the right and resist me. Now to the left.
The accessory nerve is intact. The last nerve is the hypoglossal; can you open your mouth
please? No tongue fasciculation, atrophy, or asymmetry. Can you stick your tongue straight
out? There is no deviation. Can you move you move your tongue from side to side? The
hypoglossal nerve is intact.

75
Motor examination;

Includes;

1- Inspection; Look at the patient posture, note muscle’s bulk and symmetry.
Observe for fasciculations or abnormal movements.

2- Tone examination; you need to examine the tone of the upper and lower limbs.
Ask the patient to relax and passively move each limb initially slowly throughout
looking for rigidity and then having a fast phase on extension/supination in the
upper extremity and on flexion in the lower extremity looking for spasticity.
Spasticity is velocity dependent while rigidity is not. If the patient is not fully
relaxed try distracting him/her by talking to them.

 For the upper limbs do the following; flexion and extension of the elbow,
pronation and supination, rotation at the wrist to look for cogwheel
rigidity seen in Parkinson’s.

 For the lower limbs do; internal and external rotation at the hip, flexion
and extension at the knee (with the patient lying supine and the legs fully
extended; briefly left the knee- illustrated in the video). Flexion and
extension of the ankle. Test for clonus as well.

3- Power; you need to rate each muscle group you test. On a scale from 0-5; 0
complete paralysis, 1 flicker of movement, 2; movement with gravity eliminated,
3; movement against gravity but no resistance, 4; movement against gravity and
some resistance (often graded as 4-, 4, or 4+), 5; full power. You don’t need to
mention the exact nerve or nerve roots you are testing; it is enough to name the
movements. Give the patient clear instructions; mimic the movement you need
them to do if you have to. For all muscle groups except deltoids and hip flexors
always support proximally and test distally and compare the two sides. Start with
pronator drift followed by select muscle groups. If there is weakness in a focal
area, you will need to examine the muscles in more detail (these will be written
in italics).

76
 Test for pronation drift; very sensitive for UMN weakness in the upper
extremity; ask the patient to hold his/her arms straight forward with the
palms up, fingers extended and eyes closed and notice if any of the arms
drop, pronate and fingers flex (not all need to be present for a positive
drift).

 Shoulders abduction, Shoulder adduction

 Elbows flexion and extension

 Wrists flexion and extension

 Fingers abduction, extension and flexion

 Thumb Abduction

 Hips flexion and extension and abduction and adduction

 Knees Flexion and extension

 Ankle dorsiflexion and plantar flexion and inversion and eversion.

4- Reflexes; do deep tendon reflexes (DTR). You need to mention the name of the
reflex you are doing and the nerve root. Ask the patient to relax, and warn
him/her that you are going to tap with a hummer to test their nerves. Watch and
feel for muscle contraction. Grade the reflexes on a scale from 0-4: 0; no
reflexes, 1; decreased, 2; normal, 3; increased (brisk, with spread), 4; increased
with clonus. Test the following (note an easy way to remember the roots is listed
below as 12345678);

 Ankle jerk; S1,S2

 Knee jerk; L3, L4

 Brachioradialis; C5.6

 Biceps jerk; C5.C6

 Triceps;C7, C8

If you can’t elicit a reflex, try asking the patient to clench his/her teeth for upper limbs reflexes,
and lock fingers and pull hands apart for lower limbs reflexes.

77
Examine the plantar reflex after doing the ankle jerk. Warn the patient that you are going to
stroke their foot and that it may be uncomfortable, and move the sharp part of your hammer
along the lateral side of the plantar aspect of the foot turning in just before the toes forming an
arc. Watch if the big toe goes up or down. (a Babinski sign is an up going plantar)

As part of the motor exam you need to be able to differentiate upper motor neuron (UMN)
from lower motor neuron (LMN) lesions. Refer to table 3-1 for explanation. Note that mixed
UMN and LMN signs are seen in motor neurone disease such as ALS.

UMN LMN

Muscle weakness, NO fasciculations Muscle atrophy and weakness WITH


fasciculations

Hypertonia with spasticity Hypotonia

Hyper-reflexia Hypo-reflexia

Up-going Plantar reflex Down going Plantar reflex

Table 3-1; Difference between UMN and LMN lesions.

I’m examining the motor system starting with inspection; no abnormal posturing, movements,
or fasciculations. The muscles are symmetrical with no atrophy. I will start by testing the tone.
Can you make your arm floppy for me? Flexing and extending the elbow slowly then rapidly,
now pronating and supinating the hand, rotating the wrist looking for cogwheel rigidity. I will do
the same with the other arm. The tone of the upper limbs is normal. Can you make your leg
floppy for me? Internal and external rotation of the hip, flexion and extension of the knee,
ankle dorsiflexion and plantar flexion. I will test for clonus as well. Now I will compare both
sides. Lower limbs tone is normal with no clonus. Next I will test power. I show patients the
movement I need them to do. Can you put your arms out straight in front of you with your
palms up and fingers straight and close your eyes? There is no pronator drift. Can you go like
this and resist me? Shoulder abduction and adduction. Now can you go like this and resist me;
Elbow flexion and extension. Move your wrist up and don’t let me break you, now down, wrist
flexion and extension. Spread your fingers and don’t let me bring them together; fingers
abduction. Keep your fingers straight and don’t let me bend them. Curl your fingers and don’t
let me straighten them (alternatively you can check grip strength). Finger extension and flexion.
Comparing both sides, upper limbs power is 5/5. I will now do the lower limbs with the patient

78
lying supine; Lift your leg up off the table with your knee straight and resist me pushing you
down; Hip flexion (Note ; test hip extension at the end of the exam by asking the patient to lie
prone). Bring your hips apart and don’t let me bring them in, now the opposite. Hip abduction
and adduction. Bend your knee and kick out, resist me, now in. Knee flexion and extension.
Press down with your foot as if you are pressing the gas, ankle plantar flexion now bring your
foot up, ankle dorsiflexion , now out; eversion, and in; inversion. Lower limbs power is 5/5. Next
I will do the DTR. I’m going to test your nerves by taping with a hummer, this shouldn’t hurt at
all. Starting with the ankle jerk S1, S2, comparing both sides, now the knee jerk L3,L4. Normal
lower limbs reflexes. I’m testing Brachioradialis C5.C6, then Biceps C5. C6, and finally Triceps
jerk C7. Normal upper limbs reflexes. I will now do the plantar reflex. I’m going to stroke your
foot with a sharp object, it may be a little uncomfortable, is that ok? plantars down going
bilaterally.

Sensory examination;

The sensory examination involves testing a wide variety of sensations and can take
a tremendous amount of time. Make sure you know your dermatomes very well (figure 3-4)
Practice and master the following for the sake of the examination. Always start by eliciting a
history of any sensory complaint and focus on the areas mentioned.

1- Pinprick; using a pin, which you will find on the table in the examination room, ask the
patient if he can feel it with his/her eyes open at a single location so that the sensation
is recognized. Then test the upper extremities with the patient’s eyes closed comparing
both sides in such a way that you are testing proximal sensation, distal sensation
(looking for a stocking glove pattern of peripheral neuropathy) and also dermatomes
and nerve distributions. Note that in most cases it is sufficient to test the upper and
lower limbs. Nevertheless, if spinal injury is suspected you need to test the neck and
trunk as well. The same principle apply for testing other sensory modalities like fine
touch, and temperature, but it’s unlikely to be asked to do them especially with the time
restraint of OSCE stations. Follow the link below for dermatomal distribution;

https://www.pinterest.com/pin/487162884665368556/

79
2- Vibration sense; use a 128 tuning fork , place it on the patient sternum at first to
identify the sensation. Then test over bony prominence, ask the patient to let you know
when it stops. Start with the interphalyngeal joint of the fifth metatarsals of the lower
limbs bilaterally. If vibration is preserved distally you don’t need to test more proximal
locations. If it is lost you need to test proximally until the level becomes evident. To test
vibration, you will measure the duration of sensation felt by the patient until they do
not feel it anymore and compare it to the other side as well as your thumb (presumed
normal control)

3- Proprioception; test the thumb and big toe. First show the patient what you will be
doing with eyes open before doing the test with the eyes closed. You will move a single
joint (interphalyngeal) up or down relative to the previous position. Start with large
amplitude movements followed by more subtle movements.

4- Romberg test; this is classically tested after gait assessment. Ask the patient to stand
with his/her feet together with arms either extended forward or straight by their side
and with eyes open. Observe balance. Then ask the patient to close their eyes for 5
seconds. A positive sign is a fall not swaying .Make sure to stand behind the patient to
catch him/her if needed. Balance is lost with eyes closed in cases of proprioception loss
as vision compensates for loss of joints position feeling in space. While 50% of patients
with cerebellar disease maintain their balance at 60 seconds.

Next is Sensory exam; I will start with pinprick. Do you feel a sharp poke on your hand? I need
you to close your eyes and say yes each time you feel the pin. Let me know if it doesn’t feel the
same on both sides. C4 left and right, C5,C6,C7,C8,T1. Now the lower limbs; L1, L2, L3, L4, L5,
S1. The same principle applies for testing fine touch, cold and hot sensations I will now examine
proprioception holding the patient thumb, that’s up and that’s down, can you close your eyes
and let me know if it’s up or down? I will compare both sides, then I will do the same with the
big toe; this is up and this is down, can you tell me if it’s up or down? Comparing both sides is it
up or down? (Note that you need to move each finger/toe multiple times,for example; up up
down up). Next I will test your vibration sense with a tuning fork; that’s how it feels.. Can you
close your eyes? Do you feel the vibration? Let me know when it stops. Next I will do the
Romberg test; Can you stand up and bring your feet together, extend your arms in front of you.
Now close your eyes. The patient is steady and the test is negative.

80
Cerebellar exam

Be sure to note the handedness of the patient as that may explain mild
asymmetries of coordination. Ataxia should always be out of keeping with any weakness or
sensory loss. Exam Includes;

1- Speech; note if speech is normal.

2- Extraocular movements looking for nystagmus and saccadic movements.

3- Finger to Nose test; ask the patient to touch your finger then his/her nose as fast and
accurate as he/she can. Change the position of your finger, and try to get the patient to
fully extend his/her arm as he/she tries to reach for your finger.

4- Rapid alternating movements; ask the patient to keep the palm of one hand up and tap
on it with the other hand alternating palmar and dorsal sides as quickly as possible (i.e.;
pronating and supinating the arm) Switch hands to test the contralateral side.

5- Heel-shin test; ask the patient to slide his/her heel on the shin moving up-down –up in a
straight line. You may want to show the patient how to do it. Test both sides. This test
should ideally be done while lying supine but can be done while sitting.

6- Gait; detailed next, as gait is an important part of general neurological exam as well. In
cerebellar exam ask the patient to walk heel to toes in addition to general gait
examination. Make sure to walk along with the patient to catch him/her in case he/she
is unstable.

I start my cerebellar exam by noticing that the patient’s speech is normal. Follow my finger with
your eyes, try to keep your head still and let me know if you see double. Can you touch your
nose then my finger as quickly and accurately as you can? Can you go like this for me? (Show
the patient how to do rapid alternating movement). Can you slide your heel on your shin going
from your foot up to your knees and back to the foot? (Good idea to demonstrate to the
patient how it’s done), Can you walk for me? Turn around and come back? Can you walk on
your tiptoes? Can you walk in a straight line touching your heel with your foot? (Also show the
patient how to do it) Now walk on your heels? The gait is smooth with normal base and arm
swing, normal turning, no imbalance, no difficulty with tiptoes or heels. (Always walk with the
patient and be ready to catch him/her if they lose balance)

81
Gait

Obviously you will ask the patient to walk! Observe the stance, base, smoothness of the gait,
arm swing, balance. Comment on patient’s balance. It is important to walk along with the
patient so that you can catch him/her if balance is lost. Ask the patient to turn around and ask
the patient to walk on the tip toes (S1) and heels (L4). Make sure to support the patient if
needed. If Parkinsons disease is suspected, look for festination of gait, shuffling, turning en bloc
and perform the pull test while standing behind the patient; warn the patient that you are
going to pull him/her to the back. A fall or festination (taking multiple steps to the back-one
step is normal) is a positive sign.

Refer to the cerebellar exam above for suggested verbalization of gait examination.

Glasgow Coma Scale

Do it in all trauma and coma cases if asked to assess, manage or perform a physical exam.

Score range is 3-15 covering 3 major functions;

1- Eye opening (4 points in total) ; 1; none, 2; to pain, 3; to voice, 4; spontaneous

2- Best motor response (6 points) ; 1; none, 2; Extension to pain (Decerebrate


posturing), 3; Flexion to pain (Decorticate posturing), 4; withdrawal from pain,
5; localizes pain, 6; Obeys commands

3- Best Verbal response (5 points) ; 1; none, 2; incomprehensible sounds, 3;


inappropriate words, 4; confused, 5; oriented

I will calculate Glasgow Coma Scale to assess patient’s consciousness; starting with the first
component eye opening, I notice the patient opens his/her eyes spontaneously that’s 4 for eye
opening. Then I will assess the best motor response can you left your right arm up for me
please? The patient obeys a command that’s a 6 for best motor response. Then the best verbal

82
response; what is your name? Where are you? What is todays date? Who am I? The patient is
oriented and speech is normal, that’s a 5 for best verbal response. Total Glasgow Coma Scale is
15.

83
Musculoskeletal System Examination (MSK)
Edited by Dr. Stephanie Keeling

Family doctors in Canada are the first gate to all sorts of medical problems. Unlike other
countries where orthopedic surgeons and rheumatologists may be the first physicians to assess
a patient with joint pain, family doctors in Canada often see these patients first, determine the
pathology and then refer as appropriate. Thus, a good screening musculoskeletal exam is
common in Canadian OSCEs.

The sequence of the MSK examination differs somewhat from a general examination. Do the
following;

1- Inspection: Look for SEADS; swelling, erythema (redness), atrophy of muscles,


deformity and skin changes. Inspect how the joint looks when it is moved.

2- Palpation: Feel for temperature difference first, then palpate for tenderness
naming the muscle, ligament or tendon you are palpating.

3- Range of motion: test both active (performed by the patient) and passive
(performed by you) giving the patient clear instructions and naming each
movement as you go. Look for pain, listen and feel for clicks or crepitus and
observe limitations of movement.

4- Special tests: will be specific for each joint.

5- For completeness: say you will examine the other joints including the joint above
and below, and the neurovascular compartment of the limb. In most cases, you
will run out of time after doing the special tests. It is important however to
mention that you would do all of the above. One exception to keep in mind is that
you must include neurovascular examination of the lower limbs when you
examine the back.

In this section I will cover the P.E of the hands and wrists, elbows, shoulders, back, hips, knees
and ankles.

The three most common MSK complaints include back, knee and shoulder pain. Practice them
well.

Hands and Wrists P.E

84
- Start by stating that you will examine and compare both sides, examining the elbows
and hands, and the neurovascular compartments of the arms. Focus on the affected
wrist.

- Inspection: compare both sides, looking at the palmar (volar) and dorsal aspects of the
joint. Look for SEADS (see above)

- Palpation: feel for temperature difference with the dorsum of your hands. Then feel
the affected joint with both hands looking for any tenderness or swelling.

- ROM: Includes:

 Active: Ask the patient to do the following movements and demonstrate to the
patient what you need him/her to do:

1- Flexion; by bringing the dorsum of the hands together with the fingers
pointing downwards

2- Extension; by doing the prayer sign.

3- Ulnar deviation

4- Radial deviation

5- Arm pronation

6- Arm supination

 Passive movements; Ask the patient to make his/her wrist floppy for you and
move the wrist in all 6 directions covered in active movements.

- Special tests: Include:

 Testing for Carpal Tunnel Syndrome (CTS):

1-Katz diagram; ask the patient to draw for you where exactly he/she feels the pain/tingling;
pain felt at the fingers, wrist or proximal to the wrist is classic for CTS, pain felt in the palmar
aspect and the first, second and third fingers indicates probable CTS. Pain felt in the radial
nerve distribution makes CTS unlikely.

2-Hypoalgesia: reduction of sensation to painful stimuli over the palmar side of the index (2 nd)
finger compared to the little (5th) finger.

85
3- Weak thumb abduction

4-If you have more time and can complete the power and sensation testing described below,
consider asking them to do Tinel’s (tapping over the distal crease on the median nerve
produces paresthesia the median nerve distribution) and Phalen’s (flexing the wrist at 90 for 60
seconds produces paresthesias in the median nerve distribution) tests although they have low
likelihood ratios, because they are so classic

 Allen’s test

Particularly important in trauma cases, and prior to performing a radial artery puncture for
arterial blood gases. Refer to page 14 of this chapter for full description. Feel the radial pulse
then proceed with the test. Only proceed with this if you have completed everything else
including power and sensation testing described below.

- Test the power of the wrist asking the patient to resist all the above movements. Then
test sensation over the radial, ulnar and median nerves. Do upper arm DTR and feel
the pulses. If you run out of time, inform the examiner that you would intend on
performing power and sensation testing in a clinical setting. In most cases you will not
have time to examine the joint above and below but it is always a good idea to
indicate you would do it.

I will examine both wrists, starting with the right (do the affected first), then I will examine the
elbow and the hand and the neurovascular compartment of the upper limbs. I start with
inspection comparing both sides looking at the dorsal and palmar aspects of the wrists, no
swelling, erythema, no thenar or hypothenar atrophy, no deformities, and no skin changes. Do
you have pain right now? I’m feeling for temperature difference, and now palpating the wrist
for any tenderness or swelling, also feeling over the snuff box for scaphoid fractures. I will now
check active ROM; can you copy my movements please? Extension, flexion, ulnar deviation and
radial deviation, arm pronation and supination. Next I will do the passive range of movement
looking for any limitation, pain, or clicks/crepitus, I’m going to move your wrist, can you make it
floppy/relax for me? Flexion, extension, ulnar deviation, radial deviation, pronation and
supination. No abnormalities. I will now do special tests for CTS: Do you get pain in your hands
or wrists? Can you draw for me exactly where you feel the pain (Comment if Katz diagram is
classic, probable or unlikely). Then I will test for hypoalgesia: This will feel sharp. Please close
your eyes and tell me if you feel it? Is the feeling the same over the index (2nd) and little (5th)
fingers? Then I will test the power of thumb abduction. Can you resist me pushing your thumb
please? Next I will tap on your wrist let me know if it hurts or feels tingly. Tinel’s test is negative.

86
Can you bring both your hands together with the fingers facing down and hold for 60 seconds?
Phalen’s test is negative as the patient didn’t experience numbness, pain or paresthesia

Elbows P.E

- State that you will examine both sides but will start with the affected one first and you
will examine the shoulders, wrists and neurovascular compartments of the upper
limbs

- Inspection: Expose both upper limbs; look for SEADS comparing both sides

- Palpation: feel for temperature first, and then feel the joint for any swelling, nodules
or tenderness paying particular attention to the olecranon process, medial and lateral
epicondyles.

- ROM: includes flexion, extension, pronation and supination. Test both active and
passive ROM looking/feeling for any limitations, hyperextension, pain, or crepitus. A
flexion deformity (missing full extension) will suggest an elbow effusion and should be
palpated.

- Special tests:

 Lateral epicondylitis: palpate the lateral epicondyle with the elbow extended,
forearm pronated and wrist flexed, feel for any tenderness (seen in tennis
elbow; think about the movement you make when you play tennis); pain at the
lateral epicondyle with resisted extension of the elbow suggests tennis elbow.

 Medial epicondylitis: palpate the medial epicondyle with the elbow extended,
forearm supinated and wrist extended looking for pain (seen in golfer’s elbow);
pain at the medial epicondyle with resisted flexion of the elbow suggests
golfer’s elbow.

87
I am going to examine both elbows and compare both sides but focusing on the right. Then I
will examine the shoulders and wrists and the neurovascular compartment of the upper limbs. I
start with inspection, no swelling, erythema, no muscle atrophy, no deformities, and no skin
changes. I will feel your elbow; please let me know if it hurts. I am looking for temperature
difference, feeling for effusions and palpating the dorsal aspect of the forearm, the medial and
lateral epicondyles and the olecranon process. No swelling or deformities. Next I will do the
Active ROM. Can you copy my movements please? Flexion, extension, pronation, supination.
Can you relax your arm for me and I will move it for you, doing the same movements you just
did. No limitation of movement, no crepitus, no pain and no hyperextension. Now I will test for
tennis elbow noting any pain as I palpate the lateral epicondyle and with resisted extension.
Next I’m testing for golfer’s elbow, feeling for any pain over the medial epicondyle and with
resisted flexion.

Shoulder P.E

The shoulder is probably the most complicated joint. There are so many special tests. I will
cover basic knowledge about shoulder anatomy and P.E in this section to help you as a
generalist determine if the pain is arising from the shoulder, surrounding tissues or referred
from another site.

Rotator cuff tears and tendinitis are very common so knowing the muscles that make up the
rotator cuff and their specific actions will help you conduct a meaningful P.E. Other common
pathologies include: bicipital tendinitis, impingement, frozen shoulder and anterior dislocation.

Four muscles make up the rotator cuff (SITS):

1- Supraspinatus Abduction

2- Infraspinatus External rotation

3- Teres Minor External rotation

88
4- Subcapularis Internal rotation

Follow the link below for shoulder joint anatomy;

http://www.webmd.com/pain-management/picture-of-the-shoulder

- Start shoulder P.E by stating you will examine both sides but will focus on the affected
side, and that you will also examine the joint above (cervical spine) and below (elbow)
and the neurovascular compartment of the upper limbs.

- After asking permission to examine the patient, wash your hands (or use hand
sanitizer as done in some OSCEs) and respectfully drape the patient by lowering the
gown exposing both shoulders, keep the bra on in female patients. Ask the patient to
report any pain or discomfort during the P.E.

- Inspection: compare both sides looking at the shoulders from front, back and sides.
Noticing any SEADS; swelling, erythema, muscle atrophy, deformities and skin
changes. You can ask the patient to complete a shoulder arc test at this point and
inspect the shoulder as they elevate the arm in the scapular plane then bring it down
again. You are looking for any abnormalities including difficulty completing this
motion. Alternatively, you can complete this later in the ROM section.

- Palpation: check for temperature difference. Feel for tenderness and swelling starting
from the sternoclavicular joint, palpate along the clavicle, acromion, acromioclavicular
joint, humerus head, bicipital tendon in the bicipital groove, spine of scapula,
supraspinatus and infraspinatus muscles and medial border of the scapula.

- ROM: Includes

 Active: Ask the patient to complete the following movements:

1- Flexion: Move the arm forward

2- Extension: Moving the arms back, often completed with the elbow bent

89
3- Abduction: Bring the arm away from the body in the scapular plane until
the hand is above the head, and the shoulder/upper arm is as close as it
can get to the ear without using the trapezius muscles to lift up the
shoulder girdle.

4- Adduction: Bring the abducted arm back to the side.

5- Internal rotation: Ask the patient to flex the elbow to 90 degrees, and then
medially rotate the arm at the shoulder, bringing the hand to rest on the
abdomen. The flexed elbow remains in the same position resting at the
patient’s side throughout this movement.

6- External rotation: Same position as in internal rotation but ask the patient
to rotate the arm at the flexed elbow laterally, away from the abdomen as
far as it can go. The flexed elbow remains in the same position resting at
the patient’s side throughout this movement.

7- Adduction and internal rotation: The dorsum of the hand is touching the
back (note that you have already tested each of these movements
separately, you have the option to ask the patient to do this movement as
you test more than one muscle at the same time)

8- Abduction and external rotation: The patient puts his hands behind his
head with the palms touching the head.

 Passive: Ask the patient to relax, and gently move the shoulder in all the above
directions noting any limitation, pain, clicks or crepitus.

- Special tests: NOTE: It is key to do the general physicial exam well and know a few
vital special tests. If you focus on special tests but miss the general examine
maneuvers you will score low.

As I mentioned, you have a long list to choose from. Don’t panic! The most important tests
(which are also the easiest to remember) include those that test the four rotator cuff muscles.
If you can complete the tests for the 4 SITS muscles, you will know where the pathology is for
the majority of patients. For the other tests listed, you may need to do them depending on
what the patient has, but they are less critical in the general clinic setting.

 Painful arch (Shoulder Arc Test): Pain between 70 and 100⁰ during shoulder
abduction. To save time you can mention this test when you ask the patient to

90
abduct the shoulders when testing active movements. May suggest
subacromial impingement.

 Testing the power of Supraspinatus (abductor): Ask the patient to bend the
arms (like the chicken dance when they are mimicking a bird); with flexed
arms, ask him/her to abduct their arms while you resist abduction; if there is
pain or weakness, it may suggest supraspinatus pathology.

 Testing the power of Infraspinatus and Teres Minor (external rotators): Ask the
patient to flex the elbow to 90⁰ with forearms in front of the body; then ask
the patient to externally rotate the arm at the shoulder as you provide
resistance along the forearm. Keep the elbows bent at 90 degrees against the
body during this maneuver. Pain and/or weakness suggest tear or tendinitis of
either muscle.

 Lift off test for Subscapularis(internal rotator): To test the power of


subscapularis muscle, ask the patient to place his/her hand behind his/her
back with the palms facing outwards; push his/her hand down against the back
and ask the patient to resist you (resisted internal rotation)

At this point, if you run out of time, you will still have likely been able to deduce certain things
about the pain. Important factors to consider include:

1. Pain or limitation in ROM with active & passive ROM:


suggests intra-articular process (in the shoulder joint)

2. Pain or limitation in ROM with active (but NOT


passive ROM): suggests extra-articular process (eg.
rotator cuff tendonitis or bursitis)

3. Resisted rotator cuff testing can then help isolate the


involved muscles/tendons.

Other Special Tests (Consider only if you have time)

 Drop arm test: Tests supraspinatus function. Supraspinatus initiates arm


abduction. In this test, fully abduct the patient’s arm then ask him/her to
slowly adduct it. In cases of supraspinatus tear, the patient will slowly lower

91
the arm to 100⁰, after which the movement becomes irregular and the arm
may suddenly drop down due to the disrupted supraspinatus tendon.

 Empty Can test: Another test for supraspinatus. Ask the patient to abduct the
arm to 90 degrees in the scapular plane, internally rotate the arm (as if
emptying a drink from a can). Exert a downward pressure at the elbow or wrist
which the patient resists. If there is pain or weakness, this suggests
supraspinatus pathology.

 Testing for anterior shoulder dislocation: The following 3 tests can all be done
in a single maneuver:

1- Anterior Apprehension test: Preferably done with the patient sitting; ask the
patient to hold his/her arm in the throwing position (shoulder abducted 90⁰,
elbow flexed 90⁰) and apply pressure to the posterior aspect of the humerus,
as if you are trying to anteriorly dislocate the shoulder. Apprehension or pain
indicates a positive test

2- Relocation test: With the patient lying down and the arm in the throwing
position, apply anterior pressure to the humerus as if you trying to relocate an
anteriorly dislocated shoulder. Feeling relief is a positive test

3- Anterior release test: After attempting to relocate the shoulder, suddenly


release your arm. Pain or apprehension is a positive test.

 Sulcus sign for inferior shoulder dislocation: Pull the patient’s shoulder
downwards and look for an indentation between the glenoid rim and humerus
suggestive of inferior shoulder dislocation

 Posterior apprehension test for posterior dislocation: With the patient supine
and the arm in the throwing position, apply downward pressure to the humerus
as if you are trying to dislocate the shoulder posteriorly. Pain and/or
apprehension are positive signs.

Note that anterior dislocation is much more common than posterior and/or inferior
dislocations. You can start the shoulder P.E with the patient sitting or standing and do all the
maneuvers except for relocation and release tests and posterior apprehension test. If you need
to complete these tests, ask the patient to lie down and perform them. The less you move the
patient the more organized your approach will be.

92
 Yergason’s Test: Biceps is the main supinator. In this test, you stand in front of
the patient, shake the patient’s hand, and ask the patient to supinate it while
you resist them (ie. try to pronate their arm). Pain suggests bicipital tendinitis.

I will examine both shoulders but will focus on the right for the sake of the exam. I will then
examine the neck and the elbow and the neurovascular compartment of the upper limbs. I will
be looking, feeling and moving your shoulders, please let me know at any point if you feel
uncomfortable. I start P.E with inspection, comparing both sides looking from the front, back
and sides; no swelling, erythema, muscle atrophy, no deformities and no skin changes. I will
now palpate the shoulder, noticing any temperature difference, and there isn’t any. I’m starting
at the sternoclavicular joint, palpating along the clavicle, till the acromioclavicular joint, feeling
the coracoid, the humeral head, the bicipital tendon, spine of the scapula, supraspinatus,
infraspinatus, and medial border of the scapula. There is no tenderness or swelling. Next I will
test active movements. Can you please copy my movements and let me know if you feel pain.
Flexion, extension, abduction (I notice there is no painful arc), adduction, internal rotation,
external rotation. There is no pain or limitation of movement. Can you relax your shoulder and
let me move it for you? Please let me know if you feel any pain. Flexion, extension, abduction,
adduction, internal rotation, and external rotation. Next I will complete resisted rotator cuff
testing. I will test the power of the supraspinatus rotator cuff muscle. Can you bend your hands
at the elbows like this and try to move the elbows out while I resist you? Next I will test the
power of infraspinatus and teres minor. Can you go like this for me and resist me as I try to push
your arms towards your stomach. Next I will test the power of subscapularis. Can you bring your
hand behind your back like this and try to lift off my hand as I push down on yours. I will move
next to other special tests starting with the dropped arm test looking for supraspinatus tears.
Can you please lift your arms to the side all the way above your head then drop them back to
the sides slowly. The test is negative. Next I will do Yergason’s test to detect bicipital tendinitis.
Can you twist your arm this way while I resist you as I hold your hand. The test is negative. Next
is the Empty Can test for the supraspinatus tendon. Can you please go like this and resist me as
I push down on your arms. The test is negative. I will now test for inferior shoulder dislocation.
I’m going to pull your arm downwards. The sulcus sign is negative. Next I will do the anterior
apprehension test for anterior shoulder dislocation. Can you go like this please as if you are
throwing a ball, I’m going to press on your shoulder let me know if it’s painful or uncomfortable.
The test is negative. Can you lie down for me please? I’m doing the relocation test for anterior
shoulder instability by applying anterior pressure to the humerus and seeing if there is a feeling
of relief. I notice that the test is negative. I pull my arm quickly in the release test. No

93
apprehension or pain so the test is negative. Lastly I will test for posterior shoulder instability by
doing the posterior apprehension test and the test is negative.

Back P.E;

Back pain is very common, it is very important to keep in mind the red flags for back pain;

1- Age >50

2- Previous or current history of malignancy

3- Stool incontinence or urinary retention

4- Saddle anesthesia

5- IV drug use

6- Motor or sensory deficit

7- Constitutional symptoms (significant unexplained weight loss, night sweats,


fever)

8- Immunosuppression

9- Steroid use

I will focus in this section on P.E for patients with low back pain. It is very important to examine
the motor and sensory systems of the lower limb, and to mention that you would do a rectal
exam. It is helpful to practice the back exam to ensure you can fit it in 4 minutes.

- Ask for permission then wash your hands and respectfully drape the patient by taking
the gown off, keeping pants and shorts on, and keeping the bra on in female patients.
Mention that you would examine the hips and the neurovascular compartment of the
lower limbs and would do a rectal exam at the end

94
- Inspection: With the patient standing look for SEADS: Swelling, erythema, muscle
atrophy, deformities (kyphosis, scoliosis, lumbar lordosis), skin changes (especially tuft
of hair and Café-au-lait spots). Inspect the gait; ask the patient to walk, turn around
and come back, walk on tip toes and on heels and observe the gaits’ smoothness, arm
swing, balance, any antalgic gait. Make sure to walk along with the patient to support
him/her in case of a fall.

- With the patient standing for inspection do the following tests

 Occiput to wall distance: Ask the patient to stand with his/her back against the
wall and the heels touching the wall and measure the distance between the
occiput and the wall. The occiput should touch the wall. The distance is
increased in Ankylosing Spondylitis (AS).

 Rib- pelvis distance: Measure the distance between the lower ribs margin and
the upper hip in the midaxillary line; >2cm is suggestive of vertebral fracture.

 Modified Schober test: With the patient standing straight, ask the patient if
you can draw a horizontal line between the 2 posterior superior iliac spines
(Dimples of Venus). Then mark 2 points in the midline, one 10 cm above the
line and one 5cm beneath. Ask the patient to bend forward with the legs
straight and measure the distance between the 2 midline points. An increase
of at least 3-4 cm is expected, less is seen in AS.

 Chest expansion: Measure the chest circumference at the level of the


diaphragm during inspiration and expiration ( normal is 5cm or more)

- Palpation: Palpate the whole spine with the dorsum of your hand for temperature.
Then feel the spinous processes with your palm. Notice any tenderness or deformity.
Palpate the paravertebral muscles for spasm. Then palpate the iliac crest, posterior
superior iliac spine, sacroiliac joints, anterior superior iliac spine, and indicate that you
would palpate the inguinal lymph nodes and pubic symphysis (the examiner will stop
you, but you need to mention it for completeness). If you are unsure, you can ask the
examiner if they want you to complete a particular examination.

- Percussion: Make a fist; let the patient know that you are going to tap on his/her back.
Percuss the whole spine for tenderness.

- Range of motion: Stand by your patient and demonstrate the active ROM first. Then,
ask the patient to do the following movements:

95
 Forward flexion: ask the patient to bend forward as much as he/she can
without bending his/her knees.

 Extension: Bend backwards

 Side or lateral flexion: Ask the patient to bend to the right and left by sliding
his/her hand down his/her leg.

 Rotation: Ask the patient to sit down and rotate to each side. Sitting prevents
you from involving the hips and ensures it is the thoracolumbar area you are
looking at.

 Evaluate the cervical spine in the same way you completed the lumbosacral
spine. Ask the patient to bend their head so their chin touches their chest
(forward flexion), bend the head back (extension), touch their ear to each
shoulder (or as close as possible ) (lateral flexion) and “shoulder check” or look
to each side (lateral rotation).

- Special tests: If the patient complains of back pain radiating down one leg, you should
complete one or more of the following especially straight leg raise test;

 Straight leg raise test: With the patient lying supine, elevate one leg and see if
the patient develops back or leg pain. Pain at <60⁰ is indicative of sciatica.

 Crossed straight leg raise test: Elevation of one leg causes pain of the
contralateral limb, also indicative of sciatica.

 Femoral stretch test: The patient lies prone, and with the knee flexed, the hip
is lifted into extension. Positive test reveals anterior thigh pain/back pain and
can reflect higher lumbar nerve root irritation.

- Motor and sensory examination of the lower limbs: Refer to page 29 chapter 3 for
power and reflexes examination, and to page 32 chapter 3 for sensory exam.

- If you still have time, feel the pulses of the lower limbs; palpation of pulses is detailed
in chapter 3 page 10

96
- Less commonly, if the patient complains of nerve discomfort or weakness of the upper
extremity and you have concerns about their cervical spine, perform motor and
sensory examinations of the upper limbs as described for the lower limbs.

- Indicate again that you would do a rectal exam

I start P.E of the back with inspection; no swelling, no erythema, no muscle atrophy, no
deformities namely no scoliosis, no lordosis or kyphosis or any other deformity. No skin
changes. Next I will palpate. I’m going to feel and tap on your back let me know if it’s painful.
I’m feeling for temperature difference over the spine. Then palpating the spinous processes and
paravertebral muscles, no tenderness and no muscle spasm. I’m palpating the iliac crest, the
posterior superior iliac spine and the sacroiliac joint, also feeling the anterior superior iliac
spine. I will check for inguinal lymphadenopathy and palpate the pubic symphysis. The
examiner will say “pass”. I’m percussing the spine looking for any tenderness. Can you walk for
me please? The gait is smooth with normal stance and arm swing, no antalgic gait. Can you turn
around and come back? Can you please walk on your tip toes? (S1) and your heels? (L4). Next I
will measure the occiput wall distance. Can you please stand with your back against the wall
and your heels touching it? There is no space between the occiput and the wall, as expected in
normal individuals. Next I will do the modified schober test. Do you mind if I put a small mark
on your back? I’m marking the horizontal line between the 2 superior posterior iliac spines, and
in the midline marking one point 10 cm above and another one 5 cm below. Can you bend
forward, keeping your legs straight? I’m measuring the distance between the 2 midline points.
It is 20 cm, so increased by more than 3 cm and this is normal. With the patient standing I will
measure the fingerbreadths between the ribs and hip. It is 4 (more than 2 is normal). Now I will
test the active ROM: Can you bend forward? Forward flexion. And backwards? Extension. Can
you bend to the side like this? Lateral flexion and the other side please?. Can you sit down
please? Can you rotate to the left and right like this? Thoracolumbar rotation is normal. Can
you take your chin and touch it to your chest as you bring your head down? Normal cervical
forward flexion. Can you extend your head back as far as it comfortably goes? Extension of the
cervical spine is normal. Can you look to each side as far as you can? Normal lateral rotation of
the cervical spine. Can you bend your head, trying to touch your ear to your shoulder for each
side? Lateral flexion of the cervical spine is normal. Can you lie down please? I will do straight
leg raise test. I’m going to raise your left leg let me know if it hurts anywhere. No pain in the
back and leg with the lower limb elevated >60⁰. I will now do the crossed straight leg test, lifting
the right leg straight, noticing if the patient has any pain in the left leg. The test is negative.
Next I will do the femoral stretch test. Can you roll over onto your abdomen, and I am going to
bend your knee and lift your hip up? Let me know if it hurts anywhere. There is no pain with left

97
hip extension, the test is negative. Next I will test the power of the lower limbs. I’m going to
move your legs to test the strength of your muscles. Resist me please. Hip flexion, abduction and
adduction; knee flexion and extension; ankle dorsiflexion and plantar flexion are normal. I will
test the reflexes of the lower limbs. I’m going to test your nerves, let me know if it hurts. Ankle
jerk S1, S2, Knee Jerk L4,5. I will do the other side as well. Next I will test light touch of the
lower limbs with a cotton ball. Do you feel this? Close your eyes and say yes each time you feel
it. Let me know if it feels different on either side. L1 comparing both sides, L2, L3, L4, L5, and S1.
Finally I would do a rectal exam and examine the hips

Hips P.E

- Ask for permission, wash your hands and respectfully drape the patient by taking the
gown off and keeping his/her shorts on. Indicate that you will examine both sides but
will focus on the affected side for the sake of the examination. You will also examine
the back, knees and the neurovascular compartment of the lower limbs

- Inspection: Compare both sides, looking from the front, back, and sides for SEADS;
swelling erythema, atrophy, deformity, skin changes. Ask the patient to walk (make
sure you support your patient) and look for any antalgic or Trendelenberg gait. Do the
Trendelenburg test with the patient standing; stand behind the patient, place your
hands on the iliac crest and ask the patient to stand on one leg at a time. Test both
sides. The unsupported (non-weight bearing) side stays up in normal individuals but
drops in a positive test, i.e. the affected side (the one the patient is weight bearing on)
has a weak abductor (gluteus medius or minimus) or nerve palsy (superior gluteal
nerve palsy).

- Palpation: Feel for temperature difference, and then palpate the iliac crest, anterior
superior iliac spine, posterior superior iliac spine, sacroiliac joint, greater trochanter
and bursa. Mention that you would palpate the inguinal ligament and symphysis
pubis, the examiner will stop you.

- ROM: Start with active range of motion doing the first 2 movements with the patient
standing and the rest lying down. Ask the patient to copy your movements:

 Extension: ask the patient to move each leg backwards with the knees straight

98
 Rotation: ask the patient to make a circle with each foot

 Flexion: ask the patient to bring his knees to his chest (supine)

 Abduction: ask the patient to move his leg outwards with the knee straight

 Adduction: ask the patient to move the leg inwards all the way crossing the
opposite leg with the knee straight.

Then ask the patient to relax, and move his hip in the above-mentioned directions. For passive
internal and external rotation, you will bring the patient’s knee to approximately 90 degrees,
and then internally and externally rotate it at this point to passively evaluate hip rotation. If
they have knee problems, you can log-roll the resting leg in the same motions of internal and
external rotation. You may want to ask the patient to lie on his side to test passive extension.
Next, you can test the power by asking the patient to resist you as you repeat the above
movements.

- Special tests: Done with the patient lying down:

 Leg length: Using a tape measure, measure the true leg length (from anterior
superior iliac spine to medial malleolus). Compare both sides, then measure
apparent leg length (from the umbilicus to the anterior superior iliac spine)

 Patrick’s test: With both knees extended, place the foot of one leg over the
contralateral knee. Support the hip of the extended limb while you try to lower
the other limb to the same level, which is normal.

 Thomas test (Fixed flexion deformity): Place your hand under the lumbar spine
with the palms facing up, then passively flex the patient knee to his/her chest.
The opposite leg should remain straight; if not, it suggests fixed flexion
deformity of that hip.

I will examine both hips but will focus on the right for the sake of time. I will also examine the
lumbar spine and the knee and the neurovascular compartment of the lower limbs. I start with
inspection looking from the front, back and sides, comparing both sides. No swelling, erythema,
muscle atrophy, deformity or skin changes. Can you walk for me please? The gait is smooth, no
antalgic gait and no trendelenberg gait. I’m going to feel your hip. Let me know if it hurts
anywhere. I’m palpating the iliac crest, the posterior superior iliac spine, the sacroiliac joint, the
greater trochanter and its bursa, the anterior superior iliac spine, and will also palpate the
inguinal ligament and pubic symphysis. The examiner will say “pass”. With the patient standing,

99
I will do the trendelenberg test, placing my hands on the iliac crests. Can you stand on one foot
please? And now the other? The test is negative. Next is the active range of motion. Can you
copy my movements? Extension, Rotation. Can you lie down please?. Can you bring your knees
to your chest? Flexion. Can you keep your leg straight and bring it out? Abduction. Can you
bring it all the way in crossing the other leg? Adduction. Can you relax and let me move your
leg. I’m testing the passive ROM: flexion, abduction, adduction, internal and external rotation.
Can you lie on your left side? And extension. Can you lie again on your back?. I will test the
power at the end. I will move now to special tests. I’m measuring the true leg length from the
anterior superior iliac spine to medial malleolus. And will measure the other side. Both are the
same (you can give the number) then I’m measuring apparent leg length from the umbilicus to
medial malleolus, no difference. Next I will do Patrick’s test looking for sacroiliac joint
pathology, or Iliopsoas spasm. The test is negative on the right. Then I will do the Thomas test
looking for any fixed flexion deformity of the hip and the test is negative on the right.

Knees P.E;

Knee pain and injuries are very common. It is useful to review Basic anatomy of the knee so
that P.E makes sense. Follow the link below for basic knee anatomy;

http://www.athleticadvisor.com/Injuries/LE/Knee/knee_anat.htm

- Ask for permission, wash your hands and respectfully drape the patient by exposing
both lower limbs to mid thighs

- Start by stating that you will examine both knees but will focus on the affected one for
the sake of time. State that you will examine the hips and ankles and the
neurovascular compartment of the lower limbs.

- Inspection: Done with the patient standing, walking and lying down. Look from the
front, back and sides for SEADS; Swelling, Erythema, Muscle atrophy, Varus or valgus
deformity, and skin changes. Ask the patient to walk and support him/her if necessary.
Look for antalgic gait or limitation of movement. Ask the patient to stand on one leg at

100
a time and see if he/she can’t bear weight (make sure you are there for your patient).
Ask the patient to squat down and up if they can and see if there is any pain or
limitation in their ROM. Ask the patient to lie down and look at the knees again. Ask
the patient to flex his/her knees and look behind the knee for sag.

- Palpation: Feel for temperature difference. Then palpate the knee for swelling and
tenderness feeling the following structures:

 Quadriceps tendon

 Patella and patellar tendon

 Tibial tuberosity

 Head of Fibula

 Medial and lateral joint lines

 Medial collateral ligament

 Lateral collateral ligament

 Popliteal fossa

- ROM: The knee moves in 2 directions: flexion and extension. Do active and passive
ROM and test the power. Notice any limitation of movement, pain or crepitation.

- Special test:

In this section you will test the main ligaments shown in Figure 3-5 in addition to knee effusion.
Make sure you practice and master the techniques.

 Effusion: Do the test you feel more comfortable with. You can do patellar tap
where you milk the fluid down from the lower thigh into the knee, keep your
hand above the knee and tap the patella. The other test is called the milk or
wipe test where you basically milk/move fluid from the medial to lateral
compartments of the knee with one hand, and look for any bulge on the
medial side as you milk/push the fluid from the lateral side back to the medial
side. This only works with small effusions. If there is a larger effusion, you can
also perform the Ballottement test where you move fluid from the
suprapatellar pouch (lower thigh) of the knee to the main part of the knee. You
are feel fluid shift back and forth between your hands which confirms swelling.
You can look for swelling in the back of the knee (ie. Baker’s cyst) when you are

101
inspecting the patient and they are standing. Look behind their knee (ie.
Popliteal space) and see if it looks distended or red suggesting swelling.

 Anterior Drawer sign for anterior cruciate ligament stability: With the patient
lying supine, flex his/her knee to 90 degrees and sit on the ipsilateral foot. Pull
the leg forward by holding it with both hands just below the knee. Note any
excessive movement to sugesst ACL instability.

 Posterior drawer sign for posterior cruciate ligament instability: Similar to


anterior drawer sign but you push backwards this time. Note any excessive
movement.

 Testing medial and lateral collateral ligaments: Flex the knee 20⁰ and apply
valgus stress to the knee to test for medial collateral ligament instability and
varus stress to test for lateral collateral ligament instability. Excessive
movement implies a positive test.

 McMurray test for medial meniscus injury: Place one hand on the knee joint
line and one hand on the foot. Fully flex the knee, and externally rotate the
foot as you slowly extend the knee. A snapping sensation indicates a positive
test.

 McMurray test for lateral meniscus injury: Same maneuver as above but
internally rotate the foot.

I will examine both knees but will focus on the right for now. I will also examine the hips and
ankles and the neurovascular compartment of the lower limb. I start P.E by inspecting the knees
with the patient standing comparing both sides, looking from the front, back and sides; no
swelling, no erythema, no deformity no muscle atrophy, and no skin changes. Can you walk for
me please? No antalgic gait. Can you squat for me and then come back up? I do not see any
abnormal movement. Can you lie down and bend your knees? I’m inspecting the knees again
looking for any sag and there isn’t. I’m going to feel your knees, let me know if it hurts
anywhere. I’m palpating the knees for temperature difference and there isn’t. I’m now feeling
for any swelling or tenderness; feeling the quadriceps tendon, patella, patellar tendon, medial
collateral ligament, lateral collateral ligament, tibial tuberosity, head of femur, and popliteal
fossa. No abnormality. Next is ROM, I will start with active ROM. Can you bring your knees to
your chest then fully straighten them. ROM is full with no limitation, pain and I feel no crepitus
in the medial, lateral and patellofemoral compartments. Can you relax your knees while I move
them this time? Flexion and extension. Then I will test power. Resist me as I bend your knee.

102
Resist me as I straighten it. I will now do some special tests starting with patellar tap looking for
effusion. Its negative, and the milk/wipe test also looking for effusion is negative. Next I will
test the cruciate ligaments. Can you bend your knee please? With the knee flexed to 90⁰, I’m
doing the anterior drawer test for anterior cruciate ligament and the posterior drawer test for
posterior cruciate ligament. Both tests are negative. Next I will check the collateral ligaments,
flexing the patient knee 20⁰ and applying varus stress looking for lateral collateral ligament
instability and valgus stress for the medial collateral ligament. The test is negative. Next I will do
the McMurray test, fully flexing the patient knee, and externally rotating the foot looking for
medial meniscus pathology as I extend the knee. The test is negative. Now I’m fully flexing the
knee, internally rotating the foot and extending the knee looking for lateral meniscus tear. The
test is negative.

Ankles P.E

- Ask for permission, wash your hands and respectfully drape the patient exposing the
legs and feet

- Indicate that you would examine both ankles, but will focus on the affected for the
sake of time. State that you will examine the knees, and the neurovascular
compartment of the lower limbs.

- Inspection: Done in 3 positions including standing, walking and lying down. With the
patient standing look from the front, sides, and back for SEADS; swelling, erythema,
muscle atrophy, deformity, and skin changes. Slide your hand under the foot and
noticed if the patient has high arch or flat foot. Ask the patient to walk and observe
for dropped foot or inability to bear weight on one foot. Ask the patient to walk on tip
toes and heels (always support your patient when you ask him/her to walk). With the
patient lying down, look between the toes for any skin changes, notice any nail
changes, and check for edema, look at the sole and heels for ulcers.

- Palpation: Feel for temperature difference at the ankle and foot. Then palpate for
masses or tenderness feeling 6 cm above each malleoli and feeling the medial and
lateral malleolus, the Achilles tendon (note continuity), the calcaneus, base of fifth

103
metatarsal, navicular, metatarsal heads. Compress the forefoot between thumbs and
fingers for MTP tenderness. Feel the posterior tibial and dorsalis pedis pulses

- Range of motion: Do active and passive ROM and test the power. Ankle moves in 4
directions; Dorsiflexion, plantarflexion, inversion, eversion.

- Special tests: Ask the patient to lie prone with the ankle hanging at the edge of the
examining table and do the following tests for Achilles tendon:

 Palpate the Achilles tendon for gaps in this position.

 Calf squeeze test: squeeze the calf - the normal response is ankle plantar
flexion. In the case of Achilles tendon rupture, there will be no movement.

 Knee flexion text: Ask the patient to flex his/her knees while lying prone and
measure the angle between the ankle and the leg. If the Achilles tendon is
intact, the ankles remain slightly plantarflexed and the angle is >90⁰. If it is
ruptured, the angle is <90⁰.

 Anterior Drawer Test: In the case of an ankle sprain, a common test to evaluate
for anterior talofibular ligament rupture is the Anterior Drawer test. Position
the patient’s foot in slight plantar flexion, brace the anterior shin with the
other hand, pull the heel anteriorly with main hand and check for laxity. There
should be limited movement.

Useful Tip;

Follow the link below for Ottawa Ankle rule;

http://www.ohri.ca/emerg/cdr/docs/cdr_ankle_poster.pdf

I will examine both ankles but will focus on the right for the sake of the exam. I will also
examine the knees and the neurovascular compartment of the feet. I start P.E inspecting the
feet in the standing position, comparing both sides, looking from the front, sides and back; no
swelling, no erythema, no muscle atrophy, no valgus or varus deformity or any other deformity,
no skin changes. I will slide my hand under your foot, let me know if it’s uncomfortable. No high
ach or flat foot. Can you walk for me? No drop foot or antalgic gait. Can you walk on your tip
toes? And on your heels. Can you lie down please? I’m inspecting the plantar surface and heels
for ulcers, looking between the toes, and inspecting the nails, no changes. I will now palpate the
ankles, checking first for temperature difference and there is no difference. Now I will palpate

104
for tenderness and swelling; the medial malleolus, the lateral malleolus, the Achilles tendon,
calcaneus, the base of the fifth metatarsal, the navicular, the metatarsal heads. I’m squeezing
the forefoot looking for MTP tenderness or swelling. Next is the range of motion. Can you bring
your foot up? Dorsiflexion. Now down as if you are pressing on the gas pedal of a car? Plantar
flexion. Can you bring your foot out like this? Eversion. And in? Inversion. I will now do the
passive range of motion. Can you relax your foot for me? Plantarflexion, dorsiflexion, inversion,
eversion. Next I will test the power. Can you resist me as I move your foot? Doing the same
movements, the power is 5/5. Next I will do special tests. Can you lie on your tummy for me and
hang your foot off the edge of the bed?. I’m palpating the Achilles tendon, there are no gaps; I
am doing the squeeze test, noticing plantar flexion - the test is negative. Can you bend your
knees? With the knees flexed 90⁰, I notice slight plantar flexion, the knee flex test is negative.
Can you relax your foot while I check how stable the ankle is? Anterior Drawer test is negative
for a ligamentous sprain or tear

Breast Physical Examination;


Edited by; Dr. Sadik Salman

The MCC website specifies that you will not be asked to perform a genital or rectal P.E, but
doesn’t say so for breast examination. You may encounter a male or female breast exam.

- Ask permission, wash your hands and respectfully drape the patient by exposing the
chest (ask female patient to take her bra off). Keep the patient covered from the
umbilicus down.

- Make sure vital signs are stable and the patient is comfortable.

- Inspection and palpation need to be done in two positions; sitting and standing

- With the patient sitting;

 Inspection; start with the patient hands resting on the thighs, look from the
front and sides noticing the breasts size and symmetry, changes of the shape,
skin redness, Peau’ d’orange, vascularity or skin rash or ulceration. Look at the
nipples noticing any retraction or distortion, inversion or nipple discharge.
Look for swelling or ulcerations of the areola. Inspect the axilla for masses,

105
redness or hyperpigmentation. Ask the patient to press against his/her hips
and look for any retraction. Ask the patient to put his/her hands behind his/her
back and inspect again. Pay particular attention for the inferior side of the
breasts.

 Palpation; You will palpate for 3 things;

1- Temperature; with the dorsum of your hand notice any temperature


difference

2- Palpate the breasts; start with the normal first. Palpate from the
midaxillary line between 2nd and 6th ribs to mid sternum. Divide the breast
into 4 quadrants (superior outer and inner, inferior outer and inner), use
the palmar surface of the tip of your fingers and make small clockwise
circles as you feel each quadrant. Feel under the breasts, ask the patient to
lift her breast if needed. Look for tenderness, masses. If a mass is felt
determine exact location, consistency, shape, edges, size, temperature,
tenderness and mobility. Note if the mass is attached to the skin or the
underlying muscles.

3- Lymph nodes (L.N); Palpate the supra and infra clavicular L.N, and the
axillary lymph nodes, note any tenderness. If you feel something try to
determine how many nodes, the size, consistency, exact location and
tenderness. Ask the patient to tilt his/her head down and to the right for
right supraclavicular L.Ns. and down and to the left for the left ones. It
makes it easier and more comfortable for the patient. For axillary L.Ns ask
the patient to relax his/her arm on top of yours with the elbows flexed,
slightly flex and abduct the patient’s arm.

- With the patient lying down

 Inspect again; both breasts and axilla

 Palpate only the breasts using the same technique

- Pay particular attention to the patient’s comfort and privacy

I’m going to start a breast examination; Please let me know if you feel uncomfortable. Vital
signs are stable, I’m comparing both sides, looking from the front and sides noticing that both
sides are symmetrical, no swellings, redness, Peau’ d’orange, ulceration or skin rash, no nipple
deformity or retraction, no nipple discharge. I’m inspecting the axilla; no masses, no redness.
Can you put your hands on your hips like this and press? I’m inspecting the breasts for any

106
retraction or any of the signs I just mentioned. Can you put your hands behind your head?
Inspecting the breasts and axilla. I’m going to feel your breast for masses please let me know if
it feels uncomfortable. I’m going now to palpate with the patient sitting; Feeling for
temperature difference and I don’t appreciate any. Now palpating the right breast starting from
the right upper outer quadrant, right lower outer quadrant, lower inner and upper inner; no
masses or tenderness. I will now palpate the left breast. Next I will palpate the right
supraclavicular nodes, infraclavicular nodes, now I will palpate the left side. Next I will palpate
the right axillary lymph nodes, can you relax your arm on mine; I’m feeling the medial group,
lateral, and superior, inferior, apical. Next I will feel the left axillary L.N. No palpable nodes.

Can you lie down please? With the patient lying down I’m inspecting again, no abnormalities,
and I will palpate the breasts again in this position. Let me know if you feel uncomfortable. No
masses, no tenderness.

Fundoscopy;
Edited by; Dr. Sadik Salman

- Use the rule of right, right, right when you do ophthalmoscopic examination; hold the
ophthalmoscope with your right hand, use your right eye to examine the patient’s
right eye

- Ask permission, wash your hands, no draping is warranted

- Ask the patient to focus on a point on the wall, dim the light. Start looking in the
ophthalmoscope 1 foot away adjusting the focusing wheel as needed. Note the red
reflex and come close to the eye at 45⁰ (between an imaginary vertical and horizontal
planes). Come close to the patient as much as you can, you can use place the thumb of
the contralateral hand on the patient’s forehead to avoid hitting the patient with the
ophthalmoscope accidentally. Adjust the focusing wheel as needed until you see a
retinal vessel, follow the vessel to the optic disc, note the following;

 Optic disc size, shape, margins, note any edema

107
 Blood vessels; look at the retinal veins and arteries, note any tortuous vessels,
neovascularization and any AV nodding

 The macula if you can find it

 Retinal background; Look for any cotton wool exudates or flame hemorrhages
or abnormal pigmentation.

I’m going to do fundoscopic examination, dimming the light, using my right hand and eye to
examine the patient’s right eye, I notice a normal red reflex, and as I come closer I notice a
blood vessel that I’m following, now I see the optic disc and note that it has sharp borders, no
edema, normal shape and size. I’m looking at the veins and arteries, no AV nodding, tortuosity
or neovascularization. No flame hemorrhages or cotton wool exudates. The macula looks
normal

108
References;
1- Heart murmurs, Medical Exam essentials , accessed on May 1st/2014,
http://www.medical-exam-essentials.com/heart-murmurs.html

2- Elevated JVP, Clinical exams, accessed May 1st/2014,


http://clinicalexams.co.uk/images/elevated_jvp.jpg

3- Simel, David L. The Rational Clinical Examination Evidence-based Clinical Diagnosis. New
York: McGraw-Hill Medical, 2009.

4- Macleod, John. Macleod's Clinical Examination. 12th ed. Edinburgh: Churchill


Livingstone/Elsevier, 2009.

5- McGee, Steven R. Evidence-based Physical Diagnosis. 3rd ed. Philadelphia:


Elsevier/Saunders, 2012.

6- Douglas Gelb, The detailed neurological examination in adults, In: UpToDate, Topic 5095
Version 5. , Waltham, MA. ,Accessed on October 25/214,
http://www.uptodate.com/contents/the-detailed-neurologic-examination-in-
adults?source=search_result&search=neurology+exam&selectedTitle=1~150

109
110
Chapter 4
Writing and
Counseling Tips
Edited by; Dr. Anca Tapardel

111
Introduction;
This chapter will cover common writing tasks that you will do on a daily basis as a resident.
They may also be tested in the writing part of MCCQE2. All written information becomes a legal
document. It is important to be organized and write legibly. You will not be asked to type on a
keyboard in Canadian exams, and will always be provided with a pen/pencil when you sign in.
Familiarize yourself with the Canadian style by going over the detailed examples of writing;

- Admission orders

- Progress Note

- Discharge summary

- A letter to an employer

- A prescription

- A referral letter

The next section of the chapter is about counseling. You may counsel your patient about
anything from smoking to laboratory results to life threatening diagnoses. Counseling is a very
important part in communication with our patients. Detailed examples are provided in chapter
5.

Sample writing scenarios


Admission orders

You may be given a scenario, or asked to write orders for a patient you just assessed. Make sure
you write down the patient name, hospital number and date of birth. Usually this information is
provided by labels. Remember to date and sign the orders and leave your pager number with
your name written clearly at the end. Most of the hospitals have template orders to help
standardize the charts.

In the body of the orders you need to be specific as to whom and where the patient is
admitted. Clearly state the diagnosis. Then list their recommended diet, activity level, IV fluids

112
and oxygen if needed, and the frequency with which you want their vital signs checked. Then
list the investigations and medications and indicate if any consult is warranted.

Organize investigations into blood work and imaging or others. Write the medication name,
dose, route and frequency of administration and if it is given around the clock or as needed
(prn). If you want your patient to continue her/his home medications, add them to the
admission medications.

A consult sheet request is filled by the physician (example provided in this chapter page 7). In
real life the admitting physician calls the consulted service. In rare cases the consult is faxed,
and in this case you should order the consult to be faxed. Anyhow, it is a good idea to indicate
what service is consulted and if they were called already or if the consult needs to be faxed.
This way the medical team know exactly what is being done on admission by just looking at the
orders.

Note; this example is only for demonstration. In most surgical cases in real life general surgery
will be consulted by the emergency room physician and will admit the patient under their care.

Example;

You have just assessed Mr. Smith, a 30 year old male who presented to the emergency
department with right lower quadrant pain. He had guarding and tenderness of the right lower
quadrant, and was febrile. Other vital signs were normal. He had positive signs of appendicitis.
You were informed by the examiner that he had a normal rectal exam. His WBCs were elevated.
Write admission orders for Mr. Smith.

Patient name--------------

Hospital number---------

Date of birth--------------

Date;---------

- Admit Mr Smith under Dr (Name of admitting physician) to ward (Name or ward number)

- Keep NPO (nothing per mouth), except for medications with sips of water

- Activity as tolerated

- IV fluids; normal saline 0.9% 100cc/hour for 24 hours then reassess by physician

- Vital signs every 4 hours

113
- Blood tests; CBCD, Creatinine, Urea, electrolytes (Na, K, CL),

- Urine analysis

- Abdominal Ultrasound

- Medications;

- Tylenol 325mg P.O q 4 hours prn

- Gravol 50mg IV q 6 hours prn

- Consult surgery- Surgical team was informed

Your name, signature and pager number

Progress Notes;

You have probably heard about the famous SOAP format for progress notes. It stands for;

S; Subjective (what the patient is complaining from)

O; Objective (Physical exam findings and laboratory results)

A; Assessment (Your assessment of the patient status)

P; Plan (management plan, including changing medications, or ordering further investigations


as well as the need for physical or occupational therapy or consults and the discharge plan)

Example;

Mr. Smith is a 40 year old male admitted with right middle lobe pneumonia. He was febrile,
with oxygen saturation of 92% on room air, respiratory rate of 23/min and strongly coughing
yellow-green non bloody sputum on admission. His WBCs were 16. When you saw him today he
said he is feeling much better and that his cough is not as bad. You still hear right mid lung zone
crackles, and he is afebrile with oxygen saturation of 98% on room air and respiratory rate of

114
14/min. His CXR is still the same with right middle lobe consolidation. And his sputum gram
stain and culture came back positive for Strep Pneumonia sensitive to Levofloxacin you
prescribed on admission. Blood cultures were negative.

Mr. Smith is a 40 year old male not known previously to have any chronic medical illnesses.
Admitted on (date) with right middle lobe community acquired pneumonia (CAP) and was
started on empiric Levofloxacin 750mg/ day P.O

S; He is feeling much better, his productive cough is improving no hemoptysis, no chest pain

O; His vital signs are all within normal. He still has right middle lobe crackles. His WBCs are now
11 from 16, and his sputum grew Strep Pneumonia sensitive to Levofloxacin. No complication
on his chest XR and the right middle lobe consolidation remains unchanged.

A; 40 years old previously healthy smoker male admitted with right middle lobe CAP due to
Strep Pneumonia sensitive to Levofloxacillin. Shows clinical and laboratory improvement, the
plan is;

-Continue Levofloxacin 750mg P.O daily for 7 days

-Continue to check vital signs every 6 hours

- Repeat the chest XR after 6 weeks to make sure the consolidation is fully resolved and there is
no underlying malignancy

- Counsel about smoking cessation

-Discharge plan; Plan to discharge home tomorrow with follow up with his family physician

At different levels in your training or practice you may choose to write more succinct notes.

Discharge summary

Each hospital has a form to fill in for discharge summary. You generally need to mention;

- Patient’s name, hospital number and date of birth

- Treating physician’s name

- Admission and discharge dates

- Names of all other physicians involved in patient care (It is important to fax a copy to
each one of them)

115
- Admission diagnoses

- Course in hospital including treatments given and any outstanding investigations

- Medical and surgical illnesses prior to admission

- Discharge medications

- Condition at discharge

- Follow up plan

- Recommendations for diet, activity

Example;

Will use the case of Mr. Smith page 3 of this chapter

-Patient name

Hospital number

Date of birth

-Admitting physician

Admission date

Discharge date

-Patient’s family Dr name (A copy is faxed)

-Admission diagnoses; Right middle lobe community acquired pneumonia

-Course in hospital and treatment; Mr Smith sputum grew Strep Pneumonia sensitive to
Levofloxacin. He was treated with Levofloxacin 750mg P.O daily during his hospital stay. His

116
chest XR showed right middle lobe consolidation. He improved clinically and didn’t have any
complications

- Medical illness; not known to have medical illnesses prior to admission, but is a smoker 1 pack
per day for 20 years (20pack-years)

-Discharge medications; Levofloxacin 750mg P.O daily till --------- (total 7 days)

-Discharge condition; stable, with normal vital signs including an oxygen saturation of 99% on
room air

-Follow up plan; Repeat chest XR in 6 weeks. Mr Smith doesn’t feel ready to quit smoking yet
but will call the clinic when he decides to. Mr Smith was advised to return to the emergency
department if he develops fever, chest pain, worsened cough or bloody sputum. He will follow
up with his family physician

-Diet and activity upon discharge; as tolerated

Letter to employer

Your patient may request a letter to his/her employer. This letter should be printed on a paper
with the hospital/clinic heading.

Example;

You assessed Mr. Smith, a 40 year old male in your clinic for back pain. After full investigations,
his pain deemed mechanical in origin, and no surgical intervention is warranted at this stage.
You think Mr. Smith would benefit from one week of rest. Mr. Smith works at a grocery store

117
where he often lifts heavy objects. Mr. Smith requests a letter to his employer for one week of
rest.

Clinic heading

Work absence certificate

Re; patient name and address

This letter is to certify (patient’s full name) was assessed in this clinic on (day and date) and was
unable to work due to illness/injury from (date) to (date)

Physician name and position

Date and sign

Writing a prescription

You need to include the patient name, age and health number (you can use a sticker), and the
Medication information; drug name, dose, route of administration, frequency and duration.
Sign and date the prescription and leave your pager number in case the pharmacist had any
questions.

Example;

You assessed Ms. Smith. A diabetes patient of yours who was found to have an uncomplicated
lower respiratory tract infection (cystitis). You decide to treat her as an outpatient so you
prescribe her an antibiotic and renew her diabetes medication.

Patient name

Date of birth

Health care number

118
1-Metformin 850mg P.O BID for 6 months

2-Ciprofloxacin XL 500mg P.O daily for 3 days

Date

Physician name and signature

Pager number

Writing a referral letter;

When requesting a consult from a specialist you need to ask a specific question. You need to
write the consultant a comprehensive assessment of the patient including a relevant history,
physical exam and investigations. Be sure to mention the pertinent positives and negatives. If
given investigation results in the test, write them down in your consult. I made up the example
below, so I will not detail investigations but will mention that a copy is attached. Again; write
down the investigations in the referral letter if given to you in the test.

Example;

You just assessed Mrs. Smith, a 35 years old female in your clinic. She is complaining of
recurrent swelling of her hands and fingers for the last six months. She is finding it harder to do
her job as a secretary and worried she has rheumatoid arthritis. Her mother had disabling RA .
You do a full history, physical exam and order some investigations. You decide to refer her to a
rheumatologist.

Re; Patient name, age, hospital number and contact information

To; Name and contact information of the consultant

Dear colleague;

I saw Mrs. Smith in my clinic on (date). She was complaining of recurrent MCP and IP joints
swelling that seem to be symmetrical. No other joints involved. She describes 60 min morning
stiffness that started 6 months ago when the swelling began. No skin rash, eye involvement,

119
headache, chewing or swallowing problems. No skin tightness or Raynaud’s phenomenon, no
photosensitivity and no hair loss.

Her past medical and surgical history is negative. There is a history of severe disabling
rheumatoid arthritis in her mom. She is on birth control bills and is not allergic to any
medication. She lives with her husband and not planning to have kids any time soon. Her
symptoms are affecting her performance at work. She works as a secretary downtown.

On physical exam she had normal vital signs. Her weight was---- and her height was---. She had
symmetrical swelling of the first MCP of both hands which was also tender. Other joints were
normal. No rheumatoid nodules. No deformities. No skin rash or eyes involvement. Cardiac,
pulmonary and abdominal exams were all normal.

Initial investigations included a normal CBCD, Creatinine, urea and electrolytes. Her RF, ANA
and ACCP are pending. XR of the hands showed 1st MCP soft tissue swelling and no bone
destruction. A copy of her investigations is attached.

Can you please assess her for definitive diagnosis and management?

Your help is much appreciated

Regards;

Your name, signature, date and contact information.

Counseling;
Counseling and patient education are very important aspects of patient care commonly tested
in Canadian OSCEs. These stations mainly aim at assessing your communication skills. Studies
have shown that patient education and counseling promote behavioral change. (1)

There are many ways to counsel a patient. You need to develop and practice your own style.
Key elements to keep in mind when counseling;

 Explore the patient’s view;

- Patient’s medical condition

120
- Patient’s knowledge about his/her disease, intervention

- What information is the patient seeking?

- What are the patient’s expectations?

- Social support

 Educate the patient about the subject (disease, intervention)

- Definition, mechanism, prevalence and causes

- Risk of the subject

- Options to act on the subject. Examples; treatment of disease, ways to quit smoking

- Complications and risks of acting

- Complications and risks of not acting

- Any alternatives

- What support systems are available?

- Make sure the patient understands the information

- Beware not to overwhelm the patient

- Provide reassurance as appropriate

- Do not be judgmental

- Offer links and hand outs

- Offer your support and emphasize your availability

Counseling subjects range from educating a diabetic patient about the disease and treatments
to motivating a patient to quit smoking to counseling about a Pap test results. It should be
interactive, and patient thoughts and expectations must be explored. Never lecture a patient!
Talk to your patient in a language he/she understands, and ask them what they got from what
you said to make sure you conveyed the message. Listen carefully to your patient’s responses
and observe body language at the same time. Avoid having your own agenda so rigid in your

121
mind that you forget to listen, clarify and respond to your patient. The quality of your
counseling is improved by the information you have gathered.

In OSCE the case’s stem could look like this; Counsel/Educate the patient about--------- or in XX
minutes take a focused history and counselIn such stations you must first take relevant and
brief history then counsel. You need to know your patient and give a personalized advice.
Sample cases are presented in chapter 5.

Examples of counseling subjects;

- Diabetes

- Birth Control

- Fertility Issues

- Menopause

- Genetics

- Abuse (sexual and physical)

- Hypertension

- Dyslipidemia

- Counsel a patient who had a heart attack

- Smoking Cessation

- Substance abuse

- Alcohol abuse

- Investigation results; like a pap test or incidental finding of a nodule on chest X-ray

- Medication side effects

- Obesity

- Cancer screening

- Risky sexual practices

- Immunization

122
- Allergies

- Health Screening

- Any subject the patient needs information about

References;
1- Mullen, Patricia Dolan, Denise G Simons-Morton, Gilbert Ramı ́rez, Ralph F Frankowski,
Lawrence W Green, and Douglas A Mains. "A Meta-analysis of Trials Evaluating Patient
Education and Counseling for Three Groups of Preventive Health Behaviors." Patient
Education and Counseling 32, no. 3 (1997): 157-73. Accessed November 28, 2014.
http://www.sciencedirect.com/science/article/pii/S0738399197000372.

2- Hill, Edith, and Susan Fryters. Bugs & Drugs. Edmonton: Capital Health, 2006.

123
124
Chapter 5
Sample clinical cases
Edited by; Dr. Subrata Datta
Dr. Sadik Salman
Dr. James Yeung
Dr. Abbeir Hussain
Dr. Erin Toor
Dr. Abdullah Saleh
Dr. Erica Paras

125
Introduction
This chapter is written to give you the opportunity to practice clinical cases as you prepare for
the exams. Most cases in this chapter are real patients whom I encountered during my training.
Some cases are made up, but all are based on common complaints and reviewed and edited by
Canadian physicians.

Each case is written with completeness in mind. However, I don’t guarantee that it covers all
the points on the Medical Counsel of Canada checklists. I strongly encourage you to review the
sample cases, sample checklists and common mistakes posted on the MCC website (refer to
Chapter 1 for link) before practicing the cases in this chapter.

I tried my best to simulate the exam, and presented the cases in a way that is going to make it
easy for you to practice alone or within groups. Each case starts with the door sign, then
suggested notes to write for yourself, most importantly what you are asked to do. The clinical
encounter is then presented in an interactive way to mimic reality. I will indicate which physical
exam exactly needs to be done, details of individual organ system P.E are however, provided in
chapter 3. Examples of possible questions asked by the examiner and their answers are
presented next. Then I will list basic communication and professional points that are part of
your overall evaluation. At the end included variation of the case where other differentials are
more likely when the same chief complaint is presented in a different context. Some tips and
comments are added to this last section.

Although I’m presenting the typical scenario for certain pathology, other differentials are still
possible and it is important to try to rule them in or out by focused history and physical
examination.

The cases cover both history and physical. Some cases history and counseling. Your practice
target should be to take a focused and relevant history in 4-5 min and conduct a focused and
relevant physical exam in 4 min. In this case you should be good in 10 min cases with a question
at 9 min.

Read questions carefully, and perform the task you are asked to do only. In most cases it is
straightforward as the task is history or physical or both. Nevertheless, other terms maybe used
and the scope of what you need to do may expand. Examples;

- Manage this patient; means take a relevant and focused history and perform a focused
and relevant physical exam, order investigations and treat as appropriate.
- Counsel this patient; means take a focused and relevant history and counsel.
- Explore the patient’s concerns; means take a brief history around the problem the
patient wants discussed and dig deep on what he/she knows already and what he/she

126
wants to know. Listen respectfully to the patient’s concern and answer his/her
questions. If you don’t know the answer simply say; I don’t know but I will research it
and get back to you. Never give false information to patients.
- Regardless of the type of question, always make sure the patient is stable and
comfortable

You may run into challenging situations. Those range from an angry patient who is not happy
with your care, to ethical dilemmas where patient’s confidentiality or other basic rights are at
stake. Stay calm and professional in these situations. Other challenges may test your
communication skills and sensitivity to your patient, like when a patient coughs; offer him/her
water. Or if the patient cries show empathy and offer some tissues.

Use a mix of open and closed ended questions and listen attentively to the patient. Respect the
patient’s ideas and beliefs even if they differ from yours. Answer the patient’s questions but
never give false information. Pay attention to the patient’s physical comfort during P.E.

It is a good idea to try to make a closure of the case. This is not possible in many cases as the
time is usually tight. If you finish early explain to the patient what you think is going on, what
investigations you will order and ask if he/she has any concerns. If the patient smokes briefly
mention the risks and offer a counseling appointment if the patient is interested. This applies
to other behaviors like substance abuse, or risky sexual practices. However, do not counsel in a
history and PE station – just mention it.

It is impossible to cover every single possible complaint. The best way to help you pass the
exams is to practice the cases in this chapter, and then create more cases based on other
common complaints. Please note that there are no sample psychiatry cases in this chapter.

READY?

Let’s get started…….

127
Case 1; Cough
Edited by; Dr. Subrata Datta

Door sign

Mrs. Stephanie Edwards is a 58 years old female who comes to your walk in clinic complaining
of cough.

Her vital signs are; BP 130/75, PR 87, RR 20, Temperature 37.3

In the next 10 min conduct a focused and relevant history and conduct a focused and relevant
physical exam. As you do the P.E explain to the examiner what you are doing and your findings.
At 9 min the examiner may ask you a question or questions.

Note; The NAC exam may have a similar stem but the last 2 line will look like this; in the next 11 minutes
conduct a focused and relevant history and conduct a focused and relevant physical exam. As you do the
P.E explain to the examiner what you are doing and your findings. At 8 min the examiner may ask you a
question or questions

Suggested notes to write for your self

Stepphanie Edwards 52 F RR 20 other V.S stable O2 sat?

C/C coughs URTI, Bronchitis, Pneumonia, HF, AECOPD

Hx and P.E Smoking?

Patient encounter

Questions Suggested verbalizing Patient response


History of presenting
problem
Opening start How can I help you today? My cough is getting worse
and really bothersome
Onset Tell me more about it. When Well, I always cough because
did it start? I’ve got this COPD but it is been
bad the last week
Frequency How often do you cough? A lot.. Patient coughs (offer a
glass of water)
Sputum Do you bring up any phlegm? The usual
Amount Did the amount increase Yes indeed

128
Color What color was it and what is it It was clear but now it’s
now? yellowish
Hemoptysis Did you cough up any blood? No never
Dyspnea Do you feel short of breath? Only when the cough is too bad
Chest pain Do you have chest pain? Ahh, kind of
Tell me more about it It’s right here (patient points to
the left anterior axillary line at
the level of sixth rib with the tip
of his finger), and not too bad
Radiation Does it go anywhere? No
Onset When did it start? Few days ago
Progression Is it getting worse over time? No
Frequency Is it there all the time? No, it comes and goes
Aggravating factors What makes it worse? Cough and moving
Relieving factors What makes it better? If I’m not coughing I guess
Relation to breathing Does it get worse if you take a I haven’t noticed, I don’t think
deep breath? so
Quality How would you describe it? It’s like needles
Wheezing Do you have noisy breathing or Sometimes, but not very often
hear wheezing?
URTI Do you have runny nose or feel No
congested?
Constitutional Have you had fever? I don’t think so
Have you lost weight recently? No, I gained 2 lb
Do you get drenching night
sweats that you need to change No
the sheets?
HF and PE Does it become difficult to No
breathe when you lie flat?
Do you have legs swelling? No
Have you been bed bound No
recently?
Idea component of FIFI; Is the cough affecting your Yes, I missed 2 days of work
Function function?

Sick contacts Have you been in contact with Not that I know of
sick people?
Past medical history
COPD History When were you diagnosed with 5 years ago
COPD
When was your last breathing I think I had one a year ago
test?
Do you remember the result? No
Who follows your COPD My family doctor
Have you ever been Yes, I had a bad chest infection
hospitalized? a year ago and was in the
hospital for a week

129
What treatment you received in Antibiotics and they had me on
hospital? the CPAP machine
Did you need intubation or ICU? No, none of that
Have you had any other My cough gets worse 2-3 times
exacerbation? each year but gets better with
antibiotics and steroids
Did you take your flu shot? Yup, I take it every year
Did you take your pneumonia Yes, they gave it to me in the
shot? hospital the last year
Other medical problems Do you have any other medical No
And surgeries problems particularly a clot in
your leg or lungs, heart disease
or cancer?
Have you had any surgeries? No
Medications and allergies
Type What medications are you on Symbicort inhaler
Dose and frequency How much and how often do I take 2 inhalations in the
you use it? Always compliant? morning and 2 in the evening
Side effects Do you have mouth thrush or No
any other side effects?
Have you ever been tested for No
osteoporosis?
Have you had any fractures? No
Allergies Do you have any food or Not that I know of
medications allergies?
Family history
Does anyone in your family My dad died of a heart attack
have any of; heart disease, when he was 80
diabetes, high blood pressure?
Or Cancer?
Social history
Smoking Do you smoke Not any more
When did you quit? 4 years ago
How much did you smoke? A pack a day
For how long? For 30 years
Alcohol Do you drink alcohol? Very occasional
Illicit drug use Do you use recreational drugs? I used to smoke marijuana
occasionally but stopped 5
years ago
Occupation What do you do for living? I’m a secretary
Exposures Have you ever been exposed to No
chemicals or asbestos?
Have you been in contact with No
Tuberculosis patients?
Have you traveled recently? Not for the last 10 years
Pets? Do you have pets? No

130
Sexual activity Are you sexually active? Not after my husband died a
year ago
STOP!! Respond.. do not just
continue asking your questions.
Sorry to hear that
Social support Do you live by yourself, any I have one daughter, she lives 2
other family support? blocks away, I see her every day
Idea component of FIFI; Thank you very much for all the Not really, thank you doctor
Expectations information, Do you have any
questions for me?

Physical examination;

- Take permission, wash your hands and use respectful draping


- Say that you noticed that the patient is tachypnic with a RR of 20 and that other vital
signs are normal. Indicate that you would like to get the pulse oxymetry checked. The
examiner says; 95% on room air,
- Do general inspection, looking for signs of respiratory distress, the body habitus, hands,
face, and skin. Pay particular attention for cyanosis and clubbing
- Inspect the chest, palpate, percuss and auscultate the lungs
- Auscultate the heart
- Examine JVP, if you ran out of time indicate that you would

Sample questions you may be asked by the examiner;

1- What is the most likely diagnosis?


Answer; Acute exacerbation of COPD (AECOPD)
2- What investigations would you order?
Answer; CBCD, Chest XR, Creatinine, lytes, sputum gram stain and culture, ECG, ABG,
Spirometry when the patient returns to base line
3- How would you manage this patient?
Answer;
- Short coarse corticosteroids; prednisone 40mg P.O daily for 5 days
- Antibiotics; Amoxicillin 1g P.O TID for 7 days
- Bronchodilators; Continue Symbicort, add Atrovent 500ug INH MDI q 6 hours for 7 days

Rating scales points;

- Examinee introduced self and position

131
- Addressed patient with name
- Used proper non-verbal communication
- Good organizational skills
- Examinee spoke clearly (accent didn’t get in the way)
- Listening and questioning skills
- Showed rapport with patient
- Attentive to patient physical comfort
- Medical knowledge adequate
- No misinformation was provided to patients
- No concerning ethical/legal issues
- The examinee is respectful of other health care members

Variations of the case and some tips;

1- Case; 23 years male complaining of cough

History; had flu like symptoms then persistent cough for 2 weeks. Past medical history is
significant for treated Syphilis a year ago. When you ask about sexual history you find
that he had many partners over the last year and doesn’t use condoms.

Physical exam; Temperature 37.5, bilateral crackles and wheezing. No respiratory


distress.

At 9 min the examiner tells you that the patient chest XR showed bilateral diffuse
reticular infiltrates (Note that you may be asked to read a chest XR yourself) and asks
you about the most probable diagnosis

Discussion; this presentation is typical for PJP Pneumonia (an HIV defining illness). The
history of unprotected sex and CXR findings make the diagnosis more likely. However,
some patient’s may have a normal chest XR, so it is always a good idea to check for HIV
in patients with a history of unprotected sex and lower respiratory symptoms. Keep in
mind that other differentials like pneumonia, viral bronchitis are possible, and show the
examiner you are thinking about them

2- Case; 65 male, history of 60 pack-years smoking presenting with 2 weeks history of


productive cough

History; the patient had 2 episodes of hemoptysis coughing up sputum mixed with
blood. He lost 20 lb unintentionally over the last 3 months and is complaining of
132
persistent fatigue. He may have been exposed to Tuberculosis (TB) from a co-worker
years ago

Physical exam; normal vital signs. Respiratory exam revealed dullness, increased tactile
fremitus, bronchial breath sounds and positive egophony over the right middle lobe

Discussion; this patient’s presentation has 2 differentials on the top of the list; lung
cancer and TB. The patient has right middle lobe consolidation by physical exam along
with hemoptysis and constitutional symptoms. Include chest CT scan and sputum for
acid fast Bacilli in your investigations. Note that the right middle lobe is not a typical
location for TB which tends to involve the apices. And that the patient didn’t report
fever and had normal temperature on P.E. Nevertheless, you still need to exclude this
deadly infection with a history of exposure.

3- Case; 22 year old female complaining of recurrent episodes of cough

History; For the last several months the patient has had recurrent bouts of cough and
shortness of breath especially after running for more than 5 blocks. She feels wheezy
sometimes, and coughed few times in the middle of the night. She had similar episodes
when visiting one of her friends. On further questioning you discover that the patient’s
friend has a cat, the patient has eczema and her mother is asthmatic.

Physical exam; normal

Discussion; the patient most likely has asthma, although GERD and atypical infections
are also possible. It is wise to ask about heart burn and sour taste of saliva. Initial
investigations include; CBCD, chest XR, Spirometry and a Methacholine challenge test. In
some cases you may encounter a patient with established diagnosis of asthma who had
recent deterioration of symptoms, make sure you assess severity by asking about
frequency of symptoms, the need for Ventoline, night time symptoms and days missed
from school/work. Also ask about triggers. It is necessary in such cases to watch the
patient using his/her own inhalers to make sure the technique is correct.

4- Familiarize yourself with the technique for MDI and PDI inhalers. In some cases you may
need to show the patient how to use them, or watch the patient use his/her own. The
following you tube videos are helpful;
MDI; http://www.youtube.com/watch?v=YWNcPReibZA
PDI; http://www.youtube.com/watch?v=-tyF-MC1qQo

133
5- The following website contains the Canadian guidelines for respiratory diseases;
http://www.respiratoryguidelines.ca/home

6- Always keep your differential wide. The above cases illustrate how a single complaint
can be a manifestation of a variety of diseases. You will be able to narrow down your
differential to two or more possibilities. It is important to show the examiner that you
are trying to do so. One of my patients presented with cough, lung cavities on chest XR
and a history of strong exposure to active TB from her mom. She was put on isolation
and admitted as a case of TB. To our surprise all her TB tests came back negative and
lung biopsy confirmed the diagnosis of Cryptogenic Organizing Pneumonia (COP)

134
Case 2; Fatigue
Edited by; Dr. Subrata Datta

Door sign

Anya Glenn, a 35 years old female presenting with fatigue.

Vital signs;

BP 110/70
PR 67
RR 12
Temperature 36.5

In the next 10min take a focused and relevant history and conduct a focused and relevant P.E.
As you do the P.E explain to the examiner what you are doing and your findings. At 9min the
examiner may ask you a question or questions.

Note; The NAC exam may have a similar stem but the last 2 line will look like this; in the next 11 minutes
conduct a focused and relevant history and conduct a focused and relevant physical exam. As you do the
P.E explain to the examiner what you are doing and your findings. At 8 min the examiner may ask you a
question or questions

Suggested notes to write for your self

Anya Glenn 35 F DDx; Depression


C/C; Fatigue Hypothyroid
Cancer
Chronic infection
CTD
Hx. GI
P.E
Sleep, diet, exercise, stress

Patient encounter

Questions Suggested verbalizing Patient response


History of presenting
complaint

135
Opening question How can I help you today? I’m feeling very tired, I just
have no energy
Onset and duration When did you start to feel The last 6 months
tired?
Associated symptoms Have you had other I’m also constipated
symptoms?
Details about constipation How often do you poop? Once a week if I was lucky
Is your stool hard or soft? Sometimes it’s hard
When did this start? Few months ago
How often did you poop Every day! I was very regular
before?
Do you have diarrhea as No
well?
What color is your stool? Normal brown
Did you notice any blood? No
GI system Do you have abdominal pain? No
How about nausea/vomiting? No
Did you notice that your eyes No
are turning yellow?
Hypothyroidism symptoms? Do you feel more cold than Very much, my 13 years old
usual? makes fun of me layering up
when its sunny outside
Have you noticed any hair or My hair is thinner and falling
skin changes? and my skin is dry
Do you have difficulty getting No
up from a chair?
Have you noticed swelling of No
your neck?
Did people tell you your voice No
changed?
Has there been any change to Yes, they are not regular any
your periods? more, the last 4 months I had
only one period! And I can
assure you I’m not pregnant!
Constitutional symptoms? Do you have fever? No
Have you lost weight No, I think I gained 4 lb
recently?
Do you have drenching night No
sweats that you have to
change the sheets?
Anemia Did people say you look pale? No
Note that questions about Do you feel your heart is No
menses/bleeding are covered racing?

136
Heart failure Do you feel out of breath? No
Do you have swelling around No
your ankles?
Depression screening Do you feel depressed? Well, it is depressing when
you don’t seem to get
anything done
I understand this is hard, but Not really, it’s more of
do you feel very sad? frustration
Have you lost interest in No
activities you enjoyed
before?
Did your sleep change? No
Do you wake up very early No
recently?
Did your appetite change? No
Did you notice a change to I’m slower than before
your concentration and
memory?
CTD Do you have swelling or pain No
Note that some questions are in your joints?
already covered Do you feel stiff in the No
morning that you need some
time to get going?
Chronic infection Do you have cough? No
Note that a lot of the Do you feel burning when No
questions are already you pee?
covered Have you travelled recently? No
Do you have skin rash? No

Past medical history


Have you been diagnosed No. I’ve been healthy so far
with a medical disease like
diabetes?
Medications
Do you take any No
medications?
Allergies
Do you have allergies to No
medications or food?
Family history
Does anyone in your family My mother has high blood
have a medical condition? pressure

137
Does anyone have a thyroid No
disease?
Cancer? Or heart disease? No
Social History
Smoking Do you smoke? No
Alcohol Do you drink alcohol? No
Recreational drugs Do you use recreational No
drugs?
occupation What do you do for living? I’m a sales representative
Living conditions With whom do you live? With my husband and 13
years old son
Stress Any stress at work or home? Not really no
Idea component of FIFE; How is fatigue affecting your It makes me frustrated, I
FUNCTION: Effect of fatigue life? push myself to get things
on work/home done but haven’t got in
trouble yet
Diet Do you eat healthy meals I do, I eat very healthy and
with good portion of protein, only buy organic food
carbs, fruits and veggies?
Exercise Do you exercise? I wish!
FIFE; Expectation Thank you for sharing all this Well, what is wrong with
information. Do you have any me? I never felt like this
questions for me? before

I understand your frustration,


I need to examine you first
and run some tests and
hopefully we will get an
answer

Physical Examination;

- Mention that the vital signs are normal


- Do a general exam starting with the hands, feel the radial pulse, inspect the face; Look
for jaundice or pallor; look at the buccal mucosa for hemorrhage or infection as well as
hygiene status. Inspect the skin. Examine the JVP
- You need to cover multiple organ systems; inspect the chest and auscultate the heart
and lungs (no need for palpation and percussion). Inspect and palpate the abdomen
paying particular attention for masses or splenomegaly. Feel the lymph nodes. Inspect
the lower limbs and check for edema

138
- Examine the thyroid; inspect the neck with and without the patient swallowing and
palpate the thyroid for temperature, tenderness, masses and enlargement. Examine the
cervical lymph nodes. Check for manifestations of thyroid disease; Inspect the eyes from
the side for exophthalmos, examine extraocular movements and examine for lid lag.
Test the proximal muscles of the upper and lower limbs for weakness. Check the pulse.
Test DTR. Inspect the lower limbs for pretibial myxedema

Sample questions you may be asked by the examiner;

- What is the most likely diagnosis?


Answer; Hypothyroidism
- Name a single investigation that you would do to confirm this diagnosis?
Answer; TSH (Thyroid stimulating hormone)

Rating scales points;

- Examinee introduced self and position


- Addressed patient with name
- Used proper non-verbal communication
- Organizational skills
- Examinee spoke clearly (accent didn’t get in the way)
- Listening and questioning skills
- Showed rapport with patient
- Attentive to patient physical comfort
- Medical knowledge adequate
- No misinformation was provided to patients
- No concerning ethical/legal issues
- The examinee is respectful of other health care members

Variations of the case and some tips;

1- Case; 60 year old female complaining of fatigue

History; positive for drenching night sweats and newly diagnosed breast cancer in
her sister at age 55. Screening for depression was also positive

P.E; palpable 1*1.5 right axillary lymph node. Otherwise normal including breast
examination

139
Discussion; this patient presentation strongly suggests Breast cancer, or other
malignancy. At the same time she is depressed, which could be primary or secondary
to malignancy or grief. You need to do thorough investigations including CBCD, urea,
Creatinine, lytes, LDH, INR, PTT, ALT, AST, ALK, Albumin, total protein, bilirubin, CK,
CXR, ECG. I would go straight to chest CT with axillary view in this case (no need for
mammogram or breast US, as you would end up doing CT if they were positive for
further details and if negative because malignancy is highly likely). I would still get a
chest XR because it’s easier and faster and may need it in the future if complications
arise (good to know her baseline). The psychological component is very important;
address and treat depression, and dig deep into social support. Early diagnoses and
referral could save this patient’s life. Remember to keep other differentials in mind.

2- Case; 25 year old female presenting with fatigue

History; Heavy menstrual cycles, diet is mainly fast and frozen food. Has occasional
shortness of breath and palpitations, especially on exertion. No chest pain. Wants to
get pregnant but thinks she wouldn’t be able to handle it. Appears very anxious and
worried that something serious is going on. When asked why she feels it’s serious
she shares with you that her sister in law died with cervical cancer a year ago

P.E; Pale conjunctiva and buccal mucosa. Tachycardia at 110 with regular pulse. JVP
and cardiac exams are normal. No other findings

Discussion; this patient most likely has iron deficiency anemia from her heavy
periods. B12 and folate deficiency are also possible and could be concomitant given
her diet history. The shortness of breath and tachycardia could be secondary to the
anemia or to concurrent cardiac condition that could be unrelated to or caused by
anemia. You need to know why she has menorrhagia, and obtain detailed menstrual
history, and endocrine system review.
Get a CBCD, iron, total iron binding capacity and ferritin, B12, folate, TSH, Creatinine,
urea, and lytes, and a baseline ECG and chest XR. With negative family history of
heart disease and absence of risk factors you can treat the anemia and reassess the
patient’s palpitations and shortness of breath; if persistent, an echocardiogram will
help to sort it out.

3- Case; 42 year old female complaining of fatigue and pain every where

140
History; Feeling tired most of the time for the last 5 months with diffuse poorly
localized pain that seems to migrate. Sleep is interrupted. The patient is very anxious
about it, and wants to know what’s wrong. She made some mistakes at work
because she can’t focus and is afraid she may lose her job. She saw three other
physicians before coming to see you and had a lot of investigations done, she is
frustrated because they said it’s all in her head. Comprehensive review of systems
was negative apart from the above mentioned symptoms. Screening for depression
was negative.

P.E; normal vitals, general, cardiac, respiratory, abdominal and joints P.E. You test 3
fibromyalgia trigger points; occiput at the nuchal ridge, trapezius, cervical and find
them tender

Discussion; Fibromyalgia is a debilitating condition that is poorly understood.


Exclude possible underlying diseases before arriving at the diagnosis. It is a good
idea to get the previous investigations and make sure age appropriate cancer
screening is done. Keep in mind hypothyroidism and obstructive sleep apnea.
Depression and anxiety are commonly seen in fibromyalgia patients, screen and
treat as appropriate. If everything else is negative and you think fibromyalgia is most
likely refer to a rheumatologist where multi-disciplinary individualized treatment
plan can be initiated.

141
Case 3; Chest pain in the ER

Edited by; Dr. Jay Shavadia

Door sign;

Mr. Arthurs, 65 year old male, brought by EMS with chest pain

BP 120/75
HR 91
RR 14
Temperature 37

There is a nurse in the room

In the next 9 min manage the patient. Ask the nurse to do any orders you deem
necessary. As you examine the patient explain to the examiner what you are doing
and your findings. At 10min the examiner will ask you a question or questions

Note; In this case you will be communicating with the nurse and the patient. Communication
with the nurse is presented in italic. You need to make sure the patient is stable, give emergency
medications, take a focused and relevant history, perform a focused and relevant P.E and order
investigations. The P.E is included in the case. Treat the nurse with respect, and give clear orders
specifying the dose and route of medications.

Even though it is an ER case, it is important that you introduce yourself to the nurse and patient.

Suggested notes to write for your self


Mr. Arthurs/65
ABC MI
P.E
Aortic dissection

142
General screening exam Pericarditis/Pleuritis

O₂, IV access, ASA, nitrate

Patient encounter

Your actions Suggested Patient or


verbalizing nurse
response
ABC
Can you open your
mouth Sir? The
airways are clear,
and the patient is
breathing, I’m
feeling the pulse and
it is regular
O₂ and Can you put the O₂ given,
monitoring patient on 4L oxygen monitor
and attach him to attached
the cardiac monitor
Can I get another set Unchanged
of vital signs
What is the O₂ 100% on 4L
saturation
IV and Can you establish an Done
investigations IV access and draw
blood for CBCD,
Creatinine, urea,
lytes, Troponin, CK-
MB, INR, PTT IV in, fluids
given
Can you start the
patient on normal
saline 0.9% at
75cc/hour

143
Chest pain Do you have chest No, it’s gone,
pain right now? those guys
where magical
EMS history I understand EMS
brought the patient
What was done? They did an
ECG and gave
him 2 doses of
nitroglycerine
SL 0.4 mg
The nurse
hands you the
ECG
Read the ECG, The ECG of Mr
if you don’t Arthur was done at
the examiner 10:50 am today.
will ask you to Normal sinus rhythm
do so and axis. No ST
changes. P wave, PR,
QRS and QT, are
within normal
There is T wave
inversion in leads
V2, V3, V4
Order ECG and Can I get another Ordered, ECG
CXR ECG and stat CXR will be
available in 5
min
History
Sir tell me more It was right
about this pain (ask here (patient
for character, uses his hand
continuous / to point to the
intermittent) center of his
chest) and felt
like a heavy
block lying on
my chest.

144
Continuous
over 20mins,
till the NTG
spray by EMS
Did it go anywhere? I felt it over
(shoulders, jaw, my shoulder
neck, elbow, arms,
back)
How severe was it It was 9
on a scale from one
to 10 with 10 being
the worst pain ever?
What were you I was shoveling
doing when you had the snow
it?
Did anything make it No
worse? Like
movement,
breathing, cough,
laying down
Have you had similar Not as bad, I
pain before? had the same
pain once
before but it
was 4, I was
also shoveling
but it went
away in less
than a minute
when I sat
down
Did you get any No, I didn’t see
treatment for it? a doctor for it
Did you feel dizzy I felt
today? lightheaded
when I had the
pain

145
Did you lose No
consciousness?
Did you feel your No
heart racing?
Did you feel short of No
breath?
Did you have cough? No
Did you cough up No
blood?
Did you have nausea I felt
or vomiting? nauseated
Do you have No
diabetes?
Do you have high Yes
blood pressure?
Are you on I take Coversyl
medication for it? 8mg every day
Do you have high Yes, I take
cholesterol? Lipitor for it
How much?
Not sure I
think 20mg
Do you smoke? Yes
How much and for 1 pack a day
how long? since I was 20
Do you drink I enjoy a glass
alcohol? of wine with
dinner
Did anyone in your My father died
family have a heart of a heart
disease? attack when
he was 50
The patient Did I just have a Well, It’s
asks you a heart attack doctor? possible; we
question need to do
some tests to
know exactly

146
what’s going
on.
I need to ask few Patient nods
more questions his head yes
Do you have allergy Not that I
to medications? know of
Do you have allergy No
to aspirin?
Do you have Asthma No
or other lung
disease?
Are you still doing I’m fine I don’t
ok? Have you had feel any pain
any more chest
discomfort?
Have you had a No
surgery or accidents
in the last 6 months?
How about bleeding No
from your stomach
or in your head?
Have you been No
diagnosed with
cancer?
Do you use any No
‘street drugs’?
Can you please give Given
the patient Aspirin
325mg orally
Can you give Plavix Given
300mg orally now
Is any of the The ECG is
investigations back back
You look at the ECG unchanged
ECG, and find
it’s
unchanged, if
the examiner

147
asks you to
read then
specify details
You share a The patient came Yes
summary with with ischemic chest
the nurse pain with ST
changes, resolved
with 2 doses of
Nitro, given ASA,
Plavix, Oxygen and is
now pain free,
Attached to the
monitor, vitals are
stable. He is on IV
N.S. The second ECG
is normal, other
investigations are
pending
Physical Sir do you mind if I Remember to
examination examine you? wash your
hands before
P.E
General I’m looking at the
hands for nicotine
stains, cyanosis, and
palmar erythema all
negative. I’m feeling
the pulse noticing it
is regular, no
collapsing pulse. I’m
comparing the radial
pulse of both arms
and notice it is
symmetrical. I’m
inspecting the face
looking for central
cyanosis and pallor.

148
JVP I’m going to lift the
bed up a little bit.
I’m looking at the
JVP between the 2
heads of the
sternocleidomastoid.
Measuring the
height from the
sternal angle it is 3
cm. Can you take a
deep breath? It is
going down with
respiration, is double
impulse, not
palpable and
disappears with a
pressure to the root
of the neck
Chest I’m going to feel for
examination your heart and listen
to your heart and
lungs. I’m inspecting
the chest, no visible
pulsations, scars,
deformities, dilated
veins or skin
changes. I’m feeling
the apical beat and
notice it is located in
the 5th intercostal
space midclavicular
line as expected. No
heaves, no thrills.
I’m auscultating the
heart, normal S1, S2,
no S3, S4 and no
murmurs. Can you
take a deep breath

149
in and out?
Symmetrical
vesicular breath
sounds bilaterally,
no crackles or
wheezing.
Abdomen Cover the chest, and
lift the gown up.
Lightly then deeply
palpate the
abdomen
Lower limbs Look for edema,
asymmetry, redness,
swelling calf
tenderness
Consultation
I’m going to consult Will page
cardiology them
Thanks
Sir, will get the heart
specialists involved Not now,
and take good care thank you
of you , do you have
any questions?
You’re most
welcome

150
Sample questions you may be asked by the examiner;

- What is your differential?


Answer: Acute coronary syndrome, Aortic dissection

Rating scales points;

- Examinee introduced self and position


- Addressed patient with name
- Used proper non-verbal communication
- Organizational skills
- Recognized and managed emergency effectively prioritizing actions
- Examinee spoke clearly (accent didn’t get in the way)
- Listening and questioning skills
- Showed rapport with patient
- Attentive to patient physical comfort
- Medical knowledge adequate
- No misinformation was provided to patients
- No concerning ethical/legal issues
- The examinee is respectful of other health care members

Variations of the case;

1- Case; 42 male, hypertensive, not on treatment presenting with chest pain

History; Sudden onset tearing chest pain radiating to the back. The patient is a
smoker and known to have hypertension but not on treatment

P.E; BP right arm; 190/100, left arm 168/95. Pulse weaker on the left. Diastolic
murmur is heard over the aortic area

Discussion; Aortic dissection can be easily missed, and a lot of patients don’t have
typical presentation. Always keep it in the back of your mind when assessing
patients with chest pain especially if they were hypertensive on presentation. CT is
the gold standard for diagnosis

2- Case; 35 year old female, presenting with sudden onset chest pain

151
History; The patient suddenly felt chest pain while watching TV, she was as well out
of breath and coughing. No hemoptysis. She is a smoker and has a previous history
of 3 spontaneous abortions and one still birth. Not previously diagnosed with a
medical condition
P.E; The patient was tachycardic at 120, tachypnic at 24. BP was 140/85. Normal
temperature. O₂ saturation was 88% on room air. General and chest examination
were normal. Lower limb examination revealed that the right leg was 3cm bigger
than the left.

Discussion; A history of recurrent spontaneous abortions and still birth is highly


suspicious for coagulopathy. Along with discrepancy of the size between the 2 legs
and the nature of pain makes a pulmonary embolism high on the differential.

3- 24 male, presenting with chest pain

History; Stabbing left sided chest pain worse with respiration and cough. He had a
cold 10 days ago. Otherwise healthy non- smoker, non- drinker and doesn’t use
recreational drugs

P.E: Stable vital signs. A triphasic leathery rub is heard over the apex. Otherwise
unremarkable

Discussion; Pericarditis is high on the differential. Try to rule out secondary causes
like a viral infection or connective tissue disease.

4- Although atherosclerosis is the most common cause of ischemic chest pain, keep
in mind other causes like Prinzmetal angina or cocaine induced coronary artery
spasm especially in younger populations.

152
Case 4; Chest pain in an out patient

Edited by; Dr. Jay Shavadia

Door sign;

Mr Adam Bailey, 52 years old male, comes to your clinic because he had 2 episodes
of chest pain.
Vital signs are;

BP 145/90
PR 88
RR 14
Temperature 37⁰C
O₂ saturation 99% on room air

In the next 9min take a focused and relevant history and conduct a focused and
relevant medical exam. As you do the P.E explain to the examiner what you do and
your findings.
At 9min the examiner may ask you a question or questions

Sample notes to write for yourself;


Mr Bailey 52M Details about the pain
Cardiovascular risk factors
GI system
Chest pain Respiratory
Hx and P.E

Patient encounter;
Your actions Suggested Patient response
verbalizing

153
History of present How can I help I’m worried
illness you today? doctor, I’ve had
chest pain
How many times 2 times
have you had
chest pain?
Tell me about the I was walking my
first time dog as usual, it
was a bit cold that
day, then I had
pain in my chest
right here (patient
uses his hand to
point to the
center of his
chest). I stopped
and took a couple
of deep breaths
then it went away
How long did it Maybe one or two
last? minutes
Did it go No
anywhere?
How did it feel? It felt like a rock
on my chest, it
was horrible
How severe was it It was 8
on a scale from 1-
10, 10 being the
most severe pain
ever?
Did you feel dizzy? No
Did you sweat A little bit
when you had the
pain?
Did you feel I felt a bit
nauseated or had nauseated, but
vomiting? didn’t throw up

154
Did you feel your No
heart racing?
Did you feel out of A little
breath?
How far did you 4 blocks
walk before you
had the pain?
How about the The same day
second time you when I was just
had pain? about to enter the
house, it felt
exactly the same
Did you have a No, that’s the first
similar pain time, it happened
before? the last week
Did you get No
medical help?
So it came up by yes
walking 4 blocks in
the cold air, and
went away by rest,
is that right?
Did it get worse No, it got better
with breathing?
Do you cough? No
Did you have No
fever?
Did you have a No
cold recently?
Do you feel your No
breathing is
wheezy?
Do you cough up No
blood?
Do you get heart No
burn?

155
Do you feel a sour No
taste in your
mouth?
Do you have No
abdominal pain?
Do you have No
diarrhea? No
Constipation?
Have you ever lost No
consciousness?
Past medical
history?
Do you have any I have a high
medical blood pressure
conditions? that I try to
control with diet
Do you have No
diabetes?
High Cholesterol? Not that I know of
Have you ever
been diagnosed No
with a heart
disease?
Have you ever No
been hospitalized
or had surgery?
Family history?
Did anyone in your My mom died of a
family have a heart attack when
heart attack? she was 65
How about your He had prostate
dad? cancer
Any other diseases No
in the family?
Did anyone die No
suddenly in your
family?

156
Medications and
allergies?
Do you take any No
medications?
Do you take any No
over the counter
medications or
herbs?

Do you have No
allergy to
medications or
food?
Social history
What do you do I’m a biology
for living? teacher
With whom do With my wife
you live? yes
Is she healthy?
Do you have any No
children?
Do you smoke? I tried it once
when I was young
and didn’t like it
Do you drink Very occasionally
alcohol?
Do you use No
recreational
drugs?
Do you exercise? I take the dog for
a walk every day
but haven’t done
so for the last
week
Tell me about your Well, my wife is
diet healthy and
always tries to get
me to eat like her,

157
I cheat and eat a
lot of junk food

Closure Thank you for Not at the


sharing all this moment, thank
information with you
me. This pain is
likely angina. We
need to do some
tests. Do you have
any questions?

Physical Exam;
- Mention that the vital signs are normal
- Do a general exam of the hands and face; Look for nicotine stains, palmar erythema,
cyanosis, Osler nodes, Janeway lesions, feel the elbows for tendon Xanthomas. Inspect
the face for pallor, cyanosis, corneal archus and Xanthelasma
- Examine the JVP, Listen for carotid bruits and feel the carotid pulse
- Examine the precordium
- Auscultate the lungs paying particular attention to wheezing and basal crackles
- Feel the radial and brachial pulses, comparing both sides and checking for Brachioradial
delay
- Inspect the lower limbs, check for edema, and feel the dorsalis pedis, posterior tibial
and popliteal pulses
- Inspect the abdomen for flank fullness, asymmetry or masses, Feel the abdominal aorta
and listen for bruits. Palpate for enlarged liver and check for ascites

Sample questions you may be asked by the examiner;


- What is your differential?
Answer; Acute coronary syndrome, prinzmetal angina, esophageal spasm
- What investigations would you order?
Answer; ECG, chest XR, CBCD, urea, Creatinine, lytes, INR, PTT, troponin. If ECG has
no ischemic changes, and troponin negative, then a stress test

Rating scales points;


- Examinee introduced self and position
- Addressed patient with name
- Used proper non-verbal communication

158
- Organizational skills
- Examinee spoke clearly (accent didn’t get in the way)
- Listening and questioning skills
- Showed rapport with patient
- Attentive to patient physical comfort
- Medical knowledge adequate
- No misinformation was provided to patients
- No concerning ethical/legal issues
- The examinee is respectful of other health care members

Variations of the case;


1- Case; 30 year old male, complaining of chest discomfort

History; Retrosternal burning sensation, typically felt when overeating, or upon


consuming fatty or spicy food. On occasion, the burning is also felt upon lying down.
Symptoms started a year ago, but worsened the last 3 months. The patient also has
dry cough for the past 3 months. Screening for cardiovascular risk factors was
negative.

P.E; Normal

Discussion; Even in low risk patients, always rule out cardiac causes of chest pain.
This patient’s history is typical for GERD. However, he could have esophagitis or
other concomitant complications. The cough could be a manifestation of GERD or
could be totally unrelated; the best test is improvement with GERD treatment.

2- Case; 23 F complaining of chest pain

History; One week of sharp left sided chest pain localized anteriorly from the second
through the 4th intercostal spaces. Worsened by deep breathing and moving. The
patient had the flue one month ago. Review of the cardiovascular, respiratory and
musculoskeletal system was negative. She is healthy, doesn’t use drugs and plays a
variety of sports on daily bases.

P.E; Left sided chest wall tenderness from the 2ndd to 4th spaces. Otherwise
unremarkable, V.S stable

159
Discussion; Chest wall tenderness makes Costochondritis highly likely, Other causes
of chest pain must, however, be excluded. Trauma and a viral infection are
implicated as possible causes for Costochondritis, no specific tests required. Treat
the patient with rest and anti-inflammatory medications.

160
Case 5; Back pain

Edited by; Dr. James Yeung

Door sign

Mrs. Elizabeth Peter’s is a 45 years old female, presenting with low back pain.
Vital signs;
BP 120/80
PR 82
RR 12
Temperature 36.5

In the next 9min take a focused and relevant history and conduct a focused and
relevant medical exam. As you do the P.E explain to the examiner what you do and
your findings.
At 9min the examiner may ask you a question or questions

Note; the NAC may have a similar stem but the last 3 line will look like this; In the
next 11 min take a focused and relevant history and conduct a focused and relevant
medical exam. As you do the P.E explain to the examiner what you do and your
findings. At 8min the examiner may ask you a question or questions

Sample notes to write for yourself


Elizabeth Peters Herniated disk?
45 F Infection
Malignancy
Back pain AS
Hx, PE Spinal stenosis
Degenerative disc disease
Fracture
Mechanical back pain
Cauda equina

Patient encounter

161
Your actions Suggested Patient response
verbalizing
Opening start How can I help I have back pain
you today? doctor
Tell me more It is right here
about it (patient points to
the center of
lower back)
Radiation Does the pain I feel it here
radiate (patient points to
anywhere? the right lateral
leg and foot)
Onset When did it Almost a year now
start?
Tell me about I don’t think so ,
the first time can’t remember
you had it, did doing anything
you have specific
trauma, lift
something
heavy?
Was it sudden I would say
or insidious? insidious
Intensity and How severe is It’s about five to
progression it on a scale seven
from 1-10, 10
being the
worst pain I get it once or
ever? twice a week,
Do you get it especially when I
every day? clean the house
Not really

Is it getting I take Tylenol or


worse over the Advil and rest, it
last year? helps

162
What do you
do when it’s
very bad?
Aggravating and What makes it Moving, especially
relieving factors worse? bending forward
Rest
What makes it
better?
So it worsens That’s right
with activity
and improves
with rest?
quality How would It’s aching
you describe
it?
Constitutional
Fever Have you had No
Weight loss fever?
Did you lose No
Night sweats weight over
the last year?
Do you get No
drenching
night sweats
that you have
to change the
sheets?
Associated symptoms
CTD Do you have No
joints pain or
swelling,
particularly the
shoulders or
hips? No
Do you have
skin rash? No

163
Motor/sensory deficit Do you get No
tired when you
chew food?
Do you feel
stiff in the
morning and No
need some
time to get
going?

Do you feel
your legs are Not at all
weak?
Is it hard to get No
up from a
Urinary chair?
retention/Stool Do you feel The same area
incontinence numbness or that hurts feels
tingling in your numb sometimes
legs and feet?
Saddle anesthesia
Is it hard to No
pass urine?
GI Did you No
become
incontinent of
stool?

Do you feel No
numbness in
Urinary your buttocks?

Do you have None of that


tummy pain?
Vomiting?
Diarrhea or
constipation?

164
Is it burning or No
painful to pass
urine?
Were you ever No
Obstetric/gynecologic diagnosed with
kidney stones?
Have you
noticed a No
change in urine
color or smell?

Are your Yes


periods
regular?
Have you No
noticed a
change in the
amount or
timing of
bleeding?
Do you have
vaginal None of that
discharge or
pelvic pain?
The rest of red flags Have you ever
for back pain been No
Cancer diagnosis diagnosed with
cancer?
IV drug use Do you use No, I would never
Intravenous do that
Steroids use drugs?
Trauma Do you take No
steroids?
Have you ever
had a trauma No, never
to your back?

165
Osteoporosis Have you ever No
had fractures?
Function component Is the pain Not my function,
of FIFE affecting your but it’s
function or frustrating, I want
mood? to know what’s
wrong
I understand,
we will do our
best to figure it
out, is it ok if I
ask you few
more
questions? Absolutely
Previous treatment Have you seen No
a doctor or
chiropractic
before?
Past Medical and
surgical history
Do you have No
any other
disease like;
heart disease,
diabetes or No
high blood
pressure?
Were you ever
diagnosed with No
Psoriasis,
Inflammatory
bowel disease
or a sexually
transmitted No
infection?
Have you ever
had a surgery?
Family history

166
Did anyone in No, they were all
your family healthy
have cancer?
Medications and
allergy
Do you take Tylenol once or
any twice a week
medication?
Are you
allergic to food No
or medication?
Social history
Smoking Do you smoke? No
Alcohol Do you drink Occasionally
alcohol?
Illicit drug use Do you use No
recreational
drugs?
Occupation What do you I’m a teacher
do for living?
Did you miss Not yet!
days of work
because of
back pain?
Living conditions With whom do With my husband
you live? and 20 years old
daughter
Exercise Do you No
exercise?
FIFE Thank you for No, thank you for
sharing all this being thorough
information
with me. Do
you have any
questions for
me?

Physical examination;

167
- Examine the back – page 42, chapter 5
- Pay particular attention to sensory and motor deficit and try to determine the level of
the lesion. Most common for herniated disk are; L4-L5 and L5-S1
- Inspect other joints for redness and swelling, note any skin rash or eyes redness
- As included in back examination mention that you would do a rectal exam
- Palpate the epigastrium for AAA which may cause low back pain
- P.E in this case was normal except for positive right straight leg raise

Sample questions you may be asked by the examiner;

1- What are your top 3 differentials?


Answer; Disk herniation, Lumbar muscle strain, Lumbar spinal stenosis
2- What investigations would you order?
Lumbar spine XR
Lumbar spine MRI
3- How would you treat this patient?
Answer; hot compressors, continue daily activities as tolerated, symptom guided
strengthening exercise, analgesics (NSAID like voltaren, Tylenol, opioids if needed.) ,
Cortisone injection if facet arthropathy

Rating scales points;

- Examinee introduced self and position


- Addressed patient with name
- Used proper non-verbal communication
- Organizational skills
- Looked for emergent conditions
- Examinee spoke clearly (accent didn’t get in the way)
- Listening and questioning skills
- Showed rapport with patient
- Attentive to patient physical comfort
- Medical knowledge adequate
- No misinformation was provided to patients
- No concerning ethical/legal issues
- The examinee is respectful of other health care members

Variations of the case and some tips;

168
1- Case; 67 M, known to have Prostate cancer presenting with sudden onset low back pain
and a limp

History; low back pain radiating to left lateral thigh and leg. Numbness is felt in the
same distribution. The patient had saddle anesthesia and normal bowel and bladder
function.

Physical examination; power of big toe extension and ankle plantar flexion 3/5.
Ankle jerk was absent. The patient had decreased light touch sensation over L5, S1.
When you mentioned you would do a rectal exam and examine sensations of the
perianal area the examiner says; anal sphincter tone decreased and perennial light
touch sensation is decreased.

Discussion; sudden onset Back pain combined with neurological deficit is an


emergency. Differential includes; Cauda equina, conus medullaris and epiconus
syndrome. Cauda equina is typically lower motor neuron while the other two are
upper motor neuron. Some causes include; tumors, abscess, large herniated disk
Prompt diagnosis with spinal MRI (preferred) or CT and surgical decompression
within 48 hours can save the patient’s function.

2- 28 year old male, complaining of low back pain and stiffness.

History; you find that the patient has 5 months history of moderate-severe non-
radiating low back pain, stiffness and fatigue. The pain is worse in the morning and
sometimes awakens him from sleep. No history of trauma. No constitutional
symptoms. No neurological or urinary symptoms. He had a bout of bloody diarrhea 4
weeks ago that lasted about one week. His back pain was more severe and he took a
lot of Advil. He didn’t see a doctor for the back pain or diarrhea because he doesn’t
like to take drugs but the pain is not going away. No skin, eye or other
rheumatologic symptoms. No other abdominal symptoms. He is a smoker for 8 years
1 pack per day. No constitutional symptoms, infections or unsafe sex practices. His
sister was diagnosed with Crohn’s disease a week ago

Physical exam; back movements are limited. Positive modified Schober test of 3 cm.
Right SI joint was tender

169
Discussion; As detailed in the history section, rule out neurological deficits and red
flag. This patient probably has Ankylosing Spondylitis (AS); An inflammatory
condition that mainly involves the spine and SI joint but can involve peripheral
joints. If you suspect AS make sure you cover extra articular manifestations mainly
inflammatory bowel disease (IBD), psoriasis, uveitis and dactylitis. Investigations
include HLA B27, C-reactive protein, ESR, spine XR, SI XR, and spine and SI MRI.
Encourage exercise, physical therapy and start the patient on full dose regular
NSAIDs. Refer to a rheumatologist for further evaluation and management.

3- 65 year old female complaining of low back pain radiating to both lower extremities

History; moderate low back pain and bilateral buttocks and thigh pain increased by
prolonged standing and walking and improved by sitting or bending forward. No
other symptoms. No cardiovascular risk factors apart from age. The patient had
laminectomy for a herniated disk at age 55.

Physical examination; unremarkable and normal lower limbs pulses.

Discussion; this patient is presenting with neurogenic claudication. Make sure you
rule out intermittent claudication seen in peripheral vascular disease. The presenting
complaint plus a history of back surgery make the diagnosis of lumbar spine stenosis
highly likely. Imaging studies preferably MRI aid in the diagnosis. Treatment includes
physical therapy, analgesics and decompression surgery with or without fusion for
severe cases.

4- Some tips for localization;


 L4-L5 (L5 radiculopathy) and L5-S1 (S1 radiculopathy) more common
 Foot drop, weak dorsiflexion, big toe extension and hip abduction L5
 Weak plantar and knee flexion S1
 Ankle jerk S1
 Patellar reflex L4
 Refer to chapter 3 page 13 for dermatomal distribution

170
Case 6; Knee pain

Edited by; Dr. James Yeung

Door sign

Mary Smith, 65 years old female, admitted for heart failure that was treated. Now
has knee pain and swelling. You are asked to see her for her knee pain

Vital signs are;


BP 110/70
PR 86
RR 14
Temperature 37⁰C

In the next 10 min conduct a focused and relevant history and conduct a focused
and relevant physical exam. As you do the P.E explain to the examiner what you are
doing and your findings. At 9 min the examiner may ask you a question or questions.

Note; The NAC exam may have a similar stem but the last 2 line will look like this; in the next 11
minutes conduct a focused and relevant history and conduct a focused and relevant physical
exam. As you do the P.E explain to the examiner what you are doing and your findings. At 8 min
the examiner may ask you a question or questions

Sample notes to write for yourself;


Mary Smith Gout
65 F Pseudo gout
Septic arthritis
C/C; Knee pain and swelling Rheumatoid arthritis, Inflammatory arthritis
Hx and P.E Trauma/ligamentous injury/fracture

Patient encounter

171
Your actions Suggested verbalizing Patient
response
Opening start I understand that heart Yes, I feel
failure brought you to much
hospital and is under better
control
Tell me about your knee My left
pain knee is
very
painful
since
yesterday
Onset Did it suddenly become Yes
painful
Swelling Is it swollen? Yes
Redness Is it red? Yes
Hotness Is it hot? Yes
Severity of pain How severe is the pain on It’s 9
and function a scale from 1-10, 10
being the worst pain ever?
Can you move your knee? No, it’s
very
painful
Exacerbating What makes the pain Just
factors worse beside movement? movemen
t
Relieving factors What makes the pain They gave
better me some
Percocet,
helped a
little
History of trauma Did you have trauma to No
your knee?
Swelling/pain of Have you ever had No
other joints swelling or pain of other
joints?
Fever/chills Have you had fever/chills? No
Gout precipitants

172
Diuretics/dehydra Were you given extra Yes, I’m
tion water pills the last week back to
for heart failure? my home
dose now
Meat intake but I was
Etoh given an
extra 2
pills every
day
Did you consume a lot of I eat a
meat? serving
every day
Do you drink alcohol No
Rheumatologic
Disease Do you feel stiff in the No
Morning stiffness morning that you need
some time to get going?
Do you get skin rash? No
Skin/hair changes Any loss of hair? No
Does your fingers color
Raynaud’s change in the cold? No
Do you have mouth
Mouth ulcers ulcers? No
Proximal muscle Do you find it difficult to
weakness stand up from a chair or No
comb your hair?
Additional
symptoms Do you have chest pain? No
CVS/Pulmonary Do you have shortness of Not any
breath? more
Can you lie flat? Do you
wake up at night short of Not any
breath? more
Do you have swelling
around your ankles? All gone
Do you cough? now
GI
No

173
Urinary Do you have abdominal
pain/diarrhea/constipatio No
n?
Does it burn to pee? No
Did the color of urine
change?
Constitutional
Weight loss Have you lost weight No
Night sweats recently?
(Fever covered Do you get drenching No
above) night sweats that you
need to change the
sheets?
Recent travel Have you traveled No
recently?
Gonorrhea Do you have vaginal No
infection discharge?
Have you ever had a No
sexually transmitted
infection?
Osteoporosis Have you ever had No
fractures?
Have you ever been No
diagnosed with
osteoporosis?
Do you take hormone I took it
replacement therapy? for 5
years
about 8
years ago
Past medical and
surgical history
Besides heart failure do I have
you have any other high
disease? blood
pressure
and high

174
Were you ever diagnosed cholester
with cancer? ol
Have you had any
surgeries? No
No
Medications and
allergy
What medications do you I take
take? aspirin,
water pill,
Lipitor
and a
couple
other
medicatio
ns, you
can check
with my
nurse
Sure, I will check with your because I
nurse, Are you allergic to believe
medications or drugs? they
made
some
changes
No
Family history
Does anyone in your My father
family have gout? had gout

Rheumatoid arthritis or No
other rheumatic disease?
Cancer? No
Social history
Do you smoke? No, I
never did
Do you drink alcohol? No

175
Do you use recreational No never
drugs?
Are you still working? No, I’m
retired, I
used to
be a clerk
With whom do you live? With my
How is his health? husband
He is
strong like
a horse
Do you exercise? I walk my
dog every
day
Do you have social Oh ya,
support? lots, I
have 3
children
and they
are all
wonderful
and my
husband
helps with
everythin
g
FIFE Thank you for sharing all Not at the
this information, do you moment,
have any questions for thank you
me? doctor

Physical examination:

- Examine the knee as detailed in chapter 3


- Inspect other joints for swelling, redness, deformities
- Inspect the skin for rash and mouth for ulcers
- Look at the hands and extensor surfaces for tophi seen in gout

176
Sample questions you may be asked by the examiner;

1- What are your top 3 differentials?


Answer; Gout, Pseudo gout, Septic arthritis
2- What investigations would you order?
Knee XR
Knee joint aspiration; send fluid for; gram stain and culture, crystals, cell count and
differential

Rating scales points;

- Examinee introduced self and position


- Addressed patient with name
- Used proper non-verbal communication
- Organizational skills
- Recognized and managed emergency effectively prioritizing actions
- Examinee spoke clearly (accent didn’t get in the way)
- Listening and questioning skills
- Showed rapport with patient
- Attentive to patient physical comfort
- Medical knowledge adequate
- No misinformation was provided to patients
- No concerning ethical/legal issues
- The examinee is respectful of other health care members

Variations of the case and some tips;

1- Case; 28 year old female presenting with knee pain and swelling

History; Sudden onset left knee pain while skiing; her left foot got planted, and she
felt pain and heard a pop as she was trying to move forward down the hill. Less than
an hour later her knee swelled. No other joints involved. No other symptoms. She is
finding it hard to move her knee because of the pain, Tylenol and Advil helped a
little. No previous injuries or surgeries.

Physical exam; Swollen left knee, positive effusion, flexion and extension limited by
pain, positive anterior drawer test.

177
Discussion; This patient presents with acute injury most likely of the Anterior
Cruciate ligament. The first thing to do is an XR and orthopedic consult. Pain control
is very important and the patient will most likely need opioids. Rest, cold
compressors are other things that may help till definitive diagnosis and treatment is
confirmed. Note that you need to explore the mechanism of injury in trauma cases,
particularly knee position and direction of force. Rule out previous injuries or
surgeries and do quick screens for other causes of knee pain especially infection.

2- Case; 20 year old male presenting with right knee pain and swelling

History; The patient describes 3 days history of knee pain, swelling and redness. No
history of trauma. He had chills and fever. No skin rash. He has a history of syphilis
treated a year ago. He had multiple sexual partners the last 3 months and doesn’t
use condoms. No previous or other joint pain and swelling, no rheumatologic or
other symptoms. No IV drug use.

Physical exam; Temperature 38⁰C. No skin rash. No other joints swelling. Hands and
face exam normal. Auscultation of the heart and lungs was normal. The right knee
was swollen, warm, and red with limitation of movement due to pain and positive
effusion.

Discussion; Septic arthritis is the top differential, make sure to rule out other
possibilities. Get an XR and aspirate the joint to confirm the diagnosis. Screen for
sexually transmitted infection and offer testing for HIV. Septic arthritis is treated
with empiric IV antibiotics till gram stain and cultures are back. Antibiotic choice
depends on the most probable organism and local sensitivities. In this case
gonococcus is the most likely cause; treat with ceftriaxone 1g IV daily. In IV drug
users Strep or Staph are more common in and Vancomycin is a more appropriate
initial treatment.

3- The basic principles for history apply for other joints. Patient age, presentation and risk
factors help sort out the differential. Make sure you master the physical examination of
all joints detailed in chapter 3.

178
Case 7; Diabetic Ketoacidosis (DKA)

Edited by; Dr. Sadik Salman

Door sign

Nichole Smith, an 18 years old female presenting to ER with abdominal pain


Vital signs;
BP 110/75
PR 100
RR 22
Temperature 38.2⁰C

Blood glucose 22
ABG done; PO₂ 88, PCO₂ 25 , HCO₃ 12 , PH 7.33
There is a nurse in the room

In the next 9 min manage the patient. Ask the nurse to do any orders you deem
necessary. As you examine the patient explain to the examiner what you are doing
and your findings. At 10min the examiner will ask you a question or questions

Note; In this case you will be communicating with the nurse and the patient.
Communication with the nurse is presented in italic. You need to make sure the
patient is stable, give emergency medications, take a focused and relevant history,
perform a focused and relevant P.E and order investigations. The P.E is included in
the case. Treat the nurse with respect, and give clear orders specifying the dose and
route of medications.
Even though it is an ER case, it is important that you introduce yourself to the nurse
and patient.

Sample notes to write for yourself;


Nichole Smith 18F DKA
Gastroenteritis
Appendicitis
Abd pain- Mx Ovarian abscess/torsion
Febrile, tachypnic tachycardic Inflammatory bowel disease
IBD

179
Patient encounter;

Your actions Suggested Patient or nurse


verbalizing response
ABC I will start with
ABC, The patient is
alert, breathing
spontaneously,
protecting her
airways, I’m
feeling the pulse it
is regular. The
patient is
tachypnic and
tachycardic.
Oxygen and IV The patient is
fluids maintaining her
oxygenation, can
we get 2 large
pore IVs, and give
her normal saline
0.9% 1 liter IV Given
bolus then
200cc/hour
Monitor Can you put the Done
patient on cardiac
monitor please
and check vital
signs every 15 min
Brief History
So Nichole, your Yes, for the last 6
blood sugar is years
high, Have you

180
been diagnosed
with diabetes? Yes
And it’s type 1,
right?
And what brought I had tummy pain
you to the and diarrhea,
hospital? couldn’t eat and
so didn’t take my
insulin
Details about the Is this the first Yes
abdominal pain time you have this
pain? It started
Where is it? here(patient
points at
umbilicus) then
moved her
How severe is it on (patient points at
a scale from one to right lower
10, 10 being the quadrant)
worst pain ever? It’s 9
What makes it
worse? I don’t know
What makes it
better? Advil helped a
You said you also little
had diarrhea, how
many bowel
movements? 3 watery
Any blood or
mucus in the No
stool?
Have you had any I felt a bit
other symptoms nauseated
like vomiting,
nausea, burning
urination, change
in urine or stool

181
color or flue like No
symptoms?
Did you eat in a
restaurant lately? No
Have you been in
contact with
someone sick? No
DM history What insulin type I take 18 units
and dose are you NPH in the
on? morning and 18
units in the
evening and I take
Humalog before I
eat usually 5-8
Which insulin dose
did you drop I didn’t take any
today? today because I
didn’t eat
Do you check your I hate it so I don’t
blood sugar levels? always do it, but I
usually check
once or twice a
What are the day
values? 8-12

Do you know It was 7.2 a


what’s your month ago
HBA1c?
Have you had Yes twice; when I
diabetic was diagnosed,
ketoacidosis and 2 years ago?
before?
Were you
admitted to the
ICU? No
What was it about I had the flu and
2 years ago? couldn’t eat or
take insulin

182
Did you get your Yes, a year ago,
eyes checked? and it was fine, I
take very good
care of myself I
Good for you, I’m don’t want to die
glad to hear that. from diabetes

Have you had any


other No
complications
from diabetes?
Investigations Can I get an ECG, Sure
ABG Ok, will draw
And the following blood now, results
blood work; CBCD, will take 15 min
Creatinine, urea,
Lytes, PO₄, and
urine analysis and
culture
Management Can you please Will do
give 10 units
regular insulin IV,
then 0.5 U/hour,
and do a chem
strip after an hour
Relevant review Have you had No
of systems cough?
Chest pain? No
Skin rash? No
Vaginal No
discharge/itching?
Fever/chills at I’m not sure
home?

When was your A week ago


last menstrual
period?

183
Other relevant Do you have any No
history other disease?
Have you ever had
surgeries? No
Do you take any Tylenol or Advil
other occasionally
medications?
Do you have No
allergies to food or
medications? No
Do you smoke? No
Do you drink
alcohol? No
Do you use
recreational
drugs?

Are you sexually Yes, with my


active? boyfriend
Do use We use condoms
contraception?
Have you ever had No
a sexually
transmitted
infection?
Order Can I get the Will add B-hcg to
investigations following the blood I drew
investigations as and will order
well; CXR, Do you want
B-hcg to look at the
CXR ECG?
The nurse hands
you an ECG
Thank you, Normal
rate, rhythm, and
axis. P waves and P
wave intervals are
normal. QRS, T

184
wave and QT
normal, No U
waves
Physical The patient is
examination tachypnic,
tachycardic, and
febrile. The hands
are normal, no skin
or nails changes.
No pallor, no skin
rash, no fetor
hepaticus, no
jaundice. Can you
open your mouth
please, no throat
redness, no
cyanosis. The
mucous
membranes are
mildly dry. No skin
turgor. I will test
orthostatic vitals
at the end.
I’m going to listen
to your heart and
lungs; normal S1,
S2, no S3, S4, no
murmurs, no rubs.
Breath sounds are
symmetrical
bilaterally, no
wheezes, no
crackles. No rubs.
Next I will examine
the abdomen. I’m
going to examine
your tummy, let
me know if it

185
hurts; staring with
inspection, no
masses, no
distension, the
abdomen is
symmetrical and
moving with
respiration. No
caput medusa, no
Grey turners or
Cullins sign. No
stria.
I’m now listening
to bowel sounds,
they are normal.
Do you have pain
right now? Yes in
here (right iliac
fossa)
Next I will do
superficial
palpation starting
from the left iliac
fossa, there is
tenderness in the
right lower
quadrant, then
deep palpation, no
masses, there is
tenderness in the
right lower
quadrant. I will
test now for
specific signs of
appendicitis,
pressing on
McBurney’s point,
there is pinpoint

186
tenderness. Does
it hurt when I let
go? Yes. And
rebound
tenderness. I
noticed that
rouvsings was
negative. I’m going
to move your leg,
relax it for me and
let me know if you
have pain; flexion,
then internal
rotation; Psoas
and Obturator
signs are negative.
Next I will palpate
for hepatomegaly
in the mid
clavicular line, can
you breathe in and
out for me please?
The liver is not
palpable. No I’m
palpating for the
spleen using the
same technique.
The gallbladder is
not palpable. Can
you take a deep
breath for me
please? Murphy’s
sign is negative.
Can you set up for
me? I’m checking
for renal angle
tenderness and it’s
negative.

187
Next I will examine
the lower limbs;
I’m going to look
at your legs. No
swelling, no
redness, no
deformity, no
dilated or tortuous
veins, no cyanosis,
no ulcers, no skin
or nail changes.
I’m going to press
on your calves, let The examiner says
me know if it vaginal and rectal
hurts. No calves’ examinations
tenderness. Finally normal.
I would like to do a
rectal and vaginal
exams
Investigations and Please give
management another bolus
normal saline 0.9%
IV over 1 hour
then run normal
saline at 200
cc/hour.
Give the patient
Percocet 1 tab
now with sips of
water then keep
NPO
Can you order US
of the abdomen to
rule out
appendicular or
ovarian pathology.
I will continue to
manage DKA, but

188
will consult
surgery in the
mean time for
possible
appendicitis.
Social support Thank you Nichole Thank you, Can
for all the you please
information, You explain to my
have a DKA. Your boyfriend what is
abdominal pain going on. He is
could be due to very worried.
DKA, or inflamed
appendix. I will ask
the surgeons to
come and take a
look. Would you
like me to talk to
one of your family
or your boyfriend?
No problem

Sample questions you may be asked by the examiner

1- What is your preliminary diagnosis?


Answer; DKA secondary to appendicitis, tubo-ovarian abscess/rupture or ovarian
torsion.

Rating scales points

- Examinee introduced self and position


- Addressed patient with name
- Used proper non-verbal communication
- Organizational skills
- Recognized and managed emergency effectively prioritizing actions
- Examinee spoke clearly (accent didn’t get in the way)
- Listening and questioning skills
- Showed rapport with patient
- Attentive to patient physical comfort

189
- Medical knowledge adequate
- No misinformation was provided to patients
- No concerning ethical/legal issues
- The examinee is respectful of other health care members

Variations of the case and some tips;

1- Case; 18 year old female presenting with abdominal pain, blood sugar of 25 and anion
gap metabolic acidosis

History; Type 1 DM diagnosed at age 10. Well controlled till the last year when she
had 4 episodes of DKA. Patient talks madly about her boyfriend and repeatedly
describes him as a jerk. When you ask you find that they have been together for 1
year but the relationship is not smooth. When troubles arise she threatens with
either not taking insulin or taking too much. All her previous DKAs were due to
missed insulin. She also had 5 iatrogenic hypoglycemic attacks. No ICU admission.
The abdominal pain is non-specific and she doesn’t have other symptoms. No
alcohol or drug abuse. No depression or suicidal intent, she just got mad with her
boyfriend and is seeking attention.

Physical exam; Dry mucous membranes. Otherwise normal

Discussion; This is an emergency case, so you need to focus on managing DKA. And
although the patient intentionally didn’t take insulin you still need to rule out other
possible precipitants, especially intoxication. The patient has some social stressors
and is showing features of border line personality disorder so it is important to
consult psychiatry once she is medically cleared. Ask the patient if you could speak
to her parents for collateral history.

2- In addition to stabilizing patients and initiating management in DKA patients, it is


important that you dig deep into the precipitant; 6Is; insulin missed, iatrogenic (like
glucocorticoids), infection, inflammation, ischemia and intoxication. Once the cause is
recognized, it needs to be treated.

3- Note that DKA patients can be tripped off by precipitants even if they continue to take
insulin

190
Case 8; DM history and counseling
Edited by; Dr. Sadik Salman

Door sign

Jonathan Adams, a 55 years old male who comes to your clinic for diabetes follow up.

Vital signs;

BP 120/80

PR 80

RR 13

Temperature 36.5

In the next 10 min counsel the patient. At 9 min the examiner may ask you a question or questions.

Note; The NAC exam may have a similar stem but the last 2 line will look like this; in the next 11 minutes
counsel the patient. At 8 min the examiner may ask you a question or questions

Sample notes to write for yourself

Jonathan Adams DM type, onset

55M medications

Diet and exercise

History and counsel DM monitoring and control

Complications

Feet

CV risk factors

Patient encounter

Your actions Suggested verbalizing Patient response

191
Opening start How can I help you today I’m here to check on my
diabetes

DM history

Type What type of diabetes do u Type 2


have?

When was it diagnosed?


Diagnosis 5 years ago
What medications are you on?
Medications Glucophage I take 850mg twice
a day and Gliclazide 80mg daily

For how long you’ve been on 3 years


them?

Control and monitoring Do you check your sugar levels? Yes I do

How often?
2 times every day
What values do you get? Usually 6-13

Do you have your log book? Sorry I forgot it at home

No worries, Do you know what


It was over 6 months ago, I
was your last hemoglobin A1C
think it was 7.5
and when?

Did you get your eyes checked? 5 years ago, they were fine

Have you had problems with No never


feet ulcers?
No
Have you ever had a heart
attack?
Complications
Sometimes I feel the tips of my
Do you feel numbness or
fingers are numb
tingling in your hands and feet?
No
Did anyone tell you before you
have protein in your urine?

Do you get low blood sugars? Once in a while

192
Like how often? Once a month usually if I don’t
eat and walk the dog

What is the lowest number you


got? 3

What do you do when you get a I take some sugars then I eat
low?

What symptoms do u get when


its low? I feel dizzy and become shaky

No
Did you ever need help because
of low sugars?

Have you ever had a surprise; No


you checked your sugar and it
was low but you feel fine?

Have you been to diabetes No


Education
education before?
My family doctor used to, but
Follow up
Who follows your diabetes? he retired 6 months ago
Diet and exercise

Do you follow a diabetes diet I try, but I cheat sometimes


low in sugar?

Do you exercise?
I walk my dog every day

Other cardiovascular risk factors

Hypertension Do you have high blood Yes


pressure?

Do you take a medication for it? Coversyl 4mg


What is your blood pressure
usually? 129/70

Do you have high cholesterol? Not that I know of


Dyslipidemia

193
Smoking Do you smoke? No
My father died of a heart attack
Family history of MI Did anyone in your family have in his 70s
a heart attack?

Quick review of systems Do you get chest pain? No

Do you get cough? No

Do you get shortness of breath? No

Do you have abdominal pain? No

Vomiting? Diarrhea or No
constipation?

Do you have problems with


No
urine, like pain or difficulty
when you pass urine, change in
urine color?

Have lost or gained weight


recently?

I gained 2 lb

Do you get fevers? No

Other past medical and surgical Do you have other diseases? No


history
Have you had any surgeries? No

Other medications and allergy Do you take any other No


medications? Or over the
counter medications? Non-
steroidal pain killer like Advil

Do you have allergies to


medication or food?
No

Relevant social history Do you drink alcohol? Occasionally

Do you use recreational drugs? No, never

194
What do you do for living? I own a travel agency

With whom do you live? With my wife and daughter

Any stress at work or home? No, things are fine

Well, I have to watch what I eat,


and try to stay active. It’s not
How is diabetes affecting your limiting me, but I have to do my
life and function? homework and keep an eye on
my sugar

Counseling

What does the patient know What do you know about I know I should be very careful
diabetes? with what I eat and take my
medication or I may lose my
vision and get a heart attack

Patient expectations Do you have specific questions? Well, I trust that you will look
after my diabetes now that my
family doctor retired

Sure I will, thank you for


trusting me

Education and counseling

Background info As you said diabetes damages


body organs silently. In type 2
diabetes the cells become
resistant to insulin- a hormone
secreted by the pancreas to
regulate blood sugar levels, so
blood sugar levels become high.
If it stays high for too long
damage starts

Intervention and its importance That’s why we try to lower the


blood sugar with diet , exercise
and medications, and you are
on 2 good ones

195
Monitoring In addition to you checking your
blood sugar at home, we do
check the HbA1C to see how the
sugar is doing over the last 3
months, we can lower it even
further in your case and I would
like to see it around 6.5

And how would you do that?

I will increase Gliclazide dose I see


and if needed add a new
medication

I need you to check your blood


sugar 4 times a day; before you
eat in the morning, before lunch
and dinner and before bed
time. Would you be able to do
that? Well, I can but do I need to do it
for ever?

It is better to keep checking it 2


times a day, but I need you to
do it more often for the next 3
months because we are trying
to achieve a better control, and
because we are increasing
Gliclazide dose which may cause
your blood sugar to drop. Are
we on the same page? Yes

Medication side effect Do you get side effects from the No, I feel good
medications like nausea,
stomach upset, and dizziness?

You mentioned you get low


blood sugar occasionally, I need Sure, I will
you to watch that, you may get
dizzy, shaky, flushing, sweating
and have a fast heart rate, take

196
a sugar pill and check your
blood sugar right away

Complications of diabetes

So let’s talk about the damage Yes please I’m listening


diabetes can cause and what we
will do about it

Retinopathy It can cause blindness but if we I would appreciate that


interfere early we can prevent
it, it is the time for an eye exam
for you

Nephropathy It can also damage the kidneys


if not well controlled, so we
need to check the protein in
your urine and your kidney
function. You are taking the
right blood pressure pill that
can help decrease the damage

It is better to avoid drugs that


I see, glad I’m on the right
can adversely affect the kidneys
medication
like Advil and voltaren
I will ask you before I take any
over the counter medication
Yes, please do. We need to
work together on this

Neuropathy The nerves can as well be That would be great


damaged if blood sugar is too
high for too long, I would like to
examine your nerves if that’s ok

Macrovascular Diabetes, combined with other


conditions like high blood
pressure and cholesterol, and
smoking can increase the risk of

197
heart attacks, but I see that
your blood pressure is
controlled so keep the good
work, and that you don’t
smoke, this will significantly
decrease your risk, I need to
check your cholesterol level, it is
a blood test that you need to
fast 12 hours for, is that ok For sure, I would love to get
tested

Hypoglycemia (discussed above)

Hyper osmolar coma If the sugar was out of control Ok


and went too high, you may get
dizzy or even lose
consciousness, and in this case
you need to go to the hospital

Feet ulcers and poor wound It is a good idea to develop the Oh, I will check them every day!
healing habit of looking at your feet
daily, because you may lose
your sensations, and miss some
wounds. Some people lost their
limbs because of diabetes

Summary So we talked about diabetes


and its complications and how
to prevent them. We agreed to
increase Gliclazide and do some
tests and repeat the eye exam,
do you have any questions or
concerns?
How about my diet?

Diet and exercise That’s an excellent point, you


need to avoid sugar and
carbohydrates, and keep track
of your weight. I can refer you
to the dietician for full details
about diet if you like

Yes, I need to see a dietician

198
Sure, will do. The other thing I
wanted to talk to you about is
exercise; You are doing well by
walking your dog every day. You
do need to exercise, it helps
control your diabetes, but it
needs to be balanced against
I see
your diet so that your blood
sugar doesn’t drop

Do you have any other Would I ever need insulin?


questions or concerns

Some patients with type 2


diabetes need insulin when pills
are not enough to control their
blood sugar. We try to avoid
that by encouraging patients to
eat healthy, exercise and take
their medications. You will not
need insulin any time soon, and
may never need it if you are
careful enough. Any other
concerns?

No, thank you very much

Give brochure I will give you a brochure about Thank you, I appreciate it
diabetes, I want you to know
Assurance and availability that I’m here to help you, and
that you can contact my clinic
for an appointment at any time.

Sample questions you may be asked by the examiner

1- How would you diagnose type 2 DM?

Answer; one of;

 Fasting blood sugar ≥ 7

199
 Random blood sugar ≥ 11.1

 2 hours oral glucose tolerance test with 75 g ≥ 11.1

 HbA1c ≥ 6.5

2- What is the blood pressure target in DM?

Answer; ≤ 130/80

Rating scales points;

- Examinee introduced self and position


- Addressed patient with name
- Used proper non-verbal communication
- Organizational skills
- Examinee addressed patient concerns
- Educated patient in an easy to understand language
- Examinee spoke clearly (accent didn’t get in the way)
- Listening and questioning skills
- Showed rapport with patient
- Attentive to patient physical comfort
- Medical knowledge adequate
- No misinformation was provided to patients
- No concerning ethical/legal issues
- The examinee is respectful of other health care members

Some tips

1- I strongly suggest you familiarize yourself with the Canadian diabetes guidelines;

http://guidelines.diabetes.ca/

Make sure you master the following;

 Screening and diagnosis of diabetes

 Target blood sugar and HbA1c

 Oral drugs and their side effects

200
 Insulin types, durations of action, indications, initiation of therapy and complications

 Complications of diabetes

201
Case 9; Diarrhea
Edited by; Dr. Erin Toor

Door sign

Lorraine Land is a 28 year old female who comes to your clinic complaining of diarrhea.

Vital signs

BP 100/70

PR 90

RR20

Temperature 37⁰C

In the next 10 minutes, conduct a focused and relevant history and physical examination (P.E). As you do
the P.E, explain to the examiner what you are doing and your findings. At 9 min the examiner may ask
you one or more questions.

Note: The NAC exam may have a similar stem but the last 2 lines will look like this: In the next 11 minutes
conduct a focused and relevant history and physical exam. As you do the P.E explain to the examiner
what you are doing and your findings. At 8 minutes, the examiner may ask you one or more questions.

Sample Notes to write for yourself

Lorraine Land GE

28 F

IBD

Hx and P.E IBS

Patient encounter

Your actions Suggested verbalizing Patient response

Opening start How can I help you today? I have diarrhea

202
Tell me more about it It is been going on for a month
now

History of presenting illness

Diarrhea

Onset Did it start gradually or It was gradual, with the stool


suddenly? getting more frequent over
time
Duration (mentioned) I was getting 3-5, but it’s been
How many bowel movements a
Frequency 10 for the last week
day?
Yes, I’m not sleeping well
Do you wake up at night to because of it
Nocturnal diarrhea poop?
Watery
Is the stool watery or soft?
Brown
Stool consistency What is the stool color?
There is mucous sometimes,
Stool color Is there blood or mucous in the but no blood
stool?
Presence of blood or mucous
Is the stool oily? Does it stick to
the toilet bowl and is difficult to No
flush?
Fatty stool
Have you noticed that a special
Relation to diet No, I stopped some stuff, but it
diet triggers diarrhea; like
didn’t make a difference
bread, oats, or dairy?

Do you feel the urge to go to


the bathroom, and then you No
Tenesmus can’t pass stool?

Have you had any accidents?


No
Have you had similar episodes
of diarrhea before? No
Incontinence
Does the diarrhea resolve with
Previous similar episodes
fasting?
No

203
Risk factors

Recent travel Have you travelled recently? No,

Sick contacts Have you been in contact with No


people with similar symptoms?
if yes-ask details: where,
duration of travel, consumption
of local water, undercooked
food etc
Have you eaten spoiled food or
No
at restaurants?
Eating spoiled food or at
restaurants
Do you have a disease or take No, I’ve always been proud that
Immunosuppression I don’t get sick because my
medications that make your
immunity is strong
immune system weak?

Dehydration

How many cups of water do you 10 or more with this


drink every day?
Water intake
Do you feel thirsty?
Thirst Sometimes
Do you feel dizzy? Did you ever
Dizziness No
faint?

Did you notice a decrease in the


Urine output amount of urine? Yes, I’m peeing only 3 times
every day and its dark yellow

Constitutional symptoms

Weight loss Have you lost weight recently? 2 lb over the last month

Night sweats

204
Do you get drenching night No
sweats that you need to change
Fever the sheets?

Have you had fever/chills? No

Associated symptoms

Abdominal pain Do you have abdominal pain? Occasionally around my belly


button

Nausea/Vomiting Do you have nausea or


vomiting? No

Have your eyes and skin turned No


Jaundice yellow?

Did you have periods of


No
Alternating constipation and constipation followed by
diarrhea diarrhea?

How about mouth sores? I get them once in a while

Mouth ulcers Have you had pain or discharge No


from the anus?
Perianal disease
Did you have eyes pain or
redness? No

Eye pain, redness Joints pain or swelling? No

Skin rash? No

Joints pain or swelling Do you feel more tired than Yes, I’m not as energetic as
usual? before
Skin rash
Are your periods regular? Yes
Fatigue
When was your last one? A month ago

Any chances you might be I don’t think so, we use


Menstrual irregularity
pregnant? condoms

205
Past Medical and Surgical
history

Were you ever hospitalized or No


diagnosed with another
disease?
No
Have you had surgeries?

Medications and allergies

Do you take any medications? No,

Do you take non-Steroidal pain I only take Tylenol occasionally


killers like Advil, Voltaren?

Have you taken antibiotics


recently? No

No
Do you have allergies to food or
medications?

Family history

Does anyone in your family My aunt has Crohn’s disease


have Crohn’s disease, ulcerative
colitis or cancer?

Social history

Smoking Do you smoke? No

Alcohol Do you drink alcohol? No

Illicit drugs Do you use recreational drugs? No

Occupation What do you do for living? I’m a teacher

Living conditions With whom do you live? With my husband

Stress Any stress at home or work? No

Effect on function Is diarrhea affecting your work I’m more tired than usual, and
or relationship with your its embarrassing to leave the
husband? class to go to the bathroom
many times

206
FIFE Thank you for sharing all this You asked everything, thanks
information, would you like to for being thorough
add anything?

Do you have any questions for


I want to know what’s wrong
me?

Multiple things can cause


diarrhea. I need to examine you
and run some tests and
hopefully we will figure it out

Physical examination;

- General examination: Examine the hands, face. Look for pallor, jaundice, skin rash, mouth
ulcers, joints swelling or redness and dry mucous membranes. Feel the pulse and notice if it is
thready. Assess for skin turgor

- Check postural vital signs

- Auscultate the heart and lungs

- Examine the Abdomen

- Mention that you would do a rectal exam

Sample questions you may be asked by the examiner;

1- What is your differential?

Answer: Chronic diarrhea differential: Crohn’s disease, Ulcerative Colitis, Celiac disease, Lactose
intolerance, Infectious diarrhea (particularly parasitic, CMV), Hyperthyroidism, gut malignancy, Irritable
bowel syndrome, medications (laxative abuse, PPI’s), pancreatic insufficiency, Hormonal (VIPoma,
carcinoid)

2- What initial investigations would you order?

Answer: CBCD, electrolytes (Na, K, Cl, Mg, PO4), urea, creatinine, ESR, CRP, Albumin, PT/INR, ALT, Stool
WBC, Stool for ova and Parasite, Stool culture and sensitivity, Abdominal X-Ray, will consider referral for
endoscopy/colonoscopy

3- What is your initial management?

Answer: Manage dehydration, correct electrolyte disturbances, diagnose and treat the underlying cause
of diarrhea

207
Rating scales points;

- Examinee introduced self and position


- Addressed patient with name
- Used proper non-verbal communication
- Organizational skills
- Examinee spoke clearly (accent didn’t get in the way)
- Listening and questioning skills
- Showed rapport with patient
- Attentive to patient’s physical comfort
- Medical knowledge adequate
- No misinformation was provided to patients
- No concerning ethical/legal issues
- The examinee is respectful of other health care members

Variations of the case and some tips;

1- Case: 10 year old girl presenting with diarrhea and vomiting, obtain history from the mother.

History: Nausea and vomiting started 3 hours after eating fried rice in a local Chinese restaurant.
Associated with abdominal pain, 10 hours later she developed nausea and vomiting followed by watery
diarrhea. History is suggestive of moderate-severe dehydration. No fever or other symptoms. The
mother ate the same food and had similar but less severe symptoms. No previous episodes, no other
symptoms

Physical exam: Although not asked to perform a physical exam, indicate to the mother that you need to
examine her daughter, do some blood tests and give her intravenous fluids

Discussion: Food poisoning/infectious gastroenteritis is usually diagnosed by history. The suspected


pathogen in this case is Bacillus Cereus. Supportive management that includes fluid resuscitation and
correction of electrolytes disturbances. Symptoms last 24-48 hours. If the duration is prolonged consider
other diagnoses.

2- Case: 42 year old male presenting with diarrhea

208
History: A day after returning from a trip to Mexico, he had abdominal cramps, nausea, vomiting and
diarrhea. He stayed in a 5 star resort over there. His wife, who accompanied him to the trip, had similar
symptoms. His urine output has decreased and he feels thirsty. His father died of colon cancer at age 50.
No constitutional symptoms, no similar episodes previously.

Physical examination: Normal vitals and postural vitals, dry mucus membranes. Normal chest and
abdominal physical exams.

Discussion: Diarrhea in a returning traveler could be due to traveler’s diarrhea, which might be caused
by viruses, bacteria, or parasites. Keep the differential wide. Rehydrate the patient and correct
electrolytes abnormalities. Do stool testing, and treat as needed. If diarrhea persists then investigate
further. Note that this patient is due for colon cancer screening, and once the diarrhea had subsided he
should be screened by colonoscopy. This could as well be his first presentation of malignancy.

3- 35 year old female, is being treated for Pneumonia with Ceftriaxone and Azithromycin. Now has
diarrhea

History: Diagnosed with Pneumonia and started treatment 10 days ago. She has symptoms of watery
diarrhea, cramps and fever. No Previous episodes. Her respiratory symptoms are improving. No other
symptoms. She didn’t eat spoiled food or at restaurants recently.

Physical exam: Temperature 37.9⁰C . Other vital signs normal, no postural drop. She had left lower lobe
crackles, abdominal exam was normal. No blood on rectal exam

Discussion: In addition to the regular testing, it is important to rule out Clostridium Difficile (C.diff) colitis
in cases of recent or current antibiotic use. Test the stool for C.diff toxin. Keep in mind possible
complications like toxic mega colon, electrolyte disturbances, volume depletion and bowel perforation.
Manage with fluid resuscitation, management of electrolyte disturbances if present and treat with
Flagyl. One could continue the previous antibiotic course until finished.

4- Diarrhea has a very wide differential that can be narrowed down by history and physical. Make
sure to develop your own approach.

209
Case 10; Smoking counseling
Edited by; Dr. Sadik Salman

Door sign

David McLean, is a 33 years old male who comes in to your clinic because he has questions about
cigarette smoking.

Vital signs;

BP 120/80

PR 80

RR 12

Temperature 36.8⁰ C

In the next 10 min counsel him about smoking. At 9 min the examiner may ask you a question or
questions.

Note; the NAC may have a similar stem but the last 2 lines may look like this; In the next 11 minutes
counsel the patient about smoking. At 8 min the examiner may ask you a question or questions.

Sample notes to write for yourself

David McLean General Hx

33 M Hx of smoking

Stage of change

Hx and counseling How much does he know

Risks of smoking, benefits of quitting

What to do to quit, meds details

Patient encounter

210
Your actions Suggested verbalizing Patient response

Opening start How can I help you today? I want to talk to you about
smoking

And what exactly do you want I’m thinking to quit and don’t
to discuss? know where to start

Good for you, I can definitely


help you with this

Reflective questioning and What prompted you to think My friend is 40 years old only,
listening about quitting? and he died with lung cancer.

Sorry about that, it must have It is, I decided I want to live for
been hard for you my wife and daughter. I’m
determined I will not let
smoking get to me

I understand, I admire your


determination, that’s a very
strong motivation and I will do
my best to help you.

Do you mind if I ask you few


questions about your health Not at all
first?

Current symptoms

Respiratory and cardiac Do you cough? No

Do you have chest pain? No

Do you cough up blood? No never

Do you have shortness of No


breath?

211
Do you feel dizzy? No

Do you have diarrhea or No


constipation?
GI

No
Abdominal pain?

Heart burn? No

Blood in the stool or change in No


stool color?

Have you lost weight recently? I gained 2 lb

Constitutional Do you get fevers? No

Do you get drenching night


No
sweats that you have to change
the sheets?

Do you feel fatigued? No, I’m very active

Do you feel depressed? I’m sad I lost my friend but I’m


Other symptoms not depressed

No
Have you lost interest in
activities you used to enjoy?

Do you have headache? No

Weakness? No

Problems with urine? No

Skin rash? No

Comorbidities and other cardiac Do you have other diseases like None of that
and respiratory risk factors diabetes or high cholesterol, or
asthma?

212
Were you exposed to asbestos
or other materials and
Never
chemicals?

Family history Did anyone in your family have Not that I know of
cancer?

Heart or lung disease?


My father died of a heart attack
when he was 70

Sorry to hear that

Medications and allergy Do you take any medications, I take Tylenol occasionally
over the counter medications or
herbs?

Do you have allergy to


medications or food? Not that I know of

Other substance use

Alcohol Do you drink alcohol? I drink few beers every day

How much is a few? Two.

Have you tried to cut down your No


drinking?

Do you feel guilty about


No
drinking?

Do you get annoyed by people


criticizing your drinking? No
Do you need alcohol the first
thing in the morning as an eye
Recreational drugs opener? No

Do you use recreational drugs? No

Social history

213
Occupation What do you do for living? I’m a lawyer

Living conditions With whom do you live? My wife and daughter

Current stressors Any stress at home or work? No

Exercise Do you exercise? I go to the gym 3 times a week

Good for you

Diet Do you eat healthy? Well, not really I eat a lot of


junk food

History of Smoking

Number of packs per day How much do you smoke? A pack a day

Duration For how long? For the last 15 years

Location and situation where Where and when do you smoke I take multiple short breaks at
smoking most most and who accompanies you work to smoke, some of my
when you smoke? coworkers are smokers and we
Who he smoke with enjoy a cigarette together. I
avoid smoking at home because
my wife doesn’t like it

Have you ever had a breathing


Previous investigations test or chest X-ray
No

Previous attempts to quit Have you tried to quit before? I tried once

What did you do and for how A girlfriend told me I’m


long did you quit? addicted and can’t stay away
from cigarettes so I stopped
smoking for 4 days, I didn’t
want to quit back then, just
wanted to prove a point

214
How did you feel those 4 days? Horrible! I just wanted a
cigarette, I was cranky and
couldn’t concentrate

Additional questions to consider I see, did you use anything to


when appropriate; help you stay away from it like
nicotine gums or patches? No
What symptoms did you get and
how did you deal with them?

Why did you bounce back?

Assess stage of change How ready do you feel you are 100%
to quit this time?

So, you feel ready to take


actions to stop smoking I will do whatever it takes

Good for you, You are giving


your body the best gift.

Can I share some of the benefits


Sure
of quitting?

Smoking increases the risk of


lung cancer, heart disease and
chronic obstructive pulmonary
disease. Once you quit Oxygen
level will go back to normal in
less than 10 hours, your risk of a
heart attack will go down, your
lung function will improve and
your risk of lung cancer will
drop; In 10 years your risk of
lung cancer is cut in half!

There are even more benefits;


People will no longer be
affected by your smoking; you
will set a good example to your
daughter and save lots of
money. Sometimes writing
down all the positive things that

215
you are getting may help, how
does that sound?
Great, I’m even more
determined than before

Time of change When do you think you want to I have a hectic work schedule
stop? this week, I will start next
Monday. I would like to hear if
you have any suggestions that
may make quitting easier

Taking actions

Family and friends support So we have agreed on a date, I


would suggest you speak to
Support groups your wife, friends and
coworkers about your decision,
their support will help you along
Avoid situations that may the way, explain to your
encourage smoking coworkers that you will not be
smoking with them. And find
something to do during your
break like going to a walk, how
does that sound? Sounds good, that’s a good
idea, I will join the other group
of coworkers, they eat and go
for short walks during their
That’s excellent.
breaks
It might also be helpful to avoid
situations that might trigger the
urge to smoke.

You will have some symptoms


when you quit; cigarettes
contain nicotine, and your body
is addicted to it, so you may feel
dizzy, shaky. You may get
irritable and feel more tired
than usual. Don’t push yourself,
nap if you need to and avoid

216
Nicotine withdrawal and how to extra work during this period.
cope Your sleep may get disturbed,
avoid caffeinated tea and
coffee. You may cough more
and feel tight the first few
weeks as your lungs try to clear
up tar and other toxins, drink
lots of water and take deep
breaths. If you get chest pain,
cough up blood, have a fever or
your cough is getting worse That’s very helpful, thank you
rather than better then you
should seek medical help.

You may feel hungry, and a lot


of people gain weight when I’m not worried about my
they quit. Try to eat healthy weight
food, and snacks

Do you have any questions?


Are there drugs to help these
symptom?

Details about pharmacotherapy That’s an excellent question

A lot of these symptoms are


due to the fact that your body
craves nicotine. There are
different forms of nicotine
replacement therapy like; gums,
patches, lozenges, inhalers and
nasal spray. I usually prescribe
the patch combined with the
gum to use when urge arises.
Side effects are minimal
including skin irritation,
insomnia and bad dreams; both
can be avoided by removing the
patch at night and rotating the
patch site.

217
I will give you handouts to read
more. Any questions?

I think I will take nicotine. Are


There are 2 drugs that are there other drugs?
effective in helping smokers
quit;

Bupropion (Zyban) and


Varenicline (Champix). Both are
pills

Zyban may cause headache,


insomnia, agitation, and seizure

Champix is probably more


effective than Zyban, it may
cause nausea, skin rash, visual
disturbance and impairment of
motor abilities with increased
risk of accidents and falls. It also
increases the risk of heart
attacks in already high risk
patients and you are not one of
them.

Both these drugs can increase


the risk of suicide.

I will give you some handouts to Thank you for providing all this
read and think about your information, I will read the
choices, if you decide to go with handouts and see if I want to
Champix then we need to get use drugs
started one week before you
quit. Questions?

Dealing with cravings Sure, let me know what you


think or if you have questions.
Now you will crave cigarettes,
strong craving lasts usually for
few minutes then goes away.
Try to engage yourself with
something, like a walk or a drink

218
of water. You can chew nicotine
gums as well

If you usually smoke with your


beer this may be challenging.
Be aware of this.

Encouraging statements I congratulate you for your


decision, you are on the right
track for optimal health

Support systems available

FU at clinic- visits, phones, texts, I or one of our physicians will


emails, etc… always be here for you. We can
Sounds good
arrange for follow up visits

Sure, we can do that. My clinic


can also send you encouraging Sure, emails would be great
emails and text messages if you
like

I will give you the hotline I appreciate that , thank you


number for smoking cessation
as well as contact information
of support groups

Falling off the wagon I also want to mention that you I’m very determined, I want to
may fall off the wagon, and stop
that’s ok, the key is to try again

Patient’s feelings, questions, Do you have any questions or Not at the moment
concerns and expectations concerns?

I admire your determination, Sure I will, thank you


and look forward to hear you

219
success story, please contact
me if you have any questions

P.E is not required in this case

Sample questions you may be asked by the examiner

1- If the patient decides to go with Zyban or Champix, how would you manage the increased
suicide risk?

Answer; I will explain to the patient that this is a possible side effect and educate him/her about
depression symptoms. I will ask the patient to stop the drug and call me immediately if he/she had
thoughts of suicide or depression symptoms.

Rating scales points;

- Examinee introduced self and position


- Addressed patient with name
- Used proper non-verbal communication
- Good organizational skills
- Examinee spoke clearly (accent didn’t get in the way)
- Listening and questioning skills
- Showed rapport with patient
- Explored patient concerns
- Attentive to patient physical comfort
- Medical knowledge adequate
- No misinformation was provided to patients
- No concerning ethical/legal issues
- The examinee is respectful of other health care members

Variations of the case and some tips;

1- It is essential to give a personalized advice in cases of counseling. Make sure you take relevant
history focusing on social aspects, and substance use. Explore the patient’s needs and concerns
and help them regardless of the stage of change they are at. If you encounter a contemplating
or pre contemplating patient, educate him/ her but don’t push for a change. Respecting the
stage your patient is at will build trust and make it easier for your patient to contact you should
he/ she needs.

2- I strongly encourage you to familiarize yourself with the 5 As of 3-5 min tobacco intervention;

220
http://www.sdta.ca/mrws/filedriver/DentistTobaccoInterventionAlgorithmSept06.pdf

221
Case 11; Preeclampsia (PET)
Edited by; Dr. Erica Paras

Door sign

Linda Robinson, 23 years old female. 32 weeks pregnant. Was diagnosed with preeclampsia by her
family doctor and referred to the obstetrician. You are the resident working in the clinic

Vital signs;

BP 145/95

HR 88

RR 12

Temperature 37⁰C

In the next 10 min take a focused and relevant history. At 9 min the examiner may ask you a question or
questions.

Note; the NAC exam may have a similar format but the last 2 lines will look like this; In the next 11
minutes take a focused and relevant history. At 8 min the examiner may ask you a question or questions.

Sample notes to write for yourself:

Linda Robinson how was it diagnosed

23 F Risk factors

PET history Severe PET

Complications

Treatment?

Obs/gyne and general Hx

Patient encounter

Your actions Suggested verbalizing Patient response

222
Opening start How can I help you today? My doctor referred me because
my blood pressure was high

Details about the presenting When was it diagnosed? Last Tuesday


complaint
How high was Your blood 145/95
pressure?

Where you well rested and


Yes
relaxed when it was measured?

Have you ever had a high blood


pressure before? No. this is the first time someone
tell me my blood pressure is high

Yes, he said it’s +1


Did your doctor check protein in
your urine?

Did he prescribe you any No


medication?

Symptoms of severe PET

Do you have headache? No

Did you have any change to your No


vision like double vision and loss
of vision?
No
Do you have breathing difficulty?

Do you have chest pain? No

Do you have abdominal pain? No


Epigastric pain? No
Have you had abnormal seizure No
like movements?

Eclampsia?

Fetal movement How is the baby moving? She is very active

History of current pregnancy

223
GTPAL Is this your first pregnancy? Have This is my first
you ever had abortions or
pregnancy losses?
Last menstrual period No, never
When was your last menstrual
period?

Was it your typical period? Are May 10, Yes


you sure about the dates?

So your expected date is


February 17 and you are 32 That’s right
weeks pregnant
Nausea/vomiting?
How was your date calculated-
By LMP
by your LMP or early ultrasound?
Other complications
Do you have nausea and
vomiting? I had some at the beginning of
pregnancy but not anymore
Have you had any other
complications during this No, it’s been smooth so far
Vitamins/folic acid/iron pregnancy like Infections?
Bleeding? Trauma? Diabetes?
Hospitalizations?
X-ray or medications exposure? No
Do you take multivitamins?

How about iron and folic acid? No


Did you take folic acid pre- I took folic acid
conception? Or when you found When I found out
Medical follow up/US out about the pregnancy?

How far along were you when


About a week after I missed my
you found out about the
period
pregnancy?

Were you exposed to X ray? No

Did you take medications during No


pregnancy?

No

224
Any exposure to over-the-
counter medications? Street
Blood group drugs? Alcohol?

So your family doctor followed Yes, every month


you up so far, correct?

Have you had an Ultrasound Yes, When I was 19 weeks


done? pregnant
Do you know if it’s one or more It’s one girl
babies?

Was there anything outstanding


on the US? My doctor said everything looks
good

A+
What is your blood group?

Relevant review of systems

Vaginal bleeding Do you have vaginal bleeding? No

Vaginal discharge Do you have vaginal discharge? No

Constipation/diarrhea Are you constipated? Do you A little constipated but not too
have diarrhea? bad
Acid reflux
Do you have heart burn? The usual, I’m used to it by now
Jaundice

Stool color
Did you notice your eyes and No
Easy bruising skin are getting yellow?
Dysuria Did you notice a change to your
stool color? No
Change of urine color or amount
No
Hands/face swelling Do you bruise easily?
No
Legs swelling Does it hurt to pee?
No
Pre-pregnancy weight Did you notice a change to urine
color or amount?

225
Headaches? Did your face or hands swell? My hands are puffy

Abnormal movements? How about your legs? They swell if I stand for too long,
Behaviours? but then go down

What was your weight before


pregnancy? 56 Kg

Past medical and surgical history

Have you ever been diagnosed No


with any disease like problems
with your Heart? Lungs?
Thyroid? Kidneys? Bowels? Etc.

Have you ever had surgery? I had my appendix taken out 2


years ago

No
Any hospitalizations?

Did you have any problems with


anesthesia? No
Did you have any complications No
after surgery?

Medications and allergy

Do you take any medications? No

Other than vitamins and folic No


acid, do you take any other over
the counter medications?
No
Do you take Herbs?
No
Do you have allergy to
medications or food?

Family history

Did anyone in your family have No


twins? Congenital
malformations? Recurrent

226
pregnancy losses? Severe mental No
retardation?

Does anyone in your family have


breast, ovarian or colon cancer? No

My father does
Does anyone in your family have
high blood pressure?

How about high blood pressure Not that I know of


during pregnancy?

Relevant gynecological history

Previous STDs Have you ever had a sexually No


transmitted infection like
Syphilis?
2 months before I got pregnant
When was your last pap test?
Pap test

Yes
Was it normal?

Have you ever had an abnormal When I was 20 years old there
was a problem with my test but
pap test?
my doctor repeated it after 6
months and it was ok

Social history

Smoking Do you smoke? No

Alcohol Do you drink alcohol? Not during pregnancy

Drugs Do you use recreational drugs? No

Work What do you do for living? I’m an art student

Home With whom do you live? With my husband

Social support

227
Always screen for Abuse- Besides your husband, do you Yes, both my parents and in laws
pregnancy is a very common have any other support? are in town and everyone is
time for abuse excited about the baby

It is exciting for sure

Have you ever felt unsafe?


No

FIFE What are your thoughts about I’m worried about my baby, is he
your high blood pressure? going to be ok?

Some babies are affected; they


Important to say that high BP in may be smaller or born earlier
pregnancy is a concern to mom than their time. I’m going to
and baby- we need to the treat start you on a medication to
the mom and keep her safe- if lower your blood pressure, and
she is not safe the baby can be follow up with you and your
affected- G-HTN is serious. baby more closely. How does
Much better
that sound?
Nothing right now, thank you
Do you have any other questions
or concerns?

No P.E is required

If asked to do a P.E, look for;

- General; Notice any tremor, abnormal movements or positioning and bruising. Check the vitals –
Both arms, while seated- ensuring the mom is relaxed- double check all the BP's- ensure it is not just
white coat hypertension.

- Hands, head and neck; Look at the hands, notice any pallor, skin or nail changes, swelling. Feel the
pulse and compare both sides. Look at the face for pallor or jaundice, notice any bruising. Look into
the mouth, and do fundoscopic examination.

- Auscultate the heart and lungs

- Examine the abdomen; Do inspection and general gentle palpation then do Leopold maneuver to
check fetal position. At earlier stages of pregnancy you can do full palpation. Check fundal height.
Palapate Epigastric area and RUQ for any pain on palpation

228
- Indicate that you would check fetal heart rate using a Doppler

- Examine the reflexes

- Inspect the legs and palpate the pulses, check for pitting edema

- Indicate that you would do a vaginal examination

Sample questions you may be asked by the examiner

1- What medication would you prescribe to control this patient’s blood pressure?

Answer;

- Labetolol 100-600 mg PO BID-TID, max 1200 mg/day


- Adalat XL 20-60 mg PO daily-BID, max 120 mg/day
- Methyldopa 250-500 mg PO BID-QID, max of 3000 mg/day
- can add Thiazide if inadequately controlled

2- What is the management of Preeclampsia?

Answer;

– If severe, deliver regardless of gestational age

– less than 34 weeks, an adverse symptom, proteinuria >5 g/day; deliver

– adverse symptom: BP > 160/110, Platelets < 100, LFT x 2 increase, Cr x 2 increase,
pulmonary edema, headache/change in vision

For Mild PET

– deliver at 37 weeks

– 34-37 weeks- can do expectant management

– <34 weeks- do daily kick counts, PIH labs 1-2x /week, NST/BPP with Dopplers 1-2x/week,
q3 weeks fetal growth measurements and AFI

1- What investigations would you order;

Answer; To evaluate the mother; CBCD, creatinine, urea, lytes, AST, ALT, Albumin, Bilirubin, INR, PTT,
LDH, -Protein/Creatinine Ratio. Urine Analysis, R/M, C/S

229
-Fibrinogen- it should be elevated in pregnancy, so a “normal” Fibrinogen is too low, and is a worrisome
sign,

-LDH- sign of hemolysis

-Urate- is typically the first abnormal lab. However once it is elevated we don't typically track it, as
further elevation doesn't mean a worsening PET

To evaluate the fetus; Biophysical profile or non-stress test

-Particularly fetal growth, is abdominal circumference< head circumference = brain sparing, AFI,
Umbilical cord Dopplers – changes in the end-diastolic flow patterns

Rating scales points;

- Examinee introduced self and position


- Addressed patient with name
- Used proper non-verbal communication
- Good organizational skills
- Examinee spoke clearly (accent didn’t get in the way)
- Listening and questioning skills
- Showed rapport with patient
- Explored patient concerns
- Attentive to patient physical comfort
- Medical knowledge adequate
- No misinformation was provided to patients
- No concerning ethical/legal issues
- The examinee is respectful of other health care members

Variations of the case and some tips;

1- Case; 28 year old female 36 weeks pregnant presenting with BP 190/110. Manage

Brief history; GTPAL; Primigravida 36 weeks, symptoms of severe PET; none. Complications during
pregnancy; Preeclampsia diagnosed at 34 weeks. Medications and allergies; Methyldopa 250mg BID. No
allergies. Last meal; 6 hours ago. Fetal movement; ok.

Relevant P.E; Auscultate heart and lungs, Assess reflexes, Fundoscopy, inspect and palpate the
abdomen, do Leopold maneuver, vaginal exam; examiner reports; closed cervix.

230
Management;

1. NPO

2. Foley catheter- strict monitoring of urine output

3. Call if BP > 160/105- above that stroke risk increases, hold meds if BP <130/80- fetus is
dependent on maternal blood flow, if she is hypo-tensive so will the baby

4. Continuous fetal HR monitoring

5. IVF: Total fluid intake of 100 cc/hr RL – do not want to fluid overload them and put
them into pulmonary edema, as the patient is third-spacing when they have PET

6. To prevent seizures; MgSO₄ 4g IV loading dose over 20 min then 1g /h


continue until at least 24 hours post-partum or BP decreases post-partum and the
patient is diuresing

7. Acute treatment of BP:

a. Labetolol 10-20 mg IV, then double dose q 10 minutes, max 300 mg, or can do
infusion at 0.5-2 mg/min or

b. Hydralazine- will acutely decrease BP, 5 mg IV, then can use 5-10 mg q 20-30
minutes, max 20 mg or

c. Nifedipine- 5 mg PO q 30 mins or

8. See above for investigations to order

Prepare for delivery; if maternal and fetal status stable induction of labor, if unstable emergency C/S

Discussion; One thing to add is to give corticosteroids for fetal lungs maturity if the patient was < 34
weeks pregnant. You can give Betamethasone 12 mg IM q 12 hours total of 2 doses.

- The steroids typically cause a “moon-lighting phase”- post steroids the patients lab abnormalities may
improve and her BP may improve- this is temporary- don't let it fool you, PET patients are sick and the
“moon lighting” shouldn't change your management plan overall

- <32 weeks MgSO4 is also given for Neuro-protection- this pre-eclamptic lady will be getting already.
We can mention to the patient that the MgSO4 will prevent seizures for her and decrease neonatal
death, cerebral palsy and gross motor-dysfuntion

231
2- Case; 30 years old female, just diagnosed with preeclampsia with BP of 140/90 and +2 protein
on urine dipstick. Counsel

History; The patient is a mother of 2 years old, no abortions or still births. She had a smooth pregnancy
and is 32 weeks pregnant. Not known to have hypertension before, no symptoms of severe PET, and
review of system was unremarkable. She had preeclampsia during her first pregnancy and her baby was
delivered with C section due to fetal distress during labor. Fetal weight was 7 lb and her baby didn’t
need NICU and did well after. Not on medications, and doesn’t have allergies. Doesn’t smoke, drink or
do drugs. She is a stay home mum, and enjoys lots of support from her husband and family.

Counseling; Explore what she already knows and what info she is seeking. See if she has particular
concerns or fears. Talk to her about PET; it’s definition and complications, importance of regular BP
monitoring and more rigorous follow up of herself and the baby. Talk to her about management of
chronic and severe PET. The patient was concerned about the mode of delivery and wanted to avoid C
section if at all possible. Assure her that you will document her wish and honor it, and explain to her that
C section might be needed to save her own or her baby’s lives. Then mention the indications and
complications of C section. Go over the symptoms of severe PET and make sure the patient understands
when to seek help.

3- I recommend that you go over the Canadian guidelines for hypertensive disorders during
pregnancy;

http://sogc.org/guidelines/diagnosis-evaluation-and-management-of-the-hypertensive-disorders-of-
pregnancy/

232
Case 12; Contraception
Edited by; Dr. Erica Paras

Door sign

Madeline Carter, a 31 years old female. Comes in to your clinic to talk about contraception.

Vital signs;

BP 120/75

PR 82

RR 12

Temperature 36.8

In the next 10 min counsel the patient about contraception. At 9 min the examiner may ask you a
question or questions.

Note; the NAC may have a similar stem, but the last 2 lines may look like this; In the next 11 min Counsel
the patient about contraception, at 8 min the examiner may ask you a question or questions.

Sample notes to write for yourself

Madeline Carter Obs/Gyne Hx

31F General Hx

FIFE

Contraception contraception types, efficacy, prices, S/E,

Hx, counsel

Patient encounter

233
Your actions Suggested verbalizing Patient response

Opening start How can I help you today? I need to know about
contraception methods out
there other than the pills

Sure, do you have particular I had a C section and I’m breast


questions in mind? feeding, I want something safe
for me and my baby

Congratulations, I can talk to you


about contraception methods
and answer your questions as we
go. I need however to ask you
few questions about your health
first Sure, no problem

Obstetric history So, was this your first Yes


pregnancy? No miscarriages?

When was your baby born?


May 20
So the baby is 6 weeks now? Yes

How many weeks were you 39 weeks


pregnant when you gave birth?

Why did you have a C section? The baby heart rate was going
slow, apparently the cord was
folded around his neck

I see, Did your baby need NICU? No

Do you know what was the It was 1, 7 and 9


APGAR score?

That’s not too bad, did you have A boy


a boy or a girl?

Is he healthy?
Yes
What was his birth weight?
7.3 Lbs

234
Did you have any problems No, it was very smooth
during pregnancy like bleeding,
infections, clots?

Did you have any problems after


delivery? No
Are you still having vaginal Very minimal
bleeding?

Have you had sex after your


baby was born? Not yet

And how is breast feeding going? It’s going very well, no problems
Any soreness or nipple
discharge?
I still take my prenatal vitamins
Do you take vitamins?
No
Do you take folic acid, iron or
other supplements?

Pertinent symptoms Do you have abdominal pain? No

How is the wound healing? Much better than I thought

I don’t feel it any more

Is there any pain or discharge? No

That’s good to hear

Do you have vaginal discharge or No


itching?
No
Does it hurt to pee?

Have you noticed any change in No


urine color or smell?

Do you have constipation or No. I’m pretty regular


diarrhea?

Any problems with


No
hemorrhoids?

235
Relevant Gyne history Have you ever had a sexually No
transmitted infection before?

Have you had ovarian or uterine


surgery? No

Just before I got pregnant, it was


When was your last pap test?
normal

Have you ever had abnormal


tests? No

14
At what age you had your first
period?

Are your periods regular? How Yes, every 30 days


often

I’ve been always on the pill, but


What contraception did you use the nurse said it may cause my
before? milk to go down

Detailed Contraception history


The combination pills of Really, that’s convenient. I’m
estrogen and progesterone can worried that I may forget to take
do that. But there are other pills the pill because my sleep is all
made only of progesterone that over the place after the baby is
wouldn’t do that. born.
It must be tiring. There are other
methods out there that I will
come to in a minute, I need to Sure
ask you few more questions

Past medical and surgical history Do you have any disease like No
diabetes, heart disease or blood
clots?

Any problems with your heart


valves? Migraine headaches? No
History of breast/gynecologic
cancers? Liver disease?

236
Have you ever had surgery
before?
No

Medications and allergy Do you take any medications? No

How about over the counter


medications or herbs?
No
Do you have allergy to
No
medications or food?

Family history Did anyone in your family have No


ovarian, uterine or colon cancer?

Did anyone have blood clots?


No
How about stroke and other
diseases? My grandfather died of stroke
when he was 87 years old, my
father has high blood pressure
but he is healthy otherwise

Social history Do you smoke? Yes, I enjoy 10 cigarettes every


day

For how long did you smoke? 10 years


Do you drink alcohol?
Occasionally

Do you use recreational drugs?


I never did

My husband helps a lot, and our


Who helps you with the baby? parents are both in town and
they have been a great help

Yes, I love my son very much


Are you coping well?
But I do feel tired

237
Do you feel depressed? No, I’m just tired

When are going back to work? I’m taking a whole year off
Counseling part

Summary of relevant history So far I know that you are 31


years old, a new mom of 6 weeks
old baby. Had a good pregnancy
and delivered at 39 weeks with a
C section due to cord around the
neck. You are healthy, smoke 10
cigarettes a day. You used the
That’s correct
pills before but would like to
hear about other options.

No problems. Now because you


had a C section, it’s advisable to
use an effective method of
contraception for 18 months if
you would want to have a
vaginal delivery with your next
pregnancy
Importance of effective
contraception after C section -Women can begin to ovulate
45-95 days after giving birth, this
is delayed by lactation

-A woman is only considered


anovulatory if she has : < 6
Lactation effectiveness as a
months after giving birth, I see
contraception
exclusive breast feeding and has
amenorrhea- if so the protection
is ~98%

-If don't meet the above criteria,


very important for a women to
be using contraception
Information about contraception

238
(you need to go over types, There are 2 main groups of
mode of administration, cost, contraception; Hormonal and
effectiveness, and major benefits non-hormonal.
and side effect)
The hormonal method includes
the 2 types of pills we discussed
before. The mini pill is 99.7%
Note that in this case no details Do I have to take it everyday?
effective if used perfectly.
of the combined pill are
provided because the patient
will not take them, they are very
It has to be taken at exactly the
common form of contraception
same time each day. If you miss
and if suitable for the patient
then you need to go over the pill by > 3 hours you need
back-up contraception for at
benefits and side effects
least 2 days. It is very sensitive to
changes in timing of the pill- not
good if the person is forgetful.

Have you thought about setting


No, but it sounds like a good
an alarm or reminder system?
idea. What are the side effects?

They may cause irregular


spotting, headache, bloating,
acne and breast tenderness.
Effectiveness may be decreased
by other medications.

Keep in mind the


contraindications as you gather The patch is another hormonal
history and counsel your method; It is applied to the skin
patients – If the patient has a for 3 weeks and removed for a
contraindication for a certain week where menses occurs
method, you don’t need to go
into details about it because you It is a systemic estrogen, just like
don’t want to overwhelm your the combo pill, therefore, in
patient theory it can affect milk
production

239
There is Estrogen vaginal ring. It
is also a combo, but less
I don’t like that one
systemic side effects and
therefore may not affect milk
production in the same way

There is also the shot, it is only


progesterone and you need to
get an injection every 3 months,
it’s 99.7% effective. Periods will
stop but there will be irregular
spotting. Weight gain is another
concern for some women. If you I see
want to get pregnant again after
the shot, it may take 9 months
for the ovaries to start ovulating
again.
What other options are there?

And there is the intrauterine


device, one type is hormonal and
contains only progesterone, it’s
called Mirena. It’s 99% effective.
Normal Mirena is still used by
most for 5 yrs, and the low dose
one for 3 yrs.

IUD has to be inserted and


removed by a doctor, may cause
irregular bleeding in the first 6 It sounds like a good option
months and may increase
cramps and bleeding. There is a
small chance that it may fall off.
How do you feel about it

- Expulsion is increased if
inserted < 6 weeks PP, if there

240
was a C-section and if the px is
breastfeeding- but overall the
I see
risk is low

It’s copper and is non hormonal.


It’s 99% effective and can last for
5 years. Has very similar S/E to
the hormonal intrauterine device

- Copper device more women


have heavier periods, vs. Mirena
where 80% of women have
amenorrhea after the first year.
Do you have questions?
No, that’s clear

Other contraception methods


that may work for you are non-
hormonal and can be used with
or without a spermicide. They
include barrier methods that can
be used by females like female

Condoms, cervical caps and


diaphragms. None of them
protects against sexually
transmitted infection. The
female condom is the most
affective ranging between 80-
95%. How do you feel about
them?
I haven’t tried them before, I
don’t like to worry about
condoms when we are about to
have sex, I want something more
convenient
I understand, this goes for male
condoms too, right?

241
The last methods are the
natural; like the calendar,
withdrawal and symptom-
thermal control, all but
withdrawal are hard to use in
your case because it’s hard to
predict ovulation when you are
breast feeding, and you want
something more effective at this
stage,
I want something more
-timing and withdrawal have up effective. This doesn’t work for
to 24% unintended pregnancies me
over one year
I’m thinking to go with the IUD,
it seems convenient and I don’t
have to worry about it for years.

How much does it cost?


Copper IUD cost around 150, and
the hormonal IUD around 350,
some insurance companies do
cover them

It’s pricy! I don’t have insurance.


What about the Shot?
The shot is around 35-40 $ but
you need one every 3 months

I will give you a flyer about all


the contraception methods to
Sounds excellent, I need some
read through
time to weigh my options.

Do you have any other


Not at the moment
questions?

I can see you after 1 week. If you


think of any questions write I will, thank you
them down for when we meet

242
You’re welcome!

Physical exam;

No physical exam is required in this case

Sample questions you may be asked by the examiner;

1- Do you have a concern about the pill if the patient decides to use it after weaning her baby?

Answer; the combined estrogen and progesterone pills are not recommended in women over 35 years
who smoke because of increased risk of blood clots. Absolute contra-indication if > 35 yrs and smoke
>15 cig/day, Relative contra-indication if smoke <15 cig/day

2- How does the copper intrauterine device work?

Answer; impedes sperm transport and fertilization, prevents implantation by producing a foreign body
reaction and chemical changes in the endometrial lining.

3- What contraception methods protect against HIV?

Answer; Male condom

Rating scales points;1. Examinee introduced self and position

2- Addressed patient with name


3- Used proper non-verbal communication
4- Good organizational skills
5- Examinee spoke clearly (accent didn’t get in the way)
6- Listening and questioning skills
7- Showed rapport with patient
8- Explored patient concerns

243
9- Attentive to patient, allows time for questions
10- Medical knowledge adequate
11- No misinformation was provided to patients
12- No concerning ethical/legal issues
13- The examinee is respectful of other health care members

Variations of the case and some tips;

1- Please be reminded that you need to take relevant history when asked to counsel a
patient, so that counseling is tailored to patient’s needs, and pertinent health issues are
covered

2- In Canada, you can counsel teenagers of both sexes about contraception. In this age
group it is important to educate them about sexually transmitted infections and how to
protect themselves. If the teenager doesn’t want his/her parents involved then you
must maintain confidentiality. Teenagers don’t need consent from their parents for a
contraception prescription. Always do HEADSS screen for this age group (refer to
chapter 2 for more details). Ask your teenage patient the age of her partner to rule out
legal issues. No matter the age of her partner ensure sex is consensual.

3- In cases of counseling, especially when too much medical information is provided, offer
to give your patient handouts to read through

4- SOGC Contraception guidelines @ sogc.org , and the website www.sexualityandu.ca are


great resources

244
Case 13; Postmenopausal bleeding
Edited by; Dr. Erica Paras

Door sign

Margaret Finlay, is a 52 years old female presenting with vaginal bleeding.

Vital signs;

BP 120/70

HR 80

RR 12

Temperature 37 ⁰C

In the next 10 min conduct a focused and relevant history and perform a focused and relevant physical
exam. At 9 min the examiner may ask you a question or questions.

Note; the NAC may have a similar stem but the last 2 lines may look like this; In the next 11 minutes
conduct a focused and relevant history and perform a focused and relevant physical exam. At 8 min the
examiner may ask you a question or questions.

Sample notes to write for your self

Margaret Finley Atrophy

52 F Cancer

Endometrial hyperplasia

Hx and P.E Fibroid

Non-gyne cause

Patient encounter

245
Your actions Suggested verbalizing Patient response

Opening start How can I help you today? I have vaginal bleeding

Tell me more about it I have vaginal spotting, I’m kind


of worried, women my age
shouldn’t have it

I understand your concern, we


will find out why you have it.
Do you mind if I ask you few
questions?
Not at all

Details about vaginal bleeding

Onset, duration

Amount When did the bleeding start? About 5 months ago

Frequency How many pads do you use per May be 2-3, It’s just spotting
day? Amount of blood on the not much
Precipitating factors pads?

How often do you get it?


Almost every day
Pain and bleeding after Was there a precipitant for the
intercourse No
first bleed like a
trauma/intercourse/infection/
douching?
Yes, with my husband
Are you sexually active?
Date of last menstrual cycle
Do you feel pain during sex? Yes, I have mild pain lately

At penetration or deep inside? Deep inside


Use of HRT or vaginal creams
Do you have Bleeding after
sex? No
When was your last menstrual A year ago
cycle?

246
Previous episodes Do you take hormone No
replacement therapy, Orally or
vaginally?
Vaginal discharge Do you use vaginal creams? No
Lubricants?

Vaginal itching

Pelvic/abdominal pain Did you have abnormal vaginal No


bleeding before?

Do you have vaginal discharge?


No, I feel dry
Do you have vaginal itching?
Did you notice an increase in Sometimes
the number of yeast infections?
No, I get 2 each year
Did you notice an increase in
the number of urinary tract
infections? How many in a No I get it once or twice every
year? year
Do you have pain in your No
pelvis? Pressure? Bloating?

Do you have abdominal pain?


Early Satiety? No

Constitutional symptoms Have you lost weight recently? No

Do you get fevers? No

Do you get drenching night No


sweats?

GI/urinary symptoms Does it hurt to pee? No

Bleeding diathesis Is there a change in urine color No


or blood in urine?

Do you have constipation or


diarrhea? No

Is there a change in stool color


or blood with stool? No

247
Do you bruise easily? No

Does your gum bleed? No

Anemia symptoms and other Do you feel fatigued? Not fatigued but more tired
precipitants than usual

Not really
Did other people say you look
pale?

Do you feel your heart is No


racing?

Do you have shortness of


No
breath or chest pain?

Obstetric history Do you have children? I have 3

Did you ever have a No


miscarriage?

So you had a total of 3


Yes
pregnancies, is that correct?
Normal, I’m very blessed, they
Were your kids born via normal
vaginal delivery or C section? were all born healthy

They were all around 39 weeks


At how many weeks?

No, they were all very smooth


Any significant complications
with the pregnancies? Post-
partum bleeds? Blood
transfusions?

Gyne history

At what age have you had your 13


fist menstrual cycle?

Did you have heavy, and or


No
painful periods?

When was your last pap test? The last year

248
Have you ever had an abnormal No
pap test?

Have you ever had a


mammogram? Yes, 2 years ago

Normal
What was the result?
I used the calendar all my life
Did you use contraception?
and it worked well.

Did you use the pill or IUD?


I took the pill for 2 months in
my 20s but didn’t like it

Do you have hot flashes? No

Have you ever had a sexually No


transmitted infection?

Any known uterine/ovary


I had an ovarian cyst removed
cysts/growths/fibroids/polyp?
when I was 37, it was benign
Ever require a D&C?
No

Past medical and surgical Do you have a medical disease No, I’m very healthy
history like diabetes mellitus, or heart
disease?

Have you ever been diagnosed


with cancer, specifically No, God forbid
ovarian, uterine, breast or
colon?

Have you ever had surgery


particularly to uterus and
ovaries other than the one you
told me about? No

Medications and allergy Do you take any drugs? Any No


over the counter medications
or herbs?

249
Do you take soy containing I only take a multivitamin
food supplements?

Do you have allergy to


medications or food? No

Family history Did anyone in your family have No


cancer of the uterus, breast or
colon?

When did your mother/sisters I think my mom was 50, I don’t


have any sisters
enter menopause?

Did they have any post-


menopausal bleeding? Not that I know of

Social history

Smoking Do you smoke? No

Alcohol Do you drink alcohol? Occasionally

Occupation What do you do for living? I’m an accountant

Any weight-bearing exercise? No

FIFE

Effect of bleeding on everyday How is the bleeding affecting It’s very annoying, I have to
life and relationship with your daily function and always wear a pad, and I’m not
husband marriage? enjoying sex anymore

I understand, it is for sure


inconvenient. Have you tried
I did but I felt burning
some creams or lubricants?
Patient ideas? I see, some of them have a
natural water base that may
not feel as burning. I will give
you a brochure about vaginal Thank you that would be great.
dryness of menopause and But what is causing the
Etiologies of post-menopausal
what’s available to help with it. bleeding? Do I have cancer?
bleeding: Atrophy, Endometrial
hyperplasia/cancer, Polyps,
hormone replacement therapy,
fibroids, adenomyosis, from

250
other organs, anti-coagulants, There are many causes of
medications, soy, post vaginal spotting after
radiation, infections, cervical menopause the most common
cancer- Need to rule in and rule is dryness and atrophy of
out each of these through the uterine and vaginal lining.
history Malignancy can cause spotting
too, as well as fibroids. We
need to run some tests to know
what the cause is. Do you have Not at the moment, thank you
particular questions or
concerns?

Physical examination;

3- General; Mention that the vital signs are normal. Comment on the body built and indicate that you
would check the weight and height and BMI. Inspect the hands and feel the pulse. Examine the head
and neck for pallor, jaundice, bruising or bleeding and lymphadenopathy

14- Auscultate the heart and lungs

15- Inspect and palpate the abdomen looking for masses, ascites, hepato and splenomegaly.

16- Mention that you would do a genital and vaginal exam, the examiner will say pass.

17- Mention that you would do a breast exam, the examiner may say pass, or let you do it

18- Examine the rest of the inguinal/pelvic lymph nodes – the examiner may say pass

Sample questions you may be asked by the examiner;

1- What is your differential?

Answer; Gynecologic; Endometrial and vaginal atrophy, Cancer, polyps, post-menopausal hormone
therapy, endometrial hyperplasia, fibroids. Or possibly non gynecological bleeding like urinary tract or GI
bleeding. Adenomyosis- typically resolves post-menopausal but is still on the differential

19- What investigations would you order?

CBCD, lytes, urea, creatinine and urine analysis. Hysteroscopy and endometrial biopsy, trans-vaginal
ultrasound

251
Cervical cancer screen (PAP test) is also necessary part of the pos-menopausal bleeding work-up

Rating scales points;

- Examinee introduced self and position


- Addressed patient with name
- Used proper non-verbal communication
- Organizational skills
- Recognized and managed emergency effectively prioritizing actions
- Examinee spoke clearly (accent didn’t get in the way)
- Good Listening and questioning skills
- Showed rapport with patient
- Attentive to patient physical comfort
- Medical knowledge adequate
- No misinformation was provided to patients
- No concerning ethical/legal issues
- The examinee is respectful of other health care members

Variations of the case and some tips;

 Case; 55 year old female presenting with vaginal bleeding

History; 1 month history of moderate vaginal bleeding, almost daily. The patient describes night fevers
and sweating which she thinks are due to hot flashes of menopause. Menopause at age 51. Menarche at
age 11. No HRT. No symptoms of anemia, no GI/Urinary symptoms. The patient is G1P1, last pap test
and mammogram normal, she used IUD for 20 years. Past medical history is significant for obesity with a
BMI of 40, and a history of polycystic ovarian syndrome. The patient is also a known diabetic for 3 years
on Metformin and Gliclazide. No family history of cancer.

Physical examination; Stable vital signs, BMI 40. Normal cardiac, respiratory and abdominal exam.
Vaginal exam confirmed bleeding and was otherwise unremarkable.

Discussion; Unopposed estrogen is a risk factor for endometrial cancer that must be excluded in all
women presenting with post-menopausal bleeding. Make sure to take a thorough history that covers
endometrial cancer risk factors.

252
– Risk factors that need to be addressed- obesity, nulliparous, PCOS, early menarche, late
menopause (increased duration of estrogen exposure), OCP use, tamoxifen, systemic estrogens,
HRT, polyps, Lynch syndrome, HNPCC

– Hysteroscopy is not key- Biopsy and/or Ultrasound are key

– Biopsy is warranted: anyone >40 yrs & abnormal uterine bleeding (AUB), >90 kg with AUB, post-
menopausal with any bleeding especially if endometial thickness is >4 m, age 45 to menopausal
average 51 yr with AUB, < 45 yr with persistent AUB/unopposed Estrogen/hyperplasia risk
factors/who have failed medical therapy, pre-menopausal with anovulation/amenorrhea > 6
months, atypical glandular cells on pap smear, endometrial cycles on pap smear, monitoring for
known history of hyperplasia, or screening for a women who is at high risk of endometrial
cancer

 Case; 53 year old female presenting with vaginal bleeding

History; Menopause at age 51, started estrogen only HRT almost immediately. She had irregular vaginal
bleeding the first 3 months of hormonal therapy. Couldn’t tolerate progesterone, and refused
progesterone IUD because she didn’t want a foreign body in her uterus. The last 2 months bleeding
started again, it is irregular and of mild-moderate amount. No other symptoms. She underwent
hysteroscopy and endometrial biopsy before HRT was started and refused it afterwards because it’s; a
headache. Review of systems unremarkable. No medical illnesses or family history of cancer. She never
got pregnant in her life and never used contraception as her husband is infertile.

Physical examination; Normal vital signs. BMI 23. Normal cardiac, respiratory and abdominal
examination. Vaginal examination revealed a bulky uterus.

Discussion; It is very important in this case to show respect and understanding of the patient own
choices. She could have cancer secondary to unopposed estrogen in her HRT, but you shouldn’t lecture
her how a progesterone containing IUD could have decreased her risk. In this case the patient asks if she
has to undergo this test again; Hysteroscopy and endometrial biopsy. Explain that you understand that
she doesn’t like it but is necessary at this stage because her bleeding may be due to cancer.

-If she refuses that offer her an ultrasound to assess the lining- it doesn't give histologic evidence of
cancer, but at least it is something to follow the endometrial lining with.

253
-Persistence is key- reinforcing the importance of the biopsy

254
Case 14; Abortion
Edited by; Dr. Erica Para

Door sign

Melissa Smith, a 27 years old female, 10 weeks pregnant, presents to the ER with vaginal bleeding. US
confirmed fetal loss

Vital signs;

BP 120/80

PR 80

RR 14

Temperature 36⁰C

In the next 10 min Counsel the patient. At 9 min the examiner may ask you a question or questions

Note; The NAC may have a similar stem, but the last 2 lines may look like this; in the next 11 minutes
counsel the patient. At 8 minutes the examiner may ask you a question or questions.

Sample notes to write for your self

Melissa Smith Emotional support

27 F risk factors

Previous miscarriage

OBS Hx, counseling emotional support

Patient encounter

Your actions Suggested verbalizing Patient response

255
Opening start Hi Melissa, how are you doing Miserable... patient crying, I lost
today? my baby

Offer a tissue and sympathize It must be hard for you

You allow a moment of silence

I don’t know why, I did all the


right things; I took multivitamins,
folic acid, I didn’t drink I don’t
smoke, I don’t understand, my
doctor said things are all looking
good, I don’t understand

Help vanish feelings of guilt It is not your fault, You didn’t do The patient looks calmer and
anything wrong, I want you to more receptive
understand
- Very important to re-
emphasize that the patient
should NOT Blame themselves
for the lose- it is nothing they
did, or could have done to
prevent it- 1/3 of pregnancies
miscarry- there was something
“wrong” with the pregnancy- it
would not have been a healthy
pregnancy- re-enforce to the
patient that you understand it is
difficult, but that it is NOT their
fault.

Loss of a baby this early is


usually due to abnormal baby
chromosomes and this is not due
to anything the mother had
done. You did all the right things
and it’s not your fault, it was just
an unhealthy pregnancy

But we had sex; did I lose him


because we had sex?

256
No, sexual activity doesn’t cause
miscarriage

So why?

Spontaneous abortion occurs in


8-20% of pregnancies- of
recognized pregnancies, rates
~1/3 if unrecognized pregnancies
are included. It tends to occur
more in older women who had
many pregnancies, and in
women who had prior
spontaneous abortions. Smokers
and women who consume large
amounts of alcohol or take
cocaine are at higher risk. Other
risk factors include Non-steroidal
pain killers like Advil, Voltaren.
Tylenol is not part of this group
of pain killers and doesn’t cause
abortion. Consuming large
amount of coffee like 10 cups is
a risk factor too Patient smiles and looks more
comfortable; I don’t belong to
any of those

You smile and nod your head in


understanding; absolutely

But what else may cause a


miscarriage?

In addition to problems with the


chromosomes, (50%), congenital
anomalies of the fetus, trauma,
maternal diseases, specific
infections we call TORCH.
Abnormal structure of the uterus
like having a septum can cause
miscarriage, we can do an
ultrasound to check it out, and a

257
lot of structural abnormalities I see, I want an ultrasound done
can be treated surgically

Sure, I will arrange for one,


sometimes the mother may have
a disease like a thyroid problem
that may cause a miscarriage

Do you mind if I ask you some


questions about your health to
see if we can identify a cause
No, not at all, please go ahead
that we can treat?

Obstetric history

Was this your first pregnancy? Yes, I’ve never been pregnant
before
GTPAL

Accurate dating of Pregnancy When was your last menstrual


period? March 10
Symptoms of pregnancy
Was it your normal period? Are Yes
Symptoms of abortion you certain of your dates?
Yes
Medications/Herbs/X-ray When did you find out you were
exposure Around April 15
pregnant?

By what method- blood work,


urine sample, ultrasound? I did a urine test

Did you have nausea and I felt nauseated in the mornings


vomiting? but no vomiting
Can you tell me what happened I woke up from sleep with
when you lost the baby? vaginal bleeding and some
abdominal pain. I came to the
hospital immediately and they
told me I lost the baby

258
Sorry about your loss.

Did you take any medications or Nothing at all


herbs during pregnancy?

Were you exposed to X ray?


No

Relevant review of systems

Infection Do you have fever? Chills? No

Endocrine Do you have vaginal discharge? No

Do you have skin or hair


changes? Rashes? Flu-like
symptoms? No

Do you feel intolerant to heat or


cold? No
Are you constipated or do you No, I’m very regular
have diarrhea?

Do you feel a change to your


energy level? No

Relevant Gyne history

Contraception Have you used contraception I was on the pill throughout my


before? life, I stopped 6 months ago as
STD we started trying
Pap test No
Have you ever had a sexually
transmitted infection?

When was your last pap test? A year ago

Have you ever had an abnormal No


pap test?

Past medical and surgical history Do you have any disease? No, I’m very healthy

259
Have you ever had blood clots? No

Have you ever had surgery? No

Medications and allergy Do you have allergies to No


medications or food?

Family history Did anyone in your family have No


blood clots?

Social history

Is your partner here? Yes, my husband is in the waiting


room
As implied above most points
are covered He looked sad but stayed strong
How did he take the news? for me

Would you like me to talk to Yes, that would be great


him?
Yes, I’m a receptionist
Do you work?

Do you need a note for your


work? Yes please

Closure and some How are you feeling now? Much better, it helps to know it’s
recommendations not something I did

Do you have any questions for


me Not at the moment

It takes about 8 weeks for the


changes of pregnancy (for
example; breast tenderness) to
fully resolve. Your next
menstrual cycle is expected to
resume 4-6 weeks from now. Are Not at the moment. I will take
you planning to try again? the pills again, till we both feel
ready.
I understand. I would like to see
you again in clinic to talk more
about what to expect next time.
In the mean time we will do

260
some blood work, and book the
Ultrasound. Do you have any
Not at the moment. Thank you
questions for me?

Physical examination;

No physical exam is required in this case, if required see below

Sample questions you may be asked by the examiner;

1- What is your differential for vaginal bleeding in the first trimester?

Answer; Physiologic-implantation bleeding, Complete abortion, Hetero-topic pregnancy,


cervical/vaginal/uterine pathology- polyps etc. cervical ectropion, intercourse/trauma related, septic
abortion, missed abortion, inevitable abortion, non- gynecological source of bleeding like GI or urinary
tract

2- What investigations would you order initially for a patient presenting with vaginal bleeding
in the first trimester?

Answer; CBCD, type and screen and antibodies, B-HCG- quantitative, trans-vaginal ultrasound.

Rating scales points;

- Examinee introduced self and position


- Addressed patient with name
- Used proper non-verbal communication
- Organizational skills
- Recognized and managed emergency effectively prioritizing actions
- Examinee spoke clearly (accent didn’t get in the way)
- Good Listening and questioning skills

261
- Showed rapport with patient
- Attentive to patient physical comfort
- Medical knowledge adequate
- No misinformation was provided to patients
- No concerning ethical/legal issues
- The examinee is respectful of other health care members

Variations of the case and some tips;

1. As noted in this case, when the patient is sad or upset, the priority is to provide emotional
support. Starting with the history without acknowledging the patient’s feelings is going to make
the patient even more upset and less cooperative. It may be challenging during the exam
because you are under the pressure of time, but remember that SP's are well trained and your
communications skills are tested. Inappropriate behavior or ignoring the patient feelings may
cost you the whole station

2. Case; 30 year old female, 11 weeks pregnant, presenting with vaginal bleeding

History; GA 9 weeks confirmed by date of last menstrual period. Bleeding started 2 hours ago and is
mild. No abdominal pain. This is the second pregnancy, the first pregnancy was 2 years ago; no
complications and ended with vaginal birth of healthy baby at 39 weeks. No trauma, infection, GI,
endocrine or urinary symptoms. The patient is taking folic acid and multivitamins. No exposure to
NSAIDs, smoking, alcohol cocaine or X ray. The patient is healthy, and family history is noncontributory.

Physical examination; stable vital signs, Chest and abdominal examination unremarkable. Pelvic
examination excluded local source of bleeding, vaginal examination confirmed mild vaginal bleeding,
closed cervix, and GA of 11 weeks. Fetal heart beats were detected with Doppler US.

Discussion; Threatened abortion is the most likely diagnoses. Watchful waiting is sufficient in most
cases. It is important to emotionally support the patient and counsel her. B-HCG levels help sort out the
differential along with US. Progestin may be used but its use is controversial. Bed rest is typically
recommended- although evidence on this is lacking- but in practice it is done. Important to remind the
patient that they are at increased risk of miscarriage, pre-term birth, premature rupture of membranes,

262
antepartum bleeding and growth restricted babies- it is important that if they have any more bleeding
to come directly to hospital to assess what is happening and the viability of the pregnancy.

3. Case; 29 years old female 10 weeks pregnant, presenting with vaginal bleeding and fever

History; 10 weeks GA confirmed by last menstrual period. The patient had 2 sexually transmitted
infection in the past; Gonorrhea and Chlamydia. She is presenting with fever, abdominal pain and
vaginal bleeding. She got pregnant over an IUD that was left in place. No other symptoms. The patient is
non-smoker, non-drinker and doesn’t do drugs.

Physical examination; Temperature 38.8 ⁰C, PR 110, BP 110/70, RR 18. Chest examination was normal,
abdominal examination revealed lower abdominal tenderness. No fetal heart beat detected and vaginal
examination showed an open cervix with vaginal bleeding and passing products of conception.

Discussion; Septic abortion can be fatal, prompt diagnosis and treatment can save the patient’s life. If
suspected draw blood culture and send high vaginal swabs for culture and sensitivity and start the
patient on broad spectrum antibiotics. Evacuation of uterine content is the next step; get an OBS consult
for D&C.

Antibiotic choices are:

a. Clindamycin 900 mg IV q8h & Gent 5 mg/kg/day +/- Ampillicilin 2 g IV q 4h

b. Ampicillin 2 g IV q4h & Gent 5 mg/kg/day & Flagyl 500 mg IV q8h

c. Pipracillin-Tazobactam 4.5 mg IV q6h

d. Imipenem 500 mg IV q6h

e. IV Antibiotics are given for at least 48 hours and symptoms start to improve, they are
then stepped down to PO Antibiotics for at least 10-14 days.

f. Urgent uterine evacuation is key once the patient is hemodynamically stabilized.

263
4. Induced abortion remains a controversial issue. There are no legal rules to regulate abortion in
Canada. It is considered a medical procedure and the decision is left up to the patient with the
help, guidance and counseling of her physician. Here are some important terms;

Abortion; is the termination of pregnancy before fetal viability (500g or 24 weeks gestation)-

Termination of pregnancy; is the term used to describe pregnancy termination after the age of viability

There are obvious cases where abortion or termination of pregnancy is done for medical reasons (save
the mother’s life, or where fetal anomalies inconsistent with life exist). But there are cases where
controversy arises as in unplanned pregnancy and teenage pregnancy. In all cases it is the mother’s
decision. It is important to keep in mind that such a decision is already hard for the patient and need not
be made more difficult by judgments. You need to explore the patient request for abortion; why does
she want abortion? Is she worried about her career plans? Is she afraid to keep the baby? Is money a
problem? What types of support does she have? Is the father involved in the decision? What is her
relationship with the father? Is there an emotional or physical abuse? Did she share the news of
pregnancy with any one? Would she get help from her family and friends? Would she consider keeping
the baby if more help is provided? Does she know about foster care? You may want to refer the patient
to the social worker if social and financial issues are part of the problem.

5- Be reminded that a full obstetrical and medical history is part of comprehensive assessment in
these cases.

6- You need to put your personal opinions aside and give a pure medical advice; identify medical
indications for abortion if any and go over the risks of the procedure. Timing is very important
and the mother should know that it will be hard to find a practitioner to terminate pregnancy
for no medical reason after the age of viability. However, risks are much less and abortion is
accessible in Canada for personal reasons before 20 weeks. The only option after 20 weeks
might be to go to the States and the patient should be informed about all her options.

7- Physicians have the right to choose not to be involved in abortion and in this case they should
inform the patient in a timely fashion and in a nonjudgmental way. They still have the duty to
treat any medial problem within their scope of practice. - if not refer on to another colleague
that can help

264
Case 15; Cancer patient refusing treatment
Edited by; Dr Sadik Salman

Door sign;

Mary Christopher, 72 years old female, was diagnosed with a right breast lump highly suspicious for
malignancy 2 years ago. At that time she declined further investigations and treatments including
surgery and opted to treat herself with prayers. She is presenting today after being convinced by her
husband to repeat the CT scan which showed bilateral breast lumps, lung and thoracic spine metastases.
Her family doctor referred her to the oncology clinic for further evaluation and management. You are
the resident working in the clinic.

Vital signs;

BP 120/70

PR 85

RR 14

Temperature 37 ⁰C

In the next 10 minutes explore the patient’s ideas and concerns

Note; the NAC may have a similar stem but the last line will look like this; in the next 11 minutes explore
the patient’s ideas and concerns

Take 2 min to read and understand the question, it is about exploring the patient’s view, and this is all
about your communication skills!

Sample notes to write for yourself;

Mary Christopher Emotional support

72 F Respect and explore patient choice

Hx and counsel Functional status/pain

Does the patient want treatment?

Patient ideas about treatment

265
Palliative/oncology consult

Patient encounter;

Your actions Suggested verbalizing Patient response

Opening start Mrs. Christopher, It’s my Pleasure to meet you


pleasure to meet you.

I understand that you was


That’s correct
diagnosed with a breast lump
2 years ago and didn’t want
any further investigation or
treatment at that time, is that
right?

I also gather that you had a


My doctor said I have breast
recent chest CT scan, what do
cancer that is now spread to
you know about it?
my lungs, and bones

It must be hard for you to


Patient cries
hear that

Offer a tissue and a moment


of silence

How have you been feeling? I’m very tired, I need to rest a
lot and it takes forever to get
the house work done

For how long have you been Well, I’ve been always tired
feeling tired? but the last 3 months were
bad for me

266
I see, Are you getting any My husband and daughter
help? are always around, they are
doing a lot of things for me

My daughter did the laundry


What are they helping you the other day, and my
with? husband is going shopping by
himself, I don’t have energy
to accompany him anymore

Did you ever need help with


personal care? No, I can take care of myself

Pain Do you have any pain? I get back pain right here
(Patient points to the middle
of her back)

This is where the tumor has It’s always there but not very
gone to bone, How often do bad, it’s really bad in the
Details about pain and you feel the pain? morning
exploring patient wishes
What do you do about it? I take Tylenol, but it’s not
helping

I’m sorry to hear you are in


pain; I would like to help you I will take an analgesic, but
feel better. Would you be not your fancy cancer
open to treatments we can treatment
offer for the pain? I can
prescribe you a stronger
analgesic and we can look
into radiation therapy to the
bone

I will be happy to prescribe I don’t want any


you an analgesic, and if you chemotherapy or radiation.
don’t mind I would like to This cancer is going to be
hear more about your
thoughts regarding the

267
radiotherapy for the pain; it is treated by a miracle and I’m
not a cancer treatment, it is praying everyday
only treatment for the pain

I see where you are coming


from, But don’t you think that
a medical therapy can be your
miracle?
I never thought about that

I mean you can continue to


pray and it’s great to have
hope in prayer, this can go
along with medical therapies
Patient gives you a curious
look as if you opened her
eyes to something she never
considered before
Why do you refuse medical
therapy?

Patient sighs and moves her


eyes away from you
I’m listening

I had a sister who died from


breast Cancer, she was only
50 and suffered a lot from
chemotherapy, she didn’t
believe in miracles but the
treatment killed her and I
don’t want to suffer from
your drugs, many people cure
their cancer with prayers
every day. I’d rather go in
peace

268
You allow a moment of
silence, the patient looks
I’m sorry about your loss, it is
back at you, you nod your
head encouraging her very hard to see our loved
ones suffer, I see where you
are coming from, and my goal
is to decrease suffering as
much as possible.
Unfortunately, we can ‘t cure
your cancer at this stage, but
we can help you live your What is this radiotherapy
remaining life with minimal treatment for my back pain?
suffering

Cancer cells in your bone are


causing the pain and we can
kill them by radiotherapy.
However, you need to be
evaluated by the
radiotherapist who is
experienced in this sort of
treatment and can tell you if
it can be done for you and
how well it will help And how can I see the
radiotherapist

I will refer you if you like


Yes I would like

I will prescribe you an


analgesic and refer you. Do No
you have any other pain?

What other concerns do you


have about cancer I don’t want chemotherapy, I
treatment? can’t die miserable as my
sister

269
I understand, there are other
new treatments for breast
cancer like hormonal therapy
and new drugs against
specific receptors of cancer
cells called immunotherapy.
The newer chemotherapy
No one told me this 2 years
agents are not as toxic as
ago, that’s all new for me
older ones

I’m here to help, and I can


refer you to the oncologist
who can get you tested to see
which therapy works for you
because it depends on the
type of cancer, and he can
give you more detailed
information about the drugs
that may help including their
side effects, you can think
about it and decide for your
self

I want to see an oncologist, I


can’t believe no one talked to
me about this 2 years ago
I will make sure you see an
oncologist as soon as
possible. Do you have other
questions or concerns?

Would the oncologist send


me for surgery? I don’t want
to be slaughtered by a

270
I understand your concerns, surgeon, my sister suffered a
the oncologist and his team lot from them
will evaluate your case and
may offer you surgery, and it
is your decision at the end.
Can you tell me more about
your sister’s surgery? Well, apparently they were
You offer tissues, and allow a not clean enough, and her
moment of silence wound got infected.(patient
cries)
I’m sorry your sister suffered
a lot. However what
happened to her will not
necessarily happen to you.

This oncologist better be a


human and listen to me
I will talk to him about your
concerns and I’m sure he is
going to be supportive. Do
you have any other concerns?

Thank you, I wish I was your


patient 2 years ago, the other
doctor was idiot.

We all try to do our best, I’m


here for you if you need
anything

Thank you

Physical examination;

Not required in this case

271
Sample questions you may be asked by the examiner;

None in this case

Rating scales points;

- Examinee introduced self and position


- Addressed patient with name
- Used proper non-verbal communication
- Organizational skills
- Examinee spoke clearly (accent didn’t get in the way)
- Listening and questioning skills
- Showed rapport with patient
- Attentive to patient comfort
- Was professional in responding to patient frustration
- Medical knowledge adequate
- No misinformation was provided to patients
- No concerning ethical/legal issues
- The examinee is respectful of other health care members

Variations of the case and some tips;

1- I saw this patient during my oncology rotation. It is one of the most challenging cases I saw in
my life. The oncologist treating this patient was a master in communication. I took off the highly
specialized oncology counseling and focused on the communication part. In this case the patient
is seen by a family doctor and is not yet evaluated by the oncologist. As you can see
professionalism is key and can be illustrated in the following points;

- Respect the patient decision to forgo treatment; no matter how odd it may seem to you when a
patient with a deadly disease refuses all sorts of treatment. It is the patient’s decision at the
end. If you were the person assessing the patient when she refused treatment then you need to
make sure she is competent to make the decision by excluding delirium, dementia, psychosis
and depression. If the patient is competent then her decision must be honored. Trying to involve
family members without her consent, or coerce her in a way or another is not acceptable.

272
- Explore the patient reasoning; a caring warm non-judgmental attitude encourages the patient to
open up. This can be illustrated by asking about pain, showing concern and offering to help. You
can gain your patient’s trust by understanding where she is coming from and being
compassionate and sensitive about her feelings. This way your patient will open up even more

- Non-verbal communication; is very important. Offering a tissue, allowing a moment of silence,


making appropriate eye contact are examples. Being truly sympathetic with the patient will
show in your body language. Treat SPs as you would treat your own patients; body language
can’t be faked

- Explore again; when the patient asked about surgery it was obvious there is a story behind it.
The patient was very much drawn into her sister’s tragedy that she couldn’t see other
possibilities. Helping the patient open up in a safe non-judgmental environment will allow her to
see other facets of the issue and consider new options. Always ask about further concerns

- Dealing with the patient emotions; the patient in this case was sad, frustrated, then she showed
some denial and anger. Allow the patient to ventilate her feelings and stay professional

- Focusing on patient’s well fare; when the patient denied any explanation about her case by her
doctors 2 years ago, you may have said to yourself; I’m sure they told you everything! However,
it is better to ignore the patent statement as arguing with her will unlikely take you anywhere.
The other example is when she concluded that her sister’s wound infection was medical
negligence; not going into details is best, as she is expressing her frustration about an old life
event. Reminding her that everyone is different and that her decisions will be honored
encouraged her to open the door to treatments she was absolutely refusing before is more
positive. It is your goal to help her decide what is best for her, and it’s not always easy

- I encourage you to review the basic ethical principles presented in detail in chapter 6

273
Case 16; Neonatal jaundice

Edited by; Dr. Abbeir Hussein

Door sign;

Suzan Schmitt, mother of Peter, 1 week old boy, comes to the public health center because her
son has jaundice

Vital signs;
BP 80/42
PR 120
RR 40
Temperature 37 ⁰C

In the next 10 min obtain a focused and relevant history. At 9 minutes the examiner may ask
you a question or questions

Note; The NAC may have a similar stem but the last 2 lines will look like this; in the next 11
minutes obtain a focused and relevant history. At 8 minutes the examiner may ask you a
question or questions

Sample notes to write for yourself;

History source mum Suzan Schmitt Onset, duration, extent


Peter 1 week Feeding, sleep, irritability
Prenatal, natal and postnatal
Pediatric Hx hematoma, blood group
Mum well being

Patient encounter;

Your actions Suggested verbalizing Patient response

274
Opening start Congratulations on the birth My son Peter looks really
of your son Peter, how can I yellow to me,
help you today? I know babies get jaundice,
but I want to make sure it’s
not worrisome
Sure, I need to ask you few
questions about Peter’s No problem
health to make sure
everything is ok
Onset and progression When did he start looking I noticed his eyes turned
yellow to you? yellow yesterday, but today
his face and neck are yellow
too
And he is one week, right? Yes
Related symptoms Did he have fever? No, I checked his
temperature and it’s 37 ⁰C
Fever Are you breast or formula Only breast feeding
Feeding feeding?
Sleep How often do you feed him? Every 2.5-3 hours
Cry For how long do you feed About 40 minutes
Urine output him each time?
Stool Does he throw up? Not really, he spits up a little
Blood in stool Does he wake himself up to sometimes after burping
Skin rash feed or do you have to wake No he wakes up by himself
him up?
Can you feel your let down? Yes
Is he fussy? No he’s easy to settle

How many times does he A lot, 8-10 times?


poop?
And what color are his Watery yellow with seeds
stools?
And how many wet diapers? About 12 I would say

Did you notice any blood or No


mucous on the diaper? Light yellow
What color is his urine?

Do you have concerns about No


him other than the jaundice?

Peri-natal history I’m going to ask you few


Prenatal questions about your health
during pregnancy

275
Was this your first Yes
pregnancy?
Were there any problem No
when you were pregnant?
And diabetes or high blood No, I’m very healthy
pressure?
Did you take any medications No
during your pregnancy?

Any smoking? None


Any alcohol use A glass of wine before I knew
I was pregnant
Any recreational drug use? None

Do you know your blood A+


group? A+ too
How about your husband?
How many weeks were you 39 weeks
Natal pregnant when you gave
birth? It was normal but the doctor
Was he born vaginally or by C had to use vacuum
section?
Was there swelling of Peter’s Yes, but it’s going down
head?
For how long did your water I would say about 6 hours
break before peter was born?
Was it green, or stained with No, it was clear
blood?
Did you have a fever during No
your labour and delivery?
Do you know whether you
were group b strep positive Negative
Post natal or negative?
Any vaginal lesions when No
baby was being born
Do you know what his APGAR It was 9 and 9
scores were?
What was his birth weight? 7.6 lb
Did you weigh him after? No
Did he need NICU care? No
When were your baby and The next morning
yourself discharged from
hospital?

276
Did your baby have jaundice No
before discharge?
How about fever? No
Development and growth
Does your baby look at you Yes, it’s very lovely
Not much at 1 week- feeding when you breast feed him?
and pooping covered
it’s precious for sure
Vaccination

Nothing at this age unless the


mother is Hepatitis B carrier
Medications and allergy Did Peter receive any They gave him a vitamin K
medications? shot in hospital and some
drug in his eyes
Does he have any allergies? No
Family history

Consanguinity Are you and your husband No


Genetic disorders in the related in any way, other
family than through marriage?
Does any genetic disorders Not that I know of
run in your or your husband’s
families?
Social history

Smoking, alcohol and drug


use by parents covered How are you coping with a It is tiring but we are very
new baby? happy
It can be overwhelming
sometimes
Who helps you care for him? My husband is a very
involved dad, and my parents
are always around
It’s good to hear you have
help
How much time off are you I’m taking a whole year off
taking?
FIFE From what you told me It
seems Peter is doing well,
and has what we call
physiologic jaundice that
some babies develop, it

277
usually lasts for 1 week.
However we need to weigh
him, check his temperature
and I need to examine him to No, thank you
make sure that’s what it is.
Do you have any questions
for me?

Physical exam;

Not required in this case

Sample questions you may be asked by the examiner;

1- What is the significance of head swelling the mother described her baby had?
Answer; It could represent soft tissue swelling or a hematoma. If it was a hematoma,
then the baby is at a higher risk for pathological elevation of bilirubin and kernicterus

2- What other risk factors does this baby have for neonatal jaundice?
Answer; He is exclusively breast fed

3- What investigations would you order?


Answer; bilirubin levels either; serum bilirubin (direct and indirect) or transcutaneous,
CBCD, direct coombs test, Peripheral smear, reticulocyte count

Rating scales points;

- Examinee introduced self and position


- Addressed patient with name
- Used proper non-verbal communication
- Organizational skills
- Examinee spoke clearly (accent didn’t get in the way)
- Good Listening and questioning skills
- Showed rapport with patient
- Attentive to patient comfort

Variations of the case and some tips;

278
1- Case; 8 days old with jaundice

History; term baby born at 38 weeks by normal vaginal delivery after uneventful
pregnancy. The mother is G2P2, and is healthy. Jaundice appeared on day 7, and
is extending to the chest. The baby is exclusively breast fed, the amount of milk
seems to be adequate as his urine and stool outputs are within normal. No other
symptoms and no fever. His older sibling had jaundice during the first week of
life that continued for 3 weeks, he was exclusively breast fed, investigations
were negative and no treatment was required.

P.E; none required

Discussion; This baby may have breast milk jaundice which is seen in some
exclusively breast fed babies for no apparent reason. It usually resolves on its
own. Thorough evaluation is required, and if bilirubin levels are not in the
dangerous zone then the mother is encouraged to continue breast feeding.
Breast milk jaundice is different from breast feeding jaundice. In breast feeding
the baby is not getting enough breast milk, either because of improper latch or
decreased milk supply of the mother. Evaluate breast feeding techniques and
urine output to determine if milk supply is sufficient.

2- Case; 2 days old presenting with jaundice on the day of discharge from hospital

History; Term baby, normal vaginal delivery and uneventful pregnancy. The
mother is primigravida and the baby is on formula as the mother doesn’t wish to
breast feed. Baby’s temperature was 38, and his urine was smelly. The nurse
thinks he is more lethargic than usual newborns. The mother is concerned
something is wrong with her baby and wondering if her choice of not breast
feeding is causing her son’s jaundice.

P.E; not required

Discussion; Sepsis due to UTI or other infections can cause jaundice in newborns.
It is important to do proper evaluation including blood and urine culture and
chest X ray in addition to the investigations mentioned above to evaluate any
newborn with jaundice. Supporting the mother is very important, explain to her
that her choice of not breast feeding has nothing to do with her baby’s jaundice.
It is a good idea to assure the patient that you respect her choice and offer to

279
talk to her about breast feeding benefits when she is ready. If this was a
counseling case then you need to explore the reasoning behind not breast
feeding, it could be she can’t take time off from work or school in which case you
can involve the social worker.

3- The following guidelines provide a good summary of neonatal jaundice;


http://www.nice.org.uk/nicemedia/live/12986/48679/48679.pdf

280
Case 17; A toddler with a cleaning agent into his eyes
Edited by; Dr. Abbeir Hussein

Door sign

You are about to see Mary Duggan, Mother of 3 years and 2 months old John Duggan. John accidently
got a cleaning agent into his eyes today. His mother called 911, and he is now being evaluated by
ophthalmology.

Vital signs

BP 90/60

PR 100

RR 30

Temperature 36.9 ⁰ C

In the next 10 minutes, obtain a focused and a relevant history. At 9 minutes the examiner may ask you
a question or questions.

Note; The NAC exam may have a similar stem but the last 2 lines may look like this; In the next 11
minutes obtain a relevant and focused history. At 8 minutes the examiner may ask you a question or
questions

Sample notes to write for yourself

Mary, mother What, when how, how much

John Duggan 3 years both or one eye

Any ingestion? Other injuries

Cleaning agent into eyes Symptoms, intervention

EMS assessment

281
Hx Pediatric history, screen for abuse

Patient encounter

Your actions Suggested verbalizing Patient response

Opening start I’m sorry about what The patient looks at you and
happened to john cries hysterically

I’m a bad mother, my son is


going to be blind

You offer a tissue; Patient sighs, and after about


10 seconds (seems like
I know this is quite stressful, eternity in exam settings!)
but he is getting examined by says; I hope he doesn’t lose
the eye doctor now, and we his sight. I don’t know how I
should have a clearer idea forgot to hide Lysol
about the extent of his injury
soon

You allow a moment of


silence, then say; We will do
our best, he is in good hands

Details about the incident Do you mind if I ask you few


questions?
No, go ahead

Can you tell me about what


happened? John is a good boy, always
trying to help, and likes to do
everything mommy does. I
forgot Lysol open in the
bathroom, he sneaked in and
managed to spray some on
the floor, then it looks like he
tried to wipe it with his bare
hands. I heard him screaming

282
and ran upstairs to find him
rubbing his eyes in pain

Patient bursts in cry again

It must be hard for you

You offer tissues and after a


few seconds ask; Do you feel Yes, I want you to help him
good enough to continue?

We will do our best, I promise


It’s hard to tell, a table spoon
How much do you think he maybe
sprayed?

Did he touch both eyes?


Yes
When you saw him, were his
Yes
eyes red? And tearing?
I don’t know, I hope so
Was he able to see?

Do you think he ingested It’s hard to tell if he put his


hand in his mouth before I
some of it?
reached him

Do you think he fell down or


had other injuries? No

The bathroom cleaner, I


What type of Lysol?
brought the bottle with me

Good, I will like to look at it,


to see what exactly it has in it

What did you do when you


got to him? I rinsed his eyes with water
for about 10 minutes then I

283
called 911, who instructed
me to rinse even more

You did the right thing

They checked his heart and


What did the emergency blood pressure and
responders do when they continued to rinse his eyes
arrived? all the way to the hospital

How long did it take to get to I think the whole thing was
the hospital? about 30 minutes

Did John throw up? No


Did he have difficulty No
breathing?

Did he complain of pain other


than in his eyes? No

Did John have any other He is a very active boy, he


injuries in the past? had occasional falls and one
small cut wound but nothing
major

Has anything like this ever No


happened before?

Perinatal history Do you have other children? No

Were there any concerns or


problems with the
No
pregnancy?

Was John early, late or on


time? On time

284
And when he was born, were No
there any problems?

Did he need to go to the


NICU? No he went home right away
with me
As a baby did he have any
No, he is a healthy boy
problems?

Was he ever admitted to the


hospital? No
Does he have any medical No
conditions?

Vaccination Has he received all of his Yes he is up-to-date


immunizations?

Nutrition What is John’s diet like? He loves food, I make sure


he gets enough meat and
veggies every day, he also
drinks milk

Growth and development Are you still doing ok? I want to know what does
the eyes doctor think

I will let you know as soon as


he is finished

Do you mind if I ask you a few


questions about his
Sure that’s fine
development to make sure
everything is ok?

Do you have any concerns


about his development?
No

Can he stand on one foot?


Yes, only for few seconds
Can he ride a tricycle?
Yes

285
Can he draw a circle? Yes, and a sketchy man too

Wow, that’s impressive

Do you have any concerns No


about his speech?

How much of what he says do


At least 75%
you think a stranger could
understand?

Does he know his body parts? Oh ya, it’s his favorite game
Does he play with other kids? Yes, he goes to the day care
3 times a week, and he loves
it
Do you ever see him doing
any pretend play, like for I’ve seen him do that a few
times
example pretending to feed a
baby?

Family history Are there any diseases that No


run in the family?

Medications and allergy Do you give John any No


medications?

Does he have any allergies?


Not that we know of

Social history I’d like to ask you a few Sure


questions now to get to know
more about John and his
family if that’s okay My husband, myself and
John
Who currently lives at home?

Who looks after John most of I do


the time?

Does he go to daycare or No he stays at home with me


preschool?
Yes
Is your home childproof?

286
Where do you typically keep It’s usually locked in a
the Lysol? bathroom cupboard he can’t
reach
What do you and your family
do for work? My husband works full time,
he is a lawyer, I work part
time in a salon

Does anyone in the No


household smoke?

Do you feel John is being


treated well by all people Yes
looking after him?

Closure Thank you Mrs Duggan for No, thank you, please let me
sharing all this information know what the eye doctor
with me. I will go check on thinks
John and let you know how
he is doing. Do you have any
other questions or concerns?

Physical exam;

Not required in this case

Sample questions by the examiner;

1- If the cleaning agent was an alkaline, would you attempt to neutralize it with an acid? And
why?

Answer; No, because the heat resulting from the chemical reaction can burn the eyes and make the
injury worse

287
Rating scales points;

- Examinee introduced self and position

- Addressed patient with name

- Used proper non-verbal communication

- Organizational skills

- Recognized and managed emergency effectively prioritizing actions

- Examinee spoke clearly (accent didn’t get in the way)

- Listening and questioning skills

- Showed rapport with patient, and was sensitive to patient feelings

- Was supportive and non-judgmental

- Attentive to patient comfort

- Sensitive and respectful of patient’s concerns

Variations of the case and some tips;

1- Case; You are the doctor on call. At 2 am you got a call from a freaked out mother
whose daughter fell off the bed

History; parents co-sleep with their daughter. The 2 years old moves a lot during her sleep and managed
to cruise to the foot of the bed, she fell off their 3 feet bed, and most likely bumped her head, No loss of
consciousness, no vomiting. She cried for few minutes but her mother was able to calm her down. The
mother is wondering if she should take her to the hospital for a head CT. There are no other significant
accidents. She is otherwise healthy with normal growth, development, up to date vaccination and
satisfactory nutrition. The only medication she is on is vitamin D

Discussion; Falls remain a major part of injuries in kids. Since it is a phone call, the first thing you need to
do is to document the mothers name, phone number and home address in case the line got

288
disconnected. Then reassure the mother that help is readily available. Ask the mother to keep her
daughter up for a little while and watch for symptoms. The main concern is head injury and/or fracture.
Ask the mother to watch for seizures, change in level of consciousness and vomiting. If a serious injury is
suspected then the child needs to be brought to the hospital. The mother or the father can drive if calm
enough, if not then you need to send an ambulance. If the daughter is unstable it is as well preferable to
call an ambulance. Ask if one or both parents know CPR or first aid skills, and stay on the line till
ambulance arrives. If the daughter seems stable with no serious injury there is no need to bring her to
the hospital. You may later on counsel the mother about safe sleep practices.

The following website has the most recent Canadian recommendations;

http://www.cps.ca/documents/position/safe-sleep-environments-infants-children

2- Case; 4 months old boy brought in by his mother because of bruising.

History; the mother says her son has been easily bruising for the last week. He has bruises on the face,
neck, trunk and extremities. No blood in stool or urine and no bleeding mucus membranes. The baby is
feeding well; his activity level and sleep are unchanged. No fever, no seizures, no sick contacts and no
congestion. The mother however looks exhausted. Her husband works out of town, and has been away
for 6 weeks. Her parents are out of town, they helped the first month the baby was born but then she
was on her own. She says she is burnt out and is driving her son to a day home for a couple of hours
every day for the last 2 weeks so that she can breathe! Perinatal history was normal. Vaccinations are up
to date. Growth and development are within normal. The mother is a teacher, planning to take a year
off. Neither her nor the father smoke, drink, or do drugs.

Physical exam; make sure the room is warm and inspect the whole body documenting the distribution,
size, and color of bruises. Look for other injuries. Inspect the mucus membranes. Auscultate the heart
and lungs and palpate the abdomen. Examine the tone and primitive reflexes. Do fundoscopic
examination. Palpate the limbs for swelling or broken bones. And of course check the vital signs.

You will not be asked to examine a baby. In my mind I don’t see why not to examine a dummy, in which
case treat it as a real baby, and always wash your hands and be gentle.

Discussion; bruising in a non- cruising baby is suspicious for child abuse. However, medical conditions
like thrombocytopenia must be excluded. Your job as a physician is to take care of medical conditions
and injuries and to make sure the child is safe. Use non- judgmental language; you may say; we need to
do some tests to make sure your baby doesn’t have a disease causing all these bruising. We want to

289
make sure as well that your child is not being hurt by someone. If you highly suspect abuse after the
investigation results are back, admit the child and call the social worker and child protective agency.
Never accuse or blame parents or anyone else.

In this case, the baby may simply have a medical condition. However, he may as well be abused. Care
giver exhaustion and a new care giver are important factors. Investigations include; CBCD, PTT/INR,
fibrinogen, ALT, AST, urine analysis and skeletal survey.

If the patient was a toddler, take permission from the parent to talk to him//her in private. Parents are
usually ok with it. Refusal raises suspicions for abuse.

3- Injuries in kids are varied; falls, trauma, foreign body ingestion, poisons ingestion, etc….
Take a detailed history of current and previous injuries ask about risk factors for abuse.

290
Case 18; Motor vehicle accident (MVA)
Edited by; Dr. Abdullah Saleh

Door sign;

David Smith, 23 years old man, brought in by EMS after being involved in a motor vehicle accident.

Vital signs;

BP 100/70

PR 100

RR 16

Temperature 37 ⁰C

There is a nurse in the room. In the next 10 min, manage the patient

Note; In this case you will be communicating with the nurse and the patient. Communication with the
nurse is presented in italic. You need to make sure the patient is stable, give emergency medications,
take a focused and relevant history, perform a focused and relevant P.E and order investigations. The P.E
is included in the case. Treat the nurse with respect, and give clear orders specifying the dose and route
of medications.

Even though it is an ER case, it is important that you introduce yourself to the nurse and patient.

The NAC may have a similar stem but you have 11 min to complete the case

Sample notes to write for yourself;

David Smith, 23 yo Male

Mechanism of injury, SAMPLE hx

Primary Survey: Vitals, ABCDE

Adjuncts to primary survey (CXR, Pelvic Xray, C-Spine imaging, FAST, foley, +/- NG)

Secondary Survey (Head to toe PE)

Vitals recheck

Blood work

291
Patient encounter

Your actions Suggested verbalizing Patient or nurse response

Obtaining relevant hx from Good morning, can you The accident happened 1
nurse/paramedics please tell me about the hour ago, he was hit from
mechanism, time of incident, behind at 40 km/h when he
who brought him in and what stopped to avoid hitting a
Need to ask specifically about interventions were done in deer, he was the driver, and
the details if not volunteered the field there were no other
by the nurse/paramedic passengers and no
pedestrians. He was belted,
no LOC, and the airbags did
not deploy. There was no
Mechanism of injury
significant intrusion and no
SAMPLE; delay in extrication by the
paramedics and firemen. He
S; signs and symptoms was placed in a c-collar and 2
large bore IV’s were started
A; Allergies
at the scene and he was
M; Medications given 1 L of NS.

P; Past medical and surgical


history

L; Last meal

E; events and environment


leading to injury

Intro to the patient Hi Mr. Smith, I’m Dr-----I’m


going to take good care of
David Smith
you, What is your name?

292
I’m going to examine you and
ask questions at the same
Sure
time, is that ok?

Do you remember what Pt recounts mechanism


again.
happened? Can you tell me?

Vitals and ABCD (Primary What are his vitals? BP 100/70, PR 100, RR 16,
Survey) Temperature 37 ⁰C, O₂ sat
97% on 2L via nasal prongs
His GCS is 15/15 (EVM)

The patient is in an
adequately sized and
positioned C-Spine collar. He
is speaking full sentences. Can
you open your mouth for me
please?

The airway is patent and he is


protecting it. He is breathing
comfortably and on
inspection chest rise is equal
and bilaterally. No
paradoxical movements. No
bruises are seen. On
palpation, no subcutaneous
emphysema is felt and no
deformities notes Chest is not
tender. On auscultation I hear
good AE bilaterally, no
crackles or wheezes. I’m
feeling the pulse centrally, it’s
strong and regular at 110
bpm. Peripheral pulses are
present and palpable x 4.

To the nurse, He is
tachycardic.
Can we get 2 large pore IV
Will do
cannulas (if not already
present) and run 2 L of NS and

293
let’s make sure he is Typed Sure
and Screened.
Assessment of vital signs Will do
Repeat vitals after bolus

Can you check the vitals every


IV lines in, Blood requested
15 min.

Can you keep the patient


NPO, and run normal saline Will do
0.9% IV at 150 cc/hour

Sir, do you have pain


anywhere? At the back of my neck

How severe is the pain on a about 4


scale from one to 10, 10
being the worst pain ever?
I’m good for now
Do you need a pain killer?

I will perform a gross motor


and sensory exam now

Can you left your right arm


up, excellent, now your left.
Squeeze my fingers, flex your
elbow, now extend.

Can you wiggle your toes?

(Test the power of abduction


of the shoulders, flexion and
extension at the elbow, hips
and knees)

Do you feel numbness or


tingling anywhere in your
body?

Can you feel me touching


your skin?

294
Let me know if you feel it
every time I touch you with it
and if it feels the same on
both sides (examine touch
sensation)
Exposure
GCS 15, no apparent
disability.

The patient will need to be


exposed and I will do a full
examination from head to
toe, then roll the patient
(with help) while protecting
Examining Power (refer to the C-spine and palpate the
chapter 3 for details) spine while inquiring from the
patient if there is any
Sensory exam (Refer to
tenderness.( Examiner says ;
chapter 3 for details)
move on) A DRE will need to
be performed – to check for
tone, sensation and blood per
rectum. Examiner says move
on

(Full head to toe exam and


ROS) H&N (pupils, fundoscopy,
otoscopy), CVS, Resp, Abdo,
Pelvis for stability, MSK,
Neuro (see Physical Exam
section for more detail)
BP 120/80, PR 95, other
Repeat vitals ?
vitals unchanged
Can I get the following
investigations done please:
Yes
CBC, Creatinine, urea, Lytes,
INR/PTT, AST, ALT, ALP,
Investigations
Lipase, Calcium, Type and
Screen,

12 lead ECG,

295
Portable CXR

Cross table lateral C-Spine X


ray, pelvis x- ray
Will get lab and x-rays

History So, Mr smith, I need to ask ok


you few more questions

Mechanism of injury ( for


additional details not Were you speeding? No
covered yet) There is some damage but
Was your car damaged? not too bad
SAMPLE;

S; signs and symptoms


What did you do after the I had this pain in my neck, I
A; Allergies accident? called 911,

M; Medications The ambulance guys placed


this thing around my neck
P; Past medical and surgical
history And tested my nerves just
like what you did
L; Last meal
Were you wearing your seat
Yes
E; events and environment belt?
leading to injury I was alone in the car, the
Were other people injured in
other driver seemed ok
the accident?

Do you have pain in your


chest? No
Is the patient intoxicated?
Are you still doing ok? Yes
Smoking
Do you have headache? No
Family history of bleeding
diathesis Abdominal pain? No

Pain elsewhere? None

296
Do you feel short of breath? No

Did you lose consciousness No


after the accident?

Did you have a seizure?


No
Were you under the effect of
No
alcohol or drugs when the
accident happened?

Have you ever been


diagnosed with a medical
disease or had a surgery? No, I’m healthy

Do you take any medications? None

Do you have allergy to No


medications or food?

When did you last eat?


I had lunch 6 hours ago

Do you smoke?
No
Does anyone in your family
No
have a bleeding problem?

Summary and re assessment We have a 23 years old


healthy male involved in
MVA, with possible whiplash
injury as indicated by the
accident mechanism, midline
posterior neck tenderness
and neck pain. No
neurological deficit and no
evidence of other injuries.
Investigations including
Lateral –spine X-ray are Vitals unchanged
pending. Can we get a
surgical consult and another
set of vital signs please?

297
Sir, would you like me to It would be great if you can
contact your family or call my parents, but please
friends? don’t scare them

Sure I will, is there anything I No thanks


can do for you?

Physical examination;

Included in the case

Sample questions you may be asked by the examiner;

None in this case

Rating scales points;

- Examinee introduced self and position


- Addressed patient with name
- Used proper non-verbal communication
- Organizational skills
- Recognized and managed emergency effectively prioritizing actions
- Examinee spoke clearly (accent didn’t get in the way)
- Listening and questioning skills
- Showed rapport with patient, and was sensitive to patient feelings
- Was supportive and non-judgmental
- Attentive to patient comfort
- Medical knowledge adequate
- No misinformation was provided to patients
- No concerning ethical/legal issues
- The examinee is respectful of other health care members

298
Variations of the case and some tips;

1- Case; 28 M, brought in by EMS after being involved in MVA, manage

Management; you start with your ABCD, the patient appears intoxicated and smells of alcohol. GCS is 8,
you decide to intubate, the examiner says done, so you assume that the patient is intubated. Since his
airway is secure and protected with the endotracheal tube, you inspect, palpate and auscultate the
chest for breath sounds. There is a large bruise on the R chest wall and you feel subQ emphysema. On
auscultation, you hear decreased breath sounds on R as compared to L hemithorax. The Resp therapist
tells you that it is difficult to ventilate the patient and he is becoming hypotensive. A quick examination
of the patient’s neck reveals a deviated trachea to the left and distended neck veins. You diagnose a
tension pneumothorax. The diagnosis is made clinically and there is no need to delay while waiting for a
CXR. You perform a needle decompression (2nd intercostal space, midclavicular line) and a gush of air is
released and the patient becomes easier to ventilate and normotensive. A 36 Fr Chest Tube is placed on
the Right side (5th intercostal space, anterior or mid-axillary line), a gush of air is released and blood
pools in the pleurovac. A CXR is obtain for confirmatory placement. Make sure to check the left side as
there might be a penumo bilaterally. Restart the primary survey from the beginning again. Obtain 2
large pore IV cannulas, and start IV fluids (if not already done). Continue with the circulatory
assessment, disability assessment will be difficult as the patient is intubated and presumably sedated.
The exposure should take place and the log roll + DRE. Continue to monitor the vital signs. You order
investigations; ABG, Blood glucose, CBCD, INR, PTT, AST, ALT, ALP, albumin, Calcium, TSH, lipase, CK,
blood alcohol levels, urine drugs screen, urine analysis, ECG, c spine films, pelvic xray and CXR (if not
already done). You need to consider the other adjuncts to the primary survey (foley – if no blood at the
meatus, and NG tube). Not much history is available; EMS reported finding the patient unconscious
behind the wheel, after his car struck a tree. He was wearing the seat belt. You look for a medical
bracelet and don’t find any. You start your secondary survey and physical exam. The pelvis feels
unstable. The patient is becoming hypotensive and tachycardic. It is important at this point to think of
shock and the different types of shock. In most trauma patients, hemorrhagic hypovolemic shock is the
most common. The five places patients lost blood are: thorax, abdomen, pelvis and retroperitoneum,
fractured long bones and the floor (as in bled on the scene or from scalp lacerations etc.). The xray looks
like an open book pelvic fracture and you immediately should proceed to bind the pelvis to decrease the
volume and hence attempt to tamponade the bleed. You give 2 L crystalloid and assess if patient
responds. If he continues to be hypotensive give blood. You perform a FAST screen, and consult surgery.
You ask the nurse if there was any information in the patient wallet or any numbers that you can
contact, she tells you that his mother number is the last number dialed on his phone, you say that you
will call his mother.

299
2- Note; the case above can be made even more complicated by getting the patient to fake
a seizure. Do your ABCs, make sure the patient is safe, give 2 mg Valium or versed IV,
and a loading dose of phenytoin (15-20 mg/kg, max rate 15mg/h). The seizure could be
secondary to head injury or the effect of drugs or alcohol. Order a head CT in addition to
the above investigations and recheck the pupils, DTRs, tone and Babinski sign.

300
References;
1- Macleod, John. Macleod's Clinical Examination. 12th ed. Edinburgh: Churchill
Livingstone/Elsevier, 2009.
2- Hui, David. Approach to Internal Medicine a Resource Book for Clinical Practice. 3rd ed. New
York: Springer, 2011.
3- Sabatine, Marc S. Pocket Medicine. 4th ed. Philadelphia: Wolters Kluwer Health/Lippincott
Williams & Wilkins, 2011.
4- Stephanie H Abrams, MD, MS, Robert J Shulman, MD. Approach to neonatal cholestasis. In:
UpToDate, Topic 5941 Version 19.0, UpToDate, Waltham, MA. (Accessed on November 20,
2014.). http://www.uptodate.com/contents/approach-to-neonatal-
cholestasis?source=search_result&search=neonatal+jaundice&selectedTitle=2~68
5- "Guidelines for Detection, Management and Prevention of Hyperbilirubinemia in Term and Late
Preterm Newborn Infants." Guidelines for Detection, Management and Prevention of
Hyperbilirubinemia in Term and Late Preterm. Canadian Pediatric Society. Accessed November
19, 2014. http://www.cps.ca/documents/position/hyperbilirubinemia-newborn
6- Mimi Zieman, MD.Overview of Contraception. In: UpToDate, Topic 5459 Version 83.0,
UpToDate, Waltham, MA. (Accessed on November 25, 2014.).
http://www.uptodate.com/contents/overview-of-
contraception?source=search_result&search=contraception&selectedTitle=1~150
7- "Pocket Guide to COPD Diagnosis, Managemnet, and Prevention." Global Initiative for Chronic
Obstructive Lung Disease. Accessed July 30, 2014.
http://www.goldcopd.org/uploads/users/files/GOLD_Pocket_2015_Feb18.pdf
8- "The 5A's of Breif 3-5 Minutes Tobacco Intervention." Saskatchewan Dental Therapists
Association. Accessed August 24, 2014.
http://www.sdta.ca/mrws/filedriver/DentistTobaccoInterventionAlgorithmSept06.pdf
9- Phyllis August, MD, MPH, Baha M Sibai, MD. Preeclampsia: Clinical features and diagnosis In:
UpToDate, Topic 6814 Version 58.0 , UpToDate, Waltham, MA. (Accessed on September 20/
2014.) http://www.uptodate.com/contents/preeclampsia-clinical-features-and-
diagnosis?source=search_result&search=preeclampsia&selectedTitle=1~150
10- Errol R Norwitz, MD, PhD, John T Repke, MD, Preeclampsia: Management and prognosis. In:
UpToDate, Topic 6825 Version 58.0 , UpToDate, Waltham, MA. (Accessed on September 20/
2014.) http://www.uptodate.com/contents/preeclampsia-management-and-
prognosis?source=search_result&search=preeclampsia&selectedTitle=2~150
11- Annekathryn Goodman, MD , Postmenopausal uterine bleeding In: UpToDate, Topic 5421
Version 10.0 UpToDate, Waltham, MA. (Accessed on October 25, 2014.)
http://www.uptodate.com/contents/postmenopausal-uterine-
bleeding?source=search_result&search=post+menopausal+bleeding&selectedTitle=1~33

301
Chapter 6
Ethical and Legal
Principles
Edited by; Dr. Fraulin Morales

302
Introduction;
The goal of this chapter is to touch on some basic ethical and legal issues. Culture plays an
important role in shaping what is ethically acceptable and what not. The ethical principles
reviewed here represent Canadian values. You need to understand your own patient’s values
and work towards his/her best interest. For example; while the principle of autonomy dictates
your patient makes her own decision, you may treat a patient who is willingly giving this right to
another person, like an elderly depending on his nurse daughter to make the best decision for
him. That is ok as long as the patient is not being abused or coerced. Nevertheless, you should
try your best to involve your patient.

Ethical principles are guidelines, and some cases can get complicated. The good news is there is
an ethics committee in most hospitals. And if a committee is not available, there is always a
more experienced consultant. When uncertain, ask for an ethics consult.

I strongly encourage you to read Doing Right, 3rd edition by Philip C. Hebert. It’s a very useful
book. It covers basic and controversial ethical issues in a case based approach that motivates
your own curiosity. Follow the following link for the Canadian Medical Association code of
ethics

http://policybase.cma.ca/dbtw-wpd/PolicyPDF/PD04-06.pdf

The College of Physicians and Surgeons of Alberta standards of practice can be found by clicking
on the link below;

http://www.cpsa.ab.ca/AboutUs/FAQs_Main/FAQs_For_Physicians/Standards_of_Practice_FA
Qs.aspx

As you read through this chapter, I advise you to reflect on your own values and experiences
with patients. I personally revert to my patient’s welfare whenever there is a conflict. It makes
it easier to go back to basics when faced with a dilemma.

All Canadian trainees and independently practicing doctors are required to register with the
Canadian Medical Protective Association (CMPA). CMPA provides legal advice to its members
and helps them out when a complaint is filed against them. CMPA website contains useful
information for your reference; www.cmpa-acpm.ca

This chapter covers basic ethical expectations and concepts;

- Confidentiality

303
- Autonomy, informed consent, capacity, substitute decision maker and power of
attorney
- Beneficence and non-maleficence
- Justice
- Dealing with ones’ own and colleagues’ errors

Confidentiality;
Patients share their most personal information with their doctors trusting that their privacy is in
good hands. Doctors must strive to protect their patient’s information. The following situations
illustrate how to practically apply this principle;

1- During a patient encounter, always ask the patient if it is ok for family members to stay
in the room. Repeat your question when you are about to examine your patient. Look
for non-verbal cues, and if you need to ask very personal questions; politely ask family
members to leave the room.
2- Cultural differences may arise. In some cultures males are more dominant. One of my
patient’s husbands answered all the questions for her and she was ok with it. If the
patient agrees, this is acceptable. However, in certain situations you need to make sure
you talk to your patient in private, like when treating teenagers or when spousal or
elderly abuse is suspected; or when you feel your patient is shy to ask family members
to leave.
3- You will encounter caring family members asking about your patient’s health. Always
take your patient’s permission before disclosing any information. In most circumstances,
it’s better to talk in the presence of the patient.
4- Don’t talk about patients in elevators or restaurants or other public spaces; if you
absolutely have to discuss a case, don’t mention names, and keep your voice low.
5- Discard all papers with patient information appropriately by putting them in a shredding
box not the regular garbage.
6- Protect your patient’s information when using electronic health records; always log off
and protect your password. If you exchange emails with or about patients make sure to
use appropriate email protection. Avoid saving patient’s information on your lap top,
and if you have to, use a security password, and delete information you no longer need.
7- If a patient asks for a letter for his/her work, don’t include any personal information,
simply say; medical reasons. An example of a letter to employer is found in chapter 4:
writing and counseling tips.

304
8- There are some circumstances that require you to break confidentiality and share
patient information with others; this is when your patient or someone else is at risk.
Examples include: a homicidal or suicidal patient, certain infectious diseases like sexually
transmitted infections or infections you have to report to public health. Inform your
patient that you have a duty to disclose this information to protect him/her or others.
9- A final word about medical records; the original record belongs to the treating physician
office or hospital, but the patient has the right to have a copy of his/her own chart. The
physician can provide the copy for free or charge a fee.

Autonomy, capacity, informed consent, substitute decision maker


and power of attorney;
I grouped all these concepts together because they all come back to the same point: the
patient’s right to make his/her own health decisions. The following illustrate these principles:

1- The patient has the right to make his/her own health decisions as long as he/she is
competent to do so.
2- To be able to exercise this right the patient must be well informed. It is the physician’s
duty to fully inform the patient about his/her condition, available treatments, side
effects and benefits of treatments, outcome with and without treatment and treatment
alternatives. Treatment can include drugs, surgeries, life style modifications or a
combination of these. In some situations like a patient with cancer considering
chemotherapy, the amount of information can be overwhelming and it’s a good idea to
give the patient handouts to read through. Patients may need some time to consider
their options. In cases where imminent intervention is needed, explain to the patient
that not much time is left, but don’t pressure or force your patient to make a decision.
3- To be able to make a decision, the patient must have decision making capacity. Capacity
means the patient understands his/her options, and the consequence of taking an
action or not doing anything. The patient must be mentally clear; not delirious nor
demented nor suffering from a psychiatric condition that deters his/her ability to make
sound decisions like depression, psychosis or being actively suicidal. A competent
patient understands his/her choices and their consequences and is able to appreciate
the seriousness of his/her condition.
4- A competent patient has the right to make his/her own health choices even if they seem
irrational to the health care provider, and doctors must respect them. For example, a
patient with localized rectal cancer that can be cured by surgery refuses surgery and
uses homeopathic remedies instead. In these situations make sure the patient is capable
of making the decision, and explore his/her concerns in a non-judgmental way.

305
5- Only the patient can decide if family members can be involved in decision making. This
may be different among cultures. I had a Jehovah witness patient who had a third
degree burn. His hemoglobin was 6 and his platelet count was low. His wife strongly
refused blood transfusion. When I talked to him in privacy he said he wasn’t that
religious and wanted to have the blood but was afraid to upset his wife. He finally
decided to take the blood and asked us not to inform his wife. Always speak to your
patient privately and make sure they are not being pressured or coerced. It’s ultimately
up to the patient to decide how much family and friends know about his/her health and
the extent of their involvement.
6- I want to expand more on cases where a patient with a life threatening condition
refuses a potentially lifesaving treatment. In addition to explaining to the patient his/her
options, alternatives, benefits, side effects and outcomes must also be fully discussed.
Explore his/her concerns. Patients often have just reasons but may be willing to change
their mind if encouraged to open up (as in case 15, chapter 5). It is important as well to
discuss with the patient the goals of care. Things that may come up include; would you
treat an infection? What about if the patient needs resuscitation? In a patient who
refuses a lifesaving treatment would you offer ICU care? For example; is it ethical to
resuscitate a patient with renal failure who refuses dialysis? The patient will die without
dialysis and there is no point of trying to bring him/her back temporarily, this will only
increase suffering. The doctor has the right to refuse to resuscitate in this case, and the
patient must be fully informed upfront. A lot of patients change their minds when goals
of care are discussed. Perhaps they realize the seriousness of their situation. No matter
what your patient decides, be always supportive and compassionate.
7- Situations arise where a previously competent patient loses the capacity to make
decisions. Follow the patient’s previously expressed wish if known. If unknown look for a
substitute decision maker. The substitute decision maker may know the patient’s wish,
or act in the patient’s best interest if this wish is unknown.
8- Who can act as a substitute decision maker? It is wise to ask the patient about his
wishes and who would he want to act on his behalf should he become incompetent in
advance. In most cases the spouse or another close family member is the substitute
decision maker. If no one is found, the court may appoint a representative.
9- In Canada, a lot of patients have an advance directive or another legal document like a
power of attorney or living will that states their wishes and who they would like to act
on their behalf. Always ask if your incompetent patient has one and encourage your
seriously ill competent patient to obtain one.
10- Note that the patient autonomy is limited in cases where there might be harm to
him/her or others. The best example is the involuntary admission of suicidal psychiatric
patients to hospital. Patients are also treated without consent in emergency situations.

306
11- What about children? In most cases the guardian(s) (usually the parents) makes health
care decisions on the behalf of the child. Situations arise where teenagers ask you to not
involve their parents. It is expected that you protect their confidentiality in certain
situations like treating sexually transmitted infections or prescribing birth control. The
situation becomes, however, more complicated when it comes to a child refusing life
sustaining treatments and refusing parents involvement. As with all cases; explore your
patient concerns in privacy and try to understand their reasoning. Some children are
mature enough to be labeled competent to make their own health decisions. Always ask
for ethics consult in such cases, you may as well seek a legal advice.

Beneficence and non-maleficence;


Acting in the patient’s welfare and doing no harm may seem like straightforward concepts.
Nevertheless, there are situations where these ethical principles are challenged.

1- With the advancement and expansion of medical knowledge and therapies, one must be
thoughtful about treatment offered to patients. The physician has a duty to inform
his/her patient about his/her options, their side effects and the consequence of each.
Treatments have side effects that must be weighed against benefits. In general,
treatment shouldn’t put the patient in a worse condition. This may be hard to predict
sometimes, and it’s the physician’s duty to fully inform the patient and recommend the
best option available. And as mentioned earlier, respect the patient’s decision.
2- Side effects of therapy can be physical, mental, emotional and social. We tend to focus
on physical side effects while others may be more significant to the patient.
3- Explore your patient’s values and concerns to avoid unsuspected emotional or social
harm. There are resources that can be utilized to help the patient out. I admitted a
patient with heart failure, and while I was focused on his physical wellbeing, he was
extremely worried. When asked, he said he paid parking for only 2 hours and was very
concerned his car may get towed. He was going through financial difficulties and
couldn’t afford additional expenses. I got the social worker involved, his car was moved,
and he didn’t have to pay for one week parking, or towing expenses. This was a big relief
for him, and his emotional wellbeing helped him get better. In this case patient
admission to hospital caused a stress that was easily identified and removed.
4- Harm does ensue sometimes as in the case of a patient with arrhythmia who was put on
Amiodarone that caused lung fibrosis. It is hard for us as physicians to predict all
possible side effects. However they do happen and it helps to remember that drugs are
prescribed with the best intensions. Make sure the risks and side effects are less than
that of benefit. The patient must be fully informed about his options which in this case

307
include more sophisticated electrocardiac interventions, or an alternative medication. If
you know the percentage of patients who get the side effect, mention it. The patient
can then choose what feels best for him or her.
5- Physicians should take care of themselves so they don’t get harmed when treating
patients. An example includes using appropriate infection control precautions.
6- What if parents refuse a lifesaving treatment for their children? Like when a Jehovah
witness refuses blood transfusion of a bleeding kid. The physician can override the
parents in these cases and order blood. If the child was mature enough, involve him or
her in making the decision. If the child refuses blood and is deemed competent, then
respect his/her wish and try your best to treat him/her with alternatives. If in doubt ask
for help.

Justice;
Justice in medicine can be viewed from different perspectives;

1- Physicians should use their time wisely, so that patients are given good care, yet at the
same time the physician is not spending a lot of time unnecessarily with a single chatty
patient while others are waiting to be seen.
2- Physicians should be mindful of resources. It is imperative that patients are provided
with the best care and needed investigations are ordered even if expensive. However,
physicians should try their best to avoid ordering unnecessary tests as this exhausts the
system and may delay other, sicker patient’s access to them. An example is ordering
daily CBCD in a stable non bleeding patient admitted with COPD exacerbation. Health
care in Canada is publically funded, and patients have to wait sometimes. One can
appreciate the importance of wise utilization of time and resources.
3- Physicians must treat all patients with respect and grant them all access to high quality
care without discrimination based on age, sex, color, ethnicity, religion, sexual
orientation or social class.
4- Physicians should not abuse the system. An example would be a doctor using his
connections in the emergency room to get his relative seen sooner. This means longer
waiting times to other patients and can create a delay in seeing critically ill patients
(especially if a lot of health care providers do it).
5- Physicians should keep their relationship with the industry professional and prescribe
drugs and services based on scientific evidence. It is unprofessional to accept expensive
gifts from pharmaceutical companies like a prepaid family vacation. It is ok to listen to
marketing and weigh pharmacist’s evidence as long as the focus is the patient best
interest and no secondary gain is involved.

308
Dealing with ones’ own and others errors;
It is said that if you don’t make mistakes, you do nothing! Even the most skilled and careful
doctors make mistakes. But our job in Medicine involves caring for sick people. Mistakes can be
small and non-significant or cause direct harm to the patient. The following points illustrate
these concepts;

1- A duty of care to patients principle covers; “doing no harm” under its umbrella.
2- Negligence is a breach to the duty of care that results in harm to the patient.
3- The following example illustrates the concept of negligence; Doctor A fails to review the
blood work for one of his patients. Luckily, his patient results didn’t require an
intervention and no harm was incurred. A case of negligence can’t be made. Doctor B
fails to review his patient’s blood work. His patient had hyperkalemia that was missed.
He had a cardiac arrest few days later as his potassium levels went even higher. Medical
therapy was not enough, and in addition to the pain of resuscitation, he needed dialysis.
Doctors B scenario is a strong case of negligence.
4- Note that adverse events of medications and procedures should be well explained in
advance when obtaining consent. They are not considered negligence if they occur.
5- It is a difficult situation when a doctor makes a mistake. Yet it happens. A lot of negative
feelings and blame may occur, and the doctor may feel embarrassed and try to cover
up. One may wonder what to do in these situations? It all comes back to patient
welfare. One should be professional and do what is right;
 Be honest; Patients deserve to know the truth.
 The more serious the harm is, the more important it becomes to act quickly.
 When you inform your patient of the mistake, be straightforward, apologize but
avoid blaming yourself or others, allow the patient to ventilate his/her anger or
frustration, focus on what needs to be done to help your patient and to minimize
the harm that resulted from the mistake.
6- Revise the system you work within and see what can be done to prevent this mistake
from happening in the future. Inform your patient that you will do your best so it
doesn’t happen again to him/her or other patients.
7- If you work with a group of physicians, and the mistake could have been prevented by
better communication and coordination, talk to your colleagues about it, so that you all
work together on a solution and preventative measures.
8- Honesty and acting quickly to help patients are a big relief for doctors, as it is not easy to
see patients suffer as a result of an error.
9- Physicians may worry about law suits. However, patients are less likely to sue if
informed in a timely and respectful matter. Waiting to disclose can impose more harm
and make penalties worse.

309
10- Doctors should contact CMPA in cases of law suits or to ask for advice in difficult
situations.
11- When a team of doctors treat one patient, there is usually one most responsible
physician. If the patient had a significant problem or lab result, you must do something
about it. If it is not within your scope of practice, you should inform the most
responsible physician, who will then take the appropriate action. It is a good idea to
write a letter to all treating physicians. In all cases, you should inform the patient.
12- What if your patient complains to you about another doctor?
Stay professional. Don’t take sides or blame anyone, you don’t know the other side of
the story. You may say something like: “I’m sorry this happened to you, what canI do to
help you?” You may offer to talk to the other doctor or the hospital committee to try to
prevent this error from happening in the future. Allow the patient to vent his/her
feelings, and focus on what needs to be done for your patient’s health and wellbeing.

310
References;
1- Hebert, Philip C. Doing Right: A Practical Guide to Ethics for Medical Trainees and
Physicians. 2nd ed. Don Mills, Ont.: Oxford University Press, 2009.
2- CMA code of ethics, CMA website, accessed December 25/2014,
http://policybase.cma.ca/dbtw-wpd/PolicyPDF/PD04-06.pdf

311
Appendex A;

Abbreviations;

AKI Acute Kidney Injury


CBCD Complete Blood count and Differential
COPD Chronic Obstructive Pulmonary Disease
CK Creatine Kinase
CKD Chronic Kidney Disease
CABG Coronary Artery Bypass Grafting
CHF Congestive Heart Failure
CVS Cardiovascular System
CMPA Canadian Medical Protective Association
DM Diabetes Mellitus
DKA Diabetic Ketoacidosis
DIC Disseminated Intavascular Coagulation
DVT Deep Vein Thrombosis
Etoh Alcohol
GI Gastrointestinal
IMG International Medical Graduate
INR International Normalized Ratio
LMN Lower Motor Neuron
LMP Last Menstrual Period
LDH Lactate dehydrogenase
MSK Musculoskeletal
MI Myocardial Infarction
MVA Motor Vehicle Accident
OSCE Objective Structured Clinical Examination
PE Physical Examination/ Pulmonary Embolism
PVD Peripheral Vascular Disease
PTT Partial Throbmoplastin Time
RA Rheumatoid Arthritis
SLE Systemic Lupus Erythematosus
UMN Upper Motor Neuron
U/A Urine Analysis
US Ultrasound

312
WBC White Blood Cells
XR X-Ray

Index
Confidentiality, 14, 308, 309
A
Constipation, 32, 158, 229
Abdominal pain, 32, 72, 208, 215, 301 contraception, 36, 37, 38, 186, 236, 237, 238, 239, 242,
Adolescent (teenager) history, 44 244, 247, 248, 253, 257, 264, 306
Alcohol, 34, 45, 124, 132, 140, 169, 210, 217, 227, 231, Cough, 13, 32, 130, 131
254, 317 Counsel, 117, 124, 128, 235, 237, 259
Allen test, 62 Courvoisier’s sign, 72, 73
Ankle Brachial Index, 62 Cranial nerves, 74
Ankle edema, 31
Ankles, 105 D
Arterial, 59
Ascites, 58, 70 Depression, 43, 137, 139, 143
Autonomy, 14, 309, 310 dermatomal distribution, 61, 81, 173
Developmental milestones, 41
diabetes, 29, 37, 39, 120, 132, 139, 148, 158, 168, 183,
B
185, 194, 195, 197, 198, 200, 201, 202, 203, 204, 216,
Back, 13, 95, 96, 163, 164, 171 240, 253, 280
Beneficence, 14, 309, 312 Diarrhea, 13, 32, 33, 166, 197, 205, 206, 212, 213
Biceps, 79, 80, 94 differential diagnoses, 18, 26, 27, 28, 31
breast, 37, 38, 40, 107, 108, 142, 230, 238, 240, 242, 244, Dizziness, 31, 207
253, 254, 255, 265, 269, 270, 272, 273, 280, 281, 283, drawer, 104, 180
284 Dysuria, 32, 229

C E
CAGE, 34, 35 Elbows, 78, 89
Cardiovascular, 8, 12, 31, 50, 72, 156, 317 Endocrine, 33, 263
Carpal Tunnel Syndrome, 87 ethical, 7, 18, 19, 20, 22, 129, 134, 141, 153, 161, 171, 180,
Cerebellar, 82 193, 204, 211, 224, 234, 248, 256, 266, 277, 278, 304,
Chest pain, 13, 31, 32, 131, 144, 146, 155, 156, 186 308, 311, 312
claudication, 31, 33, 173

313
F M
Fatigue, 13, 31, 33, 137, 208 M SIGE CAPS, 43
Fever, 33, 165, 175, 176, 186, 208, 280 mammogram, 38, 142, 253, 257
Fundoscopy, 13, 75, 76, 109, 234 MCC, 7, 17, 20, 22, 107, 128
MCCQE2, 7, 12, 17, 18, 19, 20, 22, 23, 114
G McMurray test, 104, 105
medications, 26, 30, 31, 37, 39, 50, 115, 116, 118, 119,
Gait, 83 132, 140, 144, 149, 159, 162, 177, 182, 186, 194, 195,
Gastrointestinal, 8, 32, 317 197, 199, 202, 207, 209, 211, 216, 227, 230, 235, 241,
Genitourinary, 32 242, 254, 262, 264, 280, 282, 291, 296, 301, 314
Glasgow, 73, 84 menopause,, 37
MMSE, 73
H MOCA, 73
Morning stiffness, 33, 175
Headache, 32, 33 motor, 41, 61, 73, 77, 79, 80, 84, 96, 98, 170, 171, 221,
HEEADSS, 44 235, 296, 298
Hematemesis, 32 murmur, 48, 52, 53, 54, 154
Hematochezia, 32 Murphy’s sign, 72, 73, 188
Hemoptysis, 32, 131 Muscle pain, 33
Hepatojugular reflux, 55 Musculoskeletal, 9, 13, 33, 85, 317
Hips, 79, 100
History, 6, 12, 17, 25, 26, 28, 29, 30, 34, 35, 36, 37, 38, 40,
42, 130, 131, 134, 135, 138, 140, 142, 143, 147, 154,
N
156, 161, 162, 171, 172, 175, 180, 183, 193, 194, 206, NAC, 7, 12, 17, 20, 22, 23, 130, 137, 164, 174, 194, 205,
212, 213, 217, 227, 235, 240, 256, 257, 267, 279, 283, 214, 226, 237, 249, 259, 270, 279, 286, 296
284, 293, 294, 301 Natal, 40, 280
hypertension, 29, 37, 39, 52, 154, 232, 235 Nausea, 32, 208, 212, 227
non-maleficence, 14, 309, 312
I
IMGs, 1, 2, 5, 6, 17, 20, 26, 48
O
injury, 33, 81, 104, 120, 174, 180, 287, 292, 293, 296, 297, Obstetrics and Gynecology history, 36
301, 302, 305 OSCE, 1, 2, 3, 6, 7, 12, 17, 18, 20, 21, 23, 26, 48, 81, 124,
318
J
Jaundice, 32, 208, 229, 283
P
Joint pain, 33 Palpitations, 31, 33
Justice, 14, 309, 313 Pap test, 38, 123, 231, 264
JVP, 50, 54, 55, 56, 57, 111, 133, 141, 142, 151, 160 Pediatrics history, 39
Perinatal, 39, 289, 294
K Physical Exam, 17, 160, 298
Post natal, 40, 280
Knees, 79, 102 Precordium, 50, 51
prenatal, 37, 39, 239
L Prenatal, 39, 279, 280
Proprioception, 82
legal, 7, 22, 35, 114, 134, 141, 153, 161, 171, 180, 193,
Psychiatry history, 42
204, 211, 224, 234, 248, 256, 266, 268, 277, 304, 308,
311, 312
LMN, 79, 80, 317

314
R T
Recreational drug use, 34 Tenesmus, 32, 206
Respiratory, 8, 12, 31, 65, 135, 156, 215
Romberg, 82 U
Rotator cuff, 90
UMN, 75, 78, 79, 80, 318
S
V
Scoring, 12, 20, 22
SEADS, 86, 89, 91, 96, 100, 102, 105 Vaccination, 40, 282, 290
Seizures, 33 Vascular, 50, 58, 318
sensory, 61, 73, 77, 81, 82, 96, 98, 166, 170, 298 Vibration, 81
Sexual history, 35, 38 Vomiting, 32, 166, 197, 208
Shortness of breath, 31, 32
Shoulder, 78, 80, 90, 92 W
Skin rash, 33, 34, 186, 208, 215, 280
Smoking, 13, 34, 45, 124, 130, 132, 140, 169, 196, 210, Weight loss, 33, 165, 176, 208
213, 217, 219, 231, 254, 282, 301 Wheeze, 32
Sputum, 32, 131
Straight leg raise test, 98 Y
substance abuse, 35, 43, 45, 129
Yergason, 94, 95
Syncope, 31

315
316
317
NAC OSCE
A Comprehensive Review
First Edition
Copyright @ 2011, Canadaprep.

All rights reserved. No part of this publication may be reproduced or transmitted in any form or by any
means, electronic or mechanical, including photocopy, recording, or any information storage and retrieval
system, without permission in writing from the publisher. Reproducing passages from this book without such
written permission is an infringement of copyright law.

Care has been taken to confirm the accuracy of the information presented and to describe generally accepted
practices. However, the authors, editors, and publishers are not responsible for errors or omissions or for any
consequences from application of the information in this book and make no warranty, expressed or implied,
with respect to the contents of the publication. The authors, editors, and publishers have exerted every effort
to ensure that drug selection and dosage set forth in this text are in accordance with current
recommendations and practice at the time of publication. However, in view of ongoing research, changes in
government regulations, and the constant flow of information relating to drug therapy and drug reactions,
the reader is urged to check the package insert for each drug for any change in indications and dosage and
for added warnings and precautions. This is particularly important when the recommended agent is a new or
infrequently employed drug.

This publication has not been authored, reviewed or supported by the Medical Council of Canada, nor is it
endorsed by the Medical Council as a review material for the NAC OSCE.
PREFACE

This book was written due to the lack of preparation material available for the National Assessment
Collaboration (NAC) Objective Structured Clinical Examination (OSCE). As an International Medical
Graduate (IMG) preparing for clinical and written exams in Canada, there is no comprehensive review
textbook available for the NAC OSCE. Due to this lack of resource materials, many students are forced to
study from sources that are not relevant to the NAC OSCE exam. This eventually hampers the candidate's
chances of a good score in the examination.

This book aims to guide you through the steps of the NAC OSCE and ensure that you are well prepared
and a step ahead of the competition. A great effort has been put into collecting and organizing relevant
content for both the clinical OSCE stations and the written therapeutic exam.
Written by medical graduates who are oriented to the NAC OSCE, this comprehensive review can be used
as a framework, complementing your clinical skills and therapeutic knowledge as you prepare for the
examination.

This book is dedicated to all the IMGs preparing for the medical licensing examinations in Canada.

“And most important, have the courage to follow your heart and intuition. They somehow already know what you
truly want to become. Everything else is secondary.” - Steve Jobs
TABLE OF CONTENTS

Introduction to NAC OSCE


General Information ......................................................................................................................1
Registration for NAC OSCE ........................................................................................................1
Fees ................................................................................................................................................1
Examination station .......................................................................................................................1
NAC OSCE scoring ......................................................................................................................2
Sample of Therapeutic written test ...............................................................................................2
Sample clinical case station ............................................................................................................3
Therapeutic Guidelines
Medicine
Cardiology .....................................................................................................................................7
Dermatology ................................................................................................................................11
Endocrinology .............................................................................................................................14
Gastroenterology .........................................................................................................................16
Hematology .................................................................................................................................19
Infectious Diseases ......................................................................................................................19
Neurology ....................................................................................................................................21
Otolaryngology ............................................................................................................................23
Pulmonology ...............................................................................................................................24
Rheumatology .............................................................................................................................26
Nephrology/Urology ...................................................................................................................29
Emergency Medicine ...................................................................................................................30
Counseling (smoking/alcohol) ....................................................................................................35
Obstetrics & Gynecology
Sexually transmitted infections ....................................................................................................38
Urinary tract infection .................................................................................................................39
Vulvovaginitis ..............................................................................................................................39
Pelvic inflammatory disease .........................................................................................................40
Dysfunctional uterine bleeding ...................................................................................................40
Dysmenorrhea .............................................................................................................................40
Endometriosis ..............................................................................................................................40
Hormone replacement therapy ....................................................................................................41
Emergency contraception ............................................................................................................41
Group B Streptococcus in pregnancy ..........................................................................................41
Pregnancy induced hypertension .................................................................................................41
Ectopic pregnancy .......................................................................................................................42
Hyperemesis gravidarum .............................................................................................................42
Drugs contraindicated in pregnancy ...........................................................................................42
Pediatrics
Acute bronchiolitis ......................................................................................................................45
Acute otitis media ........................................................................................................................45
Asthma ........................................................................................................................................45
Bacterial tracheitis .......................................................................................................................45
Bacterial pneumonia ....................................................................................................................46
Croup (Laryngotracheobronchitis) .............................................................................................48
Epiglottitis ...................................................................................................................................48
Streptococcal pharyngitis (Group A streptococcus) ....................................................................48
Whooping cough (Pertussis) .......................................................................................................48
Bacterial meningitis .....................................................................................................................49
Febrile seizures .............................................................................................................................49
Urinary tract infection .................................................................................................................49
Allergic reaction ...........................................................................................................................50
Anemia ........................................................................................................................................50
Dose of tylenol ............................................................................................................................50
Immunization schedule ...............................................................................................................50
TABLE OF CONTENTS
Psychiatry
Delerium ......................................................................................................................................52
Mania ...........................................................................................................................................53
Panic disorder ..............................................................................................................................53
Social phobia ...............................................................................................................................54
General anxiety disorder ..............................................................................................................54
Obsessive compulsive disorder ....................................................................................................55
Post traumatic stress disorder ......................................................................................................55
Dementia .....................................................................................................................................55
Depression ...................................................................................................................................56
Psychosis ......................................................................................................................................56
Mood stabilizers ..........................................................................................................................57
Medications causing sexual dysfunction ......................................................................................58
Substance abuse ...........................................................................................................................59
Clinical Examination
Abdominal examination ..............................................................................................................63
Cardiovascular examination .........................................................................................................65
Peripheral vascular examination ...................................................................................................67
Respiratory examination ..............................................................................................................69
Central nervous system examination ...........................................................................................71
Upper limb neurological examination .........................................................................................73
Lower limb neurological examination .........................................................................................75
Muscolo-skeletal system : Spine/Back ........................................................................................77
Hip ...............................................................................................................................................79
Knee .............................................................................................................................................81
Foot and ankle .............................................................................................................................83
Shoulder .......................................................................................................................................85
Elbow ...........................................................................................................................................87
Hand and wrist ............................................................................................................................88
Breast examination .......................................................................................................................90
Thyroid ........................................................................................................................................91
Mini Mental State Examination .................................................................................................93
Clinical cases
Protocol for history taking ...........................................................................................................99
Medicine
Atrial fibrillation ........................................................................................................................102
Asthma .......................................................................................................................................103
Congestive heart failure .............................................................................................................104
Cerebrovascular attack ...............................................................................................................105
Digoxin toxicity .........................................................................................................................106
Infectious mononucleosis (sore throat) ......................................................................................107
Impotence ..................................................................................................................................108
Meningitis ..................................................................................................................................109
Headache (migraine) .................................................................................................................110
Chest pain (Myocardial Infarction) ...........................................................................................111
Pneumonia .................................................................................................................................112
Post exposure prophylaxis for HIV ............................................................................................113
Pulmonary embolism .................................................................................................................114
Seizure disorder .........................................................................................................................115
Temporal arteritis ......................................................................................................................116
Viral hepatitis ............................................................................................................................117
Obstetrics and Gynecology
Abortion .....................................................................................................................................118
Antenatal visit ............................................................................................................................119
Ectopic pregnancy .....................................................................................................................120
TABLE OF CONTENTS
Infertility ....................................................................................................................................121
OCP counseling .........................................................................................................................122
Pelvic inflammatory disease .......................................................................................................123
Placenta previa ...........................................................................................................................124
Pre eclampsia .............................................................................................................................125
Pediatrics
Failure to thrive ..........................................................................................................................126
Febrile seizure ............................................................................................................................127
Measles ......................................................................................................................................128
Neonatal jaundice ......................................................................................................................129
Primary nocturnal enuresis ........................................................................................................130
Pyloric stenosis ..........................................................................................................................131
Speech delay ...............................................................................................................................132
Psychiatry
Anorexia .....................................................................................................................................133
Bulimia .......................................................................................................................................134
Delirium .....................................................................................................................................135
Dementia ...................................................................................................................................136
Depression .................................................................................................................................137
Mania .........................................................................................................................................138
Panic attack ................................................................................................................................139
Schizophrenia ............................................................................................................................140
Suicide ........................................................................................................................................141
Surgery
Back Pain ...................................................................................................................................142
Basal cell carcinoma ...................................................................................................................143
Benign prostatic hyperplasia ......................................................................................................144
Carpal Tunnel Syndrome ...........................................................................................................145
Deep Vein Thrombosis ..............................................................................................................146
Diabetic foot ..............................................................................................................................147
Difficulty swallowing (Ca oesophagus ) ....................................................................................148
Hematemesis .............................................................................................................................149
Neck swelling .............................................................................................................................150
Pain abdomen ............................................................................................................................151
Peripheral vascular disease .........................................................................................................152
Post operative fever ....................................................................................................................153
Solitary lung nodule ...................................................................................................................154
Thyroid mass .............................................................................................................................155
Trauma .......................................................................................................................................156
Counseling
Breast feeding ............................................................................................................................159
Child abuse ................................................................................................................................160
Domestic violence ......................................................................................................................161
Hormone replacement therapy ..................................................................................................161
Mammogram .............................................................................................................................163
Immunization ............................................................................................................................164
Obesity .......................................................................................................................................165
Smoking .....................................................................................................................................167
Introduction to NAC OSCE | General Info 1

Introduction to NAC OSCE

General Information
The National Assessment Collaboration, or NAC OSCE, was established to provide a system of streamlined
assessment of IMG medical knowledge and clinical skills throughout Canada. Many international medical
graduates (IMG's) find that the path to obtaining a medical license in Canada is challenging and difficult to
navigate. Different provinces and territories have their own system for assessing IMG medical knowledge and
clinical skills.
Comprised of a number of federal and provincial assessment and educational stakeholders, the NAC OSCE
aims to streamline the evaluation process through which an IMG must navigate to obtain a license to practice
medicine in Canada. Through such a system, an IMG’s path to licensure would be the same, regardless of the
jurisdiction in which he or she is being assessed. The NAC OSCE has replaced CEHPEA’s Clinical
Examination 1 (CE1), which was unique to Ontario.

Registration for NAC OSCE


Registration for the NAC OSCE starts in November, with the deadline in January the next year. Candidates
are advised to complete their registration within this time-frame. Once the deadline is over, the candidate will
not be able to register for the NAC OSCE for the entire year. The exams are scheduled for March, June,
August and September.
Visit www.mcc.ca and www.cehpea.ca for updated information.

Fees
Application Fee: $200 which is non-refundable, NAC OSCE Fee in Ontario: $1850 and Exam Date Change
Fee: $100
All fees are in (CAD) Canadian Dollars.

Examination station
The format for the National Assessment Collaboration (NAC) Objective Structured Clinical Examination
(OSCE) consists of 12 stations based on presentations of clinical scenarios. For a given administration, each
candidate rotates through the same series of stations. Each station is 10 minutes in length with two minutes
between stations.
At each station, a brief written statement introduces a clinical problem and outlines the candidate’s tasks (e.g.
take a history, do a physical examination, etc.). In each station, there is at least one standardized patient and a
physician examiner. Standardized patients have been trained to consistently portray a patient problem.
Candidates should interact with standardized patients as they would with their own patients.
The physician examiner observes the patient encounter. For most stations, the candidate will be asked to
respond to a series of standardized oral questions posed by the physician examiner after seven minutes with the
standardized patient.
Orientation videos http://www.mcc.ca/en/video/QEII-Orientation/index.html
2 NAC OSCE | A Comprehensive Review

The examination includes a separate written test of candidates’ therapeutic knowledge. This component lasts
45 minutes and consists of 24 short-answer questions testing the candidates’ knowledge of therapeutics for
patients across the age spectrum and related to pharmacotherapy, adverse effects, disease prevention and health
promotion.

NAC OSCE scoring


The candidate’s total examination score will be determined by combining the scores on the OSCE component
with the scores on the therapeutics component. The OSCE score contributes 75 per cent of the total score and
the therapeutics score contributes 25 per cent of the total score. For reporting purposes, the NAC total
examination scores are reported on a scale with a distribution ranging from 0 to 100 with a fixed passing mark
of 65.

Number of times candidates can take the examination


Starting in 2011, the NAC OSCE can be attempted once per Canadian Resident Matching Service (CaRMS)
cycle. If you pass the examination, you can register for the examination a maximum of two additional times if
your eligibility is maintained. Regardless of whether you pass or fail, you can only take the examination three
times. If you take the examination more than once, the most recent result will be the only valid result.

Sample of Therapeutic written test


Question: An otherwise healthy 65 year old woman presents with a 3 week history of aching
and morning stiffness in both shoulders with difficulty dressing. She has no temporal artery
tenderness, headache, jaw pain or visual disturbance. Her ESR (Erythrocyte sedimentation
rate) is 100 and you have made the diagnosis of POLYMYALGIA RHEUMATICA (PMP).
What would you choose as the drug of first choice for initial medical therapy? (Drug, dose,
route of administration and duration are required.)
Answer: ________________________________________________________
Answer key the marker receives:
PREDNISONE 7.5 – 20 mg PO od for 2-4 weeks following resolution of
symptoms

Question: An otherwise healthy 55 year old male with s history of childhood “chickenpox”
presents with a 2 day history of painful unilateral vesicular eruption in a restricted
dermatomal distribution. You make a diagnosis of HERPES ZOSTER (shingles).
What would you choose as the drug of first choice to promote healing and lessen the
neuropathic pain? (Drug, dose, route of administration and duration are required.)
Answer:___________________________________________________________
Answer key the marker receives:
VALACYCLOVIR (VALTREX ®) 1000 mg PO tid X 7 days OR
FAMCICLOVIR (FAMVIR ®) 500 - 750 mg PO tid X 7 days OR
ACYCLOVIR (ZORIVAX ®) 800 mg PO 5X / day X 7 days)
Introduction to NAC OSCE | General Info 3

Sample Clinical Case Station


Example instruction written outside the station

David Thompson, 59 years old, presents to your office complaining of jaundice.

In the next 7 minutes, obtain a focused and relevant history.


After the 7 minutes, you will be asked to answer questions about this patient.

Example of post encounter questions

Q1. The abdominal examination of David Thompson revealed no organ enlargement, no masses and no
tenderness. What radiologic investigation would you first order to help discriminate the cause of the
jaundice?

Q2. If the investigations revealed that this patient likely had a post-hepatic obstruction, what are
the two principal diagnostic considerations?

Q3. What radiologic procedure would you consider to elucidate the level and nature of the
obstruction?
Therapeutic Guidelines
Therapeutic Guidelines | Medicine 7

Therapeutic Guidelines

Medicine

1. Cardiology
Acute Myocardial Infarction : Immediate management in ER
ACUTE MI TREATMENT
1. Beta blockers: Inj Metoprolol 2.5-5 mg rapid IV q2-5 min, upto MNEMONIC
15 mg over 10-15 minutes, then 15 minutes after receiving 15 mg IV. B : Beta Blockers
2. Then 50 mg PO q6h x 48 hours, then 50-100 mg PO BID. M : Morphine Sulphate
3. Inj Morphine Sulfate IV 2-5 mg every 5-30 min prn O : Oxygen
N : Nitroglycerin
Pain not relieved with 3 Sublingual Nitroglycerins A : Aspirin
4. Oxygen by nasal cannula at 4 liters per minute
5. Sublingual Nitroglycerin 0.3-0.6 mg q5min up to 3 times.
6. Non-enteric coated Aspirin 325 mg PO.
7. Cardiology Consultation

Post MI drugs

Drugs Benefits Side effects Contraindication


ACE Inhibitors ↓ mortality Hypotension/dizziness Bilateral renal artery
Ramipril – 10mg hs Prevents ventricular remodelling Hyperkalemia stenosis
Lisinopril – 10 mg od ↓ proteinuria Angioedema Hx of angioedema
Enalapril – 20mg od Renal insufficiency Pregnancy
Captopril - 50 mg tid Cough, taste changes

ARB ↓ mortality Angioedema


Valsartan – 160mg bid ↓ proteinuria Cough, taste changes
Candesartan – 32 mg od

Beta Blocker ↓ mortality Decreases BP & HR Severe/poorly


Metoprolol – 100mg bid ↓ sudden death, reinfarction & Dizziness, fatigue controlled asthma
nd
Atenolol – 100mg od arrhythmias Sexual dysfunction 2 /3rd degree heart
Cardioselective : preferred for block
Carvidelol – 25mg bid May mask hypoglycemia
mild asthma and diabetes HR<50, SBP<90
Proranolol – 60-80 tid Increase risk of
cardiogenic shock Cocaine use

Statins ↓ mortality in post MI patients GI upset, muscle aches, Active liver disease,
Atorvastatin – 10mg od with high cholesterol myopathy, rhabdomyo- alcoholics, pregnancy
lysis, Impotence
Simvastatin – 20-40mg od

Anti-platelets ↓ vascular events GI upset, Recent/active


ASA – 80-162mg od hypersensitivity bleeding
Clopidogrel – 75mg od GI bleed GI intolerance or ASA
allergy
Warfarin – 1-10mg od
8 NAC OSCE | A Comprehensive Review

Atrial Fibrillation
1. To control rate:
• Inj Metoprolol 5 mg bolus IV, followed by infusion at 0.05 mg/kg/min, increasing as needed
to 0.2 mg/kg/min.
• Inj Diltiazem 20 mg bolus. Maintenance infusion of 5-15 mg/hr.
• Inj Verapamil 5-10 mg IV over 2-3 min, repeated once after 30 mins.
• Tab Amiodarone (in case of heart failure):
• Loading dose: 800 – 1600 mg PO in divided doses until response; till max 1000
mg/day divided bid-tid.
• Maintenance: 200 mg PO od.
2. To prevent thromboembolism: Assess with CHADS 2 score
• No risk: Tab Aspirin 81-325 mg PO od.
• 1 moderate risk: Tab Aspirin 81-325 mg PO od or Tab Warfarin 2-15 mg PO od to maintain
INR 2-3.
• > 1 moderate risk or very high risk: Tab Warfarin 2-15 mg PO od to maintain INR 2-3.
3. To control rhythm:
• Tab Flecainide 300-400 mg PO bolus dose, maintenance: 50-150 mg PO bid. (First choice)
• Tab Sotalol 80-160 mg PO bid. ( Second choice)
• Tab Amiodarone (in case of heart failure):
• Loading dose: 800 – 1600 mg PO in divided doses until response; till max 1000
mg/day divided bid-tid.
• Maintenance: 200 mg PO od.
• Electrical Cardioversion: 100-360 joules.

Congestive Cardiac Failure : Immediate management in the ER CHF TREATMENT MNEMONIC


L : Lasix
• Oxygen by nasal cannula at 4 liters per minute. M : Morphine Sulphate
• Inj Furosemide (Lasix) 10 mg IV stat. N : Nitroglycerin
O : Oxygen
• Inj Morphine sulfate IV 2-5 mg every 5-30 min prn. P : Positive airway pressure
• Sublingual Nitroglycerin 0.3-0.6 mg q5min up to 3 times. P : Position > 45 degrees
• Position of patient > 45 degrees. D : Dopamine (indicated in
cardiogenic shock and
hypotension)
Non pharmacological management of Heart Failure
• Exercise : Regular physical activity
• Salt restriction : symptomatic HF – 2-3g salt/day (½ tsp/day) no added salt in diet.
HF with fluid retention : 1-2g salt/day (¼ tsp/day)
• Fluid intake : 1.5/2L per day in patients with fluid retention or HF not controlled by diuretics.
• Daily weight measurement.
• Education.
• Aggressive risk reduction (BP, glucose, lipids).
• Lifestyle modifications, influenza vaccination.
Therapeutic Guidelines | Medicine 9

Dyslipidemia

1. HMG CoA Inhibitors:


• Atorvastatin : Tab Lipitor 10-80 mg qhs
• Rosuvastatin : Tab Crestor 10-40mg qhs
• S/E: GI symptoms, rash, pruritus, increased liver enzymes, myositis.
• C/I: active liver disease, muscle disease, pregnancy.
2. Fibrates: increased TG
• Fenofibrate : Tab Lipidil 67-200 mg/d
3. Bile acid sequestrants : increased LDL
• Tab Colestipol 5-30g/day
4. Cholesterol absorption inhibitors:
• Tab Ezetimibe 10mg /day.

Lipid Risk LDL Total cholesterol/HDL

HIGH (10yr CAD ≥20%) Target LDL - <2.0 Target <4

MODERATE (10yr CAD ≥10-19%) Treat if LDL - ≥3.5 Treat if ≥5

LOW (10yr CAD <10%) Treat if LDL - ≥5 Treat if ≥6

High Risk : All with CAD, CVD, most diabetes cases & chronic renal disease.

Hypertension

Non pharmacological treatment :

• Smoking cessation: smoking aggravates hypertension and remains the major contributor to
cardiovascular disease in people under 65 years.
• Weight reduction : Maintain BMI<27, particularly in patients with glucose intolerance
• Alcohol restriction.
• Sodium restriction <150mmol/day.

Blood pressure risk factors Consider treatment if BP BP target

No risk factors ≥160/100 <140/90

Isolated systolic hypertension SBP>160 SBP<140

Moderate-High risk patient ≥140/90 <140/90

Diabetes or Renal disease ≥130/80 <130/80


10 NAC OSCE | A Comprehensive Review

Commonly used anti-hypertensive drugs :

Drug Indication Side Effect

Diuretics Uncomplicated HTN, Diabetes Rash, allergic rxn, pancreatitis, sexual


Hydrochlorothiazide HCT – 12.5-25mg od with normal albuminuria, LVH dysfunction. HCT contraindicated in
Spironolactone – 25-50mg od and isolated systolic HTN gout.

Beta Blockers Stable angina, MI, LVH, Fatigue, insomnia, ↓HR, impotence,
nd rd
Metoprolol – 50mg bid or 100mg SR od uncomplicated HTN ≤60 years, dizziness. C/I – asthma/COPD, 2 /3
Propranolol – 80mg bid degree heart block, uncompensated HF
Atenolol – 50-100mg od severe PAD

ACE Inhibitors Heart failure, diabetes, post MI, Cough, loss of taste, rash, angioedema,
Ramipril – 10mg hs uncomplicated HTN, LVH, prior renal failure, ↓BP
Lisinopril – 10 mg od CVA/TIA, renal disease, all C/I – b/l renal artery stenosis, Hx of
Enalapril – 10-20mg od
coronary artery disease pts. angioedema, pregnancy
Captopril - 25-50 mg bid
Angiotensin II Receptor Blockers Diabetes, uncomplicated HTN, Fatigue, headache, rash, angioedema,
Losartan - 25-50mg od isolated systolic HTN, LVH, ↓BP, ↑K+, pancreatitis.
Valsartan – 80-160mg od patients unable to tolerate ACEI. C/I – b/l renal artery stenosis, Hx of
Candesartan – 8-16mg od angioedema, pregnancy

Calcium Channel Blockers Uncomplicated HTN, LVH, Angina, Dizziness, headache, rash, edema,
Amlodipine – 2.5-10mg od Isolated systolic HTN, diabetes gingival hypertrophy, worsen HF
Nefidipine - 10mg tid without nephropathy C/I – hypotension, recent MI with
Verapamil - 40-80mg tid pulmonary edema, sick sinus
nd
Diltiazem – 30-60mg tid syndrome, 2 /3rd AV block

Methyldopa – 125mg bid to 500mg qid First-line for hypertension in Sedation, dry mouth, hepatotoxic,
pregnancy lupus like Sx

Infective Endocarditis Prophylaxis

• Inj Ampicillin 2g IV q4h x 4weeks.


• Inj Gentamicin 1mg/kg IV q8h x 4 weeks.
• Prophylaxis: Dental/respiratory/esophageal procedure: Tab Amoxicillin 2g PO 30-60 min prior;
Tab Clindamycin 600mg PO, if allergic to penicillin.

Rheumatic Heart Disease (RHD)

• Tab Erythromycin 500mg tid PO x 10days.


• Tab Penicillin VK 500 mg PO bid x 10 days.
Therapeutic Guidelines | Medicine 11

2. Dermatology
Acne

Mild : <20 comedones (whiteheads/blackheads) or <15 inflammatory papules, or a lesion count <30
Moderate : 15-50 papules and pustules with comedones, cysts are rare, lesion count ranges from 30-125
Severe : Primarily nodules and cysts,also present are comedones, papules and pustules, scarring is present,
lesion count >125

st
T Benzoyl Peroxide (Antibacterial/Keratolytic) Indication: 1 line S/E : contact dermatitis,
O
Dose : apply to entire affected area qhs or bid medication for mild- dryness, erythema, burning
P
I moderate acne. & pruritis
C st
A Tretinoin (Retinoid) 1 line treatment for mild- S/E : erythema, dryness,
L Dose : qhs, apply 30-45 minutes after wash moderate comedones acne. burning, photosensitivity.

Oral antibiotics Indicated for moderate- S/E : GI upset, nausea,


Tetracycline - initial 500mg bid then 250-500mg od severe acne. vomiting, candidiasis.
Doxycycline – 100mg od Acne on chest, back & C/I : liver disease
Erythromycin - initial 500mg bid then 250-500mg od shoulders
S Combined Oral Contraceptive Females with moderate- C/I : Smoking, migraine with
Y
S
Diane 35/Yasmin/Alesse : od x 21 days , 7 days severe acne + seborrhea + aura, seizures
T off/cycle hirsutism, late onset acne
E
M Isotretinoin Severe nodulocystic acne, Teratogenicity : ocular
I Accutane : 0.5-1mg/kg/day x 16-20 weeks acne with scarring, failure to effects – conjuntivitsis,
C
*Important : Tests for pregnancy 30 days prior to respond to other treatments ↓night vision, premature
starting Accutane, before each refill. Patient has to epiphyseal closure, ↑LFTs,
sign an informed consent. pseudomotor cerebri,
mucocutaneous effects,
myalgias. Photosensitivity.

Burns

• Initial assessment of ABCs , consider the need for early intubation if airway is compromised.
• Humidified O2 if any suspicion for inhalational injury.
• Oxygen 100% if known carbon monoxide exposure of fire in an enclosed space. (Half life of
hemoglobin will drop from 330 to 90 mins).
• Establish IV access.
• Fluid resuscitation : Parkland formula 4mL/kg/%BSA burn, ½ over 8 hours and rest over 16 hours
• Nasogastric tube drainage for ileus.
• Bladder catheterization to monitor urinary output, minimum 1mL/kg/hr.
• Tetanus prophylaxis : 0.5 mL tetanus toxoid IM in previously immunized and 250 units TIG IM if
unimmunized.
12 NAC OSCE | A Comprehensive Review

Psoriasis

Topical Preparations :

1. Topical Corticosteroids :
• High Potency Topical Steroids (Usually indicated)
• Very high potency: e.g. Clobetasol (Temovate)
• High potency: e.g. Fluocinonide (Lidex)
• Low Potency Topical Steroids
• Face
• Genitals
• Maintenance Therapy
2. Vitamin D based topicals :
• Calcipotriene (Dovonex)
• Used in combination with Topical Corticosteroids
3. Retinoid based topicals :
• Tazarotene (Tazorac)
• More irritating than Calcipotriene
4. Immunosuppressant based topicals :
• Tacrolimus 0.1% or Pimecrolimus 0.1% creams
Effective in facial and intertriginous Psoriasis
5. Adjunctive agents in combination with above :
• Topical Salicylic Acid (Keratolytic Agent)
6. Poorly tolerated topicals (use Calcipotriene instead) :
• Historically used with UVB light exposure
• Anthralin (Anthra-Derm)
• Coal Tar (e.g. Zetar)

Ultraviolet light

• Risk of non-Melanoma skin cancer


• Protocols
• Ultraviolet B exposure alone
• Ultraviolet A exposure with psoralen (PUVA)
Increased risk of non-Melanoma skin cancer

Systemic agents (most are for higher risk)

• Immunosuppressants
• Etretinate
• Cyclosporine
• Methotrexate (unclear efficacy)
Therapeutic Guidelines | Medicine 13

• Biological agents
• Tumor necrosis factor (TNF) receptor blockers
Etanercept (Enbrel)
Infliximab (Remicade)
• Other mechanisms
Alefacept (Amevive)
Efalizumab (Raptiva)
• Thiazolidinedione (Avandia, Actos) - experimental
• Appears effective in Psoriasis even in non-diabetics
• Only small trials support to date

Cellulitis

• Cause : β Hemolytic Streptococcus , Staphylococcus


• Treatment : Tab Cloxacillin 500mg PO qid x 10-14 days
If patient is allergic to penicillin : Tab Cephalexin 500mg PO qid 10-14 days OR
Tab Clindamycin 300mg PO qid x 10-14 days

Pediculosis

• Permethrin 1% - wash hair with regular shampoo, then apply permethrin and leave for 10 mins then
rinse
• Pyrethrins with piperonyl butoxide
• Lindane 1% C/I in neonates, young children and pregnant women, causes neurotoxicity
• Wash all clothes and linen in hot water, then machine dry.

Scabies

• Permethrin 5% - massage into all skin areas, from the top of the head to the soles of the feet, leave for
8-14 hours then wash off.
• Crotamiton 10%
• Scabene (aerosol spray) Esdepallethrin
• Lindane : used only if allergic to permethrin.
• Treat family and contacts.
• Wash all clothes and linen in hot water, then machine dry.

Tinea Cruris/Pedis ( Jock itch/Athlete's foot)

• Clotrimazole 1% cream apply bid


• Ketoconazole 2% cream apply bid
14 NAC OSCE | A Comprehensive Review

3. Endocrinology
Diabetes Mellitus

Blood glucose target

A1C q3-6 months Target ≤7 Normal range≤6

Fasting plasma glucose Target 4-7mmol/L Normal range 4-6mmol/L

Post prandial blood glucose 2h Target 5-10mmol/L Normal range 5-8mmol/L

Approach to management of diabetes mellitus

1. Lifestyle modification & patient education


2. Oral hypoglycemic monotherapy :
• Biguanides (Metformin) – 250-500mg PO bid-tid (if obese or overweight)
• Sulfonylureas (Glyburide) – 80mg PO bid
• Thiazolidinedione (Pioglitazone) – 15mg PO od; Rosiglitazone – 4mg PO od
• Alpha glucosidase inhibitors (Acarbose) – 50mg PO tid
3. Oral combination therapy (2 agents often needed; after 3 years 50%, after 9 years 75%)
4. Insulin therapy +/- oral hypoglycemics

Diabetic Ketoacidosis Management

• Fluid replacement
• Initial : Give 1 liter NS bolus over first 45 minutes, repeat fluid bolus until shock corrected.
• Next : Replace first 50% volume deficit in first 8 hours, use Normal Saline or Lactated
Ringers. Replace remaining 50% deficit over next 16 hours, use D5 1/2 NS at 150-250 ml per
hour.
• Insulin (Hypokalemia must be corrected prior to Insulin)
• Initial
i. Give IV bolus of 0.15 units/kg
ii. Start 0.1 units/kg/hour Insulin Drip
• Maintenance
i. Anticipate Serum Glucose drop of 50-70 mg/dl/hour
• If inadequate drop, then increase drip
a) Increase Insulin Infusion rate by 50-100%
b) Continue at increased rate until adequate
ii. When Serum Glucose <200-250 mg/dl
a) Keep Serum Glucose at 150 to 200 mg/dl
b) Decrease rate by 50% (to 0.05 units/kg) or
c) Discontinue Insulin Drip and start SC dosing
Therapeutic Guidelines | Medicine 15

• Potassium
Do not administer Insulin until potassium >3.3
• Give KCl 40 mEq/hour IV until corrects
• Serum Potassium 3.3 to 5.0 mEq/L
i. Standard replacement: 20-30 mEq per liter
• Serum Potassium >5.0 mEq/L
i. Do not administer any potassium
ii. Monitor every 2 hours until <5.0
• Bicarbonate
Indications
i. ABG pH < 6.9 to 7.0 after initial hour of hydration
ii. Other contributing factors
• Shock or Coma
• Severe Hyperkalemia

Hyperthyroidism

• Tab Propylthiouracil(PTU) 100 mg PO tid, to max 150 mg 6-8 hours.


• Tab Methimazole 10-30 mg PO od.
• Medications associated with Hyperthyroidism:
Excess Thyroid hormone intake
Dietary Iodine
Amiodarone

Hypothyroidism

• L-Thyroxine 0.05-0.2 mg/day


• Medications associated with Hypothyroidism:
i. Inorganic iodine
ii. Iodide
iii. Amiodarone
iv. Lithium

Hyperprolactinemia

• Tab Bromocriptine 1.25-2.5 mg PO od, increase by 2.5 mg/day q3-7days to max 15 mg/day.
• Tab Cabergoline 0.25 mg PO twice weekly, may increase by 0.25 mg q4weeks up to max 1mg twice
weekly.
16 NAC OSCE | A Comprehensive Review

Medications causing hyperprolactinemia


a) Benzodiazepines
b) Buspirone
c) MAOI
d) SSRI
e) TCA
f ) Valproic acid
g) Methyldopa
h) Verapamil
i) Atenolol
j) Danazol
k) Estrogen
l) Depo-Provera
m) OCPs
n) Metoclopromide
o) Amphetamines
p) Cannabis

Impotence

• Tab Sildenafil 25-100mg per dose, to take half an hour to 4 hours prior to intercourse.
S/E: flushing, headache, indigestion
C/I: don’t take with Nitrates.

4. Gastroenterology

Appendicitis

Perioperative for 24hrs


• Inj Ampicillin 1-2g IV q4-6h.
• Inj Flagyl 500mg IV bid.
• Inj Gentamicin 3-5mg/kg/day q8h (monitor creatinine levels).
• NPO

Acute Gastroenteritis
Acute Gastroenteritis Causes
(Watery diarrhea)
• Tab Flagyl 50 mg PO bid x 5 days.
E. Coli (Traveler's diarrhea)
• Tab Ciprofloxacin 500 mg PO bid x 3 days.
CMV
• Tab Norfloxacin 400 mg PO bid x 3 days. Cryptosporidium
• Oral rehydration solution. Giardia Lamblia
Therapeutic Guidelines | Medicine 17

Acute Pancreatitis

• NPO
• Inj Flagyl 400mg IV q8h
• Inj Meperidine 75-100mg IV q2-3h
• IVF
• NG tube
• Replace calcium

Crohn’s Disease

1. Mild to moderate:
• Tab Mesalamine 800 mg PO tid. Maintenance dose 3.2 – 4g per day.
• Tab Sulfasalazine 250 mg per day and increase up to 2 g per day. Maintenance dose is 500-
1000 mg PO qid with food.
2. Moderate to severe:
• Tab Prednisone 40 mg PO qid x 8-12 weeks and taper gradually.
• Tab Azathioprine 2-2.5 mg/kg/day. Used for maintenance while tapering corticosteroids.

Diverticulitis

• Inj Flagyl 500mg IV bid.


• Inj Ciprofloxacin 500mg IV bid.

Helicobacter Pylori

1. HP-PAC (7 blister pack) 7-14 days


• Tab Lansoprazole 30mg PO bid +
• Tab Clarithromycin 500mg PO bid +
• Tab Amoxicillin 1g bid
2. 2nd LINE Quadruple : 14 days
• Tab Lansoprazole 30mg PO bid
• Tab Flagyl 500mg PO bid
• Tab Tetracycline 500mg bid
• Tab Bismuth 525mg PO qid
18 NAC OSCE | A Comprehensive Review

Hepatitis B post exposure prophylaxis

1. Known HBsAg Positive Source:


i. Unvaccinated exposed patient:
• Hepatitis B Immunoglobulin (HBIG) 0.06 ml/kg and
• Hepatitis B Vaccine 0,1and 6 months.
ii. Exposed patient with known response to vaccine:
• No treatment.
iii. Exposed patient with known failed response to vaccine:
• Patient has not yet completed second 3-dose series:
• Hepatitis B Immunoglobulin (HBIG) 0.06 ml/kg and
• Hepatitis B Vaccine (complete second 3-dose series)
• Patient has completed two prior 3-dose series:
• Hepatitis B Immunoglobulin (HBIG) 0.06 ml/kg
• Second Hepatitis B Immunoglobulin dose.
iv. Exposed patient with unknown response to vaccine:
• Test for Antibody to HBsAg
• Adequate Antibody (HBsAg Positive): No treatment
• Inadequate Antibody (HBsAg Negative)
• Hepatitis B Immunoglobulin (HBIG) 0.06 ml/kg and
• Hepatitis B Vaccine booster dose:
2. Known HBsAg Negative Source:
i. Administer Hepatitis B Vaccine Series if unvaccinated
ii. No treatment otherwise needed.
3. Unknown HBsAg Source Status:
i. Unvaccinated exposed patient
• Hepatitis B Vaccine Series
ii. Exposed patient with known response to vaccine
• No treatment
iii. Exposed patient with known failed response to vaccine
• Treat source as HBsAg positive if high risk
iv. Exposed patient with unknown response to vaccine
• Test for Antibody to HBsAg
• Adequate Antibody (HBsAg Positive): No treatment
• Inadequate Antibody (HBsAg Negative)
• Hepatitis B Vaccine initial and booster dose
• Recheck titer in 1 to 2 months
4. Infant with HBsAg Positive Mother:
i. Hepatitis B Immunoglobulin (HBIG) 0.5 ml within 12 hours of birth.
ii. Hepatitis B vaccine: Dose 1 within 12 hours of birth, Dose 2 at age 1 months, Dose 3 at age
6 months.
iii. Repeat HBsAg and HbsAb at 9 months & 15 months.
Therapeutic Guidelines | Medicine 19

Peptic ulcer disease

• Tab Omeprazole 20mg PO od.


• Tab Ranitidine 150 mg PO bid.

Ulcerative Colitis

• Tab Sulfasalazine 250 mg per day and increase up to 2 g per day. Maintenance dose is 500-1000 mg
PO qid with food.
• Tab Mesalamine 800 mg PO tid. Maintenance dose 3.2 – 4g per day.
• Rectal suppositories preferred for proctitis.

Acute Cholecystitis (Perioperative)

• Inj Cefazolin 0.5-1.5mg IV q6h


• NPO
• IVF
• NG Tube

5. Hematology

Anemia

• Iron Deficiency Anemia : Tab Ferrous fumarate(Palafer) 300 mg PO qd OR


Tab Ferrous Sulfate 325 mg PO qd
• Megaloblastic Anemia : Tab Ferrous Fumarate 300mg PO qd + Tab Folic acid 1-5mg PO qd +
Inj B12 100mg q monthly or 1000 – 2000microgram PO.

6. Infectious Diseases

Prophylaxis for opportunistic infections in HIV patients

• Pneumocystis carinii: CD4 count< 200 cells/mm 3 or oral candidiasis.


• Tab TMP/SMZ DS PO OD till CD4 counts rises.
• Toxoplasma gondii: IgG antibody positive and CD4 count < 100 cells/mm 3
• Tab TMP/SMZ DS PO OD till CD4 counts rises.
• Mycobacterium tuberculosis: Mantoux > 5 mm in immunocompromised or contact with active TB.
• Tab Isoniazid 50 mg PO OD x 9 months along with
• Tab Pyridoxine 50 mg PO OD.
• Mycobacterium avium complex: CD4 counts < 50 cells/mm 3.
• Tab Azithromycin 1200 mg PO once a week.
• Tab Clarithromycin 500 mg PO once a week.
• Varicella zoster virus: Recent exposure to chicken pox or shingles.
• Varicella zoster immune globulin within ≤ 96 hours of exposure.
20 NAC OSCE | A Comprehensive Review

HIV post exposure prophylaxis

• Start within hours of exposure (under 24 to 48 hours).


• Triple Therapy for 4 weeks:
1. First two medications: AZT and 3TC (or Combivir)
i. Tab Zidovudine (AZT) 300 mg PO bid and
ii. Tab Lamivudine (3TC) 150 mg PO bid.
2. Third medication (choose one):
i. Tab Indinavir (IDV) 800 mg PO tid or
ii. Tab Nelfinavir (Viracept) 750 mg PO tid or
iii. Tab Efavirenz 600 mg PO qhs.
• Obtain baseline labs to monitor for adverse reaction:
1. Pregnancy Test
2. Complete Blood Count with differential and platelets
3. Urinalysis
4. Renal Function Tests
5. Liver Function Tests

Malaria

1. Treatment for active infection:


i. Tab Chloroquine 1 g PO stat, then 500 mg PO 6-8 hours later, then 500 mg PO at 24 hours
& 48 hours after initial dose.
ii. Tab Mefloquine 1250 mg stat dose.
iii. Tab Primaquine 15 mg base PO od x 14 days.
2. Chemoprophylaxis:
i. Tab Chloroquine 500 mg PO once a week.
ii. Tab Mefloquine 250 mg PO once a week.

Pulmonary tuberculosis

1. Initiation Phase: Tab Rifampin 120 mg + Tab Isoniazid 50 mg + Tab Pyrazinamide 300 mg for 2
months.
2. Continuation Phase: Tab Isoniazid 50 mg + Tab Rifampin 120 mg for 4 months.
3. Add Tab Pyridoxine (Vit B6) 50 mg PO OD.

Rabies

Post exposure prophylaxis:


• Wash wound with soap and water.
• Human Rabies Immunoglobulin 20 IU/kg IM stat and half dose into the wound.
• Rabies vaccine 1 ml IM on days 0, 3, 7, 14, 28.
• Inform Public Health.
• Capture animal & observe x 10 days, then examine brain for negri bodies.
Therapeutic Guidelines | Medicine 21

Tetanus Prophylaxis : Based upon Tetanus immunization status -

Clean, minor wounds All other wounds


History of tetanus immunization
Td or Tdap* TIg** Td or Tdap* TIg

Uncertain or < 3 doses of an immunization series† Yes No Yes Yes

≥ 3 doses received in an immunization series† No‡ No No§ No

* Adult-type combined tetanus and diphtheria toxoids or a combined preparation of diphtheria, tetanus and acellular
pertussis. If the patient is < 7 years old, a tetanus toxoid-containing vaccine is given as part of the routine childhood
immunization. ** Tetanus immune globulin, given at a separate site from Td (or Tdap)
† The immunization series for tetanus is described in the text (see Schedule and Dosage).
‡ Yes, if > 10 years since last booster.
§ Yes, if > 5 years since last booster. More frequent boosters not required and can be associated with increased adverse
events. The bivalent toxoid, Td, is not considered to be significantly more reactogenic than T alone and is recommended
for use in this circumstance. The patient should be informed that Td (or Tdap) has been given.

7. Neurology

Seizures

1. Acute Management:
• Inj Diazepam 5-10mg IV q2-3mins till seizure stops. PHENYTOIN S/E
• Inj Phenytoin 20mg/kg IV at 50mg per min. P: P-450 interactions
• Inj Phenobarbital 20mg/kg IV at 50-75mg/min H: Hirsutism
E: Enlarged gums
• If all fails then RSI N: Nystagmus
2. Primary Generalized & Partial seizures: Y: Yellow-browning of skin
T: Teratogenicity
• Tab Phenytoin: Loading 300mg PO q4h x 3 doses, O: Osteomalacia
I: Interference with folic acid
then 300mg PO qhs.
absorption (hence anemia)
• Tab Valproate: Loading 15mg/kg/day, increments by N: Neuropathies: vertigo,
5-10mg/kg/day qweekly, till seizures are controlled. ataxia, headache
• Tab Carbamazepine: Start 100-200mg PO od-bid,
increments by 200mg/per q2d, if needed till max
800mg-1200mg per day.
3. Absence Seizures:
• Tab Ethosuximide 500mg PO daily in divided doses, increments by 250mg/day q4-7d prn
till max 1500mg per day.

Meningitis

• Investigations : CT then LP, CSF analysis, blood C&S, neurology consult


• Empirical adult antibiotics : 3rd generation cephalosporins + vancomycin + ampicillin
Inj Ceftriaxone 2g IV q12h
Inj Dexamethasone 10mg q6h IV x 4 days for pneumococcal meningitis
Meningococcal: give contacts Tab Rifampin 600mg PO q12h x 4 doses
22 NAC OSCE | A Comprehensive Review

CSF Findings :

Cluster headache

• Tab Triptan and Tab Prednisone at the beginning of the cycle and prophylactic treatment with
lithium(300-600mg daily initially then monitor serum levels)
• Dihydroergotamine nasal spray 4mg per 1 ml. One spray each nostril and repeat q15mins.

Migraine

1. Mild – Moderate → NSAIDS


• Tab Ibuprofen 200mg tid
• Tab Aspirin 600mg PO q4h
2. Moderate – Severe → TRIPTANS
• Tab Sumatriptan 25mg PO & repeat q 2hrs prn
• Tab Metoclopramide 10mg PO stat
3. Prophylaxis:
• Tab Propranolol 60mg PO daily
• Tab Amitriptyline 10-25mg PO qhs.

Tension headache

• Tab Tylenol 500mg PO 4-6hrs prn.


Therapeutic Guidelines | Medicine 23

Myasthenia Gravis

1. Anticholinesterase (Cholinergic)
• Tab Mestinon (Neostigmine and Pyridostigmine): 60-120 mg q3-4h.
2. Immunosuppressive therapy
• Tab Prednisone: Start at 20 mg qd, increase gradually by 5 mg every 3 days to 60mg.
Continue for 3 months or until clinical improvement stops or declines. Taper gradually to
every other day
• Tab Azathioprine (Imuran) 2 mg/kg/day. Effective when given with Prednisone. Effect not
seen for 6 months or more. Monitor CBC and LFTs.
3. Plasmapheresis (Plasma Exchange) and IV Ig: Indicated for emergent worsening/crisis.
Response rate: 70%.

Parkinson’s disease

• Tab Carbidopa/Levodopa 25/100 mg PO bid-qid, increase as needed to max 200/2000mg/day.


• Tab Bromocriptine 1.25 mg PO bid.
• Tab Pergolide 0.05 mg PO od, titrate q2-3 days to the desired effect. Maintenance dose is 3-6 mg/day
in divided doses.
• Tab Premipexole 0.125 mg PO tid, increase to 1.5 - 4.5 mg/day in divided doses.
• Tab Ropinirole 0.25 mg PO tid, increase weekly to max dose 24 g/day.
• Tab Amantadine 100 mg PO od to max 100 mg PO qid.
• Tab Selegiline 5 mg PO bid.
• Tab Benztropine 0.5-6 mg/day PO in divided doses.
• Tab Entacapone 200 mg given concurrently with Carbidopa/Levodopa.

8. Otolaryngology

Acute Sinusitis

• Tab Amoxicillin 500mg tid PO x 10 days.


• Decongestant: Tab Sudafed 60mg PO q6h
• Nasal saline.

Acute Pharyngitis

• Group A ß Hemolytic Strep: Tab Penicillin V 300mg PO tid x 10days


• Penicillin allergic: Tab Erythromycin 500mg tid x 10 days
24 NAC OSCE | A Comprehensive Review

9. Pulmonology

Asthma

1. Intermittent Asthma: Short acting beta-agonist - Salbutamol (Ventolin) Inhaler 1-2 puffs q4-6h prn.
2. Mild Intermittent Asthma:
• Long acting beta agonist - Salmeterol Inhaler 1-2 puffs bid.
• Inhaled steroids:
i. Fluticasone (Flovent) 2-4 puffs bid.
ii. Budesonide (Pulmicort) 2 puffs bid.
iii. Beclomethasone (Vanceril) 1-4 puffs (40µg) bid or 1-2 puffs (80µg) bid.
3. Moderate Persistent Asthma:
• Inhaled steroids:
i. Fluticasone (Flovent) 2-4 puffs bid.
ii. Budesonide (Pulmicort) 2 puffs bid.
iii. Beclomethasone (Vanceril) 1-4 puffs (40µg) bid or 1-2 puffs (80µg) bid.
• Long acting beta agonist – Salmeterol Inhaler 1-2 puffs bid.
• Leukotriene Receptor Antagonist:
• Tab Montelukast 10 mg PO qhs.
• Tab Zileuton 600 mg PO qid.
4. Severe Persistent Asthma:
• High dose Inhaled steroids.
• Long acting beta agonist.
• Leukotriene Receptor Antagonist.
• Systemic Steroids:
i. Tab Prednisone 2 mg/kg/day PO (max 60 mg/day).
ii. Inj Methylprednisolone (Depo-medrol) 2mg/kg IV, then 0.5 mg/kg q6h x 5days.

Acute exacerbation of COPD

• Admit with nasal O2. Keep saturation between 88-92% . If silent chest/GCS < 8 or decreased LOC
then intubate.
• Elevated bed > 45 degrees.
• IVF.
• MDI : 8 puffs of Ventolin (Salbutamol) alternate with 8 puffs of Atrovent (Ipratropium) back to back
every 20 mins 3 times.
• Nebulizer : 2cc Ventolin + 1cc Atrovent in 3cc NS q20 mins x 3 times.
• Inj Hydrocortisone 125mg IV stat, if severe.
• Inj Ceftriaxone 1-2 g IV q24h along with
• Inj Piperacillin-Tazobactam 3.375 g IV q6h.
• Inj Methylprednisolone 2mg/kg IV, then 0.5 mg/kgq6h x 5 days.
Therapeutic Guidelines | Medicine 25

Community Acquired Pneumonia

1. Outpatient management:
• Tab Doxycycline 100 mg PO bid x 7-10 days.
• Tab Erythromycin 250 – 500 mg bid x 7-10 days.
• Tab Azithromycin 500 mg PO od x 5 days.
• Tab Levofloxacin 500 mg PO od x 7–10 days.
• Tab Augmentin 500 mg/ 125 mg PO q8h x 5days.
2. Inpatient management:
• Inj Ceftriaxone 1-2 g IV bid along with
• Inj Levofloxacin 500 mg IV od x 7-10 days.
• Inj Azithromycin 50 mg IV over 1 hour od x 1-2 days.

Pulmonary Embolism

1. Investigations
• V/Q scan, spiral CT or D-dimer (if unlikely Wells' score < 4)
• CBC, INR, PTT, BUN, creatinine, ALT, AST.
2. Management: Initiation
• Start Warfarin (Coumadin) concurrent with Heparin.
• Contraindicated in pregnancy. (If contraindicated may put IVC filter)
• Start Tab Warfarin at 5 mg PO daily on Day 1-2 and Heparin 5000IU IV bolus followed by
continuous infusion 20 U/kg/hour, titrate to INR 2-3 then stop heparin within 24 hours.
• Check INR in 3-5 days.
• Therapeutic INR: 2.0 to 3.0 IU.
• Oxygen, and if pain give morphine or NSAID.
3. Management: Duration of Anticoagulation
• Very low risk: 6-12 weeks
• Symptomatic isolated calf vein thrombosis.
• Low risk patient: 3-6 months
• Reversible thromboembolism risk (transient risk such as post-operative PE).
• Upper extremity Deep Vein Thrombosis.
• Moderate risk patient: 6-12 months
• First idiopathic DVT or PE.
• High risk patient: 12 months or lifetime Anticoagulation
• Recurrent DVT or PE or Thrombophilia.
26 NAC OSCE | A Comprehensive Review

10. Rheumatology

Osteoporosis

• Tab Calcium (1500mg/day) and Tab Vitamin D (800 IU/day) intake in diet or as supplements.
• Bisphosphonates: Alendronate, Risedronate or Raloxifene.
• Hormone Replacement Therapy
• Calcitonin
• Recombinant Parathyroid Hormone
• Lifestyle modifications: Weight bearing exercises, smoking and alcohol cessation.

Osteoarthritis

• Tab Tylenol 500 mg PO tid.


• Tab Ibuprofen 200-600 mg PO tid.
• Tab Naproxen 125-500 mg PO bid.
• Tab Celecoxib 200 mg PO od.
• Other treatment:
• Tab Acetamiophen + Tab Codeine.
• Intra-articular corticosteroid injection.
• Intra-articular hyaluron injection.
• Topical NSAIDs.
• Capsaicin cream.
• Glucosamine sulfate.

Rheumatoid Arthritis

1. First Choice:
• Tab Naproxen 500 mg PO bid.
• Tab Ibuprofen 300-800 mg PO qid.
• Tab Indomethacin 25-50 mg PO bid or tid.
2. Analgesics: Tab Acetaminophen 500 mg PO tid prn.
3. Corticosteroids: given intra-articular
i. Small Joints:
• Inj Hydrocortisone 8-20 mg.
• Inj Methylprednisolone 2-5 mg.
• Inj Betamethasone 0.8 – 1.0 mg.
ii. Large Joints:
• Inj Hydrocortisone 40 100 mg.
• Inj Methylprednisolone 10 – 25 mg.
• Inj Betamethasone 2 - 4 mg.
Therapeutic Guidelines | Medicine 27

4. Disease Modifying Antirheumatic Drugs (DMARDs): Start within 3 months of diagnosis to reduce
disease progression.
i. Mild disease:
• Tab Hydroxychloroquine 200 mg PO bid.
• Tab Sulfasalazine 500 m mg PO bid to tid.
ii. Moderate disease:
• Tab Methotrexate 10-15 mg PO once weekly, then increase to 20 mg PO once
weekly.
• Combination therapy:
• Methotrexate + Sulfasalazine + Hydroxychloroquine.
• Methotrexate + Cyclosporine.
• Methotrexate + Etanercept (biological DMARD).
iii. Biological DMARDs: used in persistent disease:
• Etanercept SC.
• Infliximab IV.
• Anakinra SC.
• Adalimumab SC.
• Abatacept IV.
• Rituximab IV.
NOTE:
• If Corticosteroids are used for> 3 months, do baseline DEXA and start bisphosphonate therapy.
• S/E of Corticosteroids: Osteoporosis, cataracts, glaucoma, peptic ulcer disease, avascular necrosis,
hypertension, increased infection rate, hypokalemia, hyperglycemia, hyperlipidemia.
• C/I to Corticosteroids: Active infection, hypertension, diabetes mellitus, gastric ulcer, osteoporosis.

Gout

1. Acute Gout:
i. NSAIDs: Tab Indomethacin 25-50 mg PO tid x 10-14 days.
ii. Tab Naproxen 500 mg PO bid x 4-10 days.
iii. Tab Colchicine 0.6 mg PO q1h till pain relief (max 4-6 doses), then bid x 3-5 days.
iv. Systemic Steroids: (rule out Septic Arthritis)
• Inj Methylprednisolone 40 mg IV single dose
• Inj Dero-Medrol 80-120 mg IM single dose.
• Oral: Tab Prednisone 40 mg PO od x 5days, then gradually taper the dose.
v. Intra-Articular Corticosteroid: used in large single joints & refractory cases.
• Inj Betamethasone 7 mg or Inj ACTH 40-80 IU.
2. Recurrent Gout: Treat for 3-6 months.
i. Over producers: Tab Allopurinol 100-300 mg/day PO.
ii. Under-excreters: Tab Probenecid 250 mg PO bid (max:1500 mg bid) or Tab Sulfapyrizine 50
mg PO bid (max: 1000 mg bid).
iii. Concurrently start with Tab Colchicine 0.6 mg PO bid x 3-6 months.
28 NAC OSCE | A Comprehensive Review

Temporal arteritis

• Start high dose Tab Prednisone 60 mg PO od until symptoms subside and ESR normal
• Then 40 mg PO od for 4-6 weeks
• Then taper to 5-10 mg PO od for 2 years (relapses occur in 50% if treatment is terminated before 2
years). Treatment does not alter biopsy results if the sample is taken within 2 weeks.
• Monitor ESR regularly.
• If visual symptoms are present, or develop during treatment, the patient is admitted and given
Inj Prednisolone 1000 mg IV q12h for 5 days.

Polymyalgia Rheumatica Management

1. General measures
• Consider concurrent Temporal Arteritis (See above)
• NSAIDs
2. Prednisone (key to management)
• See Corticosteroid Associated Osteoporosis
• Efficacy: 90% response
Dramatic improvement in first 48 hours
If no response to steroids – reconsider diagnosis
Reconsider diagnosis
Consider Methotrexate
• Polymyalgia alone
Dose: 15-20 mg PO qd
• Polymyalgia with Temporal Arteritis
Dose: 40-60 mg PO qd
Symptoms and signs remit within 1 month
Decrease dose by 10% each week after improvement
• Course
• Initial: Maintain starting dose for 1 month
• First steroid taper (depends on clinical response)
Taper by 2.5 mg per month down to 10 mg/day then
Taper 1 mg per 4-6 weeks down to 5 to 7.5 mg/day
• Final steroid taper
Indicated when symptom free for 6-12 months
Do not taper until sedimentation rate normalizes
Taper by 1 mg every 6-8 weeks until done
• Anticipate 2-6 year course of steroids
Relapse common in first 18 months of steroid use
Patients off steroids at 2 years: 25%
Therapeutic Guidelines | Medicine 29

Fibromyalgia

1. ANTIDEPRESSANTS : Benefits
• Assists with local pain, stiffness and sleep
• Does not affect Tender Points
2. Tricyclic Antidepressants
• Amitriptyline (Elavil)
i. First week: 10 mg PO qhs
ii. Next three weeks: 25 mg PO qhs
iii. Later: 50 mg PO qhs
• Nortriptyline (Pamelor)
3. Novel Antidepressants
• Venlafaxine (Effexor)
• Duloxetine (Cymbalta)
4. Selective Serotonin Reuptake Inhibitors (SSRI)
• Combination: Fluoxetine and Amitriptyline

Septic Arthritis

• Gonococcal: Inj Ceftriaxone 1g IV q24h x 2-4 days, then switch to Tab Ciprofloxacin 500 mg PO
bid x 7 days.
• Non-Gonococcal: Inj Naficillin 2g IV q4h x 2 weeks, then switch to Tab Ciprofloxacin 500 mg PO
bid x 2-4weeks.

11. Urology/Nephrology

Urinary tract infection (UTI)

1. Acute uncomplicated UTI: outpatient


• Tab Bactrim DS PO bid x 3 days.
• Tab Nitrofurantoin (Macrobid) 100 mg PO bid x 5 days.
2. Drug resistant UTI: outpatient
• Tab Ciprofloxacin 500 mg bid x 3 days.
• Tab Norfloxacin 400 mg PO bid x 3 days.
• Tab Ofloxacin 200 mg PO bid x 3 days.
3. Acute complicated UTI: inpatient
• Inj Ampicillin 1-2 g IV q4-6h and Inj Gentamicin 2mg/kg IV loading dose followed by 1.7
mg /kg q8h IV OD
• Inj Ciprofloxacin 400 mg IV bid.
• Switch to oral antibiotics upon improvement for a total course of 14-21 days.
30 NAC OSCE | A Comprehensive Review

Acute Pyelonephritis

1. Outpatient management: For acute uncomplicated cases


• Tab Ciprofloxacin 500 mg PO bid x 10 days.
• Tab Gatifloxacin 400 mg PO daily x 10 days.
• Tab Moxifloxacin 400 mg PO daily x 10 days.
• Tab Levofloxacin 250 mg PO daily x 10 days.
• Tab Augmentin bid x 14 days.
• Tab Bactrim bid x 14 days.
2. Inpatient management: IV for 48-72 hours, then switch to oral agents. Total duration of treatment for
14 days.
• Inj Ceftriaxone (Rocephin) 1-2 grams IV q24 hours.
• Inj Cefotaxime (Claforan) 1 gram IV q12 hours.
• Inj Ampicillin 2 g IV q6h with Inj Gentamicin 2mg/kg IV loading dose , then 1.7mg/kg
q8h.
• Inj Piperacillin 3.375g IV q6h.

12. Emergency Medicine/Poisoning

Acetaminophen Intoxication

• Toxic level dose is more than 7.5g


• Investigations : Monitor drug level stat and then q4h (Acetaminophen nomogram), LFT, INR, PTT,
BUN, Creatinine, ABG, Glucose
• Rx : Charcoal/Gastric lavage as per presentation
N-acetyl cysteine 140mg/kg PO, then 70mg/kg q4h for 18 doses

Alcohol withdrawal

• Treatment : Inj Diazepam 10-20mg IV


Inj Thiamine 100mg IM then 50-100mg/day
Fluid resuscitation with D5W 1-2mL/kg IV

Allergic Reaction

1. Severe: Inj Epinephrine 0.3-0.5 mg SC/IM stat


2. Mild: Tab Benadryl 25-50mg PO q6h x 3d
3. Tab Prednisone 60mg PO od x 3d

Anaphylaxis

• Epinephrine autoinjector (EpiPen) if available


• Epinephrine IV or ETT : 1ml of 1:10,000 in adults
• Inj Diphenhydramine (Benadryl) 50mg IV or IM q4-6 h
• Inj Methylprednisone 50-100mg IV according to severity
• If wheezing or spasm present : Salbutamol via nebulizer.
Therapeutic Guidelines | Medicine 31

Arrhythmias

• Arrhythmias due to 2nd degree and 3rd degree heart block :


Inj Atropine 0.5mg IV while waiting for transcutaneous pacing.
Transcutaneous pacing first (give Inj Midazolam 2mg for sedation)
Admit for trans venous pacing
• Unstable patients (hypotensive systolic BP < 90, chest pain, SOB, altered mental status or
unconscious) : CARDIOVERT!
• Stable patient :
Atrial fibrillation : either chemical cardioversion (Amiodarone) or electrical (Synchronized DC
cardioversion)
Ventricular tachycardia : DC cardioversion or InjLidocaine/Amiodarone 150mg IV over 10 mins.
Ventricular fibrillation : Always defibrillate! Synchronized cardioversion not useful because there is no
QRS complex to synchronize with.
PSVT : Valsalva or carotid massage (after checking for bruit), Inj Adenosine 6mg rapid IV push.
If no response then Metaprolol, Diltiazem.

ASA Intoxication

• Investigations : Drug levels, electrolytes, ABG, BUN, Creatinine


• Rx : Gastric lavage/Charcoal
Alkalinize urine with D5W, KCl and NaHCO3
Aim : urine pH > 7.5

Diabetic ketoacidosis

• Estimated daily basal glucose requirement is 0.5U/kg


• Investigations : Blood glucose, electrolytes, ABG, serum ketones, osmolar gap, anion gap, BUN,
creatinine. Look of the cause : Urinalysis, blood C&S, chest x-ray, ECG.
• Monitor : Urine output, extra-cellular fluid volume, electrolytes, ABG, creatinine,
capillary blood glucose and level of consciousness every 1-2 hours.
• Management : Rehydration : NS 1L/h in first 2 hours followed by 0.45% NS 500cc/h then switch
to maintain blood glucose 13.9-16.6mmol/L to avoid rapid decrease of osmolality.
K+ replacement : As acidosis is corrected, hypokalemia may develop.
If K+ is 3.3-5.0 mmol/L, add KCl 20-30 mEq/L to keep it within this range.
Correct acidosis : If pH < 7.0/hypotension/coma then give 3 amp NaHCO3
(150mEq/L)
Reduce blood glucose : Start Insulin therapy with 0.15U/kg bolus and maintain
0.1U/kg/h until acidosis and blood glucose resolve.
Treat underlying precipitant.
32 NAC OSCE | A Comprehensive Review

Digoxin Intoxication

• Investigations : Plasma digoxin/digitoxin levels, ECG, electrolytes, BUN, Cr ( levels > 2.6 indicate
intoxication)
• Rx : Treat arrhythmias (common with digoxin intoxication; vfib, vtach, conduction blocks)
Gastric lavage / Charcoal (1g/kg) for ingestion
NaHCO3 or glucose and insulin
Ventricular tachycardia : Digibind 10-20 vials if dose unknown
Chronic toxicity : then Digibind 3-6 vials IV over 30 mins.
Follow ECG, K+, Mg+, Digoxin levels every 6 hours.

Hypertensive emergency

• Systolic BP ≥ 180mmHg and Diastolic BP ≥ 120mmHg (with signs of acute organ damage)
• Investigations : CBC, electrolytes, BUN, Creatinine, ABG, Urinalysis, CXR, ECG, BP in all four
limbs, Fundoscopy, Cardiology consult.
• 1st Line : Inj Sodium nitroprusside 0.3 mcg/kg/min IV OR Inj Labetalol 20mg IV bolus q 10 mins.
• Aortic dissection : Sodium nitroprusside + b blocker (esmolol)
• Catecholamine excess : Inj Phentolamine 5-15mg IV q 5-15 mins
• MI/Pulmonary edema : Inj Nitroglycerin 5-20mcg/min IV, increase by 5mcg/min every 5 min till
symptoms improve.

Hypoglycemia

• Investigations : Baseline blood glucose, insulin and C-peptide, check glucose q15 mins
until > 5mmol/L
• Rx : If patient can eat/drink : give 15g carbohydrate if BG < 4mmol/L (15g glucose tabs or ¾ caps
of juice or 3 spoons of sugar in water.)
NPO : give 25g carbohydrate if BG < 4mmol/L ( D50W 50cc IV push 1 amp OR
D10W 500cc IV OR glucagon 1-2mg IM/SC )

Methanol/Ethylene glycol intoxication

• Investigations : CBC, electrolytes, glucose, methanol level.


• Rx : Ethanol 10mg/kg over 30 mins OR Inj Fomepizole 15mg/kg IV over 30 mins.
Therapeutic Guidelines | Medicine 33

Opioid Intoxication
• Mental status effects include euphoria, sedation, decreased anxiety, a sense of tranquility and
indifference to pain produced by mild-to-moderate intoxication. Severe intoxication can lead to
delirium and coma.
• Physiological effects include the following:
Respiratory depression (may occur while the patient maintains consciousness)
Alterations in temperature regulations
Hypovolemia (true as well as relative), leading to hypotension
Miosis
Soft tissue infection
Increase sphincter tone (can lead to urinary retention)
• Treatment
IV glucose : 50% Dextrose 50ml
Inj Nalaxone 0.4mg upto 2mg IV for reversal of opioid intoxication.
Inj Thiamine 100mg IM stat & OD x 3days
O2, intubation & mechanical ventillation

Shock (Cardiogenic/Neurogenic)

• Dopamine : 1-3mcg/kg/min is the renal dose; 4-10mcg/kg/min is the inotropic dose


• Dobutamine : 2.5-5mcg/kg/min

Sprain (Ankle) RICE

• Rest
• Ice : using bag of ice, apply during the day for 5-20 mins every 2 hours.
• Compression : Tensor bandage or special supports.
• Elevation : Elevate the ankle as much as possible.
• Analgesics as needed.
• Crutches if too painful to bear weight.

Stroke

• Investigations : CBC, electrolytes, BUN, glucose, creatinine, INR/PTT, lipids, ECG, carotid doppler
if suspecting TIA, ABG, Non contrast urgent CT scan.
• Treatments : NPO, Foley catheter, DVT prophylaxis, Neurology consult
Rule out contraindications for thrombolytic treatment.
Uregent neurology consult.
Thrombolysis : rTPA within 3 hours of symptoms
Anti-coagulation : Low dose Heparin 5000 U bid, start Warfarin within 3 days,
monitor INR/PTT
If unable to thrombolyse or anti-coagulate then : Tab ASA 50-325mg od or
Tab Clopidogrel 75mg od
BP control : decrease slowly, IV Labetalol (First line treatment)
Bed rest, analgesics, mild sedation and laxatives, avoid hyperglycemia.
34 NAC OSCE | A Comprehensive Review

TCA Intoxication

• Patients who present to the ED following psychotropic drug overdose with GCS ≤ 8 should undergo
intubation at the earliest opportunity to prevent hypoventilation and aspiration pneumonia.
• Investigations : Drug levels, ECG, ABG, electrolytes, LFTs, RFTs.
• Rx : Activated charcoal 1gm/kg via NG
Diazepam for seizures
Wide QRS/Seizures : NaHCO3 ( 1-2 mEq/kg bolus dose and then 100-150 mEq in
1L D5/0.45% NaCl infused 100-200 ml/h IV)

Upper GI Bleed

• Stabilize patient with IVF, cross & type, 2 large bore IV cannulas.
• Investigations : CBC, platelets, INR, BUN, creatinine, PTT, electrolytes, LFTs
• Management : NG tube, NPO, blood transfusion if needed, upper GI endoscopy
Inj Octreotide 50mcg loading and 50mcg per hour (for varices) SC/IV
Inj Pantoprazole 50mg IV stat and 50mg q8h (gastric ulcer)

Lower GI Bleed

• Stabilize patient with IVF, cross & type, 2 large bore IV cannulas.
• Investigations : CBC, platelets, INR/PTT, BUN, creatinine, electrolytes.
• Management : NG tube, NPO, blood transfusion if needed, sigmoidoscopy, colonoscopy, angiogram
(forangiodysplasia)

Warfarin Intoxication

• Treatment according to INR levels


INR < 5 : Stop warfarin, observation, serial INR/PTT
INR 5-9 : If no risk factors for bleeding, hold warfarin x 1-2 days & reduce maintenance dose.
OR Vitamin K 1-2 mg PO, if patient at increased risk or FFP for active bleeding.
INR 9-20 : Stop warfarin, Vitamin K 2-4 mg PO, serial INR/PTT then additional Vitamin K if
needed or FFP for active bleeding.
INR > 20 : FFP 10-15ml/kg, Inj Vitamin K 10mg IV over 10 min, increase dose of Vitamin K (q4h)
if needed.
Therapeutic Guidelines | Medicine 35

13. Counselling

Smoking cessation

1. Nicotine gums: 2mg if < 25 cig/day, 4mg if > 25cig/day


• 1 piece q1-2h for 1-3mths
2. Nicotine patch:
• 21mg per day for 4 weeks
• 14mg per day for 2 weeks
• 7mg per day for 2 weeks
3. Nicotine inhaler: 6-16 cartridges per day upto 12 weeks
4. Bupropion(Zyban):
• 150mg qAM x 3days, then 150mg bid for 7- 12 weeks
• Maintenance 150mg bid for upto 6 months.
• General
Stop smoking during second week of medication
Stop Bupropion if unable to quit by 7 weeks
Minimum of 8 hours between doses
More is not better
Swallow pills whole (not crushed, divided or chewed).

Alcohol cessation

Protocol: Alcohol Dependence CAGE Questionnaire

• Lab markers C : Have you ever felt the need


Serum Gamma glutamyl transferase or to CUT down on your
drinking?
Carbohydrate deficient Transferrin A : Have you ever felt
ANNOYED at criticism of
1.Initial Management your drinking?
• Tab Thiamine 100 mg PO qd G : Have you ever felt GUILTY
about you drinking?
• Tab Folate 1 mg PO qd
E : Have you ever had a drink
• Multivitamin qd first thing in the morning
• Treat Hypomagnesemia if present (EYE OPENER)?
• Seizure precautions
2.Long-Term Abstinence Programs
• Alcoholics Anonymous
• Sponsor
• Treatment Program
• Halfway House
36 NAC OSCE | A Comprehensive Review

3.Adjunctive Medications for abstinence


1.First line (consider Naltrexone with Campral)
• Tab Naltrexone
Blocks Opioid receptors
Decreases pleasure from Alcohol
Dosing: 50 mg orally daily
Effective in short-term, but not in long-term
• Tab Campral (Acamprosate)
Balances GABA and glutamate neurotransmitters
Reduces anxiety from abstinence
Dosing: 2 tabs PO tid
2.Second line agents to consider
• Selective Serotonin Reuptake Inhibitors (SSRI)
Consider especially if comorbid depression
Prozac often used, but other SSRIs effective
• Topiramate (Topamax)
Decreases Alcohol use severity and binge drinking
Improves well being, quality of life in Alcoholics
3.Agents to avoid
• Antabuse
Taken 250 to 500 mg orally daily
Not recommended due to risk and uncertain benefit

• Delirium Tremens

General Protocol (Requires ICU observation)


• Tab Diazepam (Valium)
Dose: 10-25 mg PO q1h prn while awake
Endpoint: until adequate sedation
• Inj Lorazepam (Ativan)
Dose: 1-2 mg IV q1h prn while awake for 3-5 days
Endpoint: until adequate sedation
• Librium (Chlordiazepoxide)
Dose: 50 to 100 mg PO/IM/IV q4h (max: 300 mg/day)
Endpoint: until adequate sedation
NOTES
38 NAC OSCE | A Comprehensive Review

Obstetrics & Gynecology


1. Sexually Transmitted Infection
a. Chlamydia:
Tab Azithromycin 1g PO stat or Tab Doxycycline 100mg PO bid x 7 days
If pregnant: Tab Erythromycin 500mg PO tid x 7 days.
Treat partner, Reportable disease.

b. Gonorrhea:
Inj Ceftriaxone 125mg IM stat + Tab Doxycycline 100mg bid x 7 days.
If pregnant : Inj Spectinomycin 2g IM stat
Treat partner, Reportable disease.

c. Syphilis:
Primary, Secondary, Latent Syphilis (duration less 1 year ):
Inj Benzathine Penicillin G 2.4 MU IM for 1 dose
Treat partner, Reportable disease.
If allergic to Penicillin: Tab Doxycycline 100 mg PO bid for 14 days.
Late latent, Cardiovascular (duration over 1 year)
Inj Benzathine Penicillin G 2.4 MU IM once a week for 3 weeks
If Penicillin allergic : Tab Tetracycline 500 mg PO qid for 4 weeks or
Tab Doxycycline 100 mg PO bid for 4 weeks
Neurosyphilis : Inj Aqueous Penicillin G 3-4 MU IM every 4 hours for 10-14 days.

d. Genital herpes:
First episode: Tab Acyclovir 400mg PO tid x 10 days or
Tab Famciclovir 250 mg tid x 10 days or
Tab Valacyclovir 1 g bid x 10 days
Recurrent: Tab Acyclovir 400mg PO tid x 5 days or
Tab Famciclovir 120 mg bid x 5 days or
Tab Valacyclovir 500 mg bid x 5 days
Suppression: if more than 6 episodes per year
Tab Acyclovir 400mg PO bid x 12 months
Severe episode: Inj Acyclovir 5-10 mg/kg q8h x 5-7 days

e. Genital warts (HPV):


Local treatment with LIQUID NITROGEN repeat every 1-2 weeks
Podophyllotoxin 0.5% gel bid x 3days,then 4 days off – to be repeated for 4
weeks.
Prophylaxis for HPV (for Cervical CA & warts) – Inj Gardasil IM 0, 2 and 6 months.
Therapeutic Guidelines | Obstetrics & Gynecology 39

GENERAL INSTRUCTIONS for all sexually transmitted infections:


• Treat all partners
• Avoid sexual intercourse till treatment completion.
• Barrier contraception/ educate about safe sex practices.
• Rescreening in 3 months.

SIDE EFFECTS:
• DOXYCYCLINE: Drug induced PHOTOSENSITIVITY, use sun screen
• ACYCLOVIR: headache, GI upset, impaired renal function, tremors, agitation, lethargy,
confusion, coma

2. Urinary Tract Infection

Uncomplicated:
Tab Bactrim DS PO bid x 3 days or
Tab Nitrofurantoin 100mg PO qid x 5days. (with food)
In pregnancy: Treat asymptomatic UTI
Tab Amoxicillin 250mg PO tid or
Tab Macrobid 100mg PO bid x 10 days.
Pyelonephritis: Acute Uncomplicated:
Tab Ciprofloxacin 500mg PO bid x 10 days or
Tab Augmentin 625mg PO bid x 14 days.

Inpatient: Inj Ceftriaxone 1g IV bid for 48 hours then switch to oral drugs +
Inj Gentamicin 50mg IV q8h for 24 hours.

3. Vulvovaginitis
a. Candidiasis:
Tab Miconazole 200mg PV qhs x 3 days or
Tab Nystatin (100,00 unit) vaginal tab PV qhs x 14 days or
Tab Fluconazole 150mg PO stat dose.
Prophylaxis: 4 or more infection per year – Tab Fluconazole 150mg PO every
3days for 3 doses.
Maintenance: Tab Fluconazole 150mg PO each week. Monitor liver enzymes every 1-2
months.
b. Bacterial vaginosis:
Tab Flagyl 500mg PO bid x 7days.(with food)
c. Trichomonas vaginalis:
Tab Flagyl 2g PO for 1 dose. or
Tab Flagyl 500mg PO bid x 7days.(with food), treat partner.
d. Atrophic vaginitis:
Topical Estrogen cream 0.5 to 2g daily to be applied locally.
40 NAC OSCE | A Comprehensive Review

4. Pelvic Inflammatory Disease (PID)

a. Outpatient: Inj Ceftriaxone 250mg IM stat dose + Tab Doxycycline 100mg PO bid x 14days.
b. Inpatient: Inj Cefoxitin 2g IV q6h + Inj Doxycycline 100mg IV q12h.
Continue IV for 48 hrs & then tab Doxycycline 100mg PO bid x 14 days.
Reportable disease, treat partners, rescreening after 4-6 weeks incase of documented
infection.

5. Dysfunctional Uterine Bleeding (DUB)

a. Mild DUB:
• NSAIDs – Tab Mefenamic acid 500mg PO tid x 5 days,
• Anitfibrinolytics – Tranexamic acid 500mg PO tid x 5 days, Combined OCPS
• Mirena / Provera
• Tab Progestin one tab OD in first 10-14days.
b. Severe DUB:
• Inj Premarin 25mg IV q4h + Tab Gravol 50mg PO q4h.
• With Tab Ovral PO tid till bleeding stops (24hrs),THEN bid for 2 days, THEN od for
3days.
• Continue conventional OCPs if pregnancy not desired.

6.Dysmenorrhea

• Tab Ibuprofen 400mg PO qid from 1st day of menstrual cycle.


• Oral Contraceptive Pills.
• Important to rule out secondary causes of dysmenorrhea.

7. Endometriosis

a) NSAIDs : Tab Ibuprofen 400 mg PO qid till symptoms last.


b) Oral Contraceptive pills.
c) Tab Provera 10-20 mg PO OD.
d) Tab Danazol 600-800 mg PO OD for 6 months.
e) GnRH Agonist: Inj Leuprolide 3.75 mg IM once a month for 6 months.
Inj Goserelin 3.6 mg SC every 28 days for 6 months.
Use GnRH Agonist along with Estrogen/Progesterone add back therapy. (To reduce the side
effects of bone loss.)
Therapeutic Guidelines | Obstetrics & Gynecology 41

8. Hormone Replacement Therapy (HRT)

a) Only Estrogen - Tab Premarin 0.625mg PO OD ( only estrogen)


b) Cyclic Dose – Tab Premarin 0.625mg PO OD and Tab Provera 5-10mg PO OD from days 1-14.
c) Standard dose - Tab Prempro (premarin 0.625mg and provera 2.5mg) combination pill PO OD.
d) Pulsatile – Tab Premarin 0.625mg PO OD and Low dose Tab Provera 1.5 mg PO OD.
Given as 3 days on and 3 days off.
e) Transdermal : Estradiol transdermal patch twice daily and Tab Provera 2.5 mg PO OD.

9. Emergency contraception

• OTC no prescription needed. Take within 72 hours of unprotected intercourse.


• Tab Ovral 2 tabs PO q12h x 2 doses (has Levonorgestrel 0.5mg/dose + estrogen 0.1mg/dose) +
Tab Benadryl 10mg 1 hr before dose (emesis induced by Estrogen).
• Plan B ( Tab Levonorgestrel 0.75mg/tab) one tab q12hrs x 2 doses.

10. Group B Streptococcus (GBS) in pregnancy

• Inj Penicillin G 5 MU IV then 2.5 MU IV q4h till delivery.


• Penicillin allergic: Inj Cefazolin 2 g IV then 1 g q8h or
Inj Clindamycin 900 mg IV q8h or
Inj Erythromycin 500 mg IV q6h.

11. Pregnancy Induced Hypertension (PIH)

a. Initial: To maintain DBP<100


• Inj Labetalol 20mg IV bolus every 10-20 mins prn. (C/I asthma,CHF)
• Tab Nifedipine XL 10mg PO very 20-30 mins prn.
• Inj Hydralazine 5mg or 10mg IM every 20 mins prn, then 5 or 10mg every 3 hrs prn.
(S/E: fetal tachycardia, maternal headache, palpitations)
b. Maintenance:
• Tab Methyldopa 250-500mg PO bid – qid.
• Tab Metoprolol 25-100mg PO bid.
• Tab Labetalol 100-400mg PO qid.
c. Anticonvulsant therapy:
• Inj Magnesium sulfate 4g IV bolus over 20 min, followed by maintenance of 2-4 g/hour.
• Monitor signs of Magnesium toxicity – depressed deep tendon reflexes, decreased respiratory
reflex, anuric, hypotonic, CNS or cardiac depression.
• Antagonist to Magnesium sulfate: Calcium gluconate(10%) 10 ml IV over 2 minutes.
d. Avoid these antihypertensives:
• ACE(-) & ARBs – neonatal renal failure, teratogenic, IUGR.
• Atenolol – IUGR
• Thiazide diuretics – maternal fluid depletion.
42 NAC OSCE | A Comprehensive Review

12. Ectopic Pregnancy

a) Inj Methotrexate 50mg/m2 BSA IM stat dose.


b) Repeat beta hCG levels weekly till <1.
c) Contraception till beta hcg returns to 5mIU/ml or less.
d) Do CBC, LFTs.

13. Hyperemesis Gravidarum

Tab Diclectin (10 mg Doxylamine with 10 mg Pyridoxin) started as 1 tab qAM + 1 tab qPM +
2 tabs qhs. Maximum 8 tabs per day.

14. Drugs contraindicated in pregnancy

• Chloramphenicol: Gray baby syndrome


• Erythromycin: Maternal liver damage (used only if allergic to penicillin).
• Fluoroquinolones: Cartilage damage.
• Metronidazole: Anti-metabolite, high risk in 1st trimester and breast feeding. Can lead to miscarriage.
• Sulfa drugs: Miscarriage in 1st trimester and kernicterus in 3rd trimester.
• Tetracyclines: Staining of teeth in children.
• ACE inhibitors: IUGR, oligohydraminos, fetal renal defects.
• Anticonvulsants:
▪ i) Phenytoin: Fetal hydantoin syndrome – IUGR, facial dysmorphogenesis, cardiovascular
defects, congenital anomalies of hand & foot, umbilical hernia and congenital
anomalies.
▪ ii) Valproic acid: Lumbosacral spina bifida with meningomyelocele or meningocele, often
accompanied by midfacial hypoplasia, deficient orbital ridge, prominent forehead,
congenital heart disease, and decreased postnatal growth.
▪ iii) Carbamazepine: Unique facial appearance and underdevelopment of the fingers, toes, and
nails; developmental delay.
▪ iv) Phenobarbital: Cleft palate/lip, congenital heart disease, intra-cranial hemorrhage.
• DES: Vaginal adenosis, adenocarcinoma, uterine malformations in female fetuses.
• Lithium: Ebstein’s cardiac anomaly, goiter, hyponatremia.
• Misoprostol: Congenital facial paralysis with or without limb defects (Mobius syndrome) and Neural
tube defects.
• Retinoids: Deformities of the cranium, ears, face, limbs, and liver, hydrocephalus, microcephaly,
heart defects, cognitive defects, craniofacial alteration, cleft palate, neural tube defects,
cardiovascular malformations and kidney alterations.
Therapeutic Guidelines | Obstetrics & Gynecology 43

• Warfarin: High risk of spontaneous abortion, stillbirths, IUGR.


Fetal Warfarin Syndrome: Deformities of the axial and appendicular skeleton, hypoplastic
nose, optic atrophy, mental retardation, brachydactyly, scoliosis, mental retardation, intra-
cranial hemorrhage.
• Alcohol: High incidence of abortion and still births, IUGR.
Fetal Alcohol Syndrome: Decreased muscle tone and coordination, cognitive impairment,
ASD/VSD, narrow small eyes with large epicanthal folds, small head, small mid-face,
indistinct philtrum, thin upper lip.
• Cigarette smoking: IUGR, placental abruptio/ previa, spontaneous abortion.
• Cocaine: IUGR, microcephaly, prematurity, mental retardation.
NOTES
Therapeutic Guidelines | Pediatrics 45

Pediatrics
1. Acute Bronchiolitis

a. Mild distress: oral/IV hydration, antipyretics for fever, humidified O2, VENTOLIN 0.03cc in 3ml NS
by face mask q20min and then q1hr.
b. Moderate to severe distress: all the above + Ribavirin in high risk groups like congenital lung disease,
congenital heart disease, bronchopulmonary dysplasia, immunodeficient patients.
c. Antibiotics, ipratropium, systemic corticosteroids have no use.

2. Acute Otitis Media (AOM)

a) First line:
Tab Amoxicillin 80-90mg/kg/day PO divided q8h for 10d.
If allergic – Tab Azithromycin 10mg/kg/day OD for 3 days. To be given if child > 6months
old.
b) Second line:
Tab Augmentin 90mg/kg/day divided q12h for 10 days or
Tab Cefuroxime 30mg/kg/day divided bid for 10 days.
Avoid FLUOROQUINOLONES under 16 years age.

3. Asthma

• Classification (NIH recommendations)


a) Intermittent Asthma- Occasional exacerbations (Less than twice per week).
b) Mild Persistent Asthma- Frequent exacerbations (>twice weekly, but not daily).
c) Moderate Persistent Asthma- Daily symptoms with daily Beta Agonist use
d) Severe Persistent Asthma- Continuous Symptoms and frequent exacerbations.
• Acute Management
i. O2 (to maintain O2 saturation > 90%).
ii. Fluids, if dehydrated.
iii. β2 Agonist : Salbutamol (Ventolin)- 0.03 cc/kg in 3cc NS every 20 minutes for 3 doses then
0.15-0.3 mg/kg (not to exceed 10 mg) every 1-4 hours as needed or 0.5 mg/kg/hour by
continuous nebulization.
iv. If Severe – Ipratropium bromide (Atrovent) 1cc added to each of first 3 salbutamol masks.
v. Steroids: Inj Prednisone 2mg/kg in ER, then 1mg/kg PO OD x 4d.

4. Bacterial Tracheitis
• Airway management, keep child calm.
• Humidified O2
• Nebulized racemic epinephrine(1:1000 solution) in 3ml NS, 1-3 doses, q1-2h.
• Inj Ceftriaxone 75-100mg/kg/day q24hrs + Inj Vancomycin 40mg/kg/day in divided doses every
6-8h.
46 NAC OSCE | A Comprehensive Review

5. Bacterial Pneumonia

Newborn (under 3 weeks old)


1. Admit all newborns with Pneumonia.
2. Antibiotic regimen (Use 2-3 antibiotics combined)
a) Antibiotic 1: Ampicillin
i. Age <7 days
• Weight <2 kg: 50-100 mg/kg divided q12h.
• Weight >2 kg: 75-150 mg/kg divided q8h.
ii. Age >7 days
• Weight <1.2 kg: 50-100 mg/kg divided q12h
• Weight 1.2-2 kg: 75-150 mg/kg divided q8h.
• Weight >2 kg: 100-200 mg/kg divided q6h.
b) Antibiotic 2: Gentamicin (dosing below if >37 weeks old)
• Age <7 days: 2.5 mg/kg repeated q12h.
• Age >7 days: 2.5 mg/kg repeated q8h.
c) Antibiotic 3: Cefotaxime (optional)
• Age <7 days: 100 mg/kg divided q12h.
• Age >7 days: 150 mg/kg divided q8h.
3. Organisms requiring additional antibiotic coverage
i. Methicillin Resistant Staphylococcus Aureus (MRSA)-Vancomycin
a) Age < 7 days :
• Weight < 1.2 kg : 15 mg/kg IV OD.
• Weight 1.2 – 2 kg : 10-15 mg/kg IV q12-18h.
• Weight > 2 kg 10-15 mg/kg IV q8-12h.
b) Age > 7days , weight > 2 kg : 45-60 mg/kg/day in divided IV q8h.
ii. Chlamydia trachomatis-Erythromycin 30-50 mg/kg/d PO divided q8h.

Management: Age 3 weeks to 3 months


1. Outpatient (if afebrile without respiratory distress)
i. Azithromycin 10 mg/kg day 1, 5 mg/kg days 2-5 PO.
ii. Erythromycin 30-40 mg/kg/day PO divided q6h x10days.
2. Inpatient (if febrile or hypoxic)
i. Inj Erythromycin 40 mg/kg/day IV divided q6h and
ii. One of the following antibiotics if febrile:
• Inj Cefotaxime 200 mg/kg/day IV divided q8h.
• Inj Cefuroxime 150 mg/kg/day IV divided q8h.
3. Critically ill
i. Inj Cefotaxime as above and Inj Cloxacillin or
ii. Inj Cefuroxime alone as above
Therapeutic Guidelines | Pediatrics 47

Management: Age 3 months to 5 years


1. Outpatient (if afebrile without respiratory distress)
a) Consider initial parenteral antibiotic at diagnosis:
• Inj Ceftriaxone 50 mg/kg/day up to 1 gram IM x1 dose.
• Start oral antibiotics concurrently as below.
b) First-line oral agents:
• Amoxicillin 90 mg/kg/day PO divided q8h x7-10d.
c) Alternative oral agents:
• Amoxicillin-Clavulanic Acid (Augmentin) .
• Erythromycin.
• Clarithromycin.
• Azithromycin.
2. Inpatient (if febrile or hypoxic):
a) Inj Cefotaxime 150 mg/kg/day IV divided q6h.
b) Inj Cefuroxime 150 mg/kg/day IV divided q8h.
c) If confirmed Pneumococcal Pneumonia:
• Inj Ampicillin alone 200 mg/kg/day IV divided qh.
3. Critically ill:
a) Option 1
• Inj Cefotaxime 150 mg/kg/day IV divided q6h and
• Inj Erythromycin 40 mg/kg/day IV divided q6h.
b) Option 2
• Inj Cefuroxime 150 mg/kg/day IV divided q8h and
• Inj Cloxacillin 150-200 mg/kg/day IV divided q6h.

Management: Age 5 to 18 years


1. Outpatient:
a) First-line oral agents:
i. Erythromycin 40 mg/kg/day PO divided q6h x 7-10d.
ii. Clarithromycin 15 mg/kg/day PO divided q12h x 7-10d.
iii. Azithromycin
• Day 1: 10 mg/kg day 1 PO (maximum 500 mg).
• Days 2-5: 5 mg/kg/day PO (maximum 250 mg).
b) Pneumococcal Pneumonia confirmed:
i. Amoxicillin 90 mg/kg/day PO divided q8h x 7-10d.
2. Inpatient:
a) First line and in critical illness:
i. Inj Cefuroxime 150 mg/kg/day IV divided q8h and
ii. Inj Erythromycin 40 mg/kg/day IV divided q6h.
b) Pneumococcal Pneumonia confirmed:
i. Inj Ampicillin 200 mg/kg/day IV divided q8h.
48 NAC OSCE | A Comprehensive Review

6. Croup (Laryngotracheobronchitis)

a) Humidified O2
b) Nebulized racemic epinephrine(1:1000 solution) in 3ml NS, 1-3 doses, q1-2h :
• Child < 6mths: 0.25ml
• Child > 6mths: 0.5ml
• Adolescent: 0.75ml
c) Dexamethasone 0.6mg/kg IM/IV/PO, max dose 10mg, given as a single dose.

7. Epiglottitis

• Suspect epiglottitis if child has fever, ill looking, dyspnea, dysphonia, loss of voice, stridor, sudden in
onset.
• Investigations : Pharyngeal swab and culture
Blood culture
Lateral X-ray neck (Thumbprint sign)
ABG, CBC
Endoscopy in ER
• Treatment : Intubation
IV fluid
IV Cefuroxime

8. Streptococcal Pharyngitis (Group A Streptococcus)

McIsaac Criteria – no cough, tender anterior cervical lymph nodes, erythematous tonsils with exudate,
fever > 38°C, age 3-14 years.
a) If 1 symptom only – no culture or antibiotics needed.
b) If > 1 symptom, culture positive – treat with antibiotics:
Penicillin V 40 mg/kg/day PO divided bid x 10 days.
Erythromycin 40mg/kg/day PO divided tid x 10 days.
Acetaminophen for fever or pain.
c) Invasive GAS: needs admission –
Inj Clindamycin 40 mg/kg divided into 3-4 doses and
Inj Penicillin 250 000 – 400 000 U/kg/day divided into 6 doses x 10 days.

9. Whooping Cough (Pertussis)

a) Erythromycin 40-50mg/kg/day PO divided qid x 10d.


b) Azithromycin 10mg/kg/PO OD day1, 5mg/kg PO OD day2 to day5. (preferred)
c) Isolate for 5 days of treatment.
d) Erythromycin to all the household members.
Therapeutic Guidelines | Pediatrics 49

10. Bacterial Meningitis (Reportable disease)

a) Inj Dexamethasone 0.6 mg/kg/day IV in 4 divided doses. Start within 1 hour of 1 st antibiotic
dose..
b) Ampicillin:
i. Age< 1 month – 50 mg/kg IV q8-12h.
ii. Age>1 month – 50 mg/kg IV q6h.
c) Cefotaxime:
i. Age < 1 month – 50 mg/kg IV q8-12h.
ii. Age>1 month – 200 mg/kg/day IV divided q6-8h.
d) Ceftriaxone:
i. Age< 1 month – 50-75 mg/kg IV divided q12-24h.
ii. Age> 1 month – 100 mg/kg/d IV divided q12h.
e) Gentamycin: 2-2.5 mg/kg IV q8h.
f ) Vancomycin: 15 mg/kg q6h IV x 7-14 days.
g) Prophylaxis for contacts:
i. H. Influenzae : Rifampin 20 mg/kg/day up to 4 days.
ii. N. Meningitides :
• Rifampin
• Children: 10 mg/kg PO q12h x 2 days (max 600 mg).
• Adults: 600 mg PO q12h x 2 days.
• Ciprofloxacin (adults) 500mg PO for one dose.
• Ceftriaxone :
• Age<15 years: 125 mg IM for one dose.
• Age> 15 years: 250 mg IM for one dose.

11. Febrile Seizures

a) In ER : Inj Diazepam 0.2 – 0.5mg/kg IV q15mins till seizure stops.


b) Home : Diazepam rectal suppository.
c) Investigate & treat the cause of fever.

12. Urinary Tract Infection (UTI)

1. Oral Treatment- for 7-14 days.


i. Cefixime (Suprax) 8 mg/kg PO divided bid or
ii. Cefpodoxime (Vantin) 10 mg/kg PO divided bid or
iii. Cefprozil (Cefzil) 30 mg/kg PO divided bid or
iv. Cephalexin (Keflex) 50-100 mg/kg PO divided qid.
2. IV antibiotics
i. Inj Cefotaxime 50-150mg/kg/day divided q4-6h or
ii. Inj Ceftriaxone 50-75mg/kg/day divided q12-24h.
3. UTI Prophylaxis
i. Bactrim (2mg TMP/10mg SMZ per kg) at bedtime qhs.
50 NAC OSCE | A Comprehensive Review

13. Allergic reaction

a) General Measures:
i. ABC management.
ii. Oxygen.
iii. IVF.
iv. Nebulised beta-agonist (Albuterol).
b) Anaphylaxis with airway compromise: Epinephrine (1:1000 solution) 0.01ml/kg SC/IM(upto 0.3ml)
c) Urticaria, Pruritus or Flushing: Inj Diphenhydramine 25-50mg IM/IV every 6hrs prn. Orally same
dose q6h x 3days.
d) Prevention:
i. Medical alert bracelet.
ii. Strict avoidance of allergen.
iii. EpiPen.
iv. Allergy testing and desensitization therapy.

14. Anemia in children

• 6mg/kg/day elemental iron bid-tid.


• Investigate the cause of anemia.

15. Dose of Tylenol in children

a) 40-60 mg/kg/day PO divided q6hr prn (not to exceed 5 doses/24 hours).


b) Neonates: 10-15 mg/kg PO q6-8hr prn.

16. Immunization Schedule for Infants and Children in Canada


NOTES
52 NAC OSCE | A Comprehensive Review

Psychiatry

1. Agitation & Aggression


Agitation can be defined as excessive verbal and/or motor behavior. It can readily escalate to aggression, which
can be either verbal (vicious cursing and threats) or physical (toward objects or people).

Goals of treatment (Nonpharmacological treatment)

• Create a safe environment for treatment


• Decrease stimulation
• Permit patient to ventilate his or her feelings, but this may need to be cut short if the degree of
agitation is escalating and there is clear danger to self and others.
• Behavioral approaches include never turning your back to an agitated patient, talking softly rather than
shouting, and inquiring about what specific needs the patient may have
• Prevent further episodes of agitation or aggression

Delirium

• Delirium or acute confusional state is a common and severe neuropsychiatric syndrome with core
features of acute onset and fluctuating course, attentional deficits and generalized severe
disorganization of behavior. Treatment of delirium requires treatment of the underlying causes.
• Antipsychotics are first-line treatment. Haloperidol is the most effective medication for decreasing
agitation in delirious patients. First generation antipsychotic Loxapine and second generation
(atypical) antipsychotics such as Olanzapine,Risperidone and Quetiapine can also be used.
Benzodiazepines should be reserved for cases of alcohol withdrawal.

First-generation Antipsychotics (FGA)


• Haloperidol : 0.5-2.5 mg PO/IM bid ANTIPSYCHOTICS S/E
FGA>SGA
Extrapyramidal S/E – Acute
• Loxapine : 12.5-50 mg/day PO dystonia, Parkinsonism,
Second generation (atypical) antipsychotics (SGA) akathasia, Tardive dyskinesia.
Neuroleptic Malignant
• Olanzapine : 5-10 mg/day PO, 2.5-10 mg IM (repeat 2h and 6h prn Syndrome (NMS)
to max of 30 mg/24 h) Sedation
• Risperidone : 0.5-2 mg/day PO Weight gain, diabetes and
dyslipidemia
• Quetiapine : 25-100 mg/day PO Hyperprolactinemia and
Benzodiazepines sexual dysfunction.
• Lorazepam : 0.5-1 mg PO/IM q6-8h
• Oxazepam : 10-15 mg PO tid
Therapeutic Guidelines | Psychiatry 53

Mania

• Mania is a state of abnormally elevated or irritable mood, arousal, and/ or energy levels. Treatment of
mania involves both acute control of severe agitation by a mood stabilizer and long term mood
stabilizers. Initially atypical antipsychotics such as Risperidone, Olanzapine or Quetiapine are effective.
First-generation Antipsychotics
• Haloperidol : 5-10 mg/day PO/IM
Atypical antipsychotics
• Risperidone : 2-3 mg/day PO
• Olanzapine : 5-20 mg/day PO, 2.5-10 mg IM (repeat 2h and 6h prn to max of 30 mg/24 h)
• Quetiapine : start with 100 mg/day PO; increase by 100 mg/day as needed to 300-600 mg/day
divided BID

2. Anxiety Disorders

• Anxiety disorders are a group of conditions with exaggerated anxiousness and worry about a number of
concerns persists for an extended period of time.

Goals of treatment (Nonpharmacologic treatment)

• Stress reduction and relaxation techniques such as meditation and low impact yoga is often helpful.
• Cognitive behavioral therapy (CBT)
• Reduction of consumption of caffeine and other stimulants.
• Minimize use of alcohol

Panic disorder

• Panic attack or panic disorder involves sudden anxiety that occurs without warning. Symptoms can
include chest pain, heart palpitations, sweating, shortness of breath, feeling of unreality, trembling,
dizziness, nausea, hot flashes or chills, a feeling of losing control, or a fear of dying. Panic attacks are
extremely common - 10% to 20% of the population experience a panic attack at some point in their
life. Some people start to avoid situations that might trigger a panic attack; this is called panic attack
with agoraphobia. Panic disorder refers to recurring feelings of terror and fear, which come on
unpredictably without any clear trigger.
• SSRIs and SNRIs are the first choice in the treatment of panic disorders. Selective serotonin reuptake
inhibitors (SSRIs) like Citalopram, Escitalopram, Fluoxetine, Paroxetine and Sertraline are all
effective in reducing panic attacks. Serotonin norepinephrine reuptake inhibitor (SNRIs) eg.
Venlafaxine is also used in panic disorder.
• There is a delay in the onset of response to these drugs which may be accompanied by initial agitation.
Combining SSRI or SNRI with a brief course of low dose benzodiazepine can increase adherence to
medication and produce rapid response.
• Other medication include Tricyclic antidepressants (TCAs) eg. Imipramine, Desipramine and
Clomipramine and Monoamine oxidase inhibitors (MAOIs) eg. Phenelzine, Tranylcypromine.
54 NAC OSCE | A Comprehensive Review

Selective serotonin reuptake inhibitors (SSRIs)


• Paroxetine : 20-60 mg/day PO SEROTONIN SYNDROME
Confusion, disorientation,
• Fluoxetine : 20-80 mg/day PO agitation, fever, diaphoresis,
• Sertraline : 50-200 mg/day PO sinus tachycardia, HTN,
mydriasis, tachypnea,
• Citalopram : 20-60 mg/day PO myoclonic jerks, hyper-
• Escitalopram : 10-20 mg/day PO reflexia, muscle rigidity,
tremor, ataxia, incoordination.
Serotonin norepinephrine reuptake inhibitor (SNRIs) TREATMENT: Benzodiazepines
st
• Venlafaxine : 37.5-22.5 mg/day (1 line), Serotonin
Antagonists : Cyproheptadine,
Tricyclic antidepressants (TCAs) Methysergide, Propranolol.
• Clomipramine : 75-225 mg/day
• Desipramine : 75-300 mg/day
• Imipramine : 75-300 mg/day
Benzodiazepines
• Alprazolam : 0.25 mg tid-qid, up to 1 mg PO qid
• Clonazepam : 0.25-0.5 mg PO bid
Monoamine oxidase inhibitors (MAOIs)
• Phenelzine : 45-90 mg/day
• Tranylcypromine : 20-60 mg/day

Social Phobia

• Social anxiety, also known as social phobia, involves excessive anxiety in social situations where people
fear being embarrassed or made fun of. Situations that can trigger social anxiety include small group
discussions, dating, going to a party, and playing sports. Common symptoms of social anxiety include
blushing, sweating, and dry mouth. People with social phobia often avoid social situations that cause
anxiety.
• SSRI and SNRI are mainstay drugs for the treatment for social phobia. Escitalopram, Fluvoxamine,
Paroxetine, Sertraline and Venlafaxine may be used for milder cases.
• Simple stage fright or fear of public speaking may respond to low dose Propranolol 10mg taken 30
minutes before the event.

General Anxiety Disorder

• Generalized anxiety disorder (GAD) is associated with continual excessive anxiety and worry about a
number of things (e.g., work, money, children, and health). There is no specific source of fear.
Symptoms can include muscle tension, trembling, shortness of breath, fast heartbeat, dizziness, dry
mouth, nausea, sleeping problems, and poor concentration. CBT is the most effective psychosocial
treatment but often takes 20 or more sessions to be effective.
• SSRIs and SNRIs have become established as first line treatments for GAD. Bupropion and
Pregabalin are further choices. Low dose benzodiazepines can be used but dependence is a problem.
Buspirone has a low abuse potential and is less sedating than benzodiazepines.
• Buspirone : 5mg bid-tid, up to 60 mg/day
Therapeutic Guidelines | Psychiatry 55

• Pregabalin : Initial 150 mg/day in 2-3 divided doses, may be increased to 150 mg bid after 1 week if
necessary
• Bupropion (Wellbutrin, Zyban): Use : Smoking cessation, second line Antidepressant.
Antidepressant: Start 100 mg bid x 4 days  100 m g tid.

Obsessive-compulsive disorder (OCD)

• OCD involves recurring unpleasant thoughts (obsessions) and/or repetitive behaviours (compulsions).
The thoughts may be connected to the repetitive behaviours. For example, people who fear getting an
infection may constantly wash their hands. At times, however, there's no connection at all between the
thoughts and the behaviours.
• CBT is the psychotherapy of choice. SSRIs : Fluoxetine, Fluvoxamine, Paroxetine and Sertraline, in
the usual antidepressant dosing range are the drugs of choice in Canada. It may take 6-8 weeks for
symptoms to improve. Second line drugs include Clomipramine, Venlafaxine, Citalopram and
Mirtazapine.

Post-traumatic stress disorder (PTSD)

• PTSD is associated with extreme anxiety that appears after a traumatic experience. Symptoms usually
start within 3 months of the traumatic event but may take years to start. PTSD can be associated with
sleep problems, nightmares, irritability, and anger. Feelings of guilt and unworthiness are common with
PTSD. Traumatic experiences that can trigger PTSD include wars, plane crashes, natural disasters
(e.g., hurricane, earthquake), and violent crimes (e.g., rape, abuse).
• SSRI and SNRI antidepressants have been shown to be effective in reducing the symptoms of PTSD.
Fluoxetine, Paroxetine, Sertraline and Venlafaxine are first line options.

3. Dementia

• Dementia is a serious loss of cognitive ability in a previously unimpaired


ADL/IADL
person, beyond what might be expected from normal aging. Dementia is Dressing
not a single disease, but rather a non-specific illness syndrome in which Eating
affected areas of cognition may be memory, attention, language, and Ambulation
Toilet
problem solving. It is normally required to be present for at least 6 months
Hygiene
to be diagnosed.
• The most common causes of dementia are Alzheimer's disease and vascular Shopping
dementia. It affects about 1% of people aged 60-64 years and as many as Housekeeping
30-50% of people older than 85 years. Accounting
• Benzodiazepines must be used cautiously in the elderly patients due to Food Making
Transportation
increase risk of falls and impaired cognition.
56 NAC OSCE | A Comprehensive Review

4. Depression

Nonpharmacological treatment
• Cognitive behavioral and interpersonal psychotherapy are as effective as antidepressants in mild to
moderate depression.

Pharmacological treatment
• Take medication daily, antidepressant must be taken for 2 to 4 weeks for effect to be noticeable.
Medication must be taken even if patient is feeling better.
SSRI
• Paroxetine (Paxil): Start 20 mg qhs, increase 10mg every 2wks, max 60mg per day.
• Fluoxetine (Prozac): Start 20mg PO qd, avoid increasing more often than monthly, max 80mg PO per
day
• Sertraline (Zoloft): Start 50mg PO qAM, increase 50mg every 2 weeks, max 200mg per day
• Fluvoxamine (Luvox): Start 25mg PO qhs x 3 days -> 50mg PO qhs x 7 days -> titrate 150-250 mg
daily divided doses bid.
• Citalopram (Celexa): Start 20mg PO qd, max 60mg.
• Escitalopram (Lexapro): Start 10mg PO qd
Monoamine Oxidase Inhibitor (MAO inhibitor): Use in Atypical depression, Refractory depression.
• Isocarboxazid (Marplan) – 10 mg PO bid, max 60 mg per day.
• Phenelzine (Nardil) – 15 mg PO tid, max 90 mg per day.
• Tranylcypromine (Parnate) – 10-40 mg per day in divided doses, max 60 mg per day.
Complication: Hypertensive crisis, Serotonin syndrome. Interaction with tyramine containing foods to
be avoided strictly.
Serotonin Norepinephrine Reuptake Inhibitors(SNRI) not used these days
• Tricyclic Antidepressants: Amitriptyline 25 mg qhs, Nortriptyline (Pamelor)
S/E: Anti-cholinergic – dry mouth, constipation, blurred vision, Anti-histaminergic – sedation, weight
gain; Serotonergic – sexual dysfunction; Orthostatic hypotension; Sinus tachycardia, SVT, Ventricular
tachycardia, Prolonged QT interval, heart block; Withdrawal symptoms.
Other : Venlafaxine (Effexor) 37.5 mg PO od.

6. Psychosis
In acutely psychotic individuals, short-acting parenteral antipsychotics either alone or in combination
with a parenteral benzodiazepine may be recommended. Liquid formulations of atypical antipsychotics may be
used as an alternative to intramuscular injections, Risperidone and Olanzapine are examples.

Atypical antipsychotics :
• Clozapine – 12.5 mg PO qd or bid, titrate slowly upwards in increments of 25-50 mg/day
Target dose : 300 – 450 mg/day, max 900 mg/day.
S/E: Agranulocytosis, Diabetes mellitus, hypertriglyceridemia.
NOT 1st LINE Anti-psychotic. Order weekly blood counts for 1 month and then q2 weeks.
Therapeutic Guidelines | Psychiatry 57

• Olanzapine (Zyprexa) – Start 5-10 mg PO qd, increase in 5 mg


Neuroleptic Malignant
increments weekly as tolerated, max 20 mg/day. Syndrome
S/E: Mild sedation, insomnia, dizziness, early AST & ALT Life- threatening neurological
elevation, restlessness, weight gain, increased risk of diabetes disorder presents with muscle
rigidity, fever, autonomic
mellitus and hyperlipidemia. instability, delirium.
• Quetiapine (Seroquel) – Start 25 mg PO bid – tid, increase in 25- Elevated creatine
phosphokinase
50 mg/day increments, target 300-400 mg/day divided doses bid- Rx : dantrolene sodium
tid, max 750 mg/day.
S/E: Headache, sedation, dizziness, constipation.
• Risperidone (Risperdal) – Start 1 mg PO bid, slow titration 2-4 mg
PO daily or divided doses bid, max 16 mg/day.
S/E: Insomnia, agitation, EPS, headache, anxiety, hyperprolactenemia, postural hypotension,
constipation, dizziness, weight gain.

Typical antipsychotics:
• Haloperidol (Haldol)– 5-10 mg PO, IM, IV. May repeat q30-60mins, max 300 mg per day.
• Fluphenazine (prolixin) – 2.5 mg PO bid, max 40 mg per day.

S/E – EXTRA PYRAMIDAL SIDE EFFECTS:


Akinesia – treat with Benztropine 2mg PO /IM/IV OD
Dystonic reaction – treat with Lorazepam or benztropine.
Dyskinesia
Akathesia – treat with Lorazepam, Propranolol or Diphenhydramine.
Perioral tremor
Neuroleptic malignant Syndrome – Muscle rigidity, tremor, delirium, high fever, diaphoresis,
hypertension. Discontinue drug. Give symptomatic treatment and supportive care. Treatment
with Dantrolene or bromocriptine.
Tardive dyskinesia – Blinking, lip smacking, sucking, chewing, grimaces, choreoathetoid movements,
tonic contractions of neck / back.
Treatment – Clozapine.

7. Mood stabilizers : Used in Bipolar disorder.


• Lithium bicarbonate – Start 300 mg PO bid, then increase to 900 – 1800 mg per day divided doses.
Serum levels – 0.6 – 1.2 mEq/l, monitor RFTs, TFTs.
S/E: tremor, polydipsia, hypothyroidism, weight gain, nausea/vomiting, diarrhea, ataxia, slurred speech,
lack of coordination.
Treatment of Lithium toxicity:
Discontinue lithium.
Check serum lithium levels, BUN, electrolytes.
IV fluids – Normal saline.
Hemodialysis in case Li > 2 mmol/L, coma, shock, severe dehydration, deterioration,
unresponsive to treatment.
58 NAC OSCE | A Comprehensive Review

• Divalproex (Epival) 300-1600 mg/day PO bid.


Monitor LFTs weekly x 1 month, then monthly.
S/E: Liver dysfunction, nausea/vomiting, diarrhea, ataxia, drowsiness, tremor, sedation, hair loss, weight
gain thrombocytopenia, neural tube defects in early pregnancy.
• Carbamazepine (Tegretol) 750- 3000 mg/day PO tid.
Monitor weekly CBC due to risk of agranulocytosis.
S/E: Nausea/vomiting, hepatic toxicity, ataxia, drowsiness, confusion, nystagmus, diplopia, transient
leukopenia, agranulocytosis, drug reaction (SJS), neural tube defect in early pregnancy.

8. Medications causing sexual dysfunction


1. Antiandrogen Medications
• Spironolactone
• Ketoconazole
• Finasteride

2. Antihypertensives
• Centrally acting sympatholytics (e.g. Clonidine)
• Peripherally acting sympatholytics (e.g. Guanadrel)
• Beta Blockers
• Thiazide Diuretics

3. Antidepressant Medications
• Selective Serotonin Reuptake Inhibitors (SSRI)
• Tricyclic Antidepressants
• MAO inhibitors

4. Sedative-Hypnotic Medications
• Barbiturates
• Benzodiazepines

5. Drug Abuse
• Alcohol Abuse
• Heroin abuse
• Marijuana abuse
• Methadone
• Tobacco abuse

6. Other Medications
• Anticholinergic Medications
• Antipsychotic Medications
• H2 Receptor Blockers
Therapeutic Guidelines | Psychiatry 59

9. Substance abuse

• Alcohol withdrawal:
• Tab Diazepam 20 mg PO q1-2h prn .
• Observe for 1-2 hours and re-assess.
• Inj Thiamine 100 mg IM then 100 mg PO OD x 3 days.
• Maintain hydration.
• If oral Diazepam not well tolerated then switch to Inj Diazepam 2-5 mg IV/min – maximum
10-20 mg q1h, or S/L Lorazepam.
• If severe liver dysfunction ,severe asthma, respiratory failure or age> 65 years present –
Lorazepam PO/SL/IM 1-4 mg q 1-2h.
• Hallucination present – Haloperidol 2-5 mg IM/PO q1-4h – max 5 doses/day along with
Diazepam 20 mg x 3 doses as seizure prophylaxis.
• Wernicke’s syndrome: Thiamine 100 mg PO OD x 1-2 weeks.
• Korsakoff ’s syndrome: Thiamine 100 mg PO bid/tid x 3-12 months.

• Opioid Intoxication:
• ABCs
• IV Glucose
• Inj Naloxone (Narcan) 0.4 mg – 2mg IV.
• Intubation and mechanical ventilation may be required for decreased level consciousness.
• Cocaine Overdose:
• ABCs
• Inj Diazepam 2-5 mg IV/min – maximum 10-20 mg q1h ( to control seizures).
• Propranolol or labetalol to treat hypertension and arrhythmia.
• Hallucinogens: LSD, mescaline, psilocybin, MDMA.
• Symptomatic treatment and supportive care.
• Decreased stimulation.
• Benzodiazepines or antipsychotics might be required.
• Phencyclidine:
• Room with minimal stimulation.
• Inj Diazepam IV for muscle spasms or seizures.
• Haloperidol to suppress psychotic behavior.
NOTES
Clinical Examination
This is a blank page
Clinical Examination 63

Abdominal Examination

1. Steps before beginning examination

• Introduce yourself : “I am Dr. ________, your attending physician and I'll be examining you today. At
any point of the examination you feel uncomfortable, please let me know and I'll stop the examination
right there.”
• Wash/Sanitize hands
• Explain to the patient what you are about to do and gain informed consent.
• Look for medical equipment/therapies (e.g. drains, colostomy/ileostomy bags).
• Verbalize the steps of the examination and your findings.
• Use proper draping techniques.

2. Inspection

• General inspection of the patient : Is patient comfortable at rest? Do they appear to be tachypnoeic?
• Examine the patient's hands for presence of koilonhychia (iron deficiency), leukonychia
(hypoalbuminemia), clubbing (IBD, coeliac disease, cirrhosis), palmar erythema, tar staining or
Dupuytren's contracture.
• Ask the patient to hold their hands out in front of them looking for a any tremor and then get them
to extend their wrists up towards the ceiling keeping the fingers extended and look for flapping
(asterixis in hepatic encephalopathy).
• Examine the face, check the conjunctiva for pallor. Also check the sclera for jaundice. Look at the
buccal mucosa for any obvious ulcers which could be a sign of Crohn's disease, B12 or iron deficiency.
Also look at the tongue. If it is red and fat it could be another sign of anaemia, as could angular
stomatitis. Check state of dentition – pigmentation of oral mucosa (Peutz-Jegher's syndrome),
telangectasia, candidiasis.
• Examine the neck for an enlarged left supraclavicular lymph node. A palpable enlarged supraclavicular
(Virchow's) node is known as Troisier's Sign, may be a sign of malignancy. Virchow's node drains the
thoracic duct and receives lymph drainage from the entire abdomen as well as the left thorax.
Therefore, enlargement of this node may suggest metastatic deposits from a malignancy in any of
these areas.
• Examine the chest, in particular look for gynaecomastia in men and the presence of 5 or more spider
naevi. These are both stigma of liver pathology.
• Inspect the abdomen and comment on any obvious abnormalities such as scars, masses and pulsations.
Also note if there is any abdominal distension/ascites. Look for distended veins, striae, Cullen's/Grey-
Turner's signs (pancreatitis), Sister Mary Joseph's nodule (widespread abdominal cancer)
64 NAC OSCE | A Comprehensive Review

3. Palpation

• Palpation of the abdomen should be performed in a


systematic way using the 9 named segments of the
abdomen: right and left hypochondrium, right and left
flank, right and left iliac fossa, the umbilical area, the
hypochondrium and the suprapubic region.
• If a patient has pain in one particular area you should
start as far from that area as possible. The tender area
should be examined last as they may start guarding
making the examination very difficult.
• Initial examination should be superficial using one
hand.Once you have examined all 9 areas superficially,
you should examine deeper. This is performed with two
hands, one on top of the other.
• Feel for organomegaly, particularly of the liver, spleen and kidneys. Palpation for the liver and spleen is
similar, both starting in the right iliac fossa. For the liver, press upwards towards the right
hypochondrium. You should try to time the palpation with the patient breathing in as this presses
down on the liver.If the liver is distended, its distance from the costal margin should be noted.
• Palpating for the spleen is as for the liver but in the direction of the left hypochondrium. The edge of
the spleen which may be felt if distended is more nodular than the liver.
• To feel for the kidneys you should place one hand under the patient in the flank region and the other
hand on top. You should then try to ballot the kidney between the two hands.

4. Percussion

Percussion over the abdomen is usually resonant, over a distended liver it will be dull. Percussion can also
be used to check for 'shifting dullness' - a sign of ascites. With the patient lying flat, start percussing from
the midline away from you. If the percussion note changes, hold you finger in that position and ask the
patient to roll towards you. Again percuss over this area and if the note has changed then it suggests
presence of fluid such as in ascites.

5. Auscultation

Listen with the diaphragm next to the umbilicus for up to 30 seconds.


Listen for bowel sounds – absent (e.g. Ileus, peritonitis), tinkling (bowel obstruction)

6. You should mention to the examiner at this point that you would like to finish the examination with an
examination of the hernial orifices, the external genitalia and also a rectal examination. It is also
appropriate to perform a urinalysis at this point including a pregnancy test in females.
Clinical Examination 65

Cardiovascular Examination

1. Steps before beginning examination

• Introduce yourself : “I am Dr. ________, your attending physician and I'll be examining you today. At
any point of the examination you feel uncomfortable, please let me know and I'll stop the examination
right there.”
• Wash/Sanitize hands
• Explain to the patient what you are about to do and gain informed consent.
• Look for medical equipment/therapies (e.g. GTN spray, ECG pads, oxygen)
• Verbalize the steps of the examination and your findings.

2. Inspection

• Start by observing the patient from the end of the bed. You should note whether the patient looks
comfortable. Are they cyanosed or flushed?
• Respiratory rate, rhythm and effort of breathing.
• Chest shape, chest movements with respration (symmetrical/assymetrical), skin (scars/nevi)
• Inspect the nails for clubbing, splinter hemorrhages (infective endocarditis), koilonychia (iron
deficiency anemia).
• Inspect fingers for capillary refill time, peripheral cyanosis, osler's nodes (infective endocarditis) and
nicotine staining.
• Inspect palms for palmar erythema, Janeway lesions and xanthomas.
• Take the radial pulse, assess the rate and rhythm.At this point you should also check for a collapsing
pulse – a sign of aortic incompetence. Locate the radial pulse and place your palm over it, then raise
the arm above the patient’s head. A collapsing pulse will present as a knocking on your palm.

At this point you should say to the examiner that you would like to take the blood pressure. They will
usually tell you not to and give you the value.

• Look in the eyes for any signs of jaundice (particularly beneath the upper eyelid), anaemia (beneath
the lower eyelid) and corneal arcus. You should also look around the eye for any xanthelasma.
• Whilst looking at the face, check for any malar facies, look in the mouth for any signs of anaemia such
as glossitis, check the colour of the tongue for any cyanosis, and around the mouth for any angular
stomatitis – another sign of anaemia.
• Assess jugular venous pressure ( JVP), ask patient to turn their head to look away from you. Look
across the neck between the two heads of sternocleidomastoid for a pulsation.
• Examine the chest, or praecordium for any obvious pulsations, abnormalities or scars, remembering to
check the axillae as well.
66 NAC OSCE | A Comprehensive Review

3. Palpation

• Palpation praecordium trying to locate the apex beat and describe its location anatomically. The
normal location is in the 5th intercostals space in the mid-clavicular line.
• Palpate for any heaves or thrills. A thrill is a palpable murmur whereas a heave is a sign of left
ventricular hypertrophy. Feel for these all over the praecordium.

4. Auscultation

• Mitral valve – where the apex beat was felt.


• Tricuspid valve – on the left edge of the sternum in the 4th
intercostal space.
• Pulmonary valve – on the left edge of the sternum in the
2nd intercostal space.
• Aortic valve – on the right edge of the sternum in the 2nd
intercostal space.

How many heart sounds are heard? Are the heart sounds
normal in character? Any abnormal heart sounds? If you hear any abnormal sounds you should describe
them by when they occur and the type of sound they are producing. Are there any murmurs? Can you hear
any rub? Feeling the radial pulse at the same time can give good indication as to when the sound occurs –
the pulse occurs at systole. Furthermore, if you suspect a murmur, check if it radiates. Mitral murmurs
typically radiate to the left axilla whereas aortic murmurs are heard over the left carotid artery.

• To further check for mitral stenosis you can lay the patient on their left side, ask them to breathe in,
then out and hold it out and listen over the apex and axilla with the bell of the stethoscope.
• Aortic incompetence can be assessed in a similar way but ask the patient to sit forward, repeat the
breathe in, out and hold exercise and listen over the aortic area with the diaphragm.

5. With patient sitting up percuss back for pleural effusion (cardiac failure)

6. Finally assess for any pedal oedema.

7. Finish by thanking the patient and ensuring they are comfortable and well covered.
Clinical Examination 67

Peripheral Vascular Examination

1. Steps before beginning examination

• Introduce yourself : “I am Dr. ________, your attending physician and I'll be examining you today. At
any point of the examination you feel uncomfortable, please let me know and I'll stop the examination
right there.”
• Wash/Sanitize hands
• Explain to the patient what you are about to do and gain informed consent.
• Look for medical equipment/therapies (e.g. GTN spray, ECG pads, oxygen)
• Verbalize the steps of the examination and your findings.

2. Inspection

• General observation of the patient, arms from the finger tips to the shoulder and legs from the groin
and buttocks to the toes. Comment on the general appearance of the arms and legs, size, swelling,
symmetry, skin color, hair, scars, pigmentation including any obvious muscle wasting. Note colour and
texture of nails.
• Any signs of gangrene or pre-gangrene such as missing toes or blackening of the extremities.
• The presence of any ulcers – ensure you check all around the feet including behind the ankle. These
may be venous or arterial – one defining factor is that venous ulcers tend to be painless whereas
arterial are painful.
• Any skin changes such as pallor, change in colour (eg purple/black from haemostasis or brown from
haemosiderin deposition), varicose eczema or sites of previous ulcers, atrophic changes and hair loss.
• Presence of any varicose veins – often seen best with the patient standing.

3. Palpation

• Assess the skin temperature. Starting distally, feel with the back of your hand and compare each limb
to the other noting any difference.
• Check capillary return by compressing the nail bed and then releasing it. Normal colour should return
within 2 seconds. If this is abnormal, perform Buerger’s Test. This involves raising the patient’s feet
to 45º. In the presence of poor arterial supply, pallor rapidly develops. Following this, place the feet
over the side of the bed, cyanosis may then develop.
• Any varicosities which you noted in the observation should now be palpated. If these are hard to the
touch, or painful when touched, it may suggest thrombophlebitis.
• Palpate peripheral pulses. These are:
Carotid – only palpate one carotid at a time
Radial – use the pad of three fingers
Brachial – may use thumb to palpate
Femoral – feel over the medial aspect of the inguinal ligament.
68 NAC OSCE | A Comprehensive Review

Popliteal – ask the patient to flex their knee to roughly 60º keeping their foot on the bed,
place both hands on the front of the knee and place your fingers in the popliteal space.
Posterior tibial – felt posterior to the medial malleolus of the tibia.
Dorsalis pedis –feel on the dorsum of the foot, lateral to the extensor tendon of the great toe.
You should compare these on both sides and comment on their strength.

• Check for radio-femoral delay. Palpate both the radial and femoral pulses on one side of the body. The
pulsation should occur at the same time. Any delay may suggest coarctation of the aorta.

4. Auscultation : listen for femoral and abdominal aortic bruits

5. Special Tests

• Allen Test : Ask the patient to make a tight fist and elevate the hand. Occlude the radial and ulnar
arteries with firm pressure. The hand is then opened. It should appear blanched (pallor can be
observed at the finger nails). Release either the Ulnar or radial artery pressure and the color should
return in 7 seconds. If the palm does not redden immediately, this suggests arterial insufficiency.

• Straight Leg Raise and Refill Test (Buerger's Test) : Raise the leg 45o to 60o for 30 seconds until
pallor of the feet develops and observe empty veins. Sit the patient upright and observe the feet. In
normal patients, the feet quickly turn pink (within 10-15 seconds). If, pallor persists for more than 10-
15s or there is development of a dusky cyanosis (rubor), this suggests of arterial insufficiency.

• Test for incompetent Saphenous Vein : Ask the patient to stand and note the dilated varicose veins.
Compress the vein proximally with one hand and place the other hand 10-15 cm distally. Briskly
compress and decompress the distal site. Normally, the hand at the proximal site should feel no
impulse, however with varicose veins a transmitted pulse may be felt.

• Trendelenburg Maneuver (Retrograde filling) : Ask the patient to lie down. Elevate the leg, and
empty the veins by massaging distal to proximal. Using a tourniquet, occlude the superficial veins in
the upper thigh. Ask the patient to stand. If the tourniquet prevents the veins from re-filling rapidly,
the site of the incompetent valve must be above this level i.e. at the sapheno-femoral junction. If the
veins re-fill, the communication must be lower down.
Observing the same protocol, proceed down the leg until the tourniquet controls re-filling. As
necessary, test:
• above the knee - to assess the mid-thigh perforator
• below the knee - to assess competence between the short saphenous vein and popliteal vein
If re-filling cannot be controlled, the communication is probably by one or more distal perforating
veins.
Clinical Examination 69

Respiratory Examination

1. Steps before beginning examination

• Introduce yourself : “I am Dr. ________, your attending physician and I'll be examining you today. At
any point of the examination you feel uncomfortable, please let me know and I'll stop the examination
right there.”
• Wash/Sanitize hands
• Explain to the patient what you are about to do and gain informed consent.
• Look for medical equipment/therapies (e.g. inhalers, oxygen).
• Verbalize the steps of the examination and your findings.

2. Inspection

• General look of the patient. Check whether they are comfortable at rest, is patient tachypnoeic? Are
they using accessory muscles? Are there any obvious abnormalities of the chest? Check for any clues
around the bed such as inhalers, oxygen masks or cigarettes.
• Inspect the hands, hot, pink peripheries may be a sign of carbon dioxide retention. Look for any signs
of clubbing, cyanosis, hypertrophic pulmonary osteoarthropathy, dupytren's contacture and nicotine
staining. Assess for carbon dioxide retention flap/salbutamol tremor.
• Take the patient’s pulse. After you have taken the pulse it is advisable to keep your hands in the same
position and subtly count the patient’s respiration rate.
• Inspect the face, ask the patient to stick out their tongue and note its colour – checking for cyanosis.
- Horner's sydrome (Pancoast tumour) , plethora (polycythemia).
• Look for any use of accessory muscles such as the sternocleidomastoid muscle. Also palpate for the
left supraclavicular node (Virchow's Node) as an enlarged node (Troisier's Sign) may suggest
metastatic lung cancer.
• Examine the chest and back. Observe the chest for any deformities (barrel chest, kyphoscoliosis,
pectus excavatum, pectus carinatum), symmetry of expansion, dilated veins, intercostal recession.

3. Palpation

• Palpate the chest. Feel between the heads of the two clavicles for the trachea, see if it is deviated.
• Feel for chest expansion. Place your hands firmly on the chest wall with your thumbs meeting in the
midline. Ask the patient to take a deep breath in and note the distance your thumbs move apart.
Normally this should be at least 5 centimetres. Measure this at the top and bottom of the lungs as well
as on the back.
70 NAC OSCE | A Comprehensive Review

4. Percussion

• Percussion should be performed on both sides, comparing similar areas on both sides. Start by tapping
on the clavicle which gives an indication of the resonance in the apex. Then percuss normally for the
entire lung fields. Hyper-resonance may suggest a collapsed lung where as hypo-resonance or dullness
suggests consolidation such as in infection or a tumour. Be sure to perform this on the back as well.

5. Vocal Fremitus

Check for tactile vocal fremitus. Place the medial edge of your hand on the chest and ask the patient to say
‘99’. Do this with your hand in the upper, middle and lower areas of both lungs.

6. Auscultation

• Do this in all areas of both lungs and on front and back comparing the sides to each other. Listen for
any reduced breath sounds, or added sounds such as crackles, wheezes or rhonchi.

7. Finish by examining the lymph nodes in the head and neck. Start under the chin with the submental
nodes, move along to the submandibular then to the back of the head at the occipital nodes. Next
palpate the pre and post auricular nodes. Move down the cervical chain and onto the supraclavicular
nodes.
Clinical Examination 71

Central Nervous System Examination

1. Steps before beginning examination

• Introduce yourself : “I am Dr. ________, your attending physician and I'll be examining you today. At
any point of the examination you feel uncomfortable, please let me know and I'll stop the examination
right there.”
• Wash/Sanitize hands
• Explain to the patient what you are about to do and gain informed consent.
• Look for medical equipment/therapies (e.g. walking aids).
• Verbalize the steps of the examination and your findings.

2. Cranial Nerve Examination

1) The Olfactory nerve (CN I) is simply tested by offering something familiar for the patient to smell
and identify – for example coffee or vinegar.
2) The Optic nerve (CN II) is tested in five ways:
• The acuity is easily tested with Snellen charts. This should be assessed both with the patient
wearing any glasses or contact lenses they usually wear and without them.
• Colour vision is tested using Ishara plates, these identify patients who are colour blind.
• Visual fields are tested by asking the patient to look directly at you and wiggling one of your
fingers in each of the four quadrants. Ask the patient to identify which finger is moving.
Visual inattention can be tested by moving both fingers at the same time and checking the
patient identifies this.
• Visual reflexes comprise direct and concentric reflexes. Place one hand vertically along the
nose to block any light from entering the eye not being tested. Shine a pen torch into one eye
and check that the pupils on both sides constrict. This should be tested on both sides.
• Finally fundoscopy should be performed on both eyes.
3) Eye movements: Oculomotor nerve (III), Trochlear nerve (IV) and Abducent nerve (VI) are
involved in movements of the eye. Asking the patient to keep their head perfectly still directly in front
of you, you should draw two large joining H’s in front of them using your finger and ask them to
follow your finger with their eyes. It is important the patient does not move their head. Always ask if
the patient experiences any double vision and if so when is it worse. Also ;look for ptosis and assess
saccadic eye movements.
4) The Trigeminal nerve (CN V) is involved in sensory supply to the face and motor supply to the
muscles of mastication. Initially test the sensory branches by lightly touching the face with a piece of
cotton wool and then with a blunt pin in three places on each side – around the jawline, on the cheek
and on the forehead. The corneal reflex should also be examined as the sensory supply to the cornea is
from this nerve. This is done by lightly touching the cornea with the cotton wool. This should cause
the patient to shut their eyelids.
For the motor supply, ask the patient to clench their teeth together, observing and feeling the bulk of
the masseter and temporalis muscles. Then ask them to open their mouth against resistance. Finally
perform the jaw jerk on the patient by placing your left index finger on their chin and striking it with
a tendon hammer. This should cause slight protrusion of the jaw.
72 NAC OSCE | A Comprehensive Review

5) The Facial nerve (CN VII) supplies motor branches to the muscles of facial expression. Therefore,
this nerve is tested by asking the patient to crease up their forehead (raise their eyebrows), close their
eyes and keep them closed against resistance, puff out their cheeks and show you their teeth.
6) The Vestibulocochlear nerve (CN VIII) provides innervation to the hearing apparatus of the ear and
can be used to differentiate conductive and sensori-neural hearing loss using the Rinne and Weber
tests. For the Rinne test, place a sounding tuning fork on the patient’s mastoid process and then next
to their ear and ask which is louder, a normal patient will find the second position louder. For Weber’s
test, place the tuning fork base down in the centre of the patient’s forehead and ask if it is louder in
either ear. Normally it should be heard equally in both ears.
7) The Glossopharyngeal nerve (CN IX) provides sensory supply to the palate. It can be tested with the
gag reflex or by touching the arches of the pharynx.
8) The Vagus nerve (CN X) provides motor supply to the pharynx. Asking the patient to speak gives a
good indication to the efficacy of the muscles. You should also observe the uvula before and during
the patient saying ‘aah’. Check that it lies centrally and does not deviate on movement.
9) The Accessory nerve (CN XI) gives motor supply to the sternocleidomastoid and trapezius muscles.
To test it, ask the patient to shrug their shoulders and turn their head against resistance.
10) The Hypoglossal nerve (CN XII) provides motor supply to the muscles of the tongue. Observe the
tongue for any signs of wasting or fasciculations. Then ask the patient to stick their tongue out. If the
tongue deviates to either side, it suggests a weakening of the muscles on that side.

3. Cerebellar Examination

Gait:
• Ask the patient to stand up. Observe the patient's posture and whether they are steady on their feet.
• Ask the patient to walk, e.g. to the other side of the room, and back. If the patient normally uses a
walking aid, allow them to do so.
• Observe the different gait components (heel strike, toe lift off ). Is the gait
shuffling/waddling/scissoring/ swinging?
• Observe the patients arm swing and take note how the patient turns around as this involves good
balance and co-ordination.
• Ask the patient to walk heel-to-toe to assess balance.
• Perform Romberg’s test by asking the patient to stand unaided with his eyes closed. If the patient
sways or loses balance this test is positive. Stand near the patient in case he falls.

Co-ordination:
• Look for a resting tremor in the hands.
• Test tone in the arms (shoulder, elbow, wrist)
• Test for dysdiadochokinesis by showing the patient to clap by alternating the palmar and dorsal
surfaces of the on hand. Ask to do this as fast as possible and repeat the test with the other hand.
• Perform the finger-to-nose test by placing your index finger about two feet from the patients face. Ask
him to touch the tip of his nose with his index finger then the tip of your finger. Ask him to do this as
fast as possible while you slowly move your finger. Repeat the test with the other hand.
• Perform the heel-to-shin test. Have the patient lying down for this and get him to run the heel of one
foot down the shin of the other leg and then to bring the heel back up to the knee and start again.
Repeat the test with the other leg.
Clinical Examination 73

Upper Limb Neurological Examination

1. Steps before beginning examination

• Introduce yourself : “I am Dr. ________, your attending physician and I'll be examining you today. At
any point of the examination you feel uncomfortable, please let me know and I'll stop the examination
right there.”
• Wash/Sanitize hands
• Explain to the patient what you are about to do and gain informed consent.
• Use proper draping techniques, verbalize the steps of the examination and your findings.

2. Inspection
• General inspection of patient: general comfort, abnormal posture/movements, muscle wasting.
• The upper body should be exposed for this examination. Observe the patient's arms, look for any
muscle wasting, fasciculation’s or asymmetry.

3. Tone
• Examine the tone of the muscles. Start proximally at the shoulder, feeling how easy the joint is to
move passively. Then move down to the elbow, wrist and hand joints again assessing each one's tone in
turn.
• Assess for spastic catch, clasp-knife rigidity, led-pipe or cog-wheel rigidity.

4. Power
• Next assess the power of each of the muscle groups.
– Shoulder abduction (C5) & Shoulder adduction (C5/C6/C7)
– Elbow flexion (C5/C6) & Elbow extension (C7)
– Wrist flexion (C8) & Wrist extension (C8)
– Finger flexion (C8), Finger abduction (T1), Finger adduction (T1)
– Thumb abduction (C8)

5. Reflexes
• There are three reflexes in the upper limb - the biceps, triceps and supinator reflexes.
• The biceps reflex (C5/C6) is tested by supporting the patient's arm, with it flexed at roughly 60º,
placing your thumb over the biceps tendon and hitting your thumb with the tendon hammer. It is
vital to get your patient to relax as much as possible and for you to take the entire weight of their arm.
• The triceps reflex (C6/C7) is elicited by resting the patient's arm across their chest and hitting the
triceps tendon just proximal to the elbow.
• Finally, with their arm rested on their abdomen, locate the supinator tendon (C5/C6) as it crosses the
radius, place three fingers on it and hit the fingers. This should give the supinator reflex. If you
struggle with any of these reflexes, asking the patient to clench their teeth should exaggerate the
reflex.
74 NAC OSCE | A Comprehensive Review

6. Sensation
• This is tested in a number of ways. You should test
light touch, pin prick, vibration and joint position
sense and proprioception.

• Ask the patient to place their arms by their sides with


their palms facing forwards. Lightly touch the patient's
sternum with a piece of cotton wool so that they know
how it feels. Then, with the patient's eyes shut, lightly
touch their arm with the cotton wool. The places to
touch them should test each of the dermatomes. Tell
the patient to say yes every time they feel the cotton
wool as it felt before. Then repeat this using a light pin
prick.
• To assess vibration you should use a sounding tuning
fork. Place the fork on the patient's sternum to show
them how it should feel. Then place it on the bony
prominence at the base of their thumb and ask them if
it feels the same. If it does, there is no need to check
any higher. If it feels different you should move to the
radial stylus and then to the olecranon until it feels
normal.
• Finally, proprioception. Hold the distal phalanx of the thumb on either side so that you can flex the
interphalangeal joint. Show the patient that when you hold the joint extended, that represents 'Up'
whereas when you hold it flexed that represents 'Down'. Ask the patient to close their eyes and,
having moved the joint a few times hold it in one position - up or down. Ask the patient which
position the joint is in.

7. Coordination
• Pronator drift – Ask patient to extend arms in front of them in supination and to close their eyes. A
positive result occurs when the arm falls downwards and pronates (cerebral damage), in cerebellar
lesions the arms may rise.
• Assess for dysdiadochokinesia
• Assess for finger to nose coordination and intentional tremor.

8. Function is a very important part of any neurological examination as this is the area which will affect
people's day to day lives the most. For upper limb you should ask people to touch their head with both
hands and then ask them to pick up a small object such as a coin which each hand.

9. Finish by thanking the patient and ensuring they are comfortable and well covered.
Clinical Examination 75

Lower Limb Neurological Examination

1. Steps before beginning examination


• Introduce yourself : “I am Dr. ________, your attending physician and I'll be examining you today. At
any point of the examination you feel uncomfortable, please let me know and I'll stop the examination
right there.”
• Wash/Sanitize hands
• Explain to the patient what you are about to do and gain informed consent.
• Verbalize the steps of the examination and your findings.
• Make sure patient is adequately exposed, use proper draping techniques

2. Inspection
• Observe the patient's legs, look for any muscle wasting, fasciculations or asymmetry.

3. Tone
• Start by examining the tone of the muscles. Roll the leg on the bed to see if it moves easily and pull
up on the knee to check its tone. Also check for ankle clonus by placing the patients leg turned
outwards on the bed, moving the ankle joint a few times to relax it and then sharply dorsiflexing it.
Any further movement of the joint may suggest clonus.

4. Power
• Next assess the power of each of the muscle groups.
– Hip flexion (L1/L2) & Hip extension (L5/S1)
– Hip abduction (L2/L3) & Hip adduction (L2/L3)
– Knee flexion (L5/S1) & Knee extension (L3/L4)
– Ankle dorsiflexion (L4/L5) & Ankle plantar flexion (S1/S2)
– Big toe flexion (S1/S2)

5. Reflexes
• Test the patient's reflexes. There are three reflexes in the lower limb - the knee reflex, the ankle jerk
and the plantar reflex - elicited by stroking up the lateral aspect of the plantar surface.
• The knee reflex (L3/L4) is tested by placing the patient's leg flexed at roughly 60º, taking the entire
weight of their leg with your arm and hitting the patellar tendon with the tendon hammer. It is vital
to get your patient to relax as much as possible and for you to take the entire weight of their leg.
• The ankle jerk (S1/S2) is elicited by resting the patient's leg on the bed with their hip laterally rotated.
Pull the foot into dorsiflexion and hit the calcaneal tendon.
• Finally, with their leg out straight and resting on the bed, run the end of the handle of the tendon
hammer along the outside of the foot. This gives the plantar reflex (S1). An abnormal reflex would see
the great toe extending. If you struggle with any of these reflexes, asking the patient to clench their
teeth should exaggerate the reflex.
76 NAC OSCE | A Comprehensive Review

6. Sensation
• The final test is sensation. However, this is tested in a number
of ways. You should test light touch, pin prick, vibration and
joint position sense and proprioception.
• Ask the patient to place their legs out straight on the bed.
Lightly touch the patient's sternum with a piece of cotton wool
so that they know how it feels. Then, with the patient's eyes
shut, lightly touch their leg with the cotton wool. The places to
touch them should test each of the dermatomes - make sure
you know these! Tell the patient to say yes every time they feel
the cotton wool as it felt before. Then repeat this using a light
pin prick.
• To assess vibration you should use a sounding tuning fork.
Place the fork on the patient's sternum to show them how it
should feel. Then place it on their medial malleolus and ask
them if it feels the same. If it does, there is no need to check
any higher. If it feels different you should move to the tibial
epicondyle and then to the greater trochanter until it feels
normal.
• Finally, proprioception. Hold the distal phalanx of the great toe
on either side so that you can flex the interphalangeal joint.
Show the patient that when you hold the joint extended, that represents 'Up' whereas when you hold
it flexed that represents 'Down'. Ask the patient to close their eyes and, having moved the joint a few
times hold it in one position - up or down. Ask the patient which position the joint is in.

7. Function is a very important part of any neurological examination as this is the area which will affect
people's day to day lives the most. For the lower limb you should assess the patient's walking. Observe
their gait and check for any abnormalities. Whilst they are standing you should perform Romberg's
test. Ask the patient to stand with their feet apart and then close their eyes. Any swaying may be
suggestive of a posterior column pathology.

8. Finish by thanking the patient and ensuring they are comfortable and well covered.
Clinical Examination 77

Musculo-skeletal system : Spine/Back


1. Steps before beginning examination
• Introduce yourself : “I am Dr. ________, your attending physician and I'll be examining you today. At
any point of the examination you feel uncomfortable, please let me know and I'll stop the examination
right there.”
• Wash/Sanitize hands
• Explain to the patient what you are about to do and gain informed consent.
• Ensure patient is adequately exposed.
• Look for medical equipment/therapies
• Show empathy.
• Verbalize the steps of the examination and your findings.

2. Inspection
• Ask for patient vitals
• Observe patient : Is patient sitting comfortably? Gait? Position of comfort.
• Observe the patient from behind :
– Pelvic and shoulder symmetry, palpate the pelvic brim to check for symmetry.
– Scoliosis
– Gibbus (dorsal spines abnormally prominent)
• Observe patient from side :
– Kyphosis
– Increased lumbar lordosis
• Check the spine for SEADS : S: Swelling, E: Erythema, ecchymosis, A: Atrophy/asymmetry (muscle
bulk), D: Deformity, S: Skin changes/scars/bruising

3. Range of Motion
• Flexion : In the standing position by asking the patient to touch the toes. Normal - 90 o .The normal
spine should lengthen more than 5 cm in the thoracic area and more than 7.5 cm in the lumbar area
on forward flexion.
• Extension : Stabilize the patient, ask patient to bend backwards. Normal – 30 o.
• Lateral flexion : ask the patient to slide their hand straight down the thigh, first on the right and then
on the left, keeping the hips straight.
• Observe for restricted movement and loss of symmetry.
• Test for facet joint disease : Ask patient to extend their back as far as possible and to rotate (pain
suggests facet joint pathology).

4. Palpation
• Examine the back and palpate for areas of muscle spasm and tenderness (paraspinal muscles).
• Palpate spinous processes with thumb for tenderness
• Sacroiliac joints, sacro iliac dimples, ask for tenderness.
78 NAC OSCE | A Comprehensive Review

5. Ankylosing spondylitis tests


• Chest expansion : Measure with a tape measure (should be >5cm)
• Schober's Test : Draw a horizontal line 10cm above and one 5cm below the dimples of Venus (the
distance between these lines should increase to >20cm during lumbar flexion – in ankylosing
spondylitis the distance will not increase to >20cm)
• Distance of tragus to wall when patient is standing with their back to the wall (useful for monitoring).

6. Cervical and thoracic movements (patient sitting on edge of bed)


• Cervical movements
– Flexion (ask patient to touch chin to chest)
– Extension (ask patient to look to the ceiling as far back as possible)
– Lateral flexion (ask patient to touch their ear to the shoulder keeping the shoulder still)
– Spurling Maneuver : Extend head back & bring ear towards shoulder. Give gentle axial pressure
on the head. If patient complains of pain radiating from head to ipsilateral arm – diagnosis of
Radiculopathy is made.
– Rotation (ask patient to look over the left and right shoulder)
– Perform these movements passively if active movements are restricted.
• Thoracic rotation : ask patient to fold their arms and twist around.

7. Tests with patient lying on their back


• Straight leg raising test : ask the patient to lie with the spine on the table and to relax completely.
With the knee fully extended, first one leg and then the other is slowly lifted and flexed at the hip.
This produces stretch on the sciatic nerve, at which point sciatic pain is produced. If this maneuver
produces pain in the hip or low back with radiation in the sciatic area, the test is considered positive
for nerve root irritation. The angle of elevation of the leg from the table at the point where pain is
produced should be recorded.
• FABER (Flexion Abduction External Rotation) : Ask the patient to lie supine on the exam table.
Place the foot of the affected side on the opposite knee. Pain in the groin area indicates a problem
with the hip and not the spine. Press down gently but firmly on the flexed knee and the opposite
anterior superior iliac crest. Pain in the sacroiliac area indicates a problem with the sacroiliac joints.
• Bowstring test: Once the level of pain has been reached, flex the knee slightly and apply firm pressure
with the thumb in the popliteal fossa over the stretched tibial nerve. Radiating pain and paraesthesiae
suggest nerve root irritation.

8. Tests with patient lying on their abdomen


• Lasegue's sign: With patient supine and hip flexed, dorsiflexion of the ankle causes pain or muscle
spasm in the posterior thigh if there is lumbar root or sciatic nerve irritation.
• Femoral stretch test:With the patient prone and the anterior thigh fixed to the couch, flex each knee
in turn. This causes pain in the appropriate distributions by stretching the femoral nerve roots in L2-
L4. The pain produced is normally aggravated by extension of the hip. The test is positive if pain is
felt in the anterior compartment of thigh.
Clinical Examination 79

Hip Examination

1. Steps before beginning examination


• Introduce yourself : “I am Dr. ________, your attending physician and I'll be examining you today. At
any point of the examination you feel uncomfortable, please let me know and I'll stop the examination
right there.”
• Wash/Sanitize hands
• Explain to the patient what you are about to do and gain informed consent.
• Ensure patient is adequately exposed.
• Look for medical equipment/therapies
• Ask which hip is painful, show empathy.
• Verbalize the steps of the examination and your findings.

2. Inspection of hip (with patient standing up)


• While the patient is standing, check the hip for SEADS : S: Swelling, E: Erythema, ecchymosis,
A: Atrophy/asymmetry (muscle bulk), D: Deformity, S: Skin changes (erythema/scars/abscess/sinuses)
• Leg length discrepancy
• Whilst the patient is still standing, perform the Trendelenberg test. This is done by asking the patient
to alternately stand on one leg. Stand behind the patient and feel the pelvis. It should remain level or
rise slightly. If the pelvis drops markedly on the side of the raised leg, then it suggests abductor muscle
weakness on the leg the patient is standing on.

TYPES OF GAIT
3. Gait – ask patient to walk across the floor. Look for any abnormalities,
Antalgic – Trauma, OA
hip, knee, foot movements, length of stride. Trendelenberg – weakness
of hip adductors
Festinating – Parkinson's ds.
High stepping – Polio, MS
3. Inspection & Palpation of hip (with patient lying down) Scissor – Spastic cerebral
palsy
• Inspection for hip and groin swellings (hernia, lymphadenopathy, Stomping – Friedreich's
saphenous varix, effusion) ataxia, tabes dorsalis
Spastc – Brain tumor, sturge
• Inspect for obvious fixed flexion weber's, cerebral palsy
• Palpate anterior hip for lumps and tenderness.
• Palpate the greater trochanter for any tenderness which might
suggest trochanteric bursitis.

4. Leg-length difference
• Make an approximate judgment by aligning the medial malleoli and looking for discrepancy.
• Measure true and apparent leg-length if appropriate. True leg length discrepancy is found by
measuring from the anterior superior iliac spine to the medial malleolus. Apparent leg length
discrepancy is measured from the umbilicus to the medial malleolus.
80 NAC OSCE | A Comprehensive Review

5. Active and passive movements


• Assess active flexion, extension, abduction and adduction.
– Flexion : Flex the knee to 90 degrees and passively flex the hip by pushing the knee towards the
chest.
– Extension : is performed by placing your hand under the patient's ankle and asking them to push
your hand into the bed.
• Passively assess internal and external rotation of the hip (with hips at 90 o flexion)
– Internal rotation : performed with the knee flexed and by everting the knee for internal rotation
– External rotation : performed with the knee flexed and inverting it for external rotation.

6. Special tests
• Thomas test : Place your hand under the patient's lumbar spine to stop any lumbar movements and
fully flex one of the hips. Observe the other hip, if it lifts off the couch then it suggests a fixed flexion
deformity of that hip.

• FABER (Flexion Abduction External Rotation) : Ask the patient to lie supine on the exam table.
Place the foot of the affected side on the opposite knee. Pain in the groin area indicates a problem
with the hip and not the spine. Press down gently but firmly on the flexed knee and the opposite
anterior superior iliac crest. Pain in the sacroiliac area indicates a problem with the sacroiliac joints.
Clinical Examination 81

Knee Examination

1. Steps before beginning examination


• Introduce yourself : “I am Dr. ________, your attending physician and I'll be examining you today. At
any point of the examination you feel uncomfortable, please let me know and I'll stop the examination
right there.”
• Wash/Sanitize hands
• Explain to the patient what you are about to do and gain informed consent.
• Ensure patient is adequately exposed (up to above knees).
• Look for medical equipment/therapies
• Ask about knee locking, giving way and pain, show empathy.
• Verbalize the steps of the examination and your findings.

2. Inspection
• Gait : Ask the patient to walk for you. Observe any limp or obvious deformities such as scars or
muscle wasting. Check if the patient has a varus (bow-legged) or valgus (knock-knees) deformity.
Also observe from behind to see if there are any obvious popliteal swellings such as a Baker's cyst.
• While the patient is lying on the bed, make a general observation. Look for symmetry, redness, muscle
wasting, scars, rashes or fixed flexion deformities.

3. Palpation
• Check the temperature using the backs of your hands, comparing it with other parts of the leg.
• Palpate the border of the patella for any tenderness, behind the knee for any swellings, along all of the
joint lines for tenderness and at the point of insertion of the patellar tendon. Finally, tap the patella to
see if there is any effusion deep to the patella.
• Landmarks of the knee : Tibial tuberosity, patellar tendon, quadriceps tendon, medial and lateral
femoral condyles.
Peripatellar area : push patella medially and rub right underneath the medial facet of patella and look
for tenderness ( Patellar – femoral stress S o).
Joint line tenderness : bend the knee 90o , palpate medial and lateral joint line.

4. Range of Motion
• Active flexion and extension of knee – Observe for restricted movement and for displacement of
patella.
• Passive flexion and extension of knee – feel for crepitus.
• Patella apprehension test – Move patella around and observe patient's face for pain.
• Straight leg raise – assessment of extensor apparatus.
82 NAC OSCE | A Comprehensive Review

5. Special tests

Tests to Detect Arthritis:


• Crepitus: Crepitus is the sensation that is felt when rough cartilage or exposed bone is rubbing as the
knee is bent. The examiner will feel, and may hear, this grinding as the knee is bent back and forth.
• Deformity: As the knee cartilage is worn away, the knees may become progressively knock-kneed or
bow-legged.
• Limited Motion: The range of motion of the knee typically becomes limited if arthritis, bone spurs,
and swelling prevents normal mobility.

Tests to Detect a Torn Meniscus:


• Joint Line Tenderness
Joint line tenderness is a very non-specific test for a meniscus tear. The area of the meniscus is felt,
and a positive test is considered when there is pain in this area.
• McMurray's Test
McMurray's test is performed with the patient lying flat on his back and the examiner bending the
knee. A click is felt over the meniscus tear as the knee is brought from full flexion to full extension.

Tests to Detect an ACL Tear:


• Lachman Test
The Lachman test is the best test to diagnose an ACL tear. With the knee slightly bent, the examiner
stabilizes the thigh while pulling the shin forward. A torn ACL allows the shin to shift too far
forward.
• Anterior Drawer Test
This test is also performed with the patient lying flat on his back. The knee is bent 90 degrees and the
shin is pulled forward to check the stability of the ACL.

Tests to Detect Other Ligament Injuries:


• Posterior Drawer Test
The posterior drawer is performed similarly to the anterior drawer test. This test detects injury to the
PCL. By pushing the shin backward, the function of the PCL is tested.
• Collateral Ligament Stability
Side-to-side stability of the knee detects problems of the collateral ligaments, the MCL and LCL.
With the patient lying flat, and the knee held slightly bent, the shin is shifted to each side. Damage to
the LCL or MCL will allow the knee to "open up" excessively, a problem called varus (LCL) or valgus
(MCL) instability.

Tests to Detect Kneecap Problems:


• Patellar Grind
The patient lies supine with the leg extended. The examiner reproduces the patient's knee pain by
pushing the kneecap down and asking the patient to flex his thigh muscles. Damaged cartilage can
cause a grinding sensation called crepitus.
• Patellar Tenderness
The examiner can slightly lift up the kneecap and place direct pressure on the under surface of the
kneecap. By doing so, the examiner is looking for sensitive regions of cartilage.
• Patellar Apprehension
This is a sign of an unstable kneecap. While the examiner places pressure on the kneecap, the patient
may complain of the sensation that the kneecap is going to 'pop out' of its groove.
Clinical Examination 83

Foot and Ankle Examination

1. Steps before beginning examination

• Introduce yourself : “I am Dr. ________, your attending physician and I'll be examining you today. At
any point of the examination you feel uncomfortable, please let me know and I'll stop the examination
right there.”
• Wash/Sanitize hands
• Explain to the patient what you are about to do and gain informed consent.
• Ensure patient is adequately exposed (up to above knees).
• Look for medical equipment/therapies
• Ask if patient is able to bear weight, show empathy.
• Verbalize the steps of the examination and your findings.

2. Inspection

• Gait : watch the patient walk, observing for a normal heel strike, toe-off gait. Also look at the
alignment of the toes for any valgus or varus deformities. Assess ability to weight-bear on affected
side.
• While patient is standing check the foot arches checking for pes cavus (high arches) or pes planus
(flat feet).
• Inspection of the foot with patient sitting and feet overhanging
– Check the foot and ankle for SEADS : S: Swelling, E: Erythema, ecchymosis,
A: Atrophy/asymmetry (muscle bulk), D: Deformity, S: Skin changes/scars/bruising.
– Check the symmetry, nails (psoriasis), skin, toe alignment, look for toe clawing, joint swelling and
plantar and dorsal calluses.
• Finally you should look at the patient's shoes, note any uneven wear on either sole and the presence of
any insoles.

3. Palpation of ankle/foot

• Feel each foot for temperature, comparing it to the temperature of the rest of the leg.
• Feel for distal pulses.
• Squeeze over the metatarsophalangeal joints observing the patient's face for any pain.
• Palpate over the midfoot, ankle and subtalar joint lines for any tenderness. Feel the Achilles tendon
for any thickening or swelling. Palpate medial and lateral malleoli for any tenderness.
84 NAC OSCE | A Comprehensive Review

4. Range of Motion

• Assess all active and movements of the foot. These movements are inversion, eversion, dorsiflexion
and plantarflexion.
– Subtalar joint – inversion and eversion
– Ankle joint – dorsiflexion and plantar flexion
– Big toe – dorsiflexion and plantar flexion
– Mid-tarsal joints - which are tested by fixing the ankle with one foot and inverting and everting
the forefoot with the other.

5. Special tests

• Ankle Anterior Drawer Test - assesses for lateral ankle sprain


Patient is seated, stabilize the tibia with one hand while grasping heel and pulling it anteriorly with
the other. Greater than 3 mm anterior movement may be significant. 1 cm is significant and indicates
anterior talofibular ligament rupture.
Positive Test - laxity in the ligament with exaggerated anterior translation
• Talar Tilt Test - assesses integrity of the deltoid ligament/lateral ankle sprain
Patient is seated, stabilize the leg and foot while adducting and inverting the calcaneus apply a varus
force. The calcaneus is then abducted and everted applying a valgus force.
Positive Test - pain or laxity in the ligament
• Thompson's Test - assesses for Achilles' tendon rupture
Patient is prone, squeeze the gastrocnemius and soleus muscles while noting any movement at the
ankle and foot
Positive Test - no movement or plantarflexion at all indicates a 3rd degree strain of the Achilles'
tendon
• Plantar Fasciitis Test - assesses for inflammation of the plantar fascia
Patient is supine, dorsiflex the ankle and extends all toes then press in the medial border of the plantar
fascia
Positive Test - pain is consistent with plantar fasciitis

Ottawa Ankle rules


For taking ankle series x-rays(AP and lateral ankle)
• X-ray if there is pain over the malleolar zone AND tenderness on palpation of the posterior medial or
lateral malleolus OR
• Patient unable to bear weight immediately and in ER.

For foot series (AP and Lateral foot)


• X-ray if there is pain in midfoot zone AND bony tenderness over the navicular or base of 5 th
metatarsal OR
• Unable to bear weight immediately and in ER
Clinical Examination 85

Shoulder Examination

1. Steps before beginning examination

• Introduce yourself : “I am Dr. ________, your attending physician and I'll be examining you today. At
any point of the examination you feel uncomfortable, please let me know and I'll stop the examination
right there.”
• Wash/Sanitize hands
• Explain to the patient what you are about to do and gain informed consent.
• Look for medical equipment/therapies, ensure patient is adequately exposed.
• Ask which shoulder is painful. Verbalize the steps of the examination and your findings.

2. Inspection

• Start by exposing the joint and observe the shoulder joint looking from the back, side and front for
any scars, deformities or muscle wasting (SEADS). Also compare both sides for symmetry.
• With the patient standing, ask the patient to place their hands behind their head and behind their
back and observe for and deformities.

3. Palpation

• Feel over the joint and its surrounding areas for the temperature of the joint as raised temperature may
suggest inflammation or infection in the joint.
• Systematically feel along both sides of the bony shoulder girdle. Start at the sternoclavicular joint,
work along the clavicle to the acromioclavicular joint
• Feel the acromion and then around the spine of the scapula.
• Feel the anterior and posterior joint lines of the glenohumeral joint and finally the muscles around the
joint for any tenderness.

4. Range of Motion

• The movements of the joint should start being performed actively.


• Ask the patient to bring their arm forward (flexion), bend their arm at the elbow and push backwards
(extension),
• Bring their arm out to the side and up above their head (abduction). When testing adduction perform
the scarf test (The scarf test is performed with the elbow flexed to 90 degrees, placing the patient's
hand on their opposite shoulder and pushing back, again look for any discomfort. )
• Rotation
– Internal rotation : Ask the patient to place hands in the small of their back, and slide them up the
back as far as possible.
– External rotation : Ask patient to rotate their arms outwards, keeping the elbows flexed and by
the side of the body.
• Once all of these movements have been performed actively, you should perform them passively and
feel for any crepitus whilst moving the joints.
86 NAC OSCE | A Comprehensive Review

5. Special Tests

Tests for Rotator Cuff


i. Supraspinatus
• Empty Can Test ( tests integrity of Supraspinatus) : The patient stands with arms extended at the
elbows and abducted in the scapular plane and with thumbs pointed to the floor. The examiner applies
downward pressure to the arms and the patient attempts to resist.
Positive test : Pain, muscle weakness or both.
• Apley's Scratch Test- Reach over shoulder to "scratch" between scapula. Measure to which vertebrae
thumb can reach.
ii. Infraspinatus
• External Rotation Lag Sign : The elbow is passively flexed to 90 degrees, and the shoulder is held at
20 degrees abduction (in the scapular plane) and near maximal external rotation by the examiner. The
patient is then asked to actively maintain the position of external rotation in abduction as the
examiner releases the wrist while maintaining support of the limb at the elbow. The sign is positive
when a lag, or angular drop occurs. The magnitude of the lag is recorded to the nearest 5 degrees.
iii. Subscapularis
• Gerber Lift-Off Test : With the patient's hand on the small of the back, the arm is extended and
internally rotated. The examiner then passively lifts the hand off the small of the back, placing the
arm in maximal internal rotation. The examiner then releases the hand. If the hand falls onto the
back because the subscapularis is unable to maintain internal rotation, the test result is positive.
Patients with subscapularis tears have an increase in passive external rotation and a weakened ability
to resist internal rotation.

Tests for Shoulder Instability


• Apprehension Sign for Anterior Instability : The test is performed by abducting the shoulder to 90
degrees, and then slowly externally rotating the shoulder toward 90 degrees. A patient with anterior-
inferior instability will usually become "apprehensive" either verbally or with distressing facial
expressions.

Tests for Subacromial Impingement


• Neer Impingement Sign : Place one hand on the posterior aspect of the scapula to stabilize the
shoulder girdle, and, with the other hand, take the patient's internally rotated arm by the wrist, and
place it in full forward flexion.If there is impingement, the patient will report pain in the range of 70
degrees to 120 degrees of forward flexion as the rotator cuff comes into contact with the rigid
coracoacromial arch.
• Hawkins Impingement Sign : The examiner places the patient's arm in 90 degrees of forward flexion
and forcefully internally rotates the arm, bringing the greater tuberosity in contact with the lateral
acromion. A positive result is indicated if pain is reproduced during the forced internal rotation.

Tests for Long Head of the Biceps


• Speed's Maneuver : The patient's elbow is flexed 20 degrees to 30 degrees with the forearm in
supination and the arm in about 60 degrees of flexion. The examiner resists forward flexion of the
arm while palpating the patient's biceps tendon over the anterior aspect of the shoulder.
• Yergason test : The patient's elbow is flexed to 90 degrees with the thumb up. forearm is in neutral.
The examiner grasps the wrist, resisting attempts by the patient to actively supinate the forearm and
flex the elbow. Pain suggests biceps tendonitis.
Clinical Examination 87

Elbow Examination

1. Steps before beginning examination

• Introduce yourself : “I am Dr. ________, your attending physician and I'll be examining you today. At
any point of the examination you feel uncomfortable, please let me know and I'll stop the examination
right there.”
• Wash/Sanitize hands
• Explain to the patient what you are about to do and gain informed consent.
• Look for medical equipment/therapies, ensure patient is adequately exposed.
• Verbalize the steps of the examination and your findings.

2. Inspection

• SEADS : S: Swelling, E: Erythema, ecchymosis, A: Atrophy/asymmetry (muscle bulk), D: Deformity,


S: Skin changes/scars/bruising. Check for a fixed flexion deformity. Look at the underside of the
elbows to check for any psoriatic plaques, cysts or rheumatoid nodules.

3. Palpation

• Feel the elbow, assessing the joint temperature relative to the rest of the arm.
• Palpate the olecranon process as well as the lateral and medial epicondyles for tenderness (medial for
golfer's elbow and later for tennis elbow), and cubital fossa for tenderness.
• Palpate joint line with elbow flexed to 90o for tenderness and swelling.

4. Range of Motion

• The movements at the elbow joint are all fairly easy to describe and assess. These are flexion,
extension, pronation and supination. Once these have been assessed actively they should be checked
passively checking for power and crepitus.
• Test for varus / valgus instability.

5. Neurological Examination of hand


Motor Sensory
- Median nerve (thumb abduction) - Median nerve (pulp of index finger)
- Radial nerve (wrist extension) - Radial nerve (1st dorsal interosseous space)
- Ulnar nerve (finger abduction) - Ulnar nerve (pulp of 5th finger)

6. Special Tests

• Tennis Elbow : Tennis elbow localises pain over the lateral epicondyle, particularly on active extension
of the wrist with the elbow bent.

• Golfer's Elbow : Golfer's elbow pain localises over the medial epicondyle and is made worse by
flexing the wrist.
88 NAC OSCE | A Comprehensive Review

Hand and Wrist Examination

1. Steps before beginning examination

• Introduce yourself : “I am Dr. ________, your attending physician and I'll be examining you today. At
any point of the examination you feel uncomfortable, please let me know and I'll stop the examination
right there.”
• Wash/Sanitize hands
• Explain to the patient what you are about to do and gain informed consent.
• Look for medical equipment/therapies
• Verbalize the steps of the examination and your findings.

2. Inspection

• Inspect hands :
– Skin (rashes, Gottron's patches, nodules, Raynaud's phenomenon, slerodactyly, scars, skin
atrophy)
– Nails (pitting, onycholysis, splinter haemorrhages, clubbing)
– Muscles (swelling, wasting)
– Joints (swellings, subluxation / deviation of wrist, swan neck / Boutoniere's deformity,
Heberden's/Bouchard's nodes, Z deformity of thumb)
– Inspect palm (palmar erythema, pallor, cyanosis), muscle wasting.
• Inspect elbows :
– Psoriatic skin lesions
– Rheumatoid nodules
– Scars

3. Palpation

• Assess the temperature over the joint areas and compare these with the temperature of the forearm.
• Start proximally and work towards the fingers, feeling the radial pulses and the wrist joints. Then feel
the muscle bulk in the thenar and hypothenar eminences. In the palms, feel for any tendon thickening
and assess the sensation over the relevant areas supplied by the radial, ulnar and median nerves.
• Squeeze over the row of metacarpophalangeal joints whilst watching the patient's face for any
discomfort.
• Bi-manually palpate MCP and interphalangeal joints.
Clinical Examination 89

4. Range of Motion

• Ask the patient to perform the following movements in the sequence mentioned below and observe
for range of movement :
– Make a fist
– Pronate wrist
– Extend little finger (extensor digiti minimi is usually the first tendon to rupture in rheuatoid
arthritis)
– Extend all fingers
• Assess function
– Pinch grip
– Opposition (touch thumb to each finger)
– Power grip (ask patient to squeeze your fingers)
– Froment's test (for ulnar nerve palsy)
– Ask patient to write something / undo a button.
• Assess power
– Wrist extension (radial nerve)
– Thumb abduction (median nerve)
– Finger abduction (ulnar nerve)

5. Neurovascular Examination

Nerve Sensation Motor

Median Lateral portions of the pulp of the Resisted palmar abduction of the
index and middle fingers thumb

Ulnar Lateral pulp areas of the little finger Abduction of the fingers against
resistance

Radial Web space between the thumb and Wrist extension


index finger (anatomical snuff box)

6. Special Tests

• Phalen's test : Forced flexion of the wrist, either against the other hand or by the examiner for 60
seconds will recreate the symptoms of carpal tunnel syndrome.

• Finkelstein's test is used to diagnose DeQuervain's tenosynovitis. Patient is told to flex the thumb
and clench the fist over the thumb followed by ulnar deviation. If there is an increased pain in the
radial styloid process and along the length of the extensor pollicis brevis and abductor pollicis longus
tendons, then the test is positive for De Quervain’s syndrome.

• Tinel's sign : Use the index finger to tap over the carpal tunnel at the wrist. A positive test results
when the tapping causes tingling or paresthesia in the area of the median nerve distribution, which
includes the thumb, index finger, and middle and lateral half of the ring finger. A positive Tinel's sign
at the wrist indicates carpal tunnel syndrome.
90 NAC OSCE | A Comprehensive Review

Breast Examination

• Introduce yourself : “I am Dr. ________, your attending physician and I'll be examining you today. At
any point of the examination you feel uncomfortable, please let me know and I'll stop the examination
right there.”
• Wash/Sanitize hands
• Explain to the patient what you are about to do and gain informed consent.
• Verbalize the steps of the examination and your findings. Ask which side the problem is.
• Make sure patient is adequately exposed, use proper draping techniques

1. General Inspection (with patient sitting on side of bed)

• Inspect with :
– Patient's arm by their sides.
– Patient's arms behind their head (tenses skin)
– Patient's hands on their hips (tenses pectoralis major)
These manoeuvers test for T4 disease – invasion of chest wall / skin. Inspect for :
– Obvious masses
– Scars
– Radiotherapy tattoos
– Skin changes
– Peau d'orange
– Dimpling
– Nipple retraction
– Paget's disease.

2. Inspection (with patient lying down)

– Inspect inframammary folds


– Palpate normal breast followed by abnormal breast
– Palpate all quadrants, nipple and axillary tail of each breast.
– Describe any masses : position, size shape, mobility, number, tenderness, consistency
– Palpate axillary and supraclavicular lymph nodes.
– Ask patient about discharge.

3. Palpate for hepatomegaly.

4. Axillary examination
– Palpate for axillary, supraclavicular and infraclavicular lymph nodes

5. Auscultate lungs.
Clinical Examination 91

Thyroid Examination

1. Steps before beginning examination

• Introduce yourself : “I am Dr. ________, your attending physician and I'll be examining you today. At
any point of the examination you feel uncomfortable, please let me know and I'll stop the examination
right there.”
• Wash/Sanitize hands
• Explain to the patient what you are about to do and gain informed consent.
• Ensure patient is adequately exposed.
• Look for medical equipment/therapies
• Show empathy.
• Verbalize the steps of the examination and your findings.

2. Inspection

• Ask for patient vitals.

• Observe patient : Is patient anxious? Weight gain/loss? Note hoarseness of voice?

• Feel pulse – rate/rhythm/volume

• Face : Facial expression ( dull in hypothyroidism)


Periorbital myxedema
Loss of 1/3rd of eyebrows
Hair – texture/alopecia
Exophthalmos (look from behind patient), lid lag
Ophthalmoplegia (ask patient to follow your finger then ask for diplopia)
Chemosis (redness and watering of eyes)
• Hands : Temperature, moist palms?, texture? color?
Assess for fine tremors, palmar eythema
Pemberton's sign - is the development of facial flushing, distended neck and head superficial
veins, inspiratory stridor and elevation of the jugular venous pressure ( JVP) upon raising of
the patient's both arms above his/her head simultaneously. (Thoracic inlet obstruction
– e.g. due to goitre)
Carpal Tunnel Syndrome (Tinel's and Phalen's Test) – associated with hypothyroidism.
Arm reflexes – brisk in hyperthyroidism.
• Neck : Stand in front of the patient, inspect for neck swellings/goitre
- scars, sinuses, dilated vessels.
92 NAC OSCE | A Comprehensive Review

3. Examination of the Thyroid gland and cervical lymph nodes.

• Swallow tests – Ask patient to swallow water and observe for movement of any masses.
• Tongue protrusion – Thyroglossal cyst moves on tongue protrusion.
• Stand behind the patient and palpate. Assess size, texture, smoothness, margins and mobility of the
thyroid gland (including when swallowing). Note the temperature over gland and adjacent skin.
• Palpate cervical lymph nodes.
• Percuss over sternum – Retrosternal goitre.
• Auscultate for thyroid bruit – Grave's disease.

4. Examination of legs.

• Pretibial myxoedema
• Peripheral edema due to congestive cardiac failure.
• Delayed relaxation of ankle reflex in hypothyroidism.

5. Thank the patient after the examination.


Clinical Examination 93

Mini Mental State Examination

I. ORIENTATION Maximum score = 10


What is today's date? 1
What is the year? 1
What is the month? 1
What is the day today? 1
Can you tell me what season it is? 1
Can you also tell me the name of the location we are in? (Hospital/clinic) 1
What floor are we on? 1
What city are we in? 1
What country are we in? 1
What state are we in? 1
II. IMMEDIATE RECALL Maximum score = 3
Ask the subject if you may test his/her Ball 1
memory. Say the words “ball”, “flag”,
Flag 1
“tree” clearly and slowly. Then ask the
subject to repeat the words. Check for each Tree 1
correct response. The first repetition
determines the score. If the subject does
not repeat all three correctly, keep saying
them up to six tries until the subject can Number of Trials :___
repeat them.
III. ATTENTION AND
CALCULATION
A. Counting Backwards Test Record each response Maximum score = 5
Ask the subject to begin with 100 93 1
and count backwards by 7. Record
86 1
each response. Check one box at
right for each correct response. Any 79 1
response 7 or less than the previous
response is a correct response. The 72 1
score is the number of correct 65 1
subtractions. For example, 93, 86,
80, 72, 65 is a score of 4; 93, 86,
78 70, 62, is 2; 92, 87, 78, 70, 65 is
0.
B. Spelling Backwards Test
Ask the subject to spell the word D 1
"WORLD" backwards. Record each
response. Use the instructions to L 1
determine which are correct
responses, and check one box at R 1
right fore each correct response.
94 NAC OSCE | A Comprehensive Review

C. Final Score O 1
Compare the scores of the Counting W 1
Backwards and Spelling Backwards
tests. Write the greater of the two
Final Score : ____
scores in the box labeled FINAL
(Max of 5 or Greater of
SCORE at right, and use it in
the two scores)
deriving the TOTAL SCORE.
IV. RECALL Maximum score = 3
Ask the subject to recall the three Ball 1
words you previously asked him/her
Flag 1
to remember. Check the Box at right
for each correct response. Tree 1
V. LANGUAGE Maximum score = 9
Naming Watch 1
Show the subject a wrist watch and ask Pencil 1
him/her what it is. Repeat for a pencil.
Repetition
Ask the subject to repeat “No ifs, ands, or Correct repetition 1
buts.”
Three – Stage Command
Establish the subject's dominant hand. Give Takes paper in hand 1
the subject a sheet of blank paper and say,
Folds paper in half 1
"Take the paper in your right/left hand, fold
it in half and put it on the floor." Puts paper on the floor 1
Reading
Hold up the card that reads, “Close your
eyes." So the subject can see it clearly. Ask
him/her to read it and do what it says. Closes eyes 1
Check the box at right only if he/she
actually closes his/her eyes.
Writing
Give the subject a sheet of blank paper and
ask him/her to write a sentence. It is to be
written spontaneously. If the sentence Writes sentence 1
contains a subject and a verb, and is
sensible, check the box at right. Correct
grammar and punctuation are not
necessary.
Copying
Show the subject the drawing of the
intersecting pentagons. Ask him/her to
draw the pentagons (about one inch each Copies pentagons 1
side) on the paper provided. If ten angles
are present and two intersect, check the
box at right. Ignore tremor and rotation.
Clinical Examination 95

DERIVING THE TOTAL SCORE

Add the number of correct responses. The maximux is 30.

23 - 30 Normal
23 - 19 Borderline
Less than 19 Impaired
This is a blank page
Clinical Cases
This is a blank page
Clinical Cases – Protocol for history taking 99

Protocol for history taking

A candidate gets 2 minutes outside the station to read the clinical case senario on the door before entering. It is
essential to get yourself organised in these 2 minutes.

• Read the question properly, understand the requirement and follow instructions (e.g. if you are asked
to do a physical examination, do not start taking history. You will be losing valuable time)

• You will be given a pencil and a booklet with blank pages. It is a good practice to jot down notes.
• Write the name, age and chief complaint of the patient.
• For history of present illness, you can use the mnemonic OCDPQRSTUV+AAA.
• Past and Social History : PAM HUGS FOSS
• Write down your differential diagnosis.

So your note should look something like this :

Name of the patient, age and chief complaints

O - Onset P – Past medical history


C - Course A - Allergies
D - Duration M - Medication
P - Place H – Hospitalizations
Q - Quality U – Urinary problems
R - Radiation G – GIT problems
S - Severity S - Sleep
T - Timing F – Family history
U – You (Associate with your daily activities) O – Obstetrical history
V – Deja vu (Has it happened before?) S – Social history
A – Aggravating factors S – Sexual history
A – Alleviating factors
A – Associated symptoms
Differential diagnosis :

• Knock the door before entering, relax, take a deep breath, smile and enter the room with confidence.
Hand over the stickers to the physician examiner.

• Introduce yourself to the patient : e.g. “Good morning/afternoon/evening, Mr./Miss ______, I am


Dr. _____ and I'm your attending physician today.”
• “So, Mr./Miss _____, what complaint has brought you here today?”
• Show empathy : “Oh! I am sorry to hear that, I shall try my best to help you.”
100 NAC OSCE | A Comprehensive Review

Data Collection : OCDPQRSTUV+AAA

• Onset : When did it start? Sudden? Gradual?


• Course : Is it getting worse, better or just the same?
• Duration : You sat it started ....... ago, does it come and go? How often does it come? For how long
does it stay each time?
• Place : Show me exactly where it hurts?
• Quality : Tell me how it feels like? Is it sharp/burning/dull/crampy?
• Radiation : Does it travel to anywhere?
• Severity : On a scale from 1 to 10, with 1 being the mildest and 10 the worst pain, how bad is it?
• Timing : Is it worse at a particular time of the day?
• U (You) your daily activities : Does it interfere in your day to day activities? Does it change with your
daily activities like posture, rest, eating, exertion?
• V (Deja vu) : Has it happened before? When? What happened then? What medication?

• Aggravating factors : What makes it worse?


• Alleviating factors : What make it better?
• Associated symptoms : Have you noticed anything that occurs with it?
• A1 : Associated constitutional symptoms like fever, shortness of breath, cough, nausea,
vomiting, diarrhea, headache, fatigue (FSC NVD HF)
• A2 : Associated symptoms to particular system
Respiratory : Chest pain, shortness of breath, cough, sputum, wheezing, runny nose, post
nasal drip, contact with ill person, night sweats, questions for pulmonary embolism (leg pain,
long travel, surgery and ocp use in females)
CVS : chest pain, orthopnea, paroxysmal nocturnal dyspnea, palpitations, tachycardia.
GIT : pain in abdomen, stool, bowel movements, vomiting, jaundice, blood in stools, diet,
travel.
Neurology : headache, loss of consciousness, weakness, paresthesias
Higher Mental Function : orientation, memory, consciousness.
Motor : weakness of limbs.
Sensory : tingling sensation.
Cerebellum : gait, balance.
Cranial Nerves : speech, swallowing, vision, hearing.

PAM HUGS FOSS

• “Ok, Mr./Miss _____, Now I need to ask you about your health in general. Is that okay with you?”

• Past Medical History : What other medical problems do you have? (Diabetes/Hypertension/Asthma /
Cancer?)

• Allergies : Do you have any allergies? Are you allergic to any drugs?
Clinical Cases – Protocol for history taking 101

• Medication : Do you take medicines at present?

• Hospitalizations : (medical/surgical/trauma)

• Urinary problem :

• GIT : bowel movements?

• Sleep?

• “ I am going to ask you a few personal questions that will help me in my diagnosis. Is that okay with
you? Let me begin by asking you about your family health.”

• Family history : similar complaits in the family? Cancer in the family? Depression? Suicide?

• Obstetrical History : When way your last pap smear? Wast it normal? Any history of STIs?

• “Now I need to ask you about your sexual health. Whatever you tell me will be kept confidential. Is
that okay with you?”

• Sexual History : Are you in a physical relationship? How long?


Do you practise safe sex?
Any risk of STIs?

• Social History : Smoking, how many packs? Alcohol, amount? Recreational drugs?

• “ Is there anything else that you want to share?”

• WRAP UP
102 NAC OSCE | A Comprehensive Review

Tammy Robbins, a 48 years old lady presented with heart racing and chest discomfort for the past 3 days.
Take a focused history and perform focused physical examination.
Vitals: BP - 90/70 mm Hg, HR - 146/min, irregular, RR - 12/min, Temp - 37.5°C

Clinical Info: Ms Tammy Robbins is a known hypertensive with CAD for the past 10 years, who presented
with sudden onset of palpitations and chest discomfort for the past 3 days. Her symptoms are worsening for
the past 24 hours. She has dyspnea. She has dizziness for the past 12 hours. Pedal edema is 2 +. She had 2
vessel angioplasty done 5 years ago. ECG shows absent P waves with irregular narrow QRS complexes.
Bilateral basal rales present on lung auscultation.
Clinical Case : Atrial Fibrillation

HOPI Past History


• OCD PQRST UVW + AAA • Do you have any medical illnesses?
• How did it start? Sudden or gradual. • Are you allergic to any medications?
• Is it getting worse/better or no changes in • Any surgeries in the past?
the symptoms? • Past h/o recurrent infections?
• Duration of palpitations?
• Associated with chest discomfort? Family and Social History
• Describe the type of chest discomfort? • Do you smoke? Duration & frequency.
• Do you have chest pain? • Do you consume alcohol? Duration &
• Any shortness of breath? frequency.
• Any dizziness/light headedness? • Do you take any recreational drugs?
• Any fever/cough/nausea/vomiting? • Any family history of cancers/ medical
• Any hemoptysis? illnesses?
• Any chills/night sweats?
• Any malaise/fatigue/weakness?
• Any swelling of feet?
• Any numbness/paresthesias?
• Any visual problems?
• Any relieving factors?
• Any aggravating factors?
• Any recent trauma?
• List of current medications/compliance?

Differential Diagnosis Management


Atrial fibrillation secondary to: • Treat the primary cause.
1. Congestive heart failure. • Admit in cardiac care unit.
2. Ischemic heart disease. • Rate control by beta blockers, calcium
3. Hypertension. channel blockers or digoxin.
4. Thyroid disease. • Anticoagulation with heparin, then warfarin.
• Rhythm control by electro or medical
Investigations cardioversion.
• CBC, electrolytes, glucose. • Assess Stroke risk by using CHADS2 score.
• LFTs, RFTs, TSH.
• CK, LDH, Cardiac enzymes.
• 12 lead ECG.
• Echocardiogram.
• Chest X ray.
Clinical Cases - Medicine 103

Simon Charles, a 20 years old male presented to your clinic with shortness of breath for the past 24 hours.
Take a focused history and perform focused physical examination.
Vitals: BP - 110/80 mm Hg, HR – 110/min, RR – 22/min, Temp – 37.5°C.

Clinical Info: Mr Simon Charles has a h/o of Asthma since the past 10 years. He recently cleaned his
basement 1 day ago and his asthma symptoms exacerbated. He is having wheezing, chest tightness,cough and
SOB. He is currently on inhalers with no night symptoms. On examination, he has dyspnea and wheezing
present in all lung fields. He has mild exacerbation of his symptoms and needs only outpatient treatment.
Clinical Case : Asthma

HOPI Past History


• OCD PQRST UVW + AAA • Do you have asthma/other allergies?
• How did it start? Sudden or gradual. • Are you on any medications and compliance?
• Is it getting worse/better or no changes in • Are you allergic to any medications?
the symptoms? • Any hospitalizations for asthma?
• Do you wake up in night with shortness of • H/o any other medical illness?
breath?
• Do you have noisy breathing? Family and Social History
• H/o fever/sore throat/rash? • Do you smoke? Duration & frequency.
• Any night time cough? • Do you consume alcohol? Duration &
• Any sputum production? frequency.
• Any chest pain with deep inspiration? • Do you use recreational drugs? TRAPPED.
• Any recent activity which worsened your • Any family history of asthma/allergies?
symptoms?
• Any exposure to cold air/dust/mites?
• Any pets at home?
• Any changes in bowel & urinary habits?
• Any contact with a sick person?
• Any recent travel?
• Any recurrent episodes?
• Do symptoms affect your daily activities?

Differential Diagnosis Management


• Asthma. • Ventolin 2-4 puffs MDI q4-6h.
• Acute bronchitis. • Fluticasone 2-4 puffs bid(Inhaled steroids)
• GERD. • Asthma education:
• Pneumonia. Inhaler use.
Spacer and holding chamber use.
Investigations Symptom monitoring.
• CBC. Early recognition of exacerbation.
• Pulmonary function tests. Avoid environmental allergens.
• Peak flow meter. Encase mattress and pillow in
• Chest X ray. impermeable cover.
Reduce indoor humidity to <50%.
104 NAC OSCE | A Comprehensive Review

Larry Edwards, a 55 years old man presented with blood in sputum and shortness of breath for the past 5
days. Take a focused history and perform focused physical examination.
Vitals: BP - 160/110 mm Hg, HR - 96/min, RR - 18/min, Temp – 37.5°C.

Clinical Info: Mr Larry Edwards is a known hypertensive who presented with shortness of breath and blood
in sputum for 5 days. It is gradual in onset. He has chest pain also. No fever or recurrent pneumonia. He is
non compliant with his medications. He is on Losartan, Aspirin, Atorvas, multi vitamins. He has not taken
his anti-hypertensives for 4 weeks. Has paroxysmal nocturnal dyspnea and orthopnea.
Clinical Case : Congestive Heart Failure

HOPI Past History


• OCD PQRST UVW + AAA • Do you have diabetes or hypertension?
• How did it start? Sudden or gradual. • Are you on any medications?
• Is it getting worse/better or no changes in • Compliance with medications?
the symptoms? • Are you allergic to any medications?
• Duration of blood in sputum? • Any surgeries in the past?
• Amount / color of blood of sputum? • Past h/o recurrent infections?
• Any of shortness of breath?
• Present at rest or with exertion? Family and Social History
• H/o orthopnea? • Do you smoke? Duration & frequency.
• H/o paroxysmal nocturnal dyspnea? • Do you consume alcohol? Duration &
• Any chest pain/palpitations? frequency.
• Type of pain – sharp or dull? • Do you use recreational drugs? TRAPPED.
• Does the pain radiate or shoot anywhere? • Any family history of cancers/ medical
• Severity of pain on a scale of 1-10. illnesses?
• Do your symptoms change with time?? • Any family history of heart disease?
• H/o fever/cough/sore throat/rash?
• Any chills/night sweats?
• Any swelling of face or feet?
• Any change in weight/appetite?
• Any bruises on the body?
• Any trauma recently?

Differential Diagnosis Management


• Pulmonary Edema due to CHF. • Symptomatic treatment.
• Pulmonary embolism. • Admit in cardiac care unit.
• Bronchiectasis. • Inj Lasix 40 mg IV stat.
• Drug induced coagulopathy. • Beta blockers & ACE inhibitors.
• Pneumonia. • Inj Morphine 2-4 mg IV.
• Nasal oxygen.
Investigations • Sublingual Nitroglycerines.
• CBC, electrolytes, glucose. • Position - 45 degree elevation of head end.
• LFTs, RFTs,
• PT, PTT, INR.
• 12 lead ECG.
• Echocardiogram.
• Chest X Ray.
Clinical Cases - Medicine 105

Jack Allen, a 65 years old man presented with sudden onset of right arm weakness 4 hours ago. Take a focused
history and perform focused physical examination.
Vitals: BP - 160/90 mm Hg, HR - 96/min, RR - 12/min, Temp – 37.5°C.

Clinical Info: Mr Jack Allen presented with sudden onset of right arm weakness with numbness and
paresthesias 4 hours ago. He has slurring of speech, blurring of vision and mild headache. He has no
nausea,vomiting or head trauma. No weakness of lower limbs or left arm. No incontinence. He is hypertensive
for the past 10 years and non compliant to medications.
Clinical Case : Cerebrovascular Attack

HOPI Past History


• OCD PQRST UVW + AAA • Do you have diabetes or hypertension?
• How did it start? Sudden or gradual. • Are you on any medications/compliance?
• Is it getting worse/better or no changes in • Are you allergic to any medications?
the symptoms? • Any surgeries in the past?
• Duration of weakness?
• Where is weakness located? Family and Social History
• Is there any associated pain in the arm? • Do you smoke? Duration & frequency.
• Any numbness/tingling/paresthesias? • Do you consume alcohol? Duration &
• Any limitations in right arm movements? frequency.
• Any abnormal position of right arm? • Do you use recreational drugs? TRAPPED.
• Do your symptoms change with time? • Any family history of cancers/ medical
• Any changes in vision? illnesses?
• Any changes in speech?
• Any difficulty swallowing/drooling of saliva?
• Any headache/trauma?
• Any weakness of left arm or lower limbs?
• Any seizures?
• Any changes in facial expression?
• H/o fever/cough/sore throat/rash?
• Any chest pain/shortness of breath?
• Any changes in bowel & urinary habits?

Differential Diagnosis Management


• Cerebrovascular Attack. • Admit urgently.
• Transient Ischaemic Attack. • Urgent neurological consult.
• Sub Arachnoid Hemorrhage. • Neurovitals q 1 hourly.
• Nasal oxygen.
Investigations • Tab Aspirin 325 mg PO OD.
• CBC, ESR, Glucose. • Blood pressure control.
• Lipid profile, RFTs. • Thrombolysis to be done only if presented
• PT, PTT, INR. within 3 hours of onset of symptoms.
• ECG.
• CT ± MRI Head.
• Echo, Carotid doppler.
106 NAC OSCE | A Comprehensive Review

Allan Smith, a 70 years old man presented with light headedness and dizziness for 2 days. Take a focused
history and perform focused physical examination.
Vitals: BP - 110/80 mm Hg, HR - 56/min, irregular, RR - 12/min, Temp – 37.0°C.

Clinical Info: Mr Allan Smith is a known hypertensive for the past 20 years on medications. He is
experiencing light headedness and dizziness for the past 2 days. He has palpitations and mild chest pain for 2
weeks. He is breathless on exertion. Has 2+ pedal edema. No fainting episode. No trauma recently. No visual
changes or limb weakness. He is on Losartan, Ramipril, Digoxin, Atorvas, Aspirin, Nexium and Calcium.
ECG shows Type 2 Second Degree AV block.
Clinical Case : Digoxin Toxicity

HOPI Past History


• OCD PQRST UVW + AAA • Do you have diabetes or hypertension?
• How did it start? Sudden or gradual. • Are you allergic to any medications?
• Is it getting worse/better or no changes in • Any surgeries in the past?
the symptoms? • Past h/o recurrent infections?
• Duration of dizziness?
• Any relieving factors? Family and Social History
• Any aggravating factors? • Do you smoke? Duration & frequency.
• Any episode of fainting? • Do you consume alcohol? Duration &
• Any weakness/tingling/paresthesias of limbs? frequency.
• Any chest pain/palpitations? • Do you use recreational drugs? TRAPPED.
• Any shortness of breath? • Any family history of cancers/ medical
• Any visual changes/headache? illnesses?
• Any hearing loss/ear discharge?
• Any excessive sweating?
• Any fever/cough/vomiting?
• Any abdominal pain/loss of appetite?
• Any changes in bowel/urinary habits?
• Any swelling of ankles?
• Do your symptoms vary with change in head
position?
• List of current medications & compliance?
• Any changes in medications & dosages?
• Any recent trauma?

Differential Diagnosis Management


• Digoxin toxicity. • Admit in cardiac care unit.
• Arrhythmia. • Stop Digoxin.
• TIA. • Start Digibind in case of massive overdose or
• Anxiety disorder. refractory toxicity.
• Medication induced bradycardia. • External Pacemaker.

Investigations
• Serum Digoxin level.
• CBC, electrolytes, RFTs.
• INR/PTT, glucose.
• ECG, 24 hour Holter monitor.
• Echocardiogram,Carotid Doppler.
Clinical Cases - Medicine 107

Lisa Giroux, a 25 years old lady presented with lump in the neck for the past 7 days. Take a focused history
and perform focused physical examination.
Vitals: BP - 120/88 mm Hg, HR – 96/min, RR – 12/min, Temp – 38.5°C.

Clinical Info: Ms Lisa Giroux noticed 2 lumps on the right side of her neck below the mandible. She has
positive history of fever for 5 days,sore throat and fatigue. On examination she has 2 enlarged, tender
submandibular lymph nodes.
Clinical Case : Infectious Mononucleosis (Sore throat )

HOPI Past History


• OCD PQRST UVW + AAA • Do you have diabetes or hypertension?
• How did it start? Sudden or gradual. • Are you on any medications?
• Is it getting worse/better or no changes in • Are you allergic to any medications?
the symptoms? • Any surgeries in the past?
• Since how long have you noticed the lumps? • Past h/o recurrent infections?
• Where are the lumps located?
• How does the lumps feel like? Family and Social History
• Is there any pain associated with the lumps? • Do you smoke? Duration & frequency.
• Type of pain - sharp or dull pain? • Do you consume alcohol? Duration &
• Does the pain radiate or shoot anywhere? frequency.
• Severity of pain on a scale of 1-10. • Do you use recreational drugs? TRAPPED.
• Do your symptoms change with time? • Any family history of cancers/ medical
• H/o fever/cough/sore throat/rash? illnesses?
• Any chest pain/shortness of breath?
• Any abdominal pain?
• Any weakness or myalgia?
• Any chills/night sweats?
• Any change in voice?
• Any change in appetite?
• Any change in weight?
• Any changes in bowel & urinary habits?
• Any contact with a sick person?
• Recent travel?

Differential Diagnosis Management


• Infectious mononucleosis. • Symptomatic treatment.
• Acute bacterial pharyngitis. • Maintain adequate hydration.
• Acute viral pharyngitis. • Rest.
• Lymphoma. • Analgesics for pain.
• Toxoplasmosis. • Treat Streptococcal pharyngitis, if coexists.
• Avoid active sports.
Investigations • Steroids used only for severe symptoms.
• CBC.
• Monospot test.
• Throat swab for gram stain, culture &
sensitivity.
108 NAC OSCE | A Comprehensive Review

Jason Hardinge, a 26 years old university student wants to discuss confidential issues with a doctor. Take
focused history and address his concerns.
Vitals: BP - 120/88 mm Hg, HR - 88/min, RR - 12/min, Temp - 37.5°C
Clinical Info: Mr Jason Hardinge is having difficulty in maintaining erection during intercourse for the past 4
months. He is currently in a monogamous relationship with his girlfriend. He is on Paroxetin for his mood
disorder for 6 months. No other medical illnesses. Girlfriend is very understanding. He has no morning or
night tumescence. Has no erection with self stimulation. He is very anxious about this issue.
Clinical Case : Impotence

HOPI • Sexual orientation -


• OCD PQRST UVW + AAA heterosexual/homosexual/bisexual?
• How did it start? Sudden or gradual. • Current relationship?
• Is it getting worse/better or no changes in • Performance anxiety?
the symptoms? • Any stresses in the relationship?
• Duration of impotence? • Any stresses at home or school?
• Course & frequency of impotence? • Sexual partner's expectations?
• Description of the problem: no erection at • Are you on any medications?
all, cannot sustain erection, ejaculate too • Compliance/type of medications – details?
quickly to satisfy partner?
• Cannot achieve orgasm or orgasm without Past History
ejaculation? • Do you have diabetes or hypertension?
• Any retrograde ejaculation? • Are you allergic to any medications?
• Circumstances under which impotence • Any surgeries in the past?
occurs only with certain partners, only at • Any trauma in the past?
certain times or locations, what percentage of • Past h/o recurrent infections?
the time?
• Is impotence related to lack of sexual desire? Family and Social History
• Presence and firmness of morning or • Do you smoke? Duration & frequency.
nocturnal erections. • Do you consume alcohol? Duration &
• Can sustain erections with self stimulation? frequency.
• Associated problems: anxiety attacks, • Do you use recreational drugs? TRAPPED.
anhedonia/depression, perineal or peripheral • Any family history of cancers/medical
numbness, poor peripheral circulation. illnesses?

Diagnosis Management
Impotence secondary to antidepressants. • Complete physical exam.
• Reassurance, counseling both patient &
Causes of impotence : (IMPOTENCE) partner.
Iatrogenic Mechanical Psychological • Inform that symptoms are due to side effects
Occlusive vascular Trauma Extra factors of anti-depressants.
Neurogenic Chemical Endocrine • Symptoms are reversible by changing the
dose or the type of drug.
Investigations • Substitute with another anti-depressant:
• CBC, blood glucose, TSH. Minimal to no sexual dysfunction
• Se Testosterone. Nefazodone (Serzone)
• Urinalysis. Bupropion (Wellbutrin)
• Endocrine lab tests, if indicated: Low risk of sexual dysfunction (10-15%)
FSH,LH, Prolactin. Fluvoxamine (Luvox)
Citalopram (Celexa)
Venlafaxine (Effexor)
• Avoid alcohol/smoking.
• Medical treatment: Tab Sildenafil 25-5 mg
PO 0.5 to 4 hours prior to coitus.
Clinical Cases - Medicine 109

Taylor Jackson, a 18 years old boy presented with fever, neck stiffness and photophobia to the ER. Take a
focused history and perform focused physical examination.
Vitals: BP - 90/70 mm Hg, HR - 110/min, RR - 12/min, Temp – 39.0°C.

Clinical Info: Mr Taylor Jackson has high grade fever for the past 3 days along with neck stiffness. He has
photophobia for the past 1 day. He alert & conscious. No seizures. Has headache with nausea & vomiting. No
ear discharge. Has a purpuric rash on chest and lower limbs. No recent trauma. Has h/o contact with sick
person with similar symptoms. O/E: Febrile, Brudzinski's and Kernig's sign are positive.
Clinical Case : Meningitis

HOPI Past History


• OCD PQRST UVW + AAA • Do you have medical illnesses?
• How did fever start? Sudden or gradual. • Are you on any medications?
• Is it getting worse/better or no changes in • Are you allergic to any medications?
the symptoms? • Any surgeries in the past?
• Duration of fever? • Past h/o recurrent infections?
Continuous/intermittent/remittent?
• Duration of neck stiffness? Family and Social History
• Associated with headache/nausea/vomiting? • Do you smoke? Duration & frequency.
• Any photophobia/phonophobia? • Do you consume alcohol? Duration &
• Any fatigue/malaise? frequency.
• Any fainting/seizures/confusion/irritability? • Do you use recreational drugs? TRAPPED.
• H/o fever/cough/sore throat/rash? • Any family history of cancers/ medical
• Any chills/night sweats? illnesses?
• Any chest pain/shortness of breath?
• Any abdominal pain?
• Any changes in bowel & urinary habits?
• Any contact with a sick person?
• Recent travel?
• Recent head trauma?

Differential Diagnosis Management


• Bacterial Meningitis • Admit under isolation.
• Encephalitis. • Start empiric antibiotics prior to LP.
• Intra-cerebral abscess. Inj Cefotaxime 2 g IV q4h +
Inj Ampicillin 50 mg/kg IV q6h.
Investigations Inj Dexamthasone 10 mg q6h IV x 4 days.
• CBC, electrolytes, glucose. • Treat all close contacts.
• RFTs, ABG.
• Lumbar puncture - gram stain, microscopy,
culture/sensitivity (rule out papilledema).
• Blood culture/sensitivity, gram stain.
• Urinalysis.
• CT scan head.
• Chest X Ray.
110 NAC OSCE | A Comprehensive Review

James Irwin a 30 years old man presented to your clinic with symptoms of headache. Take a focused history
and address his concerns.
Vitals: BP - 120/88 mm Hg, HR - 96/min, RR - 12/min, Temp - 37.5°C.

Clinical Info: Mr James Irwin presented with unilateral, pulsating headache, grade 7/10 for the past 6
months. He experiences aura prior to the onset of headache. Associated with nausea,vomiting and
photophobia. Stimulated by stress and excessive caffeine intake. One episode lasts for 8-12 hours. He had 6
attacks in past 6 months. Currently on advil prn.
Clinical Case : Headache (Migraine)

HOPI Past History


• OCD PQRST UVW + AAA • Do you have diabetes or hypertension?
• How did it start? Sudden or gradual. • Any head trauma in the past?
• Location of pain? • Are you on any medications?
• Is it getting worse/better or no changes in • Are you allergic to any medications?
the symptoms? • Any surgeries in the past?
• Since how long have you noticed the pain? • Past h/o recurrent infections?
• Number of episodes of headache till now?
• How many hours pain lasts? Family and Social History
• How does the pain feel like? • Do you smoke? Duration & frequency.
• Does the pain radiate or shoot anywhere? • Do you consume alcohol? Duration &
• Severity of pain on a scale of 1-10. frequency.
• Any provoking factors like food, alcohol, • Do you use recreational drugs? TRAPPED.
caffeine, chocolate, wine, stress. • Any family history of cancers/ medical
• How does the pain decrease? illnesses?
• Do your symptoms change with time?
• H/o fever/cough/sore throat/nasal stuffiness?
• Any excessive lacrimation?
• Any visual changes prior to onset of
headache?
• Any neck pain/rigidity/seizures?
• Any weakness/muscle pain/paresthesias?
• Any mood changes/decreased sleep/energy?

Differential Diagnosis Management


• Migraine with Aura. • Symptomatic treatment.
• Tension Headache. • Avoid triggers.
• Cluster Headache. • Mild attack - ASA, Ibuprofen.
• Temporal Arteritis • Moderate attack - Anti-emetics,
Ibuprofen, Triptans, ergots.
Investigations • Severe attack - Anti-emetics, Ergotamines,
• CBC, ESR Triptans.
• Migraine prophylaxis – Propranolol,
Amytriptiline, Verapamil.
Clinical Cases - Medicine 111

Michael Smith, a 55 years old man presented with chest discomfort for the past 1 hour. Take a focused
history and perform focused physical examination.
Vitals: BP - 160/90 mm Hg, HR - 96/min, RR - 12/min, Temp – 37.5°C.

Clinical Info: Mr Michael Smith presented with left sided chest discomfort for the past 1 hour. He has pain
in his left shoulder and jaw. He has shortness of breath along with palpitations. He is a known hypertensive
and diabetics on oral medications. His wife states he is non-compliant with his medications. ECG shows ST
elevation in leads II,III and avF.
Clinical Case : Chest Pain (Myocardial Infarction)

HOPI Past History


• OCD PQRST UVW + AAA • Do you have diabetes or hypertension?
• How did it start? Sudden or gradual. • Are you on any medications/compliance?
• Is it getting worse/better or no changes in • Are you allergic to any medications?
the symptoms? • Any surgeries in the past?
• Duration of chest discomfort? • Past h/o recurrent infections?
• Location of chest discomfort?
• How does the discomfort feel like? Family and Social History
• Any pain associated with chest discomfort? • Do you smoke? Duration & frequency.
• Does the pain radiate or shoot anywhere? • Do you consume alcohol? Duration &
• Severity of pain on a scale of 1-10. frequency.
• Any shortness of breath? • Do you use recreational drugs? TRAPPED.
• Any palpitations/diaphoresis/dizziness? • Any family history of cancers/ medical
• Do your symptoms become change with illnesses?
time??
• H/o fever/cough/sore throat/rash?
• Any hemoptysis?
• Any heart burn?
• Any abdominal pain?
• Any weakness or myalgia?
• Any changes in bowel & urinary habits?
• Any swelling of feet?
• Any relieving factors?
• Any aggravating factors?

Differential Diagnosis Management


• Myocardial Infarction. • Admit to cardiac care unit.
• Unstable Angina. • Urgent cardiology consult.
• Pericarditis. • Nasal oxygen.
• Panic Attack. • Tab Aspirin 160-325 mg chewable stat.
• Inj Morphine 2-4 mg IV stat & prn.
Investigations • Sublingual Nitrolycerine 0.4 mg stat, repeat
• CBC, electrolytes, glucose. 3 times.
• INR/PTT. • Inj Atenolol 5 mg IV bolus over 5 mins, then
• Serial CK-MB & Troponin q8h X 3. repeat.
• ABG. Then switch to oral Atenolol 50 mg.
• CXR. • Cardiac monitoring every 30 minutes-1
• 12 lead ECG. hour.
• Bed rest.
• Consider PCI, in case of no
contraindications.
112 NAC OSCE | A Comprehensive Review

Adam Sawyer, a 18 years old male presented with fever, cough for 1 week along with shortness of breath. Take
a focused history and perform focused physical examination.
Vitals: BP - 110/70 mm Hg, HR – 96/min, RR – 20/min, Temp – 38.5°C.

Clinical Info: Adam Sawyer has fever and expectorant cough for the past 1 week. He has wheezing and
shortness of breath for 2 days. On auscultation of chest, there is decreased breath sounds on left side with
rales present.
Clinical Case : Pneumonia

HOPI Past History


• OCD PQRST UVW + AAA • Do you have diabetes or hypertension?
• How did it start? Sudden or gradual. • Are you on any medications?
• Is it getting worse/better or no changes in • Are you allergic to any medications?
the symptoms? • Any surgeries in the past?
• Duration of cough? • Past h/o recurrent infections?
• Type of cough - dry or expectorant?
• Is there any chest pain? Family and Social History
• Location of chest pain? • Do you smoke? Duration & frequency.
• Type of pain – sharp or dull? • Do you consume alcohol? Duration &
• Does the pain radiate or shoot anywhere? frequency.
• Severity of pain on a scale of 1-10. • Do you use recreational drugs? TRAPPED.
• Pain present with deep breathing? • Any family history of cancers/ medical
• Onset of shortness of breath? illnesses?
• Any noisy breathing present?
• Any chills/night sweats?
• Any changes in bowel & urinary habits?
• Any contact with a sick person?
• Recent travel?

Differential Diagnosis Management


• Community Acquired Pneumonia. • Outpatient treatment.
• Acute exacerbation of COPD. • Tab Doxycycline 100 mg PO bid x 7 days.
• Acute bronchitis. • Tab Azithromycin 500 mg PO OD x 5 days.
• Asthma. • Rest and adequate hydration.
• Symptomatic treatment.
Investigations • Inpatient treatment for 48-72 hours.
• CBC. • Tab Levofloxacin 750 mg PO q24h.
• Sputum gram stain with culture/sensitivity. • Tab Amoxicillin 1000 mg PO qid +
• Electrolytes. • Tab Clarithromycin 500 mg PO bid.
• Renal function tests.
• Chest X Ray.
Clinical Cases - Medicine 113

Helen Solazzo is an ICU nurse who had a needle stick injury 30 minutes ago while drawing blood sample
from a patient . Take a focused history and address her concerns.
Vitals: BP - 120/88 mm Hg, HR - 86/min, RR - 12/min, Temp - 37.0°C.

Clinical Info: Ms Helen Solazzo had a needle stick injury in the ICU 30 minutes ago. She was drawing blood
sample at that time. She was wearing gloves. She has no high risk behavior. Her immune status for
HIV/HCV/HBsAg is negative as of 1 year ago. Patient's immune status is unknown as of now.
Patient's result come back positive for HIV.
Clinical Case Diagnosis: Post exposure prophylaxis for HIV.

HOPI Past History


• Mechanism of injury. • Do you have medical illnesses?
• Was the nurse wearing gloves? • Are you on any medications?
• Were adequate occupational precautions • Are you allergic to any medications?
taken? (like gloves, mask.) • Any surgeries in the past?
• Depth of needle penetration? • Past h/o recurrent infections?
• Type of needle- hollow or solid?
• Needle gauge? Family and Social History
• Was any blood present on the tip of needle? • Do you smoke? Duration & frequency.
• Any contact with patient's body fluids? • Do you consume alcohol? Duration &
• Steps taken after injury? frequency.
• Immune status of the nurse? • Do you use recreational drugs? TRAPPED.
• When was the last immune status checked? • Any family history of cancers/ medical
• Immunization for Hepatitis A and Hepatitis illnesses?
B?
• Any high risk behavior like unprotected sex?
• Any similar incident in the past?
• Is the immune status of the patient known?
• Patient's medical history?

Investigations Management
For Healthcare professional: • Reassurance.
• CBC, electrolytes. • Refer to Infectious Disease clinic.
• RFTs, LFTs. • Report to occupational health dept within 72
• HIV, HCV, HBsAg. hours & every 2 weekly.
For the patient: inform the pt. • Certify to worker's compensation board for
• CBC, electrolytes. file claim.
• HIV, HCV, HBsAg. • Advise about safe sex practices.
• In case of positive HIV/HCV/HBsAg do • Avoid pregnancy/breast feeding.
viral loads & CD 4 counts. • Repeat blood work 6 weeks,12weeks,6
months and 12 months.
• Patient HIV + then start the nurse on post
exposure prophylaxis for 4 weeks.
• Counsel about side effects of medications.
114 NAC OSCE | A Comprehensive Review

Jacob Sandler, a 50 years old man presented hemoptysis and right sided calf swelling for the past 2 days. He
had knee replacement surgery 1 week ago. Take a focused history and perform focused examination.
Vitals: BP - 140/80 mm Hg, HR - 110/min, RR - 18/min, Temp – 37.5°C.

Clinical Info: Mr Jacob Sandler had a right knee replacement 1 week ago. He now presented with 2 episodes
of hemoptysis and right calf swelling with tenderness. He has no fever or infection of surgical wound.
Homan's sign is positive with ECG showing S1Q3T3 pattern.
Clinical Case : Pulmonary Embolism

HOPI Past History


• OCD PQRST UVW + AAA • Do you have diabetes or hypertension?
• How did it start? Sudden or gradual. • Are you on any medications?
• Is it getting worse/better or no changes in • Are you allergic to any medications?
the symptoms? • Any surgeries in the past?
• Duration of blood in sputum? • Past h/o recurrent infections?
• Amount / color of blood of sputum?
• Onset of shortness of breath/chest pain? Family and Social History
• Present at rest or with exertion? • Do you smoke? Duration & frequency.
• H/o orthopnea? • Do you consume alcohol? Duration &
• H/o paroxysmal nocturnal dyspnea? frequency.
• Duration of leg swelling? • Do you use recreational drugs? TRAPPED.
• Where is the leg swelling located? • Any family history of cancers/ medical
• Any leg pain associated with swelling? illnesses?
• Type of pain - sharp or dull pain?
• Does the pain radiate or shoot anywhere?
• Severity of pain on a scale of 1-10.
• Any pain during rest?
• Any pain in the night time?
• Any skin discoloration of legs/nail changes?
• Any skin ulceration of legs?
• Any fever/cough/cold?
• Any headache/dizziness?
• Any weakness/muscle pain?
• Any prolonged immobilisation?
• Any pain/discharge from the wound?
• Any pre-op or intra-op complications?

Differential Diagnosis Management


• Pulmonary embolism. • Admit in Intensive care unit.
• Deep Vein Thrombosis. • Elevate head end to 45 degree.
• Pulmonary edema due to CHF • Nasal oxygen.
• Myocardial infarction. • Give chewable ASA 160-325 mg
immediately.
Investigations • Secure IV access, bolus IV Lasix 40 mg.
• CBC, RFTs. • Ventolin if wheezes are heard.
• Electrolytes, glucose. • Sublingual nitro spray, if blood pressure is
• Serial CK-MB and Troponin q8h X 3. adequate.
• Arterial blood gases. • Inj Morphine 1 mg IV.
• D-dimer, PT, PTT, INR, factor assay. • Inj Heparin 7500 U IV bolus, then infuse at
• ECG, Echocardiogram. 1200 U/h, then switch to warfarin.
• Chest X Ray. • Continue anticoagulation for 3 months.
• PET Scan or VQ scan.
• Doppler of lower limbs.
Clinical Cases - Medicine 115

Jasper Preudhomme, a 16 years old boy a known epileptic presented to your clinic for the first time. Take a
focused history and address his concerns.
Vitals: BP - 120/88 mm Hg, HR - 96/min, RR - 12/min, Temp – 37.0°C.

Clinical Info: Mr Jasper Preudhomme is a known epileptic for the past 6 years. He is on regular anti-
epileptics and is non-compliant. His last seizure was 2 months ago. He recently started consuming alcohol
with friends. His main concern is to get a driver's license.
Clinical Case : Seizure disorder

HOPI Past History


• Age of onset. • Do you have diabetes or hypertension?
• Precipitants: Sleep deprivation, drugs, • Any surgeries in the past?
alcohol, TV screen,emotional upset. • Past h/o recurrent infections?
• Describe type of seizures.
• Salivation, cyanosis, tongue biting, Family and Social History
incontinence, automatisms, motor vs. • Do you smoke? Duration & frequency.
visual/gustatory/olfactory. • Do you consume alcohol? Duration &
• Frequency & duration of seizures. frequency.
• What body parts affected and in what order? • Do you use recreational drugs? TRAPPED.
• Promontory signs (presence of aura: implies • Any family history of cancers/ medical
focal attack). illnesses?
• Post-ictal state (decrease in level of
consciousness, headache, sensory
phenomena, tongue soreness, limb pains,
Todd's paralysis - hemiplegia),
• Degree of control achieved with
medications.
• Was a CT scan done when seizures were first
diagnosed?
• Number and description of recent seizures.
• Are they different from previous seizures?
• Is the patient having any new symptoms like
headache, vomiting, new neurological
deficits?
• Side effects of antiepileptics: drowsiness,
poor concentration, poor performance in
school, ataxias, peripheral neuropathy,acne,
nystagmus, dysarthria, hypertrichosis
,gingival hypertrophy.

Diagnosis Management
• Seizure Disorder • Discuss compliance of medications.
• Regular follow up.
Investigations • Avoid alcohol consumption/smoking.
• CBC, electrolytes. • Avoid recreational drugs.
• Serum drug levels. • Inform to the patient Ministry of
• EEG. Transportation regulations require patient to
be seizure free for 1 year or more.
• Notify Ministry of Transportation as
required by law.
116 NAC OSCE | A Comprehensive Review

Samantha Ho, a 56 years old woman presented to your clinic with symptoms of headache and blurry vision.
Take a focused history and address her concerns.
Vitals: BP - 130/88 mm Hg, HR - 86/min, RR - 12/min, Temp - 37.5°C.

Clinical Info: Ms Samantha Ho presented with unilateral, left temporal side pulsating headache, grade 7/10
for the past 2 weeks. She experiences headache while chewing and combing her hair. Associated with blurring
of vision and diplopia. One episode lasts for 30 minutes. Currently on advil prn, atenolol 50 mg OD and
multivitamins.
Clinical Case : Temporal Arteritis

HOPI Past History


• OCD PQRST UVW + AAA • Do you have diabetes or hypertension?
• How did it start? Sudden or gradual. • Any head trauma in the past?
• Location of pain? • Are you on any medications?
• Is it getting worse/better or no changes in • Are you allergic to any medications?
the symptoms? • Any surgeries in the past?
• Since how long have you noticed the pain? • Past h/o recurrent infections?
• Number of episodes of headache till now?
• For how long the pain lasts? Family and Social History
• How does the pain feel like? • Do you smoke? Duration & frequency.
• Does the pain radiate or shoot anywhere? • Do you consume alcohol? Duration &
• Severity of pain on a scale of 1-10. frequency.
• Any provoking factors like food, alcohol, • Do you use recreational drugs? TRAPPED.
caffeine, chocolate, stress, combing hair. • Any family history of cancers/ medical
• How does the pain decrease? illnesses?
• Do your symptoms change with time?
• H/o fever/cough/sore throat/nasal stuffiness?
• Any excessive lacrimation?
• Any visual changes with headache?
• Any neck pain/rigidity/seizures?
• Any weakness/muscle pain/paresthesias?
• Any mood changes/decreased sleep/energy?

Differential Diagnosis Management


• Temporal arteritis. • High dose corticosteroids.
• Migraine. • Tab Prednisone 60 mg PO OD until
• TIA. symptoms subside and ESR is normal, then
40 mg PO OD for 4-6 weeks. Then taper to
Investigations 5-10 mg PO OD for 2 years.
• CBC, ESR, CRP, LFTs. • Relapses occur in 50% if treatment is
• Temporal artery biopsy. stopped before 2 years.
• Visual acuity. • Monitor ESR regularly.
• Fundoscopy. • If visual symptoms present, then admit &
start Inj Prednisolone 1000 mg IV q12h for
5 days.
Clinical Cases - Medicine 117

Jason Scott, a 30 years old man presented with yellowish discoloration of eyes and skin for the past 1 week.
Take a focused history and perform focused physical examination.
Vitals: BP - 120/88 mm Hg, HR - 96/min, RR - 12/min, Temp – 38.0°C.

Clinical Info: Mr Jason Scott presented with yellowish discoloration of eyes and skin for the past 1 week. It
has progressed gradually. He has right upper quadrant abdominal pain. He has loss of appetite, malaise,
nausea and vomiting. His urine is high colored and stool is pale colored. He has low grade fever. He has few
tattoos on his body along with body piercing. He is a chronic alcoholic, smoker and IV drug user for the past
10 years.
Clinical Case : Viral Hepatitis

HOPI Past History


• OCD PQRST UVW + AAA • Do you have medical illnesses?
• How did it start? Sudden or gradual. • Are you on any medications?
• Is it getting worse/better or no changes in • Are you allergic to any medications?
the symptoms? • Any surgeries/dental procedures in the past?
• Duration of yellowish discoloration? • Past h/o recurrent infections?
• Any fever/chills/night sweats?
• Any abdominal pain/location/duration? Family and Social History
• Any nausea/vomiting? • Do you smoke? Duration & frequency.
• Any pruritus/rash? • Do you consume alcohol? Duration &
• Any bruises/spontaneous bleeding? frequency.
• Any loss of appetite/weight? • Do use recreational drugs? TRAPPED.
• Any bowel complaints/color of stools? • Any tattoos/body piercing?
• Any urinary complaints/color of urine? • Are you in a sexual relationship?
• Any fatigue/malaise? • Do you practice safe sex?
• Any confusion/irritability/seizures? • Are you at risk of sexually transmitted
• Any aggravating factors? infections?
• Any relieving factors? • Any family history of medical illnesses?
• Any contact with sick person? • Immunization history.
• Any recent travel?

Differential Diagnosis Management


• Acute Viral Hepatitis. • Symptomatic treatment.
• Alcoholic hepatitis. • Treat the primary cause.
• Bacterial hepatitis. • Educate about safer sex practices.
• Notify public health in case of HAV,
Investigations HbsAg,HCV or HIV positive serology.
• CBC, electrolytes, glucose.
• LFTs, RFTs.
• INR, PT, PTT.
• HAV, HBsAg, HCV, HIV serology.
• Ultrasound abdomen.
118 NAC OSCE | A Comprehensive Review

Anna Levy, a 32 years old lady presented with lower abdominal pain and vaginal spotting for 2 days.. Take a
focused history.
Vitals: BP - 120/80 mm Hg. HR - 90/min. RR - 12/min. Temp - 37.5°C

Clinical Info: Ms Anna Levy presented with h/o lower abdominal pain and vaginal spotting for 2 days. LMP:
6 weeks ago. Bi-manual exam has cervical motion tenderness with open os and bleeding +++.
Clinical Case : Abortion

HOPI Obstetrical History


• OCD PQRST UVW + AAA • Do you have children? If yes, then ask for
• How did it start? Sudden or gradual. • Gravidity, Term/Premature deliveries,
• Is it getting worse/better or no changes in Abortions,
the symptoms? • Live/Multiple births, complications in
• Since how long have you noticed the pain? pregnancy.
• Where is the pain located? • H/o ectopic pregnancy?
• Sharp or dull pain?
• Does the pain radiate or shoot anywhere? Past History
• Severity of pain on a scale of 1-10. • Do you have diabetes or hypertension?
• Type & amount of vaginal bleeding? • Are you on any medications?
• Number of pads changed? • Are you allergic to any medications?
• Passage of clots per vagina? • Any surgeries in the past?
• Do your symptoms change with time? • Past h/o sexually transmitted infections?
• H/o fever/nausea/vomiting?
• Any changes in bowel & urinary habits? Family and Social History
• Date of last menstrual period? • Do you smoke or consume alcohol?
• Last intercourse? • Do use recreational drugs?
• Blood group? • Currently in a relationship? How long?
• Sexual orientation?
Gynecological History • Are you at risk of abuse?
• Age of onset of menses • Any family history of cancers?
• Regular/irregular?
• Duration of menses?
• Amount of bleeding/passage of clots?
• Any dysmenorrhea?

Differential Diagnosis Management


• Spontaneous abortion. • Admit.
• Incomplete abortion. • Dilatation & curettage in case of incomplete
• Ectopic pregnancy. abortion.
• Implantation bleed. • IVF for hemodynamic stability.
• Postcoital trauma • Serial Beta HCG till levels<1.

Investigations
• CBC, electrolytes, renal function tests.
• Beta HCG
• Pelvic ultrasound.
• Blood group & type.
Clinical Cases – Obstetrics & Gynecology 119

Rachel Owens, a 42 years old primigravida who is 9 weeks pregnant. She came to your office to know about
her genetic risks. Take a focused history and address her concerns.

Clinical Info: Ms Rachel Owens conceived naturally and this is her first pregnancy. She is sure of her dates.
She didn't have any antenatal visit yet. This is her first visit. Her home pregnancy test was positive twice. No
family history of genetic disorders. She is only taking prenatal vitamins. No h/o medical illnesses.
She does not smoke or consume alcohol.
Clinical Case : Antenatal Visit

HOPI Past History


• Congratulate the patient. • Do you have diabetes or hypertension?
• When was your period? • Are you on any medications?
• Conceived naturally or with treatment? • Are you allergic to any medications?
• Any antenatal visits till date? • Any surgeries in the past?
• Any blood tests and ultrasound done? • Past h/o sexually transmitted infections?
• Any lower abdominal pain? • When was your last PAP test & results?
• Any bleeding per vagina?
• Any radiation exposure till now? Family and Social History
• Any medication taken in the past 2 months? • Do you smoke or consume alcohol?
• Any fever/nausea/vomiting? • Do use recreational drugs?
• Any vaginal discharge? • Currently in a relationship? How long?
• Intake of folic acid prior to conception? • Are you at risk of abuse?
• Blood group? • Any family history of cancers?
• Any family history of genetic disorders?
Gynecological History • Any h/o genetic disorders in the partners
• Age of onset of menses family?
• Regular/irregular? • Support system at home?
• Duration of menses? • Any stressors at work or home?
• Amount of bleeding/passage of clots?
• Any dysmenorrhea?

Obstetrical History
• Do you have children? If yes, then ask for
• Gravidity, Term/Premature deliveries,
Abortions,
• Live/Multiple births, complications in
pregnancy.
• H/o ectopic pregnancy?

Investigations Management
• CBC • Give antenatal brochures.
• Urine culture/sensitivity, microscopy. • Discuss about genetic screening &
• Beta HCG Counseling.
• ABO Rh, type. • Referral to an obstetrician.
• Blood sugar,TSH. • Nutrition & exercise in pregnancy.
• Measles,Mumps,Rubella,Varicella,VDRL. • Avoid alcohol/smoking/teratogenic
• HIV,HBsAg,HCV. medications.
• Pelvic ultrasound. • Discuss about risk of Down's/Turner's &
other genetic disorders in elderly
primigravida.
120 NAC OSCE | A Comprehensive Review

Lisa Raymond, a 28 years old lady presented to the ER with lower abdominal pain on the left side for the
past 12 hours.. Take a focused history and perform a focused examination (Page 63).
Vitals: BP - 100/70 mm Hg. HR - 98/min. RR - 16/min. Temp - 37.5°C

Clinical Info: Ms Lisa Ray, presented with h/o left side lower abdominal pain for 12 hours with mild
spotting. LMP: 2 months ago. Bi-manual exam has cervical motion tenderness & left adnexal fullness.
Clinical Case : Ectopic Pregnancy

HOPI Obstetrical History


• OCD PQRST UVW + AAA • Do you have children? If yes, then ask for
• How did it start? Sudden or gradual. • Gravidity, Term/Premature deliveries,
• Is it getting worse/better or no changes in Abortions,
the symptoms? • Live/Multiple births, complications in
• Since how long have you noticed the pain? pregnancy.
• Where is the pain located? • H/o ectopic pregnancy?
• Sharp or dull pain?
• Does the pain radiate or shoot anywhere? Past History
• Severity of pain on a scale of 1-10. • Do you have diabetes or hypertension?
• Type & amount of bleeding per vagina? • Are you on any medications?
• Do your symptoms change with time? • Are you allergic to any medications?
• H/o fever/nausea/vomiting? • Any surgeries in the past?
• Any changes in bowel & urinary habits? • Past h/o sexually transmitted infections?
• Date of last menstrual period? • When was your last PAP test & results?
• Any trauma recently?
• When was your last meal? Family and Social History
• Blood group. • Do you smoke or consume alcohol?
• Do use recreational drugs?
Gynecological History • Currently in a relationship? How long?
• Age of onset of menses • Sexual orientation?
• Regular/irregular? • Are you at risk of abuse?
• Duration of menses? • Any family history of cancers?
• Amount of bleeding/passage of clots?
• Any dysmenorrhea?

Differential Diagnosis Management


• Ectopic pregnancy. • Admit
• Threatened abortion. • IVF
• Pelvic Inflammatory Disease. • Urgent gynecology consult
• Ovarian torsion. • Rhogam if needed.
• Endometriosis. Medical treatment:
• Inj Methotrexate 50 mg/m2 given IM as stat
Investigations dose.
• CBC. • Serial Beta HCG till levels drop to <1.
• Blood group, type and cross match. • Contraception advice till completion of
• Beta HCG treatment.
• Electrolytes, Renal function tests. • Surgical treatment : Laparoscopy/
• Pelvic ultrasound. Laparotomy.
Clinical Cases – Obstetrics & Gynecology 121

Cathy Davies, a 32 years old lady presented with inability to conceive for the past 3 years. Take a focused
history and address her concerns.
Vitals: BP - 120/80 mm Hg. HR - 88/min. RR - 12/min. Temp - 37.0°C

Clinical Info: Ms Cathy Davies has been unable to conceive for the past 3 years with unprotected intercourse.
She has not taken any treatment so far. Her periods are irregular with prolonged intervals. She has weight
gain for past 2 years and hirsutism. She is in a monogamous relationship. No other stressors. She has a 5 years
old daughter conceived naturally. No other medical illnesses.
Clinical Case : Infertility

HOPI Gynecological History


• Since how long trying to conceive? • Last menstrual period?
• Frequency of sexual intercourse? • Age of onset of menses
• Awareness of ovulation period? • Regular/irregular?
• Any difficulty /pain during/after intercourse? • Duration of menses?
• Any bleeding after intercourse? • Amount of bleeding/passage of clots?
• Any weight gain recently? • Any dysmenorrhea?
• Any excessive hair growth?
• Any vaginal discharge/type/duration? Obstetrical
• Any abdominal pain/back pain? • Do you have children? If yes, then ask for
• Any swelling/masses noticed in the body? • Gravidity, Term/Premature deliveries,
• Any heat/cold intolerance? Abortions,
• Any nipple discharge/type/duration? • Live/Multiple births, complications in
• H/o repetitive infections? pregnancy.
• H/o fever/nausea/vomiting?
Past History
Male Partner History: • Do you have diabetes/hypertension/thyroid
• Any difficulty during intercourse? diseases?
• H/o premature ejaculation? • Are you on any medications?
• H/o difficulty maintaining ejaculation? • Are you allergic to any medications?
• Any abdominal surgeries? • Any surgeries in the past?
• Any trauma to the pelvic area in the past? • Past h/o sexually transmitted infections?
• Any medical illnesses? • When was your last PAP test & results?
• H/o mumps in childhood?
• Are you on any medications/allergies? Family and Social History
• Do you smoke or consume alcohol? • Do you smoke or consume alcohol?
• Do use recreational drugs? • Do use recreational drugs?
• H/o sexually transmitted infections? • Are you at risk of abuse?
• Any stressors at home or work? • Any family history of cancers?
• H/o of conception in past relationships?

Diagnosis • Hystero-salphingogram.
• Secondary infertility. • Laparoscopy.
• Semen analysis
Investigations
• CBC,FBS,TSH. Management
• Day 3 FSH,LH,PRL±DHEAS,Free • Complete physical examination of both the
testosterone. partners.
• Day 21-23 Progesterone. • Treat the cause.
• Basal body temperature monitoring. • Supportive counseling.
• Pelvic ultrasound • Timing of the intercourse in relation to
ovulation.
• Referral to infertility specialist.
122 NAC OSCE | A Comprehensive Review

Alyssa Jones, a 18 years old girl came to your office requesting for contraceptive pills. Take a focused history
and address her concerns.
Vitals: BP - 120/80 mm Hg. HR - 80/min. RR - 12/min. Temp - 37.0°C

Clinical Info: Ms Alyssa Jones is a 18 years old student with no significant history of medical illnesses. She is
an active sexual relationship for the past 4 months. Had one episode of STI 6 months ago. LMP was 1 week
ago. No family history of cancers. Currently using barrier contraception.
Clinical Case : OCP Counseling

HOPI Gynecological History


• When was the last menstrual period? • Age of onset of menses
• Do you get headaches often? • Regular/irregular?
• Do you have any bleeding disorders? • Duration of menses?
• Any thromboembolic events? • Amount of bleeding/passage of clots?
• Any liver disease? • Any dysmenorrhea?
• Any uncontrolled high blood pressure? • Current contraception use?
• Any heart disease?
• Any mass in the breast? Obstetrical History
• Any abnormal vaginal bleeding? • Have you been pregnant before?
• Any history of sexually transmitted If yes, then ask for Gravidity,
infections? Term/Premature deliveries, Abortions,
• Any abnormal PAP results? Live/Multiple births, complications in
• When was the last PAP done? pregnancy.
• Sexual orientation? • H/o ectopic pregnancy?
• Currently in a relationship? How long?
• Number of sexual partners? Past History
• Do practice safe sex? • Any hospitalizations?
• Do you smoke or consume alcohol? • Any surgeries in the past?
• Do use recreational drugs?
• Are you on any medications? Family History
• Are you allergic to any medications? • Family history of hypertension/diabetes?
• Family history of breast/ovarian/endometrial
cancers?
• Family history of bleeding disorders?

Investigations Management
• PA test & complete physical. • Tab Yasmin one tab OD for 28 days.
• Vaginal & Cervical swabs, culture/sensitivity. • Typical start (start at first Sunday after
Menses)
Benefits of OCP • Begin pill on first Sunday after onset of
• Prevention of unwanted pregnancy. Menses
• Reduced blood loss. • If Menses start on Sunday, then start pill
• Decreased dysmenorrhea. Day 1
• Cycle regularization. • Use barrier Contraception for Days 1-7
• Decreased risk of breast/ovarian/endometrial If pill started after Day 5:
cancers. • OCP may not suppress Ovulation for first
• Decreased acne. cycle
• Decreased osteoporosis. • Use barrier Contraception for first month.
• Decreased PMS symptoms. • Follow up 6 weeks after the start of the pill.
• Reversible contraception.
Clinical Cases – Obstetrics & Gynecology 123

Maria Santosa, a 28 years old lady presented with lower abdominal pain, dyspareunia and vaginal discharge
for 1 week. Take a focused history and perform focused examination (Page 63).
Vitals: BP - 120/80 mm Hg. HR - 90/min. RR - 12/min. Temp - 38.5°C

Clinical Info: Ms Maria Santosa presented with h/o lower abdominal pain for 1 week with dyspareunia and
foul smelling vaginal discharge. She has mild fever for 2 days. H/o unprotected intercourse +. H/o of past
infection 3 months ago. LMP: 1 week ago. Bi-manual exam has cervical motion tenderness & right adnexal
fullness.
Clinical Case : Pelvic Inflammatory Disease

HOPI Obstetrical History


• OCD PQRST UVW + AAA • Do you have children? If yes, then ask for
• How did it start? Sudden or gradual. • Gravidity, Term/Premature deliveries,
• Is it getting worse/better or no changes in Abortions,
the symptoms? • Live/Multiple births, complications in
• Since how long have you noticed the pain? pregnancy.
• Where is the pain located? • H/o ectopic pregnancy?
• Sharp or dull pain?
• Does the pain radiate or shoot anywhere? Past History
• Severity of pain on a scale of 1-10. • Do you have diabetes or hypertension?
• H/o painful intercourse? • Did you suffer from any diseases in the past
• Type & amount of vaginal discharge? like tuberculosis?
• Do your symptoms change with time? • Are you on any medications?
• H/o fever/cough/rash/vomiting? • Are you allergic to any medications?
• Any chills/night sweats? • Any surgeries in the past?
• Any changes in bowel & urinary habits? • Past h/o sexually transmitted infections?
• Any spotting per vagina? • When was your last PAP test & results?
• Date of last menstrual period?
• Do you practice safe sex? Family and Social History
• Number of sexual partners? • Do you smoke or consume alcohol?
• Do use recreational drugs?
Gynecological History • Currently in a relationship? How long?
• Age of onset of menses • Sexual orientation?
• Regular/irregular? • Are you at risk of abuse?
• Duration of menses? • Any family history of cancers?
• Amount of bleeding/passage of clots?
• Any dysmenorrhea? Intra uterine devices?

Differential Diagnosis Management


• Pelvic Inflammatory Disease caused due to • Inj Ceftriaxone 250 mg IM stat dose with
sexually transmitted infections, IUDs. • Tab Doxycycline 100 mg PO bid x 14 days.
• Ovarian cyst. • Reportable disease.
• Endometriosis. • Treat partners.
• Ovarian torsion. • Avoid intercourse till completion of
• Acute appendicitis. treatment.
• Practice safe sex.
Investigations • Retest with cervical swabs after 4-6 weeks.
• CBC
• Vaginal & Cervical swabs, culture/sensitivity.
• Urine culture/sensitivity.
• Beta HCG
• Pelvic ultrasound.
124 NAC OSCE | A Comprehensive Review

Julia Marshall, a 30 years old lady presented to the ER with bright red vaginal bleeding for the past 1 hour.
She is 36 weeks pregnant. Take a focused history address her concerns.
Vitals: BP - 100/70 mm Hg. HR - 100/min. RR - 14/min. Temp - 37.5°C

Clinical Info: Ms Julia Marshall is G2 T1 P0 A0 L1 at 36 weeks gestation. She has painless vaginal bleeding
for the past 1 hour. Has no contractions. Fetal heart rate is 130/minute. She has a previous history of cesarean
section.
Clinical Case : Placenta Previa

HOPI Gynecological History


• OCD PQRST UVW + AAA • Regular/irregular menses?
• How did it start? Sudden or gradual.
• Is it getting worse/better or no changes in Obstetrical History
the symptoms? • Do you have children? If yes, then ask for
• Since how long have you noticed the Gravidity, Term/Premature deliveries,
bleeding? Abortions, Live/Multiple births,
• Amount of bleeding? complications in pregnancy.
• Color of bleeding? • H/o ectopic pregnancy?
• Number of pads changed? • H/o placenta previa/ abruptio placentae?
• Any passage of clots? • H/o of cesarean section and the reason?
• Any leaking noticed per vagina? • Any surgeries on the uterus?
• Any trauma recently?
• Are fetal movements felt? Past History
• Any abdominal pain? • Do you have diabetes or hypertension?
• Any fever/nausea/vomiting? • Are you on any medications?
• Any changes in bowel & urinary habits? • Are you allergic to any medications?
• When was the last intercourse?
• Date of last menstrual period? Family and Social History
• Any complications in the antenatal period? • Do you smoke or consume alcohol?
• When was the last antenatal visit? • Do use recreational drugs?
• Any bleeding episode during pregnancy? • Are you at risk of abuse?
• Blood group? • Any family history of medical illnesses?

Differential Diagnosis Management


• Placenta previa. • Admit in the hospital.
• Vasa previa. • Left lateral decubitus position.
• Abruptio placentae. • Electronic fetal heart monitoring.
• Bloody show. • IVF.
• Maternal vital monitoring.
Investigations • Inj Rhogam, if needed.
• CBC, electrolytes, renal function tests. • GA < 37 weeks - mild bleeding - admit &
• Fetal ultrasound. observation.
• Blood group, type and cross match. • GA > 36 weeks - profuse bleeding with feto-
maternal compromise - Urgent Cesarean
section.
Clinical Cases – Obstetrics & Gynecology 125

Elaine Abraham, a 32 years old lady primigravida, at 34 weeks gestation presented to the ER with headache,
abdominal pain and blurring of vision. Take a focused history and address her concerns.
Vitals: BP - 150/100 mm Hg. HR - 90/min. RR - 14/min. Temp - 37.0°C. FHR = 148/min.

Clinical Info: Ms Elaine Abraham has a history of pregnancy induced hypertension since 28 weeks. Her BP
is controlled by dietary restrictions and low salt intake. She has epigastric pain, blurring and headache for the
past 4-6 hours. She has facial and ankle edema ++. There are no contractions. Fetal movements are felt. No
bleeding. Urine dipstick is positive for proteinuria.
Clinical Case : Pre Eclampsia

HOPI Gynecological History


• OCD PQRST UVW + AAA • Regular/irregular?
• How did it start? Sudden or gradual.
• Is it getting worse/better or no changes in Obstetrical History
the symptoms? • Do you have children?
• Since how long have you noticed the pain? • If yes, then ask for Gravidity,
• Where is the pain located? Term/Premature deliveries, Abortions,
• Sharp or dull pain? Live/Multiple births, complications in
• Does the pain radiate or shoot anywhere? pregnancy.
• Severity of pain on a scale of 1-10. • H/o ectopic pregnancy?
• Onset of headache? • Last antenatal visit?
• Location of headache?
• Type of headache? Past History
• Onset of blurring of vision? • Do you have diabetes or hypertension?
• Do your symptoms change with time? • Are you on any medications?
• H/o fever/cough/rash/vomiting? • Are you allergic to any medications?
• Any changes in bowel & urinary habits? • Any surgeries in the past?
• Any bleeding per vagina?
• Any swelling of the body? Family and Social History
• Fetal movements? • Do you smoke or consume alcohol?
• Any contractions/ leaking per vagina? • Do use recreational drugs?
• Total weight gain in the pregnancy? • Are you at risk of abuse?
• Any antenatal complications like high blood • Any family history of hypertension/diabetes?
pressure/diabetes/ seizures?

Diagnosis Management
• Gestational hypertension with Pre- • Admit in the hospital.
eclampsia. • Electronic Fetal monitoring.
• Bed rest in left lateral decubitus position.
Investigations • Hourly maternal vital signs with
• CBC, electrolytes, renal function tests. intake/output charting.
• Urinalysis, 24 hour urinary protein, liver • Inj Magnesium sulphate 4 mg IV bolus over
function tests, uric acid, LDH, albumin. 20 min,then 2-4g/h for maintenance.
• INR, PTT, Fibrinogen. • Monitor signs for magnesium toxicity.
• Non stress test, Bio-physical profile. • Inj Labetolol 20-50 mg IV q10minutes till
• Fetal ultrasound. BP< 140/90 mmHg.
• Deliver the baby.
RISK FACTORS FOR PIH:
Maternal: Primigravida or new paternity, Family hx of Preeclampsia, Diabetes Mellitus, Obesity, Maternal age >40 years,
Preexisting Hypertension, Anti-Phospholipid Antibody syndrome.
Fetal: IUGR, Oligohydraminos, GTN. Hydrops, Multiple pregnancy.
126 NAC OSCE | A Comprehensive Review

Michael Walter a 18 months old boy brought to your office by his mother regarding poor weight gain. Take
history from the mother & address his concerns.

Clinical Info: Michael's mother is concerned for poor weight gain for his age & height. He has no
fever/nausea/vomiting/cough. No h/o recurrent infections. No urinary or bowel complaints. He is picky eater
who gets distracted wile eating food. His diet consists of excessive juice & milk. No family stress present.
Clinical Case Diagnosis: Failure to thrive due to inadequate dietary intake.

HOPI • Maternal malnutrition.


• OCD PQRST UVW AAA • Maternal exhaustion or Major Depression.
• Duration of poor weight gain? • Any stress at home?
• Sudden or gradual decline in weight? • Any signs of physical/psychological/family
• Quality and Quantity of food? abuse?
• Who feeds the child? • Family history of short stature/FTT in
• Does the child feed self(e.g. spoon, cup)? siblings.
• Psychosocial events around feeding time. • Any developmental delay?
• Is the child distracted or not supervised?
• Are there food battles or food refusal? Birth history
• Discuss food preparation (e.g. formula too • Gestational age at birth and birth weight.
dilute). • Mode of delivery: cesarean, induction,
• Beverages (e.g. Milk, juice, water, soda). forceps or vacuum delivery.
• Stool habits (e.g. frequency and consistency). • Any fetal distress?Was meconium passed in
• Pica history. utero?
• Detailed nursing or breast feeding history. • APGAR score at birth, 1 minute & 5
• Infrequent brief feedings. minute?
• Current weight & height? • Was resuscitation required?
• Highest weight? • When was breast feeding started?
• Any fever/nausea/vomiting/cough? • Color of 1st stool, when was 1st stool
• Any diarrhea/constipation? passed?
• Any urinary complaints? • Color of urine, when was 1st urine passed?
• H/o recurrent infections? • Any antenatal/post partum complications.
• Maternal ingestion of alcohol/diuretics. • Immunization history.
• Inadequate milk supply. • Developmental milestones.
• Inadequate milk let down. • Detailed dietary history.

Differential Diagnosis • Karyotype.


• Physiologic cause-Familial short stature. • Wrist X ray.
• Organic cause-Cleft palate, Choanal atresia,
GERD, Celiac ds, Giardiasis, Protein losing Management
enteropathy, Milk protein allergy, Liver ds. • Complete physical examination.
• Chronic diarrhea. • Reassure parents.
• Hyperthyroidism, Immunodeficiency. • Height & weight measurement.
• Prenatal causes-Intrauterine infection, • Head circumference.
maternal malnutrition,Fetal alcohol • Assess feeding process & parent-child
syndrome, interaction.
• Chromosomal disorders. • Determine & treat underlying etiology.
• Institute nutritional therapy.
Investigations • Eat in a comfortable, stress free environment
• CBC,ESR, electrolytes, RFTs, LFTs. with positive reinforcement.
• Blood sugar,TSH,Se Ferritin,Sweat chloride. • Consume food from all four food groups.
• Urinalysis-routine microscopy, C/S. • Maintain dietary intake diary.
• Stool for fat content,ova & parasites.
• Se Calcium, Phosphate, Albumin.
Clinical Cases - Pediatrics 127

Benjamin Smith a 15 months old boy has been brought to the ER with fever and 2 episodes of seizures. He is
stabilized now. Take history & address the concerns of an over anxious mother.

Clinical Info: Benjamin Smith was having a runny nose and high grade fever for the past 3 days. His fever did
not subside with Tylenol. He had 1st episode of tonic-clonic seizure 6 hours ago at home. This was the first
occurrence. He had no other symptoms. No family history of seizures. No complications during birth or
development so far. Immunization is up to date.
No signs of child abuse.
Diagnosis: Febrile seizures.

HOPI • History of problems during the pregnancy


• OCD PQRST UVW AAA and birth.
• Describe seizure duration? • Developmental history.
• what body parts affected and in what order, • Child's medical history.
premonitory signs? • Immunization history.
• Post-ictal state (decrease in level of • Family history of seizure disorder.
consciousness, headache, weakness). • Screen for signs of child abuse.
• Previous seizure?
• Events during the seizure time?
• How did the seizure stop?
• Onset of fever? Sudden or gradual?
• Duration of fever?
• Type of fever-continuous, remittent,
intermittent.
• Any nausea/vomiting?
• Any ear/eye discharge/runny nose?
• Any rash?
• Any cough/sore throat/difficulty swallowing?
• Any difficulty breathing?
• Any bowel or urinary complaints?
• Any sick contacts?
• Ask about preceding trauma or illness or
medications taken?

Differential Diagnosis Management


• Febrile seizure. • Symptomatic treatment.
• Meningitis. • Antipyretics for fever prn.
• Encephalitis. • Maintain hydration.
• Counseling & reassurance for parents.
Investigations • Recurrence - rectal or sublingual Lorazepam.
• CBC, electrolytes, RFTs. • Treat underlying cause of fever.
• ABG, Blood glucose.
• Urinalysis.
• Blood culture & sensitivity.
• LP-gram stain, culture & sensitivity.
128 NAC OSCE | A Comprehensive Review

Nick Chang is a 15 years old boy brought by his mother with fever and rash for the past 2 days.
Take history & address her concerns.

Clinical Info: Nick has high fever for the past 2 days. He developed a diffuse rash in the last 24 hours which
is spreading from head to trunk. He also has cough, sore throat and redness of eyes. He has no altered level of
consciousness/irritability. He is alert and feeding well. Has h/o sick contacts with similar complaints in the
day care. His immunization is up to date.
Diagnosis: Measles.

HOPI • Immunization history till date?


• OCD PQRST UVW AAA • Any recurrent infections?
• Onset of fever- sudden or gradual? • Development milestones for age?
• Duration of fever? • Any similar symptoms in the past?
• Type of fever-
continuous,remittent,intermittent?
• Highest recorded temperature?
• Relieving factors for the fever?
• Onset of rash?
• Type of rash?
• Location of rash?
• Rash becoming better or worse?
• Any vesicles noticed with the rash?
• Any swelling in the body?
• Any ear/eye discharge?
• Any excessive crying/irritability?
• Any changes in alertness of the child?
• Any cough/sore throat/runny nose?
• Any nausea/vomiting/difficulty swallowing?
• Any changes in urinary/bowel habits?
• Feeding well or not?
• Any seizures?
• Any recent sick contacts?
• Any travel?

Differential Diagnosis Management


• Measles. • Symptomatic treatment.
• Rubella. • Maintain adequate hydration.
• Varicella zoster. • Rest.
• Erythema infectiosum. • Antipyretics for fever prn.
• Educate parents about complications.
Investigations
• CBC with differential.
Clinical Cases - Pediatrics 129

Marie Jones delivered a baby Anthony 36 hours old and now the newborn has jaundice, lethargy and crying.
The serum bilirubin is 220 mmol ( N < 200). Take history & address her concerns.

Clinical Info: Anthony was born to a primigravida by normal vaginal delivery. Mother noticed yellowish
discoloration of his eyes in the morning. She had no antenatal complications. She had premature rupture of
membranes prior to onset of labor at 38 weeks. She was put on antibiotics. Her labor was 18 hours long. The
labor was induced. Apgar was 9/10. Baby is a little lethargic and not feeding well. Has no fever/altered
consciousness. No seizures.
Clinical Case Diagnosis: Neonatal Jaundice due to Sepsis.

HOPI • APGAR score at birth, 1 minute & 5


Mother's obstetrical history: minute?
• GTPAL • Was resuscitation required?
• H/o neonatal jaundice in past pregnancies. • When was breast feeding started?
• Maternal medical history esp. liver disease. • Is the baby feeding well?
• Illness during pregnancy esp. diabetes, • Color of 1st stool, when was 1st stool
rubella, toxoplasmosis, herpes, CMV. passed?
• Teratogenic medications during pregnancy. • Color of urine, when was 1st urine passed?
• Radiation exposure in pregnancy? • Any vomiting/regurgitation?
• Drug and alcohol use during pregnancy? • Decreased neonatal muscle tone?
• Any pets in the house? • Any fever, irritability, lethargy,seizure?
• Maternal & Paternal blood type. •
• Complications of present pregnancy. • Past History
- Gestational hypertension or diabetes, • Do you have diabetes or hypertension?
hyper/hypothyroid, hypercoagulation. • Are you on any medications?
• Any antenatal/post partum complications? • Are you allergic to any medications?
• Any surgeries in the past?
Newborn history: • Past h/o recurrent infections?
• Gestational age at birth, •
• Mode of delivery: cesarean, induction, • Family and Social History
forceps or vacuum delivery. • Social support.
• Duration of rupture of membranes (ROM)? • Any family history of medical illnesses?
• Was ROM artificial or prolonged?
• Any fetal distress?Was meconium passed in
utero?

Differential Diagnosis Management


• Sepsis. • Prevent kernicterus.
• Breast feeding jaundice. • Treat underlying cause.
• Hemolysis. • Monitor neonatal vitals.
• Physiologic jaundice. • Maintain hydration.
• Initiate phototherapy.
Investigations • Reassurance for parents.
• CBC with retics,electrolytes, RFTs.
• Se bilirubin(conjugated & unconjugated) &
albumin.
• Blood group (maternal, paternal &
neonatal).
• Coomb's test.
• Blood & urine culture.
• TSH, G6PD.
• Chest X Ray, LP.
130 NAC OSCE | A Comprehensive Review

Sean Radcliffe is a 8 years old boy whose parents have concern about bed wetting. Take history from the
father & address his concerns.

Clinical Info: Sean has been wetting his bed since last 3 years. He never had bladder control. He has no
fever/vomiting. No h/o recurrent infections. He wets bed 2-3 times in the night. No day time wetting present.
No encoparesis. Parents have not taken any treatment so far and have tried toilet training in past with no
success. No stresses at home or school.
Clinical Case Diagnosis: Primary nocturnal enuresis.

HOPI • Enuresis in other siblings?


• OCD PQRST UVW AAA • Any stresses at home or school?
• Type of voiding - Involuntary or intentional. • Any new habits or regression to old habits?
• Number of times wets bed in the night? • Trial of toilet training in the past?
• Has the child ever been dry?(primary or • Any treatment in the past for enuresis?
secondary)
• Is there daytime Enuresis?(complicated Birth history
Enuresis) • Gestational age at birth and birth weight.
• Wetting pattern - day±night or night only. • Mode of delivery: cesarean, induction,
• Any dysuria/pyuria/foul smelling urine? forceps or vacuum delivery.
• Involuntary passage of stool in the sleep? • Any fetal distress?Was meconium passed in
• Functional bladder disorder signs like utero?
- Voids >7 per day with urgency & in small • APGAR score at birth, 1 minute & 5
volumes. minute?
- Withholds urine until last minute, wets • Was resuscitation required?
more than once nightly. • When was breast feeding started?
• Has enuresis on only a few nights per week? • Color of 1st stool, when was 1st stool
• Voids large volumes when enuresis occurs? passed?
• Bowel or bladder habit changes recently. • Color of urine, when was 1st urine passed?
• Infrequent or difficult stool passage? • Any antenatal/post partum complications.
• Any changes in appetite/weight? • Immunization history.
• Any fever/nausea/vomiting?
• Any recurrent infections?
• Amount of fluid intake prior to sleep?
• Any neurological disorders?
• Any genitourinary surgeries?

Differential Diagnosis Management


• Primary nocturnal enuresis. • Complete physical examination.
• Urinary tract infection. • Reassure parents.
• Urinary tract anomalies like small bladder. • Schedule voiding times.
• Psychological (death in the family, sexual • Bed wetting alarm.
abuse). • Void before bedtime.
• Limit fluids 1 hour before bedtime.
Investigations • Voiding diary to be maintained.
• CBC,electrolytes, RFTs, LFTs. • Positive reinforcement for dry nights.
• Blood sugar. • Pharmacological therapy like
• Urinalysis-routine microscopy, C/S. Imipramine/DDAVP.
• Ultrasound abdomen.
Clinical Cases - Pediatrics 131

Ally Singer's 6 weeks old baby boy Alex is vomiting for the past 2 days. Take history & address her concerns.

Clinical Info: Alex had 4 episodes of projectile non bilious vomiting in the past 48 hours. He vomits after
feeding. No fever. Looks lethargic & dehydrated but alert. No seizures. Had only one bowel movement in last
24 hours. No sick contacts. O/E: Palpable abdominal mass in the right hypochondrium.
Clinical Case Diagnosis : Pyloric stenosis.

HOPI Mother's obstetrical history


• OCD PQRST UVW AAA • GTPAL
• Number of episodes of vomiting? • Maternal medical history esp. liver disease.
• Duration of vomiting? • Illness during pregnancy esp. diabetes,
• Type of vomiting - projectile/non projectile? rubella, toxoplasmosis, herpes, CMV.
• Color/contents of vomitus? • Teratogenic medications during pregnancy.
• Any excessive crying? • Radiation exposure in pregnancy.
• Feeding pattern in last 48 hours? • Drug and alcohol use during pregnancy.
• Decreased neonatal muscle tone? • Complications of present pregnancy.
• Any fever, irritability, lethargy,seizure? - Gestational hypertension or diabetes,
• Last bowel movement? - hyper/hypothyroid, hypercoagulation.
• Foul smelling urine & color of urine? • Any antenatal/post partum complications?
• Current weight.
• Any sick contacts.

Newborn history
• Gestational age at birth and birth weight.
• Mode of delivery: caesarean, induction,
forceps or vacuum delivery.
• Any fetal distress? Was meconium passed in
utero?
• APGAR score at birth, 1 minute & 5
minute?
• Was resuscitation required?
• When was breast feeding started?
• H/o neonatal jaundice.
• Color of 1st stool, when was 1st stool
passed?
• Color of urine, when was 1st urine passed?

Differential Diagnosis Management


• Pyloric stenosis. • Admit.
• Tracheo-esophageal fistula. • Urgent Pediatric surgery consult.
• Duodenal atresia. • IVF to maintain hydration.
• Malrotation of gut. • Surgery-Pyloromyotomy.
• Gastro-esophageal reflux.

Investigations
• CBC,electrolytes, RFTs, LFTs.
• ABG.
• Urinalysis.
• Ultrasound abdomen.
• Abdominal X ray.
132 NAC OSCE | A Comprehensive Review

John Andrews is a 3 years old boy who is not speaking well. Take history & address his father's concerns.

Clinical Info: John Andrews has h/o recurrent ear infections. He had 3 episodes in the alst 6 months. He has
runny nose and mild cough too. He can speak in sentence of 3-4 words. He can count to 5. But for the past 3
months he is not learning new words or numbers. He responds to loud sounds. No other complaints. Social
interaction is very good. No birth or developmental complications till date.
Diagnosis: Speech delay secondary to recurrent otitis media.

• Duration of speech delay?


• Who noticed it first?
• Any ear discharge/runny nose?
• Any recurrent infections?
• Any fever/cough/sore throat?
• Does the child wake up in response to
sounds?
• Startle to loud sounds?
• Comes when called?
• Understand spoken instructions?
• Ask about swimming.
• Enquire of verbal cues.
• How many languages are spoken in the
household?
• Child's social interaction with others.
• Does the child talk less in particular
situations?
• How many words are spoken by the child?
• Detailed developmental history.
• Was the child screened for hearing at birth?
• Any regression in habits?
• Immunization history till date.
• Family history of speech delay.
• Any complications during pregnancy or
birth?
• Detailed birth history.
• Exposure to toxins during pregnancy?
• Any ototoxic drugs used in infancy?
• Screen for signs of child abuse.

Differential Diagnosis Management


• Hearing loss secondary to Otitis media. • Reassurance for parents.
• Selective mutism. • Complete physical assessment.
• Expressive speech delay. • ENT referral.
• Autism. • Speech therapy.
• Positive reinforcement & encouragement.
Investigations
• Hearing test like Pure Tone Audiometry.
Clinical Cases - Psychiatry 133

Gabriella Anderson, a 18 years old girl came to your office with complaints of gaining weight. Take history &
counsel.

Clinical Info: Ms Gabriella Anderson presented with gaining 5 lbs in last 1 month. She looks underweight
for her age and height. She is exercising 3 times a day. She doesn't binge or induce vomiting. Lately she is
taking small portions of meals due to fear of gaining weight. She has no medical illnesses. No past history of
psychiatric illness. Currently not taking any medications.
Clinical Case : Anorexia

HOPI • Do you take any medications? Ask details.


• When did you notice the change in weight? • Do you consume alcohol?
• Duration of symptoms? Amount/frequency?
• Amount of weight gain? • Do you use recreational drugs? Ask
• Lowest and highest weight you had? TRAPPED.
• Are you afraid of gaining weight?
• How do you try to control your weight? Past Psychiatric History
• How do you think your body looks? • Any similar symptoms in past?
• Does your body weight & shape have an • Any h/o
impact on your self opinion? mania/depression/delusion/delirium?
• Last menstrual • Any contacts with mental health
period/regularity/complications? professionals?
• Any abdominal pain/nausea/vomiting? • Any past problems with law?
• Bowel habits?
• Any skin changes? Family History
• Any intolerance to temperature changes? • Any family history of similar complaints?
• Any recent stressors at home or work? • Any family history of other psychiatric
• Changes in sleep pattern? illnesses?
• Feeling of • Any family history of suicide/alcohol/drug
guilt/hopelessness/helpless/worthless? abuse?
• Changes in mood?
• Any thoughts of harming self/suicide? Social History
• Any thoughts of harming someone else? • Support system at home/work?
• Any plans at the moment? • Current living situation?
• Do you feel persistently cheerful/high? • Relationship history?
• Do you have any medical/surgical illnesses? • Education history?
Ask details. • Any risk of physical/sexual/mental abuse?

Differential Diagnosis Management


• Anorexia nervosa. • Complete physical assessment.
• Bulimia nervosa. • Antidepressants.
• Mood disorders. • Supportive psychotherapy.
• Medical cause of weight loss. • CBT
• Body Dysmorphic disorder. • Make plans for weight gain.
• Community resources for eating disorders.
Investigations • Educational brochures.
• CBC, electrolytes, renal & Liver function • Admit if weight <85% of ideal weight.
tests. HR < 40 bpm.
• TSH, blood glucose, ECG. Hypovolemia.
• Urine toxicology screen. Hypokalemia
• Beta HCG, LH,FSH. Hypoglycemia.
Hypothermia.
134 NAC OSCE | A Comprehensive Review

Amanda Sawyer, a 20 years old girl brought to your office by her mother for vomiting and weight loss. Take
history & counsel.

Clinical Info: Ms Amanda Sawyer presented with vomiting about meals. She has fear of weight gain. H/o
binging & induced vomiting present. H/o laxative abuse and excessive exercise. She has no apparent psycho-
motor or suicidal ideation. She has no medical illnesses. No past history of psychiatric illness. Currently not
taking any medications.
Clinical Case : Bulimia

HOPI • Do you take any medications? Ask details.


• Onset of vomiting? • Do you consume alcohol?
• Duration of symptoms? Amount/frequency?
• Type of vomitus/contents? • Do you use recreational drugs? Ask
• Projectile/non projectile? TRAPPED.
• H/o abdominal pain/site/type?
• Do you force yourself to vomit? Past Psychiatric History
• Do you binge large amount of food? • Any similar symptoms in past?
• Amount of weight gain/loss? • Any h/o
• Lowest and highest weight you had? mania/depression/delusion/delirium?
• How do you try to control your weight? • Any contacts with mental health
• Does your body weight & shape have an professionals?
impact on your self opinion? • Any past problems with law?
• Last menstrual
period/regularity/complications? Family History
• Any skin changes? • Any family history of similar complaints?
• Any intolerance to temperature changes? • Any family history of other psychiatric
• Any recent stressors at home or work? illnesses?
• Changes in sleep pattern? • Any family history of suicide/alcohol/drug
• Feeling of abuse?
guilt/hopelessness/helpless/worthless?
• Changes in mood? Social History
• Any thoughts of harming self/suicide? • Support system at home/work?
• Any thoughts of harming someone else? • Current living situation?
• Do you feel persistently cheerful/high? • Relationship history?
• Do you have any medical/surgical illnesses? • Education history?
Ask details. • Any risk of physical/sexual/mental abuse?

Differential Diagnosis Management


• Bulimia nervosa. • Complete physical assessment.
• Anorexia nervosa. • Antidepressants.
• Mood disorders. • Supportive psychotherapy.
• Medical cause of weight loss. • CBT
• Body Dysmorphic disorder.. • Make plans for weight gain.
• Community resources for eating disorders.
Investigations • Educational brochures.
• CBC, electrolytes, renal & Liver function • Admit if weight <85% of ideal weight.
tests. HR < 40 bpm.
• TSH, blood glucose, ECG. Hypovolemia.
• Urine toxicology screen. Hypokalemia
• Beta HCG, LH,FSH. Hypoglycemia.
Hypothermia.
Clinical Cases - Psychiatry 135

Derek Paul, a 65 years old man admitted in surgical floor presented with strange behavior for the past 4
hours. You are on call surgical resident for the shift. Take history & counsel.

Clinical Info: Mr Derek Paul had partial right hip replacement 3 days ago. His post op recovery till now has
been uneventful. Evening shift nurse noticed significant change in his behavior. He is agitated, restless with
acute memory loss. He is disoriented to time, place & person. He is having delusional thoughts of ants
crawling. He is on oral antibiotics, antihypertensives, blood thinners. He is chronic alcohol abuser.
Clinical Case : Delirium

HOPI Past Psychiatric History


• When did you notice the change in • Any similar symptoms in past?
behavior? • Any h/o
• Duration of symptoms? mania/depression/delusion/delirium?
• Symptoms have worsened or improved? • Any contacts with mental health
• Do you feel persistently cheerful/high? professionals?
• Do you sense things that others around you • Any past problems with law?
don't?
• Do you hear any voices? Family History
• Do you feel restless/agitated? • Any family history of similar complaints?
• Any changes in memory? • Any family history of other psychiatric
• Orientation to time/place/person? illnesses?
• Any fever/nausea/vomiting? • Any family history of suicide/alcohol/drug
• Any chest pain/ abdominal pain? abuse?
• Any changes in bowel/urinary habits?
• Last meal? Social History
• Any changes in mood? • Support system at home/work?
• Any thoughts of harming self/suicide? • Current living situation?
• Any thoughts of harming someone else? • Relationship history?
• Any plans at the moment? • Education history?
• Do you have any medical/surgical illnesses? • Any risk of physical/sexual/mental abuse?
Ask details.
• Do you take any medications? Ask details. T : Treatment history
• List of current medications. R : Route
• Do you consume alcohol? A : Addiction / toxicity / Amount
Amount/frequency? P : Pattern of use
• Do you smoke? Duration/frequency? P : Prior abstinence
• Do you use recreational drugs? Ask E : Effects of drug
TRAPPED. D: Duration of use

Differential Diagnosis Management


• Acute Delirium. • Complete physical examination.
• Alcohol withdrawal. • Re-evaluate current medications.
• Medication induced delirium. • Maintain hydration.
• Sepsis. • Avoid sedatives,narcotics,BZD.
• Place in a quiet, well lit room.
Investigations • Vital q1hourly.
• CBC, electrolytes, renal function tests. • Inj Haloperidol 5 mg IM stat with Inj
• TSH, blood glucose, Blood gases Lorazepam 2mg IM stat.
• Urinalysis with toxicology screen. • Reassurance.
• Blood alcohol levels.
• MSE/MMSE.
136 NAC OSCE | A Comprehensive Review

Claire Wiggins, a 72 years old lady brought to your office by her son with strange behavior. Take history from
the patient and address her concerns.

Clinical Info: Ms Claire Wiggins is forgetting things and daily tasks for past 1 year. Her symptoms have
become worse for the last 6 months. Recently she forgot her way back home. She lives alone. Son has noticed
changes in her dressing and poor hygiene. She has no apparent psycho-motor or suicidal ideation. She has
hypertension. No past history of psychiatric illness. Currently on oral antihypertensives, statins, zoloft,
multivitamins.
Clinical Case : Dementia

HOPI • Do you smoke? Duration/frequency?


• When did you notice the change in • Do you use recreational drugs? Ask
behavior? TRAPPED.
• Duration of symptoms?
• Have you found yourself forgetting things? Past Psychiatric History
(establish onset, duration, degree) • Any similar symptoms in past?
• Do you ever get confused or disorientated? • Any h/o mania/ depression/ delusion/
• Do you have trouble understanding what delirium?
people say to you? • Any contacts with mental health
• Do you have trouble finding the right words professionals?
to say? • Any past problems with law?
• Symptoms getting gradually worse over
months? Family History
• Have you seen, heard or felt anything that • Any family history of similar complaints?
other people told you didn’t exist? • Any family history of other psychiatric
• Have you noticed a change in your sense of illnesses?
smell? • Any family history of suicide/alcohol/drug
• Have you had any incontinence? abuse?
• Any changes in gait?
• Any mood changes? Social History
• Any fever/nausea/vomiting/abdominal pain? • Support system at home/work?
s • Current living situation?
• Has there been a change in personality? • Relationship history?
• Do you have any medical/surgical illnesses? • Education history?
Ask details. • Any risk of physical/sexual/mental abuse?
• Do you take any medications? Ask details. • Activities of daily living & Instrumental
• Do you consume alcohol? Activities of daily living.
Amount/frequency?

Differential Diagnosis • MRI/CT Head.


• Alzheimer's disease. • MSE/MMSE.
• Multi-Infarct dementia.
• Dementia with Lewy bodies. Management
• Depression. • Complete physical examination.
• Polypharmacy. • Involve social worker.
• Living environment assessment.
Investigations • Educational brochures.
• CBC, electrolytes, renal function tests. • Access to community resources.
• Liver function tests,Vit. B 12 levels. • Orientation cues: clocks,calendars,notes.
• TSH, blood glucose, ECG. • Resources for the family.
• Urinalysis, urine toxicology screen. • Acetylcholinesterase inhibitors.
• Safely home registry.
Clinical Cases - Psychiatry 137

Julian Smith, a 56 years old lady brought to your office by her husband with strange behavior. Take history &
counsel.

Clinical Info: Ms Julian Smith has h/o of change in mood for the past 1 month after loosing her job. She has
changes in mood, sleep and appetite. She has lost 10 lbs in the last 1 month. She has lack of interest in social
activities. She has no apparent psycho-motor or suicidal ideation. She has no medical illnesses. No past
history of psychiatric illness. Currently not taking any medications.
Clinical Case : Depression

HOPI • Do you take any medications? Ask details.


• When did you notice the change in • Do you consume alcohol and smoke?
behavior? Amount/frequency?
• Duration of symptoms? • Do you use recreational drugs? Ask
• Symptoms have worsened or improved? TRAPPED.
• Any recent stressors at home or work?
• Changes in sleep pattern? Past Psychiatric History
• H/o lack of interest recently? • Any similar symptoms in past?
• Feeling of guilt/hopelessness/ • Any h/o mania/depression/delusion/
helpless/worthless? delirium?
• Lack of energy? • Any contacts with mental health
• Changes in mood? professionals?
• Are you crying a lot? • Any past problems with law?
• Is your mood always low or it alternates?
• Any changes in concentration? Family History
• Any changes in appetite? • Any family history of similar complaints?
• Any changes in weight? • Any family history of other psychiatric
• Any changes in memory? illnesses?
• Do you feel slowed down? • Any family history of suicide/alcohol/
• Do you feel restless/agitated? drug abuse?
• Any thoughts of harming self/suicide?
• Any thoughts of harming someone else? Social History
• Any plans at the moment? • Support system at home/work?
• Do you feel persistently cheerful/high? • Current living situation?
• Do you sense things that others around you • Relationship history?
don't? • Education history?
• Do you have any medical/surgical illnesses? • Any risk of physical/sexual/mental abuse?
Ask details.

Differential Diagnosis Management


• Major Depressive Episode. • Start SSRI.
• Adjustment disorder with depressed mood. • Psychotherapy/CBT.
• Bipolar affective disorder. • Educational brochures about depression.
• Anxiety disorder. • Info about the side effects of medications.
• Emphasis on long term management,follow
Investigations up and compliance.
• CBC, electrolytes, renal function tests. • Referral to community resources.
• TSH, blood glucose, urinalysis. • Contract: Pt to contact you/family doctor/
• MSE/MMSE. nearest emergency/friend/family/crisis help
line in case of suicidal or homicidal ideation.
138 NAC OSCE | A Comprehensive Review

David Rosenberg, a 26 years old man brought to the ER by the police because he was throwing stones on a
public building. Take history & counsel.

Clinical Info: Mr David Rosenberg presented with irrational behavior for the past 10 days. He is having
racing thoughts, increased activity, decreased sleep and increased vocalization. He has constant flight of ideas
during the interview with easy distractibility. He is restless while sitting and at times agitated. No medical
illnesses but is a chronic cocaine abuser.
Clinical Case : Mania

HOPI • Do you have any medical/surgical illnesses?


• When did you notice the behavioral Ask details.
changes? • Do you take any medications? Ask details.
• Duration of symptoms? • Do you consume alcohol and smoke?
• Symptoms have worsened or improved? Amount/frequency?
• Any recent stressors at home or work? • Do you use recreational drugs? Ask
• Do you get easily distracted while talking or TRAPPED.
working?
• Did you have reckless driving & drinking Past Psychiatric History
episode? • Any similar symptoms in past?
• Have you gone for shopping sprees/ • Any h/o mania/depression/delusion/
gambling/ excessive spending? delirium?
• Do you feel like a special person with special • Any contacts with mental health
talent, power or on a mission? professionals?
• Do people say you jump from topic to topic? • Any past problems with law?
• Did anyone say that you have increased
activity/energy/appetite/restless/agitation? Family History
• Changes in sleep pattern? • Any family history of similar complaints?
• Did anyone say that you are more talkative • Any family history of other psychiatric
than usual? illnesses?
• H/o lack of interest recently? • Any family history of suicide/alcohol/drug
• Feeling of guilt/hopelessness/helpless/ abuse?
worthlessness?
• Changes in mood? Social History
• Any thoughts of harming self/suicide? • Support system at home/work?
• Any thoughts of harming someone else? • Current living situation?
• Any plans at the moment? • Relationship history?
• Do you sense things that others around you • Education history?
don't? • Any risk of physical/sexual/mental abuse?

Differential Diagnosis Management


• Cocaine induced Maniac episode. • Start Mood stabilizers -
• Bipolar disorder. Lithium/Valproate/CBZ.
• Personality disorder. • Psychotherapy/CBT.
• Substance abuse. • Admit in case of acute mania.
• Info about the side effects of medications.
Investigations • Emphasis on long term management,follow
• CBC, electrolytes, renal function tests. up and compliance.
• TSH, blood glucose, urinalysis. • Referral to community resources.
• VDRL, HIV. • Contract: Pt to contact you/ family doctor/
• Urine toxicology screen. nearest emergency/friend/family/crisis help
• Urgent psychiatric consult. line in case of suicidal or homicidal ideation.
Clinical Cases - Psychiatry 139

Brad Daniels, a 22 years old man came to your office with light headedness, trembling and chest pain for the
past 4 hours. Take history & counsel.
Clinical Info: Mr Brad Daniels is a university student who presented with sudden onset of light headedness,
trembling of body and chest pain prior to his presentation in class. He also complaints of palpitations and
shortness of breath. He had similar episodes in the past. No past history of psychiatric or medical illnesses.
Not taking any medications currently.
Clinical Case : Panic Attack

HOPI • Worry about consequences of another


• When did you notice the symptoms? attack?
• Duration of symptoms? • Any thoughts of harming self/suicide?
• Symptoms have worsened or improved? • Any thoughts of harming someone else?
• Any recent stressors at home or work? • Do you have any medical/surgical illnesses?
• Do you have excessive sweating? • Do you take any medications? Ask details.
• Do you experience tremors? • Do you consume alcohol/smoke?
• Do you have unsteadiness/dizziness? Amount/frequency?
• Do you sense things that others around you • Do you use recreational drugs? Ask
don't? (Derealization) TRAPPED.
• Do you ever feel you are outside of yourself ?
(Depersonalization) Past Psychiatric History
• Do you have excessive heart rate? • Any similar symptoms in past?
• Nausea/vomiting? • Any h/o mania/ depression/ delusion/
• Any tingling/paresthesias? delirium?
• Any shortness of breath? • Any contacts with mental health
• Any fear of dying/loosing control/going professionals?
crazy? • Any past problems with law?
• Do you have chest pain?
• Do you have chills/choking sensation? Family History
• Changes in sleep pattern? • Any family history of similar complaints?
• Feeling of guilt/ hopelessness/ helpless/ • Any family history of other psychiatric
worthless? illnesses?
• Any changes in mood? • Any family history of suicide/alcohol/drug
• Any changes in concentration? abuse?
• Any changes in appetite/weight?
• Any changes in bowel habits? Social History
• Any changes in memory? • Support system at home/work?
• Do you feel slowed down? • Current living situation?
• Do you feel restless/agitated? • Relationship history?
• Amount of caffeine intake? • Education history?
• Any anticipatory anxiety? • Any risk of physical/sexual/mental abuse?

Differential Diagnosis Management


• Panic Attack. • Start SSRI.
• Panic disorder. • Psychotherapy/CBT.
• Anxiety disorder. • Educational brochures about anxiety attacks.
• Mood disorder. • Info about the side effects of medications.
• Emphasis on long term management,follow
Investigations up and compliance.
• CBC, electrolytes, renal function tests. • Muscle relaxation, deep breathing &
• TSH, blood glucose, urinalysis. biofeedback.
• ECG. • Contract: Pt. to contact you/family doctor/
nearest emergency/friend/family/crisis help
line in case of suicidal or homicidal ideation.
140 NAC OSCE | A Comprehensive Review

Liam Pinkerton, a 24 years old male was brought to the ER with complaints of alien attacks . Take history &
counsel.
Clinical Info: Mr Liam Pinkerton is brought by police with complaints of being attacked by aliens in the last
48 hours. He is talking to himself and avoiding direct eye contact. He is restless and agitated and feels
threatened. He is hearing strange voices for the past 1 month along with disorganized speech and behavior.
He is a chronic cocaine user for the past 3 years and increased consumption in last 48 hours.
Clinical Case : Schizophrenia

HOPI • Get emotional or excited about things you


Thought phenomena: are used to?
• Think your thoughts are interfered with in • Have drive and ambition for anything at the
any way? moment?
• Think others can read your thoughts? • Any mood changes recently?
• Think an outside entity is affecting your • Do you feel restless/agitated?
thoughts? • Any changes in appetite/weight?
• Hear echo’s of your thoughts like a voice? • Any fever/nausea/vomiting/trauma?
Delusions: • Currently on any medications?
• Anything bothering you at the moment? • Do you consume alcohol?
• Ever felt that someone is out to get you? Amount/frequency?
• Different understanding of things to other • Do you smoke? Duration/frequency?
people? • Do you use recreational drugs? TRAPPED?
• Do you get any special information from Past Psychiatric History
things which happen? • Any similar symptoms in past?
• Do you get special messages or • Any h/o mania/ depression/ delusion/
communication from newspapers, radio, delirium?
television or any other source? • Any contacts with mental health
• Delusions of control; influence or passivity: professionals?
• Do you feel that a force can control or • Any past problems with law?
influence you?
• Think that some of your actions or thought Family History
are not being controlled by you? • Any family history of similar complaints?
Hallucinations: • Any family history of other psychiatric
• Hear voices when there is no one present? illnesses?
• Does someone talk to you? • Any family history of suicide/alcohol/drug
• How many? Who are they? What do they abuse?
say? Social History
• Do they ever tell you to harm/kill yourself or • Support system at home/work?
others? • Current living situation?
• How often do you hear voices? • Relationship history?
• How does it make you feel? • Education history?
• Can you control or stop them? • Any risk of physical/sexual/mental abuse?
• Blunted affect / Apathy, loss of drive?

Differential Diagnosis Management


• Schizophrenia. • Admit in hospital under FORM 1.
• Substance induced Acute Psychosis. • Start Anti-Psychotics.
• Acute Delusional disorder. • Inj Haloperidol 5mg IM stat.
• Schizoaffective disorder. • Urgent psychiatric consultation.
Investigations • Detoxification program.
• CBC, electrolytes, renal function tests. • Involve social work.
• Liver function test, blood gases.
• Blood glucose, urinalysis.
• Urine toxicology screen.
Clinical Cases - Psychiatry 141

Erica McCain is a 16 years old girl brought to the ER with ASA overdose. She is stabilized now. Take
history & counsel.
Clinical Info: Ms Erica McCain a 16 years old school going girl took 30 tabs of Aspirin after smashing her
parents car in a tree. She attempted to commit suicide to prevent embarrassment. She went to her friends
house after the accident. Her grandma brought her to the ER. Has h/o previous attempt 1 year ago. Is
currently consulting a psychiatrist on regular basis. Presently on antidepressants. Show EMPATHY!
Clinical Case : Suicide

HOPI • Do you use recreational drugs? Ask


• Analyze the attempt- TRAPPED.
When? / What method? / Source of
method? / Circumstances which lead to Past Psychiatric History
the attempt? • Any similar attempts in the past?
• What were your thoughts while hurting Outcome/admission, if any?
yourself ? • Treatment given for such attempts?
• What did you think would be the outcome? • Any h/o mania/ depression/ delusion/
• Changes in mood? delirium?
• H/o lack of interest recently? • Any contacts with mental health
• Feeling of guilt/hopelessness/helpless/ professionals?
worthless? • Any past problems with law?
• Lack of energy/concentration?
• Is your mood always low or it alternates? Family History
• Any changes in appetite/weight? • Any family history of suicidal attempts?
• Do you feel restless/agitated? • Any family history of other psychiatric
• Any thoughts of harming someone else? illnesses?
• Any plans at the moment? • Any family history of suicide/alcohol/drug
• Do you sense things that others around you abuse?
don't?
• Do you have any medical/surgical illnesses? Social History
Ask details. • Support system at home/work?
• Do you take any medications? Ask details. • Current living situation?
• Do you consume alcohol? • Relationship history?
Amount/frequency? • Education history?
• Do you smoke? Frequency/duration? • Any risk of physical/sexual/mental abuse?

Diagnosis Assess Suicidal risk: SAD PERSONS


• Suicidal Attempt. Sex - Male> female
Age - Bimodal: 15-25 years and > 65 years.
Investigations Depression
• CBC, electrolytes, renal function tests. Previous attempts
• Liver function tests, Blood gases. Ethanol use
• Se. salicylate levels q2h till levels fall. Rational thinking
• Blood glucose, urinalysis. Suicide in family
Organized plan
Management No support
• Admit under FORM 1. Sickness
• Gastric Lavage. Based on the score from the scale:
• Urine alkalinization. 0-2 - Send home with family.
• Urgent psychiatric evaluation. 3-4 - Close follow up, consider admission.
• Arrange family meeting and involve social 5-6 - Strongly consider admission.
worker. 7-10 - Admit.
• Start Lithium or Clozapine.
142 NAC OSCE | A Comprehensive Review

Brandon Rodrigues, 28 young man comes with recent onset of back pain and limp. Take focused history and
preform a focused examination.

Clinical info: Mr Brandon Rodrigues had a sudden onset of sharp lower back pain 2 days ago after lifting
heavy boxes at home. Pain is located in the lumbar area, grade 8/10 and is constantly present. He has
numbness and paresthesias present in his left leg for the past 12 hours. No weakness or loss of sensation in
the lower limbs. No urinary retention or bowel incontinence. He does not smoke or consume alcohol. O/E
there is tenderness in the L4 – L5 area & decreased sensation in the L4 – L5 dermatomal distribution.
Clinical Case : Back Pain

HOPI Past History


• OCD PQRST UVW + AAA • Do you have diabetes or hypertension?
• How did it start? Sudden or gradual. • Are you on any medications?
• Is it getting worse/better or no changes in • Are you allergic to any medications?
the symptoms? • Any surgeries in the past?
• Since how long have you noticed the back
pain? Family and Social History
• Where is the pain located? (Pain worse in • Do you smoke? Duration & frequency.
back than in buttocks or legs suggests • Do you consume alcohol? Duration &
mechanical back pain. Pain worse in buttocks frequency.
suggests radiculopathy) • Do you use recreational drugs? TRAPPED.
• Type of pain -sharp or dull? • Any family history of cancers/ medical
• Does the pain radiate or shoot anywhere? illnesses?
• Severity of pain on a scale of 1-10. • Erectile dysfunction (Cauda equina
• Do your symptoms change with time? syndrome)
• Any aggravating or relieving factors? (Pain
worse lying down and bilateral leg weakness RED FLAGS (BACKPAIN)
suggests spinal stenosis or ankylosing Bowel or Bladder dysfunction
spondylosis, ) Anesthesia (Saddle) – perineal numbness
• Effect on activities of daily living, functional Constitutional Symptoms – Malignancy
limitation? K : Chronic disease
• H/o fever/ fatigue / weight loss/night Paresthesias
sweats? Age > 50
• Burning micturation, joint pain, uveitis? IV drug users
• Morning stiffness? Associated numbness, Neuromotor deficits
weakness?
• Bladder retention /bowel incontinence?

Differential Diagnosis Management


• Disc herniation. • Reassurance and education if no cause (70%
• Spondyloarthropathy. improve in 2 weeks, 90% in 6 weeks)
• Radiculopathy. • Limited bed rest.
• Mechanical back pain. • Activity modification.
• Heat/Cold therapies.
Investigations • Low stress aerobic activities in first 2 weeks.
• Lumbo-sacral X-ray. • Encourage early return to work/ activities.
• NSAIDS/ Acetaminophen.

Surgery indicated in
• Cauda Equina.
• Worsening neurological deficit.
• Intractable pain not responding to
conservative treatment.
Clinical Cases - Surgery 143

Nicole Davy, a 75 years old lady presented with enlarging mole on her nose . Take a focused history and
perform focused physical examination.
Vitals: BP - 120/88 mm Hg, HR - 86/min, RR - 12/min, Temp - 37.0°C

Clinical Info: Ms Nicole Davy has an enlarging mole on her nose which is changing color and shape over the
past 1 month. She is Caucasian retired woman who spends 6 months in Florida during winters in Canada.
Recently noticed irregular edges of her mole and got concerned. She worked as a radiation technician for 30
years prior to retirement. She had a similar mole which was cancerous and removed 10 years ago. O/E: There
is a small 0.5 x 0.5 cm pearly papule on her lateral left side of nose, with irregular rolled out margins and
minimal discharge.
Clinical Case Diagnosis: Basal Cell Carcinoma.

HOPI Past History


• OCD PQRST UVW + AAA • Do you have diabetes or hypertension?
• How did it start? Sudden or gradual. • Are you on any medications?
• Is it getting worse/better or no changes in • Are you allergic to any medications?
the symptoms? • Any surgeries in the past?
• Duration of the mole? • Past h/o recurrent infections?
• Location of the mole?
• What changes have you noticed in the mole? Family and Social History
• Any itching? • Do you smoke? Duration & frequency.
• Any ulceration? • Do you consume alcohol? Duration &
• Any discharge or bleeding noticed? frequency.
• Any change in color of the mole? • Do you use recreational drugs? TRAPPED.
• Any other suspicious moles elsewhere? • Any family history of cancers/ medical
• Amount sun exposure? illnesses?
• Any radiation exposure? • Occupation?
• Any swelling noticed in the body?
• Any fever/nausea/vomiting/decreased
appetite?
• Any weight loss/night sweats?
• Any headache/chest pain/bony pain?
• Any visual changes?
• Any changes in bowel & urinary habits?

Differential Diagnosis Management


• Basal Cell Carcinoma. • Complete physical examination.
• Squamous Cell Carcinoma. • Electrodessication & curettage.
• Nodular malignant melanoma. • Surgical excision ± microscopically
• Intradermal melanocytic nevus. controlled surgery.
• Cryotherapy.
Investigations • 95% cure rate in lesions less than 2 cm.
• Punch/excision biopsy. • Life long follow up.
• Imiquimod 5% cream locally, if surgical
treatment not possible.
144 NAC OSCE | A Comprehensive Review

Jacob Simpson, 62 year old man presents to the Emergency Department with 12 hours suprapubic discomfort
and inability to urinate. Take a focused history & perform a focused examination.

Clinical Info: Mr Jacob Simpson presented with acute urinary retention for the past 12 hours. He is having
difficulty passing urine for the past 4 months, which has gradually increased. He has hesitancy, urgency,
increased frequency and weak stream. No hematuria or UTI. O/E there is a palpable supra-pubic mass.
Catheterization yields 1200cc urine.
Clinical Case : Benign Prostatic Hyperplasia

HOPI Past History


• OCD PQRST UVW + AAA • Do you have diabetes or hypertension?
• How did it start? Sudden or gradual. • Are you on any medications?
• Is it getting worse/better or no changes in • Are you allergic to any medications?
the symptoms? • Previous renal colic / diagnosed prostate
• Duration of inability to pass urine? hypertrophy?
• Any pain associated? • H/O prostate cancer, prostatism,
• Location of pain? nephrolithiasis, UTIs? H/O pelvic radiation?
• Type of pain – sharp or dull? • Any h/o past surgeries? TURP?
• Severity of pain on a scale of 1-10.
• Do your symptoms change with time? Family and Social History
• Difficulty initiating or maintaining urinary • Any family history of cancer / similar
stream? complaints?
• H/O weak/interrupted stream/ • Currently in a relationship?
• Feeling of incomplete bladder emptying? • Practicing safe sex?
• Any nocturia/ urgency/ increased frequency? • Do you think you are in risk of getting STIs?
• Color of urine? Any blood in urine? • Do you smoke? Duration & frequency.
• H/O recurrent urinary infections? • Do you consume alcohol? Duration &
• H/O renal pain or groin pain? frequency.
• H/O fever, night sweats, weight loss, fatigue? • Do you use recreational drugs? TRAPPED.
• Any bowel complaints?
• Any perineal numbness / leg weakness?

Differential Diagnosis Management


• Benign Prostatic Hyperplasia. • DRE (Digital rectal examination).
• Urinary Tract Infection. • Watchful waiting ( 50% resolve
• Prostatitis. spontaneously).
• Prostate Cancer. • Lifestyle modifications – evening fluid
restrictions, planned voiding.
Investigations • Urological consultation.
• Urea / Creatinine. • Medication – Alpha-adrenergic antagonists
• Urinalysis, culture & sensitivity. (Terazosin, Doxazocin, Tamsulosin).
• Prostate Specific Antigen (PSA). • 5-alpha-reductase inhibitors ( Finesteride).
• Renal and pelvis ultrasound. • Surgery (TURP, open prostatectomy).
• Cystoscopy. • Minimally invasive (Stents, Microwave
therapy, Laser ablation, Cryotherapy, HIFU,
TUNA).
Clinical Cases - Surgery 145

Lydia Jones, a 30 years old office lady presented to your office with right hand numbness and weakness for 2
months. Take a focused history and address her concerns.

Clinical info: Ms Lydia Jones presented with gradual onset of right numbness and weakness for the past 2
months. Her symptoms have worsened. Associated with paresthesias and pain in fingers at the end of the day.
She has difficulty opening jars, turning keys and night pains.
She has no medical illness. Not on any medications. She is an office administrator.
Clinical Case : Carpal Tunnel Syndrome

HOPI Past History


• OCD PQRST UVW + AAA • Do you have diabetes or hypertension?
• Handedness – left or right? • Are you on any medications?
• How did it start? Sudden or gradual. • Are you allergic to any medications?
• Is it getting worse/better or no changes in • Any surgeries in the past?
the symptoms?
• Duration of numbness? Family and Social History
• Location of numbness? • Do you smoke? Duration & frequency.
• Any paresthesias/tingling/swelling? • Do you consume alcohol? Duration &
• Any weakness or muscle pain? frequency.
• Is there any associated pain? • Any family history of thyroid disease?
• Any difficulty opening jars/turning keys? • Any family history of cancers/ medical
• Any symptoms in night time? illnesses?
• Any abnormal position of the hand?
• Any neck pain/upper arm weakness?
• Similar symptoms in other parts of the body?
• Do your symptoms become change with
time?
• Any changes in vision/speech/headache?
• Any changes in gait?
• Occupation?

Differential Diagnosis Management


• Carpal Tunnel Syndrome. • Modify manual work.
• Cervical radiculopathy. • Wrist splint (often worn at night).
• TIA. • NSAIDs, local corticosteroid injections.
• Control underlying systemic contributors
Investigations (e.g. diabetes, hypothyroidism, arthritis).
• CBC, blood glucose, electrolytes. • Surgical decompression via flexor
• C-spine X ray. retinaculum release.
• Nerve conduction studies. • Orthopedic or Plastic surgery consult.
146 NAC OSCE | A Comprehensive Review

Ruth Gagnon, a 60 years old woman presented to your office with right sided calf swelling for the past 2 days.
Take a focused history and perform focused examination.
Vitals: BP - 140/80 mm Hg, HR - 86/min, RR - 12/min, Temp - 37.5°C

Clinical Info: Ms Ruth Gagnon is having right calf swelling and redness for the past 2 days. She has leg pain
also. She is a known hypertensive on medications. She recently traveled for 20 hours in an overnight flight.
She is compliant with her medications. She has past history of breast cancer treatment 5 years ago with no
complications.
Clinical Case : Deep Vein Thrombosis

HOPI Past History


• OCD PQRST UVW + AAA • Do you have diabetes or hypertension?
• How did it start? Sudden or gradual. • Are you on any medications?
• Is it getting worse/better or no changes in • Are you allergic to any medications?
the symptoms? • Any surgeries in the past?
• Duration of leg swelling? • Past h/o recurrent infections?
• Where is the leg swelling located?
• Any leg pain associated with swelling? Family and Social History
• Type of pain - sharp or dull pain? • Do you smoke? Duration & frequency.
• Does the pain radiate or shoot anywhere? • Do you consume alcohol? Duration &
• Severity of pain on a scale of 1-10. frequency.
• Any pain during rest? • Any family history of thyroid disease?
• Any pain in the night time? • Do you take any recreational drugs?
• Any skin discoloration of legs/nail changes? • Any family history of cancers/medical
• Any skin ulceration of legs? illness?
• Any fever/cough/cold?
• Any chest pain/ shortness of breath?
• Any headache/dizziness?
• Any weakness/muscle pain?
• Any recent travel?
• Any prolonged immobilization?

Differential Diagnosis Management


• Deep Vein Thrombosis. • Urgent medicine consult.
• Ruptured Baker's Cyst. • Low molecular weight heparin x 3 days.
• Cellulitis. • Then switch to Warfarin.
• Unilateral lymphedema. • IVC filters (only if anticoagulation is
contradicted).
Investigations
• CBC, ESR, Lipid profile, RFTs.
• D-dimer.
• PT, PTT, INR, factor assay.
• Deficiency of Antithrombin III, protein C or
S.
• Lupus anticoagulant.
• Doppler of lower limbs.
• Impedance plethysmography.
Clinical Cases - Surgery 147

Mathew Hobbs, a 55 years old man presented with numbness in his both feet. He is a known diabetic. Take a
focused history and perform focused physical examination.
Vitals: BP - 130/90 mm Hg, HR – 86/min, RR – 14/min, Temp – 38.5°C.

Clinical Info: Mr Mathew Hobbs is a known diabetic for the past 15 years. He is on oral hypoglycemics. His
last fasting glucose was 7.6 mmol/L. On examination, both feet were normal.
Clinical Case : Diabetic Foot

HOPI Past History


• OCD PQRST UVW + AAA • Do you have diabetes or hypertension?
• How did numbness start? Sudden or • Are you on any medications?
gradual. • Are you allergic to any medications?
• Is it getting worse/better or no changes in • Any surgeries in the past?
the symptoms? • Past h/o recurrent infections?
• Duration of numbness?
• Location of numbness? Family and Social History
• Is there any pain associated with numbness? • Do you smoke? Duration and frequency.
• Type of pain - sharp or dull pain? • Do you consume alcohol? If yes, duration,
• Does the pain radiate or shoot anywhere? amount and frequency.
• Severity of pain on a scale of 1-10. • Do you use recreational drugs? TRAPPED
• Do your symptoms change with time? • Any family history of cancers/medical
• H/o tingling? illnesses?
• Any recent injury to feet? • Exercise schedule.
• Any blisters/calluses on the feet? • Dietary restrictions for diabetes.
• Any swelling of feet? • Compliance with medications?
• Any changes in vision?
• H/o dizziness / LOC?
• H/o chest pain/orthopnea?
• H/o excessive sweating?
• Any changes in appetite?
• Any changes in weight?
• Any changes in bowel & urinary habits?
• Last fasting blood sugar, eye and foot exam?

Diagnosis Management
• Diabetic foot • Self foot exam daily.
• Foot examined regularly at physician visits.
Investigations • Perform Peripheral Neuropathy Testing.
• Fasting blood glucose. • Check for pedal pulses.
• HbA1C. • Evaluate & aggressively treat new foot
• Fasting lipids. wound.
• ECG. • Avoid foot trauma
• Fundoscopy. • Do not walk barefoot.
• Urinalysis with urine dip. • Cut nails carefully.
• Avoid excessive heat or chemicals.
148 NAC OSCE | A Comprehensive Review

Wayne Singer, 68 year old man presented with difficulty swallowing for the past 4 months. Take a focused
history and perform a focused examination.

Clinical Info: Mr Wayne Singer has difficulty swallowing for the past 4 months. It has gradually increased
from solids to liquids. He feels a lump in the throat. He has chest pain when he eats food. He has noticed
weight loss, night sweats and decreased appetite in the last 3 months. He is a chronic smoker for the past 30
years.
Clinical Case : Difficulty Swallowing ( Ca Oesophagus)

HOPI Past History


• OCD PQRST UVW + AAA • Do you have diabetes or hypertension?
• How did it start? Sudden or gradual. • Are you on any medications?
• Is it getting worse/better or no changes in • Are you allergic to any medications?
the symptoms? • Any surgeries in the past?
• Duration of difficulty swallowing? • Any history of goitre or thyroid problems?
• Is there difficulty transferring food from • Any radiation exposure?
mouth to esophagus (suggestive of
oropharyngeal dysphagia) or further down Family and Social History
(suggestive of esophageal dysphagia)? • Do you smoke? Duration & frequency.
• Is the problem worse with solids (suggests • Do you consume alcohol? Duration &
mechanical obstruction) or liquids (suggests frequency.
neuromuscular dysfunction, often can't • Any family history of thyroid disease?
swallow both solids and liquids)? • Any family history of cancers/ medical
• Is there a sensation of lump in the throat illnesses?
(globus hystericus)?
• Progression of swallowing solids difficulty to
difficulty in swallowing liquid? (Suggests a
worsening stricture of growing tumor)
• Any swelling the neck?
• Aggravating, relieving factors? The
association of intermittent obstruction and
chest pain suggests esophageal spasm.
• H/O peptic ulcer, reflux, hiatus hernia?
• H/O fever, night sweats, weight loss, fatigue,
hematemesis, black stools?

Differential Diagnosis • Chest X-ray.


• Esophageal Cancer. • LFTs, RFTs.
• Stricture due to GERD/Trauma. • Abdominal ultrasound.
• Neuromuscular obstruction – achalasia, • CBC.
cranial nerve palsy, MS, supranuclear palsy, •
stroke, motor neuron disease, myasthenia • Management
gravis, muscular dystrophy. • Urgent surgical consult.

Investigations
• Endoscopy with biopsy.
• Upper GI series.
• CT chest (for mediastinal and lymph node
involvement).
Clinical Cases - Surgery 149

Brad Chisolm, a 35 years old man presented with bloody vomiting to the ER for the past 2 hours. Take a
focused history and perform focused physical examination.
Vitals: BP - 90/60 mm Hg, HR - 116/min, RR - 12/min, Temp - 37.0°C

Clinical Info: Mr Brad Chisolm presented with acute onset of blood in vomitus, 2 episodes in 2 hours ago.
He has no history of trauma. Non alcoholic, non smoker. He has been having chronic knee pain after a
skateboarding accident 2 weeks ago. He is taking Ibuprofen for the past 2 weeks 4-5 times a day. Has
moderate epigastric pain. No hemoptysis, hematuria or hematochezia. No surgeries/ medical illnesses.
Clinical Case : Hematemesis

HOPI Past History


• OCD PQRST UVW + AAA • Do you have medical illnesses?
• How did it start? Sudden or gradual. • Any surgeries in the past?
• Is it getting worse/better or no changes in
the symptoms? Family and Social History
• Duration of blood in vomitus? • Do you smoke? Duration & frequency.
• Amount of blood? • Do you consume alcohol? Duration &
• Color of blood? frequency.
• Number of episodes? • Any family history of cancers/ medical
• Is there any abdominal pain? • Do you take any recreational drugs?
• Type of pain - sharp or dull pain? • Any family history of cancers/ medical
• Does the pain radiate or shoot anywhere? illnesses?
• Severity of pain on a scale of 1-10.
• Do your symptoms change with time?
• Any blood while coughing?
• Any blood in stools/last bowel movement?
• Any blood in urine?
• Any trauma to abdomen?
• Any fever recently?
• Any dizziness/fainting?
• Currently on any medications?
• Last meal?
• Any allergic reactions?

Differential Diagnosis Management


• Gastric ulcer. • ABC.
• Acute Esophagitis. • Admit.
• Acute gastritis. • NPO.
• Drug induced coagulopathy. • NG tube.
• IVF via large bore cannulas.
Investigations • Inj Ranitidine 50 mg IV bolus and q8h.
• CBC, electrolytes, glucose. • In case of perforated ulcer - surgery consult.
• LFTs, RFTs.
• PT, PTT, INR.
• Blood group & cross match.
• Urgent endoscopy.
• Upright abdominal X Ray.
150 NAC OSCE | A Comprehensive Review

Mary Laplante, a 40 years old lady presented with swelling in the neck for the past 1 month. Take a focused
history and perform focused physical examination.
Vitals: BP - 120/88 mm Hg, HR – 96/min, RR – 12/min, Temp – 37.5°C.

Clinical info: Ms Mary Laplante noticed this swelling in the anterior neck with no other prominent
symptoms. On examination, there is 2cm x 2cm mobile, non tender thyroid enlargement in the left lobe.
Clinical Case : Neck Swelling

HOPI Past History


• OCD PQRST UVW + AAA • Do you have diabetes or hypertension?
• How did it start? Sudden or gradual. • Are you on any medications?
• Is it getting worse/better or no changes in • Are you allergic to any medications?
the symptoms? • Any surgeries in the past?
• Since how long have you noticed the lump? • Any history of goitre or thyroid problems?
• Where is the lump located? • Any radiation exposure?
• How does the lump feel like?
• Is there any pain associated with the lump? Family and Social History
• Type of pain - sharp or dull pain? • Do you smoke? Duration and frequency.
• Does the pain radiate or shoot anywhere? • Do you consume alcohol? If yes, duration,
• Severity of pain on a scale of 1-10. amount and frequency.
• Do your symptoms change with time? • Any family history of thyroid disease?
• H/o fever/cough/sore throat? • Any family history of cancers/medical
• Any weakness or myalgia? illnesses?
• Any change in voice?
• Any change in vision?
• Any change in appetite?
• Any change in weight?
• Any changes in bowel & urinary habits?
• Any temperature intolerance?
• Any palpitations or tremors?
• Any swelling of face or feet?
• Last menstrual period?
• Any changes in menstrual cycles?

Differential Diagnosis Investigations


• Toxic nodular goitre. • TSH.
• Hashimoto's thyroiditis. • Free T3 and T4.
• Thyroid cyst. • Thyroid ultrasound.
• Thyroid adenoma. • Thyroid biopsy.
• Thyroid lymphoma. • Antimicrosomal & anti-thyroglobulin abs.
Clinical Cases - Surgery 151

Judy Frances, a 25 year old female presented to your office with lower abdominal pain for the past 1 day. Take
a focused history and perform a focused examination.

Clinical Info: Ms Judy Frances had a gradual onset of right lower abdominal pain 24 hours ago. The pain has
gradually increased in intensity, grade 7/10. She has fever, nausea and vomiting since morning. No bowel or
urinary complaints. No trauma. Her LMP was one week ago. O/E peritoneal signs are present and tenderness
at McBurney's point.
Clinical Case : Pain Abdomen /Acute Abdomen

HOPI Past History


• OCD PQRST UVW + AAA • Do you have diabetes or hypertension?
• How did it start? Sudden or gradual. • Are you on any medications?
• Is it getting worse/better or no changes in • Are you allergic to any medications?
the symptoms? • Any surgeries in the past?
• Duration of abdominal pain?
• Where is the pain located? Family and Social History
• Type of pain - sharp or dull? • Do you smoke? Duration & frequency.
• Does the pain radiate or shoot anywhere? • Do you consume alcohol? Duration &
• Severity of pain on a scale of 1-10. frequency.
• Do your symptoms change with time? • Do you use recreational drugs? TRAPPED.
• Any aggravating or relieving factors (change • Any family history of cancers/ medical
in position/food intake)? illnesses?
• Effect on activities of daily living, functional
limitation?
• H/o fever, nausea/vomiting, dizziness/faint?
• Any urinary complaints?
• Loose stools /black stools or blood in stools?
• Change in stool caliber?
• Females :
LMP?
Pain associated with periods?
Vaginal discharge?

Differential Diagnosis Management


• Appendicitis. • Admit.
• UTI. • Urgent surgical consultation.
• Renal colic due to nephrolithiasis. • IVF.
• PID. • IV Antibiotics.
• Ectopic pregnancy. • NPO, NG tube.
• Ruptured Ovarian cyst or ovarian torsion.
• Mittelschmerz syndrome.

Investigations
• Abdominal X-ray 3 views
• Abdominal & pelvic ultrasound
• CBC, Electrolytes, Urea, Creatinine
• INR/PTT, Glucose, beta HCG
• Urinalysis
• Stool for occult blood
• Cervical swabs for culture/ PAP smear
152 NAC OSCE | A Comprehensive Review

Ronald Mandel, a 65 years old man presented to your office with bilateral leg pain for the past 2 weeks. Take
a focused history and perform focused examination.
Vitals: BP - 140/90 mm Hg, HR - 86/min, RR - 12/min, Temp - 37.5°C

Clinical Info: Mr Ronald Mandel is a known hypertensive for 15 years. He was diagnosed with CAD 5 years
ago. He is having bilateral lower leg pain for the past 2 weeks. Pain starts only when he has walked for 10-15
minutes. Pain subsides after taking rest. He has paresthesias too. No weakness or night pain. He is a chronic
smoker and alcoholic for 20 years.
Clinical Case : Peripheral Vascular Disease

HOPI Past History


• OCD PQRST UVW + AAA • Do you have diabetes or hypertension?
• How did it start? Sudden or gradual. • Are you on any medications?
• Is it getting worse/better or no changes in • Are you allergic to any medications?
the symptoms? • Any surgeries in the past?
• Duration of leg pain? • Past h/o recurrent infections?
• Where is the leg pain located?
• Type of pain - sharp or dull pain? Family and Social History
• Does the pain radiate or shoot anywhere? • Do you smoke? Duration & frequency.
• Severity of pain on a scale of 1-10. • Do you consume alcohol? Duration &
• Distance or time walked before pain starts. frequency.
• Speed of walking before pain starts. • Any family history of thyroid disease?
• Degree of incline walked. • Do you take any recreational drugs?
• Any pain during rest? • Any family history of cancers/medical
• Any pain in the night time? illnesses?
• Any skin discoloration of legs/nail changes?
• Any skin ulceration of legs?
• Any chest pain/ shortness of breath?
• Any headache/dizziness?
• Any weakness/muscle pain?
• Do the symptoms come back with exertion?
• H/o impotence?

Differential Diagnosis Management


• Peripheral Vascular Disease. • Symptomatic treatment.
• Osteoarthritis. • Tab Aspirin 75-150 mg OD.
• Neurogenic claudication. • Clopidogrel.
• Baker's cyst. • Vascular surgery opinion.
• Reduction of CAD risk factors.
Investigations • Alcohol & smoking cessation.
• CBC, Lipid profile.
• Serum Homocysteine, Apolipoprotein A.
• Serum Creatinine.
• Hemoglobin A1C ,fasting glucose.
• Urinalysis.
• Ankle brachial index, Doppler of lower
limbs.
• CT Angiography.
Clinical Cases - Surgery 153

Alex Pereira, a 45 years old man presented with high grade fever with chills on day 3 after his abdominal
surgery. Take a focused history and perform focused physical examination.
Vitals: BP - 110/80 mm Hg, HR - 96/min, RR - 12/min, Temp – 38.5°C.

Clinical Info: Mr Alex Pereira was operated 3 days ago for acute appendicitis. The morning shift nurse
noticed temperature of 39.8º C. He is complaining of chills , rigors and nausea. He has pain at the wound site.
No burning in urine. Had one bowel movement in the morning. No chest pain or shortness of breath. IV
antibiotics were stopped on post op day 2. No other complications. O/E: Wound site is tender, erythematous
with yellowish discharge.
Clinical Case : Post Operative fever secondary to wound infection

HOPI Past History


• OCD PQRST UVW + AAA • Do you have diabetes or hypertension?
• How did it start? Sudden or gradual. • Are you allergic to any medications?
• Associated with any chills/rigors? • Any surgeries in the past?
• Duration of fever? • Past h/o recurrent infections?
• Fever pattern- continuous,intermittent,
remittent? Family and Social History
• H/o cough/sore throat/rash? • Do you smoke? Duration & frequency.
• Any chest pain/shortness of breath? • Do you consume alcohol? Duration &
• Any abdominal pain? frequency.
• Any burning in urine/cloudy urine? • Do you take any recreational drugs?
• Any dysuria/hematuria? • Any family history of cancers/ medical
• Last bowel movement? illnesses?
• Any pain at the wound site?
• When was the wound dressing changed?
• Any pain at the IV site?
• Any pain in calves?
• Any leg swelling?
• Current list of medications?
• Any changes in medications?
• Last meal?
• Reason for surgery?
• Any pre-op/intra-op surgical complications?

Differential Diagnosis Management


• Post op wound infection. • Start IV Antibiotics & Antipyretics.
• Urinary tract infection. • Wound drainage.
• Intra-abdominal abscess. • Wound dressing for healing by
• Septic thrombophlebitis. secondary intention.
• Vitals q4h till fever subsides.
Investigations
• CBC, electrolytes, glucose.
• LFTs, RFTs.
• Urine routine and culture/sensitivity.
• Wound swab, culture/sensitivity, gram stain.
• Blood culture/sensitivity.
• Ultrasound abdomen.
154 NAC OSCE | A Comprehensive Review

Diane Richardson, a 55 years old woman presented with incidental finding of lung nodule on chest x ray. Take
a focused history and perform focused physical examination.
Vitals: BP - 120/80 mm Hg, HR - 86/min, RR - 18/min, Temp – 37.5°C.

Clinical Info: Ms Diane Richardson has chronic cough for 2 months. It was gradual in onset. No fever or
recurrent pneumonia. A routine chest X ray revealed solitary nodule in the right middle lobe. She is a known
smoker for the past 30 years. She is a chronic alcoholic also.
Clinical Case : Solitary lung nodule

HOPI Past History


• Why was the chest X ray done? • Do you have diabetes or hypertension?
• OCD PQRST UVW + AAA • Are you on any medications?
• How did cough start? Sudden or gradual. • Compliance with medications?
• Is it getting worse/better or no changes in • Are you allergic to any medications?
the symptoms? • Any surgeries in the past?
• Duration of cough? • Past h/o recurrent infections?
• Type of cough – dry/expectorant?
• Amount / color of cough? Family and Social History
• Any of shortness of breath? • Do you smoke? Duration & frequency.
• Present at rest or with exertion? • Do you consume alcohol? Duration &
• H/o orthopnea? frequency.
• H/o paroxysmal nocturnal dyspnea? • Any family history of cancers/ medical
• Any chest pain/palpitations? illnesses?
• Does the pain radiate or shoot anywhere? • Occupation?
• Severity of pain on a scale of 1-10. • Exposure to carcinogenic substances?
• Do your symptoms change with time?
• H/o fever/chills/night sweats?
• Any change in weight/appetite?
• Any recent travel?
• Any contact with sick person?
• Any pets at home?

Differential Diagnosis Management


• Lung Carcinoma. • Symptomatic treatment.
• Solitary granuloma. • Comparison with older chest X rays.
• Pulmonary tuberculosis. • Urgent respirology consult.
• TB skin test.
Investigations • CT guided biopsy.
• CBC, electrolytes, glucose. • Bronchoscopy.
• LFTs, RFTs.
• Chest X Ray.
• CT Chest.
• Sputum cytology.
Clinical Cases - Surgery 155

Elaine Jones, a 60 years old lady presented with swelling in the neck for the past 4 months. Take a focused
history and perform focused physical examination.
Vitals: BP - 120/80 mm Hg, HR – 88/min, RR – 12/min, Temp – 37.5°C.

Clinical info: Ms Elaine Jones presented with a solitary swelling in the right lobe of the thyroid for the past 4
months. The swelling has increased in size. She has no fever, cough or sore throat. She has decreased appetite
and 5 kg weight loss in the past 3 months. She has hoarseness of voice.
Clinical Case : Thyroid mass

HOPI Past History


• OCD PQRST UVW + AAA • Do you have diabetes or hypertension?
• How did it start? Sudden or gradual. • Are you on any medications?
• Is it getting worse/better or no changes in • Are you allergic to any medications?
the symptoms? • Any surgeries in the past?
• Since how long have you noticed the • Any history of goitre or thyroid problems?
swelling? • Any radiation exposure?
• Where is the swelling located?
• How does the swelling feel like? Family and Social History
• Is there any pain associated with the • Do you smoke? Duration & frequency.
swelling? • Do you consume alcohol? Duration &
• Type of pain - sharp or dull pain? frequency.
• Does the pain radiate or shoot anywhere? • Any family history of thyroid disease?
• Severity of pain on a scale of 1-10. • Any family history of cancers/ medical
• Do your symptoms change with time? illnesses?
• H/o fever/cough/sore throat?
• Any weakness or myalgia?
• Any change in voice?
• Any change in vision?
• Any change in appetite?
• Any change in weight?
• Any changes in bowel & urinary habits?
• Any temperature intolerance?
• Any palpitations or tremors?
• Any swelling of face or feet?

Differential Diagnosis Management


• Thyroid adenoma. • Urgent surgical opinion.
• Multinodular goitre. • Radioiodine in case of hyperthyroidism
• Thyroid cyst. • Chemo/radiotherapy in case of anaplastic
• Parathyroid adenoma. tumor.
• Benign nodule. • Surgical removal in cases other than
anaplastic tumor or lymphoma.
Investigations
• TSH.
• Free T3 and T4.
• Radioactive thyroid uptake scan-
Hot nodule : Hypo/Hyperthyroidism.
Cold nodule: Thyroid malignancy.
• Thyroid ultrasound.
• Thyroid biopsy and cytology.
• Neck CT.
156 NAC OSCE | A Comprehensive Review

Joseph Quinton, a 25 years old male was brought to the ER after a motor vehicle accident with the following
vitals: BP - 80/50 mm Hg, HR - 116/min, RR - 10/min, Temp - 37.0°C, O2 sat - 80%.
Manage the patient with a nurse.

Clinical Info : Mr Joseph Quinton had a MVA 1 hour ago. He is conscious, alert and responding to verbal
commands. He is in excruciating pain & complains of difficulty breathing. He can move all limbs. On
auscultation, there are decreased breath sounds on right side of chest with dullness on percussion.
Diagnosis: Trauma - Right sided hemothorax.

• Introduce yourself.
• Call out the patient's name and assess verbal response.
• Follow universal precautions - mask,wash hands,wear gloves.
• Ask for patient's vitals.
• Ask the patient to be connected to monitors: cardiac monitor, BP cuff, pulse oximeter, temperature
probe.
• Place cervical collar with in-line traction.
• AIRWAY - Open mouth & check airway for any loose body/dentures/bleeding. Mention any
specific smell.
• BREATHING -
- LOOK - cyanosis/pallor/icterus/nasal flaring/chest movements/respiratory rate/neck venous
engorgement.
- FEEL - flow of air/tracheal shift/chest wall for crepitus/flail segments/sucking chest
wounds/subcutaneous emphysema.
- LISTEN - sounds of obstruction/breath sounds/symmetry of air entry/air escaping/noisy
breathing.
• CIRCULATION - feel for peripheral pulses/ assess for shock-capillary refill,cool extremeities.
• DISABILITY - GCS/pupillary reaction.
• Order primary INVESTIGATIONS - CBC, differentials, electrolytes, RFTs, LFTs, ABG, INR,
PTT, 12 lead ECG, urinalysis, urine toxicology screen, portable chest X ray, C-spine X ray, Blood
group, type & cross match, blood glucose.
• Place large bore IV cannulas both arms & IVF 1 l normal saline bolus stat.
• Attach to 100 % oxygen through mask/nasal cannulas.
• Ask for vitals again.
• Ask for oreintation to time/place/person, mechanism of injury/ any eye witnesses/ any loss of
consciousness/ vomiting/ pain anywhere in the body/ last meal/ any drug allergies/ TAMPLE or
SAMPLE.
• EXPOSURE/ SECONDARY SURVEY - Assess for:
- Skull/cranium fractures.
- Injuries to the face.
- Hemptympanum/ otorrhea/ rhinorrhea/ epistaxis/ battle's sign/ racoon eyes.
- Check upper extremities for fractures/ bruises/ lacerations/ tattoos/ needle track marks/ medic
alert bracelet/ scars/ wounds.
- Check abdomen for movements/ scars/ wounds/ bruises/ rigidity/ masses, bowel sounds.
- Check lower extremities for fractures/ bruises/ wounds/ tattoos/ needle track marks.
- Pelvic compression to rule out pelvic fracture.
- Deep tendon reflexes of upper & lower extremities.
- Sensory examination of upper & lower extremities.
- Motor examination of upper & lower extremities.
- Genital examination.
- Spinal examination - log roll with help to look for fracture/ step deformity.
- Digital rectal examination.
- Change rigid board to semi rigid board.
Clinical Cases - Surgery 157

• Give SECONDARY ORDERS:


- Ask for vitals again.
- Results of the investigation ordered earlier.
- Request for needle thoracostomy with 16 G needle in 2nd intercostal space.
- Ask what do you see - air or blood.
- Urgent cardio-thoracic consult for chest tube.
- Order 2L blood transfusion.
- Foley's catheter.
- Naso-gastric tube.
- Measure intake/output.

Management for specific trauma case scenarios

Management for tension pneumothorax


• Large bore IV needle in 2nd intercostal space in the mid-clavicular line.
• Cardio-thoracic consult.
• Chest tube to be inserted in the 5th intercostal space in the anterior axillary line.

Management of open pneumothorax


• 3 sided sealed dressing.
• Cardio-thoracic consult.
• Chest tube to be inserted in the 5th intercostal space in the anterior axillary line.

Management of flail chest


• Nasal oxygen.
• IV fluids.
• Pain control with Inj Morphine 2-4 mg IV.
• Positive pressure ventilation.
• Cardio-thoracic consult.

Management of pericardial tamponade


• Nasal oxygen.
• IV fluids.
• Cardio-thoracic consult.
• Pericardiocentesis.

Management of increased intracranial pressure


• Raise head end of the bed to 30-45 degrees.
• maintain neck in a neutral position.
• Hyperventilate to target PCO2 30-35 mmHg.
• Osmolar diuresis - Inj Mannitol 20% IV 1-1.5 g/kg, then 0.25 g/kg q6h to achieve serum osmolarity
of 315-320.
• Sedation.
• Paralysis with vencuronium.
• Oxygen to maintain pO2 >60 mmHg.
• Urgent neurosurgery consult.
• Order CT scan of the head.
158 NAC OSCE | A Comprehensive Review

Management of open fracture


• Remove debris and wound irrigation with normal saline.
• Sterile dressing and splint the fracture.
• Check the neurovascular status of the limb.
• Order X ray of the fracture with 1 joint above and 1 joint below.
• Start IV analgesics.
• IV antibiotics.
• Tetanus shot, if not immunized in the last 5 years.
• NPO.
• Urgent orthopedic consult.

Management of anterior shoulder dislocation


• Posture of the arm - Abducted & externally rotated.
• Order X rays: AP, trans-scapular, axillary views.
• Urgent orthopedic consult.
• Closed reduction with IV sedation & muscle relaxation.
• Obtain post reduction x rays.
• Check post-reduction neurovascular status.
• Sling for 3 weeks, followed by shoulder rehabilitation.

Management of ankle sprain


• Follow Ottawa ankle rules to order X rays.
• Ankle X rays - AP, lateral , mortise views.
• Rest to the joint.
• Ice to be used for 5-20 minutes every 2 hours.
• Compression with a tensor bandage.
• Elevate the limb.
• Analgesics.
• Urgent orthopedic consult.
• Reduce weight bearing with help of crutches.
Clinical Cases - Counseling 159

Allison George, a 28 years old primigravida came to your clinic for her antenatal visit. She wants info for
breast feeding.
Take a focused history and address her concerns.

Counseling Case : Breast Feeding

HOPI Past History


Current gestational age? • Do you have medical illnesses?
Any complications in the current pregnancy? • Are you on any medications?
GTPAL : Gravidity, Term pregnancies, • Are you allergic to any medications?
Prematurity, • Any surgeries in the past?
Abortion, Living children. • Any h/o cancer in the past?
Last fetal ultrasound.
Any maternal screening till date for genetic Family and Social History
disease? • Do you smoke? Amount/frequency.
Any genetic disorder in family ? • Do you consume alcohol?
Any breast feeding issues in previous Amount/frequency.
pregnancies/ • Do you use recreational drugs? TRAPPED.
Any recurrent infections of the breast? • Any family history of cancers (esp breast
Any h/o HIV/HCV/HBsAg/active HSV? cancer)?

Counseling for Breast Feeding


• Breast feeding has to be initiated immediately after birth.
• Initial clear breast milk called COLOSTRUM is full of nutrients and immunoglobulins.
• It is beneficial for developing immunity in the newborn.
• Full milk production starts by 3-7 days.
• Exclusive breast feeding is recommended during the first 4 months,
• Breast milk is easily digested with minimal renal load.
• Breast milk has low allergic potential than cow's milk protein.
• Lower pH promotes growth of lactobacillus in the GI tract.
• Creates parent - child bonding.
• Breast fed babies require following supplements: Vitamin K, Vitamin D, Iron (from 4months to 12
months), Fluoride(after 6 months).
• Contraindications to breast feeding-
- Mother receiving chemotherapy.
- Mother with HIV/AIDS, active TB, herpes in the breast region.
- Mother consuming alcohol/illicit drugs.
- Mother on drugs contraindicated for breast feeding like antimetabolites, bromocriptine,
chloramphenicol, metronidazole, tetracycline, lithium, cyclophosphamide.
• Complications of breast feeding - sore/cracked nipples, breast engorgement, mastitis, breast feeding
jaundice, breast milk jaundice, oral thrush in baby.
• Breast feeding helps in losing pregnancy weight.
• Lactational amenorrhea protects against future pregnancy.
• Give educational info for breast feeding.
160 NAC OSCE | A Comprehensive Review

Rachel Marshall is a 20 months old girl brought to the ER with excessive crying. She has signs of fracture of
right humerus. You also observe some old healed bruises elsewhere on her body. She is now stable. Take
history from the mother and address her concerns.

Counseling Case : Child abuse

HOPI • Are there other children in the house?


• How did the injury occur? • Have they had broken bones or other
• When did the injury occur? injuries?
• Location of injury? • Was this child a planned pregnancy,
• What was the child doing at the moment of • Problems with pregnancy, birth history.
injury? Detailed history.
• Any loss of consciousness? • Developmental milestones. Detailed history.
• Any abnormal position or posture of the • What is the typical response of caregivers
body? when the child cries or misbehaves?
• Any seizures after the trauma?
• Any vomiting? Family and Social History
• Any excessive crying? • Do you smoke? Amount/frequency.
• Any bleeding or discharge from • Do you consume alcohol?
nose/ears/mouth? Amount/frequency.
• Who are the child's care givers? • Do you use recreational drugs? TRAPPED.
• Who lives in the house or comes in contact • Alcoholism/smoking/drug abuse by other
with the child? caregivers?
• How did the child get the bruises? • Economic condition of the family?
• What happened with the other fractures? • Any problems with the law?
• Any other injuries in the past? • Were the caregivers abused as children?
• Is the child accident prone or difficult to • Is there spousal abuse, sexual abuse or incest?
handle? • Has the Children's Aid Society been
• What is the child's personality: open vs. involved with this child or other children?
withdrawn.

Counseling for child abuse


• Do a complete physical examination of the baby.
• Document and/or photograph all injuries: type, location, size, shape, color, pattern.
• Inform parents or care takers about the suspicion of child abuse.
• Order blood tests to rule out medical causes of presenting symptoms.
• Sexually transmitted infection work up.
• Skeletal survey/bone scan.
• CT/MRI.
• Fundoscopy.
• Report all suspicious cases to the CHILDREN'S AID SOCIETY.
• Admit for serious injuries.
• Involve social worker and other community resources.
• Inform that you are legally obliged to inform the Children's Aid Society.
• Your duty to report overrides patient's confidentiality.
• Evaluate the risk factors for child abuse:
- Environmental factors- Social isolation, poverty, domestic violence.
- Caregiver factors- Parents were abused as children, psychiatric illnesses, substance abuse,
single parent family, poor social & vocational skills, below average intelligence.
- Child factors- difficult temperament, disability, special needs(eg developmental delay),
premature.
Clinical Cases - Counseling 161

Nancy Alfredo, a 30 years old woman presented to your clinic with a black eye and multiple bruises on her
arms. Take history and address her concerns.

Counseling Case : Domestic violence

HOPI • Who are the biological parents of the


• How did the injury occur? children?
• When did the injury occur? • Do the children witness physical abuse?
• Location of injury/injuries? • Ask about violence to the children, sexual
• Circumstances in which the injury occurred? abuse?
• Describe violent episode, what triggered it? • Is the boyfriend willing to seek help?
• Were objects used as weapons? • Any stresses at home/work?
• Was the boyfriend remorseful afterward?
• History of previous episodes of violence or Family and Social History
loss of temper by boyfriend. • Do you smoke? Amount/frequency.
• What was patient's response? • Do you consume alcohol?
• Has patient been in an abusive relationship Amount/frequency.
before? • Do you use recreational drugs? TRAPPED.
• Is boyfriend controlling? • Does the partner abuse alcohol or other
• Does he restrict her activities? drugs?
• Question her excessively after she has been • Economic situation?
out? • Any family history of physical abuse?
• Engages in verbal abuse or threats?
• Is the violence increasing in severity?
• Are there children in the house?

Counseling for domestic violence


• Explain that the boyfriend hitting the patient is a criminal assault and an example of domestic
violence.
• Domestic violence tends to increase over time unless the victim leaves, or the abuser and couple seek
therapy.
• Very often, women don't leave their abusive partner until they are seriously hurt.
• Domestic violence between adult partners tends to be reflected in future behavior of children who
are exposed to it and there is a risk of violence to the children.
• Child abuse is a criminal act and if suspected, is reportable to police by law.
• Spousal abuse is also a criminal act but is not reportable by law.
• Recommend that the patient not return to the abuser if there is risk to her safety (e.g. not the first
assault, abuser not remorseful).
• If the patient does return, an exit plan should be developed to ensure patient safety.
• Document all evidence of abuse (pictures, sketches) and related visits; quote patient directly in chart.
• Alternatively, the patient can contact the police to obtain a restraining order on the abuser.
• Develop a plan with the patient to seek alternate living arrangements (women's abuse shelter).
• Enlist the help of patient's support structure (friends, other family members).
• Contact the police (patient should be informed that, if contacted, the police will lay charges whether
the patient wants to or not).
• Counsel patient on how to enter into controlled, safe environment & contact with the abuser to
discuss possible therapy for anger management and controlling behaviors.
• Social worker referral and provide info about community resources.
• Arrange follow up.
162 NAC OSCE | A Comprehensive Review

Sara Chang, a 55 years old lady came to your clinic to get info about Hormone Replacement Therapy. She is
menopausal for the past 2 years. She is having significant hot flushes, mood fluctuations and vaginal dryness.
It is significantly affecting her quality of life.
Take a focused history and address her concerns.

Counseling Case : Hormone replacement therapy

HOPI • Any h/o fibroids?


• Menopausal since when? • Any headaches/migraines?
• Any post menopausal bleeding/spotting? • Any liver/all bladder disease?
• Any hot flushes? • Any blood clotting disorders?
• Any vaginal discharge/itchiness/dryness?
• Any dysparuenia? Past History
• Any night sweats/sleep disturbances? • Do you have medical illnesses?
• Any chest pain/palpitations? • Are you on any medications?
• Any h/o breast lump/mass? • Are you allergic to any medications?
• Any nipple discharge? • Any surgeries in the past?
• Any weight loss/gain? • Any h/o cancer in the past?
• Any bony pains?
• Any bowel/urinary changes? Family and Social History
• Any mood changes/irritability/decreased • Do you smoke? Amount/frequency.
libido? • Do you consume alcohol?
• GTPAL - Gravidity, Term pregnancies, Amount/frequency.
Prematurity, Abortions, Living children. • Do you use recreational drugs? TRAPPED.
• When was last PAP test done? • Any family history of cancers (esp
• When was the last breast exam done? breast/endometrial cancer)?

Counseling for hormone replacement therapy


• HRT is recommended for significant vasomotor symptoms of menopause.
• Low dose of hormones is given for a short duration of < 5 years.
• Types of HRT - Oral, Transdermal (patch, gel), Combined estrogen & progestin.
• Give information brochures about HRT.
• Compliance to medication dose is very important.
• Encourage annual physical examination with a family physician.
• Side effects:
Abnormal uterine bleeding, mastodynia.
edema, bloating, heartburn, nausea, mood changes.
• Contraindications to HRT:
Pre-existing uncontrolled hypertension.
Uterine fibroids, endometriosis., migraine.
Family h/o estrogen dependent cancers.
Chronic thrombophlebitis, diabetes mellitus.
Impaired liver function/ gall bladder disease.
Hypertriglyceridemia, fibrocystic disease of the breasts.
• HRT is protective against osteoporotic fractures and colon cancers.
• HRT increases risk for:
Invasive breast cancer.
Coronary heart disease.
Deep vein thrombosis or pulmonary embolism.
Stroke.
Dementia and mild cognitive impairment.
Clinical Cases - Counseling 163

Nadia Solanski, a 45 years old lady came to your clinic to get info about mammogram.
Take a focused history and address her concerns.

Counseling Case : Mammogram

HOPI Past History


• Any h/o breast lump/mass? • Do you have medical illnesses?
• Any breast discharge? • Are you on any medications?
• Any nipple discharge? • Are you allergic to any medications?
• Any trauma to the breast? • Any surgeries in the past?
• Any abnormal skin discoloration of breast? • Any h/o cancer in the past?
• Any breast surgeries/biopsies?
• Any fever/might sweats/weight loss? Family and Social History
• Any bony pains? • Do you smoke? Amount/frequency.
• Any bowel/urinary changes? • Do you consume alcohol?
• When was your first menstrual period? Amount/frequency.
• When was the last menstrual period? • Do you use recreational drugs? TRAPPED.
• Any irregularity in the menses? • Any family history of cancers (esp breast
• Any dysmenorrhea/ menorrhagia/ cancer)?
oligomenorrhea?
• Any passage of clots?
• GTPAL - Gravidity, Term pregnancies,
Prematurity, Abortions, Living children.
• Age at which 1st child delivered?
• H/o breast feeding?
• When was last PAP test done?
• When was the last breast exam done?

Counseling for Mammogram

• Mammogram is an annual screening test for the early detection of breast cancer.
• Breast cancer is the second leading cause of cancer mortality in women.
• Every 1 in 9 women in Canada are diagnosed with breast cancer.
• It is recommended after the age of 40 years or more for every women.
• It is done annually or every 2 years as a routine preventive test.
• If there is a strong family history of breast cancer or genetic pre-disposition to breast cancer, then
mammogram is done 5-10 years prior to the age of the relative detected with cancer.
• There are two types of mammogram - Screening and Diagnostic.
• Diagnostic mammogram is done in cases of breast mass/lumps/suspicion of breast cancer.
• Mammogram is a special X ray of the breast done in a diagnostic radiology clinic.
• The procedure might cause slight discomfort or pain which lasts only few seconds.
• It can take upto 20 minutes to do a mammogram.
• Images are interpreted by an experienced radiologists.
• In case of any abnormality, further tests will be arranged.
• Ideally mammogram should be done after your period has stopped, to avoid discomfort.
• Give information brochures for mammogram.
• Encourage annual physical examination with a family physician.
164 NAC OSCE | A Comprehensive Review

Jenna Martin is a 28 years new mother who has concerns about the immunization for her 2 months old son.
Take history and address her concerns.

Counseling Case : Immunization

HOPI
• Take detailed prenatal/antenatal history?
• Any complications during pregnancy?
• Any complications during delivery time?
• Any post partum complications?
• Any h/o genetic disorders in the family?
• Any h/o egg allergies?
• Any allergies to medications?
• Feeding pattern?
• Developmental history?
• Any fever/vomiting/irritability?
• Any bowel complaints?
• Any urinary complaints?
• Any neonatal jaundice?
• Hearing & vision tests for the newborn?
• Any issues during newborn examination?
• Any h/o complications after immunization in the family?

Counseling for immunization


• Ask about the patient's concerns?
• What does she want to know and why?
• Immunization (vaccination) is a way of creating immunity to certain diseases by using small amounts
of a killed or weakened microorganism that causes the particular disease.
• Explain that vaccines protect children from diphtheria, tetanus, pertussis, polio (DPTP), mumps,
measles, rubella (MMR), influenza (Hib) and hepatitis B (Hep B).
• All of which were once common and caused serious, sometimes fatal illness in Canada, and now
these diseases are hardly ever seen because of vaccines.
• Explain that the vaccines stimulate the immune system.
• Infants are fully capable of generating protective humoral & cellular immune responses to multiple
vaccines simultaneously.
• The timing of vaccines is important; some vaccines are most effective when given to children at a
particular age or in combination with other vaccines.
• Every batch of vaccine is tested for safety and quality in Canada before it is released for public use.
• Some children have a temporary sore arm (with induration and tenderness) at the injection site,
malaise, mild fever or rash.
• Allergic reactions to vaccines also occur including urticaria, rhinitis, anaphylaxis.
• It is very rare to have a more serious reaction (seizures, encephalopathy have been reported).
• Standard modern vaccines are not known to cause disease or to have long-term deleterious effects.
• The risks of vaccinations are small compared with the health risks associated with the diseases
they're intended to prevent.
• Explain the recommended immunization schedule (below) :
Give the patient some information pamphlets, invite further questions and ask her to return
in two weeks for the child's first immunization.
Clinical Cases - Counseling 165

James Hendrik is a 40 years old man who came to your office today to discuss his weight issues. His current
weight is 250 lbs, Height 5 feet 7 inches, BMI 39.2.
Take history and address his concerns.

Counseling Case : Obesity

HOPI • Crisis in patient's life, stress, anxiety?


• Duration of weight gain? • Assess patient's self-image: does patient feel
• Amount of weight gain? underweight, overweight or normal?
• Dietary habits: frequency of meals, snacking, • Does patient feel that weight interferes with
eating at night, foods eaten, binge eating, health, activities?
guilt about food, hoarding, concealing eating • Screen for eating disorders.
from others.
• Estimated daily caloric intake. Past History
• Exercise history. • Do you have medical illnesses?
• Diseases associated with weight gain: • Are you on any medications?
hypothyroidism, DM type II, Cushing's, • Are you allergic to any medications?
major depression, anxiety disorder, some • Any surgeries in the past?
medications (TCA, steroids, OCP).
• Problems associated with overweight: gout, Family and Social History
sleep apnea, cholecystitis, back pain, • Do you smoke? Amount/frequency.
cardiovascular disease, hemorrhoids, lower • Do you consume alcohol?
limb joint pain and osteoarthritis. Amount/frequency.
• Why is patient seeking medical help for this • Do you use recreational drugs? TRAPPED.
now? • Any family history of cancers?
• Past attempts to lose weight, successes, • Any overweight relatives?
obstacles, goals.

Counseling for Obesity

• Motivation: how would being at ideal body weight improve the patient's life?
• Emphasize health, lifestyle, self esteem, relationship benefits.
• Discuss nutrition-related problems: heart disease, obesity, hypertension, osteoporosis, anemia, dental
decay, cancer, gastrointestinal disorders, respiratory compromise, high lipids, diabetes, sleep apnea,
osteoarthritis.
• Discuss diets tried and why these failed.
• Fad diets involve unusual or extreme eating patterns and are not designed to be maintained for a
lifetime therefore these should be discouraged.
• Weight loss agent Ponderal no longer available.
• SSRIs such as Paxil may assist with weight loss, unfortunately, when the drug is discontinued, most
people regain weight.
• Explain that the brain has a satiety set point which can be reset over time with reduction in caloric
intake.
• Warn that the body's ability to determine caloric content is very good, and will not be fooled by so-
called diet products.
• Recommend a balanced diet consisting of ordinary foods, with three distinct meals per day of small
size.
• No eating at night and be careful of snacks.
• Inform patient that he will be hungry for at least the first two weeks of reduced intake.
• Suggest visualization techniques, redirection of interests, and to think of hunger as a sign of positive
progress on weight loss.
• Group support can be beneficial too: Weight watchers, overeaters anonymous etc.
166 NAC OSCE | A Comprehensive Review

• Behavior modification and positive outlook is vital for weight loss regime.
• Dietary recommendations: reduce fat to 20% of caloric intake. 1200-1600 kcal/day for males.
• Ideal caloric intake can be estimated at 10-12 Cal/lbs (ideal weight) in males.
• Emphasize that caloric intake is more important for weight loss than food composition (i.e. excessive
calories lead to weight gain even if they are non-fat).
• Exercise recommendations: 30 minutes of moderate intensity exercise, 4-5 days/week.
• Sudden intense exercise in sedentary patient unwise.
• More vigorous exercise can be initiated when weight is lost.
• Behavioral modifications, self control, rewards on achieving goals.
• Arrange regular follow-up for body mass monitoring and counseling.
Clinical Cases - Counseling 167

Peter Harper is a 35 years old man, who is a chronic smoker for the past 10 years. He came to your office
today because he wants to quit smoking.
Take history and address his concerns.

Counseling Case : Smoking

HOPI • How many times have you tried to quit


• Duration of smoking? smoking?
• Frequency of smoking in a day? • If you tried to quit before, what methods you
• At what age did you start smoking? used?
• What type of tobacco do you smoke- • What challenges you faced while quitting?
cigarette/ cigars/ pipe/filter/non filter? • What methods helped you in the plan?
• When do you have your first cigarette of the • What is your motivation to quit on the scale
day? of 1-10?
• If you do not smoke for a while, do you
experience cravings or withdrawal Past History
symptoms? • Do you have medical illnesses?
• Which symptoms do you experience • Are you on any medications?
- weak & tired? • Are you allergic to any medications?
- sad or blue? • Any surgeries in the past?
- irritable or cranky? • Any h/o cancer in the past?
- difficulty concentrating?
- restlessness? Family and Social History
- anxious or jittery? • Do you consume alcohol?
• What people,places or events make you crave Amount/frequency.
a cigarette? • Do you use recreational drugs? TRAPPED.
• How does smoking help you? • Any family history of cancers?
• Does smoking help or harm your • Occupation?
relationships?
Alphabetical Index
Abortion - clinical case..........................................118 Collateral Ligament Stability...................................82
Acetaminophen .......................................................30 Community Acquired Pneumonia...........................25
Acne.........................................................................11 Congestive Cardiac Failure........................................8
Acute Bronchiolitis..................................................45 Congestive Heart Failure - clinical case.................104
Acute Cholecystitis..................................................19 COPD......................................................................24
acute confusional state..............................................52 Crohn’s Disease........................................................17
Acute Gastroenteritis...............................................16 Croup.......................................................................48
Acute Myocardial Infarction......................................7 Deep Vein Thrombosis - clinical case....................146
Acute Otitis Media .................................................45 Delirium...................................................................52
Acute Pancreatitis.....................................................17 Delirium - clinical case..........................................135
Acute Pharyngitis.....................................................23 Delirium Tremens....................................................36
Acute Pyelonephritis................................................30 Dementia..................................................................55
Acute Sinusitis.........................................................23 Dementia - clinical case.........................................136
Alcohol.....................................................................35 Depression................................................................56
Alcohol withdrawal............................................30, 59 Depression - clinical case.......................................137
Allen Test.................................................................68 Diabetes Mellitus.....................................................14
Allergic reaction.......................................................50 Diabetic Foot - clinical case...................................147
Allergic Reaction......................................................30 Diabetic ketoacidosis................................................31
Anaphylaxis..............................................................30 Diabetic Ketoacidosis...............................................14
Anemia...............................................................19, 50 Digoxin.....................................................................32
Ankle Anterior Drawer Test ...................................84 Digoxin Toxicity - clinical case..............................106
Anorexia - clinical case...........................................133 Diverticulitis.............................................................17
Antenatal Visit - clinical case.................................119 Domestic violence - counseling.............................161
Anterior Drawer Test...............................................82 Dysfunctional Uterine Bleeding..............................40
anti-hypertensive drugs............................................10 Dyslipidemia..............................................................9
Apley's Scratch Test.................................................86 Dysmenorrhea..........................................................40
Appendicitis.............................................................16 Ectopic Pregnancy....................................................42
Apprehension Sign...................................................86 Ectopic Pregnancy - clinical case...........................120
Arrhythmias.............................................................31 Emergency contraception.........................................41
ASA..........................................................................31 Empty Can Test ......................................................86
Asthma...............................................................24, 45 Endometriosis..........................................................40
Asthma - clinical case............................................103 Epiglottitis................................................................48
Athlete's foot............................................................13 Ethylene glycol ........................................................32
Atrial Fibrillation...............................................8, 102 Examination - Abdominal.......................................63
Atrial Fibrillation - clinical case............................102 Examination - Back/Spine.......................................77
Atrophic vaginitis.....................................................39 Examination - Breast...............................................90
Back Pain - clinical case.........................................142 Examination - Cardiovascular..................................65
Bacterial Meningitis.................................................49 Examination - Central Nervous System .................71
Bacterial Pneumonia ...............................................46 Examination - Elbow ..............................................87
Bacterial Tracheitis...................................................45 Examination - Foot and Ankle ...............................83
Bacterial vaginosis....................................................39 Examination - Hand and Wrist ..............................88
Basal Cell Carcinoma - clinical case......................143 Examination - Hip ..................................................79
Benign Prostatic Hyperplasia - clinical case..........144 Examination - Knee ................................................81
Bipolar disorder........................................................57 Examination - Lower Limb Neurological ..............75
Bowstring test..........................................................78 Examination - Mini Mental State ..........................93
Breast Feeding - counseling...................................159 Examination - Peripheral Vascular ..........................67
Buerger's Test...........................................................68 Examination - Respiratory ......................................69
Bulimia - clinical case............................................134 Examination - Shoulder...........................................85
Burns........................................................................11 Examination - Thyroid ...........................................91
Ca Oesophagus - clinical case................................148 Examination - Upper Limb Neurological ...............73
Candidiasis...............................................................39 External Rotation Lag Sign.....................................86
Carpal Tunnel Syndrome - clinical case.................145 FABER...............................................................78, 80
Cellulitis...................................................................13 Failure to thrive - clinical case...............................126
Cerebrovascular Attack - clinical case....................105 Febrile Seizures........................................................49
Chest Pain - clinical case.......................................111 Febrile seizures - clinical case.................................127
Child abuse - counseling........................................160 Femoral stretch test..................................................78
Chlamydia................................................................38 Fibromyalgia.............................................................29
Cluster headache......................................................22 Finkelstein's test.......................................................89
Cocaine Overdose....................................................59 General Anxiety Disorder........................................54
Alphabetical Index
Pain Abdomen - clinical case.................................151
Genital herpes..........................................................38 Panic attack..............................................................53
Genital warts ...........................................................38 Panic Attack - clinical case....................................139
Gerber Lift-Off Test................................................86 Panic disorder...........................................................53
GI Bleed...................................................................34 Parkinson’s disease....................................................23
Golfer's Elbow.........................................................87 Patellar Apprehension..............................................82
Gonorrhea................................................................38 Patellar Grind...........................................................82
Gout.........................................................................27 Pediculosis................................................................13
Group A Streptococcus............................................48 Pelvic Inflammatory Disease....................................40
Group B Streptococcus............................................41 Pelvic inflammatory disease - clinical case.............123
Hallucinogens...........................................................59 Pemberton's sign......................................................91
Hawkins Impingement Sign....................................86 Peptic ulcer disease...................................................19
Headache - clinical case.........................................110 Peripheral Vascular Disease - clinical case.............152
Heart Failure..............................................................8 Pertussis....................................................................48
Helicobacter Pylori..................................................17 Phalen's test .............................................................89
Hematemesis - clinical case...................................149 Phencyclidine...........................................................59
Hemothorax - clinical case.....................................156 Placenta Previa - clinical case.................................124
Hepatitis B...............................................................18 Plantar Fasciitis Test................................................84
HIV..........................................................................19 Pneumonia - clinical case.......................................112
Hormone Replacement Therapy..............................41 Polymyalgia Rheumatica..........................................28
HRT - counseling..................................................162 Post exposure - clinical case...................................113
Horner's sydrome.....................................................69 Post Operative fever - clinical case.........................153
Hyperemesis Gravidarum........................................42 Post-traumatic stress disorder .................................55
Hyperprolactinemia...............................................15p. Posterior Drawer Test..............................................82
Hypertension........................................................9, 32 Pre Eclampsia - clinical case..................................125
Hyperthyroidism......................................................15 Pregnancy Induced Hypertension............................41
Hypoglycemia..........................................................32 Primary nocturnal enuresis - clinical case..............130
Hypothyroidism.......................................................15 Psoriasis....................................................................12
Immunization - counseling....................................164 Psychosis...................................................................56
Immunization Schedule...........................................50 Pulmonary Embolism..............................................25
Impotence.................................................................16 Pulmonary Embolism - clinical case......................114
Impotence - clinical case........................................108 Pulmonary tuberculosis............................................20
Infectious Mononucleosis - clinical case................107 Pyelonephritis...........................................................39
Infective Endocarditis..............................................10 Pyloric stenosis - clinical case................................131
Infertility - clinical case..........................................121 Rabies.......................................................................20
Lachman Test...........................................................82 Rheumatic Heart Disease........................................10
Laryngotracheobronchitis........................................48 Rheumatoid Arthritis...............................................26
Lasegue's sign...........................................................78 Romberg’s test..........................................................72
Malaria.....................................................................20 Scabies......................................................................13
Mammogram - counseling.....................................163 Schizophrenia - clinical case..................................140
Mania.......................................................................53 Schober's Test...........................................................78
Mania - clinical case...............................................138 Seizure disorder - clinical case...............................115
McMurray's Test......................................................82 Seizures.....................................................................21
Measles - clinical case............................................128 Septic Arthritis.........................................................29
Meningitis................................................................21 sexual dysfunction....................................................58
Meningitis - clinical case.......................................109 Sexually Transmitted Infection................................38
Migraine...................................................................22 Shock........................................................................33
Mood stabilizers.......................................................57 Smoking...................................................................35
Myasthenia Gravis...................................................23 Smoking - counseling............................................167
Neck Swelling - clinical case..................................150 Social Phobia............................................................54
Neer Impingement Sign...........................................86 Solitary lung nodule - clinical case........................154
Neonatal Jaundice - clinical case............................129 Speech delay - clinical case....................................132
Obesity - counseling..............................................165 Speed's Maneuver....................................................86
Obsessive-compulsive disorder ...............................55 Sprain.......................................................................33
OCP Counseling - clinical case.............................122 Straight leg raising test.............................................78
Opioid......................................................................33 Streptococcal Pharyngitis.........................................48
Opioid Intoxication..................................................59 Stroke.......................................................................33
Osteoarthritis...........................................................26 Substance abuse........................................................59
Osteoporosis.............................................................26 Suicide - clinical case.............................................141
Ottawa Ankle rules..................................................84
Alphabetical Index
Syphilis.....................................................................38 Trendelenburg Maneuver.........................................68
Talar Tilt Test...........................................................84 Trichomonas vaginalis..............................................39
TCA.........................................................................34 Troisier's Sign...........................................................69
Temporal arteritis.....................................................28 Tylenol......................................................................50
Temporal Arteritis - clinical case...........................116 Ulcerative Colitis......................................................19
Tennis Elbow ..........................................................87 Urinary tract infection..............................................29
Tension headache.....................................................22 Urinary Tract Infection.......................................39, 49
Tetanus.....................................................................21 Viral Hepatitis - clinical case.................................117
Thomas test..............................................................80 Virchow's Node........................................................69
Thompson's Test......................................................84 Vocal fremitus...........................................................70
Thyroid mass - clinical case...................................155 Vulvovaginitis...........................................................39
Tinea Cruris.............................................................13 Warfarin...................................................................34
Tinel's sign...............................................................89 Whooping Cough ...................................................48
Trendelenberg test....................................................79 Yergason test.............................................................86
Content
Medicine
CVS Diabetes Neurology
Ankle swelling DKA Approach to
Calf pain Tiredness Headache, acute
Hypertension Headache, environmental
MI Headache, primary
Palpitation GI Headache, subacute
Syncope Abnormal LFT Headache, template
Acute abdomen Multiple Sclerosis
Diarrhea Neuropathic pain
RS Dysphagia Weakness in arm
Asthma GERD
Cough, Acute Mesenteric Iscemia
Cough, Chronic Otolaryngology
Hemoptysis Allergic rhinitis
SOB GU Lump in neck
SOB – Post Surgery Introduction
Dark urine
Geriatric wets himself
Hematology Difficulty to pass urine
High MCV
Wrong blood transfusion
HIV
Breaking news – HIV
Ophthalmology Fever and tiredness
Loss of vision Headache – HIV complications
HIV test request
Needle stick injury
Tiredness for 6 weeks

Please note that this is only a draft version based on several sources, including: Dr. Basel Mohasen’s lectures,
Toronto Notes, Therapeutic Choices and others. Edited and organized for the sake of all attendances of the
Canadian Osce Exams: NAC OSCE and MCCQE2. by: Dr. Merlyn D’Souza and Dr. Zeev Gross, Spring 2011. Page 1
Counselling Paediatrics
Abortion Introduction FTT 1
Ante-natal counselling Anaphylactic Shock Child abuse 2
Breast Feeding Cough Speech delay 3
Diabetic daughter Cry Vomiting in newborn 4
Domestic violance Diarrhea Enuresis 5
Endometrial cancer Diabetic daughter Febrile seizures 6
Epilepsy Febrile seizure Jaundice child 7
Fall Fever Joint pain 8
Febrile seizure Marijuana counseling Child with ADHD 9
HRT Osgood Schlatter Child with chronic cough 10
Lump in breast Pale Child with abdominal pain 11
Obesity Phone cases Well-baby visit 12
OCP Post-concussion Child with fever 14
Pregnancy Vomiting Yellow baby 16
Smoking Cessation Yellow discoloration Vomiting baby 17
Warfarin Counseling Immunization Baby with diarrhea 18
Pap Smear IUGR Pallor baby 19
OCP 3 Child abuse Child with chronic cough 20
Breast feeding 5 Enuresis Child with abdominal pain 21
Ante-natal 7 Child abuse 22
HIV 9 Enuresis 26
Needle stick 10 Hyperactive child 27
IUGR 11 History taking format 28
Abortion 12 Vomiting child 30
Sexual abuse 14 Anemia in a child 32
Epilepsy 15 Diarrhea in a child 34
Drug seeker 17 Enuresis 36
Smoking cessation 18 Breast feeding 37
Alcohol cessation 20 Delayed speech 39
Imptence 21 ADHD 40
HRT 22 FTT 41
Obesity 24 Child abuse 43
Anorexia nervosa 44
Vaccination 46

Please note that this is only a draft version based on several sources, including: Dr. Basel Mohasen’s lectures,
Toronto Notes, Therapeutic Choices and others. Edited and organized for the sake of all attendances of the
Canadian Osce Exams: NAC OSCE and MCCQE2. by: Dr. Merlyn D’Souza and Dr. Zeev Gross, Spring 2011. Page 2
OBGYN Psychiatry Physical Exam / Manage
Introduction Psychiatry Introduction Introduction to Physical Exam
Amenorrhea Anorexia Nervosa Acute and acute on chronic
Ask for file after C/S Bipolar Disorder abdomen
HRT Delusions – contamination Back
Infertility Delusions – persecutory Blood transfusion
Pre-eclampsia Depression Cranial Nerves
Request for C/S Forms CVS
Vaginal bleeding Diabetic Foot
Insomnia
Vaginal discharge ER: Trauma and Non-Trauma
Mania
First trimester bleeding 1 Hand-Laceration
Third trimester bleed 2 Marijuana in the bag
Hand - CTS
Pre-eclampsia 3 MMSE
Hematemesis
High risk pregnancy 4 MMSE-Delirium
Hip
Counseling breast feed MSE-Psychosis
Knee
5 Panic Attack Neck
Vaginal bleeding 6 Personality disorders PVD
Vaginal discharge 7 Psychiatric assessment Respiratory system
Amenorrhea 8 Req. admit (Delusion) Secondary Hypertension
OCPs 9 Req. Admit (Borderline) Shoulder
Counseling HRT 10 Request to stop Lithium Unconscious Patient
Screen for Breast Ca 11 Somatization disorder Volume Status
Pap smear 12 Suicide Attempt MI
Depression 21 GI Bleeding 3
Ethics Manic episode 24 DKA 5
Decision to forgo treat 3 Sleep hygiene 26 Asthmatic attack 7
Delivering bad news 4 Delusional disorder 35 TCA Overdose 8
Woman abuse 5 Schizophrenia 36 Seizure 9
Telling the truth 6 Panic disorder 39 Anaphylaxis 11
Death before arrival 7 MMSE 42 Acute aortic dissection 12
Pharmacist refusal 8 MSE 43 Subarachnoid hemorrhage 14
Organ retrieval 9 Dementia 44 Violent patient 17
Confidentiality 10 Delirium 46
Decision maker 11 Suicidal attempt 48
Substitute Decision M 12 Competency=Capacity 50
(Admition) notes 13 Alcohol abuse 55
Borderline personality D. 57

Please note that this is only a draft version based on several sources, including: Dr. Basel Mohasen’s lectures,
Toronto Notes, Therapeutic Choices and others. Edited and organized for the sake of all attendances of the
Canadian Osce Exams: NAC OSCE and MCCQE2. by: Dr. Merlyn D’Souza and Dr. Zeev Gross, Spring 2011. Page 3
Tool Setting Meaning
OCD Any S&S Onset, Course, Duration

Position, Quality, Radiation, Scale, Time/Triggers, How


PQRSTUV Pain
it affects YOUR life? Deja Vu – is it the first time?

COCA-B Secretion Colour, Odour, Consistency, Amount, Blood

Psychiatric Mood, Organic, Anxiety, Psychosis,


MOAPS screening Suicide/Homicide/Self care

Home, Education, Alcohol, Activities (hobies), Diet,


HEAADDDSSSS Teen ager
Dating, Drugs, Sexual activity, Stress, Sleep, Suicide,

OBGYN
MGOS questions in Hx
Menstrual, Gynecological, Obstetrics, Sexual

Mood, Interest, Psychomotor retardation, Appetite,


MI PASS ECG R/O Depression
Sleep, Suicide, Energy, Concentration, Guilt
Sex: male; Age: >60; Depressed; Previus attempts; Ethanol/drugs;
SAD PERSONS Risk of suicide Rational thinking loss; Suicide in family; Organized plan; No
support; Serious illness/pain

Distractability, Impulsivity, Grandiosity, Flight of ideas,


DIG FAST R/O Mania
Goal directed Activity, Sleep, Talkative

HRT or OCP Active liver disease, Blood diathesis, Cance (breast,


ABCD C/I endometrial), DVT

Benefits of Anemia and Acne – reduced; Benign breast disease decreased; Cancer
ABCDE OCPs
(ovarian) decreased, Cycles regulated, Cervical mucous increased (reduces
STIs), Dysmenorrhea decreased, Ectopic pregnancy/ pregnancy reduced.

SPIKE Bad news Setting, Perception, Invitation, Knowledge, Empathy

Birth, Immunization, Nutrition, Development,


BINDE Child
Environment

Allergies, Medications, PMHx, Last


AMPLE Trauma
menstruation/meal/tetanus shot, Event

SEADS Joint P/E Swelling, Erythema, Atrophy, Deformity, and Scars

Please note that this is only a draft version based on several sources, including: Dr. Basel Mohasen’s lectures,
Toronto Notes, Therapeutic Choices and others. Edited and organized for the sake of all attendances of the
Canadian Osce Exams: NAC OSCE and MCCQE2. by: Dr. Merlyn D’Souza and Dr. Zeev Gross, Spring 2011. Page 4
Ever felt the need to Cut down on drinking?
Screening for Ever felt Annoyed at criticism of your drinking?
CAGE alcoholism Ever feel Guilty about your drinking?
Ever need a drink first thing in the morning (Eye opener)
Causes to high Methanol; Uremia; DKA; Paraldehyde; Isopropyl; Lactate;
MUD PILES AG Met. Ac. Ethanol; Salicylates

Causes of Non Hyperalimintation; Acetazolamide; RTA; Diarrhea; Ureteroentric


HARD UP AG Met. Ac. fistula; Pancreaticodudenal fistula

Autoimmune hepatitis; Hepattis B; Hepatitis C; Drugs or toxins;


Causes of AST Ethanol; Fatty liver; Growths (i.e. tumors); Hemodynamic disorders
ABCDEFGHI-M and ALT raise (CHF); Iron (hemochromatosis), copper (Wilson’s disease) or alph1-
antitrypsin deficiency; Muscle injury

Please note that this is only a draft version based on several sources, including: Dr. Basel Mohasen’s lectures,
Toronto Notes, Therapeutic Choices and others. Edited and organized for the sake of all attendances of the
Canadian Osce Exams: NAC OSCE and MCCQE2. by: Dr. Merlyn D’Souza and Dr. Zeev Gross, Spring 2011. Page 5
WARFARIN COUNSELLING
40 M came to clinic as he was informed by clinic to come as his INR result was 1. Next 10 mins
take history & provide counselling
Divide into 2 parts:
1. History ------> 5 mins
2. Counselling ------> 5mins

General scheme:
1. Event
2. Symptoms at the time of prescription
3. Compliance
4. Risk of bleeding from other sites
5. R/O relapse of DVT
6. Drugs and diet that interfere with warfarrin: Grapefruit, Antibiotics, NSAIDs, Antifungals,

Restart INR (fresh person)


X3 dose of 10mg/d  change INR every day (change the warfarrin accordingly)
Check three time a week the INR if three consecutive are in the goal range  once aweek 
than for every two weeks check once  than every month (or depend on the condition)
If INR>10 or patient is bleeding  Vit. K (if active bleeding give FFP according to setting)

Greeting: Good afternoon Mr.Hendricks,I’m Dr.X with you & will be your physician for today.
As I understand, you’re here to discuss your blood reports.
1. Why was the blood test done?
[Pt had DVT x 5 wks ago,& was having regular checkups till last week when he decided to
stop as he’d read some alarming information on the internet & did not like warfarin (or
other scenario,his friend who was on warfarin had a stroke)]
2. When was DVT Diagnosed?
3. How was it diagnosed?
4. What was done??
5. WAS HE TREATED AS AN OUTPATIENT OR WAS HE ADMITTED?/If Yes: How many days?
6. What were the symptoms at that time?
7. Was there pain & swelling?
8. Was there SOB (lung involvement)
DO NOT LOOK FOR FACTORS THAT CAUSE DVT
9. Which medicines were you treated with? -----> Blopd thinners/Warfarin?
10. Is INR done on a regular basis?
11. What was the last time it ws done?
12. What was the target?
13. What was the level?

Please note that this is only a draft version based on several sources, including: Dr. Basel Mohasen’s lectures,
Toronto Notes, Therapeutic Choices and others. Edited and organized for the sake of all attendances of the
Canadian Osce Exams: NAC OSCE and MCCQE2. by: Dr. Merlyn D’Souza and Dr. Zeev Gross, Spring 2011. Page 6
NOW BREAK THE NEWS
Your INR is ONE; do you know the reason why?
IF Pt vague, give him options:
1...Do you take your medications on a regular basis?
1. Do you take your meds by yourself or do you need help?
2. Any chance you were skipping a dose?
3. Did you start any new medications or antibiotics?
4. Diet: Are you eating a lot of spinach?
5. Any Vit K supplements?
[If Pt expresses concern about bleeding S/e:Adress it & say it is a reasonable enough concern.
I’ve to ensure that you do not have any bleeding at that time.
Did you notice any blood from your gums,nose,bruises in body,coughing up blood?
Neuro Sx:.....
Since you stopped the meds, I want to ensure that there is no Relapse of your DVT:
Do you have: Swelling/Calf pain/SOB/Heart racing/Chest tightness?
2 Qns about PMH:
H/o long term illness or surgery

FH

COUNSELLING:
What is your understanding of DVT?....clot
Why did it occur? .......
The concern about this clot is that if not treated, there is a chance of relapse, or it may recur &
this chance is: 8%
To decrease this chance to 0.8% we use warfarin
If DVT occurs more than twice – take life time medication.

If not convinced: In addition to local recurrence there is damage to veins in the legs& valves& if
this happens more than once warfarin has to be taken for a longer period
In addition these clots formed in your legs may dislodge & travel all the way to your heart,&
This is serious. If large, can cause, sudden death. Can travel to lungs & can cause a condition
called PE which again is a very serious condition

Of course the main side effect is bleeding which is very rare if properly monitored. As long as
INR is in normal limits chances of bleeding are minimal i.e: 1%
We’ve to restart with Heparin & warfarin & monitor INR on a daily basis

Back to Content

Please note that this is only a draft version based on several sources, including: Dr. Basel Mohasen’s lectures,
Toronto Notes, Therapeutic Choices and others. Edited and organized for the sake of all attendances of the
Canadian Osce Exams: NAC OSCE and MCCQE2. by: Dr. Merlyn D’Souza and Dr. Zeev Gross, Spring 2011. Page 7
EPILEPSY COUNSELLING

Young 16 yr old male for driving License counsel


Always ask Qn as to why he wants a driver’s license. Usually a Dr does not give such a note
unless there is an underlying condition.

General scheme:
1. Intro
2. Event – before in and after the attack. When was the drug level checked?
Any other medications that might interfere with epileptic drugs (e.g. OCPs).
3. If it is only seizure  go to secondary causes of seizures  refer to neurology
4. Which medication, and compliance
5. HEADDSSS –
6. Triggers – sleep deprivation
7. MOAPS
8. Counsel: needed to be seizure free for one year. Invite him again for f/u after one year.
Risk behaviour: drivint, swimming, hicking, bath door open and don’t take bath but can take
shower, no heavy machines
You have to take it for your whole life – if you have any attack let me know and we’ll discuss
it. Talk with your physician about any new medication you want to take.
Valproic acid 500mg.
OCD:
1. Age of onset
2. When was the Ds
3. What was the Ds
4. How long does each attack last?
5. How frequently do the attacks occur?
6. +/- LOC
7. Aura prior to attack
8. How does she feel after the attack?
9. What meds is she on/Is she compliant/Were the drug levels checked?
10. Any other meds (if female ask about OCs)
11. When was last attack?
12. What happens during an attack? Does she shake/All over/Partly/roll up her eyes/bite her
tongue?

I’ve to ask questions to R/o any new pathology:

Please note that this is only a draft version based on several sources, including: Dr. Basel Mohasen’s lectures,
Toronto Notes, Therapeutic Choices and others. Edited and organized for the sake of all attendances of the
Canadian Osce Exams: NAC OSCE and MCCQE2. by: Dr. Merlyn D’Souza and Dr. Zeev Gross, Spring 2011. Page 8
System review:
CNS: Head trauma/HA/Vi
CSx:
MOOD: Any chance you may hurt yourself?
PMH: h/o Dm
HEAADDS
HOME: With whom do you live/How is your relationship with parents/siblings
EDUCATION: How is school? How’re your grades? Any recent change in grades?
ACTIVITIES: what are your hobbies?
ALCOHOL: Sometimes kids your age might smoke or take alcohol & experiment with drugs, any
of your friends do it? How about you?
If YES: How much/How often?
DIET: How is your diet?
DATING: Are you dating? Are you sexually active?
STIMULANT USE:
STRESS:
SLEEP: Do you have enough sleep?
MAKE SURE that he knows what a seizure is
What do you know about epilepsy?
It is a common condition due to increased electrical activity in the brain, some people lose
consciousness, and some do not. It does not cause learning disability or damage the brain
In those who have seizure attacks:
If lasts for a few minutes there is no brain damage
If lasts for > 30 minutes, will cause brain damage

People with epilepsy should AVOID dangerous activities such as:


1. Driving
2. Mountain climbing
3. Swimming
4. Operate machinery
5. Boating
6. Chewing gum
7. Tub baths (have a shower bath & never lock bathroom door)
You can have a driver’s licence only after you’ve been seizure free for 1 year
It is my duty as your physician to inform the Ministry of transportation

Please note that this is only a draft version based on several sources, including: Dr. Basel Mohasen’s lectures,
Toronto Notes, Therapeutic Choices and others. Edited and organized for the sake of all attendances of the
Canadian Osce Exams: NAC OSCE and MCCQE2. by: Dr. Merlyn D’Souza and Dr. Zeev Gross, Spring 2011. Page 9
Mention TRIGGERS
 If you drink alcohol, it decreases the point at which ea seizure occurs and can cause an
attack
 Sleep deprivation also can cause it
 So also flashing lights

If you want to take any other meds, speak to your Dr


I will check the blood levels of your medications to see if it is at the therapeutic level
If you want to get pregnant consult your Dr
There are support groups
Regular F/u
Any Concerns?

Back to Content

Please note that this is only a draft version based on several sources, including: Dr. Basel Mohasen’s lectures,
Toronto Notes, Therapeutic Choices and others. Edited and organized for the sake of all attendances of the
Canadian Osce Exams: NAC OSCE and MCCQE2. by: Dr. Merlyn D’Souza and Dr. Zeev Gross, Spring 2011. Page 10
OCP COUNSELLING
21 F for OCPs Counsell x 10 mins

General scheme:
Intro:
Good morning xxx,I’m Dr...... As I understand,you’re here today because you want a
prescription for Birth Control pills.
During the next few minutes, I will ask some questions that will help me
2 Questions here:
1. Have you ever used any form of contraception before ?
2. Why do you want to use it?
2.1. If in stable relationship
2.2. If sexually active
2.3. Do you practise safe sex?
2.4. How do you feel about this relationship?
2.5. Prior to this were you in any other relationship?
2.6. Whose idea was it/ Yours or His?

MGOS
MENSES:
MENSES Use the word ―period‖
1. When was your last period?
2. Are your periods regular / not
3. How often?
4. How many days or How long does it last?
5. How many pads do you use/change?
6. Are the pads full?
7. Are they heavy?
8. Do you see clots?
9. Between periods do you have spotting?
10. From your last menstrual period was your period different from the current one?
11. At what age did you start your periods?
12. Were they regular/irregular?
13. When did it become regular?
14. Are your periods painful / painless?
15. If irregular from beginning?
16. Discharge – ask if pregnant and when LMP
GYENECOLOGY
1. Any history of Gyn. Disease – polyps or cysts
2. History of pelvic surgery (if yes – when?)
3. Have you used any birth control?
4. When/type/any complications
Please note that this is only a draft version based on several sources, including: Dr. Basel Mohasen’s lectures,
Toronto Notes, Therapeutic Choices and others. Edited and organized for the sake of all attendances of the
Canadian Osce Exams: NAC OSCE and MCCQE2. by: Dr. Merlyn D’Souza and Dr. Zeev Gross, Spring 2011. Page 11
5. Pap’s smear
OBSTETRICS:
Have you ever been pregnant?
Have you ever had an abortion or miscarriage?
SEXUAL HISTORY:
Any STIs?
Any PIDs?
Any partner with STI?

CONTRAINDICATIONS:
To find out if you’re a suitable candidate,I need to ask a few more questions:
ABCD (Active liver disease, Bleeding, Cancer, DVT)
1. Any abnormal vaginal bleeding?
2. Any active liver disease: (Ac & Ch)
3. CVS:Have you ever had clots in your calves/DVT/Very High blood pressure
4. H/o Migraine headaches?
5. FH of Ca breast/Uterine or Liver

AGREE to give if No CI

EXPLAIN what are OCs


1. Combination of hormones Estrogen & Progesterone or sometimes only a progesterone
2. These come in packs of 21 or 28 pills
3. They prevent pregnancy by interfering with hormone signals in our body & prevent ovulation
4. Also make the inner lining of the womb & makes it hostile for conception
5. Thickens secretions at the mouth of the uterus & prevents conception

MISSED PILL;
To be taken at same time every day, so chances of forgetting is less & constant blood levels
1St pill on 1st Sunday of period, or 1st day of periods
1st month use back up method of Cx like a condom
In first 2 weeks:
If miss one pill: Take 2 pills next day & use condom x 1 week
If miss 2 pills: Take 2 pills same day + 2 pills day after + Condom x 7 days
If miss 3 pills: Stop,use condom & restart new pack
In 3rd week:
If you miss any pills restart new pack

Explain BENEFITS:
1. Help regulate cycle if periods are irregular
Please note that this is only a draft version based on several sources, including: Dr. Basel Mohasen’s lectures,
Toronto Notes, Therapeutic Choices and others. Edited and organized for the sake of all attendances of the
Canadian Osce Exams: NAC OSCE and MCCQE2. by: Dr. Merlyn D’Souza and Dr. Zeev Gross, Spring 2011. Page 12
2. Will eliminate pain
3. Less blood loss during periods
4. Less chances of benign breast disease & ovarian Ca
But like any other medications, there are also the SIDE EFFECTS:
 MILD
N/V,Wt gain (5lbs)breast heaviness,mood changes,Spotting may occur in the initial months
If these occur,you can change brand
 SEVERE;
Severe Ha/SOB Chest pain -----> If these occur STOP the pill & sek urgent medical attentiomn

DRUG INTERACTIONS:
If takes any other medications,let her Dr know she s on the pill

SAFE Sex:

PAP’s Smear
If Teenager: HEAADDS

Last any Concerns or qns?

Back to Content

Please note that this is only a draft version based on several sources, including: Dr. Basel Mohasen’s lectures,
Toronto Notes, Therapeutic Choices and others. Edited and organized for the sake of all attendances of the
Canadian Osce Exams: NAC OSCE and MCCQE2. by: Dr. Merlyn D’Souza and Dr. Zeev Gross, Spring 2011. Page 13
ABORTION
1. Young woman 19 yrs asking for abortion x 10 mins counsel
2. Can be a teenager with a vague complain
a. Read body language & assure Confidentiality
b. When did sexual contact occur?
c. Who was the partner?
d. Was she raped or was it against her will?
e. Is she being regularly abused?
f. Do her parents know?
M (Signs of pregnancy: engorgement of breast, urine frequency, n&v)
O
G
S
PMH
SHx:
HEADDSSS:
Home enviorenment & parental attitude
MOOD & Interest SUICIDAL IDEATION NOW?

When pt tells outright she wants an abortion:


How do you feel about having an abortion?
If she says she feels there is no other option:
Explain that there are other options, Do you want to know them?
When did she find out she was pregnant?
Here be sensitive if she found out last night, she is probably in a panic, but if she has known it
for a week, she has thought about it well, & is more decisive.

Before it can be done, I need to ask you some qns


1. How did she find out she was pregnant?
2. Was she using any contraception?
3. Has she spoken to her partner/family/friend?
4. Would she like to talk about it?
LMP: How was her LMP, was it similar to her previous LMP? Or was it shorter, lighter?
Is there Nx/V,visiting washroom more frequently?
Breast engorgement?
O
Have you been pregnant before?
Have you ever had an abortion/miscarriage?
G
Has she ever used any kind of birth control before?
Any Pelvic surgeries
Any Pap smear (depending on age)
H/O STIs

Since it is the first time I’m seeing you, I need to ask about
PMH;
Please note that this is only a draft version based on several sources, including: Dr. Basel Mohasen’s lectures,
Toronto Notes, Therapeutic Choices and others. Edited and organized for the sake of all attendances of the
Canadian Osce Exams: NAC OSCE and MCCQE2. by: Dr. Merlyn D’Souza and Dr. Zeev Gross, Spring 2011. Page 14
Any H/o HTN/Liver disease/DM?
Surgeries/Anaesthesia complications
Blood Group
Any Medications/Allergies

SOCIAL Hx:
With whom do you live?
How do you support yourself financially?
If young teen: HEAADDS

Whatever you choose to decide, I will support you. Is she decides to go in for an abortion:
I will refer you to an abortion clinic
However it is difficult to get an obstetrician who will do it after 20 weeks
She has to make a decision fast
Also here ask about her own support system (family/boyfriend)
I will also get you connected with a support group, who are women who’ve had abortions
before & will help you cope with it.

Now in addition to abortion there are OTHER OPTIONS:


Would you like me to tell you about them?
1. If your concern is financial, you can carry on this pregnancy & there are a lot of support
groups as well as the government who will help you.
2. You can carry on this pregnancy & give up the baby for adoption, a lot of people are looking
for a child also nowadays you can have visitation rights in certain cases.

If Pt is still going for an abortion:


For now, I will do
1. PAPs test
2. Blood tests: Sr B HCG & Blood group & Rh typing
3. USound
Once your pregnancy has been confirmed by us, I will send you to the abortion clinic
From now, until the time you’ve the abortion, you’ve to;
Quit Smoking/Alcohol/Drugs
If you happen to see any dr during this time period, you’ve to inform him you’re pregnant

Back to Content

Please note that this is only a draft version based on several sources, including: Dr. Basel Mohasen’s lectures,
Toronto Notes, Therapeutic Choices and others. Edited and organized for the sake of all attendances of the
Canadian Osce Exams: NAC OSCE and MCCQE2. by: Dr. Merlyn D’Souza and Dr. Zeev Gross, Spring 2011. Page 15
Smoking Cessation
According to the type of patients we will allocate the time:
Neutral: Hx (4m), Why (3m), How (3m)
Willing: Hx (4m), Why (1m), How (5m)
Unwilling: Hx (4m), Why (5m), How (1m)

General scheme:
1. Intro (“Very good decision”)
2. Hx
2.1. Impact: breathing, coughing, phlegm
2.2. RF: HTN, DM, Hyperlipidemia,
Questions about target organs: heart, lung
2.3. Gain from quitting: what do you think you are going to gain from quitting
2.4. Hx from previous quit – what support do you need? What led to relapse?
Withdrawal symptoms? What is the longest time you quitted?
2.5. In which situations you need to smoke?
2.6. SHx: do you smoke in front of your children?
3. Counseling
What is your motivation to stop smoking (scale 1 to 10)
Different people from different reasons…what is the reason for your smoking?
3.1. Why
3.1.1. Effects of the smoking on different of the body
3.1.2. Reduced risk for diseases – time frame
3.1.3. Influence on other household
3.1.4. Economical effect
3.2. How
3.2.1. Set a quit date within 2 weeks – reduce gradually within 2 weeks
3.2.2. The support you’ll need – tell your family. Found someone who wants to quit.
3.2.3. Diary
3.2.4. Exercise, healthy diet
3.2.5. Things you can do instead of smoking
3.2.6. If taking nicoting replacement – stop smoking.
3.2.7. Medication: Ziban (bupropion) 150mg (only in the morning for three days and
than increase to bid to 7-12 weekly up to 6months)
S/E Insomnia and dry mouth
C/I seizure, eating disorder, MAOI;
Varenicline (Champex)
S/E nausea
C/I Previous psychiatric conditions

Please note that this is only a draft version based on several sources, including: Dr. Basel Mohasen’s lectures,
Toronto Notes, Therapeutic Choices and others. Edited and organized for the sake of all attendances of the
Canadian Osce Exams: NAC OSCE and MCCQE2. by: Dr. Merlyn D’Souza and Dr. Zeev Gross, Spring 2011. Page 16
Intro
Hello Mr./Ms. …..
As I understand you are here today to seek help to quit smoking. I am really happy to hear that –
can you tell me what made you come to that decision?
What are your expectations from this visit?
Motivation can be assessed by asking the following two questions:
1. “Given everything going on in your life right now, on a scale of 1 to 10, where 10 is the most
important thing to do right now, how important is it for you to quit smoking altogether?”
2. “Given everything going on in your life right now, on a scale of 1 to 10, where 10 is the most
confident you have felt about anything, how confident do you feel you will be able to quit
smoking altogether?”

Ask about the smoking now – how long, how much, since when
Impact of smoking of his life: breathing and coughing, weakness, relationship and sex, CSx
RF: HTn, DM, Cholesterol, FHx of CAD and Cancers,
In your opinion - what are the good things you will gain from quitiing?
Have you tried to quit before?
What stopped you from quitting before?
What support will you need in order to quit?
What are the situations in which you usually smoke?

Please note that this is only a draft version based on several sources, including: Dr. Basel Mohasen’s lectures,
Toronto Notes, Therapeutic Choices and others. Edited and organized for the sake of all attendances of the
Canadian Osce Exams: NAC OSCE and MCCQE2. by: Dr. Merlyn D’Souza and Dr. Zeev Gross, Spring 2011. Page 17
Please note that this is only a draft version based on several sources, including: Dr. Basel Mohasen’s lectures,
Toronto Notes, Therapeutic Choices and others. Edited and organized for the sake of all attendances of the
Canadian Osce Exams: NAC OSCE and MCCQE2. by: Dr. Merlyn D’Souza and Dr. Zeev Gross, Spring 2011. Page 18
Smoking Hx
What is the reason that made you decide to smoke?
How much you smoke, how long (More than 10pk/y -

Withdrwal symptoms:
Heart racing, sweating, shakiness, Irritable

What is the longest time you quit


smoking? (every time you quit you have a
better chance of success.)

Sleeping pills;
Please note that this is only a draft version based on several sources, including: Dr. Basel Mohasen’s lectures,
Toronto Notes, Therapeutic Choices and others. Edited and organized for the sake of all attendances of the
Canadian Osce Exams: NAC OSCE and MCCQE2. by: Dr. Merlyn D’Souza and Dr. Zeev Gross, Spring 2011. Page 19
Because you have been smoking for long time I’d like to see how this smoking has affected your
health:

Target organs: Heart, Lung, GI, PVD, Sexual,


CHx
MOAPS
FHx (also addiction, cancer, suicide, depression)
SHx
Do you smoke in front of your children?

Counselling
Why
Different people smoke from different reasons - what is the reason you smoke?
In your opinion – what are the advantages of smoking?
Do you know what the active components in cigarettes are? (It is Nicotine, and when you
smoke it you have a sense of well being. To maintain the same effect you keep increasing the
number of cigarettes and by that tolerance develops. So, when you stop you get withdrawal
symptoms, and therefore it is habit forming and difficult to quit. In addition to the effect on our
brain it causes narrowing of our blood vessels all over the body.
In the heart it causes heart attacks which are leading cause of death in our society.
In the brain it causes stroke which is the third leading cause of death.
In the GI it causes peptic ulcers.
It can cause erectile dysfunction.
In addition to nicotine, cigarettes can contain few thousands of other substances – some of
these affect the lungs and cause COPD which is an irreversible condition which there is no
treatment.
On top of that, smoking is associate with cancer in a lot of different organs of which lung cancer
is the leading cause of death from cancer worldwide.
In addition to medical impact it affects also family members and expose them to most of the
harmful effects mentioned previously.
It is expensive, staining and smelling, increases hazards (fire).
If you quit smoking you are taking the right step and can expect to gain the following:
 After 1 year, the risk of coronary heart disease is cut in half
 After 5 years, the risk of stroke falls to the same as a non-smoker
 After 10 years, the risk of lung cancer is cut in half and the risk of other cancers
decreases significantly
 After 15 years, the risk of coronary heart disease drops, usually to the level of a non-
smoker

Please note that this is only a draft version based on several sources, including: Dr. Basel Mohasen’s lectures,
Toronto Notes, Therapeutic Choices and others. Edited and organized for the sake of all attendances of the
Canadian Osce Exams: NAC OSCE and MCCQE2. by: Dr. Merlyn D’Souza and Dr. Zeev Gross, Spring 2011. Page 20
How
1. Set a quit date. It should be within the next two weeks. Avoid a time when you will be under
stress.
2. Think about why you want to quit and all the good things that you expect as a result of
quitting.
3. If you have tried to quit before, you have probably learned some valuable tips of what not to
do this time. Think about what was most difficult last time and why you gave up trying.
Think about the things you need to avoid this time.
4. Decide what kind of support will be most helpful over the next six months to a year. For
example, you can join a smoking cessation group or plan to meet regularly with a health
professional (such as a pharmacist, nurse or doctor).
5. Tell your family and friends that you are quitting. Ask them to help you to stick to your plan.
If they smoke, ask them to respect your decision to quit and to not smoke in front of you.
Think of things you can do to avoid smoking while with them.
6. Find someone you know who does not smoke and ask them to help you to quit.
7. Make a diary for a few days to keep track of when and why you smoke.
8. Think of ways to avoid situations when you usually smoke.
9. Buy a brand you don’t like. Buy one pack at a time. Increase the time of lighting it, and
smoke only part of it.
10. Think of things you can do instead of smoking (for example, chewing gum, sipping water,
holding a fake cigarette).
11. Most people gain weight while quitting. You can avoid this by healthy eating and increased
activity. Keep healthy snacks around for times when you get the urge to nibble.
12. Keep busy with healthy activities like walking or an exercise program. Starting a new
activity will help to break old habits connected with smoking.
13. If you are taking medication to help you to quit, be sure to follow the instructions carefully.
14. If you are taking nicotine replacement therapy, do not continue smoking, as this is very
dangerous to your health.

Please note that this is only a draft version based on several sources, including: Dr. Basel Mohasen’s lectures,
Toronto Notes, Therapeutic Choices and others. Edited and organized for the sake of all attendances of the
Canadian Osce Exams: NAC OSCE and MCCQE2. by: Dr. Merlyn D’Souza and Dr. Zeev Gross, Spring 2011. Page 21
Back to Content

Please note that this is only a draft version based on several sources, including: Dr. Basel Mohasen’s lectures,
Toronto Notes, Therapeutic Choices and others. Edited and organized for the sake of all attendances of the
Canadian Osce Exams: NAC OSCE and MCCQE2. by: Dr. Merlyn D’Souza and Dr. Zeev Gross, Spring 2011. Page 22
OBESITY COUNSELLING

38 YOM 6 ft height,weight=260Ib 10 min.HX. and Education


Keys:
 Explore motivations for wt.loss.
 Provide information about the consequences of obesity (health and psychological well-
being) and nutrition.
 Set realistic goals, target BMI
 Offer support/reinforcement throughout the weight-losing process.
Hx
DIET & EXERCISE:
Motivation for losing wt. now?
Self –image,
Health concerns? ………
Can you tell me more.
Good decision to come today, I am glad you came.
If patient request first surgery, tell him that sound reasonable however, surgery is not the first
line can be done based on BMI if >40
If you like I can calculate it for you; kg/m2.

I need > information about your condition:


WEIGHT:
1. What is your weight today?
2. Highest weight.
3. When started to gain wt?
4. When started to be concerned?
5. Have you tried any wt- loss programs?
6. Which one?
7. How long?
8. Did you lose wt?
9. Why did you stop?

I am going to ask you Qs to see


WHY YOU’RE GAINING WEIGHT:
Do you calculate your calorie intake?
How many meals do you take/day including snacks?
Tell me more about your DIET:
 What do you eat?
 How much fat, fruit, veg bread?
Eat while watching TV,
Before bed,

Please note that this is only a draft version based on several sources, including: Dr. Basel Mohasen’s lectures,
Toronto Notes, Therapeutic Choices and others. Edited and organized for the sake of all attendances of the
Canadian Osce Exams: NAC OSCE and MCCQE2. by: Dr. Merlyn D’Souza and Dr. Zeev Gross, Spring 2011. Page 23
Breakfast daily,
Ever eat to relax or when stressed?
Binge eating?
Do you feel guilty about your eating?
Do you induce vomiting/purging?
ALCOHOL
How about your ACTIVITY,
Do you exercise?

IMPACT:
I am going to ask you how this Affects your life?
1. Difficulty sleeping,
2. Tiredness,
3. Heart burn,
4. Nausea, vomiting,
5. GB stones, bowel motion,
6. Back pain,
7. Jt pain.
PMH: HTN, DM,
Medications: anti-psychotic, OTC, steroids, thyroid disease, OCP.
Social Hx: With whom do you live? Any change in sexual desire?
How it affects your self esteem,mood and interest?
Do you smoke? Drink? Recreational drugs?
FHx : obesity
Education:
There are some genetic factors that influence wt. We can’t modify these but we can modify our diet and
exercise. In some people, diseases are the underlying cause for obesity.
Give patient their ideal wt. for ht. >20% ideal wt is obesity.
Being overwt increases the risk of
Hypertension,CVD,CAD,GB disease,DM,fatty liver,cancers(breast,bowel),OA,sleep apnea,spinal
dysfunction.
 We recommend to lose 10% of your body wt.over 6 months (gradually).guidance is BMI
There are 2 methods to lose wt: Decrease intake or Exercising more.
If you like I can refer you to a dietician.
We also recommend dividing your meals into 3 small and in between snacks ( carrot, veg.or fruit)
55% CHO, 15% protein,30% fat
Avoid saturated fat, cheese, alcohol
Give patient a target caloric intake:
to lose 1 Ib/week,should take 300-500 kcal less
1g fat-9kcal, 1g CHO-4kcal, 1g protein-4kcal
Do not recommend diet medications and fad diet, these may be harmful and are of no long-term
benefit.
If BMI>27 + RF (DM, Htn...) or BMI > 30 start pharmacotherapy:
Xenical=increase bulk of stools, leakage, decrease absorption of fat sol.Vit.
Meridia (sibutramine) = suppress appetite, cause heart racing, hypertension.
IF BMI>35 + RF or BMI > 40 recommend Baratric srgery

Please note that this is only a draft version based on several sources, including: Dr. Basel Mohasen’s lectures,
Toronto Notes, Therapeutic Choices and others. Edited and organized for the sake of all attendances of the
Canadian Osce Exams: NAC OSCE and MCCQE2. by: Dr. Merlyn D’Souza and Dr. Zeev Gross, Spring 2011. Page 24
Exercise:begin with walking,regularly 30 min,4-5 times per week
Reach 60-80% maximum heart rate (220-age)
Self-monitoring, group support
Follow-up: advise patient to come back in a week with food intake diary etting sick

Please note that this is only a draft version based on several sources, including: Dr. Basel Mohasen’s lectures,
Toronto Notes, Therapeutic Choices and others. Edited and organized for the sake of all attendances of the
Canadian Osce Exams: NAC OSCE and MCCQE2. by: Dr. Merlyn D’Souza and Dr. Zeev Gross, Spring 2011. Page 25
Back to Content
70 yr old female with H/o fall at home .Brought in by ambulance
personell to the ER.
She is medicaly cleared;
In the next 20 minutes take history & Counsel;
Diff/Diag (Dd):
1.Poly pharmacy
2. Recent hypovolemeia
 Diarrhea/Vomiting
 Lack of intake
 Recent bleeding
3.Orthostatic hypotension
4.Hypoglycemia
5.Elder abuse
FALL:
1. When did the fall occur?
2. Where did it occur?
3. Were you alone?
4. Could you get up by yourself or did you need help?
5. How long before you got help?
6. Did you trip or just feel your legs give way?
7. If there was a witness around ask permission to speak to witness after you finish talking
to Pt to obtain collateral history

3 parts of history relating to the EVENT:


A.Before fall
B.During the Fall
C.After the fall

Events assoc with the Fall:


A.Before:
1. Did you feel
2. Light headed/ Spinning/ Hungry/ Heart racing & Sweating --- HYPOGLYCEMIA
3. Chest pain/ Palpitations/ Shortness of breath----CVS
4. Lights flashing/ Strange smell/ Strange feeling in body--Seizure
5. Weakness/Numbness/Dificulty finding words/Visual disturbances --CNS/STROKE
6. Was the lighting good?
7. How is your vision
8. Is your footwear comfortable?

Please note that this is only a draft version based on several sources, including: Dr. Basel Mohasen’s lectures,
Toronto Notes, Therapeutic Choices and others. Edited and organized for the sake of all attendances of the
Canadian Osce Exams: NAC OSCE and MCCQE2. by: Dr. Merlyn D’Souza and Dr. Zeev Gross, Spring 2011. Page 26
B.During the Fall:

1. Did your wife mention that you were shaking or making jerky movts?
2. Did you wet yourself?
3. Turn blue & were stiff?
4. Bite your tongue?

C.After the fall:


1. Nausea/Vomiting
2. Weaknes
3. Difficulty finding words
4. Any vision difficulties
5. Loss of sensation in the arms or legs
6. Ringing in ears

Has this ever happened before


1. When & where
2. Did you seek medical help then
3. What were you advised?

CONSTITUTIONAL SYMPTOMS:
 Fever & Chills & Night sweats
 Wt loss & Loss of appetite
 Lumps & Bumps
Sx related to CVS:
Chest pain/SOB/Palpitations
Sx of CNS:
Weakness/Numbness/Loss of vision/LOC
Past Medical History;
1. Are you taking any medications?
2. Can you take them by yourself or does your caregiver give them to you?
3. Do you take them regularly as prescribed?
4. Can I see them please?
Please see the meds
Was there a recent change in the meds
5. Besides these do you take any additional OTC products or herbal medications?
6. Do you take alcohol? ..............
 How much do you take regularly?
 Did you take alcohol prior to the fall?
7. Do you have high blood pressure?
 When was it last checked?

Please note that this is only a draft version based on several sources, including: Dr. Basel Mohasen’s lectures,
Toronto Notes, Therapeutic Choices and others. Edited and organized for the sake of all attendances of the
Canadian Osce Exams: NAC OSCE and MCCQE2. by: Dr. Merlyn D’Souza and Dr. Zeev Gross, Spring 2011. Page 27
 What did your doctor have to say about it?
8. Do you have high blood sugar or Diabetes?
 When was it last checked?
 What did your doctor have to say about it?

9. Did you ever have a stroke or heart attack?


10. Were you ever diagnosed with Cancer
11. Were you hospitalized at any time in your life?

I need to ask a few more questions concerning your lifestyle that will aid me to help you.
It is all confidential & my duty is to help you (When you suspect Elder abuse)
1. With whom do you live?
2. Are you happy living with XXXXX
3. Who prepares your meals?
4. Do you do your own shopping?
5. Do you manage your own finances?
6. Do you go out of the house & meet up with friends & have your own social life?
7. Do you get into arguments with XXXX?
8. Have you ever been hit or yelled at or threatened by XXXX?

COUNSELLING FOR POLYPHARMACY (Orthostatic Hypotension)

Based on what you’ve told me most likely the reason of your fall is a condition called
“Orthoststic Hypotension”.Have you ever heard about it?..........
When you change position from lying to sitting or standing blood pools to the legs & Bld vessels
narrow to maintain BP.
In pts with OH because of Age,Medications,DM or a combination of these condts body might
fail to react,& blood pools in the legs & thus BP drops & there is not enough bld reaching the
brain.
There is a possibility that this might happen again & from now on whenever you change your
posn from lying get up slowly,sit at edge of bed & slowly get up.
I need to get in touch with your doctor & modify the dosage of your meds or change them.
Is it alright with you?
I need to talk now to your wife & do an ECG to check your heart

Back to Content

Please note that this is only a draft version based on several sources, including: Dr. Basel Mohasen’s lectures,
Toronto Notes, Therapeutic Choices and others. Edited and organized for the sake of all attendances of the
Canadian Osce Exams: NAC OSCE and MCCQE2. by: Dr. Merlyn D’Souza and Dr. Zeev Gross, Spring 2011. Page 28
HA DOMESTIC VIOLENCE

Domestic Violence common presentations:


1. HA
2. Abd Pain
3. Ac Abd
4. Insomnia
5. Sleeping pills
6. Vaginal Bleeding

Sx
1. No good eye contact
2. Vague complaints
3. Non communicative

OCD/PQRST
CONSTITUTIONAL Sx:
R/o Migraine & Tension HA
RISK FCTS:
Smoke/Alcohol/Recreational drugs
PMH:Are you on any meds/OTC/Herbal meds?
Were you hospitalized at any time?
FH:
SOCIAL HISTORY: Important**
All information you give here is entirely confidential & will not be released unless you authorize
it
Who lives with you?
Any recent changes/Stress in your relationship
SCREEN FOR DOMESTIC VILOENCE:
Does your Partner:
1. Hit you?
2. What happens during an argument?
3. when he is angry,does he :
4. Shout/Swear & call you names or demean you?
5. Has your partner ever ridiculed you or cut you off from other relationships with
friends/family?
6. Have you ever sought help from others in health care?
ABUSE RISK FCTS:
1. Drink alcohol,drink more now than before?
2. Does he have access to firearms?
3. Does he ever get angry to the point where he gets physical & hits you?Did you ever have to
go to the ER? Was there a serious consequence?
4. Are you having more arguments now

Please note that this is only a draft version based on several sources, including: Dr. Basel Mohasen’s lectures,
Toronto Notes, Therapeutic Choices and others. Edited and organized for the sake of all attendances of the
Canadian Osce Exams: NAC OSCE and MCCQE2. by: Dr. Merlyn D’Souza and Dr. Zeev Gross, Spring 2011. Page 29
5. Does he get more angry now,& How has all this affected your self esteem?
6. How does it make you feel?
7. Does he ever force you to have sex against your will?
8. Who controls the finances & spending?
9. Has he ever mistreated you in front of the children?- If yes: it is emotional abuseto
children & has to be reported to CAS
10. Has he ever misRxed th children?
11. Have you ever thought of putting an end to your life or his life?
Have you spoken to anybody abt this?
Do you have some support?
COUNSELLING: Empowering & Education
3 kinds of Pt:
1.She wants Help
2.She might Consider getting help
3.She does not want to get help & thinks he is right

I’d like you to know that what you’re experiencing is called “Domestic Violence “or Spousal
abuse. It is a crime against the law & not acceptable.
It is not your fault & you should not accept it & feel guilty
It can get out of hand & you can get harmed seriously
Call Police (Never Call Police from your office)
Contact Social worker, who will help you with housing, finding a job & finances & child support
If she is considering
Escape Plan
Keep a bag with important documents,change of clothes & hide it
DOCUMENT

Fup x 3 days

Back to Content

Please note that this is only a draft version based on several sources, including: Dr. Basel Mohasen’s lectures,
Toronto Notes, Therapeutic Choices and others. Edited and organized for the sake of all attendances of the
Canadian Osce Exams: NAC OSCE and MCCQE2. by: Dr. Merlyn D’Souza and Dr. Zeev Gross, Spring 2011. Page 30
Diabetic Daughter 2y, Counsel
Either she is not doing well in school
Not seeing well
Not playing well, tired
DKA

Is it regular f/u or something special you wanted to discuss?


When was the last f/u?
How was she diagnosed?
What happen then?
What were the symptoms?
Is there any pain / vomiting?
Are you feeling eating/drinking/peeing more?
Is there any weight loss or blurred vision?
From the last f/u till now have you had DKA? How about before?
Have you had low blood sugar?
Talk with the father: which medication does she take? How does she take? When was the last
time?
Do you take insulin or somebody else gave it to you? Do you take it all the time? DO you skip
dose? Does she need any help to take insulin? DO you measure blood sugar regularly? When
was the last time? Do you record them in the machine? (The glucometer should be used by only
one patient).
There is a blood work called ―Hemoglobin A1C‖ it is done every three month – did you do it?
Did you start new medication? How about your diet?
DO you have your log book?
What do you eat?
Have you ever seen by a dietician?
PMHx
FHx

Counseling
A lot of people have diabetes and she is not the only one. What’s your understanding of
diabetes?
Whenever we eat food contains sugar it is absorbed in our stomach and goes to the blood and
from there to different parts of our body. Sugar act in our body like a fuel, in order for our body
to use this energy it needs insulin. Patients having diabetes have not enough insulin. Sugar will
be built up in your blood. The body tries to get rid or it, by peeing extra sugar – this will lead to
thirsty and tiredness.
This can be avoided by controlling the blood sugar. If you control your blood sugar you’ll be
able to play again. If not controlled – may end in DKA, hypoglycaemia and serious
consequences.
Always be aware of hypoglycaemic symptoms: loss of conscious, sweating, heart racing, hungry.
Since you might lose conscious it is important to carry MedAlert Caed or Bracelet which will
clarify your situation.

Please note that this is only a draft version based on several sources, including: Dr. Basel Mohasen’s lectures,
Toronto Notes, Therapeutic Choices and others. Edited and organized for the sake of all attendances of the
Canadian Osce Exams: NAC OSCE and MCCQE2. by: Dr. Merlyn D’Souza and Dr. Zeev Gross, Spring 2011. Page 31
Medical Error, Wrong blood transfused
When there is a mistake, always there is a kind of unintentional medical error.
(to the nurse) when informed about wrong blood – ask: ―did you stop the blood?‖ say: ―Well
done!‖ If she asks not to tell the patient...ask her what her believe she may loose her job, and it is
too early to determine who is responsible. Errors take place in medical practice. We don’t know
what exactly happened. We will stabilize patient and ensure he’s fine and later deal with this
issue.
(to the patient) Intro: I am the doctor in charge, and it looks like it was an unintentional medical
error took place. We need to make sure you are stable. We don’t know who is responsible, there
are at least 15 steps and in each step could have been an error. We will fill an incident report and
as soon as we get result we will inform you. You can sue, it is your right at the moment it is my
priority to stabilize you.
ABCD
A – Open your mouth (check for anaphylaxis, no swelling in mouth, ask for any itchiness, or
difficulty breathing),
Oxygen saturation.
Normal air entry.
Normal S1, S2
Vitals again
Remove blood unit and keep cannula.
Start new IV line.
Once new line, don’t give fluids if stable.
Send blood: CBC, Lytes, INR, PTT, LFT, Cr, BUN, FDP, Haptoglobulin, Direct coombs test;
Urinalysis: hemoglobulinuria
Unit to be sent to blood bank for cross matching.
Ask nurse to call the blood bank and keep original blood.

D
D1 – I’d like to shine a light in your eyes. Pupils are round, active, and symmetrical. Squeeze my
finger, wriggle...wriggle...
D2 – (if febrile) give tylanol
Please prepare for me benedril (Diphenhydramine) 50mg. Steroids (Hydrocortisone) and
Epinephrine

Secondary survey
Hx (two parts:) condition (how is he feeling now) and the other is: ―Why blood was given?‖
Condition: Do you feel warm? Chills? Itchiness? Tinglings? Diffculty breathing? Wheezing?
Swelling in lips / fingers? Hives?
Before transfusion did you have fever?
Check for haemolytic reaction – any back or flank pain?
P/E – no oozing at IV line
Then press on flank and back – no pain for haemolytic reaction.
Is it the first time?
Why did you receive blood?
If received blood before – was there any complications?
Any long term diseases?

Please note that this is only a draft version based on several sources, including: Dr. Basel Mohasen’s lectures,
Toronto Notes, Therapeutic Choices and others. Edited and organized for the sake of all attendances of the
Canadian Osce Exams: NAC OSCE and MCCQE2. by: Dr. Merlyn D’Souza and Dr. Zeev Gross, Spring 2011. Page 32
Counseling
Mr. X what do you know about blood transfusion?
It is a life saving measure, and a lot of measures are taken to make sure it is safe. However, like
any other medication with blood transfusion there could be side effects, and these side effects
could be serious.
The most common side effect is febrile reaction (3%), usually it is self limited and can happen
again. Next time you receive blood we will give you tylanol.
Anaphylactic reaction. It is a severe allergic reaction, and it is very serious and we cannot predict
it. However, we have good measures to deal with it, and your symptoms make it less likely that
you have had an anaphylactic reaction.
The yhird reaction reaction is more serious and called haemolytic reaction. Usually happens
when patients receive blood belonging to another blood group. The fact that this blood is same as
your blood group, and the symptoms are not consistent with haemolytic anemia make it less
likely that this is not the case here. The blood is sent to the blood bank and once results are back
we will get final confirmation, we will able to reassure you.

Back to Content

Please note that this is only a draft version based on several sources, including: Dr. Basel Mohasen’s lectures,
Toronto Notes, Therapeutic Choices and others. Edited and organized for the sake of all attendances of the
Canadian Osce Exams: NAC OSCE and MCCQE2. by: Dr. Merlyn D’Souza and Dr. Zeev Gross, Spring 2011. Page 33
Febrile Seizure
A child brought to the ER because of febrile seizure.
Next 10m counsel him.
He is stable. During the next few minutes I’ll ask you few questions, and after that I’ll go with you to see
him.
You should r/o meningitis. Educate, and what to do next time.
Did you see him? (Started to shake. All over his body? Bite his tongue / roling up his etes / wet himself).
After the seizure does he have any neurologic deficits.
How long did it last, or did you come on your by his own or medcial staff.
Did he stop seizing on his own or after medical interv.
Is it the 1st time?
Ask about fever? (if it started a week ago – did you seek medical assistance? Discharge? Did they give
you any treatment? Did they give it to him or no?)
Why! Some studies show you can treat OM without antibiotics. You should look for the reason not to
give the antibiotics (negligence?). Is he having any vomiting? Skin rash? Coughing? Head to toe...
If you find nothing – ask when he got his last shot? (up to 72 hours he can have fever).
R/O meningitis, pneumonia.
Any family history of febrile seizures, epilepsy
BINDE (especially immunization).

Counseling:
Your child has condition called febrile seizure (FS).
It is a condition that might happen from 6m to 60m. We don’t know exactly why – we believe it is a
sudden change in the temp. This might lead to the seizure. This condition might happen again.
Any time your child has fever – seek medical admition. Give tylanol and sponge to decrease his temp.
Most of the children will outgrow this condition by the 6th year.
They don’t recommend Diazepam because it might make him drowsy.

If it stopped less than 5m or more than 5m including neurological symptoms seek ER immediately.
Brochure.

“This is Dr. ... (immediately should introduce yourself). I am the Dr. In charge in the ER. I am calling that
your child swallow medication. I know you are stressed, I need to take your phone number and address,
and how far it is from the hospital.”
Stay calm. Your son needs you, I am going to give you some instructions and you need to follow them. Is
your son is alert or not? Is he conscious? Can he talk to you? Can he recognize you? If he doesn’t – do
you know how to do CPR and start with that.
He’s crying. What is colour? Pink. Hold him and try to calm and sooth him. If he his conscious – try to
hold him and check his mouth. Is he breathing? We’ll send the ambulance for you. When did it happen?
How long was he alone? Which medications did he take? Do you have the container? (don’t go to the
next room to bring them). Do you know what condition your father have (was it vitamins, sleeping pills,
or any other?) how much the amount? Don’t use any ippecak?
Is it happened before? What is the weight of the child? BINDE (was it full date, did he needed special
attention after term, does he have any special conditions). Weight for two reasons – antidote and
estimate neglect.

Please note that this is only a draft version based on several sources, including: Dr. Basel Mohasen’s lectures,
Toronto Notes, Therapeutic Choices and others. Edited and organized for the sake of all attendances of the
Canadian Osce Exams: NAC OSCE and MCCQE2. by: Dr. Merlyn D’Souza and Dr. Zeev Gross, Spring 2011. Page 34
Post encounter Q: what are the first four steps you do when he arrives? (ABC, Monitor vitals, IV line, NG,
Foley as needed, Blood works – CBC, Lytes, BUN, Cr, Osmolality, Coagul, LFTs, Tox screen – blood and
urine).
List three risk factors for this child.
What is the antidote for betablocker (glucagon) and for CaChannel is (Calcium gluconate).
CAS and Poisoning centre.

Second scenario – while he is seizing just put him on the side, and not start any CPR. Before I proceed I’d
like to take your phone number and address. Is it the same time or happened before. If it is the second
time – more than 15m he needs intervention.
Is he seizing right now? Try to put him on the floor on the left side (the right bronchus is shorter than
the lt.). Observe him. What is his colour? Is he still shaking? You send the ambulance. Can you tap on his
shoulder? If he is not responding – can you do CPR? Can you feel his pulse? Is he alert? Can he talk to
you? Can he move his legs? Was he shaking? Does your child have fever? Did you seek medical
attention? What prevented you from giving the medication?
Post Concussion.
2 scenarios (Osgood schlatter and Post-concussion)
Decision will based whether the child can tolerate pain or not?

#1 About to see the father of 14yom with Osgood Schlatter.


Make sure that the child best interest are preserved. What was done to diagnose the child?
OCD PQRST compare to the other knee, is the first time or not, was any trauma.
What is the child wish? (Don’t go for HEADDSSS since it is the father).

Counseling
What is your understanding of OS.
Let me explain to you what is the mechanism for OS.
Avoid him from playing, especially jumping. But he can continue with ice presses and pain killers. The
rule is that he can continue up to his limit of his pain.

Back to Content

Please note that this is only a draft version based on several sources, including: Dr. Basel Mohasen’s lectures,
Toronto Notes, Therapeutic Choices and others. Edited and organized for the sake of all attendances of the
Canadian Osce Exams: NAC OSCE and MCCQE2. by: Dr. Merlyn D’Souza and Dr. Zeev Gross, Spring 2011. Page 35
54 year old female comes to clinic concerning about using
HRT.
.
When a patient has concern about any subject, address it very soon. Don't wait to the end.
Dr: As far as I understand you're here as you have concern about using HRT.
Patient: yes Dr. I feel I am confused about using HRT.
Always ask what do you mean by HRT. So the patient will tell you how much they known about HRT.
Dr: I'm glad you're here so we can discuss about it and address your concerns and hopefully by the end
of the session you can make a decision regarding using HRT. Or hopefully by doing this discussion you
will have a better understanding of HRT.

Or you can say: I agree with you as there are a lot of confusion about HRT and the reason for this
confusion is that in the past because it was used to be given routinely to all women who reach a certain
age, however 10 years ago there was study called " women health initiative" in which the authors found
that the numbers of the patients with serious side effects are very high. However those ladies used HRT
for a long time.

Serious side effects are


Cancer,
Heart attacks and Strokes.
For that reason the routine use of HRT was stopped.
Nowadays we have a better understanding and have better guidelines.
Not only that we do it on the individualized basis.
We use it only for short time, they don't exceed five years.
So using HRT within five years is safe.
So I would take some information from you and we will discuss about the risk factors and if you are a
good candidates we can make a decision to prescribe it or not.
Dr: What makes you interested in HRT?
Patient: because of hot flushes.
At this stage if the patient gives you the symptom, it is your chief complaint.
But if patient doesn't give you any symptoms, you should start with her LMP
If she starts with the symptom of hot flushes, ask the patient
1. When did hot flushes start,
2. Is it all the time,
3. On & off or continues,
4. How many attacks,
5. Day or night,
6. How do you feel that you have it.
7. Night episodes, you have any night sweating, does it wake you up.
Asked patient if the hot flushes wake her up during the night and if she needs to change her gown of
night’s sweats.

1. Affect your sleep and how does it affect your concentration.


2. Change in your mood, anybody has told you that your short tempered, and if you
3. feel tired.
4. Some women with the same symptoms may notice some change in their sexual life.
a) So the doctor should ask with whom do you live?
b) Are you sexually active?
c) Any dryness or pain during the intercourse?
5. Any change in your urination?
6. Have you ever lost control?
7. Last period?

Please note that this is only a draft version based on several sources, including: Dr. Basel Mohasen’s lectures,
Toronto Notes, Therapeutic Choices and others. Edited and organized for the sake of all attendances of the
Canadian Osce Exams: NAC OSCE and MCCQE2. by: Dr. Merlyn D’Souza and Dr. Zeev Gross, Spring 2011. Page 36
 Are you periods regular or not?
 If it's irregular, when did it start to become irregular?
 Are your periods heavy or not?
 Any clots?
 *Any bleeding or spotting between periods? This is a very important point.
8. Bone pain? Any fractures? Any family history of osteoporosis? If yes, tell the patient that you will
discuss this in another meeting. Because that's another session to discuss about using steroids,
smoking, alcohol, caffeine, warfarin and diet. If she takes calcium supplements.

MGOS for GYN cases: Menstural, Gynecologic diseases, Obstetrics, Sextually transmitted disease

Dr: any history of gynecological disease like polyps, cysts, any pelvic intervention/instrumentation,
surgeries.
Dr: did you use to take any oral contraception? If yes, which one and did you have any side effects?
Also you should ask about her last smear.
Because she is 50+ you should ask about her mammogram.
At any age you ask about Pap smear, once you reach 50 to ask about mammogram and when the patient
pass 65 you should add bone density.
You can ask about her obstetrics history, like have you ever been pregnant if yes how many times you
have been pregnant?
Now use the transition...
Because this is the first time I met you, I would like to ask you about your past medical and social history.
Is there any long-term disease, hospitalization before, any surgery, diabetes, or hypertension. Any history
of allergy, and the medication she takes.
ABCD: Active liver disease, vaginal Bleeding, Cancer, DVT

For A you ask about any history of Active liver disease. Have you ever been yellowish? Any dark urine or
pay stool?

For B you should ask about any vaginal Bleeding? ... You have already asked these question before

For C you should check about Cancer. I would like to ask about constitutional symptoms here to see if
there is any endometrial cancer. Fever, chills, weight loss, appetite, lumps & bumps. A history of cancer in
yourself or family (breast cancer, endometrial cancer,and colon cancer).

For D you should ask about any history of swelling in the legs (DVT), any history of heart attacks,
pulmonary embolism or stroke.

Social history: smoking, taking alcohol, recreational drugs, how does she support financially herself, how
does this affect her life and ask about osteoporosis.

Usually in this set of scenario, you tell her on the basis of the history you are good candidates for HRT.

However as I told you it is an important information to tell you to make your decision.

As we go through different stages of life usually for ladies, we go to the stage called menopause which is
vary between person to person.
At this stage there is hormonal changes and ovaries start to produce less hormones specialty estrogen
and progesterone and that changes affect the whole body. It can explain about dryness, decreasing or
absence of periods. And that's why we try to replace those decreased hormones by HRT. They are the
same hormones but we give it through external sources either tablets or skin patches.

Please note that this is only a draft version based on several sources, including: Dr. Basel Mohasen’s lectures,
Toronto Notes, Therapeutic Choices and others. Edited and organized for the sake of all attendances of the
Canadian Osce Exams: NAC OSCE and MCCQE2. by: Dr. Merlyn D’Souza and Dr. Zeev Gross, Spring 2011. Page 37
As I told you before there is a balance it's your decision to make. And the balance is to use it up to five
years. Using more than five years would increase the risk of stroke, heart attack or some cancers
depending on what we call it estrogen dependent that includes breast and endometrial cancer. And some
studies showed that it might increase the risk of Alzheimer's disease.

So the risk of use for less than five years is not significant and still acceptable. So if you want to use it the
shorter the better.

To get rid off the hot flushes that are other measures like exercise or herbal supplements that you can try
to improve the symptoms.

The HRTs are the same as OCP's but in this smaller doses and you can take one tablet a day. They have
a few side effects like weight gain, bloating, nausea, abdominal distention and pain but they improve by
time.

This serious side effects are headaches, swelling of the legs or chest tightness which whenever happen
you should go to emergency room. By using these HRT's your periods may stop or you may see
spottings.

If the patient had hysterectomy before you only give estrogen without progesterone, otherwise you should
give both.

Because you take it regular shootout regular ultrasound scans to check the thickness of the endometrium

Back to Content

Please note that this is only a draft version based on several sources, including: Dr. Basel Mohasen’s lectures,
Toronto Notes, Therapeutic Choices and others. Edited and organized for the sake of all attendances of the
Canadian Osce Exams: NAC OSCE and MCCQE2. by: Dr. Merlyn D’Souza and Dr. Zeev Gross, Spring 2011. Page 38
CARDIO
1.45 yr old Chest Pain x 45 mins ER History x 5 mins (MI)
Onset:
1. When did it start?
2. What were you doing at that time?
3. How did you get here today?
4. If you came in by Ambulance, did the paramedics give you a tablet to be kept under
your tongue?
Course:
Was it sudden or gradual?

Position:
Where exactly is it hurting you the most?
Quality:
Can you describe the pain? Is it crushing? Knifelike?.......
Radiation:
1. Does it move anywhere else in your body
2. Does it move to the back?
Severity:
On a scale of 1 to 10 where one is minimum & 10 is highest, where would you place this pain?

Associated symptoms:
CVS:
 N/V,Sweating?
 Heart racing?
 SOB/Orthopnoea/PND?
 Have you been under stress recently?
 Cough with blood tainted sputum?
GI
 Acid taste in mouth?
 Heart burn?
 Dysphagia
 Pud?
MSK
 Have you had any trauma to the chest
 Are there any blisters on chest?
RS
 Did you have any flu recently?
 Cough with Phglem?
CONSTITUNIOL Sx
 Do you have night sweats
 Loss of appetite & Loss of wt?
Please note that this is only a draft version based on several sources, including: Dr. Basel Mohasen’s lectures,
Toronto Notes, Therapeutic Choices and others. Edited and organized for the sake of all attendances of the
Canadian Osce Exams: NAC OSCE and MCCQE2. by: Dr. Merlyn D’Souza and Dr. Zeev Gross, Spring 2011. Page 39
Alleviating Fcts:
What makes you feel better?
1. Rest?
2. GTN?
3. Antacid?
4. Sitting forward?
Aggravating Factors:
1. Exercise/exertion?
2. Movements?
3. Deep inspiration?
4. Lying down?
5. Eating?

RISK FCTS:
I need to know additional information that could be related to your pain right now, and need to
ask some further questions......
1. Do you have a high Blood pressure?
 When,& what did your doctor have to say about it?
 Were you put on medicatn?

2. Were you diagnosed at any time with an elevated Blood Sugar or were told you had
diabetes?
 When,& what did your doctor have to say about it?
 Were you put on medication?

3. Have you ever had your cholesterol checked?


If yes:
 When,& what did your doctor have to say about it?
 Were you put on medication?

4. Do you smoke?
If Yes;
How many & Since how long?
5. Do you take alcohol
6. Have you used recreational drugs?
Cocaine?
7. .Do you find time for regular physical activity?
8. Do you eat a lot fast food?
9. In your family has anyone had a heart attack under the age of 50?

CAUSES/Consequences & Symptoms:


1. Did you ever have a stroke?

Please note that this is only a draft version based on several sources, including: Dr. Basel Mohasen’s lectures,
Toronto Notes, Therapeutic Choices and others. Edited and organized for the sake of all attendances of the
Canadian Osce Exams: NAC OSCE and MCCQE2. by: Dr. Merlyn D’Souza and Dr. Zeev Gross, Spring 2011. Page 40
2. Did you have a weakness or numbness?

Past Medical History:


1. Do you take any medications?/OTC or herbal products?
2. Are you allergic to anything?
3. Were you hospitalized at any time?
4. Did you ever undergo any surgery?
5. Were you ever diagnosed with Cancer?
6. Do you have a bleeding disorder?
7. Did you have any head/facial trauma since last 3 mo?

I need some more information about your family


HTN/DM/MI/Stroke

Social History:
1. With whom do you live?
2. How do you support yourself?

MANAGEMENT:

 Rapid, targeted history and physical examination, with particular attention to onset of
symptoms, contraindications to use of thrombolytic agents

Absolute contraindications:
1. Previous intracranial hemorrhage;
2. Known malignant intracranial neoplasm,
3. Known cerebral vascular lesion,
4. Ischemic stroke within 3 mo EXCEPT acute stroke within 3 h;
5. Suspected aortic dissection;
6. Active bleeding or bleeding diathesis (excluding menses);
7. Significant closed head or facial trauma within 3 mo.

Relative contraindications:
1. History of chronic severe, poorly controlled HTN,
2. Severe uncontrolled HTN (BP > 180/110 mm Hg)c;
3. Prior CVA greater than 3 mo or known intracerebral pathology not covered above;
4. Traumatic or prolonged (> 10 min) CPR or
5. Major surgery (< 3 wk);
6. Noncompressible venous punctures;
7. recent (2–4 wk) internal bleeding; pregnancy;
8. active peptic ulcer;
9. current use of anticoagulants.

Please note that this is only a draft version based on several sources, including: Dr. Basel Mohasen’s lectures,
Toronto Notes, Therapeutic Choices and others. Edited and organized for the sake of all attendances of the
Canadian Osce Exams: NAC OSCE and MCCQE2. by: Dr. Merlyn D’Souza and Dr. Zeev Gross, Spring 2011. Page 41
and evidence of high-risk features (tachycardia, hypotension, congestive heart failure)

Management
1. ECG STAT, then every 8 hours for the first 24 hours, then daily for 3 days.
a. In addition, repeat the ECG with each recurrence of chest pain
2. Baseline troponin STAT, (creatine kinase if troponin is unavailable) and then every 8
hours until enzymatic confirmation of the diagnosis
3. CBC to rule out the presence of anemia,
4. Baseline electrolytes,
5. Creatinine,
6. Fasting lipid profile (within 24 hours of presentation)
7. Liver function tests
8. Portable chest x-ray (CXR) STAT
9. Echocardiography to assess LV function after stabilization and treatment.
Echocardiography is also used emergently when there is suspicion of acute mechanical
complications post-MI
Therapeutic Tips
 The goal for thrombolytic treatment is a door-to-needle time of 30 minutes or less.
 The goal for primary PCI is a door-to-dilatation time of 90 minutes or less.
 Careful attention to maximum pain relief is important.
 In patients with right ventricular infarcts:
o avoid nitrates and diuretics
o use fluids and inotropes to treat hypotension
 Administer beta-blockers early to all patients without contraindications. Increase the dose
every 12 hours (every 24 hours for once-daily beta-blockers), if tolerated (monitor blood
pressure and heart rate), until the patient has reached adequate beta-blockade (HR ≤ 55-
65 BPM).
 Start ACE inhibitors early. The choice of agent can depend on practitioner preference,
hospital formulary or financial constraints for the individual patient.
 In smokers, the need to quit smoking should be reinforced early (within 24 hours) and
frequently.
 Stool softeners are often used in the immediate post-MI period to prevent straining with
bowel movements.
 Anxiolytics are often used on an as-needed basis in the immediate post-MI period.

Early Management of STEMI

Please note that this is only a draft version based on several sources, including: Dr. Basel Mohasen’s lectures,
Toronto Notes, Therapeutic Choices and others. Edited and organized for the sake of all attendances of the
Canadian Osce Exams: NAC OSCE and MCCQE2. by: Dr. Merlyn D’Souza and Dr. Zeev Gross, Spring 2011. Page 42
Please note that this is only a draft version based on several sources, including: Dr. Basel Mohasen’s lectures,
Toronto Notes, Therapeutic Choices and others. Edited and organized for the sake of all attendances of the
Canadian Osce Exams: NAC OSCE and MCCQE2. by: Dr. Merlyn D’Souza and Dr. Zeev Gross, Spring 2011. Page 43
Hx
OCD
PQRST (if it is suspected to be ACS - stop at R and start primary survey)
Primary Survey (If patient talks – Airway preserved, take Oxygen saturation and start Oxygen
Stat – 4L/m through nasal prongs)
Vitals
Auscultation: normal air entry and normal S1, S2
IV lines (normal NaCl 50ml/hr to keep line open, from the other side take blood for: Troponin,
CK-MB, Cr, BUN, Lytes, CBC, INR, PTT, LFT, Toxic., Alcohol, Lipids; and finger prick for Glucose)
ECG 12 leads and continue monitoring
Ask about Allergy and Viagra (if negative)
Give ASA chewable (325mg)
Non-ST elevation: give Nitro x3 (S.L) if there is no benefit – give Morphine.
Contin

Please note that this is only a draft version based on several sources, including: Dr. Basel Mohasen’s lectures,
Toronto Notes, Therapeutic Choices and others. Edited and organized for the sake of all attendances of the
Canadian Osce Exams: NAC OSCE and MCCQE2. by: Dr. Merlyn D’Souza and Dr. Zeev Gross, Spring 2011. Page 44
PQRST
AA&A
How do you feel now?
Ask Hx on CVS and GI (especially peptic ulcer)
CSx
RS
DVT

ST Elevation: do not go for DDx, Vitals (again)


RF
Nitro (2nd dose)
Examination:
JVP
Listen to heart
Base of lung
Compare BP in both Upper extremities to r/o coarctation of Ao
CXR
Once there is no Aortic Dissection  Thrombolytics (should be clear to r/o: Peptic ulcer, recent
surgery, pericarditis, aortic dissection, brain tumor, and stroke)
Based on ECG – counselling

Counseling
Based on your ECG it is most likely you are having an heart attack. If stable – BP and HR are
stable, but it is a serious condition, however it is treatable. Heart attack means that greater
than one blood vessel supporting your heart is blocked by a clot that has to be reimoved. The
medications are called clot busters. Based on ECG and no sign of pericarditis or signs of aortic
dissection you are a good candidate for treatment. It is an effective medication, needs consent.
1% chance of stroke and we can start heparin.

Back to Content

Please note that this is only a draft version based on several sources, including: Dr. Basel Mohasen’s lectures,
Toronto Notes, Therapeutic Choices and others. Edited and organized for the sake of all attendances of the
Canadian Osce Exams: NAC OSCE and MCCQE2. by: Dr. Merlyn D’Souza and Dr. Zeev Gross, Spring 2011. Page 45
PALPITATIONS
A.37 M/6wks [H&PE]
B.30F/4wks[H&C]
Dd:
VITAMINS C
VASCULAR: SVT,Rapid atrial fibrillation,& V Tach
METABOLIC:Fever,Anemia,Hyperthyroidism,Acromegaly
NEOPLASTIC: Pheochromocytoma
SUBSTANCE ABUSE & PSYCHIATRIC DRUG INGESTION (sympathomimetic) Drug
Withdrawl,Anxiety
CONGENITAL:WPW Syndrome

>48 hrs not panic attack

OCD:
O: When did it start? How did it Start?
Sudden/Gradual
What were you doing at that time?
C:
Does it come in bouts or Continous?
How often does it occur?
What was the duration of the attack?
D:
How long since you’ve had these palpitations?
How long does each episode last? / ? > 48 hrs?
PQRSTUV
Q:
Ask Pt to tap with his fingers the heart beat.
Does it Miss abeat/Racing/Slowing of heart beat?
S:
On a scale of 1 – 10 How has it affected the quality of your life?
T:
Does it occur even at night?Is it the first time or has it happened before?
Emphatize: I know it can be a fairly scary feeling
AA&A
A: What makes it worse:Coffee/Recreational
drugs/Stress/Smoke(extrasystoles)Choclates/Alcohol
A:
Anything makes it better?
How was your health prior to the palpitations?

ASOC Sx:

Please note that this is only a draft version based on several sources, including: Dr. Basel Mohasen’s lectures,
Toronto Notes, Therapeutic Choices and others. Edited and organized for the sake of all attendances of the
Canadian Osce Exams: NAC OSCE and MCCQE2. by: Dr. Merlyn D’Souza and Dr. Zeev Gross, Spring 2011. Page 46
CVS: Chest Pain/SOB/Orthopnea/PND/Dizziness/Sweating/N/VSwelling of feet/Cough
CNS: Weakness/Vision loss/Difficulty in finding words/Numbness or loss of sensation
THYROID: Do you feel hot/cold
Do you have wt loss inspite of increased appetite?/Tremors?Shakiness?Sweaty palms & moist
skin?
PHEOCHROMOCYTOMA: Repeated headaches,with increased sweating
CARCINOID:Flushing/Diarrhoea
CNS:In last few weeks did you notice any difficulty in Walking,numbness,finding words? (Look
for Sx/o Embolism)
CONSTITUTIONAL Sx: Fever/Chills/Loss of appetite/Lumps & Bumps

RISK FCTS: Smoking/Alcohol (Holiday Heart)/Recreational drugs

PMH:
 Are you on any medications?
 Have you seen a psychiatrist?
 Any OTC/Herbal products/Cold meds/Asthma meds
 Are you allergic to anything?
If Allergic to Penicillin:
1. When did you find out?
2. Where did you take it?
3. Why did you take it?
Do you have any Heart disease/HTN/DM/Stroke/Ca/MI
H/O Rheumatic fever as a child?
Did you get repeated sore throat infections as a child?
Did you receive Penicillin injections regularly as a child?

Back to Content

Please note that this is only a draft version based on several sources, including: Dr. Basel Mohasen’s lectures,
Toronto Notes, Therapeutic Choices and others. Edited and organized for the sake of all attendances of the
Canadian Osce Exams: NAC OSCE and MCCQE2. by: Dr. Merlyn D’Souza and Dr. Zeev Gross, Spring 2011. Page 47
HTN(GAO Pg:95)
History:

OCD
o O:
* When did you notice your BP was high?
* When was your last (N) BP
o Duration of hypertension,
* Usual level of blood pressure and
* Any sudden change in severity of hypertension
o History of antihypertensive drug use,
* Reason for changing therapy,
* effectiveness,
* side effects and intolerance (IMPOTENCE)
o Drugs that may cause hypertension drugs that may interact with antihypertensive drugs
(those that induce or inhibit metabolism)
o Adherence with lifestyle recommendations and drug therapy

HOME MONITORING

END ORGAN DAMAGE:


1. Angina/Mi: Chest pain/SOB
2. TIA/Stroke:LOC/Vision changes/weakness/Numbness
3. Peripheral Vascular disease/Leg pain/ED/
4. Kidney disease
SX of SECONDARY HTN, ,
1. Pheochromocytoma (hyperadrenergic symptoms)Do you have episodes of
palpitations/HA/Sweating?
2. Hyper- and hypothyroidism: Feel Hot/Cold/Tremors
3. Cushing’s syndrome: Bruising of skin/Wt gain
4. Renal/urinary symptoms or a past history of renal disease
RISK FCTS:
1. Cigarette and alcohol use,
2. Usual physical activity
3. Usual diet and sodium intake,
4. Current weight and recent weight change, waist circumference,
5. Diabetes
6. Dyslipidemia

PAST MED HISTORY


1. Medications Pt is on/OTC/Herbal
2. Allergies
3. Hospitalizations/Surgery

FAMILY HISTORY
 Hypertension,
Please note that this is only a draft version based on several sources, including: Dr. Basel Mohasen’s lectures,
Toronto Notes, Therapeutic Choices and others. Edited and organized for the sake of all attendances of the
Canadian Osce Exams: NAC OSCE and MCCQE2. by: Dr. Merlyn D’Souza and Dr. Zeev Gross, Spring 2011. Page 48
 Cardiovascular risk factors
 Premature cardiovascular disease

SOCIAL HISTORY
Nonpharmacologic Choices
Effect of Lifestyle Changes on Blood Pressure in Adults with Hypertension

Intervention Change in Blood Pressure


(systolic/diastolic) mm Hg

1. Reduction in sodium ↓ by 1800 mg (78 −5.8/−2.5


intake mmol) per day

2. Weight loss 4.5 kg −7.2/−5.9

3. Reduction in alcohol ↓ by 2.7 drinks/day −4.6/−2.3


intake

4. Exercise 3 times/week −10.3/−7.5

5. Dietary DASH dieta −11.4/−5.5


recommendations

 Weight loss of 4 kg or more if overweight (target body mass index: 18.5 to 24.9 kg/m 2; waist
circumference <102 cm in men and <88 cm in women).

 Healthy diet—high in fresh fruits, vegetables, soluble fibre and low-fat dairy products, low in
saturated fats and sodium, e.g., DASH diet available at Sodium intake of 1500 mg (65 mmol) per
day for those aged 19–50 years, 1300 mg (56 mmol) per day for those aged 51–70 years and
1200 mg (52 mmol) per day in those 71 years and older.
 Regular, moderate intensity cardiorespiratory physical activity for 30–60 minutes on most days.
 Low risk alcohol consumption (0 to 2 drinks/day, < 9 drinks/week for women and <
14 drinks/week for men).
 Smoke-free environment.

Back to Content

Please note that this is only a draft version based on several sources, including: Dr. Basel Mohasen’s lectures,
Toronto Notes, Therapeutic Choices and others. Edited and organized for the sake of all attendances of the
Canadian Osce Exams: NAC OSCE and MCCQE2. by: Dr. Merlyn D’Souza and Dr. Zeev Gross, Spring 2011. Page 49
SYNCOPE
Volume depletion and drugs
Volume depletion
 Diarrhea
 Diminished oral intake
 Polyuria

Drugs

ACE inhibitors

o Alcohol
o Alpha- and beta-adrenergic blockers
o Antiparkinsonian drugs
o Diuretics
o Nitrates
o Phosphodiesterase type 5 inhibitors (sildenafil, tadalafil, vardenafil)
o Vasodilators

Orthostatic intolerance disorders

 Reflex syncope syndromes


o Carotid sinus hypersensitivity
o Vasovagal syncope syndromes

 Autonomic neuropathies
o Pure autonomic failure syndromes
o Multiple system atrophy syndromes
 Arrhythmias
1. Bradycardias

o Complete (third degree) and bifasicular heart block


o Sinus node disease

2. Tachycardias

o Supraventricular arrhythmias (uncommon)


o Torsades de pointes polymorphic ventricular tachycardia
o Ventricular tachycardia

Obstruction

 Aortic stenosis
 Pulmonary emboli
 Many other rare causes
Please note that this is only a draft version based on several sources, including: Dr. Basel Mohasen’s lectures,
Toronto Notes, Therapeutic Choices and others. Edited and organized for the sake of all attendances of the
Canadian Osce Exams: NAC OSCE and MCCQE2. by: Dr. Merlyn D’Souza and Dr. Zeev Gross, Spring 2011. Page 50
Investigations
In patients with transient loss of consciousness perform a complete cardiovascular and neurologic
history and physical examination. Rule out seizures, then screen for life-threatening causes such as
obstruction, ventricular tachycardia and asystole or heart block

 Tailor laboratory investigations to the individual patient:


o ECG (most patients)
o older patients ( >55 years) should have ambulatory ECG monitoring unless the history is
strongly persuasive for vasovagal syncope.
o echocardiogram or other noninvasive measure of left ventricular function if structural
heart disease is suspected
o coronary angiography as indicated
o refer patients with structural heart disease for electrophysiologic assessment
o unless contraindicated, carotid sinus massage should be performed in patients >50
years old to screen for carotid sinus hypersensitivity (do not perform in patients with
carotid bruits)
o tilt table testing might be useful in diagnosing vasovagal syncope in patients with
atypical symptoms
 After potentially fatal causes are eliminated and reversible causes are removed, most patients
will have one of several syndromes of orthostatic intolerance:3
o reflex syncope syndromes
 vasovagal syncope
 carotid sinus hypersensitivity in the elderly
o pure autonomic failure syndromes
o multiple system atrophy syndromes
 The orthostatic intolerance syndromes can be distinguished based on history and a simple stand
test in the office.

To perform the stand test, first measure blood pressure


and heart rate after the patient has been supine for 5
minutes, then after 2 and 4 minutes of standing. These
responses are seen:
Normal and vasovagal syncope: modest rises in heart
rate (about 10 BPM) and blood pressure (about 10 mm
Hg).
Autonomic failure: progressive fall in blood pressure of
≥20 mm Hg systolic or ≥10 mm Hg diastolic with
development of presyncope; often no increase in heart
rate.

Please note that this is only a draft version based on several sources, including: Dr. Basel Mohasen’s lectures,
Toronto Notes, Therapeutic Choices and others. Edited and organized for the sake of all attendances of the
Canadian Osce Exams: NAC OSCE and MCCQE2. by: Dr. Merlyn D’Souza and Dr. Zeev Gross, Spring 2011. Page 51
Therapeutic Choices

Treatment is directed at the cause of syncope. Treat any reversible causes. Refer patients with
syncope secondary to bradycardia (asystole or complete heart block) for a permanent pacemaker.
Refer patients with suspected or diagnosed ventricular tachycardia, and all patients with
structural heart disease to a cardiologist, preferably an electrophysiologist. The following
addresses treatment of syndromes of orthostatic intolerance.

Nonpharmacologic Choices

 Reassure the patient that this syndrome is not life threatening and that it is a physical problem,
not a psychiatric disorder. Encourage increased dietary salt intake of about 3–5 g daily, in the
absence of contraindications such as hypertension or heart failure.5 , 8
 Teach the patient to use physical counterpressure manoeuvres at the onset of presyncope.9 , 10
These include squatting, crossing the legs with isometric contraction if standing, and vigorous
hand clenching with upper girdle isometric contraction. All should be tried. The evidence is
based on a good physiologic study10 and an open label randomized clinical trial.9
 Pacemaker therapy is no longer indicated, based on the results of an adequately powered
randomized placebo-controlled trial.11 The occasional patient with asystole documented during
vasovagal syncope might benefit, and these uncommon patients should be assessed at a tertiary
referral cl

Back to Content

Please note that this is only a draft version based on several sources, including: Dr. Basel Mohasen’s lectures,
Toronto Notes, Therapeutic Choices and others. Edited and organized for the sake of all attendances of the
Canadian Osce Exams: NAC OSCE and MCCQE2. by: Dr. Merlyn D’Souza and Dr. Zeev Gross, Spring 2011. Page 52
65/F Calf Pain x 10 weeks
How many blocks you could go? How many now?
How fast the pain disappears after resting?
Is the pain alleviated by bending forward or extending backward?
Is it awakening you at night?
Ddx:
1. Spinal Stenosis (Pain disappears about 15min after resting, alleviate by leaning forward)
2. Disc herniation (Pain disappears about 15 min after resting, alleviated by extending)
3. Intermittent Claudication (After resting – pain disappears by few minutes)
4. PE
5. Cellulitis
6. Ruptured Baker’s cyst
CC:
Unilateral Vs (B)
O:
Can you tell me when it all started?
Sudden Vs Gradual
What were you doing at that time?
What made you come in today?
C:
Is it Increasing,decreasing or same?
Has the intensity increased?
*Does it awaken you up at night?
D:
How often does each episode occur?
How long does it last?
PQRSTUV:
R:Does it move anywhere else in the body? Buttock/Toes/Feet/Thigh
T:When does it come on?
When you walk?
How many blocks can you walk when it comes? ------ >Now & at the beginning?
Reproducible pain

U
V:
 Has it happened before?
 If Yes When?
 How Often?
AAA:
AGGRAVATING:
Stand/Sit?

Please note that this is only a draft version based on several sources, including: Dr. Basel Mohasen’s lectures,
Toronto Notes, Therapeutic Choices and others. Edited and organized for the sake of all attendances of the
Canadian Osce Exams: NAC OSCE and MCCQE2. by: Dr. Merlyn D’Souza and Dr. Zeev Gross, Spring 2011. Page 53
When you walk uphill or downhill?
When you raise your leg?
ASSOC. SX:
CONSTITIONAL Sx: Fever/ /night sweats/Loss of appetite & los of Wt/Lumps & bumps.
Local Sx:
1. Swelling/Raised temp
2. Back Pain or Trauma to back or knee/Morning stiffness
3. Numbness/Tingling/Weakness/Burning sensation
4. Change in nails/Hair loss/Skin is it shiny/any Ulcers?
5. Are your feet cold?
CARDIAC:
Chest pain/Palpitations/SOB
*How is your sex life? Desire & Erection?........ How has this affected your life?
RS:
Cough
RISK FCTS:
I need to ask you some more qns that will help me arrive at a diagnosis of your pain:

1. Do you Smoke?
2. Drink Alcohol?
3. Have you recently travelled a long distance in an airplane?
4. Were you at any time Diagnosed as HTN,Is it Rxed & Under control? When was the last time
you saw your Dr.?
5. Were you at any time Diagnosedwith high blood sugar?Is it Rxed & Under control? When
was the last time you saw your Dr.?
6. Have you checked your cholesterol? .......
7. Did you have recent surgery?

PMH:
I need some information about your health in general:
 Are you on any meds?/OTC/Herbal products?
 Allergies?
 Were you ever hospitalized?
FHx
SOCIAL:
 Who lives with you?
 How do you support yourself financially?

Back to Content

Please note that this is only a draft version based on several sources, including: Dr. Basel Mohasen’s lectures,
Toronto Notes, Therapeutic Choices and others. Edited and organized for the sake of all attendances of the
Canadian Osce Exams: NAC OSCE and MCCQE2. by: Dr. Merlyn D’Souza and Dr. Zeev Gross, Spring 2011. Page 54
ANKLE SWELLING 30 M x 10 days
A. Gout
B. CHF
Dd:
A. UNILATERAL
1. Trauma
2. Arthritis: Gout/SepticA
3. Cellulitis
4. Varicosities

B. BILATERAL
1. CHF
2. Nephrotic
3. Liver failure
4. Myxoedema
5. Protein losing enteropathy

First Qn: Unilateral or Bilateral

OCD:
O; Sudden/Gradual
Off & On/Continous
Everyday/Certain time of day
Related to activity/Standing
PQRSTUV:
P:Above kne/Below knee
Posture
AA&A
What makes it worse: Activity/Alcohol & diet(Gout)/Standing
Alleviating fcts:Rest with elevation of feet....
ASSOC Sx:
LOCAL Sx:
In addition to the swelling did you notice any:
Pain/Stiffness/Fullness/Redness
Did you notice swelling anywhere else?
Face/Eyes particularly did you feel your eyes were puffy in the morning/Increasing waist
size/Rings are tighter?
CONSTITUTIONAL Sx: Fever/Wt loss/Night sweats/lumps & bumps
CARDIAC Sx: Chest Pain/SOB/Heart racing
GI/LIVER Disease:Yellow discoloration of skin/Pale stools/Dark urine
KIDNEY DISEASE:Change in the amount of: Urine/Color/Consistency (Frothy/Cloudy)/Odour

Please note that this is only a draft version based on several sources, including: Dr. Basel Mohasen’s lectures,
Toronto Notes, Therapeutic Choices and others. Edited and organized for the sake of all attendances of the
Canadian Osce Exams: NAC OSCE and MCCQE2. by: Dr. Merlyn D’Souza and Dr. Zeev Gross, Spring 2011. Page 55
RISK FCTS:
I need to ask you some more details to get more insight into your condt:
Do you Smoke,Take Alcohol or Recreational drugs?
What is your diet like? Do you eat a lot of red meats? (GOUT)

PMH:
*
Any meds you’re currently taking?( Aspirin & Thiazides for gout)/OTC/Herbal products?
Have you ever been diagnosed with HTN/DM/MI/Stroke/Ca?
Were you ever hospitalized or undergone any surgical procedure?

FH:
Does anyone in the family have a similar condt?
DM/HTN/MI/Stroke?

SOCIAL H: Who lives with you


How do you support yourself financially?

Back to Content

Please note that this is only a draft version based on several sources, including: Dr. Basel Mohasen’s lectures,
Toronto Notes, Therapeutic Choices and others. Edited and organized for the sake of all attendances of the
Canadian Osce Exams: NAC OSCE and MCCQE2. by: Dr. Merlyn D’Souza and Dr. Zeev Gross, Spring 2011. Page 56
24 female, acute asthma in ER, 3 dasys ago,asthmatic for the last 3yr
comes to you at clinic for F up
As I understand you were in ER 3 days ago with an attack of Ac asthma.
HOW DO YOU FEEL RIGHT NOW?

Event –
Before
Event
After
OCD
Can you tell me what happened at that time?
SX:
1. Sudden/Gradual
2. Was there Wheezing?
3. Chest tightness?
4. Were you able to talk?
5. How many times did you use the puffer?
6. How did you get to the ER?
7. What did they do in the ER?
8. Were you intubated?
9. What medicines did they give you?

Asthma history
Let us talk about Asthma history:
1. When were you diagnosed?
2. How were you diagnosed?
3. Are you on regular f/Up?
4. When was the last time you were seen at F/up?
5. Have you visited the ER before?
6. Did you notice any increase in nos of attacks?
7. Do you have attacks at rest? Attacks at Night?
8. Did you dr adjust your meds at that time?
9. Which meds do you use?
10. How often do you have to use your medicines?
11. Are you using the meds more frequently?
12. Triggers
13. Do you suffer from heart burn or condt called GERD?
14. Do you user a peak flow meter?

I need to ask more qns which will help me to clarify as to why you had an attack recently?
1. Have you had recently any flu/infection? (any chest infection upto 10 wks post infection
hyperreactive airways)

Please note that this is only a draft version based on several sources, including: Dr. Basel Mohasen’s lectures,
Toronto Notes, Therapeutic Choices and others. Edited and organized for the sake of all attendances of the
Canadian Osce Exams: NAC OSCE and MCCQE2. by: Dr. Merlyn D’Souza and Dr. Zeev Gross, Spring 2011. Page 57
2. Can you show me how you take medication? (Shake it, put it in your mouth, take deep breath
when puffing).
3. Do you make sure your medication are not expired and stored expired?
4. Did you started any new medication that might interfere (beta blocker / aspirin)with your
asthma?
5. Outdoor –
cold weather, pollens, exercise, construction, dust
6. Indoor –
a. Do you smoke or anyone around you,
b. Pets or people around,
c. New curtain, indoor plants, carpets, curtains, pillows.
d. Basement – mould, renovations, paintings;
7. Relation to any type of food;
8. Strong odour

Important to classify – mild / moderate / severe

I want to ask you…

What do you do for living?


PAST H:
Any HTN/DM
Hospitalizations/Surgery
FAMILY H:
Same Condt
DM/HTN/Stroke
SOCIAL H
Do you take alcohol/recreational drugs?
How do you support yourself financially?

Please note that this is only a draft version based on several sources, including: Dr. Basel Mohasen’s lectures,
Toronto Notes, Therapeutic Choices and others. Edited and organized for the sake of all attendances of the
Canadian Osce Exams: NAC OSCE and MCCQE2. by: Dr. Merlyn D’Souza and Dr. Zeev Gross, Spring 2011. Page 58
ER Rx of Ac Asthma

Please note that this is only a draft version based on several sources, including: Dr. Basel Mohasen’s lectures,
Toronto Notes, Therapeutic Choices and others. Edited and organized for the sake of all attendances of the
Canadian Osce Exams: NAC OSCE and MCCQE2. by: Dr. Merlyn D’Souza and Dr. Zeev Gross, Spring 2011. Page 59
Asthma Treatment
Very mild, intermittent asthma may be treated with fast-acting beta2-agonists taken as needed.
Inhaled corticosteroids (ICS) should be introduced early as the initial maintenance treatment for asthma,
even in individuals who report asthma symptoms less than 3 times a week. Leukotriene receptor
antagonists (LTRAs) are second-line monotherapy for mild asthma.
If asthma is not adequately controlled by low doses of ICS, additional therapy should be considered. A
long-acting beta2-agonist (LABA) should be considered first as add-on therapy only in combination with
an ICS. Increasing to a moderate dose of ICS or addition of an LTRA are third-line options. Theophylline
may be considered as a fourth-line agent in adults. Severely uncontrolled asthma may require additional
treatment with prednisone. Omalizumab may be considered in individuals 12 years of age and over with
poorly controlled atopic asthma despite high doses of ICS and appropriate add-on therapy, with or without
prednisone. Asthma symptom control and lung function tests, inhaler technique, adherence to asthma
treatment, exposure to asthma triggers in the environment and the presence of comorbidities should be
reassessed at each visit and before altering the maintenance therapy. After achieving proper asthma
control for at least a few weeks to months, the medication should be reduced to the minimum necessary
to maintain adequate asthma control.
Short-acting Inhaled Beta2-agonists (SABAs)
Salbutamol and terbutaline are selective beta2-agonists that are agents of first choice for treatment of
acute exacerbations and for prevention of exercise-induced asthma. They are best used as required
rather than on a fixed schedule. Although potent bronchodilators, they have little effect on the late
(inflammatory) phase of an exacerbation. If patients use a short-acting beta2-agonist more than 4 times
per week (including any doses used to prevent or treat exercise-induced symptoms), initiate therapy with
4
an anti-inflammatory agent. Isoproterenol and epinephrine are not recommended for the treatment of
asthma because of lack of beta2-selectivity and potential for excessive cardiac stimulation, especially at
high doses.

Long-acting Inhaled Beta2-agonists (LABAs)


Salmeterol and formoterol are long-acting beta2-agonists for regular twice daily treatment of asthma.
Salmeterol has a slow onset of action and should not be used for immediate relief of bronchospasm.
Formoterol is rapid acting and can be utilized for rescue therapy. These drugs help to prevent exercise-
5
induced bronchospasm. Both should be used only in patients already taking inhaled corticosteroids and
may be particularly useful for the prevention of nocturnal symptoms. Adding long-acting beta2-agonists to
6
inhaled corticosteroids may permit decreasing the latter’s dose.

This information was originally published in Canadian Respiratory Journal 2010;17(1):15-24.

Back to Content

Please note that this is only a draft version based on several sources, including: Dr. Basel Mohasen’s lectures,
Toronto Notes, Therapeutic Choices and others. Edited and organized for the sake of all attendances of the
Canadian Osce Exams: NAC OSCE and MCCQE2. by: Dr. Merlyn D’Souza and Dr. Zeev Gross, Spring 2011. Page 60
ACUTE COUGH Cough for the last 5d
Local Cause
Community acquired Pn
HIV
In young Pt can ask directly H/o HIV Status
In HIV(Pneumocytis Jevorici there is (B) chest pain & night sweats
OCD:UV
O
How did it start: Sudden/Gradual?
C:
Is it first time or have you had it before?
Is it increasing/Decreasing or same intensity now as it was in the beginning?
Does it wake you up from sleep? EMPATHY if awakens him up
NATURE:
Dry/Wet
If Wet: COCA
Color
Odour
Consistency
Amount
Blood
CHEST PAIN:
PQRST
Constitutional symptoms
RESPIRATORY Sx:
1. Shortness of breath,
2. Tightness,
3. Wheezing,
4. Ear pain
5. Sore throath
CARDIAC Sx:
SOB/
Heart racing
Chest ain
S/o Meningitis:
RISK FCTS:
1. Recent contact with sick people,
2. Vaccination for flu
3. TRAVEL H
4. Smoking
5. Alcohol
6. Drug abuse
Past Med H
Any meds/OTC/Allergies
Asthma/DM/Similar condt

Please note that this is only a draft version based on several sources, including: Dr. Basel Mohasen’s lectures,
Toronto Notes, Therapeutic Choices and others. Edited and organized for the sake of all attendances of the
Canadian Osce Exams: NAC OSCE and MCCQE2. by: Dr. Merlyn D’Souza and Dr. Zeev Gross, Spring 2011. Page 61
Hospitalizations/Surgery
FAMILY H
SOCIAL H:
Habits
IF HIV + PT:

DO you know your HIV diagnoses?


* When?
* Where?
* Regular follow up?
* Medication? Taking/or not?
* Last CD4?
* H/O Thrush?
PAST MED H
Allergy
COUNSELLING:
PE/ CXR & Bl work
Admit today

Back to Content

Please note that this is only a draft version based on several sources, including: Dr. Basel Mohasen’s lectures,
Toronto Notes, Therapeutic Choices and others. Edited and organized for the sake of all attendances of the
Canadian Osce Exams: NAC OSCE and MCCQE2. by: Dr. Merlyn D’Souza and Dr. Zeev Gross, Spring 2011. Page 62
Cough for the last 6w
Increasing, not improving
DID you seek medical attention?, what make you come today? Is it the first time?
OCD
O
How did it start: Sudden/Gradual?
C:
Is it first time or have you had it before?
Is it increasing/Decreasing or same intensity now as it was in the beginning?
Does it wake you up from sleep? EMPATHY if awakens him up
What made you come in today/
NATURE:
Dry/Wet
If wet first & then dry
2 elements:
1. Previous episode Sx:
When productive:Fever/chills/Night sweats/ muscle pain & joint ache/COCA
2. Dry cough Sx:
From that time till now,do you have fever?chills,muscle pains?
Sx of infection?
PATTERN OF COUGH:
1. Whole day?
2. How often?
3. How may attacks?
4. How long each attack?
5. Any particular time of the day?or
6. Do you wake up in morning with cough?(NIGHT COUGH: GERD/Asthma/CHF)
(MORNING COUGH: PND/GERD)
7. When you cough do you cough to the extent that you’ve:
a. SOB
b. Difficulty talking
c. Wheezing
d. Chest pain
e. Sweating
f. (in children----> vomit)
RISK FCTS:
I need to ask some qns now that could lead me to the cause:
 H/o repeated sinusitis
 Facial pain
 Ned to clear throath
 Runny nose
 GERD: H Burn
Acid taste in mlouth

Please note that this is only a draft version based on several sources, including: Dr. Basel Mohasen’s lectures,
Toronto Notes, Therapeutic Choices and others. Edited and organized for the sake of all attendances of the
Canadian Osce Exams: NAC OSCE and MCCQE2. by: Dr. Merlyn D’Souza and Dr. Zeev Gross, Spring 2011. Page 63
Relation to lying down/bending forward
How many pillows do you need at night
Do you get up in night gasping for breath/
 ASTHMA;
h/o Asthma
Relation between cough & outdoors or indoors
 SMOKING H:
Self: How many/
How long
Those around you
 Pets
What do you do for a living/
Any exposure to dust?
Any perfumes?
D/D
Other causes of Ch Cough
1. Medications:
a. HTN; ACE/Aspirin/NSAIDS
2. Swelling in legs
3. Rcent travel
4. Contact with Tb
5. H/o Lung Ca

CHEST PAIN:
PQRST
Constitutional symptoms
RESPIRATORY Sx:
1. Shortness of breath,
2. Tightness,
3. Wheezing,
4. Ear pain
5. Sore throath
CARDIAC Sx:
SOB/
Heart racing
Chest ain
S/o Meningitis:

Please note that this is only a draft version based on several sources, including: Dr. Basel Mohasen’s lectures,
Toronto Notes, Therapeutic Choices and others. Edited and organized for the sake of all attendances of the
Canadian Osce Exams: NAC OSCE and MCCQE2. by: Dr. Merlyn D’Souza and Dr. Zeev Gross, Spring 2011. Page 64
Back to Content

Please note that this is only a draft version based on several sources, including: Dr. Basel Mohasen’s lectures,
Toronto Notes, Therapeutic Choices and others. Edited and organized for the sake of all attendances of the
Canadian Osce Exams: NAC OSCE and MCCQE2. by: Dr. Merlyn D’Souza and Dr. Zeev Gross, Spring 2011. Page 65
67Male with Hemoptysis:

Same check list as H Uria look for pulmonary Sx

Intro:
As I understand you’re here because you’ve cough x 1 week?
Any chance you may be vomiting?
OCD
Duration: Night?
COCA + Bl
Sx of Hypovolemeia: Dizziness/Faint/Tiredness/Loc
A&A
ASx:
RS
CSx:
Hoarse voice
Risk Fcts;
Smoking
Contact with TB/Screened for TB
Travel outside Canada
Exposed to asbestos
H/O Dvt,Calf pain,redness,swelling of calves
H/O Hd: PND/Orthopnea
H/o Blood thinners
Bleeding from any parts of body
CNS:
PMH

Back to Content

Please note that this is only a draft version based on several sources, including: Dr. Basel Mohasen’s lectures,
Toronto Notes, Therapeutic Choices and others. Edited and organized for the sake of all attendances of the
Canadian Osce Exams: NAC OSCE and MCCQE2. by: Dr. Merlyn D’Souza and Dr. Zeev Gross, Spring 2011. Page 66
SOB/67 F x 6wks [5min/H] HF
Dd:
VITAMINS
VASCULAR:CHF,ACS,PE
Precipatants of CHF:
 Meds:
* Stopped
* NSAIDS
 Increased Na intake
INFECTIONS:Pneumonia

TRAUMATIC:Pneumothorax

METABOLIC:DKA

IDIPATHIC/IATROGENIC:COPD/Asthma/Massive atelactasis

NEOPLASTIC: Large pleural effusion

FIRST evaluate:ABC,ask pt if she is comfortable.......

OCD:

O:Sudden/Gradual
What were you doing when you had this SOB? ---Exertion/Lying down?
C:
*SOB first always ask if difficulty in breathing is for: Breathing IN or Breathing OUT
Does it occur all the time or only now
Is it related to activity or does it occur even at rest
If brought on by walking? How many blocks can you walk now as compared in the
beginning?
If at Rest?
Do you sleep well?
How many pillows do you need?
Do you wake up at night gasping for breath?
D:
How long?
If assoc Leg swelling,---- How long since leg swelling?
PQRSTUV:
S:
On a scale of 1 – 10?
T:
Has it ever happened to you before?If so,how often?
When was the last time you had SOB? How did you cope?

Please note that this is only a draft version based on several sources, including: Dr. Basel Mohasen’s lectures,
Toronto Notes, Therapeutic Choices and others. Edited and organized for the sake of all attendances of the
Canadian Osce Exams: NAC OSCE and MCCQE2. by: Dr. Merlyn D’Souza and Dr. Zeev Gross, Spring 2011. Page 67
U:
How has it affected your life & how do you manage? Do you have someone to help you?
[EMPATHY]
AAA
AGGRAVATING FctS:
Exercise
POsition
Exposure to cold air?
Infection?
Allergies?
ALLEVIATING FctS;
ASSOCIATED SYMPTOMS:

CONSTITUTIONAL Sx;
Fever/Chills/Wt loss/Lumps/Bumps
RS:
Cough/Sputum
GI:
Dec appetite (Liver & GI congestion)
Increase in waist size (Ascitis)
CVS:
Wt gain/weakness/Fatigue (Decreased cardiac Output)
Chest pain/Sweating/N/V/Heart racing/dizziness/Nocturia
Leg Pain/Leg Swelling/Wt gain
THYROID:
Do you feel cold/Hot /tremors
RISK FCTS:
I need some more details about you to get a better understanding abt your condt & hence need
to ask you a few more qns
Do you smoke?
Take alcohol?
* Take your meds regularly?
* Any change in your diet recently? --- Are you eating more canned foods or have you
been taking salted nuts
* Do you measure the Na in your diet?
Did you notice you’re pale?
PMH:
Were you ever diagnosed with HTN? What meds do you take? Have you taken your meds
regularly?
Were you ever diagnosed with a heart condition?
Which HD?When were you diagnosed?
How were you diagnosed?
Do you have regular follow ups?
Please note that this is only a draft version based on several sources, including: Dr. Basel Mohasen’s lectures,
Toronto Notes, Therapeutic Choices and others. Edited and organized for the sake of all attendances of the
Canadian Osce Exams: NAC OSCE and MCCQE2. by: Dr. Merlyn D’Souza and Dr. Zeev Gross, Spring 2011. Page 68
When was your last F up?
During your last visit, did your Dr add or remove any medications? How were you doing?
Were your symptoms controlled at that time?
Particularly ask about Dixogin:
How much/How long/Dose/did you have your levels checked/any chance that you might’ve
missed a dose?
Particularly ask about Water pills:
NSAIDS
DM/MI/Cholesterol/Stroke/Ca/Hospitalization/Surgery
FH:
FH of premature deaths

Back to Content

Please note that this is only a draft version based on several sources, including: Dr. Basel Mohasen’s lectures,
Toronto Notes, Therapeutic Choices and others. Edited and organized for the sake of all attendances of the
Canadian Osce Exams: NAC OSCE and MCCQE2. by: Dr. Merlyn D’Souza and Dr. Zeev Gross, Spring 2011. Page 69
SHORTNESS OF BREATH – POST SURGICAL
Surgery 3 days ago: SOB x 45 mins
D/D:
1. Volume status (low & High)
2. Atelectasis
3. Pneumonia
4. Heart Failure
5. Embolism
6. Fat embolism
INTRO:
As I understand, you’d surgery 3 days ago & I’ve to do a PE on you
VITLAS please
I would like to R/O orthostatic hypotension
Respiratory Rate
G/E:
Orientation:
 Time
 Place
 Person
Head: Sclera & Pallor
Mouth:
S/o dehydration
No Central cyanosis
No nasal flaring or pursed lips
No S/o Respiratory distress
HANDS:
Capillary refill
Clubbing
Cyanosis
Skin: Hot/Cold
Pulse: Rate & Volume
LEGS:
Dorsalis Pedis
Temperature
Pedal oedema
Feel for DVT
Measure diameter of (B) calves
Homann’s sign
NECK:
JVP
Trachea
S/o respiratory distress & use of accessory muscles of respiration
L Nodes for pneumonia

Please note that this is only a draft version based on several sources, including: Dr. Basel Mohasen’s lectures,
Toronto Notes, Therapeutic Choices and others. Edited and organized for the sake of all attendances of the
Canadian Osce Exams: NAC OSCE and MCCQE2. by: Dr. Merlyn D’Souza and Dr. Zeev Gross, Spring 2011. Page 70
CHEST:
Inspection:
Symmetrical
No IC retraction
No accessory muscles
No obvious pulsations
No PMI seen
FEEL
Apex beat
Back to Content

Please note that this is only a draft version based on several sources, including: Dr. Basel Mohasen’s lectures,
Toronto Notes, Therapeutic Choices and others. Edited and organized for the sake of all attendances of the
Canadian Osce Exams: NAC OSCE and MCCQE2. by: Dr. Merlyn D’Souza and Dr. Zeev Gross, Spring 2011. Page 71
LUMP In Breast/Neck
40/F h/o lump in Breast x 8 wks
INTRO:
As I understand you’ve a lump in your breast since 8 weeks, can you tell me more about it since
it all started?
OCD:
O:
HOW did you notice it? Routine examination or Accidentally?
C:
From that time to now, is it increasing, decreasing or remaining the same?
V:
Is it the first time, or have you noticed it before?
Any relation to periods?
Did you notice it on the upper or lower half of breast or inner or outer side?
Do you feel it reaching into the arm?
SIZE:
How do you estimate the size?
Chickpea/Olive/egg/Orange?
Hard/soft/rubbery?
Pain+/-
Skin: slides or fixed?
Changes in skin above: redness/ulcers
Lumps in other breast
Nipples: Dischareg/changes/ulcer?
CSX:
TRAUMA to breast?
METASTASIS:
Ha/Nx/Vx/Back pain/Cough/numbness in hand/Tired/pruritus?
I’m going to ask you qns that may explain this:
RISK FCTS:
1. H/o Ca in breast or other breast?
2. If any biopsy was performed on the breast?
3. FH of Ca breast
4. LMP
5. Have you ever been pregnant & at what age your first pregnancy?
6. Have you breast fed?
7. Any OCs & for how long?
8. Do you smoke/alcohol?
9. H/o Ovarian or uterine Ca
10. Fatty Diet?

Back to Content

Please note that this is only a draft version based on several sources, including: Dr. Basel Mohasen’s lectures,
Toronto Notes, Therapeutic Choices and others. Edited and organized for the sake of all attendances of the
Canadian Osce Exams: NAC OSCE and MCCQE2. by: Dr. Merlyn D’Souza and Dr. Zeev Gross, Spring 2011. Page 72
LUMP IN NECK:16/F X 2 WKS
D/d:
Reactive Adenitis --------> Recent Flu
Tooth problems --------->
Lymphoma ----------> Hard
IM
HIV
INTRO:
WHERE?
HOW : OCD:
SIZE:
FEEL:
Pain +/-
SKIN changes
Any other lumps?
CSx:
Sore throat / dyspahgia?
Hx/Nx/Vx?
Skin rash?
Ear discharge?
Sinusitis/cough/Pglem?
H/o Ca or malignancies?
HEADSS
Abd pain
Vaginal discharge
Urine changes
PE:
Vitals:
Look & asses the lump
Look for any other lumps: Cervical LN/Supraclavicular/axillary
Mouth
PA:
Liver & spleen
Groin LN
Popliteal fossa
Pelvic & Vaginal exam
Rectal exam
Back to Content

Please note that this is only a draft version based on several sources, including: Dr. Basel Mohasen’s lectures,
Toronto Notes, Therapeutic Choices and others. Edited and organized for the sake of all attendances of the
Canadian Osce Exams: NAC OSCE and MCCQE2. by: Dr. Merlyn D’Souza and Dr. Zeev Gross, Spring 2011. Page 73
TIREDNESS
45M with tiredness x 6 weeks
As I understand you’re having Tiredness since 6 weeks, can you tell me more about it since it all
started?
Pt says he is concerned. STOP & ask about his concern.
He says he is Air traffic controller & his vision has been blurry
Pt says, he is also never been so tired before. Clarify: What do you mean about Tiredness?
 Sometimes I do not feel refreshed after sleep.
 Do you feel lack of energy? Like you cannot move your arm above your head.
The Statement: NOT REFRESHED ANYMORE points to an organic cause
OCD + Relation to sleep +/- Mood
If Mood Sx + ------ MOAPS
If Organic cause ---- Red Flags
OCD
At onset you can ask if there were any flu-like Sx initially (Thyroiditis)
 Do you sleep more
 When do you go to bed?
 Do you wake up in the middle of the night
 Which time of day/Night do you feel most?
 Ask Nature of work;
If shift
 With whom do you sleep?
 Does your partner C/o you snoring or jerky movts of limbs? (Restless Leg)
 Do you feel better in morning or evening(if tired in morning-->Depression If evening-------
-->Organic cause)

1. Ask Constitutional Sx first:


Fever/Wt loss/Night sweats/Chills/Lumps & Bumps
Then quick review of Sx:
2. Cardiac: Chest Pain/SOB/Palpitations
3. Pulmonary: Cough/Wheezing/Phglem
4. GI: N/V Abnormal bowel movts/Diarrhoea/Malena/
5. Liver: Dark urine/Yellow sclera/Abd pain/Loss of aooetite/Pale stool/Itchy skin
6. GU: Change in color of urine/Amt of urine/Cloudy/Frothy urine/Dysuria/Facial swelling
7. Anemia: Bleeding gums/Easy bruising/Malena/Haematuria/Female: Meorrhagia & LMP

Please note that this is only a draft version based on several sources, including: Dr. Basel Mohasen’s lectures,
Toronto Notes, Therapeutic Choices and others. Edited and organized for the sake of all attendances of the
Canadian Osce Exams: NAC OSCE and MCCQE2. by: Dr. Merlyn D’Souza and Dr. Zeev Gross, Spring 2011. Page 74
8. Autoimmune Disease: Joint Pains/Skin rashes/Oral Ulcers
9. Endocrine: Thyroid: Feel hot/Cold Skin Moist/Dry/Tremors/Wt loss
10. DM: Risk fcts: FH & Lifestyle
Once Pt has DM in history, GO over ALL Sx & Sy:
A. Fluctuating 6 Sx
3 High:
1. Increased eating
2. Inc Drinking
3. Inc urination
3:
4. Tired
5. Wt loss
6. Blurry vision
B. Complications:
 Micro:
1. Retinopathy: Black spots
2. Neuropathy: Tingling/ numbness
3. Nephropathy: Inc Urine
4. Impotence: Sometimes people with this condition have marital problems; Do you have
changes in desire or difficulty in having or maintaining an erection?
5. Autonomic Dysfn:
 Orthostatic hypotension
 Gastroparesis
 Diarrhea
 Voiding difficulties
 HTN
 Macro:
1. Coronary Artery Disease: Chest Pain/SOB/
2. CVA: Weakness/numbness/difficulty finding words/Sudden visual loss
3. Peripheral Vascular disease: Pain in calves/Cramps/Cold feet
Skin infections
Candidial infections in women

RISK FCTS:
1. Diet
2. Exercise
3. FH

Please note that this is only a draft version based on several sources, including: Dr. Basel Mohasen’s lectures,
Toronto Notes, Therapeutic Choices and others. Edited and organized for the sake of all attendances of the
Canadian Osce Exams: NAC OSCE and MCCQE2. by: Dr. Merlyn D’Souza and Dr. Zeev Gross, Spring 2011. Page 75
4. Smoking
5. Alcohol
6. Recreational drugs

PAST MEDICAL HISTORY:


Any medications/OTC/Herbal/LMP
Surgery/Hospitalization

FAMILY HISTORY:
DM/HTN/Stroke/MI

SOCIAL H:
Habits
With whom do you live?
How do you support yourself financially?

Please note that this is only a draft version based on several sources, including: Dr. Basel Mohasen’s lectures,
Toronto Notes, Therapeutic Choices and others. Edited and organized for the sake of all attendances of the
Canadian Osce Exams: NAC OSCE and MCCQE2. by: Dr. Merlyn D’Souza and Dr. Zeev Gross, Spring 2011. Page 76
Here Counselling should be short Focus on Blurry vision in relation to DM & why he needs to
correct it.
As I told you DM is a lifelong disease & it is imp to have the BSL controlled.
Without proper BSL control, the increasing Blood sugar damages the blood vessels in our body,
& leads to heart attacks, Strokes, Kidney failure. Also visual loss & feet ulcers
I will have to refer you to a diabetic clinic
However I will do basic blood inv & ECG first

Back to Content

Please note that this is only a draft version based on several sources, including: Dr. Basel Mohasen’s lectures,
Toronto Notes, Therapeutic Choices and others. Edited and organized for the sake of all attendances of the
Canadian Osce Exams: NAC OSCE and MCCQE2. by: Dr. Merlyn D’Souza and Dr. Zeev Gross, Spring 2011. Page 77
35 M in hospital setting had a DKA 3 days ago x 5 mins H He is
diabetic since 25 years

Here we’ve to look for Rf & see that are not repeated

As I understand you were seen in the hospital by my colleagues & Rxed for a condt
called DKA 3 days ago.
Can you tell me what exactly happened to you at the time?
Xxxxx
1. Was there abdominal pain
2. Vomiting?
3. Were you drowsy?
4. Was there loss of consciousness?
5. How did you get to the ER?

6. When were you Diagnosed ad Diabetes?


7. How were you Dsed?
8. What type of Dm?
9. Which medications do you take/
10. Insulin?
10.1 How much?
10.2 When was the last time your insulin was adjusted?
10.2 What was the wt at that time/
11. Any loss or gain of wt?
12. How often do you monitor your bl Glucose?
13. When was the last HbA1c
14. Did you have DKA before?
15. When was the last time you had it/Are you under regular F/U?
16. How were you doing for the last few mths?
Ask about Sx control:
Blurry vision
Inc thirst
Inc Peeing
Wt loss

Any idea why Sx are not controlled?


DIET
What about diet?
How often do you party?
Do you modify your diet when you party?
Do you adjust insulin when you party/
If you take alcohol do you count it as a part of your calories
Please note that this is only a draft version based on several sources, including: Dr. Basel Mohasen’s lectures,
Toronto Notes, Therapeutic Choices and others. Edited and organized for the sake of all attendances of the
Canadian Osce Exams: NAC OSCE and MCCQE2. by: Dr. Merlyn D’Souza and Dr. Zeev Gross, Spring 2011. Page 78
EXERCISE:
Any RECENT INFECTION/FEVER?

LONG TERM COMPLICATIONS:


Have you ever had an eye check/
When was the last time you saw an ophthalmologist?
Any retinal complications/
Any H/o heart attack?
How are your feet/
Any feet ulcers?
How is your urine,is it frothy/Cloudy/
Do you have tingling/numbness
Thank you for the information,we will have to do further assessment

38M requesting a note to say he is well & fit to work


Cannot see at work, works as an accountant.

Back to Content

Please note that this is only a draft version based on several sources, including: Dr. Basel Mohasen’s lectures,
Toronto Notes, Therapeutic Choices and others. Edited and organized for the sake of all attendances of the
Canadian Osce Exams: NAC OSCE and MCCQE2. by: Dr. Merlyn D’Souza and Dr. Zeev Gross, Spring 2011. Page 79
ABDOMINAL PAIN/ACUTE ABDOMEN
1. Acute abdomen x 24 hrs 45/M Er x 5min H
2. Dysphagia x 6 wks 55M
3. Inc LFT:
a. 35 M ALT>>AST
b. 55M AST > ALT
4. Diarrhoea:
a. Ch Diarrhoea x 6 wks H & PE
b. Ch Diarrhoea x 6 wks H & C
c. Ac Diarrhoea x 3 days H x 5mins

AC ABD: Acute abdomen x 24 hrs 45/M Er x 5min H


As I understand you’ve abdominal pain since 24 hrs.
Are you comfortable sitting down?
Please bear with me for 5 mins till I find the cause of this pain & then I can give you
something for relief
OCD:
O: Gradual/Sudden
C:
From the onset till now,is it: Inc/Dec
Off & On/Ct-ous
How was it at first?
When change in course,QUALITY & SEVERITY is significant
P
Q: Ask nature from beginning
R
S
T: Any relation with time?
Before eating/After eating
U: How was your sleep?
V:
Alleviating: Eating/Antacids/Defecation/Meds?
Aggravating Fcts: Food/Posn/Alcohol/Meds?
Assoc Sx:
 Nausea
 Vomiting: If +ve:
* Color
* Odour
* Consistency
* Amount

Please note that this is only a draft version based on several sources, including: Dr. Basel Mohasen’s lectures,
Toronto Notes, Therapeutic Choices and others. Edited and organized for the sake of all attendances of the
Canadian Osce Exams: NAC OSCE and MCCQE2. by: Dr. Merlyn D’Souza and Dr. Zeev Gross, Spring 2011. Page 80
* Forceful
* Does vomiting relieve pain?
* What started first: Vomiting or pain?
(If V 1st --Infection/ If Pain 1st Surgical)
BOWEL CHANGES:
* When was the last bowel movt?
* Any abdominal distension?
* Is there any blood in stool/Dark stool?
* Are you passing any gases?
APPETITE CHANGES:
Sx Dehydration:
* Dizziness
* Dark Urine
LAST MEAL:
* Did you eat alone?
* Was there anything new?
* Did others have same Sx?
RISK FCTS:
* Any Past H/o Abdominal Surgeries
* Crohn’s disease?
* Groin surgery(hernia repairs)
* Gall Bladder stones?
* Pancreatic stones
* Diverticulitis?
* Smoking
* Alcohol

CONSTITUNIOL Sx:
Fever/Chills/wt loss/Lumps 7 bumps
CVS: Chest Pain/Hd/HTN
FH;
Ca Colon/Polyposis
PAST Medical h:
Meds: Aspirin/OTC/Herbals/LMP

Back to Content

Please note that this is only a draft version based on several sources, including: Dr. Basel Mohasen’s lectures,
Toronto Notes, Therapeutic Choices and others. Edited and organized for the sake of all attendances of the
Canadian Osce Exams: NAC OSCE and MCCQE2. by: Dr. Merlyn D’Souza and Dr. Zeev Gross, Spring 2011. Page 81
71F/Abd pain after meals x 4 weeks [10 min H] (ischemic)

Dd:
1. GERD
2. PUD
3. Gastric Ca
4. Ischemic Mesentry
5. Pancreatic failure
6. Ischemic colitis
Here Pt was concerned that spouse died of Ca Stomach.Reassuare her that unlike Flu Ca
Stomach is not contagious, but because they were married for so long there might be a
chance that they were exposed to a risk fct.But you will conduct a thorough History &
Inv
OCD
PQRSTUV
A&A
Assoc Sx:

 Constitutional Sx:
 GI Sx:
* N/V/D
* Malena
* Bowel movts
* Abdominal bloating
 Cardiac Sx:
* Chest Pain/Palpitations/SOB

RISK FCTS: HTN/DM/Smoking/FH of HD/Cholesterol?HD


If +ve for HD
 Ascertain type & when diagnosed,enquire abt F ups
 When/what & if intervention
PMH:
 Any meds/OTC/Herbal
 Hospitalization & Surgery
FH
Social H:
Here since Pt had Cardiac Sx.Your cardiac history is of concern to me & I’ve to do a PE &
perform some blood inv & an ECG & then ask for a Surgical Consult
Back to Content
Please note that this is only a draft version based on several sources, including: Dr. Basel Mohasen’s lectures,
Toronto Notes, Therapeutic Choices and others. Edited and organized for the sake of all attendances of the
Canadian Osce Exams: NAC OSCE and MCCQE2. by: Dr. Merlyn D’Souza and Dr. Zeev Gross, Spring 2011. Page 82
DIARRHOEA
Bloody Diarrhoea:
Awakens at night?
Any constant pain?
Think Inflammatory bowel disease

Always ask about antibiotic use: if yes:


1. Why
2. Which Ab
3. For how long
4. When
In acute Diarrhoea look for dehydration
In ch diarrhoea look for Wt loss & then dehydration

Bad Diarrhoea if:


+ve for fat droplets
Floats
Undigested food
Bulky

After intro:
OCD
Frequency
COCA + Blood
Impact
PAIN
ASx
D/d:
1. GE: if fever/N/V
2. Travel
3. New restaurant
4. Antibiotic
5. Osmotic
PMH
Fh
Sh
Please note that this is only a draft version based on several sources, including: Dr. Basel Mohasen’s lectures,
Toronto Notes, Therapeutic Choices and others. Edited and organized for the sake of all attendances of the
Canadian Osce Exams: NAC OSCE and MCCQE2. by: Dr. Merlyn D’Souza and Dr. Zeev Gross, Spring 2011. Page 83
24 yr old diarrhoea x 3 days
Introduction:
Tell me more about is since the moment it all started?
O
C:
How often do you go to the wash room?
Estimated amount of stool passed?
Is it Tarry?
Mixed with blood?
Any undigested food?
Bulky?
Is it offensive?
Does it float in toilet bowl?
Is it difficult to flush?
A&A
Did you try any meds/did they help

ASx:
Do you feel dizzy/Thirsty?
Do you drink enough fluid?
Have you lost wt?
Do you have any additional Sx like abdominal pain?
Does bowel movt relieve your pain?
N/V
CSx:
Did you eat in a new place? (raw food----Shigella)
Did you recently take any antibiotics?
Anybody else in the family has diarrhoea?
Any FH of Bowel disease? Or condt called Crohn’s Disease?
FH
SH

Please note that this is only a draft version based on several sources, including: Dr. Basel Mohasen’s lectures,
Toronto Notes, Therapeutic Choices and others. Edited and organized for the sake of all attendances of the
Canadian Osce Exams: NAC OSCE and MCCQE2. by: Dr. Merlyn D’Souza and Dr. Zeev Gross, Spring 2011. Page 84
24 yr M Diarrhoea x 6 weeks
(Crohn’s)

Can you tell me about it?


O
C:
All the day?/At night/
D
COCA + Bl
Bulky
+ Fat droplets
Floats & difficult to flush
Undigested food
PAIN
Relieved by diarrhoea or not
Distension/gases
U:
How has it affected your life
How do you deal with it?
V:
CSx:
Extraintestinal Sx:
Skin rashes/nail changes/Joint pains/back pain/Hx of psoriasis/Back pain/red eyes/Morning
urethral discharge/
D/D:
Thyroid :tremors/sweating
Relation of diarrhoea to food like dairy products
Travel or camping history
Sx of Liver disease
FH of Ulcerative colitis/Crohn’s
SH

Back to Content

Please note that this is only a draft version based on several sources, including: Dr. Basel Mohasen’s lectures,
Toronto Notes, Therapeutic Choices and others. Edited and organized for the sake of all attendances of the
Canadian Osce Exams: NAC OSCE and MCCQE2. by: Dr. Merlyn D’Souza and Dr. Zeev Gross, Spring 2011. Page 85
DIARRHOEA x 6 wks (Irritable bowel disease)
Dd:
1. IBD
2. Infection: camping/travel
3. Hyperthyroidism
4. IBS
5. Lactose intolerance
6. Coeliac Disease
7. HIV
8. Ch Liver & pancreratic failure
OCD
COCA + Blood + mucus
PAIN:if +ve which started first Pain or diarrhoea
Does bowel movement relieve the pain?
Does it awaken you at night?
Do you feel you want to go back to wash room again?
Distension/gases
U
What made you come in today?
V:
DIET: dairy products
C Sx:
In IBS R/o Organic cause:
Stress?
What type; Family/Job
How do you cope with it?
How is your mood/Interest?
FH: of Ca Colon at young age
SH

Back to Content

Please note that this is only a draft version based on several sources, including: Dr. Basel Mohasen’s lectures,
Toronto Notes, Therapeutic Choices and others. Edited and organized for the sake of all attendances of the
Canadian Osce Exams: NAC OSCE and MCCQE2. by: Dr. Merlyn D’Souza and Dr. Zeev Gross, Spring 2011. Page 86
42 M with Bld work LFT INCREASED:
ALT:300 AST:100

Cc: Bld works


Intro
Talk about blood works:
1. WHY?
2. Is it 1st time
3. When
4. Who
5. If done before: what were results then?

SPIKE
EXPLAIN Results
CONSEQUENCES: Sx. Ac & Ch
CAUSES OF LIVER DISEASE:
PMH
FH
SH

Intro:
As I understand you’re here today to discuss some of your blood tests results
Is it 1st time/
Why/who/When
EXPLAIN:
Results show that there is an increase in the markers to measure the function of the liver, called
liver enzymes, this indicates that there is an injury to the liver cells
There are different causes, but before coming to the causes, I want to know if there are any
Symptoms of liver disease
Ac Sx;
1. Yellow discoloration of skin/eyes
2. Pale stools/Dark urine
3. Itchy skin
4. Loss of appetite/nausea/distaste for cigarettes
5. Flu like Sx few weeks ago
Ch Sx;
CAUSES OF LIVER DISEASE
1. Have you ever been diagnosed as liver disease before?
2. Have you ever ben screened for liver disease before?
3. Have you ever been vaccinated for Hepatitis A or Hepatitis B before?
TRANSITION

Please note that this is only a draft version based on several sources, including: Dr. Basel Mohasen’s lectures,
Toronto Notes, Therapeutic Choices and others. Edited and organized for the sake of all attendances of the
Canadian Osce Exams: NAC OSCE and MCCQE2. by: Dr. Merlyn D’Souza and Dr. Zeev Gross, Spring 2011. Page 87
I want to ask qns now as to whether you were exposed to liver disease:
1. Do you smoke? Take alcohol/Recreational drugs? Past use IV drugs
2. Tattoos
3. Any past surgeries/hospitalizations
4. FH of liver disease
5. Long term disease in past
6. Any H/o bleeding disease
Thank you for all this info

Need to do some further assessment

Back to Content

Please note that this is only a draft version based on several sources, including: Dr. Basel Mohasen’s lectures,
Toronto Notes, Therapeutic Choices and others. Edited and organized for the sake of all attendances of the
Canadian Osce Exams: NAC OSCE and MCCQE2. by: Dr. Merlyn D’Souza and Dr. Zeev Gross, Spring 2011. Page 88
52 M/Dysphagia x 6 wks ----5 mins H
D/d:
1. Oesophageal Ca
2. Scleroderma
3. O.Stricture
4. O.
5. DES
6. web/Ring
7. HIV

FIRST CLARIFY:
If difficulty to initiate swallowing or food coming out from nose ---- Neuro Sx/Stroke/MS
Pain on swallowing ------ AIDS/CMV/Ca/Decreased immunity/Leukemia
If food gets stuck -> Can you show where it gets stuck?

ONSET:How did it start?


COURSE:
Intermittent /progressive
Intermittent for both solids & liqds ----> SPASM
Intermittent for only SOLIDS---------------> Oesophageal web/ring check by endoscopy
Progressive: Starts with fluid & Solids later ------>Achalasia & Scleroderma
Progressive: Starts with solids & later liquids ------> M/Canical obstruction: Ca
Here initially you could flush it down with water, but unable to do so now

ASSOC Sx:

N/Vx
Repeated chest infections
Chest pain/Tightness in chest
Do you bring up undigested food?
Cough
Change in voice
GI Sx:
Abd pain/Abd distension
Change in bowel pattern
Any blood in stool or vomit?
CONST Sx:
Fever/ Night sweats/Change in appetite/Chills /Lumps & bumps
If wt loss,how much Wt loss over how long?
Please note that this is only a draft version based on several sources, including: Dr. Basel Mohasen’s lectures,
Toronto Notes, Therapeutic Choices and others. Edited and organized for the sake of all attendances of the
Canadian Osce Exams: NAC OSCE and MCCQE2. by: Dr. Merlyn D’Souza and Dr. Zeev Gross, Spring 2011. Page 89
Tiredness
*Mets: Liver: Yellow discoloration of skin/Dark urine & pale stools

RISK FCTS:
I’m going to ask you a few qns to reach the diagnosis:
1. Any H/o heart burn?
2. Have you ever been diagnosed with a condt called GERD?----If Yes:
How long ago?
Did you seek medical attention?
Was an endoscopy performed (A tube with a camera put down your food pipe to view)
3. Were you ever diagnosed with a condt called: Barret’s Oesophagus?
4. Do you smoke?/Drink alcohol?
5. FH of Oesophageal Ca
6. H/o swallowing acid or alkalies
7. H/o Chest radiation
8. H/o Achalasia
9. Any skin tightness
10. Change in color of digits when exposed to hot or cold enviorenments
11. CNS:
H/o stroke/weakness
H/o DM
H/o HIV

Back to Content

Please note that this is only a draft version based on several sources, including: Dr. Basel Mohasen’s lectures,
Toronto Notes, Therapeutic Choices and others. Edited and organized for the sake of all attendances of the
Canadian Osce Exams: NAC OSCE and MCCQE2. by: Dr. Merlyn D’Souza and Dr. Zeev Gross, Spring 2011. Page 90
Chest Pain 45 M x 6 wks
GERD
ONSET:
When did you first notice it?
How did it start?
What were you doing at that time?
Course:
1. Is it the same intensity it was at the beginning?
2. Is it increasing in intensity?
3. Is it decreasing in intensity
4. Is the frequency increasing or decreasing or same?
Duration:
Foe how long now you’ve the pain?
POSITION:
Can you tell me exactly where it hurts you?
Quality
Can you describe the nature of your pain? Burning,Tightnes....
Radiation:
Does it move anywhere else in your body?
To the (L) arm,Jaw,Neck,Back?
Severity:
On a scale of 1 –10 ......
How has this pain affected your life?
Time
Does it occur at a particular time?
Does it awaken you at night?
Aggravating fcts:
1. Exercise/Stress
2. Food: (peppermint,fatty food,Citrus fruit)
3. Tobacco
4. Alcohol
5. Hot or cold food
Relieving fcts:
1. Antacid
2. Elevation of head end of bed
Assoc Fcts:
Do you have
1. Heart burn
2. Acid reflux
3. Difficulty swallowing
4. Dark stools?

5. Cough

Please note that this is only a draft version based on several sources, including: Dr. Basel Mohasen’s lectures,
Toronto Notes, Therapeutic Choices and others. Edited and organized for the sake of all attendances of the
Canadian Osce Exams: NAC OSCE and MCCQE2. by: Dr. Merlyn D’Souza and Dr. Zeev Gross, Spring 2011. Page 91
6. Hoarse voice
7. Sore throath
8. Wheezing
9. Dental problems (dental erosions)

10. Palpitations
11. SOB/PND/Orthopnea

12. Constitutional Symptoms:


13. Loss of appetite & Wt loss
14. Fever & Chills
15. Lumps & bumps
16. Lupus
To gain more insight into your condition, I need to ask you some details about your life:
Do you smoke
Drink Alcohol
*** ? Recreational drugs?
Do you eat a lot of fatty foods?
How soon afer dinner do you go to bed?

PAST Med H
1. Are you on any medication/
2. Do you take Aspirin or any pain relievers or any OTC products?
3. Is there any herbal medication you’re on?
4. Are you allergic to anything?
5. Have you ever been diagnosed to have a High BP or high BSL?
6. Have you ever had a heart attack or stroke?
7. Were you ever hospitalized?
8. Did you ever undergo any surgical procedure?

FAMILY History
Social History

MGment:
Nonpharmacologic Choices
 Dietary modifications (avoid chocolate, caffeine, acidic citrus juices, large fatty meals)
 Weight loss if obese (BMI > 25 to 30 kg/m2)
 No snacks within 3 hours before bedtime
 No lying down after meals
 Reduce alcohol intake
 Elevate legs under the head of the bed on 10 to 15 cm blocks
 Stop smoking
 Avoid tight clothing

Please note that this is only a draft version based on several sources, including: Dr. Basel Mohasen’s lectures,
Toronto Notes, Therapeutic Choices and others. Edited and organized for the sake of all attendances of the
Canadian Osce Exams: NAC OSCE and MCCQE2. by: Dr. Merlyn D’Souza and Dr. Zeev Gross, Spring 2011. Page 92
Pharmacologic Choices
When possible, eliminate drugs that impair esophageal motility and lower esophageal sphincter tone
(e.g., calcium channel blockers, theophylline, tricyclic antidepressants, beta-blockers, anticholinergic
agents).

Back to Content

Please note that this is only a draft version based on several sources, including: Dr. Basel Mohasen’s lectures,
Toronto Notes, Therapeutic Choices and others. Edited and organized for the sake of all attendances of the
Canadian Osce Exams: NAC OSCE and MCCQE2. by: Dr. Merlyn D’Souza and Dr. Zeev Gross, Spring 2011. Page 93
6th Feb

GU
If CC Urinary Sx:
I. Obstruction
II. Irritation
III. Urinary changes
OSTRUCTION 4 qns:
1. Difficulty initiating urine ----->Do U need to strain?
2. Did you notice change in stream?
3. Dribbling?
4. After passing urine, do you still need to pass more?
IRRITATION:
1. How many time do you need to go to Wash room: Now,Before At Night?
2. Does it affect your sleep?
3. Do you feel you need to rush to WC
4. Are you able to make it in time?
5. Have you ever lost control?
6. Burning sensation
7. Flank pain
8. Fever
URINE CHANGES:
COCA + Bld
Consistency,remember:
1. Frothy urine
2. Cloudy urine
3. Not clear urine
Back to Content

Please note that this is only a draft version based on several sources, including: Dr. Basel Mohasen’s lectures,
Toronto Notes, Therapeutic Choices and others. Edited and organized for the sake of all attendances of the
Canadian Osce Exams: NAC OSCE and MCCQE2. by: Dr. Merlyn D’Souza and Dr. Zeev Gross, Spring 2011. Page 94
67 M ER reten48 hrs/Colleague passed F Catheter & got 1.2 L urine
As I understand, you’re here today because you’d difficulty in passing urine since 24 hrs.& one of my colleagues
has passed a Foley catheter & drained 1.2L urine.
HOW DO YOU FEEL RIGHT NOW?
I’m glad to know you’re better
If Pt c/o pain: Bear with me few minutes as soon as I finish with asking you a few Qns I will deal with it.
Can you tell me about it since it all began?
OCD -------- U V
How many times did you try to void?
Were you able to pass any amt?
Is it the first time/Can be first time to this extent
Recently have you noticed any changes in your urine?you.g: Do U need to rush?
When did it first start?
From that time till now, is it Increasing/Decreasing?
 Sx of Obstruction:
1. Difficulty initiating urine ----->Do U need to strain?
2. Did U notice change in stream?
3. Dribbling?
4. After passing urine,do U still need to pass more?
EMPATHY
 Sx of irritation:
1. How many times do U need to go to Wash room: Now, Before At Night?
Does it affect your sleep?
2. Do U feel U need to rush to WC
Are you able to make it in time?
Have you ever lost control?
3. Burning sensation
4. Flank pain
5. Fever
 Urine changes:
COCA + Bld
Ask if H/o passing stones in urine
C Sx:
ASx:
Trauma to back:Back pain
Sx of GU: asked in earlier Qns
Mets to Liver: Sx of Liver Disease
Mets to lung: Cough/Haemoptysis
Mets to CNS:

RISK FCTS:
1. Have you ever been screened or diagnosed as prostrate disease?

Please note that this is only a draft version based on several sources, including: Dr. Basel Mohasen’s lectures,
Toronto Notes, Therapeutic Choices and others. Edited and organized for the sake of all attendances of the
Canadian Osce Exams: NAC OSCE and MCCQE2. by: Dr. Merlyn D’Souza and Dr. Zeev Gross, Spring 2011. Page 95
2. Have you ever done the blood test for PSA
3. FH of Ca Prostrate: Who & at what age?
4. Do you smoke
5. Take alcohol
6. How is your diet?

D/D:
1. Are you on any medications?
2. Do you take psychiatric medications
3. Meds for Glaucoma?
4. H/O Stroke
5. H/o Urethritis
6. Sx of renal failure: Puffy face, swollen ankles

PMH:
Since this is the first time I’m seeing you, I need to ask you some qns regarding your
Past Medical History:
Do you have DM/HTN
Hospitalized or had any surgeries?
SOCIAL H:

Back to Content

Please note that this is only a draft version based on several sources, including: Dr. Basel Mohasen’s lectures,
Toronto Notes, Therapeutic Choices and others. Edited and organized for the sake of all attendances of the
Canadian Osce Exams: NAC OSCE and MCCQE2. by: Dr. Merlyn D’Souza and Dr. Zeev Gross, Spring 2011. Page 96
67 M brought in by daughter,as she is concerened that he is not himself
Greeting: As I understand ..........
Whenever a pt is brought in by someone always ask:DO U AGREE?
If Yes:...I’m glad you’re here as we can find a working solution
If NO: I would appreciate that you’re here just to make you daughter happy, I promise you I will be as
fast as I can.

Tell me more about the concern:


She is worried that I’m not going out as I used to before.
Do you stay at home?
I’m not going away to far places or out with my friends
When did this happen?
Do you share her concern?
My daughter overreacts
I’m Glad you’re here
What prevents you from going out?
GIVE CONFIDENTIALITY
I wet myself
When
How many times?
How did you react at that time?

The go to Urine History: Obstructn/Iritation/Urine

If Pt does not divulge, Go to Geriatric History:


1. What Medications?do you take & ask to see list or bag of meds
Do you take sleeping pills (specifically ask for that as it affects memory &
gait)OTC/Herbal products
2. Screen for Mood & Memory
3. Vision & Hearing/Balance & Falls
4. Urine: Retention in males & Incontinence in females
5. Sleep: How many hours
5.1 When do you go to bed?
5.2 Do you get up frequently?

Follow event..........
If still refuses to talk,do review of Sx
Back to Content

Please note that this is only a draft version based on several sources, including: Dr. Basel Mohasen’s lectures,
Toronto Notes, Therapeutic Choices and others. Edited and organized for the sake of all attendances of the
Canadian Osce Exams: NAC OSCE and MCCQE2. by: Dr. Merlyn D’Souza and Dr. Zeev Gross, Spring 2011. Page 97
67 M Dark urine x 1 week (H x 5 min)

D/D:
1. Bleeding/SE of warfarin
2. PSGN
3. Stone
4. Nephrotic syndrome
5. Renal or bladder Ca
6. Trauma
7. Berger’s disease
8. Infection

If CC is Dark Urine, make sure it is haematuria & not Jaundice


What do you actually mean? Dark like Tea/Cola or Red?
OCD
O:Sudden/gradual
C: from that time till now,is it:
Off & on or continous/Same
D:
Has it ever happened before?
P
Does it occur at the:
1. Beginning of stream?(Urethra)
2. End of the stream? (bladder)
3. Whole of the stream? (Kidney)
V:
Urine changes:COCA + Bld
Consistency,remember:
1. Frothy urine
2. Cloudy urine
3. Not clear urine
Obstrn
1. Difficulty initiating urine ----->Do you need to strain?
2. Did you notice change in stream?
3. Dribbling?
4. After passing urine, do you still need to pass more?
Irritation:
1. How many time do you need to go to Wash room: Now,Before At Night?
2. Does it affect your sleep?
3. Do you feel you need to rush to WC
4. Are you able to make it in time?
5. Have you ever lost control?
6. Burning sensation
7. Flank pain
8. Fever

Please note that this is only a draft version based on several sources, including: Dr. Basel Mohasen’s lectures,
Toronto Notes, Therapeutic Choices and others. Edited and organized for the sake of all attendances of the
Canadian Osce Exams: NAC OSCE and MCCQE2. by: Dr. Merlyn D’Souza and Dr. Zeev Gross, Spring 2011. Page 98
C Sx:
ASx:
1. Back trauma
2. H/O recent Sore throat or skin infection
If yes to sore throat: When was that/Was there swelling of feet/Puffy face?
3. H/o bleeding tendencies or blood thinners
If Yes to blood thinners: Which one/Why/How long/How much do you take/When was last
F/U/what was your last INR/What is the target/Any new medications/Any antibiotics?
4. Did you notice bleeding from any other sites?
Gums/Nose/Malena/CNS:Numbness,weakness,difficulty finding words ......
5. H/o stones

Back to Content

Please note that this is only a draft version based on several sources, including: Dr. Basel Mohasen’s lectures,
Toronto Notes, Therapeutic Choices and others. Edited and organized for the sake of all attendances of the
Canadian Osce Exams: NAC OSCE and MCCQE2. by: Dr. Merlyn D’Souza and Dr. Zeev Gross, Spring 2011. Page 99
Neuropathic Pain

Neurologic Disorders: Neuropathic Pain

Table 1: Types of Neuropathic Pain

Peripheral Neuropathic Pain Central Neuropathic Pain

 Nerve root pain  Central post-stroke pain

 Carpal tunnel syndrome  Spinal cord injury pain

 Trigeminal neuralgia  Brain injury

 Postherpetic neuralgia  Multiple sclerosis

 Incisional neuralgia  Syringomyelia

 Nerve trauma (causalgia)

 Phantom limb pain

Investigations
History with attention to:
o temporal profile and characteristics of the pain
o functional status, mood, quality of life, insomnia, sexual function, previous and current
treatments, especially concurrent medications
o present or past chemical dependency, especially if opioids are considered
 Physical examination:
o determine areas of sensory loss (hypoesthesia) and skin sensitivity characteristic of
neuropathic pain determine other neurologic findings that might indicate a progressive
lesion requiring imaging and surgery
o determine concurrent conditions that contribute to the pain problem, e.g., concomitant
muscular pain and psychological factors
 Other investigations:
o imaging with CT or MR scanning if a space-occupying lesion is suspected
o electromyography
o diagnostic sympathetic blockade if complex regional pain syndrome is suspected
o although there is no established therapeutic range, monitoring serum levels of tricyclic
antidepressants (TCAs) and antiepileptic drugs may help to assess adherence and guide
dosage

Guideline for Use of Opioids in Chronic Nonmalignant Pain


 Consider after other reasonable therapies have failed.

Please note that this is only a draft version based on several sources, including: Dr. Basel Mohasen’s lectures,
Toronto Notes, Therapeutic Choices and others. Edited and organized for the sake of all attendances of the
Canadian Osce Exams: NAC OSCE and MCCQE2. by: Dr. Merlyn D’Souza and Dr. Zeev Gross, Spring 2011. Page 100
 Perform a complete pain and psychosocial history, physical examination and
appropriate diagnostic tests. A history of substance abuse, tension-type headaches,
frequent migraine headache, muscular pain (myofascial pain, fibromyalgia) or pain
that appears to be largely determined by psychologic factors is a relative
contraindication to the use of opioid therapy.
 A single physician/prescriber/pharmacy is optimal. The prescriber may choose to set
up a contract with the patient. The agreement should specify the drug regimen,
possible side effects, the functional restoration program and that violations may
result in termination of opioid therapy.
 The opioid analgesic of choice should be administered around the clock and may
include a provision of “rescue doses” for breakthrough pain. Controlled-release
preparations include morphine, oxycodone, hydromorphone, tramadol and
transdermal fentanyl. Avoid meperidine primarily because of accumulation of its
excitotoxic metabolite normeperidine. Codeine is a poor analgesic for moderate to
severe pain because it has to be metabolized to morphine. Drug administration
should include a titration phase to minimize side effects. If a graded analgesic
response to incremental doses is not observed, the patient may not be opioid-
responsive, and opioid treatment should probably be terminated.
 The patient should be seen monthly or more often for the first few months and every
2–3 months thereafter. At each visit

1. assess pain relief (0–10 scale),


2. mood,
3. side effects,
4. quality of life,
5. adherence to functional goals and
6. presence of drug-related behaviour.
Optimally, affix a copy of the prescription and drug therapy flow sheet to the medical
record.
 The goal of opioid therapy is to make the pain tolerable. For some patients with
chronic noncancer pain (e.g., postherpetic neuralgia), the administration of an opioid
analgesic can mean the difference between bearable and unbearable pain.
Therapeutic Tips

 Two to three months constitutes a reasonable trial of medication for neuropathic pain.
 While patients frequently say they have used amitriptyline or carbamazepine or other agents,
these drugs have often been used in too high or too low a dose and for too short a period of
time. It is useful to re-institute these drugs to evaluate their effectiveness when used
appropriately: start low, go slow, increase dose until relief of symptoms or side effects occur and
treat side effects when possible.
 Be sure the patient understands the goals of therapy: reduction in pain from moderate or severe
to mild, at the price of some side effects that may be tolerable or treatable.
 Use a pain assessment tool, such as a scale of 0–10 where 0 is no pain and 10 the worst pain
imaginable, to evaluate pain with and without activity, and before and after medication.
 As a matter of course, prescribe an artificial saliva mouth spray with TCAs and a stool softener
with TCAs or opioids.
 Use controlled-release formulations of carbamazepine and opioids

Please note that this is only a draft version based on several sources, including: Dr. Basel Mohasen’s lectures,
Toronto Notes, Therapeutic Choices and others. Edited and organized for the sake of all attendances of the
Canadian Osce Exams: NAC OSCE and MCCQE2. by: Dr. Merlyn D’Souza and Dr. Zeev Gross, Spring 2011. Page 101
 It may be possible to reduce or gradually withdraw medication after initial control of pain and a
period of relief of 1–3 months (pain such as postherpetic neuralgia may resolve spontaneously
and trigeminal neuralgia may go into remission). Gradual reduction is important to avoid
withdrawal symptoms.
 Always consider combining pharmacotherapy with appropriate psychological and physical
measures.
 Try different drugs within a class (e.g., a TCA or a gabapentinoid such as gabapentin or
pregabalin), drugs of different classes and combination therapy (polypharmacy) for a possible
additive or synergistic effect; do not combine TCAs with SNRIs.
 If opioids are used, guidelines are important and should be worked through with the patient.
 A trial and error approach of scientifically unproven treatments is reasonable if standard
therapy fails.
 Repeated visits can provide important psychological support and hope for desperate patients as
trial and error approaches are utilized.
 If chronic neuropathic pain is being managed in general practice, semi-annual or annual visits to
a pain specialist (where available) help provide support to the family practitioner for
contentious approaches such as opioids, and offer the chance of a novel therapy for the patient

Back to Content

Please note that this is only a draft version based on several sources, including: Dr. Basel Mohasen’s lectures,
Toronto Notes, Therapeutic Choices and others. Edited and organized for the sake of all attendances of the
Canadian Osce Exams: NAC OSCE and MCCQE2. by: Dr. Merlyn D’Souza and Dr. Zeev Gross, Spring 2011. Page 102
HEADACHE:
Dd:
1. Tension H
2. Cluster H
3. Migraine
4. Temporal A
5. Cervical Spondylitis
6. Meningitis
7. SOL
8. SAH
9. Depression
10. Spousal abuse

Red Flags for Serious Headache


1. Age of onset Middle-aged to elderly patient
2. Type of onset Severe and abrupt
3. Temporal sequence Progressive severity or increased
frequency
4. Pattern Significant change in headache
pattern
5. Neurologic signs Stiff neck, focal signs, reduced
consciousness
6. Systemic signs Fever, appears sick, abnormal
examination
Caution: If headache does not fit typical pattern, a serious diagnosis can be
missed.
*****
Chronic Daily Headache & Medication
overuse Headache
Chronic headache occurs daily or almost daily for 15 days per month, for 6 months or longer. The most
common causes of these headaches are transformed migraine and chronic tension-type headache. In the
former there is history of migraine attacks and over several years the migraine attacks become more
frequent. Soon the migraine characteristics give way to chronic daily headache with a daily or near-daily
background headache that often resembles a typical “tension-type headache.” People with chronic
tension-type headache may have no history of distinct migraine.
Patients with these disorders frequently use excessive amounts of abortive agents, including
ergots, acetaminophen, ASA and opioid analgesics. They can have rebound headaches as a result
of medication-overuse, while some may have symptoms of depression or other psychological
comorbidities. Rebound headaches can also occur with the overuse of triptans. Most will
improve in days or a few weeks with the discontinuation of these medications, especially mixed
analgesics.
Please note that this is only a draft version based on several sources, including: Dr. Basel Mohasen’s lectures,
Toronto Notes, Therapeutic Choices and others. Edited and organized for the sake of all attendances of the
Canadian Osce Exams: NAC OSCE and MCCQE2. by: Dr. Merlyn D’Souza and Dr. Zeev Gross, Spring 2011. Page 103
Generally, simple analgesics should be used less than 15 days per month in primary headache disorders
such as migraine or tension-type headache or they will lead to the development of medication-overuse
headache and chronic daily headache. Further, if chronic daily headache develops, other useful abortive
and prophylactic medications usually have less efficacy.
Management includes recognition of these disorders, tapering and stopping the offending
agent(s), and starting a prophylactic medication such as amitriptyline or another agent listed in.
During withdrawal, particularly in patients with transformed migraine, use abortive agents such
as DHE or a triptan for treatment of the migraine headaches that emerge. Short-term admission
to hospital may be required to use the Raskin protocol (using DHE) and give support. If
psychological comorbidities such as depression are present, they must be managed and treated.
Consider referral to a multidisciplinary pain management clinic for cases failing to respond to
therapy.
Therapeutic Tips
 Give abortive treatment, without exceeding recommended dosages, as soon as possible.
 Use simple analgesics less than 15 days per month, and ergots, triptans, opioids or analgesic
combinations less than 10 days per month.
 A calendar or diary of headaches is useful in follow-up assessment.
 Keep a record of medications (usefulness, dosage and side effects).
 If migraine that does not respond adequately to symptomatic therapy occurs more than 3–4 times
per month, try prophylactic medications for several months and then discontinue if possible, to
assess ongoing need.2
 Different medications may need to be tried, including different members of the same class, such as
triptans.
 Follow-up is most important in managing chronic headache.
 Reassurance and explanation are most important to the patient in the long term.
 Always offer hope to patients with chronic headache even if no cure is available; most primary
headaches can be controlled.

Back to Content

Please note that this is only a draft version based on several sources, including: Dr. Basel Mohasen’s lectures,
Toronto Notes, Therapeutic Choices and others. Edited and organized for the sake of all attendances of the
Canadian Osce Exams: NAC OSCE and MCCQE2. by: Dr. Merlyn D’Souza and Dr. Zeev Gross, Spring 2011. Page 104
ACUTE HA x 10 days: 67M in ER
HISTORY:
OCD PQRSTUV
Onset: Sudden Vs Gradual
Course:
 All the time
 Is it increasing or decreasing or is it the same?
 VARIATION: Did you notice any variation?
 Is it the same throughout the day
 Does it awaken you at night? (EMPATHIZE++++)
Duration:
 How long the whole disorder
 How long each attack
 How frequent: off & on
Posn:
 Unilareral/Bilateral
 Where is it exactly?
 Does the part where it hurts is tender (Temporal Arteritis) & do you feel like a cord-like
structure there?
Quality:
 Throbbing
 Burning
 Tightness/Pressure
 Ice pick like
Radiation?
front,side,back of head,or in the eyes,ears or throats?
Severity:
On a scale of 1 – 10
Can you say It is the worst HA of your life?
Timing
Triggers:(not when single episode)
U:
Qns for empathy**
 How has it affected U in your daily life?
 How r U coping with it?
 How do U feel abt it?
 What r your expectations from today’s visit?

V= deja Vu
Has it happened before?
Aggravating factors:
 Eating (Jaw claudication)

Please note that this is only a draft version based on several sources, including: Dr. Basel Mohasen’s lectures,
Toronto Notes, Therapeutic Choices and others. Edited and organized for the sake of all attendances of the
Canadian Osce Exams: NAC OSCE and MCCQE2. by: Dr. Merlyn D’Souza and Dr. Zeev Gross, Spring 2011. Page 105
 Bending forwards/Coughing/Lifting/Lying down (Inc ICP)
 Lights/Certain foods etc (Migraine)
 Eyestrain (vision correction)
 Alcohol (cluster H)
Alleviating fcts:
Did you try any meds & were they helpful?
Assoc.Symptoms:
In addition to your headache did you notice any other symptoms:
(Try & do constitutional sx first as you may forget them)
Fever/Neck pain/Photophobia/Skin rash/Ear infection
NEURO Screening:
1. Vision changes:
What type of problem?
2. Hearing abnormalities
3. Difficulty swallowing
4. Weakness/Numbness
5. Difficulty finding words
6. Difficulty in balance or repeated falls
7. Changes n bowel/Urine Loss of bladder control
8. LOC
9. H/o Seizure

MOOD Changes
MEMORY problem
Changes in CONCENTRATION
Has anyone told you that you’ve ben acting strangely?

MSK Screening
Is there pain in your joints
For how long?
Can you raise your arms above your head?

H/O INJURY:
To head
Did you have a fall & hurt your head?

EXTRACRANIAL:
EYE:
Did you notice any redness or need eyeglasses?
Sinusitis:
Facial pain/flu
Throath pain
Dental pain

Please note that this is only a draft version based on several sources, including: Dr. Basel Mohasen’s lectures,
Toronto Notes, Therapeutic Choices and others. Edited and organized for the sake of all attendances of the
Canadian Osce Exams: NAC OSCE and MCCQE2. by: Dr. Merlyn D’Souza and Dr. Zeev Gross, Spring 2011. Page 106
RISK FCTS:
Do you smoke
Take EtOh
Take recreational drugs?

PMH:
Have you taken pain killers,if +ve: How much & for how long (rebound HA)
Did you take any OTC or herbal meds?
Are you allergic to anything?
Have you ever been diagnosed with HTN/DM/Stroke/MI/Ca?
Were you ever hospitalized or underwent Surgery?
FAMILY H:
HTN/DM/Stroke/MI
SOCIAL H:
Who lives with you?
How do you support yourself financially?
DIAGNOSIS: Temporal Arteritis

Back to Content

Please note that this is only a draft version based on several sources, including: Dr. Basel Mohasen’s lectures,
Toronto Notes, Therapeutic Choices and others. Edited and organized for the sake of all attendances of the
Canadian Osce Exams: NAC OSCE and MCCQE2. by: Dr. Merlyn D’Souza and Dr. Zeev Gross, Spring 2011. Page 107
24 M/HA 6 wks Office 10 mins H & C

Primary Headache Secondary Headache


Type % Type %
Migraine 16 Systemic infection 63
Tension-type 69 Head injury 4
Cluster 0.1 Vascular disorders 1
Idiopathic stabbing 2 Subarachnoid hemorrhage <1
Exertional 1 Brain tumor 0.1

Cluster headache is a rare form of primary headache

The pain is deep, usually retroorbital, often excruciating in intensity, nonfluctuating, and
explosive in quality.

A core feature of cluster headache is periodicity.

At least one of the daily attacks of pain recurs at about the same hour each day for the duration
of a cluster bout.

The typical cluster headache patient has daily bouts of one to two attacks of relatively short-
duration unilateral pain for 8–10 weeks a year; this is usually followed by a pain-free interval
that averages 1 year.

Cluster headache is characterized as chronic when there is no period of sustained remission.


Patients are generally perfectly well between episodes.

Onset is nocturnal in about 50% of patients, and men are affected three times more often than
women. Patients with cluster headache tend to move about during attacks, pacing, rocking, or
rubbing their head for relief; some may even become aggressive during attacks. This is in sharp
contrast to patients with migraine, who prefer to remain motionless during attacks.

Cluster headache is associated with ipsilateral symptoms of cranial parasympathetic autonomic


activation: conjunctival injection or lacrimation, rhinorrhea or nasal congestion, or cranial
sympathetic dysfunction such as ptosis.
OCD
ONSET:
If pt says this time was worst: Ask Prev episode,if present:
1. How long ago?
2. Did you seek medical attention then?
3. What was the diagnosis?
4. What Rx was given?

Please note that this is only a draft version based on several sources, including: Dr. Basel Mohasen’s lectures,
Toronto Notes, Therapeutic Choices and others. Edited and organized for the sake of all attendances of the
Canadian Osce Exams: NAC OSCE and MCCQE2. by: Dr. Merlyn D’Souza and Dr. Zeev Gross, Spring 2011. Page 108
5. Is this current HA different from from the previous one?
NOW GO to THE CURRENT HA.
Finish with the current HA & can go back to previous HA
ONSET:Gradual/Intermittent
COURSE:
Inc/Dec/Same
DURATION:
How Often?
How long does each episode last?
Everyday,few hrs,wkends longer& awaken at night?
POSN
QUALITY:
RSTUV
AlLEVIATING FCTS:
Sleep/Pacing/Dark room/Lying down
AGGRAVATING FCTS:
 Flashing lights
 Lack of sleep
 Certain food
 Alcohol (Cluster )
CONSTITUTIONAL Sx
Fever/chills/N Sweats/Loss of appetite & loss of wt/Lumps or bumps anywhere
TRAUMA:
RISK FCTS:
Are you under stress?
How do you handle stress?
Do you Smoke?......
Do you take Alcohol:
How much How long Why??
Have you used recreational drugs?
How is your MOOD?
Any chance that you may be depressed?
*MI PASS ECG
Mood:
Interest
Psychomotor retardation
Appetite
Sleep
Suicidal ideation
Energy
Concentration
Guilt
If M& I are +ve Look for depression

PMH:

Please note that this is only a draft version based on several sources, including: Dr. Basel Mohasen’s lectures,
Toronto Notes, Therapeutic Choices and others. Edited and organized for the sake of all attendances of the
Canadian Osce Exams: NAC OSCE and MCCQE2. by: Dr. Merlyn D’Souza and Dr. Zeev Gross, Spring 2011. Page 109
Are you taking any meds?/OTC/Herbal products?
Were you ever Diagnosed with HTN/DM/Ca
Were you ever hospitalized or had surgery?
FH:
SOCIAL HISTORY:
Who lives with you?
How do you support yourself financially?

Acute Attack Treatment


Cluster headache attacks peak rapidly, and thus a treatment with quick onset is required. Many
patients with acute cluster headache respond very well to oxygen inhalation. This should be
given as 100% oxygen at 10–12 L/min for 15–20 min.
Sumatriptan 6 mg subcutaneously is rapid in onset and will usually shorten an attack to 10–15
min;. Sumatriptan (20 mg) and zolmitriptan (5 mg) nasal sprays are both effective in acute
cluster headache, offering a useful option for patients who may not wish to self-inject daily. Oral
sumatriptan is not effective for prevention or for acute treatment of cluster headache.
Preventive Treatments
The choice of a preventive treatment in cluster headache depends in part on the length of the
bout. Patients with long bouts or those with chronic cluster headache require medicines that are
safe when taken for long periods. For patients with relatively short bouts, limited courses of oral
glucocorticoids or methysergide (not available in the United States) can be very useful.
A 10-day course of prednisone, beginning at 60 mg daily for 7 days and followed by a rapid
taper, may interrupt the pain bout for many patients.
When ergotamine (1–2 mg) is used, it is most effective when given 1–2 h before an expected
attack.
Patients who use ergotamine daily must be educated regarding the early symptoms of ergotism,
which may include vomiting, numbness, tingling, pain, and cyanosis of the limbs; a weekly limit
of 14 mg should be adhered to.
Lithium (600–900 mg qd) appears to be particularly useful for the chronic form of the disorder.
Table 15-9 Preventive Management of Cluster Headache

Short-Term Prevention Long-Term Prevention


Episodic Cluster Headache Episodic Cluster Headache & Prolonged
Chronic Cluster Headache
Prednisone 1 mg/kg up to 60 mg qd, Verapamil 160–960 mg/d
tapering over 21 days
Methysergide 3–12 mg/d Lithium 400–800 mg/d
Verapamil 160–960 mg/d Methysergide 3–12 mg/d
Greater occipital nerve injection Topiramatea 100–400 mg/d

Gabapentina 1200–3600 mg/d

Back to Content
Please note that this is only a draft version based on several sources, including: Dr. Basel Mohasen’s lectures,
Toronto Notes, Therapeutic Choices and others. Edited and organized for the sake of all attendances of the
Canadian Osce Exams: NAC OSCE and MCCQE2. by: Dr. Merlyn D’Souza and Dr. Zeev Gross, Spring 2011. Page 110
35F HAx 6 wks H & C
Can you tell me abt it since you first noticed it?
I’m glad you came in today,do you have the HA now?Any particular reason as to why you came
in today?
OCD
Empathize+++
PQRSTUV
CONSTITUTIONAL Sx
Local Sx:
RISK FCTS:
Smoking/alcohol/recreational drugs
PMH:
 Are you on any medications
 Are you on the Contraceptive Pill?
 Was it changed recently?
 Did you notice any relation to the HA & the Pill?
 Any OTC/Herbal meds?
 HTN?DM?MI?STROKE?CA?
Any hospitalizations or Surgery?
FH:
Similar HA in any one of your family members?
FH of HTN/DM/Stroke/MI
SOCIAL H:
Stress in your life?
Who lives with you? Look out for Domestic Violence....
How do you support yourself financially?
COUNSELLING:
Migraine can be related to the pill
Disct the OC or change the particular pill & switch to another form of pill or contraception like
IUCD or barrier method
o avoid triggers, especially in migraine, e.g., too much or too little sleep, irregular
meals, lack of regular exercise, extremes of stress or relaxation, known dietary
triggers
o apply ice; sleep or rest in a dark, noise-free room
DIAGNOSIS:MIGRAINE
Back to Content

Please note that this is only a draft version based on several sources, including: Dr. Basel Mohasen’s lectures,
Toronto Notes, Therapeutic Choices and others. Edited and organized for the sake of all attendances of the
Canadian Osce Exams: NAC OSCE and MCCQE2. by: Dr. Merlyn D’Souza and Dr. Zeev Gross, Spring 2011. Page 111
45 M HAx 4 wks
OCD
PQRSTUV
This scenario Pt has typically gets HA at work,better at wkends & when he is drivng home.
Alert to possibility to exposure to something at work.
On H/o :
 What sort of Job, he was a forklift operator.
 Ask which sort of Environment he works whether it is:
1. Open or closed
2. Operated by electricity or gas
3. Presence or absence of ventilation
4. Presence of Carbon monoxide alarm,whether it has ben checked
5. If anybody else in the work place has a similar HA

Back to Content

Please note that this is only a draft version based on several sources, including: Dr. Basel Mohasen’s lectures,
Toronto Notes, Therapeutic Choices and others. Edited and organized for the sake of all attendances of the
Canadian Osce Exams: NAC OSCE and MCCQE2. by: Dr. Merlyn D’Souza and Dr. Zeev Gross, Spring 2011. Page 112
40F/Looks older Weakness (R) arm x 6 hrs History x 5 mins & review of Sx

D/d: Vitamin D
Vascular:Stroke/ICH/TIA
INFECTION: Abscess/Meningitis/Encephalitis
Traumatic: Head Injury
Autoimmune:Vasculitis
Metabolic: electrolyte abnormalities/Hyperthyroidism/Uremia
Idiopathic:Syncope/MS
Neoplastic: Mets or Pirmary Brain T
Drugs: EtOH/Cocaine/Phencyclidene/Amphetamine

OCD:
O: Sudden/Gradual
What were you doing at that time it occured
C: Is it getting worse?
D:

PQRST UV
Quality of defeciet: Sensory/Movt/Power
1. How weak is it?
2. Can you move at all?
3. Partially weak?
U:How has it affected your life? (ADLs)
Gross motor:(Reaching shelves/Opening doors)
Fine Motor:Buttoning shirt/using keys/writing
V: Have you had such episodes previously?
OTHER LIMB: what abt (R) Leg/(L) Arm & (L) Leg

Assoc Sx:
Local Sx:
 Parasthesias/Pain
 Calf Pain/Swelling
 Recent travel/Immobilization
CNS:
HA/Dizziness/LOC/Visual disturbances (amaroux Fugax)/Slurred speech
CVS:
Palpitations/Chest pain

Please note that this is only a draft version based on several sources, including: Dr. Basel Mohasen’s lectures,
Toronto Notes, Therapeutic Choices and others. Edited and organized for the sake of all attendances of the
Canadian Osce Exams: NAC OSCE and MCCQE2. by: Dr. Merlyn D’Souza and Dr. Zeev Gross, Spring 2011. Page 113
CONST Sx:
Fever/Chills/wt loss/Lumps & Bumps
Trauma: or injury
Bladder: any urinary problems (R/O MSclerosis)
RISK FCTS:
Smoke/Alcohol
Was your blood ever checked for cholesterol & Sugar? .......
When/if on any Rx .........
OC
DM/HTN/Stroke/Ca/MI
Do you have a form of regular exercise?

PMH:
 Are you on any Meds/OTC/Herbal products (particularly Asa/Warfarin/Blood thinners)
 Do you have any allergies
 Were you ever hospitalized or had any Surgeries?
 Do you have any Peptic Ulcers

FH:
Does anybody else in Family have such a condition
HTN/DM/Stroke?MI
SOCIAL H:
Who lives with you?
How do you support yourself financially?

Please note that this is only a draft version based on several sources, including: Dr. Basel Mohasen’s lectures,
Toronto Notes, Therapeutic Choices and others. Edited and organized for the sake of all attendances of the
Canadian Osce Exams: NAC OSCE and MCCQE2. by: Dr. Merlyn D’Souza and Dr. Zeev Gross, Spring 2011. Page 114
Alteplase in Acute Ischemic Stroke: Treatment Criteria
Treatment criteria
1. Ischemic stroke in a patient ≥ 18 years
2. Stroke onset > 1 h and ≤ 4.5 h before alteplase administration
3. Stroke deficit that is disabling or measurable on the NIH Stroke Scale
4. No intracranial hemorrhage on CT or MRI scan
Exclusion criteria
1. Time of stroke onset unknown or > 4.5 h
2. Any hemorrhage on brain CT or MRI scan
3. Symptoms suggestive of subarachnoid hemorrhage
4. CT or MRI signs of acute hemispheric infarction involving more than 1/3 of the MCA
5. History of intracranial hemorrhage
6. Stroke or serious head or spinal trauma within the preceding 3 mo
7. Seizure at stroke onset
8. Systolic blood pressure ≥ 185 mm Hg or diastolic blood pressure ≥ 110 mm Hg or aggressive
treatment (intravenous medication) necessary to reduce blood pressure to these limits
9. Recent major surgery

Please note that this is only a draft version based on several sources, including: Dr. Basel Mohasen’s lectures,
Toronto Notes, Therapeutic Choices and others. Edited and organized for the sake of all attendances of the
Canadian Osce Exams: NAC OSCE and MCCQE2. by: Dr. Merlyn D’Souza and Dr. Zeev Gross, Spring 2011. Page 115
10. Arterial puncture at a noncompressible site within the previous 7 days
11. Elevated activated partial thromboplastin time
12. International normalized ratio > 1.7
13. Platelet count < 100 × 109/L
14. Blood glucose concentration < 2.7 or > 22 mmol/L
15. Any other condition that could increase the risk of hemorrhage after alteplase administration

Alteplase in Acute Ischemic Stroke: Monitoring5


Blood Pressure and Neurological Signs

• Baseline, then Q15min × 2 h after starting alteplase


• Then Q30min × 6 h
• Then Q1H until 24 h after starting alteplase
• Call MD if the systolic BP is > 180 mm Hg or if the diastolic BP is > 110 mm Hg on 2 or more occasions
taken 5–10 min apart
• Stop the infusion, obtain emergency CT scan and notify MD if there is neurologic deterioration, severe
headache, or new onset of nausea or vomiting
Blood Glucose
• Call MD if glucose > 12 mmol/L
Lines and Tubes
• Delay placement of nasogastric tubes, indwelling catheters or intra-arterial pressure catheters
Medications
• No ASA, ticlopidine, clopidogrel, heparin or warfarin for 24 h
• Acetaminophen 650 mg po or pr Q4H if body temperature is ≥ 38°C or for analgesia
• O2 via nasal prongs or face mask to keep O2 saturation > 90%
• After the alteplase infusion is completed, continue iv normal saline (with or without KCl)
Investigations
• CT brain scan after 24 h
Carotid endarterectomy (CEA)2
Patients with carotid territory transient ischemic attack or nondisabling stroke and ipsilateral 70–
95% internal carotid artery stenosis should be offered carotid endarterectomy within 2 weeks of
the incident transient ischemic attack or stroke unless contraindicated.
CEA is also appropriate for selected patients with moderate (50–69%) symptomatic stenosis.
These patients should be evaluated by a physician with expertise in stroke management. Carotid
stenting may be considered for patients who are not CEA candidates for technical, anatomical or
medical reasons.

Antiplatelet therapy

 If intracranial hemorrhage is excluded by CT scan, but alteplase is not indicated, give ASA 160 mg
immediately. This is followed by ASA 80–325 mg daily.
 When alteplase is used, wait until intracranial hemorrhage is excluded by CT scan 24 hours later
and give ASA 160 mg once. This is followed by ASA 80–325 mg daily.

Please note that this is only a draft version based on several sources, including: Dr. Basel Mohasen’s lectures,
Toronto Notes, Therapeutic Choices and others. Edited and organized for the sake of all attendances of the
Canadian Osce Exams: NAC OSCE and MCCQE2. by: Dr. Merlyn D’Souza and Dr. Zeev Gross, Spring 2011. Page 116
 Administer ASA as a suppository or via nasogastric tube to dysphagic patients. Use enteric-coated
formulation for patients who can swallow. No evidence supports the use of ASA doses greater
than 325 mg/day for secondary stroke prevention. The GI side effects of ASA are dose related.
 For patients who were taking ASA prior to their stroke, consider other antiplatelet agents, such as
clopidogrel 75 mg daily or a combination of ASA and sustained-release dipyridamole 25/200 mg
twice daily, although these regimens have not been tested in acute stroke.
 The combination of ASA and clopidogrel is not recommended for long-term secondary stroke
prevention.

Anticoagulant therapy

 Immediate systemic anticoagulation with unfractionated heparin, low molecular weight heparin,
heparinoids or specific thrombin inhibitors is not recommended in the setting of acute ischemic
stroke, not even for patients in atrial fibrillation (AF), because there is no evidence of short- or
long-term benefit. Specifically, reduction in early recurrent ischemic stroke is completely offset by
an increase in major intracranial and extracranial bleeding.18
 ASA is as effective as warfarin for secondary stroke prevention in patients in normal sinus rhythm,
and does not require laboratory monitoring.
 For patients in AF, use warfarin at a dose to maintain the INR in the range 2.0 to 3.0, provided
there are no contraindications to anticoagulation. For patients who cannot take warfarin, use
enteric-coated ASA 80–325 mg daily.
 The best time to initiate anticoagulant therapy is unclear. For patients with minor strokes, start
warfarin as soon as intracranial hemorrhage has been excluded by CT scan. For patients with
major strokes, delay warfarin until a CT scan done about a week or two after the stroke has
excluded hemorrhagic transformation of the infarct.

Blood pressure lowering treatment


Randomized controlled trials have not defined the optimal time to initiate blood pressure lowering
therapy after stroke.19 Oral blood pressure lowering treatment should be initiated (or modified)
prior to discharge from hospital in patients whose blood pressure is ≥ 140/90.

IV. Restore Function of the Individual


 Outcomes are optimized by care on a stroke unit provided by a coordinated interdisciplinary team
(Start rehabilitation as soon as the patient is medically stable.
 Family and community supports are important for social reintegration.

Therapeutic Tips
 The effectiveness of thrombolytic therapy with alteplase is exquisitely time dependent; delays of
any sort should not be tolerated. A minority of patients present to hospital within the first 90
minutes of stroke onset, leaving limited time to act. Immediate contact with the patient, rapid
triage, and (most importantly) staying with the patient continuously during the clinical
assessment, CT scan, blood tests and consent procedures are vital in ensuring that the
appropriate steps are being taken as rapidly as possible prior to alteplase administration. For
example, it is not necessary to wait for hospital porters to take the patient to the CT scanner.

Please note that this is only a draft version based on several sources, including: Dr. Basel Mohasen’s lectures,
Toronto Notes, Therapeutic Choices and others. Edited and organized for the sake of all attendances of the
Canadian Osce Exams: NAC OSCE and MCCQE2. by: Dr. Merlyn D’Souza and Dr. Zeev Gross, Spring 2011. Page 117
 Determining the time of stroke onset is critical in deciding to use alteplase, but checking the clock
is not a natural reaction in the setting of an acute stroke. Encourage patients and families to think
of “time anchors” (e.g., what was on the radio or TV at the time, or at what point in the patient's
daily routine did the symptoms first occur).
 Patients with acute stroke are often unable to communicate. When possible, the next-of-kin
should travel with the patient to hospital (or between hospitals if the patient is transferred) to
provide collateral history and consent for treatment before the time window for intervention
closes.
 If the patient is referred to a tertiary care hospital, have the stat blood work (CBC, INR) drawn at
the community hospital and the results faxed to the referral centre as soon as possible.
 Point-of-care INR testing , if available, can provide results quickly.
 Signs of infarction on a CT scan done within 4.5 hours of stroke onset are usually subtle. If the CT
scan of a patient being considered for treatment with alteplase shows a very definite infarct in a
location that explains the presenting clinical symptoms and signs, recheck the time of onset.

Back to Content

Please note that this is only a draft version based on several sources, including: Dr. Basel Mohasen’s lectures,
Toronto Notes, Therapeutic Choices and others. Edited and organized for the sake of all attendances of the
Canadian Osce Exams: NAC OSCE and MCCQE2. by: Dr. Merlyn D’Souza and Dr. Zeev Gross, Spring 2011. Page 118
A 30 YOF with right arm weakness for 10 hours, Hx for 5m
Intro
Where is your weakness?
Can you still work with your hand or no?
Do you have burning or tingling sensation on your hand or shoulder? How about numbness?
Any problem on your right foot? Lt. arm or leg?
Is it the first time? (If the patient says that she had it before than: “When was it? How long did
it last? Which medication did she take?)
Did you fall or lost your consciousness?
Any change in your vision? Loss of vision? Double vision? Blurry vision?
Any change in your hearing? Buzzing sound? Diffucult in finding words? Any change in balance?
Any change in urination and bowel movement?
When you bent your neck do you fill electrical shock along your spine?
Do you difficult to swallow?
Have you ever had dizziness, headlightedness, loss of consciousness, jerky movement, seizure?
How is your mood / concentration / memory?
Any change in your personality?
When you touch your face do you feel any electrical shock?
Uhthoff’s sign: when they get hot water or hot weather – trigger for their symptoms (especially
optic neuritis).
Review systems from head to toe: chest pain, heart racing, sob, cough and phlegm, abdominal
pain, nausea and vomiting, joint pain, skin rash, diabetes, thyroid disease, anemia
CSx
RF for MS, PMHx, FMHx

Back to Content

Please note that this is only a draft version based on several sources, including: Dr. Basel Mohasen’s lectures,
Toronto Notes, Therapeutic Choices and others. Edited and organized for the sake of all attendances of the
Canadian Osce Exams: NAC OSCE and MCCQE2. by: Dr. Merlyn D’Souza and Dr. Zeev Gross, Spring 2011. Page 119
Please note that this is only a draft version based on several sources, including: Dr. Basel Mohasen’s lectures,
Toronto Notes, Therapeutic Choices and others. Edited and organized for the sake of all attendances of the
Canadian Osce Exams: NAC OSCE and MCCQE2. by: Dr. Merlyn D’Souza and Dr. Zeev Gross, Spring 2011. Page 120
PAEDS
1. Pediatrics (30)
2. Psychiatry (30)
3. Physical exam (30)
4. Management (12)
5. OBGYN (
6. Communication Skills (10)
7. Counseling (10)
8. Medicine (CVS 15, Neu 15, Med 20)

Pediatrics
Consider abuse
There are no children in the room, only parents.
Maternity leave – either husband or wifes.

Please note that this is only a draft version based on several sources, including: Dr. Basel Mohasen’s lectures,
Toronto Notes, Therapeutic Choices and others. Edited and organized for the sake of all attendances of the
Canadian Osce Exams: NAC OSCE and MCCQE2. by: Dr. Merlyn D’Souza and Dr. Zeev Gross, Spring 2011. Page 121
A child 9m – chronic diarrhea (CF, Celiac, HIV; Lactose deficien cy)

Back to Content
5y.o fever – take history.
Skin rash – ask questions about it (distribution, relation to vfever)
HSP

Back to Content
Son, 3y.o is coughing for 4wk and they want to renew his antibiotics
This shows there was a condition
Ask What Ab,for which condt,When?.
Don’t waste your time –R/O: Hyperactive airways or is it infection that has not cleared,or could be
asthma.

Back to Content
A mother just delivers a baby who is IUGR
all questions should about pregnancy and delivery.
1. Reassure her,
2. Note appearance of the child
3. Note Paediatrician’s visit,
4. History of pregnancy/Obstetric History
5. Family history.
In case mum was smoking,taking alcohol & drugs & asks if her fault if child has IUGR
Don’t reproach her – it is NOT her mistake.
It is a multi-factorial condition.Can be due to various causes,some genetic,pregnancy,related to baby
Because safe levels of smoking,drugs & alcohol not known,
We always recommend not to smoke or drink for futurepregnancies.

Parents are concerned that their child is not growing enough: [AGEx2+8]
What his weight in birth.

14m 8Kg (birth weight 3.5Kg). He is underweight.

CC
OCD
COCA-B
AA
ASx
PMHx

Please note that this is only a draft version based on several sources, including: Dr. Basel Mohasen’s lectures,
Toronto Notes, Therapeutic Choices and others. Edited and organized for the sake of all attendances of the
Canadian Osce Exams: NAC OSCE and MCCQE2. by: Dr. Merlyn D’Souza and Dr. Zeev Gross, Spring 2011. Page 122
SHx
0-6m: BINDE
Birth – Pregnancy:
Was it a planned pregnancy?
1. Did you have any regular follow-up?
2. Did you have any US? Was it normal or not?
3. During your pregnancy did you have any fever or skin rash?
4. Any contact with sick person or cats?
5. Any medication/smoking/drugs/alcohol?
6. Screened for HIV/Syphilis/GBS/Hepatitis B? Blood group?
Birth – Delivery:
1. Was it in term or not?
2. What is the route? (Cs/NVD)
3. How long it took? (18hr is normal for primi, 12hr for multi),
4. Early gush of water?
5. Any need for augmentation?
6. What was the APGAR score?
7. Did the baby cry immediately?
8. Did your baby need any special attention?
9. Any bulging or bruising in his body?
10. When were you sent home?(C/S 3d, V/D – 1d).
11. After delivery did you have any fever/vaginal discharge/any medication?
12. Were you told that your baby had any congenital deformity?

Immunization – if he says that the child is not immunized you have to inquire for the reason.
If he is not vaccinated because the parent is busy – look for child abuse RED FLAG.Ask wt &
milestones
If it is due to religion believe – you don’t have to ask more. Otherwise – ask about nutrition.

Nutrition –
WEIGHT:
1. What his weight today,
2. Weight at birth,
3. Highest weight,
Growth chart.
X (birth), 5m-2x, 1y-3x, 2y-4x. Weight: Agex2+8
H (birth, about 50cm), 1y-1y, 2y-1.75H (half of his adult height), 4y-3.5H
HC (at birth): 35cm
What do you FEED your baby
If formula: –
When did you start the formula
If B Fed at all
Did you consider B feeding?
what type of formula do you use?
How do you prepare it?

Please note that this is only a draft version based on several sources, including: Dr. Basel Mohasen’s lectures,
Toronto Notes, Therapeutic Choices and others. Edited and organized for the sake of all attendances of the
Canadian Osce Exams: NAC OSCE and MCCQE2. by: Dr. Merlyn D’Souza and Dr. Zeev Gross, Spring 2011. Page 123
Was there any changes in the feeding?
Did you add any solid food or supplements (any fortified serials or iron) do you feed him with
any bread, solid food –
when started the diarrhea (before the solid food or after?)

Development –
At the end 1y they use words, 2y – two words at one sentence, 3y – 3words in one sentence;
4y – speak normally.
Gross motor: role – 4m, seat- 6m, crowling – 9m, standing – 1y, climbing upstairs – 18y,
riding bicycle – 3y

Environment –
with whom do you live at home?
Any other children?
Relation between your child and other households?
Who spends most of the time with the child?
Financially how do you support yourself?
Do you live in your own house?
Do you have basement in your house?
Anybody drinks or uses drugs?
Building – basement (mold) and
Old houses (lead poisoning).
6-12m: School Performance: comparing the grades between now and previous.
>12-14 yrs: HEAADDSSS
Home: with whom do you live?
Education: Which grade?
How are your marks?
What do you want to be?
Recent drop in grades?
Activities: Any hobbies? (in case of epilepsy – ask for the risky activities)
Alcohol: do you smoke, drink, (a lot of people of your age might experiment with drugs? How
about you?), Smoking
Diet: do you have any special diet?
Drugs: have you ever tried recreational drugs?
Smoke
Sexual: are you in relationship?
Suicide: how is your mood?

Questions for dehydration: Does baby have tears when crying?


How many times you pee?
How many times you change his diapers?

Back to Content

Please note that this is only a draft version based on several sources, including: Dr. Basel Mohasen’s lectures,
Toronto Notes, Therapeutic Choices and others. Edited and organized for the sake of all attendances of the
Canadian Osce Exams: NAC OSCE and MCCQE2. by: Dr. Merlyn D’Souza and Dr. Zeev Gross, Spring 2011. Page 124
5 Day old infant with yellow discoloration since he was 2 days old
A mother who is after 5days from deliver (if she uses “jaundice” – what do you mean be that?).
Is it early in the second day is it pathological.
Late in the second day – it has no value.
Make sure that the baby is stable – Red flags:
1. High pitched crying,
2. Poor feeding & Poor sucking
3. Floppy baby,
4. If above three are present, it is a problem, decide to reassure her or tell her you’ve to do a
physical exam& admit
If you have to take history and counsel ------>Reassuarence
Only history-------->Pathological
m/p it is not physiology

What is the name of your child?


(He is yellow)
Good you are here; I hope you can reassure you at the end.
OCD
Tell me more about it since the moment it started
O:
Is it early in the second day is it pathological.
Late in the second day – it has no value
C:
D:
Where did you notice it?
Is it spreading?
Did it reaches the legs?
Is it getting darker?
In addition to it did you notice any dark urine?
Pale stool (in bliary atresia – pale stool from the beginning?)
ASx:
INFECTION:
In addition to that did you notice any
fever, cough, discharge from ear,discharge diarrhea, vomiting, foul smell urine,
is he crying, is he floppy, is he sucking well, (RED FLAGS)
Rash,
Dehydration:
how many diapers did he change, any tears
Transition: I am going to ask you some questions to see if any conditions cause this issue back to
your pregnancy
FHx: of liver disease and blood disease. If she are concerned – why you are concerned (will he be
mental retarded).

Back to Content

Please note that this is only a draft version based on several sources, including: Dr. Basel Mohasen’s lectures,
Toronto Notes, Therapeutic Choices and others. Edited and organized for the sake of all attendances of the
Canadian Osce Exams: NAC OSCE and MCCQE2. by: Dr. Merlyn D’Souza and Dr. Zeev Gross, Spring 2011. Page 125
Mike Tyson, Child crying for the last 10d, 6w years old.
A child who’s with infantile colic.
How do you feel when your child is crying.
Crying child could be colic,screen for abuse,see how parents handle it
The crying might cause abuse

Weight, dehydration,
Start to observe the body language of the father.
OCD
O;
At that time was there any illness,like fever,runny nose
C;
Off & On/all the time
Every single day,every day,how many days/week
also “is he crying during the night?” – how does it affect you and your wife?)
Aggravating FCts:
Any chance he is hungry?
Any chance he is wet & neds a diaper change?
Diaper rash?
Any chance he is too hot or cold?
Alleviating Sx:
Do you soothe him/hug him/carry him & walk/take hime for a ride/listen to music?
Do you burp him/rock him/Skhe him?
If Yes: How many times?
When was the last time?
What happens to him when you shake him?
Does he stop crying?
Does he pass out?
(Children at this age cannot express their discomfort& only means of communication is by
crying“I am going to ask some questions to see if there is any reason for this crying?”
ASx;
INFECTION:
Fever, sweating, tender points in his body
Running nose, coughing, vomiting, discharge from his ear, yellow discoloration, fowel smell
urine,
GI
does he have distension of abdomen
Gases
Does he draw up his legs & cry
Any relation to feeds

BINDE (Partial)
N 1st
P
Planned pregnancy,
Reg F/u
Was it term P
Please note that this is only a draft version based on several sources, including: Dr. Basel Mohasen’s lectures,
Toronto Notes, Therapeutic Choices and others. Edited and organized for the sake of all attendances of the
Canadian Osce Exams: NAC OSCE and MCCQE2. by: Dr. Merlyn D’Souza and Dr. Zeev Gross, Spring 2011. Page 126
Any illness
Smoke/Drugs, Term, Complicated, Needed special attention, separation,
any congenital abnormality
Environment: financial how do you support yourself, any financial stress, with whom do you live,
repeated visits to ER, anyone in home have psychiatric problems/drugs/alcohol, relationship
with your partner
PMHx – diseases, hospitalizations

Back to Content

Please note that this is only a draft version based on several sources, including: Dr. Basel Mohasen’s lectures,
Toronto Notes, Therapeutic Choices and others. Edited and organized for the sake of all attendances of the
Canadian Osce Exams: NAC OSCE and MCCQE2. by: Dr. Merlyn D’Souza and Dr. Zeev Gross, Spring 2011. Page 127
Rita Gordon, mother of 5w who vomit for the last 10d
Pyeloric stenosis
GERD
Infection
Alleric to milk
overfeeding
Not Pyloric stenosis
If the colour of vomit is yellow or greenish discolouration
Not projectile + Wt loss

GERD
No wt loss at 6 weeks
Wt loss at 18 mo due to anaemia,due to bleeding due to oesophageal bleeding

Confidential – give it early according to the cues.


Depressed: after my son become vomit or preceding the vomit. Not reliable history.

CC
OC fD
O:
C;
Off & on/All the time
How many/day
Increasing/Decreasing or same?
COCA±B
Forceful
Feed: Formula/Breast?
COCA
How much F do you give? & How much does he vomi t out?
AA:
Any particular posn improves it? ( GERd upright better)
IMPACT:
WT & Dehydration
Do you feel he is still hungry after you feed him?
How many diapers do you change,Now & at the beginning?
ASx: wt & s/o dehydration
Gerd – no weight loss
Pyeloric stenosis – yellowish colour (ask specifically about the colour, relation to feeding – up to
half hour can be related to PS, what about position, do you burp him?)
ASx:
Infection – any signs of infection
BINDE:
N 1st
If formula fed?
Did you change the Formula?
Have you considered breastfeeding? Is there any reason not to breast feeding?

Please note that this is only a draft version based on several sources, including: Dr. Basel Mohasen’s lectures,
Toronto Notes, Therapeutic Choices and others. Edited and organized for the sake of all attendances of the
Canadian Osce Exams: NAC OSCE and MCCQE2. by: Dr. Merlyn D’Souza and Dr. Zeev Gross, Spring 2011. Page 128
Overfeeding – overweight
Allergy – less likely if she uses it from birth
Abdominal distended
B
PWas it planned pregnancy
Were there regular F/u?
How do yu feel about being a mum?
MOOD & INTERSET
Any chance of being depressed?
Any chance you feel like harming the baby or yourself?
Do you have any support at home?
“I see you are preoccupied / overwhelmed”
Child abuse/neglect
If there is a growth chart – it it is from the beginning.
4 min on the child, last 1 min to concentrate to the mother.

Back to Content

Please note that this is only a draft version based on several sources, including: Dr. Basel Mohasen’s lectures,
Toronto Notes, Therapeutic Choices and others. Edited and organized for the sake of all attendances of the
Canadian Osce Exams: NAC OSCE and MCCQE2. by: Dr. Merlyn D’Souza and Dr. Zeev Gross, Spring 2011. Page 129
Sandra Bullock, 19 y.o, 8m child, pale.
Intro
Name of the child
CC: Anemia
?
OCD
IMPACT
Causes: Red Flags (bleeding & BINDE)
Past MH
FH
ENv: Old house
Pale – what do you mean? Who told you that
OCD

If told by another person/ If you think about it,any chance he was pale before that or just
now?,& you were unaware
I like to see how it has affected your son:
IMPACT: Is he as active as before?
Crawl? Playful as before? LOC? Heavy breathing? Stop to breath when you feed him?
T:
I’m going to ask you some questions that could be the cause of this?
Asx:
Infection:
Sweat, Fever, Loose of weight, Painful points?
Does your child have bleeding? Bruises? Coughing blood? Tarry stool?
BINDE:
N 1st
What do you feed him? (B/F)
Any solids /supplements
P:
Was it a term Preganacy?
IMMUNIZATION:
ENV:
With whom do you live?
Any financial concerns?
Old/new home
Do you’ve a supportive family?
PMH:
FH:
Any bleeding disorder?
Repeated lver disease
Any gall bladder disease or splenectomy
Certain blood disease are more common in particular parts of the world & for that reason I need
to know your & partner’s ethnic background.

Back to Content
Please note that this is only a draft version based on several sources, including: Dr. Basel Mohasen’s lectures,
Toronto Notes, Therapeutic Choices and others. Edited and organized for the sake of all attendances of the
Canadian Osce Exams: NAC OSCE and MCCQE2. by: Dr. Merlyn D’Souza and Dr. Zeev Gross, Spring 2011. Page 130
ANEMIA: 29/F MCV Inc (Counsel)
D/d:[TN10/H21]
A. MEGALABLOBALSTIC:
a) B12 defeciency:
I. Diet (vegan)
II. Gastric:
a) Mucosal atrophy
b) Pernicious An
c) Post G-ectomy
III. Intestinal Absorption
a) Malabsorption (Crohn’s,celiac sprue,pancreatic disease)
b) Stagnant bowel (blind loop,stricture)
c) Fish tapeworm
d) Resection of ileum
b) Folate deficiency
I. Diet
II. Intestinal malabsorption
III. Drugs/Chemicals:
a) Alcohol
b) Anticonvulsants
c) Methorexate
d) Birth control pills
IV. Inc demands:
V. Pregnancy/Hemolysis/Hemodialysis/Psoriasis
c) Drugs (Methroxate,azathioprine)
B.Non Megalobalstic:
I. Liver disease
II. Alcohol
III. Hypothyroid
IV. Myelodysplastic syndromes
Start By saying: I’ve the results of your test with me & before I proceed I need to get some
information abt you that will help me understand:
If Pt asks if Serious: STOP & ask WHAT is her concern.
There can be many reasons for this result,though most are simple,however some can be
serious,
2QNS:
1. What is the reason for doing the test
2. Is it the first time?
Then explain the results

Please note that this is only a draft version based on several sources, including: Dr. Basel Mohasen’s lectures,
Toronto Notes, Therapeutic Choices and others. Edited and organized for the sake of all attendances of the
Canadian Osce Exams: NAC OSCE and MCCQE2. by: Dr. Merlyn D’Souza and Dr. Zeev Gross, Spring 2011. Page 131
 Search for the cause of iron deficiency, including very careful consideration of occult
gastrointestinal bleeding
 Menorrhagia must be convincing before it is accepted as the sole cause of iron deficiency. This
may be an opportunity for early recognition of a gastrointestinal malignancy—don’t miss it!
 A reticulocyte response should be evident within one week of beginning iron therapy, with
subsequent improvement in the Hgb of about 10 g/L every 7–10 days.
 If the Hgb fails to respond as anticipated, consider that there may be:
o ongoing blood loss
o use of other medications that impair iron absorption
o a different or concurrent cause of anemia and/or an impaired erythropoietic response
o compliance issues
 Gastrointestinal side effects are the most common reasons for non-compliance:
o use a graduated approach to dosing. Begin with a single tablet taken after a meal. On a
weekly basis, as tolerance permits, add another tablet until the patient is taking one
dose with each meal. Thereafter, gradually shift the timing of the doses to the beginning
of meals
o small oral doses may be adequate in patients that are susceptible to gastrointestinal
upset.
In the elderly, daily doses of elemental iron as low as 15 to 50 mg are effective in the
treatment of iron deficiency anemia9
In pregnant women, 20 mg/day of elemental iron, started at 20 weeks' gestation, is
sufficient to prevent iron deficiency11
o iron contained in enteric-coated tablets is poorly absorbed. These products should be
avoided
 Some physicians replenish iron stores while others prefer to stop therapy when the Hgb
normalizes, so that further blood loss will not be masked by robust iron stores. As a
compromise:
o completely replenish iron stores when the cause of iron deficiency has been identified
and corrected
o do not replenish iron stores when investigation has failed to

Please note that this is only a draft version based on several sources, including: Dr. Basel Mohasen’s lectures,
Toronto Notes, Therapeutic Choices and others. Edited and organized for the sake of all attendances of the
Canadian Osce Exams: NAC OSCE and MCCQE2. by: Dr. Merlyn D’Souza and Dr. Zeev Gross, Spring 2011. Page 132
Therapeutic Tips

 Search for the cause of iron deficiency, including very careful consideration of occult
gastrointestinal bleeding
 Menorrhagia must be convincing before it is accepted as the sole cause of iron deficiency. This
may be an opportunity for early recognition of a gastrointestinal malignancy—don’t miss it!
 A reticulocyte response should be evident within one week of beginning iron therapy, with
subsequent improvement in the Hgb of about 10 g/L every 7–10 days.
 If the Hgb fails to respond as anticipated, consider that there may be:
o ongoing blood loss
o use of other medications that impair iron absorption
o a different or concurrent cause of anemia and/or an impaired erythropoietic response
o compliance issues
 Gastrointestinal side effects are the most common reasons for non-compliance:
o use a graduated approach to dosing. Begin with a single tablet taken after a meal. On a
weekly basis, as tolerance permits, add another tablet until the patient is taking one

Please note that this is only a draft version based on several sources, including: Dr. Basel Mohasen’s lectures,
Toronto Notes, Therapeutic Choices and others. Edited and organized for the sake of all attendances of the
Canadian Osce Exams: NAC OSCE and MCCQE2. by: Dr. Merlyn D’Souza and Dr. Zeev Gross, Spring 2011. Page 133
dose with each meal. Thereafter, gradually shift the timing of the doses to the beginning
of meals
o small oral doses may be adequate in patients that are susceptible to gastrointestinal
upset.
In the elderly, daily doses of elemental iron as low as 15 to 50 mg are effective in the
treatment of iron deficiency anemia9
In pregnant women, 20 mg/day of elemental iron, started at 20 weeks' gestation, is
sufficient to prevent iron deficiency11
o iron contained in enteric-coated tablets is poorly absorbed. These products should be
avoided
 Some physicians replenish iron stores while others prefer to stop therapy when the Hgb
normalizes, so that further blood loss will not be masked by robust iron stores. As a
compromise:
o completely replenish iron stores when the cause of iron deficiency has been identified
and corrected
o do not replenish iron stores when investigation has failed to

Please note that this is only a draft version based on several sources, including: Dr. Basel Mohasen’s lectures,
Toronto Notes, Therapeutic Choices and others. Edited and organized for the sake of all attendances of the
Canadian Osce Exams: NAC OSCE and MCCQE2. by: Dr. Merlyn D’Souza and Dr. Zeev Gross, Spring 2011. Page 134
32 M Fever & Tiredness x 6 wks ---- 10 mins focused History

3 Scenarios:
H/O Splenectomy
IV drug user
Unprotected Intercourse

After introduction,Analyse Fever


Constitunat Sx
Then go to causes from Head to toe
End with Liver
Risk Fcts
Travel
Drugs
PAST MH
FH
Social H (Which is linked to Risk Fcts)

Can you tell me more about your fever from the moment it started?
O;
Sudden/Gradual
When it first started did you have any other illness?
Did you seek medical attention then?
What made you come in today?
C;
1. Is it on & Off/All the time/everyday
2. Does it inc/Dec or is it the same?
3. Any variation during the day, like more in morning? Any particular patern? 3 rd or 4th day
or alt days
4. Did you measure it?
5. How often do you measure it?
6. Which was the highest temp?
7. Does it increase at night?
8. Did you take any meds?/were they helpful?
9. Is it the first time or have you ever had it before?
10. Anything increases or decreases it?

Please note that this is only a draft version based on several sources, including: Dr. Basel Mohasen’s lectures,
Toronto Notes, Therapeutic Choices and others. Edited and organized for the sake of all attendances of the
Canadian Osce Exams: NAC OSCE and MCCQE2. by: Dr. Merlyn D’Souza and Dr. Zeev Gross, Spring 2011. Page 135
CONSTITIONAL Sx:
Fever/chills/N Sweats/wt loss/Lumps/bumps

TRANSITION:
I’ve to ask a couple of more qns to help me come to a diagnosis. If you’ve concerns at any time
please tell me & I will answer them
CNS:
HA/N/Vx/Photophobia/neck pain/Neck stiffness
Ear pain/Discharge from ear/runny nose/Facial pain/Sinusitis/Sore th/Difficulty swallowing
Dental pain/Tooth ache
CVS:
H racing/Chest pain/SOB
RS:
Cough/Phglem/wheezing/H-maemesis
Contact with TB/Have you ben screened for TB?
GI
Abd Pain/Diarrhoea/Malena
GU
Flank pain/burning urine/bld in urine/Inc freq in passing urine
MSK
Jt pain/Swelling/Skin rash/Ulcers in mouth/red eyes
Have you ever been Dsed as a condt called Autoimmune Disease? Or has anyone else in your
family been diagnosed?
LIVER DISEASE:
 Have you ben screened for liver disease?
 Have you been vaccinated against Hepatitis A & B?
 Sx of Ac Liver Disease: Yellow discoloration of skin & nails/Pale stools/Dark urine/Itchy
skin
 Sx of Ch L Disease: Inc abd girth/bruises /leg swelling/vomiting bld/memory changes

TRANSITION:
I’ve to ask you some questions to see if you were exposed to liver disease without being aware
of ,some of these qns may be personal, but it is imp that I ask them.All that you tell me is
confidential & the information will not be released without your permission, unless I’m
requested by law
TRAVEL & CAMPING H
Travel outside Canada
H/o eating raw fish,raw shell fish.Have you visited a new restrauant?
Please note that this is only a draft version based on several sources, including: Dr. Basel Mohasen’s lectures,
Toronto Notes, Therapeutic Choices and others. Edited and organized for the sake of all attendances of the
Canadian Osce Exams: NAC OSCE and MCCQE2. by: Dr. Merlyn D’Souza and Dr. Zeev Gross, Spring 2011. Page 136
H/O Surgery/Hospitalizations
Donated/recvd bld
Tattooing/Piercings
Smoke/Drink Alcohol/Recreational drugs?
Any injectable drugs?

SOCIAL H:
Whom do you live with?
How long have you been with your partner?
If for a specified time with a partner,ask if had any other sexual partners,though this qn is
personal,I’ve to ask it as it is imp:
When was the last time you’d sex with another partner/ Did you use a condom then?
If YES:
Ask Discharge/Lumps in groin/Genital ulcers
How is wife:Does she have: Fever/Sx/Discharge?
RISK FCTS:
How do you support your self financially?
Have you ben exposed to body fluids/TB
H/Ca
Any contacts with fever?

HIV
SEXUAL HISTORY:
Before marriage or before current relationship;
1. Did you have sexual partners?
2. At what age were you sexually active?
3. From that time till now, how many partners did you have?
4. Did you practise Safe Sex (Use of condoms?)
5. What is your sexual preference? M? F? Or Both?
6. What type of sexual activities do you prefer? Anal/vaginal/oral
7. Were you ever screened or diagnosed for STIs?
8. Did you have any sexual relationship besides your regular partner

Laboratory investigations:
o HIV antibody test (repeat to rule out lab error)

Please note that this is only a draft version based on several sources, including: Dr. Basel Mohasen’s lectures,
Toronto Notes, Therapeutic Choices and others. Edited and organized for the sake of all attendances of the
Canadian Osce Exams: NAC OSCE and MCCQE2. by: Dr. Merlyn D’Souza and Dr. Zeev Gross, Spring 2011. Page 137
o plasma HIV RNA level (viral load) with the CD4 lymphocyte count
is the best prognostic marker for progression to AIDS and survival.
o viral drug resistance mutations become harder to detect over time. Therefore
conduct a resistance test at entry into treatment program even if use of
antiretroviral treatment is not currently contemplated2
o CD4 lymphocyte count and percentage is useful in determining where a patient
lies in the continuum of HIV disease and the need for specific intervention (Table
1). Knowledge of the CD4 count can also help to narrow the differential diagnosis
in a symptomatic HIV-infected patient. In adults, a CD4 count of 430 to 1360
cells/μL (0.43 to 1.36 Giga/Litre or G/L) is considered normal in most
laboratories
o screen all patients for the presence of the HLA-B*5701 allele before starting or
restarting abacavir.2 , 3 A positive result indicates a very high risk for severe
allergy to abacavir and should be filed in the patient's chart
o perform a tropism assay to determine the chemokine receptor status (CCR5,
CXCR4 or dual-mixed tropic) if considering use of the CCR5 inhibitor
maraviroc. A plasma viral load of at least 1000 copies/mL is required to perform
this test
o CBC, differential and platelet count
o liver (AST, ALT, GGT, LDH, CPK, alkaline phosphatase, bilirubin, INR,
albumin) and renal (BUN, creatinine, electrolytes, urinalysis) profiles
o metabolic profiles (fasting glucose and lipids—total cholesterol, LDL, HDL,
triglycerides)
o hepatitis B, hepatitis C, syphilis, cytomegalovirus (CMV) and toxoplasmosis
serologies
o cultures and smears for sexually transmitted diseases as indicated
o tuberculosis skin tests, sputum cultures and smears for mycobacteria as indicated
o chest x-ray

Management of Patients with HIV Infection


CD4 Count Action
(cells/μL)
At all levels  General counselling (safer sex, nutrition, need for follow-up,
importance of adherence, etc.)
 History and physical examination every 3– 6 mo
 Plasma viral load and CD4 count at least every 3– 4 mo
 Herpes suppression if frequent recurrences (more than 4–6
episodes per year)
 Syphilis serology
 Pneumococcal vaccine; hepatitis A and B vaccines if appropriate;
update diphtheria, tetanus and inactivated polio vaccines as
needed; consider annual influenza vaccinations
 TB skin test and isoniazid prophylaxis if indicated (consider

Please note that this is only a draft version based on several sources, including: Dr. Basel Mohasen’s lectures,
Toronto Notes, Therapeutic Choices and others. Edited and organized for the sake of all attendances of the
Canadian Osce Exams: NAC OSCE and MCCQE2. by: Dr. Merlyn D’Souza and Dr. Zeev Gross, Spring 2011. Page 138
CD4 Count Action
(cells/μL)
repeating skin test yearly)

< 500  Plasma viral load and CD4 count every 3– 4 mo


 Clinical evaluations and laboratory investigations at least
bimonthly if symptomatic, diagnosed with AIDS, or on
antiretroviral therapy

< 200  Start prophylaxis for Pneumocystis jirovecii pneumonia (PCP)

< 100  Start toxoplasmosis prophylaxis if seropositive and not on


trimethoprim/sulfamethoxazole for PCP prophylaxis

< 75  Consider MAC prophylaxis

< 50  Screen by an ophthalmologist for early CMV retinitis (repeat at


3– 6 mo intervals) or consider CMV prophylaxis

Advise patients with HIV infection and immunosuppression that their risk of
infections can be reduced by following good hygienic practices.
1. Ensure thorough hand washing after contact with potentially
contaminated substances (diapers, soil, uncooked meat and produce) or
handling pets
2. Avoid raw or uncooked meat and eggs, e.g., Caesar salad
3. Drink from treated water sources only
4. Avoid handling sick animals or pet (especially cat) litter
5. Avoid cat scratches and do not allow cats to lick wounds
6. Avoid contact with reptiles

Back to Content

Please note that this is only a draft version based on several sources, including: Dr. Basel Mohasen’s lectures,
Toronto Notes, Therapeutic Choices and others. Edited and organized for the sake of all attendances of the
Canadian Osce Exams: NAC OSCE and MCCQE2. by: Dr. Merlyn D’Souza and Dr. Zeev Gross, Spring 2011. Page 139
45M with tiredness x 6 weeks
As I understand you’re having Tiredness since 6 weeks, can you tell me more about it since it all
started?
Pt says he is concerned. STOP & ask about his concern.
Pt says, he is also never been so tired before. Clarify: What do you mean about Tiredness?
 Sometimes I do not feel refreshed after sleep.
 Do you feel lack of energy? Like you cannot move your arm above your head.

The Statement: NOT REFRESHED ANYMORE points to an organic cause


OCD + Relation to sleep +/- Mood
If Mood Sx + ------ MOAPS
If Organic cause ---- Red Flags
OCD
At onset you can ask if there were any flu-like Sx initially (Thyroiditis)
 Do you sleep more
 When do you go to bed?
 Do you wake up in the middle of the night
 Which time of day/Night do you feel most?
 Ask Nature of work;
If shift
 With whom do you sleep?
 Does your partner C/o you snoring or jerky movts of limbs? (Restless Leg)
 Do you feel better in morning or evening(if tired in morning-->Depression If evening-------
-->Organic cause)
 Depression 1st Low mood then tired
 In organic 1st Tired the Low mood

11. Ask Constitutional Sx first:


Fever/Wt loss/Night sweats/Chills/Lumps & Bumps
Then quick review of Sx:
12. Cardiac: Chest Pain/SOB/Palpitations
13. Pulmonary: Cough/Wheezing/Phglem
14. GI: N/V Abnormal bowel movts/Diarrhoea/Malena/
15. Liver: Dark urine/Yellow sclera/Abd pain/Loss of aooetite/Pale stool/Itchy skin
16. GU: Change in color of urine/Amt of urine/Cloudy/Frothy urine/Dysuria/Facial swelling
17. Anemia: Bleeding gums/Easy bruising/Malena/Haematuria/Female: Meorrhagia & LMP

Please note that this is only a draft version based on several sources, including: Dr. Basel Mohasen’s lectures,
Toronto Notes, Therapeutic Choices and others. Edited and organized for the sake of all attendances of the
Canadian Osce Exams: NAC OSCE and MCCQE2. by: Dr. Merlyn D’Souza and Dr. Zeev Gross, Spring 2011. Page 140
18. Autoimmune Disease: Joint Pains/Skin rashes/Oral Ulcers
19. Endocrine: Thyroid: Feel hot/Cold Skin Moist/Dry/Tremors/Wt loss
If Pt has thyroid Sx,ak if on Thyroxine
When Dsed
If thyroxine levels are monitored?
20. DM: Risk fcts: FH & Lifestyle
Once Pt has DM in history, GO over ALL Sx & Sy:

RISK FCTS:
7. Diet
8. Exercise
9. FH
10. Smoking
11. Alcohol
12. Recreational drugs

PAST MEDICAL HISTORY:


Any medications/OTC/Herbal/LMP
Surgery/Hospitalization

FAMILY HISTORY:
DM/HTN/Stroke/MI

SOCIAL H:
Habits
With whom do you live?
How do you support yourself financially?

Hypothyroidism (underactive thyroid) is a condition in which your thyroid gland doesn't produce
enough of certain important hormones.

Hypothyroidism upsets the normal balance of chemical reactions in your body. It seldom causes
symptoms in the early stages, but, over time, untreated hypothyroidism can cause a number of health
problems, such as obesity, joint pain, infertility and heart disease.

The good news is that accurate thyroid function tests are available to diagnose hypothyroidism, and
treatment of hypothyroidism with synthetic thyroid hormone is usually simple, safe and effective once
the proper dosage is established.

Back to Content

Please note that this is only a draft version based on several sources, including: Dr. Basel Mohasen’s lectures,
Toronto Notes, Therapeutic Choices and others. Edited and organized for the sake of all attendances of the
Canadian Osce Exams: NAC OSCE and MCCQE2. by: Dr. Merlyn D’Souza and Dr. Zeev Gross, Spring 2011. Page 141
NEEDLE STICK INJURY
Michael Jackson, Nurse in hospital, Needle stick 20min ago; History and counsel, 10min
Variations: Janitor who was pricked in junk yard (here touch TT prophylaxis)
0.3% - HIV; 3% HCV; 30% HBV

“The treatment will be the same no matter what is the situation of the other patient...”
Did anybody talk to him? Did he accept to get his HIV status? “By law we are not allowed to take his
blood without his consent”
QUESTIONS RELATED TO EVENT:
1. Size of needle
2. Blunt/hollow
3. Any blood on it
4. How deep was the injury?
5. What was gauge of needle?
6. Where was the location of the prick?
7. Any bleeding after that?
8. Whether he was wearing gloves?
9. What measures did he take? (Wash hands?)
10. Is it the first time?
If Pt insists on doing HIV testing of the contaminated pt:
I know it is of great concern about the pt’s HIV status, however from the ethical point of view we cannot
do the HIV test without the pt’s consent. I can go after our interview & personally request him
If Still he insists or ask for CD4 count:
How do you think this will help us? It is a reasonable way of thinking.
There are different conditions reflecting CD4 count & ethically not the right step We do it to obtain Pt’s
information & not for the best interest of the pt
However whatever the CD4 count it makes no sense in our management.
We’ve to follow protocol:
RISKS
Give him the risks of being infected with N Stick injury:
HIV------->0.3%
Hepa C----->3%
Hepa B-------> 30%
.” In order to know what is the best line for you I need to ask you more questions.
Do you know what the chances for getting infected are? (Out of 1000 people – only 3 will be affected).
ASSESMENT:
Being a health care provider –
1. Have you been vaccinated before for Hepa A & B
1.1 How many doses?
1.2 When was the last dose?
2. Liver Disease:
Have you ever been yellowish? Itchiness? Dark urine? Pale stool? Btuises in body?
C Sx:
Repeated infections?
Chronic diarrhea?

Have you been screen for HIV or HCV?

Please note that this is only a draft version based on several sources, including: Dr. Basel Mohasen’s lectures,
Toronto Notes, Therapeutic Choices and others. Edited and organized for the sake of all attendances of the
Canadian Osce Exams: NAC OSCE and MCCQE2. by: Dr. Merlyn D’Souza and Dr. Zeev Gross, Spring 2011. Page 142
I am going to ask you some questions if you were exposed before for any of the viruses mentioned
above
1. Any travel outside Canada?
2. Any surgery
3. Any blood transfusions/
4. Tattoos/Piercings
SH
With whom do you do live.
For how long have you been together

COUNSELLING:

Whenever we face such a situation,we’re faced with three possible infections that could be transmitted:
HIV------->0.3%
Hepa C----->3%
Hepa B-------> 30%

Good news – HBV high risk but good plan; we are going to measure the titer of antibody in your blood.
If Okay,you need not worry,if low you may need an Immunoglobulin or revaccination

What do you know about HIV.


If infected,some bcome carriers,not all develoe into AIDS,we will screen you today:
For screening you we need to sample today to have base line. Most of the patients don’t react until 6wk,
few until 6m – we have to take it in these times.
If at the end of 6 mo tests come back negative, you’re cleared
If not you’re infected
Other options: we will also refer you to occupation clinic – Who will start you on prophylaxis treatment.
It consists of three medications usually.
They will explain to you which medication and describe the SE. It will decrease the chances by ...%
HCV – This is of concern as We don’t have prophylaxis yet
there is more than 50% to be carrier, more than 50% of them become chronic, 50% of them will get
cancer.
However certain medications might help like Interferon & Anti retrivirals

“How do you feel about it?” Sometimes patients are overwhelmed and might harm yourself or others”
From now till the results of your blood tests:
Practise Safe sex
Do not donate blood
Joint a support group.”
I wii file an incident report.”
Back to Content

Please note that this is only a draft version based on several sources, including: Dr. Basel Mohasen’s lectures,
Toronto Notes, Therapeutic Choices and others. Edited and organized for the sake of all attendances of the
Canadian Osce Exams: NAC OSCE and MCCQE2. by: Dr. Merlyn D’Souza and Dr. Zeev Gross, Spring 2011. Page 143
35y.o, male, counselling about HIV test
Wants to do HIV test as his partner has tested =ve

As I understand you’e here as you want a blood test.Can you tell me which blood test
you specifically need?
Can you tell me what made you come in today?
**PT: I feel I’m at risk screen me for disease
We cannot order all bl works, we’ve to look for a specific disease e.g: for TB we do a
CXR/DM BSL/HIV Bl tests
** My Partner has tested for HIV +
EMPATHY:
I’m sorry, when was that? How is she doing now?
How long have you two been together?
How has it affected you?
How do you feel?
This can be a difficult for you, & you’ve done the right thing,& definitely we can arrange
for a blood test
“20 years ago we had no options, now even if you are positive we will have treatments and
prophylaxis.”
In order to get the diagnosis we need to do more questions.”
1. “Have you ever been screened for HIV”or HCV
If Yes: When & where?
2. Any Sx OF HIV
CSx, Mouth, Ulcers,difficulty swallowing
RS;Cough
,Diarrhea, Discharge,Ulcers,Skin rash/yellowish, Dark urine/Pale stool
I am going to ask you some questions if you were EXPOSED BEFORE for any of the viruses
mentioned above
1. Any travel outside Canada?
2. Any surgery
3. Any blood transfusions/
Tattoos/Piercings?
SEXUAL HISTORY
Relationships now and before / Sexual predilection / Sexual practice
When were you sexually active?
How many partners have you had?
Did you practise safe sex?
PMH
Any long term disease/hospitalization/allergies/medications

COUNSELLING
“What do you know about HIV?”
“Nowadays we have better control over the disease. Once they start get the infection they called
AIDS patients.”

Please note that this is only a draft version based on several sources, including: Dr. Basel Mohasen’s lectures,
Toronto Notes, Therapeutic Choices and others. Edited and organized for the sake of all attendances of the
Canadian Osce Exams: NAC OSCE and MCCQE2. by: Dr. Merlyn D’Souza and Dr. Zeev Gross, Spring 2011. Page 144
“HIV is a virus which affects our immunologic system. It is different if you are the carrier as
oppose to have symptoms when you are an AIDS patient.
HIV is a virus. HIV attacks the immune system itself - the very thing that would normally get rid
of a virus
It takes around ten years on average for someone with HIV to develop AIDS
” Is that reasonable?
Am I clear? Do you have any questions?
In order to know whether you are infected or no we need to do a blood work. We need your
consent for that. They will give you the results within two weeks. If the results are positive – they
will call you back. If it negative – they will not call you.
Options to send the sample:
Nominal – with your name
Non-nominal – put a bar-code (the public health and the doctor will know the identity)
Anonymous – put a barcode on the sample and only you know the results
(needs a lot of counselling.
“How do you feel about it?” Sometimes patients are overwhelmed and might harm yourself or
others”
From now till the results of your blood tests:
Practise Safe sex
Do not donate blood
Joint a support group.”
I like you to know that in 2011 there are a lot of options open, with Rx it is controllable & people
can live with it for a long time.
If you test positive you’ve a have legal obligation to inform your partner.

Back to Content

Please note that this is only a draft version based on several sources, including: Dr. Basel Mohasen’s lectures,
Toronto Notes, Therapeutic Choices and others. Edited and organized for the sake of all attendances of the
Canadian Osce Exams: NAC OSCE and MCCQE2. by: Dr. Merlyn D’Souza and Dr. Zeev Gross, Spring 2011. Page 145
Freddy Mercury, 37y.o, Male, HIV results came back and are
positive
Divide time: 2min telling the results, 3min assessing symptoms, 2min explain about the virus,
3min the plan
HIV treatment in Ontario is covered.
“Nice to meet you. Or Hello”
Because this is the first time I see you I am going to ask you some questions, to get a better
understanding of your results:
“Why/Who/Is the first time/When you did it?”
** Somebody I knew died from it last week
“Who is the person that you got it from him?” (nature of the relationship)
“People don’t get it from normal daily contact. Was there any direct contact?”
SPIKE
Setting
Perception – what do you know about HIV?
―What did you think was going on with you when you felt the lump?‖
―What have you been told about all this so far?‖
“Are you worried that this might be something serious?”
Invitation – how much details you want me to discuss? DO you want someone else to be
present?
―Are you the kind of person who prefers to know all the details about what is going on?‖
―How much information would you like me to give you about your diagnosis and treatment?‖
“Would you like me to give you details of what is going on or would you prefer that I just tell you
about treatments I am proposing?”
Knowledge
―Unfortunately, I’ve got some bad news to tell you, Mr. Andrews.‖
“Mrs. Smith, I’m so sorry to have to tell you….”
Empathy – “What are your expectations from this visit?”
If he is not very anxious you might take some time until giving him the results. Otherwise you
give them immediately.

“I wish I’d better news for you. Unfortunately the results came back and I am very sorry to tell
you that the result is positive.”
Silence.
Wait x 10 sec if he cries
“How do you feel right now?”
Do you need more time/
Do you need water?
Do you want me to proceed?
**If mistake?
“Whenever we do a screening test we confirm it if it positive. So the result is very accurate.”
The initial test is ELISA & then we do a confirmatory test called Western Blot
Part of F/u atre other tests like the CD4 count & Viral load

SOCIAL Hx:
We will ask you several questions concerning your sexual partners.
Drug use
Asx:
Please note that this is only a draft version based on several sources, including: Dr. Basel Mohasen’s lectures,
Toronto Notes, Therapeutic Choices and others. Edited and organized for the sake of all attendances of the
Canadian Osce Exams: NAC OSCE and MCCQE2. by: Dr. Merlyn D’Souza and Dr. Zeev Gross, Spring 2011. Page 146
CSx:
AIDS Sx:
PMH: any long term disease? HTN/DM?
Any hospitalizations/Surgeries?

If does not want to inform his wife


From experience it is not necessarily that your partner will leave you. From that reason we need
to inform your wife. Part of the public health job is to tell her. The same measures we are taking
for you we should do for her. It is better that she will know it from you rather than from the
Public Health – otherwise she’ll loose the trust in you.

Do you have any symptoms relating to HIV?


PMHx and drugs.

COUNSELLING
If asked what he knows about HIV – don’g repeat. Otherwise you explain here.
From HIV+ to AIDS.
Significance of CD4 and Viral load:
Viral load,amt of HIV virus existing in your body, lower the viral load,& higher the CD4 count,
better condition
We should think about HIV these days like a chronic disease as DM or HTn,it can be controlled
but not cured.
Part of your treatment is to refer you to HIV clinic – they will treat you based on these
parameters. The newer medications are effective and control your disease – however they have
side effects.
How’s your mood, how you feel about that, there are a lot of support groups. I’ll give you “hot
lines” number.
From now on you have to practice safe sex & do not donate blood

In case of the resident who was asked to backup his supervisor orthopaed
1. I am competent – to emphasize
2. Short term – we don’t have time so we need to see her urgently
3. Long term – solve the situations that it wouldn’t occur again

Dr. Smith, Chief of staff of the hospital, ask another doctor to talk with the doctors me because they
smell alcohol from the doctor.
You smelled like alcohol.
People have different ways to relieve their stress – how do you relieve your stress?

Team worker – interpersonal relation


It is better to provide prescription than getting pregnancy
“I may share your point of view – that doesn’t say it gives me the right to impose my beliefs.”
My concern is if we face the same situation in the future...what will we do?

Please note that this is only a draft version based on several sources, including: Dr. Basel Mohasen’s lectures,
Toronto Notes, Therapeutic Choices and others. Edited and organized for the sake of all attendances of the
Canadian Osce Exams: NAC OSCE and MCCQE2. by: Dr. Merlyn D’Souza and Dr. Zeev Gross, Spring 2011. Page 147
Why won’t we contact the College?
Don’t give any names. In case of report – it should be reported to the college.
“We are here to help you. Moving to new place can be stressful. The reason of this meeting we have
received two complaints – they claimed they have smelled alcohol from you. Is that happened?
If you don’t mind me asking few more questions: do you drink more, or you did it on lunch time? Before
working here – where else did you work? Did you ever have a complain about drinking?
I would recommend that you will contact the program for doctors who drink. They will suspend your
license. After stop drinking you will resume your work. At the end you will have your career back.

Back to Content

Please note that this is only a draft version based on several sources, including: Dr. Basel Mohasen’s lectures,
Toronto Notes, Therapeutic Choices and others. Edited and organized for the sake of all attendances of the
Canadian Osce Exams: NAC OSCE and MCCQE2. by: Dr. Merlyn D’Souza and Dr. Zeev Gross, Spring 2011. Page 148
24M: HA 6 wks ER 10 mins H & Counsel
OCD PQRSTUV

INC ICP:
 SOL: Brain Tumors
 Mets
 Infection
 Toxoplasmosis

HA + Inc ICP
Constitutional Symptoms: fever: Always ask what came first: fever or headache
* If primary tumor;
FH of Malignancy
H/o Cancer,Leukemia,Melanoma
H/o HIV:
 Ask if HIV status known,
 Have you ever been checked?
 I’m concerned because of the risk factors involved

IF HIV status known ask:


 When was the last time you saw your Dr
 What was the last CD4 count
 Are U on any Anti AIDS meds?
 TB Skin test results
 Syphillis tst results
 Date & results of PAP’s smear

* Ocurence f opportunistic infections,malignancies,


* STIs: Hepatitis B & C,Syphillis,Gonorrhoea,Chlamydia,Molluscum contagious
* Other bacterial infections,fungal infections,Malignancies
* Travel History,illness while away & use of preventative vaccines
* Medication History:
* Antiretroviral History (including response,CD4,Viral load)
adherence,toxicity,any resistance testing & results
If HIV or AIDS & not on meds:
Asses condt by asking:
Constitutional symptoms:
 Fever
 Repeated chest infections
 Cough thrush

Please note that this is only a draft version based on several sources, including: Dr. Basel Mohasen’s lectures,
Toronto Notes, Therapeutic Choices and others. Edited and organized for the sake of all attendances of the
Canadian Osce Exams: NAC OSCE and MCCQE2. by: Dr. Merlyn D’Souza and Dr. Zeev Gross, Spring 2011. Page 149
 Odonophagia
 TB
 In Female: Cx al Ca

OCD
ONSET:
COURSE:
When Pt says HA now Ct ous
Ask:
 When did it become constant?
 In beginning how often did you have it?
 What time of the day?
 Is it more in the morning? Or is it worse in the evening?
 Does it wake you up?
DURATION:
PQRST UV
S: How was it in the beginning as compared to ‘Now’

U Qns for empathy**

 How has it affected U in your daily life?


 How r U coping with it?
 How do U feel abt it?
 What r your expectations from today’s visit?

Aggravating fcts:
Coughing/leaning forwards/lying down
Alleviating fcts:

ASOC Sx:
CONstitonal Sx:
Fever/nightsweats/chills (if before headache indicates patho) Ask when Wt loss started
LOCal Sx:
NEURO Screening:
1. Vision changes:
What type of problem?
2. Hearing abnormalities
3. Difficulty swallowing
4. Weakness/Numbness
5. Difficulty finding words
6. Difficulty in balance or repeated falls
Please note that this is only a draft version based on several sources, including: Dr. Basel Mohasen’s lectures,
Toronto Notes, Therapeutic Choices and others. Edited and organized for the sake of all attendances of the
Canadian Osce Exams: NAC OSCE and MCCQE2. by: Dr. Merlyn D’Souza and Dr. Zeev Gross, Spring 2011. Page 150
7. Changes n bowel/Urine Loss of bladder control
8. LOC
9. H/o Seizure

MOOD Changes
MEMORY problem
Changes in CONCENTRATION
Has anyone told you that you’ve been acting strangely?
H/O INJURY:
To head
Did you have a fall & hurt your head?

EXTRACRANIAL:

EYE:
Did you notice any redness or need eyeglasses?
Sinusitis:
Facial pain/flu
Throath pain
Dental pain
PMH:
RISK FCTS:
Do you smoke
Take EtOh
Take recreational drugs? Route
Tattoo

COUNSELLING:
Do you have any qns for me?
Based on what you’ve told me,the symptoms are concerning & I need to admit you
today, as you’d stopped your meds, you may be exposed to an infection
Your HA may be caused by this infection.
I will refer you to an Infectious Disease Specialist
Also do some blood investigations & Imaging of your head.

DIAGNOSIS: CNS Toxoplasmosis in HIV +ve Male

Back to Content

Please note that this is only a draft version based on several sources, including: Dr. Basel Mohasen’s lectures,
Toronto Notes, Therapeutic Choices and others. Edited and organized for the sake of all attendances of the
Canadian Osce Exams: NAC OSCE and MCCQE2. by: Dr. Merlyn D’Souza and Dr. Zeev Gross, Spring 2011. Page 151
22F sudden loss of vision x 2 wks seen by 2 drs one opthal
(Somatization GAO -207)
As I understand you’re here because you’d loss of vision in (R) eye x 10 days, I understand
you’ve been seen by 2 Drs

 What did the drs tell you?


 What diagnosis did they give you?
 Any investigations were done?
 How’re you doing today/Tell me how it all began...... OCD

ONSET: Sudden/gradual
What were you doing at that time? Anything particular happened at that time?
Is the loss all the time or off & On?
Ask a little about local Sx: Pain Photophobia/Injury
U
V
SOCIAL H:
 Speak about confidentially here
 Who lives with you?
 What is your relationship?
 Any difficulties in your relationship? Emphasize confidentiality
 Was there a stressful situation before you lost your vision?
 Let us go back to that day........
 Make sure no suicidal/Homicidal ideation
COUNSELLING:
This seems like a stressful situation for you & sometimes when we face such situations our
brain finds it difficult to deal with it & this stress can be manifested by loss of function.
In this case you saw something that made you lose your sight.It is not uncommon & called
CONVERSION DISORDER
I will refer you to a therapist to help you understand the stressor in your life & learn how to
handle it.

Please note that this is only a draft version based on several sources, including: Dr. Basel Mohasen’s lectures,
Toronto Notes, Therapeutic Choices and others. Edited and organized for the sake of all attendances of the
Canadian Osce Exams: NAC OSCE and MCCQE2. by: Dr. Merlyn D’Souza and Dr. Zeev Gross, Spring 2011. Page 152
LOSS OF VISION: 32 M/F
Pt presented with vision loss x 2 weeks

OCD
Onset after car accident
Here enquire about the nature of accident Ask whether she or other driver was hurt & how is
her driving now, does she still drive?
Gradual
C:
Is it increasing now?/Same/Varies at certain times of day
U &V
CONSTITUNAT SX:
Assoc Sx:
*HA: Here +ve,OCD: PQRST:
When does it occur? Morning or evening
Vomiting +/Quality---- projectile
Weakness/Numbness/Difficulty finding words
ENDOCRINE:
Thyroid Disease: feel hot/cold/Warm & moist skin/tremors
Pituitary:Sometimes Pts in similar situations can notice breast engorgement & secretions from
breast (For Males) & changes in sexual life ,desire & habits
In Females ask directly about amenorrhoea galactorrhea
SX of Acromegaly: Inc size of shoes/Tight ring
FH of Kidney stone/Pancreatic Ca/Diarrhea/Foul smelly stools

Past H
Fh
Social H

Please note that this is only a draft version based on several sources, including: Dr. Basel Mohasen’s lectures,
Toronto Notes, Therapeutic Choices and others. Edited and organized for the sake of all attendances of the
Canadian Osce Exams: NAC OSCE and MCCQE2. by: Dr. Merlyn D’Souza and Dr. Zeev Gross, Spring 2011. Page 153
VISION LOSS
1. 40 M difficulty in vision 2 wks 10 mins H & C
2. 22F sudden loss of vision x 2 wks seen by 2 drs one opthal (Somatization GAO -207)
3. 40M diificulty in vision x 4 wks seen by optometrist counsel All D/vision:
Screening Qn: What do you mean?
Pt will answer: I’m not seeing well
Now ask close ended qns:
1. One/(B) eyes
2. Blurry V
3. Double vision:
a. Relieved by covering one eye?
b. Horizontal/Vertical/Oblique
c. Worse in one direction of gaze?
d. Fluctating or constant? (Gets worse at end of day)
4. Loss of vision
5. Curtain falling
6. Dark spots/flashes
7. Difficulty seeing on sides/when you drive do you have difficulty changing lanes?
8. Do you bump into objects when walking?
9. Do you see halo around objects?

OCD:
O;Suden/gradual
Painless/Painful
PAIN: Assoc with:
 Blinking
 Eye movts
 HA/N/V
 Brow/Temporal pain
 Photophobia
 Gritty sensation
How has it affected your life
Has it happened before?
Asso Sx:
Fatigue wt loss,joint SxNight sweats,ever
Polyuria/poly dipsia

Please note that this is only a draft version based on several sources, including: Dr. Basel Mohasen’s lectures,
Toronto Notes, Therapeutic Choices and others. Edited and organized for the sake of all attendances of the
Canadian Osce Exams: NAC OSCE and MCCQE2. by: Dr. Merlyn D’Souza and Dr. Zeev Gross, Spring 2011. Page 154
Tingling /Numbness
Past Occular H: Use of eyeglasses/Contacts: Duration
H/O Occular surgery,Laser Rx,Infection,trauma,FB
Presence of Ch eye disease: Glaucoma/DM
PMH: Htn/DMMS/HIV
Asthma
Allergies
Meds:Occular meds Current+Past

Back to Content

Please note that this is only a draft version based on several sources, including: Dr. Basel Mohasen’s lectures,
Toronto Notes, Therapeutic Choices and others. Edited and organized for the sake of all attendances of the
Canadian Osce Exams: NAC OSCE and MCCQE2. by: Dr. Merlyn D’Souza and Dr. Zeev Gross, Spring 2011. Page 155
Somatization Disorder
22 YOF Somatization disorder
4-2-1-1
If seen by a surgeon – suspect somatisation.
What did the surgeon tell you.
OCD
PQRST
During the day or night.
CSx
Jaundice, white stool and dark urine
Foul smell, bulking, droplets
Change in bowel movement
First time to have this pain or had it before (V)
MRI – why do you think it is important?
Somatic pain disorder / Somatization
Pains: headache, joints, back, pain with intercourse
Sexual:
You are here because you are concern. The pain you have, and multiple doctors – all these are
consistent with somatisation.
Once every 2-3weeks

See Zu09: 207-8

Please note that this is only a draft version based on several sources, including: Dr. Basel Mohasen’s lectures,
Toronto Notes, Therapeutic Choices and others. Edited and organized for the sake of all attendances of the
Canadian Osce Exams: NAC OSCE and MCCQE2. by: Dr. Merlyn D’Souza and Dr. Zeev Gross, Spring 2011. Page 156
Introduction to OBGYN
OCD
COCA +/- Blood
ΑA
ASx:– which organism
MSGO
PMHx

OBGYN MAP

CC
Menses
Gynecology history
Obstetriscs h
Sexual h

VAGINAL DISCHARGE.
OCD
COCA
ΑA
AsSx – which organism
MSGO
PMHx

AMENORRHEA
OCD
MAGOS

VAGINAL BLEEDING
OCD
COCA +/- Dc
ΑA
M
ASx
G
O
SHx
PHx
FHx

INFERTILITY
I+O
Tr
Partner
C
M
G
S
Intro
How many months trying to conceive?

Please note that this is only a draft version based on several sources, including: Dr. Basel Mohasen’s lectures,
Toronto Notes, Therapeutic Choices and others. Edited and organized for the sake of all attendances of the
Canadian Osce Exams: NAC OSCE and MCCQE2. by: Dr. Merlyn D’Souza and Dr. Zeev Gross, Spring 2011. Page 157
If less than 35 y – wait for 1yr
If around 40 – wait for 6m
If greater than 40 – immediately
O – if children from previous relationship
Transition: ―In order for a couple to achieve pregnancy both partner involves should be relatively healthy
and capable of having children. For that reason I need to ask questions about your and your partner’s
health. Some of these questions can be personal, but important to ask, but I can assure that everything
king is strictly confidential. The male factor is responsible for 40%.‖
COITAL H.
How often do you have intercourse?
Do you monitor tmp?
MGOS
All causes of secondary amenorrhea:
Endometriosis
Past medical history
Family History
Social history

MENSES
Use the word ―period‖
1. When was your last period?
2. Are your periods regular / not
3. How often?
4. How many days?
5. How many pads do you use/change?
6. Are the pads full?
7. How long does it last?
8. Are they heavy?
9. Do you see clots?
10. Between periods do you have spotting?
11. From your last menstrual period was your period different from the current one?
12. At what age did you start your periods?
13. Were they regular/irregular?
14. When did it become regular?
15. Are your periods painful / painless?
16. If irregular, from beginning?
17. Discharge – ask if pregnant and when LMP

GYENECOLOGY
1. Any history of Gyn. Disease – polyps or cysts
2. History of pelvic surgery (if yes – when?)
3. Have you used any birth control?
When/type/any complications?
4. If less than 50 – have you ever done PAP (if yes – when and what were the results?)
5. If 50 and older – in addition ask for mammogram
6. If more than 65 – ask for bone density

OBSTETRICS
Have you ever been pregnant
How many times, how about abortion and termination/ Abortion – termination medically;

Please note that this is only a draft version based on several sources, including: Dr. Basel Mohasen’s lectures,
Toronto Notes, Therapeutic Choices and others. Edited and organized for the sake of all attendances of the
Canadian Osce Exams: NAC OSCE and MCCQE2. by: Dr. Merlyn D’Souza and Dr. Zeev Gross, Spring 2011. Page 158
How many live children, what was the route, any complication?
Were there any complications with the children?
During pregnancy: any HTM/GDM/Vaginal bleeding
How do you feel about (miscarriage?)
If NULLIPAROUS:FH of HTN/DM?cong anomalies/repeatd C S/Twins

SEXUAL Hx:
1. With whom do you live?
2. How long have you been together? (a relationship below 6 month is not stable)
3. If you live alone – are you in relationship?
4. Are you sexually active?
5. Do you practice safe sex – using condoms?
6. When did you start to be sexually active?
7. How many partners you had last years?
8. What is your sexual preference?
9. What type of sexual activity do you practice?
10. Have you ever been diagnosed with PID
11. Any Vaginal discharge?
12. How about your partner? Does he have any symptoms
have you ever been screned for HIV?

PREGNANT IN T3:
Reg F/U:
No-----> Social Hx
Yes:------->When
If recent ask 2-3 qns about PET:
1. What was your BP
2. Was there swelling?
MUM’s STABILITY:
1. Abd pains
2. Abd cramps
3. Vaginal bleeding or discharge
4. Any gush of water
BABY:
1. Is baby kicking like before?
2. 10 movts/12 hrs

Back to Content

Please note that this is only a draft version based on several sources, including: Dr. Basel Mohasen’s lectures,
Toronto Notes, Therapeutic Choices and others. Edited and organized for the sake of all attendances of the
Canadian Osce Exams: NAC OSCE and MCCQE2. by: Dr. Merlyn D’Souza and Dr. Zeev Gross, Spring 2011. Page 159
19 year old with Vaginal discharge for the last 10 days
History 5 minutes
VAGINAL DISCHARGE.
OCD
COCA
ΑA
AsSx – which organism
MSGO
PMHx: Recent use of Ab + DM

Intro: How can I help you today?

O:
Sudden(Allergey)/Gradual
How did it start?
C:
1. Is it all the time or on and off?
2. Is it increasing, decreasing or the same?
COCA + BL
1. Can you estimate the amount for me?Do you use pads? How many?
2. How about the colour? Is it greenish, whitish or yellowish?
3. How about the consistency? Is it thick or watery?
4. Is the smell offensive?
5. Is this your first time?
A & A:Does it increase after IC?
I would like to ask you a few personal questions, hope you don’t mind?

Sexual Hx: as part of A & A


1. Are you sexually active?
2. Any relation to your periods or with intercourse?
3. Do you have any pain with intercourse?
4. Any itching, redness, blisters or ulcers?
U:
V:
Associated symptoms:
LOCAL Sx:
Pain with IC
Itching/redness/blisters/warts/ulcers
GU: Any burning in urination in urination?
GI: Any change in bowel movements?
PID : Any abdominal pain? If yes, then OCD, PQRST.
:MSK: Any ulcers in mouth, difficulty swallowing, joint pain, skin rash or red eyes?

Constitutional Sx:

Menstrual Hx:

1. When was your last menstrual period?

Please note that this is only a draft version based on several sources, including: Dr. Basel Mohasen’s lectures,
Toronto Notes, Therapeutic Choices and others. Edited and organized for the sake of all attendances of the
Canadian Osce Exams: NAC OSCE and MCCQE2. by: Dr. Merlyn D’Souza and Dr. Zeev Gross, Spring 2011. Page 160
2. Are your periods regular?
3. Was the last period the same as before?
Gynecological Hx:
1. Do you use any form of contraception?
2. Have you had a pap smear? When and what was the result?
Obsteterics Hx:
1. Have you ever been pregnant?
2. Have you ever had an abortion or miscarriage?

Sexual Hx:
As I understand you’re in a relationship…
How long?
Do you practise safe sex?
Does the partner have any urinary symptoms,discharge?
Before this?
What age were you sexually active?
How many partners in last one year?
st
PMH:Since it is 1 time I’m seeing you,I need to ask some qns about PMH:

SHx:

Please note that this is only a draft version based on several sources, including: Dr. Basel Mohasen’s lectures,
Toronto Notes, Therapeutic Choices and others. Edited and organized for the sake of all attendances of the
Canadian Osce Exams: NAC OSCE and MCCQE2. by: Dr. Merlyn D’Souza and Dr. Zeev Gross, Spring 2011. Page 161
RX:

Infectious Diseases: Sexually Transmitted Infections

Please note that this is only a draft version based on several sources, including: Dr. Basel Mohasen’s lectures,
Toronto Notes, Therapeutic Choices and others. Edited and organized for the sake of all attendances of the
Canadian Osce Exams: NAC OSCE and MCCQE2. by: Dr. Merlyn D’Souza and Dr. Zeev Gross, Spring 2011. Page 162
Table 1: Differential Diagnosis of Vaginal Discharge1 , 2

Candidiasis Trichomoniasis Bacterial Vaginosis

Signs/symptoms:

Pruritus + + –

Odour – + + (fishy)

Discharge white, clumpy & off-white or yellow, grey or milky, thin, copious
curdy frothy

Inflammation + + –

Simple tests:

pH < 4.5 > 4.5 > 4.5

"Whiff" testa – +/- +

Microscopic findings:

Specific budding yeast, motile clue cells,b predominant Gram-


psuedohyphae trichomonads negative curved bacilli and
coccobacilli

PMNs ++ +++ –

Lactobacilli + – –
a.
Malodour often intensified after addition of 10% potassium hydroxide (KOH).
b.
Clue cells are vaginal epithelial cells covered with numerous coccobacilli.

Back to Content

Please note that this is only a draft version based on several sources, including: Dr. Basel Mohasen’s lectures,
Toronto Notes, Therapeutic Choices and others. Edited and organized for the sake of all attendances of the
Canadian Osce Exams: NAC OSCE and MCCQE2. by: Dr. Merlyn D’Souza and Dr. Zeev Gross, Spring 2011. Page 163
36 weeks pregnant with vaginal bleeding for 2 hrs
History 5 min

Differentials:

1. When did the bleeding start?


2. How long has it been?
3. Were the pads fully soaked? Any clots?
4. What were you doing at the time?
5. Any H/o trauma?
6. Did you have any abdominal pain?
If yes:
 Did the pain start first or the bleeding?
 Are you having any pain now?
 OCD, PQRST
7. Any gush of water?
8. Are you under regular F/U?
9. When was the last F/U ? If missed, why?
10. Symptoms of pre-eclampsia:
a. Weight gain?
b. Headache?
c. High blood pressure?
d. Flashing lights or disturbance in vision?
e. Swelling?
11. Stability of Mother: .........................?
Have you had an U/S?
When was the last one?
# of babies?
Position of placenta?
Amount of fluid?
12. Stability of fetus:
Is your baby kicking like before?
Obsteterics Hx:
1. Have you ever been pregnant?
2. Have you ever had an abortion or miscarriage?
3. Any complications during previous pregnancy?

Please note that this is only a draft version based on several sources, including: Dr. Basel Mohasen’s lectures,
Toronto Notes, Therapeutic Choices and others. Edited and organized for the sake of all attendances of the
Canadian Osce Exams: NAC OSCE and MCCQE2. by: Dr. Merlyn D’Souza and Dr. Zeev Gross, Spring 2011. Page 164
39 year old with vaginal bleeding for 50 days

History 5 min

OCD + COCA +/- Discharge

AA

ASx:

Risk Fcts: GPOS

D/d:

PMH

Fh

SH

INTRO:

1. What made you come here today?


2. Did you seek medical attention before?
OCD:
O:
How did it start?
What were you doing at the time?
Did it start gradually or suddenly?
C:
Is it on and off or all the time?
Is it increasing, decreasing or the same?
COCA
Can you estimate the amount for me?
Foul smell?
A&A:
Is there any relation to periods or with intercourse?

Menstrual Hx:

1. When was your last menstrual period?


2. Are your periods regular?
3. Was the last period the same as before?
4. Can you differentiate this bleeding from periods?

Please note that this is only a draft version based on several sources, including: Dr. Basel Mohasen’s lectures,
Toronto Notes, Therapeutic Choices and others. Edited and organized for the sake of all attendances of the
Canadian Osce Exams: NAC OSCE and MCCQE2. by: Dr. Merlyn D’Souza and Dr. Zeev Gross, Spring 2011. Page 165
I’m going to ask a few qns to see how it has affected your life:
IMPACT:
Are you having any dizziness? Hrt racing? LOC?

Associated symptoms:
CSx:
Local & Mets
1. Local symptoms:
Any itchiness, redness, discharge, pain during intercourse?
Itching/rednes/blisters/warts
1. Any abdominal pain?

When was your last Pap’s smear?


What was result?
O

Sexual Hx:
1. With whom do you live?
2. Are you sexually active?
D/D;
Hypothyroidism
Bl thinners
Bleding disorders
PMH
FH of Ca

Back to Content

Please note that this is only a draft version based on several sources, including: Dr. Basel Mohasen’s lectures,
Toronto Notes, Therapeutic Choices and others. Edited and organized for the sake of all attendances of the
Canadian Osce Exams: NAC OSCE and MCCQE2. by: Dr. Merlyn D’Souza and Dr. Zeev Gross, Spring 2011. Page 166
AMENORRHOEA 22 yr old Female:

PEP:
1. what is your Ds?
2. What is your inv?
3. What is your Rx?

Always R/o pregnancy


Intro: As I understand you’re here today because you did not have your periods since last 6
mo….. Can you tell me more about it?
CC------->
When was your LMP?
Any spotting in between?
Let us talk about your periods from the beginning……..
st
1. When did you have your 1 period?
2. Was it regular from the start? Or was it irregular?
3. When did it become irregular?
4. When your periods were regular,How often did they come?
5. How long did each cycle last?
6. When irregular,How often did they come? How many days did they last?
7. When periods were regular, were you using any contraception?
If Yes:
How long?
When did you stop?(if Inj Depo provera I yr post injection amenorrhoea OC can be upto 6 mo
Amenorrhoea)
With whom do you live?
Are you sexually active? Any chance that you may be pregnant?
Ask Sx of pregnancy:
 Breast tenderness
 N/Vx
 Increased visits to washroom
OB Hx: Any time you were pregnant?
Any abortions or miscarriages?
RISK FCTS:
I’m going to ask you questions to help reach what could be the cause
HYPOTHALAMIC:
1. Are you under stress?
2. Are you losing wt?
3. How do you perceive yourself when you look into the mirror?
4. Do you exercise excessively?
PITUITARY:
1. Any change in vision/Any difficulty in seeing on sides or changing lanes when driving?
2. Discharge from nipples & breast engorgement?
THYROID:
1. Do you feel hot when others around feel cold or do you feel cold when others around feel
cold?

Please note that this is only a draft version based on several sources, including: Dr. Basel Mohasen’s lectures,
Toronto Notes, Therapeutic Choices and others. Edited and organized for the sake of all attendances of the
Canadian Osce Exams: NAC OSCE and MCCQE2. by: Dr. Merlyn D’Souza and Dr. Zeev Gross, Spring 2011. Page 167
2. Do you have constipation/Diarrhoea?
OVARIAN:
PCO:
1. Any acne
2. Increased facial hair?
3. Are you concerned about your weight?
4. Are you trying to lose weight?
5. Is there h/o DM? (ask for Sx of DM)
6. Any FH of PCOs or infertility?
Premature Ov Failure:
1. Hx of Chemotherapy/Radiation to pelvis
2. Hot flushes
3. Night sweats
Ovarian tumors:
1. Increased muscle bulk
2. Change in voice
C Sx:
Gyn Hx:
Sexual Hx;
Any H/o STis
PMH:
st
Since it is 1 time I see you,do you have any H/o HTN,DM
Have you ever seen a psychiatrist before or used antipsychotic medications/

Back to Content

Please note that this is only a draft version based on several sources, including: Dr. Basel Mohasen’s lectures,
Toronto Notes, Therapeutic Choices and others. Edited and organized for the sake of all attendances of the
Canadian Osce Exams: NAC OSCE and MCCQE2. by: Dr. Merlyn D’Souza and Dr. Zeev Gross, Spring 2011. Page 168
31 year old woman with 36 weeks pregnancy:BP155/110
Urine Protein +++

Intro: As I understand…..,you’re here today for a F/u visit,& nurse measured your BP & did a urine
test.I’ve your results here & will discuss them with you, But I need to ask you some qns to gain a better
insight into your condition
1. Were you ever diagnosed with increased blood pressure prior to this pregnancy?
2. When was your last F/U visit?
3. What was your Bp the last time?
4. What about your blood tests?
5. Were you anemic?

Based on your BP & urine, these results are consistent with pregnancy induced HTN,& I need to ask
you qns,to see if you’ve Sx pertaining to that.
It could be a serious condition
1. Do you’ve H/O: HA ---------> OCD
2. How is your Vision ------------->Do you see flashes of light/Blurring
3. CNS --------> Weakness/numbness
4. Nx/V/Chest pain/SOB
5. ABD PAIN?
6. Bruises on body?
7. Yellow discoloration OF SKIN/ITCHINESS/Pale stool/Dark Urine
8. Swelling feet/Tight shoes/Rings tight
9. Difficulty opening eyes in morning/Inc wt gain
10. Vaginal bleeding/Discharge?
 When was the last US:
1. How many babies
2. Is the baby kicking
When is the due date?
Have you been pregnant before?
PMH
FH of PET

COUNSELLING: Do you like me to explain it to you?


It is a very serious condition
What do you know about it?

Preeclampsia is a condition of pregnancy marked by high blood pressure and excess protein in your
urine after 20 weeks of pregnancy. Preeclampsia often causes only modest increases in blood
pressure. Left untreated, however, preeclampsia can lead to serious — even fatal — complications for
both you and your baby.

Preeclampsia develops only during pregnancy. Risk factors include:

1. History of preeclampsia. A personal or family history of preeclampsia increases your risk of


developing the condition.
2. First pregnancy. The risk of developing preeclampsia is highest during your first pregnancy
or your first pregnancy with a new partner.

Please note that this is only a draft version based on several sources, including: Dr. Basel Mohasen’s lectures,
Toronto Notes, Therapeutic Choices and others. Edited and organized for the sake of all attendances of the
Canadian Osce Exams: NAC OSCE and MCCQE2. by: Dr. Merlyn D’Souza and Dr. Zeev Gross, Spring 2011. Page 169
3. Age. The risk of preeclampsia is higher for pregnant women younger than 20 and older than
40.
4. Obesity. The risk of preeclampsia is higher if you're obese.
5. Multiple pregnancy. Preeclampsia is more common in women who are carrying twins,
triplets or other multiples.
6. Prolonged interval between pregnancies. This seems to increase the risk of preeclampsia.
7. Gestational diabetes. Women who develop gestational diabetes have a higher risk of
developing preeclampsia as the pregnancy progresses.
8. History of certain conditions. Having certain conditions before you become pregnant —
such as chronic high blood pressure, migraine headaches, diabetes, kidney disease,
rheumatoid arthritis or lupus — increases the risk of preeclampsia.
Most women with preeclampsia deliver healthy babies. The more severe your preeclampsia and the
earlier it occurs in your pregnancy, however, the greater the risks for you and your baby.
Preeclampsia may require induced labor and delivery by Caesarian section. Complications of
preeclampsia may include:
Lack of blood flow to the placenta. Preeclampsia affects the arteries carrying blood to the
placenta. If the placenta doesn't get enough blood, your baby may receive less oxygen and fewer
nutrients. This can lead to slow growth, low birth weight, preterm birth and breathing difficulties for
your baby.
1. Placental abruption. Preeclampsia increases your risk of placental abruption, in which the
placenta separates from the inner wall of your uterus before delivery. Severe abruption can
cause heavy bleeding, which can be life-threatening for both you and your baby.
2. HELLP syndrome. HELLP — which stands for hemolysis (the destruction of red blood cells),
elevated liver enzymes and low platelet count — syndrome can rapidly become life-threatening
for both you and your baby. Symptoms of HELLP syndrome include nausea and vomiting,
headache, and upper right abdominal pain. HELLP syndrome is particularly dangerous because
it can occur before signs or symptoms of preeclampsia appear.
3. Eclampsia. When preeclampsia isn't controlled, eclampsia — which is essentially preeclampsia
plus seizures — can develop. Symptoms of eclampsia include upper right abdominal pain,
severe headache, vision problems and change in mental status, such as decreased alertness.
Eclampsia can permanently damage your vital organs, including your brain, liver and kidneys.
Left untreated, eclampsia can cause coma, brain damage and death for both you and your
baby.
4. Cardiovascular disease. Having preeclampsia may increase your risk of future
cardiovascular disease.

Admit you ……..

Stabilize you
MgSo4
IV Labetolol
May consider Steroids for babe

Back to Content

Please note that this is only a draft version based on several sources, including: Dr. Basel Mohasen’s lectures,
Toronto Notes, Therapeutic Choices and others. Edited and organized for the sake of all attendances of the
Canadian Osce Exams: NAC OSCE and MCCQE2. by: Dr. Merlyn D’Souza and Dr. Zeev Gross, Spring 2011. Page 170
REQ FOR CS
34 wks pregnant primi requesting for CS

Intro: As I understand you’re 34 weeks pregnant & arte requesting a Cs.


Before I proceed any further,I need to ask qns to reach the best plan.
WHY?
Pt: I believe it is painfull
 What makes you believe it is painful?
 Have you’d a prior experience?
Pt: My sister had a NVD
Never imply that sister did not receive best care
There are 2 options:
There are a lot of options to control pain nowadays & different people have a different pain
threshold

 Are you on regular F/u?


 Is it a planned pregnancy?
 When was the last time you saw a doctor?

if not on reg F/u as she is alone BF left etc always Empathize,Ask how she is coping with him
leaving & how she is handling the stress
How do you support yourself financially?
I can see that this is a very difficult period & I want you to know that there are a lot of help &
resources available in the community. I will make sure you’re connected to a social worker who
will help you support you & your child & will help you to start your life

If on reg F/U
 When was the last time you’d your BP measured?
 Sx of PE: Any HA/Nx/V
Blurry vision/Abd pain......
MUM’S STABILITY:
Abd pain
Contractions
Vaginal bleeding
Vaginal discharge
BABY:
Is baby kicking like before
DUE DATE
OBG Hx:
Have you been pregnant before?
How many times
How about abortions/Mc
If +
 At how many weeks?
 What reason
 When
 Any complications
 How did you feel about that?
PMH:
Risk fcts
NEXT come to PAIN CONTROL:

As I understand you want a CS,however if pain is a major concern, there are several options:
You can attend antenatal classes that will teach you to breathe, meditate
During your delivery a person will accompany you to give you support & emotionally support you
If that does not work, there is another very popular method of delivery: EPIDURAL Anaesthesia ...
explain....

Please note that this is only a draft version based on several sources, including: Dr. Basel Mohasen’s lectures,
Toronto Notes, Therapeutic Choices and others. Edited and organized for the sake of all attendances of the
Canadian Osce Exams: NAC OSCE and MCCQE2. by: Dr. Merlyn D’Souza and Dr. Zeev Gross, Spring 2011. Page 171
An epidural block is a common type of anesthesia for labor and delivery. During labor, a needle is
placed in the epidural space, which is just outside the spinal canal. A small, hollow tube called a
catheter is inserted through this needle. Once the catheter is in place, the needle is removed and
medication is injected through the catheter to numb your lower abdomen and birth canal. It may take
10 to 20 minutes to feel pain relief from an epidural block. As labor continues, the medication can be
adjusted to help keep you comfortable.
Painless & effective
SE: rarely it may causeHa/Dizziness/infection & may prolong labour

If still does not accept:


What is your understanding about CS?
It is a major surgery which is effective & lifesaving.
However if there is no real indication,NVD is preferred as it is natural
With CS there is a scar
Longer recovery
More bleeding
Higher risk of infection

Why don’t you think about it


Give brochures
Refer to Obstetrician for 2nd opinion
F/U in 2 weeks

Back to Content

Please note that this is only a draft version based on several sources, including: Dr. Basel Mohasen’s lectures,
Toronto Notes, Therapeutic Choices and others. Edited and organized for the sake of all attendances of the
Canadian Osce Exams: NAC OSCE and MCCQE2. by: Dr. Merlyn D’Souza and Dr. Zeev Gross, Spring 2011. Page 172
32F with 34 weeks pregnancy in hosp clinic,3yrs ago had
an urgent CS due to cord prolapsed,needs her
file,counsel her
See if request is logical
Ask type of Cs
Why not happy with last Cs
May be bad experience pain/bleeding/Complications
Maybe dead baby
Was it 1st Cs or 2nd
Was it CLASSICAL Cs? Then always Cs
Risk of rupture of Classical Cs ------12% of which 10% will die
LSCS risk of rupture is 1%

INTRO: As I understand you’re here cuz you want your file & based on your report you’d a
hospital delivery because of cord prolapsed & it was an urgent Cs.
Why?
PT: Delivery by midwife who wants to look at it
We will give you the file, but until then I want to discuss
Pt: in a hurry
Because you’ve had a previous Cs & you want a midwife. In order to make a proper decision
you’ve some imp info to know
What happened the last time?
When did you know?
How many week s were you?
How did you feel?
What was done?
Did they explain it to you?
Was there any bleeding/Infection
How was the recovery period/
How is the baby/ Is it a boy/Girl
How old?
Is the baby healthy?
If baby was fine & no complications:
Looks like it was a right decision & the outcome was good
What is your understanding about cord prolapsed?
Cord is squeezed between head of baby & pelvic bones.It is a life threatening condition & needs
urgent intervention

Have you ever been pregnant other times?


How are you doing in this pregnancy?
What was your last F/u
Bp?
US
Baby kicking
PMH
Due Date

Waht is your understanding of Cs?


There are different methods:
Classical Section Vertical incision
Lowere Segment C Section ----->transverse incision

Most common is the transverse sectionThe cut is parallel to the fibres & thus it is a strong scar

If you go into labour there is a lot of pressure & tension on the scar & with continuous pressure
there can be rupture of scar this is concerning

Please note that this is only a draft version based on several sources, including: Dr. Basel Mohasen’s lectures,
Toronto Notes, Therapeutic Choices and others. Edited and organized for the sake of all attendances of the
Canadian Osce Exams: NAC OSCE and MCCQE2. by: Dr. Merlyn D’Souza and Dr. Zeev Gross, Spring 2011. Page 173
There will be a lot of bleeding we might not be able to help you & the mechanism of delivery will
stop
Chances of rupture in cl S is 12% of which 105 will die
However if you want to continue the decision is yours Your life & the baby’s are endangered

With a transverse Cs We can give you chance of normal delivery in hospital,as in case we need to
do an urgent Cs we can

If Not ConVinced:
Why don’t you go back to your midwife & talk to her & mention
She is trained & qualified We share the same guidelines

We can arrange 2nd opinion


F/U

Back to Content

Please note that this is only a draft version based on several sources, including: Dr. Basel Mohasen’s lectures,
Toronto Notes, Therapeutic Choices and others. Edited and organized for the sake of all attendances of the
Canadian Osce Exams: NAC OSCE and MCCQE2. by: Dr. Merlyn D’Souza and Dr. Zeev Gross, Spring 2011. Page 174
54 year old female comes to clinic concerning about using
HRT.
.
When a patient has concern about any subject, address it very soon. Don't wait to the end.
Dr: As far as I understand you're here as you have concern about using HRT.
Patient: yes Dr. I feel I am confused about using HRT.
Always ask what do you mean by HRT. So the patient will tell you how much they known
about HRT.
Dr: I'm glad you're here so we can discuss about it and address your concerns and hopefully by
the end of the session you can make a decision regarding using HRT. Or hopefully by doing this
discussion you will have a better understanding of HRT.
Or you can say: I agree with you as there are a lot of confusion about HRT and the reason for
this confusion is that in the past because it was used to be given routinely to all women who
reach a certain age, however 10 years ago there was study called " women health initiative" in
which the authors found that the numbers of the patients with serious side effects are very high.
However those ladies used HRT for a long time.
Serious side effects are
Cancer,
Heart attacks and
Strokes. For that reason the routine use of HRT was stopped.
Nowadays we have a better understanding and have better guidelines. Not only that we do it on
the individualized basis.
We use it only for short time, they don't exceed five years. So using HRT within five years is
safe.
So I would take some information from you and we will discuss about the risk factors and if you
are a good candidates we can make a decision to prescribe it or not.
Dr: What makes you interested in HRT?
Patient: because of hot flushes.
At this stage if the patient give you the symptom, it is your chief complaint.
But if patient doesn't give you any symptoms, you should start with her LMP
If she starts with the symptom of hot flushes, ask the patient
1. When did hot flushes start,
2. Is it all the time,
3. On & off or continues,
4. How many attacks,
5. Day or night,
6. How do you feel that you have it.
7. Night episodes, you have any night sweating, does it wake you up.
Asked patient if the hot flushes wake her up during the night, and if she needs to change her
gown of nights sweats.
1. Affect your sleep and how does it affect your concentration.
2. Change in your mood, anybody has told you that your short tempered, and if you
3. feel tired.
4. Some women with the same symptoms may notice some change in their sexual life.
d) So the doctor should ask with whom do you live?
e) Are you sexually active?
f) Any dryness or pain during the intercourse?
8. Any change in your urination?
Have you ever lost control?

Please note that this is only a draft version based on several sources, including: Dr. Basel Mohasen’s lectures,
Toronto Notes, Therapeutic Choices and others. Edited and organized for the sake of all attendances of the
Canadian Osce Exams: NAC OSCE and MCCQE2. by: Dr. Merlyn D’Souza and Dr. Zeev Gross, Spring 2011. Page 175
9. Last period?
 Are you periods regular or not?
 If it's irregular, when did it start to become irregular?
 Are your periods heavy or not?
 Any clots?
 *Any bleeding or spotting between periods? This is a very important point.
10. Bone pain? Any fractures? Any family history of osteoporosis? If yes, tell the patient that you
will discuss this in another meeting. Because that's another session to discuss about using
steroids, smoking, alcohol, caffeine, warfarin and diet. If she takes calcium supplements.
MGOS for GYN cases: Menstural, Gynecologic diseases, Obstetrics, Sextually transmitted
disease
Dr: any history of gynecological disease like polyps, cysts, any pelvic
intervention/instrumentation, surgeries.
Dr: did you use to take any oral contraception? If yes, which one and did you have any side
effects? Also you should ask about her last smear.
Because she is 50+ you should ask about her mammogram.
At any age you ask about Pap smear, once you reach 50 to ask about mammogram and when
the patient pass 65 you should add bone density.
You can ask about her obstetrics history, like have you ever been pregnant if yes how many
times you have been pregnant?
Now use the transition...
Because this is the first time I met you, I would like to ask you about your past medical and
social history. Is there any long-term disease, hospitalization before, any surgery, diabetes, or
hypertension. Any history of allergy, and the medication she takes.
ABCD: Active liver disease, vaginal Bleeding, Cancer, DVT
For A you ask about any history of Active liver disease. Have you ever been yellowish? Any
dark urine or pay stool?
For B you should ask about any vaginal Bleeding? ... You have already asked these question
before
For C you should check about Cancer. I would like to ask about constitutional symptoms here to
see if there is any endometrial cancer. Fever, chills, weight loss, appetite, lumps & bumps. A
history of cancer in yourself or family (breast cancer, endometrial cancer,and colon cancer).
For D you should ask about any history of swelling in the legs (DVT), any history of heart
attacks, pulmonary embolism or stroke.
Social history: smoking, taking alcohol, recreational drugs, how does she support financially
herself, how does this affect her life and ask about osteoporosis.

Usually in this set of scenario, you tell her on the basis of the history you are good candidates
for HRT.
However as I told you it is an important information to tell you to make your decision.
As we go through different stages of life usually for ladies, we go to the stage called menopause
which is vary between person to person.
At this stage there is hormonal changes and ovaries start to produce less hormones specialty
estrogen and progesterone and that changes affect the whole body. It can explain about
dryness, decreasing or absence of periods. And that's why we try to replace those decreased
hormones by HRT. They are the same hormones but we give it through external sources either
tablets or skin patches.
As I told you before there is a balance it's your decision to make. And the balance is to use it up
to five years. Using more than five years would increase the risk of stroke, heart attack or some

Please note that this is only a draft version based on several sources, including: Dr. Basel Mohasen’s lectures,
Toronto Notes, Therapeutic Choices and others. Edited and organized for the sake of all attendances of the
Canadian Osce Exams: NAC OSCE and MCCQE2. by: Dr. Merlyn D’Souza and Dr. Zeev Gross, Spring 2011. Page 176
cancers depending on what we call it estrogen dependent that includes breast and endometrial
cancer. And some studies showed that it might increase the risk of Alzheimer's disease.
So the risk of use for less than five years is not significant and still acceptable. So if you want to
use it the shorter the better.
To get rid off the hot flushes there are other measures like exercise or herbal supplements that
you can try to improve the symptoms.

The HRTs are the same as OCP's but in smaller doses and you can take one tablet a day. They
have a few side effects like weight gain, bloating, nausea, abdominal distention and pain but
they improve by time.
The serious side effects are headaches, swelling of the legs or chest tightness which whenever
happen you should go to emergency room. By using these HRT's your periods may stop or you
may see spottings.
If the patient had hysterectomy before you only give estrogen without progesterone, otherwise
you should give both.
Because you take it regular shootout regular ultrasound scans to check the thickness of the
endometrium and sometimes we should take a sample

Back to Content

Please note that this is only a draft version based on several sources, including: Dr. Basel Mohasen’s lectures,
Toronto Notes, Therapeutic Choices and others. Edited and organized for the sake of all attendances of the
Canadian Osce Exams: NAC OSCE and MCCQE2. by: Dr. Merlyn D’Souza and Dr. Zeev Gross, Spring 2011. Page 177
INFERTILITY
(Sometimes it is not easy/Sometimes it takes time/I’m glad you’re here)
Intro:
As I understand you’re here because you’ve been to get pregnant for the last 14
mo, during the next few minutes tell me more about this difficulty.
Did you seek medical attention before?
(<35 –1yr/35 -40 6m0/>40 ASAP)
PID & other med condts: ASAP
How long have you been in this Relationship?
How long have you tried?
Have you ever been pregnant before?
Have you ever had Mc or Abortion?
Spouse: has he had children from a previous relationship?
Let us talk about your Partner:
(If less time Fast otherwise get details)
Fast: Was he ever investigated?
Did he have Semen analysis?
What was his sperm count?
Detailed:
How is his health
Does he have (Htn?DM/On meds)
Any back trauma,back pain?
Any Surgeries
Any H/o mumps in childhood?
H/O Ca, Rxt Cxt,STIs?
Any Psy meds,Stress,travel a lot?
Exposed to heat at work or recreational way?
COITAL Hx
Some qns about intimacy:
How often do you have IC with husband?
How do you monitor your temp?
How do you measure your urine test?
Is your husband capable of having an erection & ejaculation?
Do you use any lubrication?
MENSTURAL
When was your LMP?
GYN:
SEXUAL:
Any STIs

Back to Content

Please note that this is only a draft version based on several sources, including: Dr. Basel Mohasen’s lectures,
Toronto Notes, Therapeutic Choices and others. Edited and organized for the sake of all attendances of the
Canadian Osce Exams: NAC OSCE and MCCQE2. by: Dr. Merlyn D’Souza and Dr. Zeev Gross, Spring 2011. Page 178
16 YOF information about Pap Smear,
counselling, health maintaining issues (comes with HEADDSSS).
Whenever there is counselling – take history.
.
Pap smear –
What do you like to know about Pap smear?
Usually we offer it for people who are sexually active, for that reason I’d like to know if you are in a
relationship?
Are you sexually active?
When did you start?
Any other relationships or partners prior?
Do you use protection?
What oprotection do you use?
Any STD (blisters, ulcers, warts) in the last 6 mo?
MGOS:
M:LMP
How often do you get your menses?
Are they regular?
Are your periods painful?
Are they heavy?
G – any gynaecologic disease? Any pelvic exam?
O: Any H/o pregnancies/Abortions?
Past medical Hx?
HEADDSS .... Counsel about Seat belts
Mood & Risks of suicide?

Counselling: Why? & How?


I am glad you came here today to talk about Pap smear. As a matter of fact PS is one of the most
successful screening tests to pick up one type of dangerous cancer called “Ca Cx” which is caused by
a virus called Human Papilloma virus & a condom does not protect you from this.
It is important to pick it up early, since by time it starts to give symptoms it is too late. Let me explain it
to you.
The area connecting the vagina to womb is called Cervix, & from the outside it looks like this:(Draw the
circle with a dot) from this part we’ve to get a sample it gets infected with HPV virus which is similar to
wart virus, but in the cervix, it leads to cancer. It should be done a week after your menses. It has to be
done in a certain way, there will be a nurse with me,& you will be on your back, the exam bed has
pedals to support your feet. We will use a speculum which come in different sizes & are plastic &
disposable & we use a water based lubricant.
If spatula rotate it to 360 & put on a slide, fix it & send to lab.
If brush, rotate it 5 times put in fluid & send to lab. Results will be back in 2 weeks. If all is well, we will
not contact you.
PS has to be done every year .Once results are normal for 3 yrs & you’re with same partner you casn
do it every 2 years till 69 years old, when you can stop
,If you change your partner, you’ve to do it yearly again. Other hazards – drinking and driving.
Back to Content
YOUNG WOMAN: ANTENATAL COUNSELLINg
Please note that this is only a draft version based on several sources, including: Dr. Basel Mohasen’s lectures,
Toronto Notes, Therapeutic Choices and others. Edited and organized for the sake of all attendances of the
Canadian Osce Exams: NAC OSCE and MCCQE2. by: Dr. Merlyn D’Souza and Dr. Zeev Gross, Spring 2011. Page 179
History
Ask how Pt feels about being pregnant
When did you do the test? How did you find out?
Congratulate if she is happy. It is a very exciting time of your life.
Ask questions about the pregnancy
LMP:
LLMP was it similar to prev menses or less bleeding?
Calculate EDD; - 3mo + 7 days
Sx:,
N/V/Breast engorgement/Inc visits to washroom
Rh status
If nausea severe, ask about dizziness
O: GTPAL
Any complications in previous pregnancies: HTN/DM/Twins/Congenital anomalies
G: (surgeries, infections, PAP’s)
S: Any STIs
PMH:
Vaccinations, diabetes, hypertension, heartdisease, genetic diseases, kidney diseases, immunological
diseases
Past history of surgery-especially childhood
Family Hx:
Genetic disease, prematurity, early onset deafness
SHx:Medications, Smoking, Alcohol, Recreational drugs
OTC if on Aspirin ask to stop & change to Tylenol
Social Hx:
partner, support, provisions for child
With whom do you live?
How does your partner feel about this pregnancy?
Do you feel safe in this relationship?
Do you have pets?
If has cat,not to change litter.

COUNSELLING:
I will confirm pregnancy by blood work
Physical
Vitals, weight, full exam including PAP smear (if not done in last 6 mo)and cultures
Investigations
CBC, Lytes, INR/PTT, Urea, Creatinine, Blood Type, VDRL, Rubella antibody,
Serum folate, Hepatitis, +/- HIV, Urine dip and microscopy, ECG if indicated,
+/- sickle cell and thalessemia screens.
Nuchal Translucency at 12 weeks
Maternal serum screen at 16 weeks
Anatomy ultrasound at 18-20 weeks
Glucose challenge test at 24 weeks
+/- Rhogam at 28 weeks
Diet, smoking, alcohol, exercise, medications, morning sickness
Average weight gain is 25-35 lbs with 5-10ibs up to 20 weeks and then 1lb/week thereafter
Risks of Down’s 1/200 at 35
Consult MD prior to meds
For morning sickness eat bland foods, small portions, Diclectin is an option
Hemorrhoids, back pain, heartburn and increased vaginal discharge are common
Visits are every 4 weeks until 28 weeks then every 2 weeks
Back to Content
19/2/2011

Please note that this is only a draft version based on several sources, including: Dr. Basel Mohasen’s lectures,
Toronto Notes, Therapeutic Choices and others. Edited and organized for the sake of all attendances of the
Canadian Osce Exams: NAC OSCE and MCCQE2. by: Dr. Merlyn D’Souza and Dr. Zeev Gross, Spring 2011. Page 180
Introduction
What to write on the note before entering the room:
 Name
 Age
 CC
 What required
 DDx

First buzz – turn and read the stem (2m)


Second buzz – knock the door and enter.
The examiner might tell you that at the end he will ask you 1 or 2 questions.
In the next buzz – it will be the time for the questions. If he doesn’t ask – complete the task.
After another minute there will be a longer bip sound.

Short station (history, physical exam, s/e of psychiatric medications):


1.5 m buzz – enter
Short buzz after 4.5 m (do
“Thank you for the information, I’ll do the physical exam and I’ll take it from there.”

CC
“As I understand you have ...for ... can you tell me more from the moment you started to notice it.”
“I am glad you took the time to come here

In case of physical exam:


Intro: name, position, why you are here, time that I am going to spend with you and for which purpose.
“Good morning, my name is...I am the attending physician here. I understand you are here because....In
the following 5 minutes I will perform physical exam, hopefully towards the end we will reach a
workable plan. If you feel any discomfort please inform me. Do you have any questions?”

Imaging, blood work


“Hello, good afternoon Mr. ... as I understand you are here to get your blood work results, since this is
the first time I see you before I am discussing the results with you I have some questions to discuss with
you.”
Than: Why, 1st time, who and when.
In case of breaking bad news: SPIKE
Setting
Perception (What is your understanding about the test, and why you are doing that”
Invitation (How much details do you like to discuss with you? DO you like anyone to be with you?)
Knowledge (What do you know about the condition?)
Expectations (What are your expectations from today’s visit? What is your expectation from the result?)
Gives the result.

Questions for telephone session:


What’s your number?
Where do you live?

“...did I say it right?”


Please note that this is only a draft version based on several sources, including: Dr. Basel Mohasen’s lectures,
Toronto Notes, Therapeutic Choices and others. Edited and organized for the sake of all attendances of the
Canadian Osce Exams: NAC OSCE and MCCQE2. by: Dr. Merlyn D’Souza and Dr. Zeev Gross, Spring 2011. Page 181
Connecting with a colleague:

Psychiatry

Psychosis
1. 55 yo, believe that have strange feeling in hands. Do mental exam.
Either organic, late onset of schizophrenia, not complying with medication.
2. 35 yo, believes that the RCMP chasing him.
Persecutory delusions. Reassurance about his safety. DDx substance abuse.
3. 24 yo, brought by his roommate because haven’t been himself in the last 10days.
Can be acute psychosis, substance abuse, HIV, mania
4. 30 yo, wants to arrange DNA test for his children.
5. 17 yom, worried about contamination – wants to be admitted to get rid from it. 10min –
councsel.
6. 22yo, diagnosed with schizophrenia 6wk ago, concerned about his condition.
Think about suicide!
7. 17 yo male, pain in his neck.
s/e of drugs.
8. 35 yo, brought by the police because he wanted to slaughter his children (thinks he his
Abraham).
Ask him “Who is Abraham?”
Ask early about: “How is your mood today?” – To differentiate from mania.

Mood (Presenting symtoms)


MI PASS ECG
1. Low mood for the last 6w.
2. Patient with difficulty to sleep:
2.1. 22 yof
2.2. 35 yof
2.3. 75 yof asking for sleeping pills
3. Suicide case.
4. Presentation with tiredness 34 yo.
5. 40 yom hasn’t been himself for the last 3w – his wife concerned.
6. 70 yo has back pain for 3w (x2 cases).
7. Dysthemia case. A young lady with low mood for years

DIG FAST
1. Impulsive behaviour – might be presented with intoxication to the ER. Sexual activity with no
protection. Issues with the law (fighting in the bar, waking up the neighbours).
2. Grandiosity – some delusional ideas.
3. Patient who wants to discontinue the medication.

Anxiety
1. Panic attack – heart racing, sob, dizziness, tingling, numbness (hyperventilation – hypocapnea)
STUDENTS FEAR 3C’s

Please note that this is only a draft version based on several sources, including: Dr. Basel Mohasen’s lectures,
Toronto Notes, Therapeutic Choices and others. Edited and organized for the sake of all attendances of the
Canadian Osce Exams: NAC OSCE and MCCQE2. by: Dr. Merlyn D’Souza and Dr. Zeev Gross, Spring 2011. Page 182
2. Patient already diagnosed recently with PA or Panic disorder or generalized anxiety – discuss the
treatment.
Delirium and Dementia – Cognition disorders
1. 57 yom difficulty with her memory. History and mental status exam (mini mental). 5min.
2. 67 yof difficulty with her memory. Score for mini-mental 20.
3. 67 yom came with his wife, concerned about his memory for the last 3m. Next 15min talk with
him.
4. 70 yom, s/p hip replacement 3d ago. Didn’t sleep last night (reversed sleep cycle) – delirium.
Fragmented sleep cycle – dementia.
5. His dad is not being himself. You talk with the son. You cannot do mini-mental to the son.
6. Talk to the son about his mom that is in senior home. He is concerned – she was given 15u
instead of 5u of Insulin. “It looks like there is some kind of medical error.”

Eating disorder, borderline, schizotypal, conversion


1. 16yof, the parents concern that she loses weight.
Part of the DDx is figure out that she has amenorrhea.
2. 22 yof wants to be admitted. She wants to kill herself.
If is the first time – you need to admit her.
If it is several times – it’s not necessary to admit her.
You have to finish the assessment.
If she lives the room before finish the interview – you will write form #1.
Usually people with psychiatry problem have: social worker and case manager.
Have you ever seen by psychiatry.
3. Schizotypal disorder
4. Sudden loss of function. Seen by two doctors, one of them specialist in that field – it means it is
conversion.
4.1. Loss of vision in her rt eye. Seen by ophthalm.
4.2. 22 abdominal pain for 3w, seen by a surgeon a week ago. Counsel.
4.3. Headache for the last 6 mo, she wants to renew her thylanol 3 (x2)
5. Alcholism
5.1. AST>ALT, GGT elevated
5.2. His wife concerned he is not himself for the last 3m
6. Suicide (SAD PERSONS – score more than 4 you have to admit).
6.1. Overdose of aspirin. Medically clear.

Back to Content

Please note that this is only a draft version based on several sources, including: Dr. Basel Mohasen’s lectures,
Toronto Notes, Therapeutic Choices and others. Edited and organized for the sake of all attendances of the
Canadian Osce Exams: NAC OSCE and MCCQE2. by: Dr. Merlyn D’Souza and Dr. Zeev Gross, Spring 2011. Page 183
Psychiatry Assesment
In PSY Ds look for:

I. TIME
II. CRITERIA

If CC is psychiatric, make an early decision in MOAPS format, where:

M=Mood
Depression --MI PASS ECG
 Mood How is your mood? Do you feel down? Do you cry a lot?
Have you felt that before?
“You look down for me – is there any chance you are depressed?”
Is your mood always down or does it alternate?
Have you been very happy at times? if YES: enquire about Mania
 INTEREST: Have you lost interest in activities in doing activities that were enjoyable to
you?
“Anything makes you happy?”
If he doesn’t it any more – “Why?” (Doesn’t have time, no energy, or doesn’t enjoy it)
 PSYCHOMOTOR RETARDATION/AGITATION:
“DO you feel things are getting slower? Do you need more time to do things you did
before?”
 APPETITE
“Did you lose weight deliberately?”
 SUICIDAL Ideation
“Any plan?”
“Did you live a note?”
“Did you start to give your belongings to others?”
 SLEEP
“When you go to sleep? When wake up? Do you feel fresh?”
 ENERGY
“Do you feel tired?”
 CONCENTRATION
“When you read an article can you finish it to the end?”
“Do you find to focus to concentrate in one subject?”

Please note that this is only a draft version based on several sources, including: Dr. Basel Mohasen’s lectures,
Toronto Notes, Therapeutic Choices and others. Edited and organized for the sake of all attendances of the
Canadian Osce Exams: NAC OSCE and MCCQE2. by: Dr. Merlyn D’Souza and Dr. Zeev Gross, Spring 2011. Page 184
 GUILTY
“Do you feel guilty?”
“Do you feel there is no hope in life?”
After getting two depression episodes. If they are at least two month apart – Major depressive
For teen age istead of mood and interest is replaced by irritability and droped in school
performance.
In elder person you might have need somatic disorders.
Bipolar I (Mania) - DIG FAST (elevated mood + at least three out of the seven for a
week) sometimes it is irritated mood – than you need 4 out of seven for a week. Usually they
don’t last a week – so if they end up in hospital look for the criteria even for less than a week.
 DISTRACTATIBILITY:
“DO you find difficult to focus on one subject?”
“Are you working on more than one project at the same time?”
“How many projects do you work in?” – “Are you able to finish it or not?”
 IMPULSIVITY
“Are you spending more time than before?”
“Are you borrowing money from other people?” “For what reason?”
“Are you drinking more than before? Do you use cocaine? Which happen first? –
elevation of mood or using cocaine?”
“With whom do you live? Are you sexually active? How many partners do you have? Do
you practice safe sex?”
“Do you have any problems with the law? Speeding tickets? Any fights?
 GRANDIOSITY:
“Do you believe you’re a special person?”
“Do you believe you deserve to be treated in a special manner?”
“Do you feel you’ve a special power?”
“Do you feel you’ve a special mission?--- if Yes Always ask what is the mission? &
probe deeper & inquire about Delusions*
 FLIGHT OF IDEAS:
Do you feel thoughts racing in your head?
Do people say you’re jumping from topic to topic
 GOAL DIRECTED ACTIVITY:
“How much time you spend in your activity?”
 SLEEP
 TALKATIVE
“Anybody mentioned that you are talking faster or more than others?”
Ask:If first episode or has it occurred before?

Please note that this is only a draft version based on several sources, including: Dr. Basel Mohasen’s lectures,
Toronto Notes, Therapeutic Choices and others. Edited and organized for the sake of all attendances of the
Canadian Osce Exams: NAC OSCE and MCCQE2. by: Dr. Merlyn D’Souza and Dr. Zeev Gross, Spring 2011. Page 185
Also look for OPPOSITE mood
Relapse rate for the first time: 60% next time it is 80% third time 95%.
Intro
Why?
Concern
Assess mood today
How you were diagnose with bipolar I? When? Why? Were there any
serious consequences? Regular follow up? When you saw last your
doctor? What was the level of Lithium that time? How do you feel about
Lithium? Did you notice any s/e?
Have you ever forget to take the drug? (It will be easier in the
counselling).
“I know that you have been this question before but I am going to ask
you again – do you hear any voices. Do you worry a lot...”
Counselling
Compare mania to depression. What is your understanding of mania. It
is a condition...

O=ORGANIC (I MAD):
 ENDOGENOUS (ILLNESS);
Depression: Hypothyroid/Lupus/Ca Pancreas/Post MI/CVA
 EXOGENOUS: (Substances: MAD)
M: Medications: Dosages/duration/SE/Toxicity
A: Alcohol: How much/day?
D: Drugs:
1. What drugs have you tried?
2. When
3. How much
4. Any Hx of O/D,W/d,SE,hospitalizations?
5. Which drugs NOW?

A=ANXIETY SCREEN:
Do you worry a lot?
Interview Questions to Establish Specific Anxiety Diagnosis
Please note that this is only a draft version based on several sources, including: Dr. Basel Mohasen’s lectures,
Toronto Notes, Therapeutic Choices and others. Edited and organized for the sake of all attendances of the
Canadian Osce Exams: NAC OSCE and MCCQE2. by: Dr. Merlyn D’Souza and Dr. Zeev Gross, Spring 2011. Page 186
Questions Further Inquiry
1. Do you have sudden episodes of intense
anxiety? Establish nature of attack *
2. Do you have difficulty going to places to Inquire about crowded places, line-ups,
which you used to be able to go? movies, highways, distance from home.
3. Do you have difficulty talking to people Establish situations (one-on-one or groups).
in authority or speaking in public?
4. Are you afraid of blood, small animals or Establish precise feared situation.
heights?
5. Do you repeat actions that you feel are Ask about washing, counting, checking and
excessive? hoarding.
6. Do you have thoughts that keep going in Ask nature of thoughts (illness, harm, sex)
your mind that you can't stop? Relieved by washing hands/praying.
Do these thought cause stress for you?
How do you relieve this stress?
7. Have you experienced any emotionally Establish the nature (accident, sexual,
stressful events? torture) and timing of the trauma.
When & What happened?
8. Do you worry a lot of the time? Ask about worries related to health, family,
job and finances.

P=PSYCHOSIS
HALLUCINATIONS:
VISUAL HALLUCINATIONS:
1. Do you sense things that are not actually there?
2. Do you see things that others do not see?
3. What do you see?
4. Can you describe what you see?
5. Does it have a message for you?
6. Does the message ask you to harm yourself?
7. How do you feel about it?
8. Is this the first time?
AUDITORY HALLUCINATIONS:
1. Do you hear voices other people cannot hear?
OR :
a) If you’re alone & nobody with you, do you hear voices?
b) Do you hear voices inside your head?
2. How many voices?
3. Are the voices familiar?
4. Do you recognize the voices?
5. Do they talk to you?

Please note that this is only a draft version based on several sources, including: Dr. Basel Mohasen’s lectures,
Toronto Notes, Therapeutic Choices and others. Edited and organized for the sake of all attendances of the
Canadian Osce Exams: NAC OSCE and MCCQE2. by: Dr. Merlyn D’Souza and Dr. Zeev Gross, Spring 2011. Page 187
6. Do they talk about you?
7. What are they asking you to do?
8. Do they ask you to harm yourself?
9. Do they ask you to harm anybody else?
If YES:
10. What is preventing you from doing this?........Screens for INSIGHT
11. How do you feel about these voices? (“Some people feel comforted when
they hear these voices, others feel threatened”).
DELUSIONS:
1. Do you feel anyone wants to hurt you or harm you?
If YES: WHO & WHY?
2. Anybody tries to control you?
3. Anybody wants to put thoughts into your head? (Thought Insertion)
 Anybody wants to steal thoughts from your head? (Thought withdrawl)
4. Others can read your thoughts? (Thought broadcasting)
5. When you’re watching TV or reading the News, do you feel they’re talking
about you? (Ideas of reference)
6. Do you feel any part of your body is rotting?
7. Do you feel everybody is falling in love with you?

S=SELF CARE
 HOMICIDE:
 SUICIDE
 SOCIAL HISTORY:
o With whom do you live?
o How do you care for yourself?

PAST PSY HISTORY:


1. Any similar Sx/Ds in past?
2. Any Other psy Sx/ Ds in past? If YES:
3. Analyse Sx/extent of incapacity/Rx recvd/names of hosp/Compliance
PAST MEDICAL HISTORY:
R/OAny medical illness:DM/HTN/Thyoriod/Surgery/Head trauma/HIV/AIDS/Syphillis
SAD – Smoking, Alcohol. Drugs (especially long use of cocaine)
Screen for anxiety – are you fear a lot? any fears, especially from open places?
Screen for psychosis – “anybody wants to harm you? Sometimes people having similar
experience – they might hear voices or see things other people don’t see. How about you?”
FHx:
Please note that this is only a draft version based on several sources, including: Dr. Basel Mohasen’s lectures,
Toronto Notes, Therapeutic Choices and others. Edited and organized for the sake of all attendances of the
Canadian Osce Exams: NAC OSCE and MCCQE2. by: Dr. Merlyn D’Souza and Dr. Zeev Gross, Spring 2011. Page 188
 Anyone in family with similar Sx/Ds
 Anyone in family with other pSy Sx/Ds
 Drinking / hospitalized from psychiatry reason?
 Relationships
SADD FHx:
 Suicide
 Alcohol
 Depression/Divorce/Drug
PERSONAL Hx:
1. Prea-dulthood
2. Adulthood:
Social activity: Support system, Friendships (depth/duration/Quality) isolated, asocial
With whom do you live? If he lives alone – do you have any friends you talk with?
Current Living Situation:
Where/with whom/Relationships at home/financial support/Assistance
OCCUPATIONAL HX:
How do you support yourself?
What are your ambitions/goals/relationships/Conflicts at work?/STRESSES/ Job changes
MARITAL & RELATIONSHIP Hx; Age/Duration/areas of (dis) agreements,outcomes
MILITARY Hx Gen Adjustement,combat,Injury
Educational Hx: Highest grade/Area of interest
Religion:Strict/Permissive attitude towards suicide

PSYSOCIAL Hx:
Sx/attitudes/orientation/practises/STDs (HIV)
1. Are you currently in a relationship?
2. Are you sexually active?
3. Are you active with males, females or both?
4. How long have you been in the current relationship?
5. Are you practising safe sex?
6. Are you using condoms all the times or just sometimes?
7. Is there a risk for you to be at a risk for STDs like HIV/HBV/Syphilis?
8. How about your partner?
9. How about your previous partners? Or the previous partners of your partner?
10. Have you or your partner tested for HIV,HBV or Syphilis?/When/Outcome
11. Are you currently seeing anyone else?
12. What other relationship have you had in the past?/ Anytime with more than one person at a
time
13. Have you ever paid/received money for Sex?

Please note that this is only a draft version based on several sources, including: Dr. Basel Mohasen’s lectures,
Toronto Notes, Therapeutic Choices and others. Edited and organized for the sake of all attendances of the
Canadian Osce Exams: NAC OSCE and MCCQE2. by: Dr. Merlyn D’Souza and Dr. Zeev Gross, Spring 2011. Page 189
Allergies

Back to Content
MSE/MMSE

APPEARANCE:
1. Well dressed
2. Well groomed
3. Dress matches weather
4. Given age matches chronological age
BEHAVIOUR:
1. Agitated
2. Psychomotor retardation
3. Eye Contact
4. Co operative
5. Non hostile
6. No abnormal movts/Jerking/lip smacking
C/SPEECH:
1. Volume
2. Tone
3. Fluency
4. Articulate
MOOD& AFFECT:
Mood;Subjective Sx in pts own words
Affect (qarms)
1. Quality: Euthymic/depressed/elevated/Anxious
2. Appropiateness to thought content
3. Range:Full/Restricted/Flat/Blunted
4. Mood Congruence
5. Stability: Fixedt/Labile
PERCEPTION:
Hallucination
Illusion
THOUGHT PROCESS:
Coherence/Incoherent
Logical/Illogical
Please note that this is only a draft version based on several sources, including: Dr. Basel Mohasen’s lectures,
Toronto Notes, Therapeutic Choices and others. Edited and organized for the sake of all attendances of the
Canadian Osce Exams: NAC OSCE and MCCQE2. by: Dr. Merlyn D’Souza and Dr. Zeev Gross, Spring 2011. Page 190
Circumstantiality/Tangentiality
THOUGHT CONTENT:
 Suicidal/Homicidal Ideation
1. Low-- fleeting thoughts,no formulated plan,no Intent
2. Intermediate--More frequent ideation,well formulated plan,No active intent
3. High --Persistent ideation & profound hopelessness/Anger,well formulated plan,active
intent,believes suicide,homicide is only helpful option available
 Obsession:
1. Recurrent or persistent thoughts,impulses or images that cannot be stopped which is
intrusive or inappropriate
2. Cannot be stopped by reason & Causes marked anxiety & distress
 Preoccuption:
 Overvalued Ideas:
 Ideas of reference:
 Delusions:
 Magical thinking:
 First Rank Sx of Shz:Thought insertion/T withdrawal/T broadcasting
COGNITION: MMSE
Level of consciousness
Orientation in time/place/person
Memory: immediate,remote,recent
Attention & Conc
Global evaluation of intellect: Intellectual Fns:
INSIGHT:
JUDGEMENT:

Back to Content

Please note that this is only a draft version based on several sources, including: Dr. Basel Mohasen’s lectures,
Toronto Notes, Therapeutic Choices and others. Edited and organized for the sake of all attendances of the
Canadian Osce Exams: NAC OSCE and MCCQE2. by: Dr. Merlyn D’Souza and Dr. Zeev Gross, Spring 2011. Page 191
DELERIUM
MMSE
1st reassuare the pt,calm him down talk & do MMSE

O-O-O-O-O = 5 = Time: Year/Season/Month/Day/Date

O-O-O-O-O = 5 = Place:Country/Province/City/Street/No
O-O-O = Immediate recall:Black/Honesty/Tulip (if he makes mistakes,correct him but give
_ve
O-O-O-O-O = Concentration: Can you spell WORLD backwords?

O-O-O = Delayed recall

O-O-O = Comprehension: 3 step command

O-O = Naming 2 objects (pencil & paper)

O = Reading ; write a sentence: Close your eyes & ask him to follow the command

O= Writing

O = Repeating; No ifs ands or buts

O= Copying
Why Delerium:
Fever
Ha/photophobia?
Did you eat last night?
Abdominal pain /Flank pain?
Calf pain?
Medications
Alcohol (Last time & now)
CSx:
Ask examiner for I/O chart
& medication chart

Back to Content

Please note that this is only a draft version based on several sources, including: Dr. Basel Mohasen’s lectures,
Toronto Notes, Therapeutic Choices and others. Edited and organized for the sake of all attendances of the
Canadian Osce Exams: NAC OSCE and MCCQE2. by: Dr. Merlyn D’Souza and Dr. Zeev Gross, Spring 2011. Page 192
FORMS to Be filled: [TN10-PS52]
If during an interview a pt decides to leave & not finished......
If pt wants to kill someone or himself....ADMIT
If Pt refuses to be admitted & insists on leaving:
INVOLUNTARY ADMISSION----------FORM 1
And another doctor must come & asses him.
Cannot hold in hospital for > 72 hrs

If a wife /partner brings & dr assesses & there may be a chance that the Pt may commit
suicide/homicide, pt can be sent home, on condition that if Pt detoriates she should call back &
immediately & bring
Can file FORM1
If pt refuses voluntary admission with first dr,but second D. Can assess & can discharge if he
feels fit for discharge,or admit on VOLUNTARY basis

Admission always better on voluntary basis

If second dr admits on involuntary basis it is FORM 3 & valid for 2 wks


During these 2 wks,pt improves,& so can be discharged, or gets voluntary admission
When admission voluntary --- FORM 4
Form 4 is renewed
Released on --FORM 5
Thus FORMS 3 & $ are for Rx

Back to Content

Please note that this is only a draft version based on several sources, including: Dr. Basel Mohasen’s lectures,
Toronto Notes, Therapeutic Choices and others. Edited and organized for the sake of all attendances of the
Canadian Osce Exams: NAC OSCE and MCCQE2. by: Dr. Merlyn D’Souza and Dr. Zeev Gross, Spring 2011. Page 193
PANIC Attack
ONSET:
1. When did it start
2. How did it start?
3. What were you doing at that time?
COURSE:
1. Is the Intensity same now as it was when it all started?
2. How about the frequency?
3. What made you come in today?
DURATION:
How long does each attack last?
PQRSTUV
1. Where exactly does it hurt you?
2. Can you describe the pain?
3. Does it move to anywhere else n your body?
4. On a scale of 1 to 10,wher 1 is mild & 10 is max,where would you rate this pain?
5. Has this affected your life in any way?
6. Is there a particular time it comes on?
7. Did you ever have this before?
AAA
Alleviating Fcts:
What makes it better?
What Aggravates it?
1. Exercise
2. Stress
3. Certain situations or places?
4. Coffee?
5. Medications?
6. When passing urine or having a bowel movt or cough?
ARE YOU AFRAID THAT AN ATTACK IS COMING?
Assoc Sx: first R/o cardiac then GI & then shift to Psy
 N/V/Diarrhoea
 Heart racing/ Sweating/ Dizzy
 Decreased wt & Increased appetite/ Tremors
 Headache
 Tingling & Numbness
 Nervous & Out of control
 Do you feel you’re going to die?
 During these attacks do you feel things are unreal?
 During these attacks do you feel you can see yourself?
 Do you feel you’re going crazy?

Please note that this is only a draft version based on several sources, including: Dr. Basel Mohasen’s lectures,
Toronto Notes, Therapeutic Choices and others. Edited and organized for the sake of all attendances of the
Canadian Osce Exams: NAC OSCE and MCCQE2. by: Dr. Merlyn D’Souza and Dr. Zeev Gross, Spring 2011. Page 194
 Do you feel worried about being in places or situations where escape might not be
possible
 e.g: Crowded places
 Closed spaces
If YES:
* What place or situation?
* What happens in such a situation?
* How has it impacted your life?
* How have you dealt with this issue?
Here ask for anxiety disorder: Are you a person who worries a lot?
Interview Questions to Establish Specific Anxiety Diagnosis
Questions Further Inquiry
1. Do you have sudden episodes of intense Establish nature of attack
anxiety?
2. Do you have difficulty going to places to Inquire about crowded places, line-ups,
which you used to be able to go? movies, highways, distance from home.
3. Do you have difficulty talking to people in Establish situations (one-on-one or
authority or speaking in public? groups).
4. Are you afraid of blood, small animals or Establish precise feared situation.
heights?
5. Do you repeat actions that you feel are Ask about washing, counting, checking and
excessive? hoarding.
6. Do you have thoughts that keep going in Ask nature of thoughts (illness, harm, sex).
your mind that you can't stop?
7. Have you experienced any emotionally Establish the nature (accident, sexual,
stressful events? torture) and timing of the trauma.
8. Do you worry a lot of the time? Ask about worries related to health, family,
job and finances.
a.
MOAPPS
MOOD
RISK FCTS:
To gain more insight into your condition, I need to ask some questions about your personal life :
Do you:
 Smoke
 Take alcohol
 Recreational drugs (in case of cocaine – ask if sniffs or injects it. If injects – continue by
r/o HIV symptoms)
 Are you on any medications?
 Did you take anything for a cold or flu
 Are you taking OTC products/herbal remedies?
 Are you allergic to anything?
 When was your last period?
Are you going through a stressful situation in your life?
Please note that this is only a draft version based on several sources, including: Dr. Basel Mohasen’s lectures,
Toronto Notes, Therapeutic Choices and others. Edited and organized for the sake of all attendances of the
Canadian Osce Exams: NAC OSCE and MCCQE2. by: Dr. Merlyn D’Souza and Dr. Zeev Gross, Spring 2011. Page 195
How are you coping with it?
FAMILY H:
Does anyone in your family have a similar condt?
SOCIAL H:
COUNSELLING:
From what you’ve told me, your chest pain seems related to a condition called “Panic Attack”
It is a fairly common condt
It's not known what causes panic attacks or panic disorder. Things that may play a role include:
 Genetics
 Stress
 Certain changes in the way parts of your brain function
Some research suggests that your body's natural fight-or-flight response to danger is involved in panic
attacks. For example, if a grizzly bear came after you, your body would react instinctively. Your heart
rate and breathing would speed up as your body prepared itself for a life-threatening situation. Many
of the same reactions occur in a panic attack. But it's not known why a panic attack occurs when
there's no obvious danger present
Nonpharmacologic Choices
 Caffeine or other stimulant use should be reduced and controlled.
 Alcohol use should be minimal; it should not be used to control anxiety.
 Reduce the “as-needed” use of short-acting benzodiazepines as much as possible; ideally, such
use should not be continued for longer than 4 days.
 Stress reduction, including relaxation training and time management, is often helpful initially.
 Specific cognitive behavioural therapy (CBT) may be required;

he selective serotonin reuptake inhibitors (SSRIs) citalopram, escitalopram, fluoxetine, fluvoxamine,


paroxetine and sertraline are all effective in reducing panic
SSRIs and SNRIs have become first-choice agents in treating panic disorder with or without
agoraphobia.2There is usually a delay in response to these agents that may be accompanied by initial
agitation. Combining the SSRI or SNRI with a brief course of low-dose benzodiazepine augmentation
therapy (i.e., no longer than 8 weeks) can increase adherence to medication and produce a more rapid
response than with antidepressants alo

Panic Disorder with Agoraphobia

The pharmacologic treatment of panic disorder with agoraphobia is the same as for panic
disorder. However, much of the disability in panic disorder with agoraphobia arises from the
avoidance behaviour rather than the panic attacks. This can be addressed with cognitive
behavioural therapy (CBT), even if medication reduces or eliminates panic attacks. CBT can be
more effective alone than when it is combined with medication.8 However, access to specialized
CBT is often limited.

Back to Content

Please note that this is only a draft version based on several sources, including: Dr. Basel Mohasen’s lectures,
Toronto Notes, Therapeutic Choices and others. Edited and organized for the sake of all attendances of the
Canadian Osce Exams: NAC OSCE and MCCQE2. by: Dr. Merlyn D’Souza and Dr. Zeev Gross, Spring 2011. Page 196
Personality
Disorders

Please note that this is only a draft version based on several sources, including: Dr. Basel Mohasen’s lectures,
Toronto Notes, Therapeutic Choices and others. Edited and organized for the sake of all attendances of the
Canadian Osce Exams: NAC OSCE and MCCQE2. by: Dr. Merlyn D’Souza and Dr. Zeev Gross, Spring 2011. Page 197
“Am I crazy?”
“There is no medical condition called like that, however sometimes patient have some
difficulties with their thoughts and reality, it is called schizophrenia.”
Mental Status Exam:
Appearance wise...dressed, gromed
Behavioral wise:
Speach wise:
Mood wise:
Perception:
Thought processing:
Thought content:
Judgement:

Mini-Mental: delirium, dementia, post-concussion

Writing a chart (SOAP):


Subjective
Objective
Assessment
Plan

Borderline Personality
Work on this event and previous attempts.
If she was diagnosed – “have you ever seen by psychiatrist? What was the diagnosis? What you
didn’t contacted your case manager/psychiatrist? ER or ICU or Weapons?
In Toronto – contact with the case manager/psychiatrist.
What is the trigger that makes her come today?
In case of crisis – do you have anybody to contact? Refer to crisis team/Social worker/
“In order to determine if I can admit you or not I need more information...
Always the same pressure like today?
If the patient mentions work – “what do you do for work?”
Anything happen recently? Have you had any other relations? Is it difficult for you to maintain
relation?
Mood, Anxiety, Drugs/Alcohol/

Back to Content

Please note that this is only a draft version based on several sources, including: Dr. Basel Mohasen’s lectures,
Toronto Notes, Therapeutic Choices and others. Edited and organized for the sake of all attendances of the
Canadian Osce Exams: NAC OSCE and MCCQE2. by: Dr. Merlyn D’Souza and Dr. Zeev Gross, Spring 2011. Page 198
22 yof wants to be admitted.
Borderline personality

When Pt wants to be admitted she may say,if you do not admit her,something bad may happen
like last time.
Pick up early when she says this ......
Start with EVENT:
Check previous attempts at suicide
Pick up early when she says something bad happened
Ask: WHAT happened?
If Suicide attempt.......
1. When & How many times before
2. Was she seen by psychiatrist?
3. Has she been to ER before?
4. Was she diagnosed & Rxed
5. Why can;t she contact her psychiatrist?
6. Was she admitted in ICU?
7. Which Rx programme does she have?
8. Does she have a crisis team & case manager?
9. Why didn’t she contact them?
If repeated attempts at suicide:
Which treatment programme does she have?
Is admission one of it?
CRITERIA FOR BPD:
 Fluctuating mood either very happy or sad
 Splitting
 Feeling of emptiness
 Failure in maintaining a relationship both on social & employment areas
 Impulsivity Drugs & Sex
 Was sexually abused as a child
Let her go or admit
1st episode admit needs Psy assessment
Look for TRIGGERS that made her come in today
Do not let her manipulate you
I really like to help you,I’m on Er duty
My job is to asses you & admit you Once admitted another Dr will asses you

 Can you tell me why you want to be admitted?


 I really want to help you...
 What bad thing will happen?

Please note that this is only a draft version based on several sources, including: Dr. Basel Mohasen’s lectures,
Toronto Notes, Therapeutic Choices and others. Edited and organized for the sake of all attendances of the
Canadian Osce Exams: NAC OSCE and MCCQE2. by: Dr. Merlyn D’Souza and Dr. Zeev Gross, Spring 2011. Page 199
In order to admit you or not, I need more info & therefore I need to ask you, & admitting you is
one option
If pt says Something Bad will happen
o What do you mean?
o When did it start?
o Did it happen before?
o How many times before?
NO EMPATHY
When did it happen the 1st time?
When was the last time?
What was done?
Were you admitted to ICU?
In addition to slashing your wrists have you used any other methods?...Like weapons or
medications?
Pt says she feels some pressure
Ask if the pressure felt today is the same as the pressure felt last time
I want to help you looks like the last few days were stressful
Have you been seen by Psy?
What was the diagnosis?
Do you still see the Psy?
When was the last time you saw him/Her?
Why did you stop?
What medication were/are you taking?
In addition to psy is there a case manager?
In case of crisis do you have anybody to talk to/ or contact?
What prevented you from talking to them today?

Ask prior to Event?


What happened 6 wks ago?
How did you lose your job?
What sort of job were you doing?
What happened?
Prior to that what sort of job were you doing?
Is it difficult for you to stay in one job?
Have you tried to find another job?
Anything happened last night?
If Boy friend left her.....How long have you been together?
How does she feel about it?
Was he supportive?
At what age was she sexually active?
Does she find it difficult to stay in a relationship?
MOOD
DEPRESSED: Ask for HOMICIDAL ideation

Please note that this is only a draft version based on several sources, including: Dr. Basel Mohasen’s lectures,
Toronto Notes, Therapeutic Choices and others. Edited and organized for the sake of all attendances of the
Canadian Osce Exams: NAC OSCE and MCCQE2. by: Dr. Merlyn D’Souza and Dr. Zeev Gross, Spring 2011. Page 200
Manic Ask Drugs/Spending/Impulsivity
ANXIETY
After boyfriend ask about fly support......
Back to Content

Please note that this is only a draft version based on several sources, including: Dr. Basel Mohasen’s lectures,
Toronto Notes, Therapeutic Choices and others. Edited and organized for the sake of all attendances of the
Canadian Osce Exams: NAC OSCE and MCCQE2. by: Dr. Merlyn D’Souza and Dr. Zeev Gross, Spring 2011. Page 201
Depression
Sleep
If comes Tired  Sleep  Energy  Mood
Sleep – how does it affect you?
DO you feel tired?

 ―During the past month have you often been bothered by feeling down, depressed or
hopeless?‖
 ―During the past month have you often been bothered by little interest or pleasure in doing
things?‖

MI PASS ECG (TO diagnose depression – needs 5 out of the 9, in which one them should be
either M or I. In teenagers irritability can replace either M or I, in elderly it can come with a
somatic presentation)
If it is one episode it is called: Major Depressive Episode (need 2w in which most of the days
with depressed mood, and 4 more criteria). If there are 2 or more MDE within the same 2m – it is
Major Depressive Disorder, if it is more than 2m – it is Recurrent Major Depressive Episodes.

M
How do you feel recently?
How is your mood?
Any chance you are depressed?

I
What do you enjoy doing?
Are you still enjoy hobies?
Anything brings happiness to you?
Why don’t you enjoy any more?
No time? No energy?

P
Do you think things are getting slower?
Do you think you need more time to do things you used to do before?

A
Any change in appetite?
Did you lose weight?
How much weight did you lose?
Was it intentional or not?

S
How about sleep?
How many hours do you sleep?
When do you go to bed?
How long before you fall asleep?
DO you wake up at night?
Please note that this is only a draft version based on several sources, including: Dr. Basel Mohasen’s lectures,
Toronto Notes, Therapeutic Choices and others. Edited and organized for the sake of all attendances of the
Canadian Osce Exams: NAC OSCE and MCCQE2. by: Dr. Merlyn D’Souza and Dr. Zeev Gross, Spring 2011. Page 202
Why do you wake up at night?
DO you feel refreshed at the morning?

S
Do you feel any chance that you might harm yourself, end your life, or any ones?
If patient says I wish I am dead, consider either he has only a feeling or a plan (active)
Do you have a plan? What is preventing you?
Did you leave a note? Did you start giving your belongings to others?
(These are definite questions for a plan)

E
DO you feel tired?

C
Do you find it difficult to focus on a specific task (for example if you are watching TV – you can
stick to the same program all through? Can you finish an article?)

G
DO you feel there is no hope in your life?
Do you feel guilty?

Regardless of any specific psychotherapy, measures to enhance treatment compliance are useful;
e.g., providing psychoeducation with the following 5 simple messages is effective:19

 Take medication daily


 Call this number for questions about side effects or other issues
 Remember that it might take 2–4 weeks to see a noticeable effect from antidepressants
 Continue to take medication even if you are feeling better
 Do not stop taking the antidepressant without checking with the physician

Please note that this is only a draft version based on several sources, including: Dr. Basel Mohasen’s lectures,
Toronto Notes, Therapeutic Choices and others. Edited and organized for the sake of all attendances of the
Canadian Osce Exams: NAC OSCE and MCCQE2. by: Dr. Merlyn D’Souza and Dr. Zeev Gross, Spring 2011. Page 203
42 yom hasn’t been himself, his wife arranged for the
meeting.
INTRO:
As I understand you’re here today,as your wife has some concerns about you.
Can you tell me more about it?
Give confidentiality.
Counseling (last 2-3m):
Based on what you told me your symptoms are consistent with a condition called “depression.” We
believe it is caused because of imbalance in some of the chemicals in the brain. Sometime there is an
event in life or cause that triggers that situation. It is common and treatable. We need to r/o other
causes – and for that we need to do some blood work.

What did I do wrong to feel so depressed?


Depression does not occur because someone has done something "wrong". Like any other medical illness,
depression is caused at least in part by biochemical changes in the brain, which lead to depressive
symptoms. This is why medications which help correct chemical imbalances in the brain relieve
depression. In fact, if a chemical imbalance is not present, antidepressant medications will not have any
effect - they will not make a person "happy" when they are not clinically depressed.

How long before I feel better?


Generally speaking, people will start to notice improvement in symptoms such as sleep disturbances or
crying spells and energy levels a few weeks after starting their treatment. Improvement in depressed
mood is usually slower, and it may take six to eight weeks before people notice they are feeling much less
depressed. If someone has not improved after three to four weeks of therapy, the dose of the initial
medication may be optimized, a different drug may be added, or the initial drug may be substituted. Up to
80% of people with depression do get better with the right medication.

Will my depression come back?


The likelihood of depression recurring depends on how many previous episodes you have had.
For people who are experiencing their first depression, the likelihood of having a second episode is
around 50%.
For people who've had two depressive episodes, chances of having a third are around 70% and for
Those who've had three and more episodes, all but 10% will experience further illness.
Having someone else in your family who has depression makes it more likely your own depression will
recur. Other risk factors for recurrent depression are the presence of chronic medical problems, a history
of early trauma or abuse, dysthymia, onset of depression younger than 25 years or older than 60 years,
and a long pattern of negative thinking, low self-esteem and relationship difficulties. A depression which
does not completely resolve with treatment, as well as severe depression, also increase the likelihood
depression will recur.
This is why most people with depression need to be treated for at least six to nine months to prevent
relapse, and for greater than 12 months if someone is being treated for a recurrent episode. Depending
on the likelihood of depression recurring, some people stay on the same dose of their medication for
long-term maintenance therapy. The saying doctors have is, "The dose that gets you well is the dose that
keeps you well" and people will do better over the long run if the same dose is used throughout.

Please note that this is only a draft version based on several sources, including: Dr. Basel Mohasen’s lectures,
Toronto Notes, Therapeutic Choices and others. Edited and organized for the sake of all attendances of the
Canadian Osce Exams: NAC OSCE and MCCQE2. by: Dr. Merlyn D’Souza and Dr. Zeev Gross, Spring 2011. Page 204
Can I pass depression on to my children?
Certain types of depression, especially, bipolar affective disorder, would appear to run in families.
However, even identical twins do not share an equal risk to develop depression, and depressive illness
appears to be a combination of vulnerability to depression (part of which may be inherited but not
necessarily), difficult life events and biochemical imbalances in the brain.

I have trouble reaching orgasm now that I'm taking an SSRI. Can I stop my medication on weekends to
improve my sexual function?
Some doctors recommend drug holidays where people stop taking their medication on the weekend. The
biggest concern about stopping and starting medication revolves around compliance issues, but there is
some evidence that people may not respond as well to the medication if treatment is continuously
interrupted. For these reasons, drug holidays are not recommended and an alternative antidepressant or
an additional medication to offset unwanted sexual side effects are better solutions.

Regardless of any specific psychotherapy, measures to enhance treatment compliance are useful;
e.g., providing psychoeducation with the following 5 simple messages is effective:19

 Take medication daily


 Call this number for questions about side effects or other issues
 Remember that it might take 2–4 weeks to see a noticeable effect from antidepressants
 Continue to take medication even if you are feeling better
 Do not stop taking the antidepressant without checking with the physician

There are good options to treat it. If you choose to go to talk therapy I can refer you to a psychology. On
the other hand we can use medications which are generally safe. Called SSRI similar to Prozac, however
like any other medical intervention have some side effects. Most of them are minor, usually improve
with time – headache, sexual...however the improvement of your mood will lag behind your
improvement in your energy, we call that the window gap, and this is of concern to us.
All contracts are verbal, besides the drug contracts – “I promise I will not use ...again...”

Back to Content

Please note that this is only a draft version based on several sources, including: Dr. Basel Mohasen’s lectures,
Toronto Notes, Therapeutic Choices and others. Edited and organized for the sake of all attendances of the
Canadian Osce Exams: NAC OSCE and MCCQE2. by: Dr. Merlyn D’Souza and Dr. Zeev Gross, Spring 2011. Page 205
Mania
DIG FAST
For diagnosis we need elevated mood + 3 criteria of the above 7 for a whole week.
Sometimes irritable – you need 4 criteria.

D
DO you find it difficult to focus on one subject?
Are you working on more than one project on the same time?
How many projects are you working on?
Can you finish it on time?

I
Are you spending more money than before?
Are you borrowing money from others?
Are you maxing out on your credit card?
Do you drink alcohol?
Are you drinking more than before?
DO you smoke or take recreational drugs? (If taking recreational drugs – feeling high)
If taking cocaine – what happened first: the episode or the taking the drug?
With whom do you live?
Are you sexually active?
How many partners have you had recently?
Have you used protection?
DO you have any problems with law?
Any speeding tickets?
Any fights?

G
DO you feel you are special?
Do you feel you deserve to be treated differently?
DO you have special powers?
Do you have special mission?

F
DO you have thoughts racing in your head?
What kind of thoughts?

A
How much time do you spend in your activities?

S
Lack of sleep?

T
Did anyone tell you that you are talking faster than before?
Please note that this is only a draft version based on several sources, including: Dr. Basel Mohasen’s lectures,
Toronto Notes, Therapeutic Choices and others. Edited and organized for the sake of all attendances of the
Canadian Osce Exams: NAC OSCE and MCCQE2. by: Dr. Merlyn D’Souza and Dr. Zeev Gross, Spring 2011. Page 206
Manic
1 Manic episode = Bipolar I
Ask if it is the first time or has it happened before. How about the opposite. Have you ever felt
high? Greater than 7 days in a row?

Insomnia in elder
Difficulty in sleeping for 6 months
Can you tell me more about it since it started?
Did you seek medical attention?
OCD
Anything at that time?
From that time till now – every night? When do you go to bed? Whe do you fall asleep? When
do you wake up?
Before you fall asleep what do you think? What comes to your mind?
When you sleep – do you wake up?
Any nightmares?
If she says she has to wake up for breakfast ask why she has to wake up?
How old is your son?
Has he been always with you or is he left and come back?
Can’t the prepare breakfast for himself?
Anybody else at home?
How about your husband?
CSx
PMHx
SHx
How does son support himself?
Is he under stress?
What is the nature of your relationship?
Give confidentiality?
How do you support yourself financially?
Any financial concern?
Ask if son contributes to finances?
If son consumes Alcohol? (How much? Does he loose control/shouts?)
Does he get angry to swear to get physical?
Does he get accesses to your finances?
Did you talk with anybody about it?
Do you feel safe going back home?
Does he have access to fire arms?
DO you have suicidal or homicidal ideation?
Based on what you told me – your sleeping troubles seems to be related to stresses in your life
called ―Elder Abused‖ which is illegal and crime against law. It is nor your mistake and you
should not accept that. You need to call the police. From studies it has shown that police
interevention improves such situations.
Son needs help – can you convince him?

Please note that this is only a draft version based on several sources, including: Dr. Basel Mohasen’s lectures,
Toronto Notes, Therapeutic Choices and others. Edited and organized for the sake of all attendances of the
Canadian Osce Exams: NAC OSCE and MCCQE2. by: Dr. Merlyn D’Souza and Dr. Zeev Gross, Spring 2011. Page 207
Police will protect you and son will be sent to rehab and anger management.
I’ll be giving you sleeping pills for three days and f/u within 3d.

Back to Content

Please note that this is only a draft version based on several sources, including: Dr. Basel Mohasen’s lectures,
Toronto Notes, Therapeutic Choices and others. Edited and organized for the sake of all attendances of the
Canadian Osce Exams: NAC OSCE and MCCQE2. by: Dr. Merlyn D’Souza and Dr. Zeev Gross, Spring 2011. Page 208
Marijuana Counselling
(Mother comes in to see you as she has discovered Marijuana in her son’s belongings)

INTRO:
As I understand you’re here because you’re concerned about your son.
What is his name?
What is your concern?
How much did you find?
Did you ask him about it?
 WHAT MAKES YOU BELEIVE IT IS MJ ?
Is he using it? Or Is he carrying it?
Is it the first time you’ve found it?
 Did you notice any CHANGES in his behaviour?
Is he excited?
Laughing out of nowhere?
Is he preoccupied?
Does he stare at a wall?
Does he talk to himself?
Is he aggressive?
Any problems with the law?
Any fights?
Any criminal records?
Is he more isolated?
 How is his MEMORY?
Is he more forgetful/lose his stuff?
Does he take more time to react?
Does he spend more time in his room?
How much time do you spend with him?
How much time is he out of the home?
How much time does he spend with his friends?
Do you know any of his friends?
What kind of activity are they involved in?
 Does he have a lot of MONEY?
Does he ask for money?
Do you believe he steals money?
Do you think he smokes/or drinks alcohol?
 How would you describe his MOOD?
Is he depressed?
Is he still interested in his hobbies?
Does he worry a lot?
Does he have excessive fears & avoid situations?
Do you have concerns that he may harm himself or anyone else?
EDUCATION:
How is he doing in school?
Have his grades dropped?
DIET:
How is his general health?
Please note that this is only a draft version based on several sources, including: Dr. Basel Mohasen’s lectures,
Toronto Notes, Therapeutic Choices and others. Edited and organized for the sake of all attendances of the
Canadian Osce Exams: NAC OSCE and MCCQE2. by: Dr. Merlyn D’Souza and Dr. Zeev Gross, Spring 2011. Page 209
Have you ever seen a psychiatrist?
Fhx: SAD
COUNSELLING:
Based on what you’ve told me.There are no changes in his health & behaviour (assumed that
there were no changes in behaviour as per mum)
When it comes to Marijuana it is a commonly used drug by teenagers, sometimes only once for
experiment. When we talk about Substance Abuse & drugs we talk about different categories.
Marijuana is a SOFT DRUG,others like: Coccaine,Heroin& Amphetmanies are HARD DRUGS
Let us talk about Marijuana first.
It is from the Cannabis family & affects the brain by feeling happy, excited & enhances
experience.Sometimes with prolonged use or in high doses can cause side effects including
apathy.
It interferes with memory,& can interfere with his studies & function & fine motor skills & may
not be able to operate machinery
It impairs judgement & he might take risks.
Can cause Lung cancer
In some teens,in high doses unmasks schizophrenia & cause psychosis
Interferes with sexual function & can cause infertility & weight gain
By itself marijuana is not strongly addictive & hence he can stop it at any time with help.One of
the concerns of Marijuana though is it acts as a bridge to Hard drugs which are addictive i.e
you’ve to increase the dose to have the same effect,which is called “TOLERANCE”,& then one
cannot stop the drug as it causes withdrawal .
It is a crime to use,hold hard drugs.People can lose their jobs.
If injected increases risk of HIV,Hepa B & C
PLAN
If you like,bring your son here I can talk to him.
It is better to be a confidante to him. Try to be close to him, someone he can trust & can talk
to.Try to make sure who’re his friends,& make sure you know what he is doing.Keep him busy
with activities.
If there are any druh prevention programmes in your community or his school,get him to attend
them & gets the knowledge.

In case of the resident who was asked to backup his supervisor orthopaed
4. I am competent – to emphasize
5. Short term – we don’t have time so we need to see her urgently
6. Long term – solve the situations that it wouldn’t occur again
Back to Content

Please note that this is only a draft version based on several sources, including: Dr. Basel Mohasen’s lectures,
Toronto Notes, Therapeutic Choices and others. Edited and organized for the sake of all attendances of the
Canadian Osce Exams: NAC OSCE and MCCQE2. by: Dr. Merlyn D’Souza and Dr. Zeev Gross, Spring 2011. Page 210
INSOMNIA:
The Sleep History
1. Time data (can also be collected as part of a sleep diary –
1. Did you nap or lie down to rest today? If yes, when and for how long?
2. What time did you go to bed last night?
3. What time did you put out the lights?
4. How long did it take you to fall asleep?
5. How many times did you awaken last night?
6. How long was your longest awake period; when was it? What time did you
finally awaken?
7. What time did you get out of bed?
8. How many hours sleep did you get last night?

2. Questions about the sleep period


1. Do physical symptoms, such as pain, prevent you from falling asleep?
2. Do mental or emotional symptoms (e.g., worry or anxiety) prevent you from
falling asleep?
3. When you awaken during the night, what awakens you? (Snoring? Gasping for
air? Dreams/nightmares? Noise?)
4. When you get up for the day, do you have any symptoms? (Headache?
Confusion? Sleepiness?)

3. Questions for the patient's bed partner


1. Does your partner snore, gasp or make choking sounds during the night?
2. Does your partner stop breathing during the night?
3. Do your partner's legs twitch, jerk or kick during the night?
4. Has your partner's use of alcohol, nicotine, caffeine or other drugs changed
recently?
5. Has your partner's mood or emotional state changed recently?
6. What do you think is the cause of your partner's sleep problem?

Hygiene Guidelines

1. Keep a regular sleep–wake schedule, 7 days per week.


2. Restrict the sleep period to the average sleep time you have obtained each
night over the preceding week.
3. Avoid sleeping in, extensive periods of horizontal rest or daytime napping;
these activities usually affect the subsequent night's sleep.
4. Get regular exercise every day: about 40 minutes of an activity with
sufficient intensity to cause sweating. If evening exercise prevents sleep,
schedule the exercise earlier in the day.
5. Avoid caffeine, nicotine, alcohol and other recreational drugs, all of which
disturb sleep. If you must smoke do not do so after 7:00 p.m.
6. Plan a quiet period before lights out; a warm bath may be helpful.
7. Avoid large meals late in the evening; a light carbohydrate snack (e.g.,
crackers and warm milk) before bedtime can be helpful.
Please note that this is only a draft version based on several sources, including: Dr. Basel Mohasen’s lectures,
Toronto Notes, Therapeutic Choices and others. Edited and organized for the sake of all attendances of the
Canadian Osce Exams: NAC OSCE and MCCQE2. by: Dr. Merlyn D’Souza and Dr. Zeev Gross, Spring 2011. Page 211
8. Turn the clock face away and always use the alarm. Looking at the clock
time on awakening can cause emotional arousal (performance anxiety or
anger) that prevents return to sleep.
9. As much as possible, keep the bedroom dark and soundproofed. If you live
in a noisy area, consider ear plugs.
10. Use the bedroom only for sleep and intimacy; using the bed as a reading
place, office or media centre conditions you to be alert in a place that should
be associated with quiet and sleep. If you awaken during the night and are
wide awake, get up, leave the bedroom and do something quiet until you feel
drowsy-tired, then return to bed.
Note: Pharmacologic (or any) interventions will be less effective if these
guidelines are not followed. In mild cases of insomnia, sleep hygiene
guidelines, practised consistently and together, may be sufficient to reinstate a
normal sleep pattern.

Difficult sleeping for the last 3m


OCD
How did it start?
Suddenly / Gradually?
From that time is it all the time or “on and off?”
Shift to “whom do you live with?”
U:
How did that lack of sleep affect your life?
Give confidentiality.
R/O:
Depression,
Drinking (her or husband), “How much? How often? Does he drink more? What is the reason?
Any change in your life? When your husband gets angry – does he start shout at you? (“Sometimes
when people are drinking it can

Did you go to the ER? How often you go to the ER? Did he ever shout at you? Does he
swear?/Shout?/Call your names? How does it affect your self esteem? Did he ever become anger to the
extent that he becomes physical? Pushing? Did he ever force you to have sex against your will? Did he
ever hit the children? Did he ever abused you in front of the children? Who’s controlling spending? (If
she says that the children are safe – you can say that children are smart and realize that).

Counselling
Based on what you told me it is called “spouse abuse” it is illegal, it is a crime and against the law. You
shouldn’t feel guilty about that. We know from studies that this situation will deteriorate, and without
proper of help it might end badly. If you are concerned with the economic situation I’d like to know that
there are a lot of resources. I’ll give the number of social support that
We know from studies...he will have some restraining...usually situations might improve.
Always give them follow up in three days.
Back to Content
Please note that this is only a draft version based on several sources, including: Dr. Basel Mohasen’s lectures,
Toronto Notes, Therapeutic Choices and others. Edited and organized for the sake of all attendances of the
Canadian Osce Exams: NAC OSCE and MCCQE2. by: Dr. Merlyn D’Souza and Dr. Zeev Gross, Spring 2011. Page 212
55 yo, believe that have strange feeling in hands. Do mental exam.
Either organic, late onset of schizophrenia, not complying with medication

INTRO:
3 ways:
1.How did it start?
OCD:
What were you doing at that time?
What happened at that time?

Ms Franco 55/F strange feeling in (R) hand x 6 mo in ER talk to her for 10 mins

Can U tell me about it?


Is it one or (B) hands?
How did it start?
Right now,how do you feel/
Right now u look concerened,anything bothering U?
U’re in the right place

What were U doing when it all started?


Anything special happened at that time?
Ask What events?.......
Or Do you remember how it started 1st time?
Is it all the time or off & on
Any particular time of the day
Any particular settings
What is special in that setting?
In your opinion what is responsible for it?
May show a pic
It may be like a radiation for you,but not for me

Where exactly do you feel it?


Anywhere else?Any weakness/numbess
Ask for HOMICIDAL/SUICIDAL ideation

Ask for Hallucinations:


P=PSYCHOSIS
HALLUCINATIONS:
VISUAL HALLUCINATIONS:
9. Do you sense things that are not actually there?
10. Do you see things that others do not see?
11. What do you see?
You look preoccupied
REASSUARE HER THAT SHE IS IN SAFE PLACE
Please note that this is only a draft version based on several sources, including: Dr. Basel Mohasen’s lectures,
Toronto Notes, Therapeutic Choices and others. Edited and organized for the sake of all attendances of the
Canadian Osce Exams: NAC OSCE and MCCQE2. by: Dr. Merlyn D’Souza and Dr. Zeev Gross, Spring 2011. Page 213
DO NOT LOSE NERVE
12. Can you describe what you see?
13. Does it have a message for you?
14. Does the message ask you to harm yourself?
15. How do you feel about it?
16. Is this the first time?
AUDITORY HALLUCINATIONS:
12. Do you hear voices other people cannot hear?
OR :
c) If you’re alone & nobody with you, do you hear voices?
d) Do you hear voices inside your head?
13. How many voices?
14. Are the voices familiar?
DO NOT LOSE NERVE

15. Do you recognize the voices?


16. Do they talk to you?
17. Do they talk about you?
18. What are they asking you to do?
19. Do they ask you to harm yourself?
20. Do they ask you to harm anybody else?
If YES:
21. What is preventing you from doing this?........Screens for INSIGHT
DELUSIONS:
8. Do you feel anyone wants to hurt you or harm you?
If YES: WHO & WHY?
9. Anybody tries to control you?
10. Anybody wants to put thoughts into your head? (Thought Insertion)
 Anybody wants to steal thoughts from your head? (Thought withdrawl)
11. Others can read your thoughts? (Thought broadcasting)
12. When you’re watching TV or reading the News, do you feel they’re talking
about you? (Ideas of reference)
13. Do you think any part of your body is rotting?
14. Do you feel everybody is falling in love with you?
MOOD
ORGANIC:
Since it is first time I’m seeing you.I’ve to ask you questions
Please note that this is only a draft version based on several sources, including: Dr. Basel Mohasen’s lectures,
Toronto Notes, Therapeutic Choices and others. Edited and organized for the sake of all attendances of the
Canadian Osce Exams: NAC OSCE and MCCQE2. by: Dr. Merlyn D’Souza and Dr. Zeev Gross, Spring 2011. Page 214
Any long term diseases?
Look for S/e of meds (Streoids,smoking,drugs)
Head Injury
Fever
Csx:
Look for Social Hx
Past Psy Hx
Self Care

Admit

If Pt asks: Am I crazy?
Thre is no medical condt called crazy.Sometimes some pts have difficulty in handling their thoughts &
this is called “Schizophrenia”

MSE
APPEARANCE:
1. Well dressed
2. Well groomed
3. Dress matches weather
4. Given age matches chronological age
BEHAVIOUR:
1. Agitated
2. Psychomotor retardation
3. Eye Contact
4. Co operative
5. Non hostile
6. No abnormal movts/Jerking/lip smacking
C/SPEECH:
1. Volume
2. Tone
3. Fluency
4. Articulate
MOOD& AFFECT:
Mood;Subjective Sx in pts own words
Affect (qarms)
1. Quality: Euthymic/depressed/elevated/Anxious
2. Appropiateness to thought content
3. Range:Full/Restricted/Flat/Blunted
Please note that this is only a draft version based on several sources, including: Dr. Basel Mohasen’s lectures,
Toronto Notes, Therapeutic Choices and others. Edited and organized for the sake of all attendances of the
Canadian Osce Exams: NAC OSCE and MCCQE2. by: Dr. Merlyn D’Souza and Dr. Zeev Gross, Spring 2011. Page 215
4. Mood Congruence
5. Stability: Fixedt/Labile
PERCEPTION:
Hallucination
Illusion
THOUGHT PROCESS:
1. Coherence/Incoherent
2. Logical/Illogical
3. Circumstantiality/Tangentiality
THOUGHT CONTENT:
 Suicidal/Homicidal Ideation
Low-- fleeting thoughts,no formulated plan,no Intent
Intermediate--More frequent ideation,well formulated plan,No active intent
High --Persistent ideation & profound hopelessness/Anger,well formulated plan,active
intent,believes suicide,homicide is only helpful option available
 Obsession:
Recurrent or persistent thoughts,impulses or images that cannot be stopped which is
intrusive or inappropriate
Cannot be stopped by reason & Causes marked anxiety & distress
 Preoccuption:
 Overvalued Ideas:
 Ideas of reference:
 Delusions:
 Magical thinking:
 First Rank Sx of Shz:Thought insertion/T withdrawal/T broadcasting
COGNITION: MMSE
Level of consciousness
Orientation in time/place/person
Memory: immediate,remote,recent
Attention & Conc
Global evaluation of intellect: Intellectual Fns:
INSIGHT:
JUDGEMENT:
Back to Content

Please note that this is only a draft version based on several sources, including: Dr. Basel Mohasen’s lectures,
Toronto Notes, Therapeutic Choices and others. Edited and organized for the sake of all attendances of the
Canadian Osce Exams: NAC OSCE and MCCQE2. by: Dr. Merlyn D’Souza and Dr. Zeev Gross, Spring 2011. Page 216
24 yo, brought by his roommate because hasn’t been
himself in the last 10days.
D/d:
1. Ac. psychosis,
2. Substance abuse,
3. HIV,
4. Mania
If started 10 days ago, why brought in today? (could’ ve been homicidal or suicidal)
If carrying a book, ask Reason
Ask Delusions for grandiosity :“Do you feel you’ve a special mission?--- if Yes Always ask what is
the mission? & probe deeper & enquire about Delusions
Mission imp May be Suicidal or Homicidal ideation

Back to Content

Please note that this is only a draft version based on several sources, including: Dr. Basel Mohasen’s lectures,
Toronto Notes, Therapeutic Choices and others. Edited and organized for the sake of all attendances of the
Canadian Osce Exams: NAC OSCE and MCCQE2. by: Dr. Merlyn D’Souza and Dr. Zeev Gross, Spring 2011. Page 217
17/M worried about contamination – wants to be admitted to get rid from it.
10min – counsel
Delusions: Dd:
1. Schizophrenia
2. Schiziod personality Disorder
3. Schizotypal PD
4. Isolated PD (older pt in 40s & usually delusions about fidelity)

Qns about Delusions


DELUSIONS:
15. Do you feel anyone wants to hurt you or harm you?
If YES: WHO & WHY?
16. Anybody tries to control you?
17. Anybody wants to put thoughts into your head? (Thought Insertion)
 Anybody wants to steal thoughts from your head? (Thought withdrawl)
18. Others can read your thoughts? (Thought broadcasting)
19. When you’re watching TV or reading the News, do you feel they’re talking
about you? (Ideas of reference)
MOOD:
How is your mood,is it down,Up or N ?
ORGANIC:R/O
Head Injury
CSx: HIV/Meningitis
Medications: Steroids,smoking,alcohol,drugs
S/e of meds,Pt may have stopped anti psychotics due to Se

Back to Content

Please note that this is only a draft version based on several sources, including: Dr. Basel Mohasen’s lectures,
Toronto Notes, Therapeutic Choices and others. Edited and organized for the sake of all attendances of the
Canadian Osce Exams: NAC OSCE and MCCQE2. by: Dr. Merlyn D’Souza and Dr. Zeev Gross, Spring 2011. Page 218
35 yo, believes that the RCMP chasing him.
Persecutory delusions. Reassurance about his safety. DDx substance abuse.

INTRO;
Early on reassure pt that this is a safe place, & invite him to sit down

As I understand you’re here because you have worries that the RCMP is chasing you.
I want you to know that this is a safe place & please come & sit.
I want to help you so please sit down
Make sure he sits in front of you.
Ask him:
 Why chasing?
 How long chasing?
 How affecting him?
 How does he handle it?
 Does he talk to anyone about it?
Here there is persecutory delusion
Besides police does anyone else want to hurt him?
Does he have special powers?
FINISH the delusions
Go to Hallucinations
VISUAL HALLUCINATIONS:
 Do you sense things that are not actually there?
 Do you see things that others do not see?
 What do you see?
 Can you describe what you see?
 Does it have a message for you?
 Does the message ask you to harm yourself?
 How do you feel about it?
 Is this the first time?
AUDITORY HALLUCINATIONS:
 Do you hear voices other people cannot hear?
OR :
 If you’re alone & nobody with you, do you hear voices?
 Do you hear voices inside your head?
 How many voices?
 Are the voices familiar?
 Do you recognize the voices?
 Do they talk to you?
 Do they talk about you?

Please note that this is only a draft version based on several sources, including: Dr. Basel Mohasen’s lectures,
Toronto Notes, Therapeutic Choices and others. Edited and organized for the sake of all attendances of the
Canadian Osce Exams: NAC OSCE and MCCQE2. by: Dr. Merlyn D’Souza and Dr. Zeev Gross, Spring 2011. Page 219
 What are they asking you to do?
 Do they ask you to harm yourself?
Also ask for tactile hallucinations
I see you’re scratching your hands
Any other areas are scratching?
When & How long?.....Pt will answer.... I do not know .......
Jump to cocaine
Do you smoke/Take alcohol/Drugs
I f Pt stands, you stand, reassure him & bring him back & ask again about drugs
Did you take an increased amt recently?
How do you take it? Snort/Smoke/IV?
If IV ask about CSx;
MOOD
R/o Mania & depression
Suicide & Homicide
If Pt leaves tell I want to file form 1 & call security

Back to Content

Please note that this is only a draft version based on several sources, including: Dr. Basel Mohasen’s lectures,
Toronto Notes, Therapeutic Choices and others. Edited and organized for the sake of all attendances of the
Canadian Osce Exams: NAC OSCE and MCCQE2. by: Dr. Merlyn D’Souza and Dr. Zeev Gross, Spring 2011. Page 220
17/M worried about contamination – wants to be admitted to get rid
from it. 10min – counsel
Delusions: Dd:
Schizophrenia
Schiziod personality Disorder
Schizotypal PD
Isolated PD (older pt in 40s & usually delusions about fidelity)

Qns about Delusions


MOOD:
How is your mood,is it down,Up or N ?
ORGANIC:R/O
Head Injury
CSx: HIV/Meningitis
Medications: Steroids,smoking,alcohol,drugs
S/e of meds,Pt may have stopped anti psychotics due to Se

 A key feature of bipolar disorder is recurrent nonadherence to medication; including the patient
in decision-making, together with psychoeducation, promotes a strong therapeutic alliance and
enhances medication adherence.
 Patients taking lithium need to maintain their usual salt and caffeine intake and monitor fluid
intake and output, making adjustments in the event of unexpected losses due to vomiting or
diarrhea.
 During acute manic episodes, patients may exhibit increased tolerance to lithium.
 Advise patients taking antipsychotics about antipsychotic-associated body temperature
dysregulation and strategies to prevent heat stroke (e.g., hydration, sun protection).
 For lithium-associated cognitive impairment, check lithium level and thyroid function. Lowering
the dose or using a slow-release formulation may improve cognitive function.
 Patients who experience tremor while taking lithium may benefit from elimination of dietary
caffeine, lithium dose reduction or addition of a beta-blocker such as propranolol or atenolol.
 Patients who experience diarrhea while taking slow-release lithium preparations may fare better
with immediate-release formulations,17 particularly the oral liquid citrate salt.18
 Back to Content

Please note that this is only a draft version based on several sources, including: Dr. Basel Mohasen’s lectures,
Toronto Notes, Therapeutic Choices and others. Edited and organized for the sake of all attendances of the
Canadian Osce Exams: NAC OSCE and MCCQE2. by: Dr. Merlyn D’Souza and Dr. Zeev Gross, Spring 2011. Page 221
Pt wants to discontinue his Li
st
If stop Li Relaspe 1 time: disct Rx:40 – 60 %
2nd time:80%
3rd time:>95%
Can control BP1 but not cure
1. Want s to stop Li as handwriting not like before,
Ask if any other concerns .....
Seems reasonable....
INTRO:
As I understand, you’re her cuz you’ve been diagnosed with BP1 3 yrs ago & want to disct
your Rx,during the next few minutes I will take Hx & towards the end hopefully we will
reach a working plan
Ask:
Why do you want to discontinue?
ASSES:
MOOD disorder whether Mania/Depression
Go back to mania specially when diagnosed
Li if SE
Asses:
Psychosis
Anxiety
Organic
Past Medical Hx
Fhx
Social Hx
Fhx of Depression & BP1
Suicidal 7 Homicidal ideation
Self care
COUnselling

INTRO
Can you tell me more about your decision?
Why?
Any other reasons?
These seem reasonable enough concerns & I’m glad you’re here today to talk about it
Let me ask some qns
How would you describe your mood today?
Even if he says good.....
Go through DIG FAST

Grandiosity:
Ask for opposite mood
Do U feel Low
Please note that this is only a draft version based on several sources, including: Dr. Basel Mohasen’s lectures,
Toronto Notes, Therapeutic Choices and others. Edited and organized for the sake of all attendances of the
Canadian Osce Exams: NAC OSCE and MCCQE2. by: Dr. Merlyn D’Souza and Dr. Zeev Gross, Spring 2011. Page 222
MI PASS ECG
1. How were U Dsed as BP1?
2. What was done at that time?
3. Were U hospitalized?
4. Was there serious consequences?
5. Are you under reg F/U?
6. When was the last time you saw your Dr?
Li
1. Which medications are you on besides Li?
2. How much Li?
3. Is it measure d on a regular basis?
4. What was the level?
5. Any new meds/ or increase in dose?
6. How do you feel about taking Li?
Any SE
Have U got TSH measured?
When was the last time it was measured?
Do you feel cold? Inc wt/Dec conc?
Drink more/Pee more?
Any urine analysis?
Screen for Ataxia:Any shakiness/falls/difficulty in balance?
Nx/Vx/Abd pain?
If TSh Inc ct with Thyroxine
If Di early Stop later Ct with Thaizude
GI Stop
Tremor B Blocker

Have you ever discontinued Li in past few yrs?


TRANSITON: I know you’ve been asked this qn before, but I need to ask these qns
PSYCHOSIS
ANXIETY
Past MH
Fhx of depression/Suicide/BP1
How do u support urself financially?
COUNSELLING:
Compare between Mania & Depression
What is your understanding about Mania?
Mania is a condt that affects mood,in which people feel elevated it is one of the mood
disorders it is common. Most people have depression, where people feel low & lose interest
& Rx is often Talk therapy & medications.
The Rx for Mania is lifelong,similar to DM,in which we can control Sx,but not cure it.There is
a lot of research going on & ne day we hope to find a cure for it.
If you choose to discontinue it,your chances of relapse are high upto 60%

Please note that this is only a draft version based on several sources, including: Dr. Basel Mohasen’s lectures,
Toronto Notes, Therapeutic Choices and others. Edited and organized for the sake of all attendances of the
Canadian Osce Exams: NAC OSCE and MCCQE2. by: Dr. Merlyn D’Souza and Dr. Zeev Gross, Spring 2011. Page 223
Coming to Li levels if 1.2 upper level of (N) & we can decrease the dosea bit & see how it
affects you. But you’ve to PROMISE me that at any time you spend more,sleep less etc
contact me or go to ER ASAP.
Pt may accept .
When it comes to writing Thought block is not one of the SE of Li,give it time ,& see if it
improves
If S/o depression it is the other component of BP1 & I will refer you to psy.

Therapeutic Tips

 A key feature of bipolar disorder is recurrent nonadherence to medication; including the patient
in decision-making, together with psychoeducation, promotes a strong therapeutic alliance and
enhances medication adherence.
 Patients taking lithium need to maintain their usual salt and caffeine intake and monitor fluid
intake and output, making adjustments in the event of unexpected losses due to vomiting or
diarrhea.
 During acute manic episodes, patients may exhibit increased tolerance to lithium.
 Advise patients taking antipsychotics about antipsychotic-associated body temperature
dysregulation and strategies to prevent heat stroke (e.g., hydration, sun protection).
 For lithium-associated cognitive impairment, check lithium level and thyroid function. Lowering
the dose or using a slow-release formulation may improve cognitive function.
 Patients who experience tremor while taking lithium may benefit from elimination of dietary
caffeine, lithium dose reduction or addition of a beta-blocker such as propranolol or atenolol.
 Patients who experience diarrhea while taking slow-release lithium preparations may fare better
with immediate-release formulations,17 particularly the oral liquid citrate salt.18

Back to Content

Please note that this is only a draft version based on several sources, including: Dr. Basel Mohasen’s lectures,
Toronto Notes, Therapeutic Choices and others. Edited and organized for the sake of all attendances of the
Canadian Osce Exams: NAC OSCE and MCCQE2. by: Dr. Merlyn D’Souza and Dr. Zeev Gross, Spring 2011. Page 224
21/2/2011
Introduction to physical exam:
“...
If it is after history taking. “Thank you for the information. Now I’ll do some physical exam...hopefully
towards to the end we’ll reach a working plan...”

During the physical exam – talk aloud. Verbalize everything.


Don’t fail to drape the patient.
Doing after vitals and general inspection.
Lack of empathy (Warm hands and stethoscope and try it on your arm). You warn the patient before the
exam, but also before any manoeuvre.
If there is painful area – don’t repeat it.
If the patient having

34 physical exams, and 11 management stations (ER):


Physical Exam:
1. CVS
1.1. Essential Htn for the last 30y, 65 yo for f/u 10m
1.2. 35yo dgn recently with Htn, do focused physical exam (CVS) 5m
1.3. 25yo recently HTN, relevant physical exam 5m
1.4. 60yo, Pain in calf muscles – history and physical
1.5. 35yo palpitations, history and physical. 10m
1.6. 30-40yo, Cardiac murmur. 5m.
1.7. 70yo A surgery 3d ago doesn’t pass urine 4hours – 10m (do JVP and vitals)
1.8. 50yo SOB, 3d ago had surgery, 5m focus physical exam (emboli)
1.9. Car accident 24 ago, SOB, P/E 5m (fracture, fat emboli))
1.10. SOB for the last, surgery 3d ago, 10m
1.11. History of heart failure for the last 10y, 3d ago SOB, 10m physical exam
2. RS
2.1. Cough, for the last 3d, focused p/e 5m
2.2. Female Hx breast cancer, mastectomy, chemotherapy and radiation 5y ago, a week cough or
SOB (Pulm. Fibrosis)
2.3. Coughing blood 1w, 67yo, history and p/e 10m
3. GI
3.1. Lower abdominal pain last 24hr, 22 yof, 5m focused p/e
3.2. 35 yof, came to the ER abdom. Pain 2h,
3.3. 22 yof Hx of Crohn, abdominal pain 24hr, 5m focused p/e
3.4. 30 yom abdominal pain, 5m focused
3.5. 61 yom Hx alcoholic patient vomits blood, 5m p/e
3.6. 25yof, epistaxis, bruising in skin, hematologic exam (ITP)
4. Neuro
4.1. HIV positive, headache for the last week – do cranial nerve exam (wear gloves)
4.2. 40 yo, Difficulty in vision, Hx and p/e 10m
4.3. Crooked face (Bells palsy) - hearing and than 7th nerve, 5m focused p/e
4.4. Weakness in the Rt. Or Lt. Hand – see the power, reflexes and tone, history and physical 10m
4.5. Diabetic foot – do physical exam, 5m (monofilament test – 10m exam)
4.6. Unconcious – do neurologic exam for , 5m

Please note that this is only a draft version based on several sources, including: Dr. Basel Mohasen’s lectures,
Toronto Notes, Therapeutic Choices and others. Edited and organized for the sake of all attendances of the
Canadian Osce Exams: NAC OSCE and MCCQE2. by: Dr. Merlyn D’Souza and Dr. Zeev Gross, Spring 2011. Page 225
4.7. Shakeness in his Rt. Hand (Parkinson) – 5m
4.8. Back pain
5. M/S – all joints except elbow.
5.1. Neck (level of the lesion)
5.2. Shoulder pain
5.3. Hand
5.3.1.Laceration in the wrist
5.3.2.CTS
5.4. 35 yom – Hip pain (gonorrhoea), otherwise
5.5. Knee – Osgood Schletter, Chondromalacia patella and osteoarthritis
5.6. AP cruciate ligament,
5.7. Ankle – counsel patient. There is no fracture or rupture of ligaments. 10m
5.8. Back pain
5.8.1.Acute (3d ago)
5.8.2.Acute superimposed on chronic (fracture on metastasis)
5.8.3.Chronic back pain (young – Ankylosing spondylitis, old – spinal stenosis or osteoarthritis)

Intro
Vitals

GI (I, A, Per, PS-PD, ST), DRE, Pelvic exam for females


RS (I, Pal, Per, Aus, ST)
CVS (I, Pal, A, ST), PV exam and JVP in the neck
MS (I, Pal (Temp, Tenderness, Crepitus), ROM, ST) one below and one above joint.
Neuro (I and orientation, CN, U and LE, Coord., Cortical sensation)
InsBulk/PulpTonePowerReflexesSensation
SEADS (Swelling, Erythema, Atrophy, Deformity, Scars)

Vitals: BP, HR, Temp, RR (“Based on the vital patient is stable I am going to do...)
If there are no vitals you will say “I am going to start my physical exam by measuring the vitals by taking
BP, pulse, temp...)
If there are only 3 out of the 4 parameters – “Before I proceed I’d like to know what is the
temperature...”
Weight and height in pounds and inches. (5 feet is 150cm, 6 feet is 180, 5,6 is 165cm)
General inspection: lying down comfortably, no signs of distress
Specific inspection:
SEADS for each joint (Is it OK for you to lower your gown... )
Neck – no scars, erythema, atrophy, + muscle contractions;
Normal cervical and thoracic curvatures.
Back – no SEADS. From side – normal cervical, thoracic and lumbar curvatures.
Shoulders – both shoulders are symmetrical, clavicles deltoid and scapulae are in the same level and
angle.
Hand – SEADS + thenar and hypothenar muscles.
Hip – I’d like to have full exposure. Hip joints are deeply seated joints – I am looking for any obvious
SEADS and gluteal folds on the same level, and mentioning the lumbar curvature.

Back to Content
Please note that this is only a draft version based on several sources, including: Dr. Basel Mohasen’s lectures,
Toronto Notes, Therapeutic Choices and others. Edited and organized for the sake of all attendances of the
Canadian Osce Exams: NAC OSCE and MCCQE2. by: Dr. Merlyn D’Souza and Dr. Zeev Gross, Spring 2011. Page 226
Knee
INSPECTION:
(Stand, walk and lie down).
1.
Stands up:
By inspection
1. (B)knees are symmetrical
2. (B) knee jts are normally aligned
3. No genu varus or valgus.
2. ask him to WALK: & look for:
1. Gait
2. popliteal fossa.(no bulge in popliteal fossa)
3. LIES DOWN:–
SEADS
(B) Quadriceps muscles are in the same bulk.
Ankle – SEADS, no open fracture no bruises. The last thing in the ankle is the gait.
Gait – do in every joint besides shoulder, arm and hand.

TTC (Temp, Tenderness, Crepitus)


If the patella is the same temp as the rest of the knee – there is inflam.
Both patellae are the same temp. And colder as the rest of the leg.
Quadriceps, suprapatellar pouch, patella (press and swing) – there is no signs consistent with
chondromalacia patella, along the ...press on the collateral ligament, press to the back, when bending
the knee: Up, Down, and In.
Crepitus – no crepitus.
Effusion –
Bulging sign and milky test.
Bend your knee all the way. Normal or limited flexion. Can you push against my hand and pull.
ST – cruciate ligaments, medial/lateral, and anterior posterior.
Medial and lateral collateral ligament – varus and valgus stress test.
McMeri test for the meniscus.
Examine the other knee, pulse (popliteal).
Examine above and below the knee (just mention it).
Patrick’s test (sacroiliac joint) and Thomas’ test (fist in the lumbar region – in case of osteoarthritis they
need to do flexion by increasing their lordosis and feel less pressure).

Please note that this is only a draft version based on several sources, including: Dr. Basel Mohasen’s lectures,
Toronto Notes, Therapeutic Choices and others. Edited and organized for the sake of all attendances of the
Canadian Osce Exams: NAC OSCE and MCCQE2. by: Dr. Merlyn D’Souza and Dr. Zeev Gross, Spring 2011. Page 227
KNEE
Intro: Good afternoon Mr XXX as I understand you’ve a pain in your (R) knee for the next few
minutes I will be examining you,& if you feel any pain please let me know.
I will also be reporting my findings to the examiner.
Is that Okay with you?
Can I proceed?
Can I get the vitals please?
On General examination:
Pt sitting comfortably in no obvious distress
Mr xxxx Can you please stand up?
Can you please hold up your gown?
By inspection
1. (B)knees are symmetrical
2. (B) knee jts are normally aligned
3. No genu varus or valgus.
Can you please WALK: & look for:
1. Gait
2. popliteal fossa.(no bulge in popliteal fossa)
Thank you,
Could you please turn around walk back & lie down,
I’m going to drape you
LIES DOWN:–
O/Inspection:
1. SEADS
2. (B) quadriceps muscles are in the same bulk
I’m Going to feel your knee
PALPATION:
TTC (Temp, Tenderness, Crepitus)
1. Both patellae are the same temp & colder as the rest of the knee.
2. (B) Knees are symmetrical & there is no increase in temperature
3. (B)Quadriceps are normal in bulk
4. Suprapatellar pouch (N)
5. Patella (press and swing) – there is no signs consistent with chondromalacia patella,
6. Go along patellar tendon & end on Tibial tuberosity
7. No tenderness of T Tuberosity
8. press on the Medial collateral ligament,
9. press on the lateral collateral ligament
10. press to the back, for pop[liteal fossa
Bend the knee: Open joint fully
11. Up for Femoral condyles
12. Down for ,Tibial condyles
13. In for the lateral & medial meniscus

Please note that this is only a draft version based on several sources, including: Dr. Basel Mohasen’s lectures,
Toronto Notes, Therapeutic Choices and others. Edited and organized for the sake of all attendances of the
Canadian Osce Exams: NAC OSCE and MCCQE2. by: Dr. Merlyn D’Souza and Dr. Zeev Gross, Spring 2011. Page 228
Relax your knee,I’m going to move your knee & examine for crepitus
EFFUSION:
 Eliminate the suprapatellar pouch
 Press on patella ------->Patellar tapping ------>Bounce = Fluid
 No bounce on patellar tapping
 Bulging sign
 Milking Sign
ROM;
Can you please bend your knee all the way
(N) flexion Full flexion & extension
POWER:
I need to examine the stability of the knee:
3 tests:
1. Medial & lateral collateral lig Varus & Valgus stress test
2. Cx ligament: Ap drawer test
3. Meniscus : Mcmurray test
Examine other knee
POpliteal pulse
Dorsalis pedis pulse
I would like to examine one Jt below & one jt above
Ankle joint & back

Back to Content

Please note that this is only a draft version based on several sources, including: Dr. Basel Mohasen’s lectures,
Toronto Notes, Therapeutic Choices and others. Edited and organized for the sake of all attendances of the
Canadian Osce Exams: NAC OSCE and MCCQE2. by: Dr. Merlyn D’Souza and Dr. Zeev Gross, Spring 2011. Page 229
HIP JOINT
Intro:
Vitals
G/E:
Would you please stand up?
Do you need help?
Turn to (L) side------ go to back
Ask examiner:
Can I have full exposure?
Can you please Roll up shirt,I’m going to look at your hip
INSPECTION
The hip is a deeply seated joint,however I’m looking for SEADS
(B) hips are symmetrical
(B) gluteal folds are same level
Lumbar curvature Normal
PALPATION:
I’m going to feel your joint,plz inform me if you’ve pain:
1. SI jt (N)
2. Post superior Iliac spine (N)
3. Iliac crest (N)
4. Ant superior iliac spine (N)
5. Greater trochanter (N)
Plz walk to the wall,do you need help?
Gait (N)
No limping
Can you please turn & come back?
When standing look for EXTENSION
Trendelenbergh test
Can you please lie down
DRAPE
I would like to continue my inspection anteriorly
SEADS
PALPATION:
1. Along inguinal ligament
2. Head of Femur
3. Symphysis Pubis
Examiner will say (N)
(Inspection & palpation done)
ROM
1. Extension done when standing
2. Bend knee to abdomen as much as you can (flexion)
3. Abduction & adduction
4. Passive & active

Please note that this is only a draft version based on several sources, including: Dr. Basel Mohasen’s lectures,
Toronto Notes, Therapeutic Choices and others. Edited and organized for the sake of all attendances of the
Canadian Osce Exams: NAC OSCE and MCCQE2. by: Dr. Merlyn D’Souza and Dr. Zeev Gross, Spring 2011. Page 230
length
Discrepancy in true length Hip lesion

Patrick test
Thomas test
Sensory fn
Knee joint & Lumbar joint

Back to Content

Please note that this is only a draft version based on several sources, including: Dr. Basel Mohasen’s lectures,
Toronto Notes, Therapeutic Choices and others. Edited and organized for the sake of all attendances of the
Canadian Osce Exams: NAC OSCE and MCCQE2. by: Dr. Merlyn D’Souza and Dr. Zeev Gross, Spring 2011. Page 231
SHOULDER
Frozen shoulder – active and passive are limited.
Rotator cuff – four muscles.
 Complete tear (initiation of abduction is lost 1st 30 -60) swing the hand or tilting and
doing flexion and abduction. Cannot initiate and has painful arm and dropping.(DROP
ARM)
 Partial tear or tendinitis or impingement with same presentation (u/s or MRI can help to
differentiate between them). Painful arch – can move, but it may ease him to turn the
hand in supination. The empty can test – his arms fall.
 Anterior dislocation – apprehension test positive. For posterior dislocation – push the
elbow backward.
 Bicepts tendinitis – supination and flexion (Jargonson test).
Flexing against resistance (Job’s test).
 Infraspinatus and teres minor – external rotation against resistence. Internal rotation for
subscapularis (lift-off test).

INTRO:
Is it Ok to untie your gown & is it Okay to kep it in your lap?
INSPECTION:
1. (B) shoulders are symmetrical
2. (B) Deltoids are symmetrical
3. (B) clavicles are at same angle
4. (B) Scapulae are at same level
5. No SEADS
PALPATION:
I’m going to feel your joint,
1. Temp (N)
2. I’m going to press your joint
3. Sternal notch NT
4. (B) Sterno clavicular joint NT
5. (B) Clavicles NT
6. Acromio clavicular jts NT
I’m going to focus on (R) shoulder & then (L) shoulder
Press on:
1. Acromian
2. Spine of scapula till medial aspect of scapula
3. Tip of scapula
4. Spinal process of neck
5. Insertion of Supraspinatous NT (Greater Tuberosity)
6. Glenohumeral joint NT
Sulcus Sign _ve (Pull down on shoulder)
CREPITUS

Please note that this is only a draft version based on several sources, including: Dr. Basel Mohasen’s lectures,
Toronto Notes, Therapeutic Choices and others. Edited and organized for the sake of all attendances of the
Canadian Osce Exams: NAC OSCE and MCCQE2. by: Dr. Merlyn D’Souza and Dr. Zeev Gross, Spring 2011. Page 232
Relax I’m going to move your shoulder & feel the movts
MOVE TO NECK & examine neck
To ENSURE that shoulder pain not related to neck pain
ROM:
Please put your gown back stand up & face me
Would you mind copying me
Full flexion & extension
Push back (extension)
Int rotation
Cross arms--- Adduction
Move to sides all the way up to the head------ Abduction Ct moving down. Hold below (No
painful arc)
No drop arm
SPECIAL TESTS:

POWER
PULSE

Back to Content

Please note that this is only a draft version based on several sources, including: Dr. Basel Mohasen’s lectures,
Toronto Notes, Therapeutic Choices and others. Edited and organized for the sake of all attendances of the
Canadian Osce Exams: NAC OSCE and MCCQE2. by: Dr. Merlyn D’Souza and Dr. Zeev Gross, Spring 2011. Page 233
Chronic back x 6m0
INTRO:
VITALS
G/E
Can you please stand up?
CAN YOU PLZ UNTIE YOUR GOWN?
INSPECTION:
st
If Hx,:1 inspection of face
1. Eyes for rednes
2. Mouth for ulcers
3. Nails: No pitting/ulcers/or skin changes
Look at back
Curvature ---Side:Normal cervical,Thoracic & Lumbar curvatures
Back: No Scoliosis
SEADS
PALPATION:
Warm hands & tell Pt:
I’m GOING TO FEEL YOUR BACK, tell me if you feel pain
Feel temp
Press Spinous processes individually Identify C7
Iliac crest: L 4-5
Press; Para vertebral muscles
SI Jts
TIE THE GOWN BACK & Ask Pt to lie down DRAPE
I’d like you to do some movts for me:
ROM
1. Can you touch your toes with your fingers without bending knees?
2. Arch your back backwards without bending your knees
3. Can you slide your arm along your thigh as low as you can?
4. Can you cross your arms & rotate the shoulder (Fix the hip)
“Because I noticed you have restricted ROM of movements in all directions
I’ll do a test called Shubert test. I will draw some lines on your back which are washable
Dimples of venus – sacroiliac joints for line A
10 cm above ----- Line B
5 cm below ------Line C
Try & touch toes without bending back
the difference from line B it should be at least 15cm. Less than 15cm – it is restricted.
The 5cm below is for control
WALK to wall:
Gait (N) No limping

Please note that this is only a draft version based on several sources, including: Dr. Basel Mohasen’s lectures,
Toronto Notes, Therapeutic Choices and others. Edited and organized for the sake of all attendances of the
Canadian Osce Exams: NAC OSCE and MCCQE2. by: Dr. Merlyn D’Souza and Dr. Zeev Gross, Spring 2011. Page 234
Stand against wall
Occipital from wall test.(When there is Shubert test positive )
Stand On toes – S1,
Stand on heels L5.
Pitting changes in the nails, psoriatic changes.
CAN You please lie down? DRAPE
SLR
Patrick test
Listen to his heart for Aortic Insuff.
Chest expansion – measurement in max inspiration and expiration (changes should be more
than 5cm).

ACUTE BACK PAIN

Intro:
Always ask Pt if he prefers to lie down or stand
OCD:
ONSET:
What were you doing at that time?
Did you lift heavier than usual?
Did you hear a snapping sound? Did you have to stop what you were doing?
C
PQRST:
R: Does it move to the leg?,reach toe or thigh?
Which bothers you more,The Leg or Back?
EMPATHY...........
Did you try any pain killer?
A &A:
Lying down?
Stretching?
Bend/Move?
U:
V;
ASx:
1. Weakness
2. Numbness
3. Tingling
4. Loss of balance & falls
5. Do you need to drag your foot?
6. How about Urine & Bowel symptoms: Some patients with similar condition may soil
underwear
7. Numbness in buttock area?
Please note that this is only a draft version based on several sources, including: Dr. Basel Mohasen’s lectures,
Toronto Notes, Therapeutic Choices and others. Edited and organized for the sake of all attendances of the
Canadian Osce Exams: NAC OSCE and MCCQE2. by: Dr. Merlyn D’Souza and Dr. Zeev Gross, Spring 2011. Page 235
8. H/O trauma to back?
9. Urinary: Dysuria/Flsnk pain?

SOCIAL Hx:
Smoke
Alcohol
Drugs: ......Particularly IV drug use

PE:
G/E:
Vitals please
Can you please turn to side (so examiner can see)
Can you please untie your gown?
Dorso lumbar spine looks (N) curvature
From Back: No scoliosis
SI Joints appear (N)
Tie gown
ROM:
Forward flexion & extension
WALK to bed & wall (Make sure that Pt does not FALL!!)
Walk on heels & toes (support Pt)
I’m going to raise your leg, please lie down & if it causes pain please let me know
Can you please lie down?
SLR
SENSORY:
Start with Little toe:S1
1st Web:L5 (common peroneal nerve)
Medial malleolus:L4
Knee:L3
Mid thigh:L2
REFLEXES:
Knee
Ankle
Clonus
Babinski
DRE
Femoral stretch test
End with Dorsalis pedis PULSES

Please note that this is only a draft version based on several sources, including: Dr. Basel Mohasen’s lectures,
Toronto Notes, Therapeutic Choices and others. Edited and organized for the sake of all attendances of the
Canadian Osce Exams: NAC OSCE and MCCQE2. by: Dr. Merlyn D’Souza and Dr. Zeev Gross, Spring 2011. Page 236
Red Flag” Symptoms/Signs in Assessment of Low
Back Pain
Condition Symptoms/Signs Investigations
Herniated Nucleus Pulposus Positive SLR (leg pain at < 60°); MRI of lumbar spine
weak dorsiflexion of ankle (L4-5) or
great toe (L5-S1 or L4-5); reduced
ankle reflex (L5-S1); reduced light
touch in L4, L5 or S1 dermatomes
of foot/leg1

Cancer Age > 50; previous cancer history; Positive laboratory tests
unexplained weight loss; failure to (including elevated ESR,
improve after 1 mo therapy2 reduced hematocrit) 2 and
imaging showing erosion
or blastic lesions
Spinal Osteomyelitis Intravenous drug abuse; sources of Positive laboratory tests
infection (e.g., skin, teeth, urinary and imaging
tract, or indwelling catheter); fever;
vertebral tenderness3

Spinal Age > 50, female gender, major Positive laboratory tests
Fracture/Compression trauma, pain and tenderness, and a including plain x-rays
Fracture distracting painful injury;4 also
consider a history of osteoporosis or
corticosteroid use

Cauda Equina Syndrome Acute urinary retention or overflow Emergency laboratory


incontinence; loss of anal sphincter assessment and imaging
tone/fecal incontinence; perineal
numbness; change in sexual
function; weakness of legs1

Please note that this is only a draft version based on several sources, including: Dr. Basel Mohasen’s lectures,
Toronto Notes, Therapeutic Choices and others. Edited and organized for the sake of all attendances of the
Canadian Osce Exams: NAC OSCE and MCCQE2. by: Dr. Merlyn D’Souza and Dr. Zeev Gross, Spring 2011. Page 237
Factors Adversely Affecting Prognosis of Low Back Pain
Psychosocial Factors Mental Status Indicators of
Significant Anxiety or Depression
1. Duration of work absence 1. Insomnia or nightmares
2. High levels of self-reported functional 2. Irritability
disability 3. Withdrawal
3. Self-report of extreme pain and constant 4. Panic episodes or anxiety
pain in multiple body areas during the day or night
4. History of prolonged sick-listing after 5. Persistent tearfulness
previous injuries 6. Poor concentration
5. Prior history of absenteeism 7. Inability to enjoy (anhedonia)
6. Delays/obstacles in work re-entry 8. Poor appetite/weight loss
process 9. Poor libido
7. Patients who believe that they will 10. Thoughts that ―life is not worth
never return to work living‖
8. Adversarial attitude toward employer
9. Long-standing history of psychiatric
distress or maladjustment

Please note that this is only a draft version based on several sources, including: Dr. Basel Mohasen’s lectures,
Toronto Notes, Therapeutic Choices and others. Edited and organized for the sake of all attendances of the
Canadian Osce Exams: NAC OSCE and MCCQE2. by: Dr. Merlyn D’Souza and Dr. Zeev Gross, Spring 2011. Page 238
Back to Content

Please note that this is only a draft version based on several sources, including: Dr. Basel Mohasen’s lectures,
Toronto Notes, Therapeutic Choices and others. Edited and organized for the sake of all attendances of the
Canadian Osce Exams: NAC OSCE and MCCQE2. by: Dr. Merlyn D’Souza and Dr. Zeev Gross, Spring 2011. Page 239
NECK EXAM
INTRO:
Vitals Pt stable
G/E:
INSPECTION:
I’d like to take a look at your back, can you please untie your gown?
(N) Cxal curvature----
Look from side look from back
SEADS
PALPATION:
I’m going to feel
1. (N) Temp
2. Press along individual spinous proceses (C1 to C7)
3. P Vertebral muscles
4. Trapezius
5. Sternocleido mastoids
6. Mastoid process
7. LN
8. Thyroid (ask the patient to swallow)
ROM: I’m going to examine ROM
Copy me,
1. Touch chest to chin-----> Flexion
2. Look at ceiling -------> Extension
3. Turn to R/L Rotation
4. Touch shoulder to ear ---- R & L Lateral flexion
5. Check Streno Cleido mastoid by pressing against my hand & push to back (? Not done!)
6. Neck pain not associated with muscle spasm
7. Can you cough? ------ “No neck pain with Valsava’s manoeuvre”
Part of my exam is to check your UppExt:
Can you roll up your sleeves?
INSPECTION:
1. (B) U extremities are symmetrical
2. No abnormal posture or contracture
3. Bulk is symmetrical
PALPATION:
See & feel deltoids, biceps, Triceps, forearm, Thenar & hypothenar muscles
TONE:
WRIST:
No cogwheel rigidity
Elbow
No Pb pipe
Please note that this is only a draft version based on several sources, including: Dr. Basel Mohasen’s lectures,
Toronto Notes, Therapeutic Choices and others. Edited and organized for the sake of all attendances of the
Canadian Osce Exams: NAC OSCE and MCCQE2. by: Dr. Merlyn D’Souza and Dr. Zeev Gross, Spring 2011. Page 240
No clasp knife rigidity
SENSORY:
C6 C7 C8 C4 ------
REFLEXES
POWER:
In U/E Deltoid
Biceps
Fan fingers
Power of thumb
SPECIAL TEST: Spurling test
Ask Pt to stand: Check Clonus & gait
CNErve exam

Back to Content

Please note that this is only a draft version based on several sources, including: Dr. Basel Mohasen’s lectures,
Toronto Notes, Therapeutic Choices and others. Edited and organized for the sake of all attendances of the
Canadian Osce Exams: NAC OSCE and MCCQE2. by: Dr. Merlyn D’Souza and Dr. Zeev Gross, Spring 2011. Page 241
HAND
Laceration
Hx:
1. AMPLE + Tetanus
2. Mood
3. Handedness (occupation : can affect if Pianist, Speech therapist, Plastic surgeon)
4. X ray
5. 5.
6. Irrigate with NS
7. Antibiotic prophylaxis
8. NPO
INTRO:: As pt has an injury,I would like to get gloves for protection
Greet Pt & ask for vitals
G/E;
Remove bandage & describe the wound:
Position: wound on palmar aspect: 3 cm in length/2mm width/depth cannot be
assessed
5-10 cm proximal to wrist on Volar aspect
No active bleeding/No oozing/Margins clear & not elevated
(B) hands are symmetrical
SEADS
Colour similar
I’m going to FEEL:
Temp (B) hands is normal
(N) Capillary refill
I’m going to feel your hands to see if there is damage to the arteries (N) radial artery &
ulnar
SENSATIONS: Lt touch Ulnar/Median/Radius
Tenderness to PALPATION:
Distal radius/Styloid process/distal part of ulna & styloid process/base of thumb
Press carpal bones & metacarpal bones
ROM: try to do on table & not move elbow
Ulnar deviation
Radial deviation
MPOTOR FN OF MEDIAN N: OK Sign
Ulnar N :Able to hold peice of paper betn Adducted finger & resist pulling

Please note that this is only a draft version based on several sources, including: Dr. Basel Mohasen’s lectures,
Toronto Notes, Therapeutic Choices and others. Edited and organized for the sake of all attendances of the
Canadian Osce Exams: NAC OSCE and MCCQE2. by: Dr. Merlyn D’Souza and Dr. Zeev Gross, Spring 2011. Page 242
RADIAL N: Extend thumb Thumbs up
THUMB: Make a fist & fan out fingers
Can you touch ........your thumb to the tip of your little finger? (flexion)
Take it all the way to other side ? (extension)
Point to ceiling (Abduction)
Put close to your hand?(Adduction)
Touch thumb to tips of fingers? (opposition)
FLEXION:
Can you bend your fingers one by one?----- Flexor digitorum profondus
Flexor Doigitorum superficilias
Back to Content

Please note that this is only a draft version based on several sources, including: Dr. Basel Mohasen’s lectures,
Toronto Notes, Therapeutic Choices and others. Edited and organized for the sake of all attendances of the
Canadian Osce Exams: NAC OSCE and MCCQE2. by: Dr. Merlyn D’Souza and Dr. Zeev Gross, Spring 2011. Page 243
CTS
Pain in (R) wristy x 2 wks: Hx & PE
OCD PRTY UV A&A ASx CSx
D/d:
1. CTS
2. Spinal stenosis/OA/Cervical disc herniation
3. TIA
4. Thoracic outlet syndrome
OCD:
O:
C:
How often?
Daily?
Since when daily?
Before that?
At Night?
D:
How long each attack?
What brings these attacks?
What relieves it?
What do you do for a living?
PQRST:
P: Can you show me where it is?
Q;
S:
U;
V:
A & A:
Movts/Medications/Repeated movts
Local Sx: Swelling/Numbness/Weakness/Other hand/Leg/Bladder & bowel disturbances
CSx:
AETIO:
I’ve to ask you qns as to the presence of any condt that might have caused this:
1. Hx & Sx of DM:
2. Hx & Sx of Hypothyroid
3. Hx & Sx of Acromegaly
4. Trauma
5. Fall
6. HX of RA
D/d:

Please note that this is only a draft version based on several sources, including: Dr. Basel Mohasen’s lectures,
Toronto Notes, Therapeutic Choices and others. Edited and organized for the sake of all attendances of the
Canadian Osce Exams: NAC OSCE and MCCQE2. by: Dr. Merlyn D’Souza and Dr. Zeev Gross, Spring 2011. Page 244
Neck pain
Past MH: HTN/any long term disease
Social Hx;SAD
Fhx:
Thank You for this information,I will now proceed to the PE
GE:
INSPECTION:
1. (B) hands are symmetrical
2. No SEADS
3. No Bouchardfs nodes
4. No Swan neck deformity
5. (B) Thenar & Hypothenar muscles equal bulk
FEEL:
1. Temp & capillary refill
2. Palpate distal part of radius of hand
3. Bulk of thenar & hypothenar muscles
ROM of wrist
THUMB:
Power check against resistance,pu;ll up & down & pull
Hook thumb …..
BICEPS: Check Power & Reflex
NECK: ROM to R/o C6
SENSORY:
1. Little finger :Ulnar
2. Ring Finger:Ulnar aspect for Ulnar nerve & radialaspect to R/o median nerve
3. 2 POINT DISCRIMINATION: Only in Index finger
SPECIAL TESTS:
1. Tinel’s Tap at medial aspect of wrist x3 times Ask if feels numb
2. Phalen’s sign
PULSE: Radial
T

Back to Content

Please note that this is only a draft version based on several sources, including: Dr. Basel Mohasen’s lectures,
Toronto Notes, Therapeutic Choices and others. Edited and organized for the sake of all attendances of the
Canadian Osce Exams: NAC OSCE and MCCQE2. by: Dr. Merlyn D’Souza and Dr. Zeev Gross, Spring 2011. Page 245
TREMORS
PARKINSONISM

INTRO:
Vitals:
G/E:
Pt is sitting There is an obvious tremor in (R) hand
(N) elbow
No tremor in shoulder
Ask patient to count from10 to 1 backwards: & observe the tremor....
 Tremor does not disappear on mental activity but increases, which is consistent with
Parkinsonism, & R/O Anxiety related tremor
Please extend arms & fingers:
 No fine tremors R/O Thyroid disease
 No flapping tremors R/O Liver disease
 Can you touch Finger to my finger & then to your nose? No intention tremor R/O
Cerebellar disease
There is no dysdiadokinesia
Pt has a limited facial expression
Limited eye blinking
No drooling

INSPECTION:
Tremors in (R) hand, which are pill rolling & involve the (R) arm
Pt does not have tremors in (L) hand, arm & shoulder
NO head nodding
I want to examine the TONE:
 Cog wheel
 Pb pipe
 Clasp knife
Ask pt to please stand
There is difficulty in initiating movt
Stooped posture
Decreased arm span
Festinant gait
Turns in block
Ask Pt to say: British Constitution
(N) articulation
Ask Pt to write; Micrographia
I want to check for orthostatic hypotension
Difficulty in rapid alternating movts:
Please note that this is only a draft version based on several sources, including: Dr. Basel Mohasen’s lectures,
Toronto Notes, Therapeutic Choices and others. Edited and organized for the sake of all attendances of the
Canadian Osce Exams: NAC OSCE and MCCQE2. by: Dr. Merlyn D’Souza and Dr. Zeev Gross, Spring 2011. Page 246
1.
2.
3.
I would like to arrange for a MMSE which can happen later.

Back to Content

Please note that this is only a draft version based on several sources, including: Dr. Basel Mohasen’s lectures,
Toronto Notes, Therapeutic Choices and others. Edited and organized for the sake of all attendances of the
Canadian Osce Exams: NAC OSCE and MCCQE2. by: Dr. Merlyn D’Souza and Dr. Zeev Gross, Spring 2011. Page 247
HIV Pt with HA/PE (Cranial nerve exam)
st
1 nerve. (Coffee and ammonia). I’m going to skip the first nerve. I’ll ask the patient if he has
any difficulty smelling.
2nd nerve.: OPTIC NERVE (5 tests):
ACUITY:
Ask Pt for best vision or if he wears EYE GLASSES
Hold Snellen’s chart with (R) hand & cover Lt. eye. Choose a mid-line, jump two lines below,
and finally last line.
COLOUR VISION:,
then the other eye, change eyes, ask colour first in a reversed order and if he sees in the same
intensity. Go straight to last line and ask to read backwards.
VISUAL FIELDS:
(DDx one eye blindness, bitemporal and homonymous hemianopia)
PUPILLARY REACTION:
I am going to shine the light in your eyes it might might bother you:
first shine at the (R) Look at the (R). Eye,
second shine in rt. Look at left side, 2 shine light in eyes & see pupillary reaction
: 2-3nerve (2- afferent, 3-efferent)
FUNDOSCOPY:
verbalize (DM: microaneurysms, cotton wool spots, neovascularisation; Htn: flame
hemorrhage, disc edema, nipping of veins)

3rd, 4th and 6th


I am going to examine the nerves which cause movement of the eyes.
INSPECTION:
(B) eyes are symmetrical,
No deviation.
No head tilting (4th nerve),
No ptosis (3rd nerve),
No nystagmus.
Tell patient to look at the tip of pen and follow with eyes and when you see double vision please tell me.
Start from middle and create an H. Then go to centre and check conversion.
“Normal extraoccular vision, no limitation, no nystagmus.”

5th nerve: motor and then sensory.


INSPECTION:
No atrophy in temporal and masseter area.
I am going to examineSENSORY:. This is a piece of cotton, I’ll put in on your chest – this is how it feels.
Now I am going to touch different parts in your face while your eyes are closed. Whenever you feel it
touches you tell me. Then ask him if it is the same feeling.
MOTOR:
clench teeth and relax twice. Feel the bulk of the temporal and masseter – they should be similar bulk.
Can you push your jaw against my hand?

Please note that this is only a draft version based on several sources, including: Dr. Basel Mohasen’s lectures,
Toronto Notes, Therapeutic Choices and others. Edited and organized for the sake of all attendances of the
Canadian Osce Exams: NAC OSCE and MCCQE2. by: Dr. Merlyn D’Souza and Dr. Zeev Gross, Spring 2011. Page 248
7th is mostly motor. Sensory for the tongue (anterior 2/3). Corneal reflex efferent limb.
INSPECTION:
Face symmetrical,
Normal nasoliable fold,
No drooling,
No deviation of angle of mouth.
Now copy me: raise your eyebrows, frown, close your eyes and don’t let me open them, puff cheeks and
don’t let me blow out, show your teeth, and whistle.
I would like to check corneal reflex in the eye

8th nerve.
I am going to whisper words in your ear. Repeat after me (“horse” and “house”).

9th and 10th


1. Normal voice, no hoarseness.
2. Swallow for me – swallowing is normal.
3. Say AHAA – soft palate symmetrical, uvula is central.
4. Gag reflex I’d like to do.

11th nerve – shrug your shoulders (“Normal trapezius”).


Turn your head to the right against resistant and feel the bulk of the sternocleidomastoid.

12th nerve – no atrophy/ fasciculation of the tongue no deviation of the tongue. Wiggle your
tongue left and right.

Back to Content

Please note that this is only a draft version based on several sources, including: Dr. Basel Mohasen’s lectures,
Toronto Notes, Therapeutic Choices and others. Edited and organized for the sake of all attendances of the
Canadian Osce Exams: NAC OSCE and MCCQE2. by: Dr. Merlyn D’Souza and Dr. Zeev Gross, Spring 2011. Page 249
Unconcious Patient
INTRO: Hello, Mr….DO you hear me. If you hear me open your eyes. I am Dr. … one the
physicians working in the clinic.
FIRST CHECK PUPILS:
Pupillary reaction – pupils are round and symmetrical and reactive. Not dilated or constricted.
There is no pupillary discrepancy. If one is reacting & the other not reacting – call neuro.
If you hear me – can you move your eyes up and down? “There is no locked-in syndrome”.

Vitals (Cushing triad absent; If the patient has fever we will verbalize it).

GCS

Cranial nerve:
1. 2-fundoscopy,
2. 2-3 – pupillary reflexion,
3. 3-4-6 – eye deviation,
4. 5-7 – corneal reflex,
5. 7 – inspection of face symmetry,
6. 9-10 – gag reflex

Upper extremity:
Inspection (symmetrical, normal position, no movement, no contractures), tone, reflexes
(biceps, triceps, brachioradialis)

Lower extremity:
Inspection (symmetrical, normal position, no movement, no contractures), tone, reflexes (knee,
ankle, Babinski)

Meningeal signs
Neck stiffness,
Kernigs,
Brudinsky
Babinsky

Special test:
Caloric test, Dolls eyes

Back to Content

Please note that this is only a draft version based on several sources, including: Dr. Basel Mohasen’s lectures,
Toronto Notes, Therapeutic Choices and others. Edited and organized for the sake of all attendances of the
Canadian Osce Exams: NAC OSCE and MCCQE2. by: Dr. Merlyn D’Souza and Dr. Zeev Gross, Spring 2011. Page 250
ER

Trauma Non-trauma
Hx + Transition ABCD
A History of present illness
B CC
C PQRST (Head to toe)
D ASx
AMPLE Α
Head to Toe PMHx (Risk Fcts)
Focused P/E
Orders Orders
In ER don’t be comfortable till after primary survey and IV lines.
If non-trauma patient in ER – you do primary survey (shortened), more time on history and
focus on CC.

Case of Trauma
I’d like to initiate ATLS protocol and I’d like protection to my team and myself (gown, goggles,
mask, and gloves).
When walk to patient ask the nurse:
How is the patient doing? What was done till now?
If not wearing collar – tell nurse to fix the head, tell patient not to move “we need to fix your
neck collar for your neck”.
Take a small history: “how do you feel right now?” (to see if he can talk).
If complains of severe pain (empathy: I can see you are in pain, please bear me with me for a
few minutes, as soon as I can I will give you a pain killer. At the moment I want to make sure
you are stable for that reason, I am going to give some orders to the nurse, and as soon as I am
done I’ll ask you more questions).
A - Airway
Please open your mouth. Mouth clear, the
Flip your tongue: “there are no clots, foreign bodies, broken teeth, and
Patient is talking to me – that mean airways are patent.
B
Please note that this is only a draft version based on several sources, including: Dr. Basel Mohasen’s lectures,
Toronto Notes, Therapeutic Choices and others. Edited and organized for the sake of all attendances of the
Canadian Osce Exams: NAC OSCE and MCCQE2. by: Dr. Merlyn D’Souza and Dr. Zeev Gross, Spring 2011. Page 251
Can I get the saturation?”
Give oxygen. If saturation is 95% than you give oxygen – ask if the saturation improved.
Oxygen + saturation is a part of B.
Open the neck collar and look for trachea deviation, Jugular vein.
Trachea J Veins Air Entry Heart Sounds
Normal Central - Bilateral S1, S2
Tension Pneum. Opposite side Increased Decreased same S1, S2
side
Hemothorax Opposite side Low/Normal Decreased same S1, S2
side
Cardiac Central Increase Bilateral Muffled
Tamponade

Pneumothorax – large bore needle in 2nd intercostals, midclavicular line, upper margin of the
third rib.
Hemothorax – insert chest tube in 5th intercostals space mid-axillary line. Ask nurse “how much
blood” (If > 1.5litre – ask for thoracic surgeon, also if greater than 800cc in 4 hours). If less –
monitor.
Cardiac tamponade – ask for thoracic surgeon.
Pericardiocentesis – needle in mid-xyphoid 45 degrees towards the tip of the scapula and look
for blood. Continuous ECG.

C
Vitals (every 10 minutes), and blood orders.
“I’d like to get the vitals.
Comment if hypotensive tachycardia.
I’d like to start 2 IV lines 16G in both anti-cubital fossa.
2 litre bolus Ringer lactate in one side, and from the other take blood.
If no improvement after 2 litres – give another bolus. If deterioration in vitals – give blood.
Finger prick glucose;
BLOOD for:
 CBC, Lytes, Group, Cross match,
 INR, PTT, LFT, BUN, Cr,
 Toxoc screen, Alcohol level.
If unable to get the IV line – insert intraosseous line (IO).
 Order 6 units of blood: 2 O positive for male or negative for female in reproductive age
and add 4 units of cross matched blood (pRBC).
 Continuous ECG±cardiac enzymes (troponine, cpk-mb),
Ask the change in vitals again & results of blood glucose.
Ask if 2litres were given. If stable – OK.
If UNSTABLE:– look for source of bleeding.
Start with
ABDOMEN:
Please note that this is only a draft version based on several sources, including: Dr. Basel Mohasen’s lectures,
Toronto Notes, Therapeutic Choices and others. Edited and organized for the sake of all attendances of the
Canadian Osce Exams: NAC OSCE and MCCQE2. by: Dr. Merlyn D’Souza and Dr. Zeev Gross, Spring 2011. Page 252
INSPECTION: listen, and palpation.
If bruising – ask for surgical consult stat.
If not available ask for FAST.
If technician is not available – then DPL (Diagnostic Peritoneal Lavage).
Then go for the
PELVIS:– I am going to press on hips to see if there is any pain. If complain of pain tell “I suspect
pelvic fracture” Wrap sheet and call ortho stat.
Look at the LOWER EXTREMITY:
No internal/external rotation, feel there is any pain, difference in the legs. If you suspect
fracture ask for Thomas splint and check the pulse before and after.
Log roll – check for spinal process and DRE.
D
D1- Deficit – Gross Neurological:
Shine light to both eyes “Both pupils normal size reacting to light.
Can you squeeze my finger, wiggle your toes.
Touch his sides of both upper and lower limbs – can you feel my touch.
“Patient is grossly neurologically intact.”
If unconscious – check papillary reaction and assess GCS.
D2 – universal antidote
Thiamine, Glucose,& Naloxone.
D3 – specific antidote.

AMPLE
Allergy
Medication
PMHx
Last meal, Last tetanus shot, LMP (if female)
Event – describe the event (Rear end, T bone, Head on);
Were you driver/passenger/alone?
Have you had any head trauma? Do you remember anything before or after the event?
Do you have nausea/vomiting/headache.

Head-Toe examination

Orders

Back to Content

Please note that this is only a draft version based on several sources, including: Dr. Basel Mohasen’s lectures,
Toronto Notes, Therapeutic Choices and others. Edited and organized for the sake of all attendances of the
Canadian Osce Exams: NAC OSCE and MCCQE2. by: Dr. Merlyn D’Souza and Dr. Zeev Gross, Spring 2011. Page 253
Hypertension/Secondary
As I understand you came here today because you were diagnosed with increased blood
pressure. I’ll do a physical exam on you.
Can I get the VITALS:please?
1. Patient have (B)systolic & diastolic blood pressure raised.
2. Patient does not have tachycardia, r/o pheochromocytoma and thyrotoxicosis.
3. Patient does not have bradycardia – r/o hypothyroidism.
4. I’d like to compare BP in upper and lower extremity to r/o coarctation of aorta.
5. I’d like to r/o orthostatic hypotension for pheochromocytoma.
6. Check orientation: Time, Place, and Person
On general examination:
Patient sitting comfortably
 No sign of truncal obesity
 No cervical fat pad.
 Face is symmetrical
 No moon like face
EYES:
 Normal eye brow,
 No puffiness around the eyes
 No exophthalmus,
 Please Follow my finger – there is no lid-lag or lid retraction.
 Sclera for anemia or pallor.
 No xanthelasma or arcus senilis
 On fundoscopy there are no signs of Htn.
 No loss of visual fields (acromegally).
NOSE:
 Nose OK (septal perforation in cocaine abuse).
HANDS:
 Symmetrical,
 skin normal not dry or moist, no sign of drug abuse (needle puncture).
 Normal capillary refill,
 No clubbing
 No nicotine staining.
 Please stretch your hands – no fine tremor.
 Pulse – regular, normal volume and contour.
 Compare both pulses. (When lies down – take radio-femoral delay.)
 Abduct shoulders to check proximal weakness for Cushings.
NECK:
 feel thyroid, swallow,
 ask patient to lie down, put bed at 45 degrees and ask for JVP.
 Check for carotid bruit (first listen than palpate)

Please note that this is only a draft version based on several sources, including: Dr. Basel Mohasen’s lectures,
Toronto Notes, Therapeutic Choices and others. Edited and organized for the sake of all attendances of the
Canadian Osce Exams: NAC OSCE and MCCQE2. by: Dr. Merlyn D’Souza and Dr. Zeev Gross, Spring 2011. Page 254
BACK:
 Listen between scapula for collateral circulation and bruit (COA)
 Base of lungs for creps and heart failure.
 Press on sacrum and ankle for edema.
CHEST: (lies down, please lower your gown)
 chest is symmetrical.
 No obvious pulsation.
 I am going to feel.
 Feel for apex beat, fine and identified PMI position and size, not enlarged not displaced,
not sustained.
 No parasternal heave.
 Listen to mitral area – normal S1, S2
 Move to bell and lie on side: no S3 and S4
ABDOMEN:
 abdomen non-distended, symmetrical, no pulsation, no striae, no caffe au lait, no
obvious masses.
 I am going to listen to the abdomen. 2 inches above umbilicus is the aortic bruit, renal is
2 inches on the same level, and the iliac are 2 inches below on 45 degree below.
 Tap, feel dullness in renal area for masses. No supra-renal masses.
 Femoral-radial delay,
 No peripheral edema.
Neuro:
Kneel on chair and do ankle reflex a
Quick neuro.

Back to Content

Please note that this is only a draft version based on several sources, including: Dr. Basel Mohasen’s lectures,
Toronto Notes, Therapeutic Choices and others. Edited and organized for the sake of all attendances of the
Canadian Osce Exams: NAC OSCE and MCCQE2. by: Dr. Merlyn D’Souza and Dr. Zeev Gross, Spring 2011. Page 255
P/E of CVS
INTRO:
VITALS: (Thank you for the Vitals)
Both Sys and Dia BP are elevated, HR is normal.
Orientation: time, place, person
G/ E:no obvious obesity
HEAD: Eyes - ±pallor/arcus senalis, no xanthelasma
Mouth: no dehydration
FUNDOSCOPIC: exam
HAND:Temp/Capillary refill/Clubbing/Nicotine stain
Pulse: regular/normal volume and contour
NECK: at 45o look for JVP, listen to carotids one by one, then palpate carotids
CHEST: ask to lower gown
INSPECTION:
Sit and look for pulsation
PALPATION:
PMI
Feel apex/thrills/heaves
AUSCULTATION: Aortic/tricuspid/MV, lay patient on left side, no S4
Sit up and lean forward, breathe out and hold it – listen if there is aortic regurgitation (?)
Listen to base of lung
Press on sacrum for edema.
Ask patient to lie down on bed
ABDOMEN: listen to bruit (aortic, renal and iliac)
LOWER EXTREMITY: temp, capillary refill, dorsalis pedis and peripheral edema

Three places you look for orientation:


 Volume status
 Malignant hypertension
 Hypoxia and SOB

Back to Content

Please note that this is only a draft version based on several sources, including: Dr. Basel Mohasen’s lectures,
Toronto Notes, Therapeutic Choices and others. Edited and organized for the sake of all attendances of the
Canadian Osce Exams: NAC OSCE and MCCQE2. by: Dr. Merlyn D’Souza and Dr. Zeev Gross, Spring 2011. Page 256
Volume Status
79 Hip replacement 3d ago, nurse asked to come and see, not passed urine for four hours. Do
Volume status exam.

INTRO:
VITALS: (and mention that BP should be done twice – while lying and sitting)
After measuring BP in one position, there are two minutes before you measure the second
position, meanwhile you do (the cuff of the BP should be on the same level of the heart).
Width of cuff is equal to 40% of circumference of arm.
1. ORIENTATION:Time, Place, and Person.
2. Listen to the base of the lung.
3. Look for sacral edema.
4. Look for sclera for pallor.
5. Mouth: open and look for dehydration. Flip tongue for central cyanosis.
6. Look for hands, skin (moist and dry).
7. Capillary refill – should be less than 2s.
Measure HR again and BP.
If there is no increase in pulse more than 20bpm and no decrease in SBP more than 20 or DBP
more than 10 – there is no orthostatic hypotension. If one of them is positive – Orthostatic
hypotension.
Put patient at 45o to do JVP. Press on base to see if JV disappears. Measure.
Take deep breath and hold – Kussmaul Sign absent.
Untie the shirt and do hepato-jugular reflex.
INSPECTION – S3 and S4 and all cardiac exam.
ABDOMEN: percussion at suprapubic to see if bladder is full.
Pedal edema, than look at examiner and ask for:
input-output chart & weight charts.
(If there is cathter:) I’d like to make sure that the catheter is not kinking.

Back to Content

Please note that this is only a draft version based on several sources, including: Dr. Basel Mohasen’s lectures,
Toronto Notes, Therapeutic Choices and others. Edited and organized for the sake of all attendances of the
Canadian Osce Exams: NAC OSCE and MCCQE2. by: Dr. Merlyn D’Souza and Dr. Zeev Gross, Spring 2011. Page 257
PVD
Pain in calf for three months.
Vitals (“patient is stable”), if patient is wearing socks ask him to remove them.
INSPECTION: (B) Feet: SEADS+3:
 Normal hair distribution
 Skin non-tight and shiny
 No hypertrophy of nails

PALPATION: (I will feel your feet):


 Peripheral temperature
 Capillary refill
 Pulse (both sides): DP, PT, Pop. & Femoral.
Drape the patient and listen to his abdomen for bruits.
Feet – LIGHT TOUCH SENSATION:
Bergers test – raise legs for two minutes – any you feel tingling/numbness tell me.
After two minutes tell: “no pallor, pain, numbness, tingling.” Sit up and dangle his feet – “No
rubour on depandance.”
I would like to do ANKLE BRACHIAL INDEX

Back to Content

Please note that this is only a draft version based on several sources, including: Dr. Basel Mohasen’s lectures,
Toronto Notes, Therapeutic Choices and others. Edited and organized for the sake of all attendances of the
Canadian Osce Exams: NAC OSCE and MCCQE2. by: Dr. Merlyn D’Souza and Dr. Zeev Gross, Spring 2011. Page 258
Diabetic Foot
Intro (As I understand you are here today cause you have DM for 2 y and ulcers in your Rt leg. I
have to do a P/E)
Vitals – stable.
Drape and remove socks.
INSPECTION:
Look at sole of foot.
Ulcer – 3cm in diameter, round, margin not elevated, no active bleeding or oozing, located at
base of 1st metatarsal.
I am going to look for other ulcers at the base of the toes (Between medial and lateral
maleolus.)
Check SEADS + 3.
PALPATION:
Temp and capillary refill.
Shift to NEUROLOGICAL EXAMINATION:
LIGHT TOUCH SENSATION in glove and stocking manner.
Start with big toe and go to level – and than up and down until finding the right level.
Light touch absent or decreased at a level to distal point. For example – above wrist.
And then say to the patient: “Thank you and open eyes.”
POSITION SENSE: close eyes and move the big toe up and down five to six times. “Thank you.
Open your eyes.”
VIBRATION SENSE: tuning fork – put on sternum to show how it feels, then put it on the 1st
interphalangeal joint. If doesn’t feel – vibration sense absent. Start with first joint, and second
joint (you check vibration also for (1) medial maleolus, (2) tibial tuberosity, anterior superior
iliac spine, sternum, chin, and forhead). (1) and (2) are for spinal injury.
ANKLE REFLEX:
MONOFILAMENT TEST:
press on sole or foot. Feel or no-feel.
Increase the pressure & bend the monofilament. “He has lost light touch and pressure.”
But if feels when pressure, say: “pressure present but light touch gone.”
You check the same way in 9 points on the sole.
PULSES: DP, TP
Auscultation (?) and ABI (Ankle-Brachial Index)

Acute Abdomen, Physical exam


Intro
General inspection: the patient is lying comfortably and I see no signs of distress. Can I get the
vitals please?
The patient is stable, normal temperature, BP and HR.
You are going to face – can I take a look at your eyes: there is no jaundice, no sign of anemia.
Please open your mouth – there is no sign of dehydration and obvious ulcer in the mouth.
Upper extremity: capillary refill is normal.

Please note that this is only a draft version based on several sources, including: Dr. Basel Mohasen’s lectures,
Toronto Notes, Therapeutic Choices and others. Edited and organized for the sake of all attendances of the
Canadian Osce Exams: NAC OSCE and MCCQE2. by: Dr. Merlyn D’Souza and Dr. Zeev Gross, Spring 2011. Page 259
Abdomen (drape appropriately): please move the cover – by inspection: the abdomen is not
distended, umbilical inverted, abdominal moves with breathing, no scars, no bruises. Ask
patient to look aside and cough twice (once look at his face to see for cough tenderness and
then for abdominal bulging).
Now I am going to listen (warm the sthetoscope): “Normal bowel sounds, no bruits – aorta,
renal, and iliac.”
Percussion: I am going to tap – show me where it pains. First tap away from painful area, than
tap over the 9 areas – the painful area last.
Pulpation: I am going to feel – no tender in epigastric/Rt. And Lt. Hypochondral/Rt. And Lt. Iliac
regions/ Umbilical / Suprapubic.
Deep pulpation: I am going to apply more pressure – there is no guarding in deep pulpation,
there are no obvious masses.
Now I am going to feel your kidneys – there is no enlargement of your kidneys.
Now I am going to do some special tests.
Murphy sign
Rebound tenderness.
McBurny sign.
Rovsing sign.
Psoas sign.
Obturator sign.
“Please sit up. Now I am going to tap your back. There is no tenderness on costo-vertebral
angle. Now listening again to the base of the lungs. There is no crepitus at the base of the
lungs.”
“I’d like to finish my exam by doing pelvic exam, vaginal exam for bleeding or discharge or
bimanual examinations. Looking for any cervical motion tenderness, and adnexal masses.”
“In DRE looking for any bleeding or haemorrhoids.”

Acute on chronic abdomen (like Crohn’s Dis.)


Add to the above:
General inspection: moon faces, truncal obesity, redness in sclera, nails – pitting and clubbing,
no skin rushes, no striae, no erythema nodosum on legs, sacroiliac joints look normal.

Back to Content

Please note that this is only a draft version based on several sources, including: Dr. Basel Mohasen’s lectures,
Toronto Notes, Therapeutic Choices and others. Edited and organized for the sake of all attendances of the
Canadian Osce Exams: NAC OSCE and MCCQE2. by: Dr. Merlyn D’Souza and Dr. Zeev Gross, Spring 2011. Page 260
Acute Abdomen, Physical exam
Intro
General inspection: the patient is lying comfortably and I see no signs of distress. Can I get the
vitals please?
The patient is stable, normal temperature, BP, RR and HR.
You are going to face – can I take a look at your eyes: there is no jaundice, no sign of anemia.
Please open your mouth – there is no sign of dehydration and obvious ulcer in the mouth.
Upper extremity: capillary refill is normal.
Abdomen (drape appropriately): please move the cover – by inspection: the abdomen is not
distended, umbilical inverted, abdominal moves with breathing, no scars, no bruises. Ask
patient to look aside and cough twice (once look at his face to see for cough tenderness and
then for abdominal bulging).
Now I am going to listen (warm the sthetoscope): “Normal bowel sounds, no bruits – aorta,
renal, and iliac.”
Percussion: I am going to tap – show me where it pains. First tap away from painful area, than
tap over the 9 areas – the painful area last.
Pulpation: I am going to feel – no tender in epigastric/Rt. And Lt. Hypochondral/Rt. And Lt. Iliac
regions/ Umbilical / Suprapubic.
Deep palpation: I am going to apply more pressure – there is no guarding in deep palpation,
there are no obvious masses.
Now I am going to feel your kidneys – there is no enlargement of your kidneys.
Now I am going to do some special tests.
Murphy sign
Rebound tenderness.
McBurny sign.
Rovsing sign.
Psoas sign.
Obturator sign.
“Please sit up. Now I am going to tap your back. There is no tenderness on costo-vertebral
angle. Now listening again to the base of the lungs. There is no crepitus at the base of the
lungs.”
“I’d like to finish my exam by doing pelvic exam, vaginal exam for bleeding or discharge or
bimanual examinations. Looking for any cervical motion tenderness, and adnexal masses.”
“In DRE looking for any bleeding or haemorrhoids.”

Acute on chronic abdomen (like Crohn’s Dis.)


Add to the above:
General inspection: moon faces, truncal obesity, redness in sclera, nails – pitting and clubbing,
no skin rushes, no striae, no erythema nodosum on legs, sacroiliac joints look normal.

Back to Content

Please note that this is only a draft version based on several sources, including: Dr. Basel Mohasen’s lectures,
Toronto Notes, Therapeutic Choices and others. Edited and organized for the sake of all attendances of the
Canadian Osce Exams: NAC OSCE and MCCQE2. by: Dr. Merlyn D’Souza and Dr. Zeev Gross, Spring 2011. Page 261
Hematemesis
“Because it is hematemesis I’d like to initiate a ATLS protocol for me and my team, please can I
can get gloves, goggles, masks, and gowns.”
Intro
How do you feel right now?
I want to make sure you are stable and therefore I’ll give some orders to the nurse. Once you
are stable I’ll ask you some questions.
ABCD
Vitals
OCD + COCA
How did it start?
Forceful and retching?
Did you vomit once or more?
How much?
Dark /bright blood?
Any clots?
Any smell?
IMPACT±PAIN
-PAIN  Liver
+PAIN  GIT

If No Pain:
Hx:
Any Hx of liver disease?
Any screening for liver disease?
Any bruising in body?
Increase in abdominal size lately?
Alcohol: how long? How much?
Hx of PUD
Heartburn
Any nausea
When was your last bowel movement? Colour? Any tarry stool/fresh blood?
Any Hx of bleeding disorder?
Any NSAIDs (Aspirin) – how much? How long? Why? Who prescribed?
Any blood thinner?
CSx (Ask for weight loss)
Long term disease

Physical exam:
Vitals
If suspected liver disease (no pain):
Sclera – no yellow discoloration, pallor
Enlargement of parotid glands

Please note that this is only a draft version based on several sources, including: Dr. Basel Mohasen’s lectures,
Toronto Notes, Therapeutic Choices and others. Edited and organized for the sake of all attendances of the
Canadian Osce Exams: NAC OSCE and MCCQE2. by: Dr. Merlyn D’Souza and Dr. Zeev Gross, Spring 2011. Page 262
Mouth: Fetor hepaticus, mouth is clear no bleeding no clots
Hands: no clubbing, capillary refill, no atrophy of thenar or hypothenar, no palmar erythema,
no dupytren’s contraction, no flapping tremor
Chest: no spider nevi, no gynecomastia, normal chest hair, no bruising
Abdomen: not distanded, umbilicus normal, caput medusa, collateral veins, no bruising.
Auscultation: bowel sounds normal, bruits (aorta, renal, iliac), no hepatic rub, hum, or bruit; No
splenic rub, hum
Tapping: four taps – general percussion, percussion for liver (upper and lower margin), spleen
(Castle sign), shifting dullness
DRE
Testicular atrophy
Peripheral edema
Epigastric tenderness
Gastroenterologist consult and admit to ICU
Endoscopy and IV PPI
Back to Content

Please note that this is only a draft version based on several sources, including: Dr. Basel Mohasen’s lectures,
Toronto Notes, Therapeutic Choices and others. Edited and organized for the sake of all attendances of the
Canadian Osce Exams: NAC OSCE and MCCQE2. by: Dr. Merlyn D’Souza and Dr. Zeev Gross, Spring 2011. Page 263
Acute Abdomen – management
Abdominal pain 24 hr with vomiting and diarrhea, BP 90/60, Pulse 140
Diagnosis: acute pancreatitis
Intro: “As I understand... please bear with me, as your BP is low I’ll give orders to the nurse, and
as soon as you become stable, I’ll give you something to relieve your pain.”
A
B – Vitals, Oxygen saturation
C – because he is hypotensive and tachycardic you give IV fluids;
Take blood to: (add amylase to the other blood work)
D – Gross neurological exam

Hx
What happened?
Pain: OCD, PQRST, AA
Vomiting: how much, how many times, amount, forceful, blood/coffee ground
IMPACT
RF (Alcohol, Gall stones, Hypertriglyceridemia, DM, Viral infection, Medications)
Recent trauma
Alcohol: how much, when was last time, last drink (was it more than normal?)
Hx of gall bladder disease
Recent flu
DM
Medications
CSx
Hx of HTn (R/O Aortic dissection)
Chest pain
Cough, phlegm
Flank pain
Liver disease
PMHx
FHx
SHx
Vitals

2min stabilize, 4min Hx

P/E
Look for liver disease: sclera, tongue, and hands
Abdomen: Drape
No Cullens and Great Turner signs.
Look for cough tenderness.
Auscultate bowel sounds: no aortic/renal bruit.
Feel or tap abdomen

Please note that this is only a draft version based on several sources, including: Dr. Basel Mohasen’s lectures,
Toronto Notes, Therapeutic Choices and others. Edited and organized for the sake of all attendances of the
Canadian Osce Exams: NAC OSCE and MCCQE2. by: Dr. Merlyn D’Souza and Dr. Zeev Gross, Spring 2011. Page 264
Groing exam
DRE
Orders: Meperidine, NPO, NG Tube, Admit to ICU, Foley catheter, Input-output chart,
Imaging: AXR, Abdominal U/S and CT, surgical consult

Back to Content

Please note that this is only a draft version based on several sources, including: Dr. Basel Mohasen’s lectures,
Toronto Notes, Therapeutic Choices and others. Edited and organized for the sake of all attendances of the
Canadian Osce Exams: NAC OSCE and MCCQE2. by: Dr. Merlyn D’Souza and Dr. Zeev Gross, Spring 2011. Page 265
MI – Management
Hx
OCD
PQRST
(if it is suspected to be ACS - stop at R and start primary survey)
Primary Survey (If patient talks – Airway preserved,
Take Oxygen saturation and start Oxygen Stat – 4L/m through nasal prongs)
VITALS:
Auscultation:
 Air entry (N)
 S1 & S2 (N)
IV lines :
NaCl 50ml/hr to keep line open,
from the other side take blood for: Troponin, CK-MB, Cr, BUN, Lytes, CBC, INR, PTT, LFT, Toxic.,
Alcohol, Lipids; and finger prick for Glucose)
ECG 12 leads & continous monitoring
Portable X-ray (r/o dissection)

Ask about Allergy for Aspirin and Viagra (if negative)[12hrs for Viagra & 36 hrs for
Cialis]
Give ASA chewable (325mg)

Non-ST elevation
Nitro x3 (S.L)
Morphine
Continue now with:
PQRST
AA&A
How do you feel now?
Ask Hx:
CVS
GI (especially peptic ulcer)
CSx
RS
DVT

ST Elevation: do not go for DDx


Nitro (IV Nitro is C/I in IWMI)
Morphine (5mg if ALMI, and 1mg if PWMI)
VITALS: (again)

Please note that this is only a draft version based on several sources, including: Dr. Basel Mohasen’s lectures,
Toronto Notes, Therapeutic Choices and others. Edited and organized for the sake of all attendances of the
Canadian Osce Exams: NAC OSCE and MCCQE2. by: Dr. Merlyn D’Souza and Dr. Zeev Gross, Spring 2011. Page 266
R/O Contra Indications for Thrombolytics:
 Peptic ulcer & Recent surgery,
 Pericarditis, Aortic dissection,
 Brain tumor, & Stroke

Start Thrombolytics:
Tpa
 Ask for heparin protocol
 Start B Blocker

RISK FACTORS:
 HTN
 DM
 FH
 Coccaine
Nitro (2nd dose)
O/E:
 JVP
 Listen to heart
 Base of lung
 Compare BP in both Upper extremities to r/o coarctation of Ao
CXR
Once there is no Aortic Dissection  Thrombolytics (should be clear to r/o: Peptic ulcer, recent
surgery, pericarditis, aortic dissection, brain tumor, and stroke)
Based on ECG – counselling

Counseling
Based on your ECG it is most likely you are having an heart attack. If stable – BP and HR are
stable, but it is a serious condition, however it is treatable. Heart attack means that greater
than one blood vessel supporting your heart is blocked by a clot that has to be reimoved. The
medications are called clot busters. Based on ECG and no sign of pericarditis or signs of aortic
dissection you are a good candidate for treatment. It is an effective medication, needs consent.
1% chance of stroke and we can start heparin.

Back to Content

Please note that this is only a draft version based on several sources, including: Dr. Basel Mohasen’s lectures,
Toronto Notes, Therapeutic Choices and others. Edited and organized for the sake of all attendances of the
Canadian Osce Exams: NAC OSCE and MCCQE2. by: Dr. Merlyn D’Souza and Dr. Zeev Gross, Spring 2011. Page 267
Respiratory System – P/E
 General inspection: comfort, colour, pursed lips, flare nose, intercostal retractions, auxiliary
muscles
 Eyes, nose (perforated septum), mouth (ulcers, thrush in HIV, central cyanosis, moist
tongue)
 Hands: peripheral cyanosis, clubbing, capillary refill
 Cervix: trachea, lymph nodes
 Chest: inspection (symmetry, expansion, intercostals retractions);
Palpations for any pains, estimating chest expansion
Tactile phremitus (“99”)
Tappings (including sides): dullness/tympanic, diaphragmatic excursion
Auscultation (including sides): vesicular sounds
Vocal phremitus: “E”
Whispering pectoriloqui: “1,2,3”
 Heart: pulses and auscultation (r/o AF and Rheumatic disease).
 Other lymph nodes: axial, femoral, popliteal
 Lower leg: no signs of Caposi sarcoma, DVT (Homan sign, measuring calf in case of
tenderness or suspicious calf swelling).

Please note that this is only a draft version based on several sources, including: Dr. Basel Mohasen’s lectures,
Toronto Notes, Therapeutic Choices and others. Edited and organized for the sake of all attendances of the
Canadian Osce Exams: NAC OSCE and MCCQE2. by: Dr. Merlyn D’Souza and Dr. Zeev Gross, Spring 2011. Page 268
Diabetic Daughter 2y, Counsel
Either she is not doing well in school as she is not seeing well due to vision problems
Not playing well, as she is tired
DKA

Is it regular f/u or something special you wanted to discuss?


When was the last f/u?
How was she diagnosed?
What happen then?
What were the symptoms?
Any pain / vomiting?
Are you feeling eating/drinking/peeing more?
Any weight loss or blurred vision?
From the last f/u till now have you had DKA?
How about before?
Have you had low blood sugar?
Talk with the father:
Which medication does she take?
How does she take?
When was the last time?
Do you take insulin or somebody else gave it to you?
Do you take it all the time?
DO you skip dose?
Does she need any help to take insulin?
DO you measure blood sugar regularly?
When was the last time?
Do you record them in the machine? (The glucometer should be used by only one patient).
There is a blood work called ―Hemoglobin A1C‖ it is done every three month – did you do it?
Did you start new medication?
How about your diet?
DO you have your log book?
What do you eat?
Have you ever seen by a dietician?
PMHx
FHx

Counseling
A lot of people have diabetes and she is not the only one. What’s your understanding of
diabetes?
Whenever we eat food contains sugar it is absorbed in our stomach and goes to the blood and
from there to different parts of our body. Sugar act in our body like a fuel, in order for our body
to use this energy it needs insulin. Patients having diabetes have not enough insulin. Sugar will
be built up in your blood. The body tries to get rid or it, by peeing extra sugar – this will lead to
thirsty and tiredness.
Please note that this is only a draft version based on several sources, including: Dr. Basel Mohasen’s lectures,
Toronto Notes, Therapeutic Choices and others. Edited and organized for the sake of all attendances of the
Canadian Osce Exams: NAC OSCE and MCCQE2. by: Dr. Merlyn D’Souza and Dr. Zeev Gross, Spring 2011. Page 269
This can be avoided by controlling the blood sugar. If you control your blood sugar you’ll be
able to play again. If not controlled – may end in DKA, hypoglycaemia and serious
consequences.
Always be aware of hypoglycaemic symptoms: loss of conscious, sweating, heart racing, hungry.
Since you might lose conscious it is important to carry MedAlert Caed or Bracelet which will
clarify your situation.

Back to Content

Please note that this is only a draft version based on several sources, including: Dr. Basel Mohasen’s lectures,
Toronto Notes, Therapeutic Choices and others. Edited and organized for the sake of all attendances of the
Canadian Osce Exams: NAC OSCE and MCCQE2. by: Dr. Merlyn D’Souza and Dr. Zeev Gross, Spring 2011. Page 270
Medical Error, Wrong blood transfused
When there is a mistake, always there is a kind of unintentional medical error.
(to the nurse) when informed about wrong blood – ask: ―did you stop the blood?‖
say: ―Well done!‖
If she asks not to tell the patient...ask her what her believe she may lose her job, and it is too
early to determine who is responsible. Errors take place in medical practice. We don’t know
what exactly happened. We will stabilize patient and ensure he’s fine and later deal with this
issue.
Remove blood unit and keep cannula
(to the patient)INTRO:
I am the doctor in charge, and it looks like it was an unintentional medical error took place. We
need to make sure you are stable. We don’t know who is responsible, there are at least 15 steps
and in each step could have been an error. We will fill an incident report and as soon as we get
result we will inform you. You can sue, it is your right at the moment it is my priority to stabilize
you.
I will start PRIMARY SURVEY:,
ABCD
A – Open your mouth
(check for anaphylaxis, no swelling in mouth, ask for any itchiness, or difficulty breathing),
Oxygen saturation.
Normal air entry.
Normal S1, S2
VITALS: Pleaese
. Remove blood unit and keep cannula
C:
Start new IV line.
Once new line, don’t give fluids if stable.
Send blood: CBC, Lytes, INR, PTT, LFT, Cr, BUN,
FDP, Haptoglobulin, Direct coombs test; Urinalysis: hemoglobulinuria
Unit to be sent to blood bank for cross matching.
Ask nurse to call the blood bank and keep original blood.

D
D1 – I’d like to shine a light in your eyes. Pupils are round, active, and symmetrical. Squeeze my
finger, wriggle...wriggle...
D2 – (if febrile) give tylenol
Please prepare for me :
 Benadryl (Diphenhydramine) 50mg.
 Steroids (Hydrocortisone) and
 Epinephrine

SECONDARY SURVEY:
Hx (two parts:)

Please note that this is only a draft version based on several sources, including: Dr. Basel Mohasen’s lectures,
Toronto Notes, Therapeutic Choices and others. Edited and organized for the sake of all attendances of the
Canadian Osce Exams: NAC OSCE and MCCQE2. by: Dr. Merlyn D’Souza and Dr. Zeev Gross, Spring 2011. Page 271
1. Condition (how is he feeling now)
2. ―Why blood was given?‖

CONDITION:
Check out for Anaphylactic shock:
Do you feel warm? Chills? Itchiness? Tinglings? Diffculty breathing? Wheezing? Swelling in
lips / fingers? Hives?
Before transfusion did you have fever?
Check for Haemolytic reaction – any back or flank pain?
P/E – no oozing at IV line
Then press on flank and back – no pain for haemolytic reaction.
Is it the first time?
WHY did you receive blood?
If received blood before – was there any complications?
Any long term diseases?

COUNSELLING:
Mr. X what do you know about blood transfusion?
It is a life saving measure, and a lot of measures are taken to make sure it is safe. However, like
any other medication with blood transfusion there could be side effects, and these side effects
could be serious.
The most common side effect is:
 Febrile reaction (3%), usually it is self limited and can happen again. Next time you
receive blood we will give you tylenol.
 Anaphylactic reaction. It is a severe allergic reaction, and it is very serious and we
cannot predict it. However, we have good measures to deal with it, and your symptoms
make it less likely that you have had an anaphylactic reaction.
The third reaction is more serious and called
 HEMOLYTIC reaction. Usually happens when patients receive blood belonging to
another blood group.
The fact that this blood is same as your blood group, and the symptoms are not consistent
with haemolytic anemia make it less likely that this is not the case here. The blood is sent
to the blood bank and once results are back we will get final confirmation, we will able to
reassure you.
Back to Content

Please note that this is only a draft version based on several sources, including: Dr. Basel Mohasen’s lectures,
Toronto Notes, Therapeutic Choices and others. Edited and organized for the sake of all attendances of the
Canadian Osce Exams: NAC OSCE and MCCQE2. by: Dr. Merlyn D’Souza and Dr. Zeev Gross, Spring 2011. Page 272
Son has anaphylactic shock, is stable now.
Next few minutes I’ll talk with you and hopefully will come to a good plan.
Yawning – give empathy.
Hx (Short)
It happened at home you should take history.
If not – don’t take history.
 Itchiness,
 Swelling,
 Hives.
 Was he able to talk, wheezing, chest tightness,
 Lost his consciousness,
 Turned blue?
Start immediately with Epinephrine.
What have you done at the event? What did they do?
Any other children at home with anaphylactic shock?

Management
Based on the Hx your child has anaphylactic shock.
Explain: a kind of severe allergic or hypersensitivity, from birth or develop later. Usually people get
allergic to foods, medications, or chemicals.
Any questions?
At certain stage the immune system starts to interact with some elements of the peanut which are
called antigens. From now on when your son will be exposed to the same antigens it will lead to release
of some chemicals which will affect his skin, widening blood vessels which will become leaky and
different parts of your body will become swollen.
When not enough blood will reach the brain it will lose conscious, difficulty breathing. The concern we
have is that it might happen again. It is common.
Plan: the best treatment is prevention.
After that I need to go and talk with your child.
 You have to check the ingredients of any food you buy – make sure it is peanut free.
 IF there are other children at home they must be informed as well.
 In case that your child was exposed to peanuts by mistake, you should use EpiPen – this is a special
pen, has a cap at the top, which is needed to be activated by removing the cap, press it against his
thigh for ten seconds. This increases the blood pressure for about 20 min, in that time you should
seek help.
 Your son should carry with him two pens – one at home and one on his bag.
 he should carry Med Alert. In case your child become unconscious
 I will refer him to allergist specialist.
 Aspirin, stress test, and imaging...
 Some children will outgrow it.

Back to Content

Please note that this is only a draft version based on several sources, including: Dr. Basel Mohasen’s lectures,
Toronto Notes, Therapeutic Choices and others. Edited and organized for the sake of all attendances of the
Canadian Osce Exams: NAC OSCE and MCCQE2. by: Dr. Merlyn D’Souza and Dr. Zeev Gross, Spring 2011. Page 273
Marijuana Counselling
(Mother comes in to see you as she has discovered Marijuana in her son’s belongings)

INTRO:
As I understand you’re here because you’re concerned about your son.
What is his name?
What is your concern?
How much did you find?
Did you ask him about it?
 WHAT MAKES YOU BELEIVE IT IS MJ ?
Is he using it? Or Is he carrying it?
Is it the first time you’ve found it?
 Did you notice any CHANGES in his behaviour?
Is he excited?
Laughing out of nowhere?
Is he preoccupied?
Does he stare at a wall?
Does he talk to himself?
Is he aggressive?
Any problems with the law?
Any fights?
Any criminal records?
Is he more isolated?
 How is his MEMORY?
Is he more forgetful/lose his stuff?
Does he take more time to react?
Does he spend more time in his room?
How much time do you spend with him?
How much time is he out of the home?
How much time does he spend with his friends?
Do you know any of his friends?
What kind of activity are they involved in?
 Does he have a lot of MONEY?
Does he ask for money?
Do you believe he steals money?
Do you think he smokes/or drinks alcohol?
 How would you describe his MOOD?
Is he depressed?
Is he still interested in his hobbies?
Does he worry a lot?
Does he have excessive fears & avoid situations?
Do you have concerns that he may harm himself or anyone else?
EDUCATION:
How is he doing in school?
Have his grades dropped?
DIET:
How is his general health?
Please note that this is only a draft version based on several sources, including: Dr. Basel Mohasen’s lectures,
Toronto Notes, Therapeutic Choices and others. Edited and organized for the sake of all attendances of the
Canadian Osce Exams: NAC OSCE and MCCQE2. by: Dr. Merlyn D’Souza and Dr. Zeev Gross, Spring 2011. Page 274
Have you ever seen a psychiatrist?
Fhx: SAD
COUNSELLING:
Based on what you’ve told me.There are no changes in his health & behaviour (assumed that
there were no changes in behaviour as per mum)
When it comes to Marijuana it is a commonly used drug by teenagers, sometimes only once for
experiment. When we talk about Substance Abuse & drugs we talk about different categories.
Marijuana is a SOFT DRUG,others like: Coccaine,Heroin& Amphetmanies are HARD DRUGS
Let us talk about Marijuana first.
It is from the Cannabis family & affects the brain by feeling happy, excited & enhances
experience.Sometimes with prolonged use or in high doses can cause side effects including
apathy.
It interferes with memory,& can interfere with his studies & function & fine motor skills & may
not be able to operate machinery
It impairs judgement & he might take risks.
Can cause Lung cancer
In some teens,in high doses unmasks schizophrenia & cause psychosis
Interferes with sexual function & can cause infertility & weight gain
By itself marijuana is not strongly addictive & hence he can stop it at any time with help.One of
the concerns of Marijuana though is it acts as a bridge to Hard drugs which are addictive i.e
you’ve to increase the dose to have the same effect,which is called “TOLERANCE”,& then one
cannot stop the drug as it causes withdrawal .
It is a crime to use,hold hard drugs.People can lose their jobs.
If injected increases risk of HIV,Hepa B & C
PLAN
If you like,bring your son here I can talk to him.
It is better to be a confidante to him. Try to be close to him, someone he can trust & can talk
to.Try to make sure who’re his friends,& make sure you know what he is doing.Keep him busy
with activities.
If there are any druh prevention programmes in your community or his school,get him to attend
them & gets the knowledge.

In case of the resident who was asked to backup his supervisor orthopaed
7. I am competent – to emphasize
8. Short term – we don’t have time so we need to see her urgently
9. Long term – solve the situations that it wouldn’t occur again

Back to Content

Please note that this is only a draft version based on several sources, including: Dr. Basel Mohasen’s lectures,
Toronto Notes, Therapeutic Choices and others. Edited and organized for the sake of all attendances of the
Canadian Osce Exams: NAC OSCE and MCCQE2. by: Dr. Merlyn D’Souza and Dr. Zeev Gross, Spring 2011. Page 275
Pregnant 35 YO has concern about breast feeding.
Tell me more about your concern.
Is it: General information you like to discuss or some specific concerns?
If worried about pain: Ask:
 Have you ever been pregnant before?
 Have you ever breast fed before?
 What makes you believe it is painful?
 In addition to that any other concern?
Mostly my concern is...
It is a good concern,& I’m gald you came in today.
How is your pregnancy?
When was your last F/U?
When is your due date?
Let us talk about the pain.....
Breast feeding is a natural physiological process & usually it does not cause any pain.
However, sometimes it might cause some discomfort.
If it pains there should be some reason for that.
Most of those causes are treatable
Most commonly – cracks and fissures. They are caused because of not proper care of the nipple.
You have to make sure that they are moist, clean them, and don’t use soap.
To learn appropriate technique it might take some time.
I can send you to some classes that might guide you.
Retracted nipple and inflammation of the breast (mastitis) you can still continue to breast.
Localized condition, like abscess, we still recommend to continue to feed breast from the other side.
Before we proceed further I’d like to make sure if you are a good candidate for breast feeding.
 Do you have any long term diseases,
 Do you take any medication or radiation,
 Do you smoke or taking any drugs,
 Have you been screened for TB or HIV.
 Do you plan chemotherapy or radiation therapy?
Based on what you’ve told me,you’re a good candidate for Breast feeding
COMPARE BETWEEN BREAST MILK & FORMULA
The reason we recommend breast feeding is that we cannot match it with formulas.
The first 24 hours secretion is “ Colostrum” ,it is a special kind of milk& has a lot of antibiotics,
immunoglobulin & essential amino acids, which are essential for your baby which will give him
protection.
With time the milk becomes more mature and suits the needs of your baby. It has the right amount of
carbohydrate & fat. The quality of the fat is better. It has more whey relative to casein. The iron is less
than cow milk but is more available (50% as to 20%).
Less load on the kidneys
There are other benefits to your baby and you – there is emotional connection which is important to
both of you.
Babies breast fed have
 Less chances of having allergies
 Less chance of having diarrhea
 Less chance of being obese
 Less ear infection,
Please note that this is only a draft version based on several sources, including: Dr. Basel Mohasen’s lectures,
Toronto Notes, Therapeutic Choices and others. Edited and organized for the sake of all attendances of the
Canadian Osce Exams: NAC OSCE and MCCQE2. by: Dr. Merlyn D’Souza and Dr. Zeev Gross, Spring 2011. Page 276
 Some studies even suggest that they might have higher IQ.
There are some benefits for you (the mom)
It helps to lessen the postpartum bleedings (because the oxytocin) the uterus go back to it’s size
Helps you to regain the figure you’d prior to pregnancy.
It is clean, available, always at the right temperature, even though you don’t pay for it –
It is one of the most important things you can give your child.
If chooses to breast feed:
I will send you to clinic who will teach you.
At the beginning the breast feeding is on demand & with time it regulates ,& you need to feed every
3 – 4 hours & at least 10 min in each breast.
Monitor weight gain to ensure that the baby is adequately fed.
Occasionally the baby may be jaundiced & sometimes stool may be loose.
If you choose to breast feed you’ve to be careful whenever you take medications or alcohol.
You can go back to work, after the Maternity Leave (ask for how long)
Breast feeding can be continued. You can use some special pumps. Even if there is engorgement you
can use the pump. Make sure it is always clean. Breast milk can be stored at 6hr in room temp, 24 hr in
fridge, and 6m in freezer. Don’t put it in the microwave for heating.
Breast feeding is not reliable method of contraception. Recommend the minipill or barrier method.

Back to Content

Please note that this is only a draft version based on several sources, including: Dr. Basel Mohasen’s lectures,
Toronto Notes, Therapeutic Choices and others. Edited and organized for the sake of all attendances of the
Canadian Osce Exams: NAC OSCE and MCCQE2. by: Dr. Merlyn D’Souza and Dr. Zeev Gross, Spring 2011. Page 277
Febrile Seizure
A child brought to the ER because of febrile seizure.Next 10m counsel him.
 EVENT
 TRIGGER r/o meningitis.
 EDUCATE & what to do next time
INTRO:
As I understand you’re here because your child had a seizure 20 minutes ago,& my colleagues are
looking after him & he is stable. During the next few minutes I’ll ask you few questions, and after that I’ll
go with you to see him.
EVENT:
 Describe the event.
 What happened?
 Did you see him? (Started to shake. All over his body? Bite his tongue / rolling up his eyes / wet
himself).
 Did he fall from a height?
 How long did it last?
 Did he stop seizing by himself or did he need medical intervention?
 How did he regain consciousness?
After the seizure does he have any neurologic deficits:
 Was he drowsy
 Did not recognize you
 Able to move his arms & legs
Is it the 1st time? Or happened before
If first time:
Ask about fever?
(if it started a week ago – did you seek medical assistance? Any ear discharge? Did they give you any
treatment? Did you give it to him or no?)
Why! Some studies show you can treat OM without antibiotics. If reason medication not given was
because parent was busy.....You should look for the reason not to give the antibiotics (negligence?).
Was he playful,eating,drowsy,
Is he having any vomiting?
Skin rash?
Coughing & phlegm SOB, Wheezing
Foul smelling urine & painful peeing
Head to toe...
If you find nothing – ask when he got his last shot? (up to 72 hours he can have fever).
 R/O meningitis, pneumonia.
 Any family history of febrile seizures, epilepsy
BINDE (especially immunization to R/o Measles)
COUNSELLING:
Most likely on what you’ve told me, your child has condition called febrile seizure (FS).Do you know
what it is?
It is a special condition in children that might happen from 6m to 60m. We don’t know exactly why – we
believe it is a sudden change in the temp & as the brain is not developed fully thes e changes might lead
to the seizure. This condition might happen again. The best treatment is: PREVENTION
Therefore from now whenever your child has a temperature
Please note that this is only a draft version based on several sources, including: Dr. Basel Mohasen’s lectures,
Toronto Notes, Therapeutic Choices and others. Edited and organized for the sake of all attendances of the
Canadian Osce Exams: NAC OSCE and MCCQE2. by: Dr. Merlyn D’Souza and Dr. Zeev Gross, Spring 2011. Page 278
Seek medical attention. Give Tylenol and sponge bath to decrease his temp.
Most of the children will outgrow this condition by the 6th year.
Chances of epilepsy later in life are higher
In FHx of epilepsy,it is a risk fct for development of epilepsy.
They don’t recommend Diazepam because it might make him drowsy.
I will give you brochures

If it stopped less than 5m or more than 5m including neurological symptoms seek ER immediately.
Brochure.

Back to Content

Please note that this is only a draft version based on several sources, including: Dr. Basel Mohasen’s lectures,
Toronto Notes, Therapeutic Choices and others. Edited and organized for the sake of all attendances of the
Canadian Osce Exams: NAC OSCE and MCCQE2. by: Dr. Merlyn D’Souza and Dr. Zeev Gross, Spring 2011. Page 279
PHONE CASE:
“This is Dr. ... (immediately should introduce yourself). I am the Dr. In charge in the ER.As I understand,
you’re calling as your child has swallowed some medication. I know that you’re stressed & it is a difficult
time for you. I need your phone number now & it is important, as if we get disconnected I will call you
back.
What is your address?
How far away from the hospital are you?
Try & stay calm. Your son needs you, I am going to give you some instructions and you need to follow
them.
FIRST STEP:
Is your son is alert or not?
Is he conscious?
Can he talk to you?
Can he recognize you? (If he doesn’t – do you know how to do CPR and start with that.)
He’s crying?
What is his colour? Pink?
Hold him and try to calm & soothe him.
Try to hold him and check his mouth,if there are medications there,remove them.
Is he breathing?
We’ll send the ambulance for you.
When did it happen?
How long was he alone?
Which medications did he take?
Whose medications did he take?
Do you have the container?
(Don’t go to the next room to bring them,when the paramedics arrive then you can go & get the
container).
Do you know what condition your father have (was it vitamins, sleeping pills, or any other?)
How much the amount?
Don’t use any ipecac? Do not induce vomiting.
Is it happened before?
What is the weight of the child?
BINDE (was it full date, did he needed special attention after term, and does he have any special
conditions). Weight for two reasons – antidote and estimate neglect.
Are his shots up to date?
Are there other children at home?
Have you visited the Er frequently?
Post encounter Q: what are the first four steps you do when he arrives?
ABC,
Monitor vitals,
IV line, NG,
Foley as needed,
Blood works – CBC, Lytes, BUN, Cr, Osmolality, Coagul, LFTs, Tox screen – blood and urine).
List three risk factors forneglect for this child.
What is the antidote for betablocker (glucagon) and for CaChannel is (Calcium gluconate).
CAS & Poisoning centre.

Please note that this is only a draft version based on several sources, including: Dr. Basel Mohasen’s lectures,
Toronto Notes, Therapeutic Choices and others. Edited and organized for the sake of all attendances of the
Canadian Osce Exams: NAC OSCE and MCCQE2. by: Dr. Merlyn D’Souza and Dr. Zeev Gross, Spring 2011. Page 280
Second scenario Phone case: Febrile Seizure
This is Dr. ... (immediately should introduce yourself). I am the Dr. In charge in the ER.As I understand,
you’re calling as your childis having a seizure. I know that you’re stressed & it is a difficult time for you. I
need your phone number now & it is important, as if we get disconnected I will call you back.
What is your address?
How far away from the hospital are you?
Try & stay calm. Your son needs you, I am going to give you some instructions and you need to follow
them.
While he is seizing just put him on the side, and not start any CPR.
Is he seizing right now? Try to put him on the floor on the left side (the right bronchus is shorter than
the lt.).
Observe him.
What is his colour?
Is he still shaking? You send the ambulance.
Can you tap on his shoulder?
If he is not responding – can you do CPR?
Can you feel his pulse?
If stopped seizing...... Good
Is he alert?
Does he respond?
Can he talk to you?
Can he move his legs?
EVENT:
Can you describe what happened?
OCD
Fever +/-
Does your child have fever?
Did he have Hx/Nx/Vx/Skin rash/Neck stiffness
Any long term disease?
Did you seek medical attention? What prevented you from giving the medication?
Is it the same time or happened before. If it is the second time – more than 15m he needs intervention.

Back to Content

Please note that this is only a draft version based on several sources, including: Dr. Basel Mohasen’s lectures,
Toronto Notes, Therapeutic Choices and others. Edited and organized for the sake of all attendances of the
Canadian Osce Exams: NAC OSCE and MCCQE2. by: Dr. Merlyn D’Souza and Dr. Zeev Gross, Spring 2011. Page 281
Post Concussion.

Hx & PE

INTRO:
As I understand you had a head injury 3 days ago when playing hockey.
Start with EVENT: Before & After
If LOC ask How long?
Do u remember what happened? What was done?Were you hospitalized?
Was a CT Scan done?
HOW DO YOU FEEL TODAY?
Full neuro assessment
Headache:
OCD PQRST U V A&A
Vomiting,bending,Nausea
Balance,vision falls weakness numbness
Difficulty finding words
Past Med Hx:
HEADDS
PE:
Vitals
CN
Power
Reflexes
Sensory
GAIT check Tendem gait

Conclusion:
I know you’re eager to play hockey
Since you’ve headache you’re not ready to play again as you still have active Sx.
If you start to play again your tolerance for injury is lower & if you are injured again,your tolerance is
lower & recovery time is longer & there might be serious consequences.
Why don’t you wait till full recovery time
I will refer you to a PT & with gradual step up exercises you can get back to your game:
One week with warming up, after that stationary activity, after that skiing, than drilling without contact
(seven steps of rehab.).

Back to Content

Please note that this is only a draft version based on several sources, including: Dr. Basel Mohasen’s lectures,
Toronto Notes, Therapeutic Choices and others. Edited and organized for the sake of all attendances of the
Canadian Osce Exams: NAC OSCE and MCCQE2. by: Dr. Merlyn D’Souza and Dr. Zeev Gross, Spring 2011. Page 282
Osgood Schlatter
2 scenarios (Osgood schlatter and Post-concussion)
Decision will based whether the child can tolerate pain or not?

#1 About to see the father of 14yom with Osgood Schlatter.


Make sure that the child best interest are preserved. What was done to diagnose the child?
OCD PQRST compare to the other knee, is the first time or not, was any trauma.
What is the child wish? (Don’t go for HEADDSSS since it is the father).

Counseling
What is your understanding of OS.
Let me explain to you what is the mechanism for OS.
Avoid him from playing, especially jumping. But he can continue with ice presses and pain killers. The
rule is that he can continue up to his limit of his pain.

Back to Content

Please note that this is only a draft version based on several sources, including: Dr. Basel Mohasen’s lectures,
Toronto Notes, Therapeutic Choices and others. Edited and organized for the sake of all attendances of the
Canadian Osce Exams: NAC OSCE and MCCQE2. by: Dr. Merlyn D’Souza and Dr. Zeev Gross, Spring 2011. Page 283
IMMUNIZATION
(Newcomer come to Canada from Ukraine, concerned about immunization)
May need interpreter: Ask:
Do you understand?
Can I talk slower?
Do you need interpreter?
INTRO: As I understand you’re here as you’re concerned about vaccines & my understanding is
that you’re new to Canada, Welcome to Canada!
What is your concern?
 Pt: My neighbour told me vaccines are not safe
Dr: what do you mean?
Pt: Concerned about autism & vaccine
Dr: This is a reasonable enough concern,& I’m glad you came here.
There is a misinformation about between vaccines & autism.The origin of this misunderstanding
is due to a study done in the UK & the author of that study found a connection between autism
& vaccines. Because vaccines are lifesaving & important for our children’s protection,further
studies were done,also in other countries,& then it was definitely proved that there is no
connection between autism & vaccines.The only connection was coincidence between time of
the vaccine & time when symptoms of autism were picked up by parents.
 Another common concern is that mercury was used as a preservative for MMR vaccine.
It is no longer used now.
When we find out why this study gave such a result it was found out that there was a bias in
sample & thus led to the wrong conclusion.
 Another common Qn pt may ask:
These diseases do not exist in Canada, so why give my child the vaccine if there is no disease
here.
The world is getting smaller & even though we do not have these diseases in Canada, because
we have the vaccines, it does exist around the world & people travel.So we do not want your
son to get affected whenever there is an outbreak somewhere in the world.
Hx: Let me ask some questions about your son:
 How old is he?
 Has he received any vaccines so far?
 Were there any side effects?
 Any reason why he was not vaccinated?
 Does he have any congenital medical condition?
 Does he have any allergies (egg??)
 Any neurological history?
Inform that baby is a good candidate
As I told you vaccines are life saving, before vaccines many children died from measles, rubella.
The reason being children are not fully protected & may get the infection & die. Once
vaccinated, children get the immunity
Any Qns?
HOW:
Please note that this is only a draft version based on several sources, including: Dr. Basel Mohasen’s lectures,
Toronto Notes, Therapeutic Choices and others. Edited and organized for the sake of all attendances of the
Canadian Osce Exams: NAC OSCE and MCCQE2. by: Dr. Merlyn D’Souza and Dr. Zeev Gross, Spring 2011. Page 284
We take different bugs like bacteria, viruses or products of these bugs & process them so that it
does not harm the body,& inject it into our bodies by needles. Our body reacts by forming
elements that fight these antigens, so later in life when your son is exposed to the real factor,
these antibodies will protect him. Some of these antibodies will last forever; some will need
booster doses.Because there are a lot of disease we need to minimize the number of injections
& we’ve to give greater than one needle for vaccination.
There is a combination vaccine e.g: PEDISIL = DPT + HiB + Polio
This is given as a single shot at 2,4,6 & 18 Mo.
We will give you a schedule to remind you each time you’ve to come to the clinic
Concerning the SIDE Effects, the benefits clearly outweigh the S/E,however:
A febrile reaction can develop & you can give Tylenol if this occurs
Pain & swelling at injection site
Some children can have prolonged crying
Others may become floppy
In still rare conditions can have a seizure
Very rarely,can gt an anaphylactic reaction
Since you’re a newcomer & not got your insuarence there are some organizations that will help
you out.

Back to Content

Please note that this is only a draft version based on several sources, including: Dr. Basel Mohasen’s lectures,
Toronto Notes, Therapeutic Choices and others. Edited and organized for the sake of all attendances of the
Canadian Osce Exams: NAC OSCE and MCCQE2. by: Dr. Merlyn D’Souza and Dr. Zeev Gross, Spring 2011. Page 285
IUGR

INTRO:
As I understand you’ve just given birth to a baby,& my colleagues are looking after him,& I’m
here to talk to you.
How do you feel right now?
Have you seen your baby?
Did anyone tell you about your baby?
If at this point mother voices a concern that she saw her baby covered with green stuff.
Your baby was covered with “Meconium” one of the substances in the fluid surrounding your
baby.It is normal for the baby when under stress during delivery.
I’ve been told that your baby has a condition called “IUGR”,& I need to ask questions as to why
it happened
Qns about Pregnancy:
Smoked/Alcohol/Drugs
Qns about Delivery:
Term or preterm
MGOS:
O:Previous pregnancy/abortions/miscarriage, & if yes how many?
G:If Hx of Cancer or chemotherapy
Any congenital disease in her or husband’s family or Consanguinity
If she asks whether her mistake:
Don’t reproach her – it is NOT her mistake.
It is a multi-factorial condition. Can be due to various causes, some genetic, pregnancy, related to baby
Because safe levels of smoking, drugs & alcohol not known,
We always recommend not to smoke or drink for future pregnancies.
Back to Content

Please note that this is only a draft version based on several sources, including: Dr. Basel Mohasen’s lectures,
Toronto Notes, Therapeutic Choices and others. Edited and organized for the sake of all attendances of the
Canadian Osce Exams: NAC OSCE and MCCQE2. by: Dr. Merlyn D’Souza and Dr. Zeev Gross, Spring 2011. Page 286
CHILD ABUSE:
 # Femur
 # LE & (B) limbs
 Spiral #
 # post ribs
1.EVENT
2.1st time or prior
3.BINDE:
4.Past MH for osteoporosis imperfecta
4. COUNSELLING

INTRO:
As I understand you’re here as your child had a #.My colleagues are looking after the child, who is stable
now.
EVENT:
How?......Describe what happened….if fall from couch: How high is the couch?
When? …If time log……Why bring the child now??? If at night? Did he sleep or was he crying?
Were you there?
Did you see it?
Any LOC?
Is it the 1st TIME or has it happened before?
If before?
How many times?
Type of #?
Did you come to the same hospital or to a different one?
Any other children at home?
BINDE:
Planned pregnancy
Term pregnancy
Any cong anomalies
During pregnancy: SAD
Immunizations up to date or not
Weight today
Development: Is he a difficult child?
Environment:
PARENT-CHILD RELATIONSHIP:
Stress at home
Who is primary caregiver or who feeds the child?
How do the parents punish the child?
Financial problems
SAD
Any Psy Hx in either parent
PAST MH:
Here specifically ask about Osteogenesis Imperfecta
COUNSELLING:
I can see that you’re going through a difficult time. Sometimes it is challenging to work & care for a
child. From the history you gave me about the injury is not enough to explain such an injury. Children at

Please note that this is only a draft version based on several sources, including: Dr. Basel Mohasen’s lectures,
Toronto Notes, Therapeutic Choices and others. Edited and organized for the sake of all attendances of the
Canadian Osce Exams: NAC OSCE and MCCQE2. by: Dr. Merlyn D’Souza and Dr. Zeev Gross, Spring 2011. Page 287
this age have very flexible bones which are difficult to break by jumping off the couch. I’m sure you
share my concerns with me about the safety of your child & in this situation we contact the CAS. The
CAS will come & ask questions & talk to you & your partner:
If does not accept & says will take my child…….
Ask what makes you think like that? Do you have any experience about these matters?
It is not neccassarily,they will asses the situation & if the family is considered safe…
If pleads etc: Tell I’ve a legal responsibility to report to the CAS

Back to Content

Please note that this is only a draft version based on several sources, including: Dr. Basel Mohasen’s lectures,
Toronto Notes, Therapeutic Choices and others. Edited and organized for the sake of all attendances of the
Canadian Osce Exams: NAC OSCE and MCCQE2. by: Dr. Merlyn D’Souza and Dr. Zeev Gross, Spring 2011. Page 288
SUICIDE ATTEMPT
16/F suicide attempt ASA overdose
Sex:
AGe
Depression
Previous attempts
Ethanol use
Rational thinking loss
Suicide in family
Organized plan
No support (here put HEADSS)
Serious illness
INTRO:
As I understand,you’re here because you overdosed on Aspirin last night & I’ve been told that
you’ve been seen by my colleagues.My understanding is that you’re stable now,& I can talk to
you.
HOW DO YOU FEEL RIGHT NOW?
(if she is playing around with phone etc ask her to disconnect & speak to you)
Can you tell me more about what happened?
Pt: I went home & took Aspirin
Dr; why?
Pt: I was frustrated
Dr: Why?
I can see that you’re busy with your phone,is it important?
Can you stop for a few minutes?
I’d Like you to know that whatever you tell me is confidential here
Give confidientiality
Dr: Why are you angry?
Pt:I made a car accident
When?
Were you alone,or with someone else?
Were you driver or passenger?
Was anyone else hurt?
How Much aspirin?
Did you talke aspirin alongwith any other medication?
Did you lose consciousness?
Who called for help?
Did You seek help ort someone else did?
Was it IMPULSIVE or PLANNED?
Did you leave a note?
Have you recently been giving away your belongings to others?
Is it the First time?
Any suicidal attempt in the family?

Please note that this is only a draft version based on several sources, including: Dr. Basel Mohasen’s lectures,
Toronto Notes, Therapeutic Choices and others. Edited and organized for the sake of all attendances of the
Canadian Osce Exams: NAC OSCE and MCCQE2. by: Dr. Merlyn D’Souza and Dr. Zeev Gross, Spring 2011. Page 289
Do you usually take alcohol,or take drugs?
R What did you think about ending your life?
R/O Psychosis:
Sometimes when people want to end their lives they see a vision or hear voices,did you experience any
of such?
How is your mood, for the LAST 2 WEEKS (last 48 hrs make no difference)
HEADDSS
Do you have a driver’s liscence?
What made you take the car?
It seems an important trip, where did you go?
After you leave the hospital, what do you plan to do?
Finish SAD PERSONS
If parents separted ask about the other parent
If score <4 can send home
Nancy I know this is a stressful time for you.Based on the interview I think it is OK for you to go home.
Do you feel safe at home?
PT: Would you tell my mum?
Dr: Why can’t you tell her?
How is your relationship with your mother?
I do see your point,& it is better you tell her yourself,I can help you delivering the news.we can arrange a
meeting where a social worker be present & you can deliver the news.
Life is stressful & you’ve to learn how to handle these situations in the future.
I also want you to know in the future you may face a similar situation & if you ever feel like this
again,Please seek help
Call 911,go to the ER,Talk to your Family Physician

Back to Content

Please note that this is only a draft version based on several sources, including: Dr. Basel Mohasen’s lectures,
Toronto Notes, Therapeutic Choices and others. Edited and organized for the sake of all attendances of the
Canadian Osce Exams: NAC OSCE and MCCQE2. by: Dr. Merlyn D’Souza and Dr. Zeev Gross, Spring 2011. Page 290
ANOREXIA NERVOSA
Here inform parents as Pt lacks insight
* In Bulimia do not inform parents Pts can have Borderline personaliy disorder,Depression,Impulsive
behaviour or Kleptomaniac.
Here ther is Loss of Control, Large amount of food within 2 hrs & then feels guilty & induces vomiting
+ excessive exercise 2/week x 3 months
INTRO:
Hello,as I understand you’re here today as your parents are concerened about your weight
Can you tell me more about their concern?
What about you?
I’m glad you’re here today,as I can reassuare your parents
* WEIGHT
1. When did you start losing weight?
2. What was your weight when you started?
3. What was your highest weight?
4. What is your target?
5. When you look into the mirror,how do you perceive yourself?
6. Do you like to dress in baggy outfits?
7. Why do you want to lose weight? (Often carrer choice is of,Model,dancer or actress)
8. How do you plan to achieve that?
* DIET:
1. How’re your meals?
2. What snacks do you take?
3. Analyse the meals
4. How many calories?
5. Do you eat alone or with others?
6. At any time did you lose control & consume a large amount of food which is more than normal?
7. & how did you feel with that?
8. Did you try & compensate by exercise or purging?
* EXERCISE:
1. How long and often do you exercise?
2. What other activities are you involved in? E.g: dancing,walking
* Any other measures to lose wt like:
Water pills,medications like Xenical,Meredia,stool softeners?
* How has this IMPACTED your health?
1. No periods? For how long?
2. When was your last period?
3. Heart racing at night? Muscle cramping at night? (Hypokalemia)
4. Dry skin?
5. Constipation?
6. Fine hair on your body?
7. Pigmentation on your legs?
8. Any bone pain or fracture?
* MOAPPS
* HEADDDSSS
Back to Content
Please note that this is only a draft version based on several sources, including: Dr. Basel Mohasen’s lectures,
Toronto Notes, Therapeutic Choices and others. Edited and organized for the sake of all attendances of the
Canadian Osce Exams: NAC OSCE and MCCQE2. by: Dr. Merlyn D’Souza and Dr. Zeev Gross, Spring 2011. Page 291
22 /F Somatization disorder
4-2-1-1
If seen by a surgeon – suspect somatisation.
INTRO:
As I understand you’re here because you’ve abdominal pain for 3 weeks
Can you tell me more about it since it first started?
What did the surgeon tell you?
What investigations were done?
What Diagnosis was given?
Analyse pain TODAY
OCD
(PQRST) Non Specific
During the day or night.
U:How has it impacted your life?
How are you coping with It?
What happens at work
V:First time to have this pain or had it before
If BEFORE:
When
How often
Similar type
Seen by Dr?
Any Interventions?

CSx
Jaundice, white stool and dark urine
Foul smell, bulking, droplets in stool
Awakens at night?
Nx/Vx
Change in bowel movement?
MRI – why do you think it is important?
To differentiate: Somatic pain disorder / Somatization
In addition to abdominal pain: do you have:
Any other pain?
Headache, joints, back, pain with intercourse
With whom do you live?
Are you sexually active?
Any pain with IC
How is your interest in sex?
Do you feel interested after sexual activity?
NEUROLOGICAL:
Do you have tingling/numbness?
MOAPPS:
How is your mood?
Organic
Self Care? With whom do you live?

Please note that this is only a draft version based on several sources, including: Dr. Basel Mohasen’s lectures,
Toronto Notes, Therapeutic Choices and others. Edited and organized for the sake of all attendances of the
Canadian Osce Exams: NAC OSCE and MCCQE2. by: Dr. Merlyn D’Souza and Dr. Zeev Gross, Spring 2011. Page 292
Is your partner supportive?
How do you support yourself?
Any FH of suicide?

Back to Content

Please note that this is only a draft version based on several sources, including: Dr. Basel Mohasen’s lectures,
Toronto Notes, Therapeutic Choices and others. Edited and organized for the sake of all attendances of the
Canadian Osce Exams: NAC OSCE and MCCQE2. by: Dr. Merlyn D’Souza and Dr. Zeev Gross, Spring 2011. Page 293
39 YOF Pregnancy. Counseling. 10m

Hx=4mins
Counselling = 6mins

35yo – 1:180 (congenital) – half of them Down synd (1:360)


45yo – 1:45 (congenital) – 1:20 Down (5%)
 Always ask about concerns
 Why worried about Down’s
Take Hx: Age/FH/Ca/Cxt/RXT/Medications

Please note that this is only a draft version based on several sources, including: Dr. Basel Mohasen’s lectures,
Toronto Notes, Therapeutic Choices and others. Edited and organized for the sake of all attendances of the
Canadian Osce Exams: NAC OSCE and MCCQE2. by: Dr. Merlyn D’Souza and Dr. Zeev Gross, Spring 2011. Page 294
M: LMP + Sx of pregnancy +Pregnancy test
G
O: Imp
S
Past MH: Risk Fcts
FH: Congenital
EDUCATION:
CVS Amniocenthesis
Accuracy 97% 99.9%
Age 10-11 wks 14-16 wks
S/E: Limb defects --
Results: 48 hrs 2 weeks
Risk of abortion: 2% 0.5%
-- Checks for other
conditions

INTRO: As I understand you came here today, because you found out you were
pregnant last night & have requested an urgent meeting with me. What is the
reason?
Not ask if it is planned or not.
Ask if was on any contraception.
Confirm pregnancy by exact date.
What do you feel about the pregnancy?
What is your concern?
If she says it is her age or concern about Down;s
Ask: Any experience with Down’s syndrome?
Any Fhx of congenital anomalies?
It is a reasonable enough concern & I’m glad that you’re here. There are some
measures we can take to screen for some deformities, yet it is not 100%

How do you know you’re pregnant?


When did you find out?
LMP
Sx of pregnancy: Nausea/Vx/Breast tenderness/Inc visits to wash room to pee.
Prev pg?
Any contraception?
PAP’s smear?
STIs?

Please note that this is only a draft version based on several sources, including: Dr. Basel Mohasen’s lectures,
Toronto Notes, Therapeutic Choices and others. Edited and organized for the sake of all attendances of the
Canadian Osce Exams: NAC OSCE and MCCQE2. by: Dr. Merlyn D’Souza and Dr. Zeev Gross, Spring 2011. Page 295
FH of Cystic fibrosis
Did you discuss this with the father of the baby?
Any reason why not?
Past MH:
Medications
Chemotherapy
Radiotherapy Smoke/Alcohol/Drugs

COUNSELLING
If wants abortion,shift to abortion.There are some important figures you need to
know
Most of the time we’re concerned about Down;s SyndromeUsually the risk of
having a child with congenital. Abnormality at the age of 35 is around 1:180 and
half of these children are DS.
To r/o we do a test called amniocentesis, and the reason we offer it is because the
risk of complications of miscarriage is lower than the risk of congenital
abnormality. 0.5% (abortion) compared to 0.55% for DS.
If not willing then go to amniocentesis.
We can arrange for it. First we confirm pregnancy by US to get the exact date.
Then insert a fine needle guided by US into the womb & take sample of the fluid
surrounding your baby and send it to the lab. Usually we do it around 14-16w and
the results will come around 2w later. It is very accurate >99%.
Help us to r/o other conditions is very safe. Like neural tube defects
Risk of abortion is very low < 5%.
At the age of 35 the risk of having a miscarriage due to amniocentesis complication
is less than the risk of having a child with a congenital anomaly.
As always in medicine, we’ve to find a balance between risk & benefit.
If pt wants an earlier test.
Another option, not commonly done is Chorio venous Sampling
Here risk of abortion is 2%, & thus higher than amniocentesis.
The results are back in 48 hrs,however the chances of having a high false negative
is 3% when baby is still affected
Also there is a risk of limb injury
Genetic abnormalities: cystic fibrosis, thalasemia, etc. For that reason you should
be referred to a genetic counselling.

Please note that this is only a draft version based on several sources, including: Dr. Basel Mohasen’s lectures,
Toronto Notes, Therapeutic Choices and others. Edited and organized for the sake of all attendances of the
Canadian Osce Exams: NAC OSCE and MCCQE2. by: Dr. Merlyn D’Souza and Dr. Zeev Gross, Spring 2011. Page 296
From Mayo Clinic:
Amniocentesis is a procedure in which amniotic fluid is removed from the uterus
for testing or treatment. Amniotic fluid is the fluid that surrounds and protects a
baby during pregnancy. This fluid contains fetal cells and various chemicals
produced by the baby.
With genetic amniocentesis, a sample of amniotic fluid is tested for certain
abnormalities — such as Down syndrome and spina bifida. With maturity
amniocentesis, a sample of amniotic fluid is tested to determine whether the
baby's lungs are mature enough for birth. Occasionally, amniocentesis is used to
evaluate a baby for infection or other illness. Rarely, amniocentesis is used to
decrease the volume of amniotic fluid.
Although amniocentesis can provide valuable information about your baby's health,
the decision to pursue invasive diagnostic testing is serious. It's important to
understand the risks of amniocentesis — and be prepared for the results.
Before amniocentesis, you can eat and drink as usual. Your bladder must be full
before the procedure, however, so drink plenty of fluids before your appointment.
Your health care provider may ask you to sign a consent form before the procedure
begins. You may want to ask someone to accompany you to the appointment for
emotional support or to drive you home afterward.

During the procedure


First, your health care provider will use ultrasound to determine the baby's exact
location in your uterus. You'll lie on your back on an exam table and expose your
abdomen. Your health care provider will apply a special gel to your abdomen and
then use a small device known as an ultrasound transducer to show your baby's
position on a monitor.

Next, your health care provider will clean your abdomen with an antiseptic.
Generally, anesthetic isn't used. Most women report only mild discomfort during
the procedure.

Guided by ultrasound, your health care provider will insert a thin, hollow needle
through your abdominal wall and into the uterus. A small amount of amniotic fluid
will be withdrawn into a syringe, and the needle will be removed. The specific
amount of amniotic fluid withdrawn depends on the number of weeks the pregnancy
has progressed.

Please note that this is only a draft version based on several sources, including: Dr. Basel Mohasen’s lectures,
Toronto Notes, Therapeutic Choices and others. Edited and organized for the sake of all attendances of the
Canadian Osce Exams: NAC OSCE and MCCQE2. by: Dr. Merlyn D’Souza and Dr. Zeev Gross, Spring 2011. Page 297
You'll need to lie still while the needle is inserted and the amniotic fluid is
withdrawn. You may notice a stinging sensation when the needle enters your skin,
and you may feel cramping when the needle enters your uterus. The entire
procedure usually takes about an hour, although most of that time is devoted to
the ultrasound exam. In most cases, the fluid sample is obtained in less than two
minutes. The small amount of amniotic fluid that's removed will be replaced
naturally.

After the procedure


After the amniocentesis, your health care provider may use ultrasound to monitor
your baby's heart rate. You may experience cramping or a small amount of vaginal
bleeding immediately after the amniocentesis. Your health care provider may
suggest resting after the procedure. You may want to ask someone to drive you
home. You'll likely be able to resume normal activities the next day.

Meanwhile, the sample of amniotic fluid will be analyzed in a lab. For genetic
amniocentesis, some results may be available within a few days. Other results may
take one to two weeks. Results of maturity amniocentesis are often available within
hours.

If you develop a fever after amniocentesis or if vaginal bleeding, loss of vaginal


fluid or uterine cramping lasts more than few hours, contact your health care
provider.

Back to Content

Please note that this is only a draft version based on several sources, including: Dr. Basel Mohasen’s lectures,
Toronto Notes, Therapeutic Choices and others. Edited and organized for the sake of all attendances of the
Canadian Osce Exams: NAC OSCE and MCCQE2. by: Dr. Merlyn D’Souza and Dr. Zeev Gross, Spring 2011. Page 298
39 YOF high grade squamous endometrial ca.,
ASK HER ABOUT RISK FACTORS
In Lab work you always ask Why?
And is it the first time?
SPIKE
Explain
Local symptoms
CSx
MGOS
PMHx
Plan (colposcopy)

INTRO:
...because it is the first time I want to ask you some questions so as to get a better
understanding of your results.
Why? Is it the first time? When was it done?
Any reason prevented you from doing it?
What was your result at that time?
If done long ago?
Some people want to know in
―Are you the kind of person who prefers to know all the details about what is going
on?‖
―How much information would you like me to give you about your diagnosis and
treatment?‖
―Would you like me to give you details of what is going on or would you prefer that
I just tell you about treatments I am proposing?‖
If not anxious:
What do you know about Pap smear?
Yes, we look for changes in the cervix including cervical cancer
What do you know about Ca Cx?
It is a common cancer & we pick it up with Pap’s smear & if detected early, outlook
is good
What are your expectations of today’s visit?
The results are back & ―I wish I had better news for you but unfortunately it
shows you have some changes in the pap smears, & these changes are called ―
HGSIL‖.& these changes if Ca or not are not detected by PAPs smear. We need to
do further assessment to determine whether it is Ca or not.

Please note that this is only a draft version based on several sources, including: Dr. Basel Mohasen’s lectures,
Toronto Notes, Therapeutic Choices and others. Edited and organized for the sake of all attendances of the
Canadian Osce Exams: NAC OSCE and MCCQE2. by: Dr. Merlyn D’Souza and Dr. Zeev Gross, Spring 2011. Page 299
Let me ask you some questions to see if you have some symptoms related to it:
Local, Meta, Constitutional
LOCAL: Vaginal bleeding/Discharge/Ulcers/Blisters/Warts?
Pain with intercourse
Bleeding with intercourse
Lumps, bumps in groin area?
Fever,wt loss,Back pain?
MGOS
M:At which age you had your first period,
G: Any Gyn surgery
Contraception?
O: Have you been pregnant?
How many times?
At which age was your first pregnancy?
S: At what age were you sexually active?
How many partners did you have?
With whom do you live? How long have you been in this relationship?
Before this relationship,How many partners did you have?
STDs, Smoke,
How do you support yourself financially
Past Medical Hx
Family Hx
―I have bad news. The colposcopy result came back and consistent with cervical
cancer.
We need to take further steps & I will refer you to a gynaecologist.
If you want future babies they will use local options & do something called a Cone
biopsy
If you do not want any more children the uterus & cervix will be removed & the
prognosis is excellent

Back to Content

Please note that this is only a draft version based on several sources, including: Dr. Basel Mohasen’s lectures,
Toronto Notes, Therapeutic Choices and others. Edited and organized for the sake of all attendances of the
Canadian Osce Exams: NAC OSCE and MCCQE2. by: Dr. Merlyn D’Souza and Dr. Zeev Gross, Spring 2011. Page 300
Allergic Rhinitis, Counsel
Intro
OCD (seasonal: caused by pollens from trees. Summer, spring, early autumn – usually last several weeks,
disappears and recurs following year at the same time; Perennial: occurs intermittently for years with no
pattern or may be constantly present);
P: is the nasal congestion is only in one side (allergic rhinitis) or varies from side to side (vasomotor
rhinitis)
COCA-B (should be clear rhinorrhea, under microscope it contains increased eosinophils);
Watery/mucoid: allergic, viral, vasomotor, CSF leak (halo sign)
Mucopurulent: Bacterial, foreign body
Serosanguinous: Neoplasia
Bloody: Trauma, neoplasia, bleeding disorder, hypertension/vascular disease
ΑA:
Allergic rhinitis (hay fever): most common inhaled allergans - house dust, wool, feathers, foods,
tobacco, hair, mold; most common ingested allergans – wheat, eggs, milk, nuts;
Vasomotor rhinitis: caused by – temperature change, alcohol, dust, smoke, stress, anxiety, neurosis,
hypothyroidism, pregnancy, menopause,
Drugs: parasympathomimetic drugs and estrogens (OCPs, HRTs);
Beware of rhinitis medicamentosa: reactive vasodilation due to prolonged use (>5 days) of nasal drops
and sprays (Dristan, Otrivin)
ΑSx: Itching eyes with tearing, frontal headache and pressure, hypothyroid symptoms, change in
menstruation (pregnancy/menopause); MOAPS (especially – anxiety, neurosis, and drugs);
Complications: signs of sinusitis (pain in the face, post nasal drips, fever, severe headaches, teeth pain,
PMHx of sinusitis); Ask for diagnosis of nasal polyps or obstruction in breathing through the nose when
there is no sign of allergy; Ear pain (especially serous otitis media)
SHx:

Counsel:
From the Hx I’ve just taken it is most likely that you suffer from a condition called: allergic
rhinitis/vasomotor rhinitis. This condition is very common and is caused by exposure to irritants in the
environment which are called alergans. These alergans trigger the immune system to release substances
which cause the congestion in your nose. Finding and eliminating the appropriate trigger/s can prevent
this condition. For that reason I am sending you to do some allergy testing.
Meanwhile I can recommend several options to alleviate your symptoms.
For allergic rhinitis:
 Nasal irrigation with saline
 Spray, nasal drops, or tablets with antihistamines (e.g. diphenhydramine, fexofenadine)
 Oral decongestants (e.g. pseudoephedrine, phenylpropanolamine)
 I wouldn’t recommend to use topical decongestants since they may lead to a condition called
“rhitinitis medicamentosa” which may increase and deteriorate your condition. In case of necessity
– you may use a topical decongestant up to five days.
 There are many other medications that might help in case of serious condition – like steroids
(fluticasone), or for prevention (disodium cromoglycate), also ipratropium bromide. If very severe
oral steroids may be used.
 Desentization by allergen immunotherapy is also an option in some cases.
For vasomotor rhinitis:

Please note that this is only a draft version based on several sources, including: Dr. Basel Mohasen’s lectures,
Toronto Notes, Therapeutic Choices and others. Edited and organized for the sake of all attendances of the
Canadian Osce Exams: NAC OSCE and MCCQE2. by: Dr. Merlyn D’Souza and Dr. Zeev Gross, Spring 2011. Page 301
 Some relief can be achieved by exercise (increased sympathetic tone)
 Drugs that called parasympathetic blockers (e.g. Atrovent nasal spray)
 In serious conditions – steroids (e.g. beclomethasone, fluticasone)
 There are also some invasive procedures that might be used in stubborn cases. Surgery (which is
often with limited lasting benefit), electrocautery and cryosurgery which use hot or cold instruments
to affect the lining or your nose.
Overall this condition can be annoying but it is not dangerous and there are many ways to treat it.
However, in most cases it is repeated and the benefit of each treatment should be well balance against
its risks.

Back to Content

Please note that this is only a draft version based on several sources, including: Dr. Basel Mohasen’s lectures,
Toronto Notes, Therapeutic Choices and others. Edited and organized for the sake of all attendances of the
Canadian Osce Exams: NAC OSCE and MCCQE2. by: Dr. Merlyn D’Souza and Dr. Zeev Gross, Spring 2011. Page 302
Enuresis
Intro, ask about the concern
Ask about the name and age of the child
OCD - Analyze the problem:
Since when is he wetting his bed? Is it primary or secondary?
Does the child lose control on his bladder during day or only at night?
When did the child control his bladder and toilet?
How does the child feel about it? How do his caregivers feel about it?
R/O organic causes (red flags):
DM: Drinking too much, going more often to pee, feeling tired, lost weight
DI: Hx of meningitis, encephalitis (brain infection), head trauma
Seizure
UTI: Dysuria, odd smell or colour of urine
Neuro: Bowel dysfunction, leg weakness or numbness, trauma or surgery to back
Stress: Any stress or problem or new event
Others: Sickle cell disease, pinworms, constipation, and the most common cause for diurnal
dieresis is micturition deferral
PMHx – including medications (diuretics) and allergies
FHx
BINDE (briefly – because the child is 8-9 y.o)
How was the pregnancy (any problem)
How was the delivery (NVD vs. C/S)
Term or pre-term
Are his regular shots up to date?
How is his nutrition (does he eat well balanced diet)?
How is his school performance?
Who is the primary care giver? Who else live with them at home? Is he the only child?

Counsel
 Explain what is happening – say it is m/p regression of his development because of the
current stresses in his life
 It is caused by maturational lag in bladder control while asleep. It is self limiting and you
need to give the child some time and he will adapt very well to the changes. About 20%
of the children resolve spontaneously each year.
 The prevalence of this problem: 10% of 6 y.o, 3% of 12 y.o, 1% of 18 y.o
 Treatment by changing life style: limiting nighttime fluids and voiding prior to sleep,
engaging child using rewards, bladder retention exercises, scheduled toileting
 You can try a method called ―conditioning‖: ―wet‖ alarm wakes child upon voiding – this
method has 70% success rate
 As last resort you can try even medication: DDAVP by nasal spray or oral tablets, but
there is high relapse rate and it is costly. Other medical options: oxybutynin (Ditropan),
imipramine (Tofranil) – the latter is rarely used since it is lethal in overdose and has
cholinergic side effects.

Please note that this is only a draft version based on several sources, including: Dr. Basel Mohasen’s lectures,
Toronto Notes, Therapeutic Choices and others. Edited and organized for the sake of all attendances of the
Canadian Osce Exams: NAC OSCE and MCCQE2. by: Dr. Merlyn D’Souza and Dr. Zeev Gross, Spring 2011. Page 303
Important Drugs to Remember

Enuresis
Desmopresin 0.2-0.6mg at bed time

Torticulosis
Treatment: Diphenhydramine 50mg

Warfarin counselling
Enoxaperin 20mg OD (low risk)
Enoxaperin 40mg OD (high risk)

Want to stop Li
Normal level 0.5-1.2

Breast feeding counseling

Please note that this is only a draft version based on several sources, including: Dr. Basel Mohasen’s lectures,
Toronto Notes, Therapeutic Choices and others. Edited and organized for the sake of all attendances of the
Canadian Osce Exams: NAC OSCE and MCCQE2. by: Dr. Merlyn D’Souza and Dr. Zeev Gross, Spring 2011. Page 304
OSCE guide
Third edition
Table of contents

Table of contents ............................................................................................................................. 2


History taking – Medicine ............................................................................................................... 6
General review:............................................................................................................................ 7
Notes .......................................................................................................................................... 12
Chest pain – ACUTE ................................................................................................................. 14
Chest pain – CHRONIC ............................................................................................................ 16
Headache.................................................................................................................................... 18
Blood results / Macrocytic Anemia / B12 Deficiency................................................................. 21
Difficulty swallowing ................................................................................................................ 23
Elevated liver enzymes .............................................................................................................. 24
Drinking / Alcohol ..................................................................................................................... 26
Fever / Tired .............................................................................................................................. 28
Diarrhea – ACUTE .................................................................................................................... 29
Diarrhea – CHRONIC ............................................................................................................... 30
ASTHMA .................................................................................................................................. 32
COPD management ................................................................................................................... 35
Ankle swelling – Bilateral ......................................................................................................... 36
Ankle swelling – Unilateral ....................................................................................................... 37
Congestive heart failure – CHF ................................................................................................. 38
Heart racing ............................................................................................................................... 40
Fall ............................................................................................................................................. 42
Peripheral vascular disease: ....................................................................................................... 44
Urinary symptoms: .................................................................................................................... 45
Anuria ........................................................................................................................................ 46
Hematuria .................................................................................................................................. 47
Renal stones ............................................................................................................................... 48
Incontinence............................................................................................................................... 50
Lump – Neck Swelling .............................................................................................................. 51
Lump – Breast............................................................................................................................ 52
Dizziness.................................................................................................................................... 55
INR – Counselling ..................................................................................................................... 57
Patient is receiving blood – counsel for adverse reactions ........................................................ 59
Counselling – Ventilator............................................................................................................ 61
Ethical questions ........................................................................................................................ 62
HIV post-test counselling .......................................................................................................... 63
Lung Nodule .............................................................................................................................. 64
High Creatinine.......................................................................................................................... 66
Impotence / Erectile Dysfunction .............................................................................................. 67
Rheumatology – History Taking................................................................................................ 68
Multiple Sclerosis ...................................................................................................................... 69
Obesity....................................................................................................................................... 70
Epilepsy Counselling ................................................................................................................. 71
Medical note .............................................................................................................................. 73
Pre-diabetes – Counselling ........................................................................................................ 74
Emergency Medicine..................................................................................................................... 75
Emergency Room ...................................................................................................................... 76
Trauma ....................................................................................................................................... 77

OSCE-guide-III.doc Page 2 of 255


Unconscious Patient – Neuro..................................................................................................... 81
Unconscious Patient – Diabetic ................................................................................................. 82
Unconscious Seizing Patient – DT / Epilepsy / Brain tumour / …............................................ 84
Heart Attack – Chest Pain (MI or Heart Block) ........................................................................ 86
Case 1: Chest pain with initial normal ECG.............................................................................. 87
Case 2: Chest pain with STEMI ................................................................................................ 89
Case 3: Chest pain – v fibrillation / v tachy............................................................................... 90
Case 4: Chest pain – v fibrillation – intoxicated patient ............................................................ 90
Heart Block................................................................................................................................ 91
Headache.................................................................................................................................... 92
Acute Abdominal Pain............................................................................................................... 93
Upper GIT bleeding ................................................................................................................... 98
Lower GIT bleeding................................................................................................................... 99
ECG ......................................................................................................................................... 100
Phone calls ............................................................................................................................... 103
Physical Examination .................................................................................................................. 106
Medical Physical Exam ........................................................................................................... 107
Abdominal examination:.......................................................................................................... 109
Liver Examination ................................................................................................................... 111
Nasal bleeding  Hematological Examination....................................................................... 112
Chest Examination................................................................................................................... 114
Pneumonia ............................................................................................................................... 115
Cardiac Examination – Essential HTN case ............................................................................ 117
Secondary Hypertension .......................................................................................................... 119
Hypertension............................................................................................................................ 120
SOB – shortness of breathe...................................................................................................... 121
DVT ......................................................................................................................................... 122
Peripheral Arterial Disease Examination................................................................................. 125
Diabetic Foot ........................................................................................................................... 126
Neurological Examination ....................................................................................................... 127
Cranial Nerves Examination.................................................................................................... 129
Tremors.................................................................................................................................... 131
Thyroid Exam .......................................................................................................................... 132
Dermatomes ............................................................................................................................. 133
Neck Examination.................................................................................................................... 134
Carpal Tunnel Syndrome ......................................................................................................... 135
Hand Laceration / Wrist Laceration......................................................................................... 136
Back Pain ................................................................................................................................. 137
Acute Back Pain....................................................................................................................... 138
Chronic Back Pain ................................................................................................................... 139
Back Joint Examination ........................................................................................................... 140
Ankle Twist ............................................................................................................................. 141
Shoulder Joint .......................................................................................................................... 142
Elbow....................................................................................................................................... 143
Hip Joint................................................................................................................................... 144
Knee Joint ................................................................................................................................ 145
Obstetrics and Gynecology.......................................................................................................... 146
History taking – OB-GYN ....................................................................................................... 147
OB/GYN cases......................................................................................................................... 147
MGOS history questions:......................................................................................................... 148
History of pregnant lady – third trimester................................................................................ 149

OSCE-guide-III.doc Page 3 of 255


Vaginal Discharge.................................................................................................................... 150
Vaginal Bleeding – Non-Pregnant / Not-Known Pregnant...................................................... 151
Vaginal Bleeding – Pregnant / Ante-Partum Hemorrhage....................................................... 152
Abnormal Uterine Bleeding (AUB)......................................................................................... 153
Amenorrhea ............................................................................................................................. 155
Infertility .................................................................................................................................. 156
Counselling – pre-eclampsia.................................................................................................... 157
Caesarean Section – Counselling – wants to have CS............................................................. 159
Caesarean Section – Counselling – does not want to have CS ................................................ 161
OCPs / Contraception Counselling .......................................................................................... 163
HRT counselling...................................................................................................................... 166
Needle Stick Counselling – HIV ............................................................................................. 168
Counselling – PAP smear ........................................................................................................ 170
Antenatal Counselling.............................................................................................................. 172
Endometriosis .......................................................................................................................... 174
Woman wanting an abortion.................................................................................................... 175
Osteoporosis – Counselling / OR / Short Case ........................................................................ 176
Pediatrics ..................................................................................................................................... 177
History taking – Pediatrics....................................................................................................... 178
Jaundice ................................................................................................................................... 182
IUGR........................................................................................................................................ 184
Crying Baby............................................................................................................................. 185
Chronic Cough – Asthma ........................................................................................................ 186
Anemia..................................................................................................................................... 188
Vomiting .................................................................................................................................. 189
Diarrhea ................................................................................................................................... 191
Mother worried about her child weight ................................................................................... 193
Fever ........................................................................................................................................ 195
Runny Nose / Flu / URTI......................................................................................................... 196
Rash ......................................................................................................................................... 197
Delayed Speech........................................................................................................................ 198
Seizing child counselling ......................................................................................................... 199
ADHD counselling .................................................................................................................. 200
Vaccination counselling........................................................................................................... 202
Child with DM counselling...................................................................................................... 204
Bed wetting counselling / Nocturnal Enuresis......................................................................... 205
Breast feeding counselling....................................................................................................... 207
Psychiatry .................................................................................................................................... 209
Mental status exam – the psychiatry interview........................................................................ 210
DSM-IV-TR............................................................................................................................. 213
History taking – Psychiatry...................................................................................................... 214
Psychosis.................................................................................................................................. 216
Schizotypal personality disorder.............................................................................................. 218
Panic attack.............................................................................................................................. 219
Tiredness OR weight loss ........................................................................................................ 221
Sleep / fatigue notes................................................................................................................. 222
Insomnia .................................................................................................................................. 223
Domestic Violence – Spouse Abuse ........................................................................................ 224
Child Abuse ............................................................................................................................. 226
Domestic abuser....................................................................................................................... 228
Depression ............................................................................................................................... 230

OSCE-guide-III.doc Page 4 of 255


Depression management / counselling..................................................................................... 230
Dysthymia................................................................................................................................ 231
Premenstrual Dysphoric Disorder (PMDD)............................................................................. 232
Abdominal Pain / Headache .................................................................................................... 233
Somatoform disorders DD ....................................................................................................... 234
Drug seeker .............................................................................................................................. 236
Lithium discontinuity............................................................................................................... 238
Manic patient ........................................................................................................................... 240
Suicidal attempt ....................................................................................................................... 241
Eating disorder......................................................................................................................... 243
Mini-mental status exam:......................................................................................................... 245
Dementia.................................................................................................................................. 246
Delirium................................................................................................................................... 248
Case 1: Dad has not been himself / not sleeping well.............................................................. 248
Case 2: DT ............................................................................................................................... 249
Smoking Cessation – counselling ............................................................................................ 250
Refusal to treatment – counselling........................................................................................... 251
Truth telling ............................................................................................................................. 252
Organ Donation........................................................................................................................ 253
OCD......................................................................................................................................... 254
NOTES .................................................................................................................................... 255

OSCE-guide-III.doc Page 5 of 255


History taking – Medicine

History taking – Medicine

OSCE-guide-III.doc Page 6 of 255


History taking – Medicine

General review:

 Introduction:
 Chief complaint
 History of present illness
 Past medical history
 Family history
 Social history

Introduction

Chief complaint

TIME: Os Cf D
Analysis of
Character: PQRST
CC
↑ ↓
 Chronic diarrhea: dehydration
HPI Impact  Anemia: fatigue
 Cancer: metastasis
Constitutional symptoms
Red flags
Risk factors
Differential
diagnosis

A Allergy
M Medications
PMH P PMH: diseases (DM, HTN, heart attack, stroke, cancer)
L LMP / Last tetanus shot
E Events: hospitalization / surgery

Any long term disease


FH
Any specific disease

How do you support yourself financially?


SH With whom do you live?
SAD

OSCE-guide-III.doc Page 7 of 255


History taking – Medicine

Introduction:
- Knock the door
- Go to the examiner  give stickers  use alcohol rub (disinfective)
- Stand at the edge of the chair
- Good evening Mr …, I am Dr …, I am the physician in charge today,
o < 18 years: use first name
o > 18 years: use Mr / Mrs / Ms
- I understand that you are here because of …,
Examples:
- History Taking: Good evening Mr …, I am Dr …, I am the physician on duty now, and I
understand that you are here today because of …. In the next few minutes I will ask you some
questions about your cc, to figure out a working plan that can help you. If you have any
concerns or questions, please fell free to stop me and let me know.
- Physical examination: Good evening Mr …, I am Dr …; I am the physician in charge now. I
understand that you are here today because of …. In the next few minutes I will do a physical
exam on your (e.g. shoulder), during which I will ask you to do some movements that may
cause some discomfort and may be some pain, if you feel either, please do not hesitate to stop
me. And if you have any concerns, please let me know. And I will be telling the findings to
the examiner while we proceed.
1- Chief complaint
[A] If the CC not known
- How can I help you today?
1. Start to ask based on the age:
MALE FEMALE
> 65 yrs  Do you take meds on regular basis? Do you have a list of it? Or the
bottles? Do you take sleeping pills?
 Do you have difficulties with sleeping?
 Do you have difficulties with your balance1, any falls?
 Do you have difficulties with urination (incontinence / retention)?
 Do you have changes in your vision / hearing?
 Do you have changes in your mood / memory?
50s  Do you have problem drinking  use CAGE
 Depression2  identify through social history
 ED / Impotence
30s  Psychiatric problems
 SAD  social history
Teen / 20s  Premature ejaculation  Abortion
 STDs  STDs
 Eating disorders

2. Special conditions:
 Fatigue Domestic abuse
 Insomnia
 Headache
 Abdominal pain
 Vaginal bleeding

1
Normal pressure hydrocephalus: ataxia / incontinence / dementia
2
Common triad association: alcohol / depression / suicide

OSCE-guide-III.doc Page 8 of 255


History taking – Medicine

 Chest pain Panic attack


 SOB
 Heart racing
 Dizziness
 Numbness

[B] If the CC is known


CLEAR NOT clear
 Any pain: headache, chest pain  Vague symptoms: dizziness, tired
 Cough  When the patient uses medical terms:
 Fever abortion, jaundice, palpitation
 When the patient uses the words: change /
difficulty
Clarify: Clarify:
1. Start first open-ended questions: - Use closed ended questions
- What do you mean?
- Can you tell me more about this
2. Active listening:
- Do not interrupt
- Do not duplicate
3. Body language: nod your head

2- History of present illness


TIME (Os Cf D):
- Onset:
o How did it start (sudden / gradual)?
o Setting: what were you doing?
- Course: from that time till now, is your cc all the time or is it on and off (continuous vs.
intermittent)?
o Continuous:
 From the beginning
 Does it ↑ or ↓ or the same?
 Frequency: is it your first time?
o Intermittent:
 Frequency: how often did you have it in the last (…)?
 Are these attacks similar or different?
• ↑ in duration (longer) or ↑ in severity (more severe) or ↑ in
frequency (more often)?
 What brings it? On doing certain thing, occurring at rest, awakening you
from sleep?
- Duration:
o Usually given in the question
o If > 24 hours  empathy: were you able to sleep

Character:
+ PAIN: PQRST always ask “from the beginning?”
- Position: where did it start? Can you point with one finger on it?
- Quality: how does it feel like? Squeezing, tightness, sharp, stabbing, burning?
- Radiation: does it shoot anywhere?

OSCE-guide-III.doc Page 9 of 255


History taking – Medicine

- Severity: on a scale of 10, 1 being the mildest pain you have ever had and 10 is the most
severe, how much do you rate this one?
o If bad pain  empathy: this must be difficult
- Timing:
o Does it change with time; is it more in morning or towards the end of the day?
o Any variation?
- Triggers:
o What brings your headache?
o Is it related to: stress / lack of sleep / over sleep / flashing lights / smells?
o If female: is it related to your periods? Are you taking any meds or OCPs?
o Any diet triggers?

+ Fluids (e.g. bleeding, diarrhea, vomitus):


- COCA±B/D: colour, odour, contents/consistency, amount ± blood/discharge

What ↑ or ↓:
- What increases or decreases your cc?
- Examples: noise / quiet places / movements / resting / coughing / leaning forward / lying
down /

[D] ASSOCIATED SYMPTOMS:


- In addition to your cc, did you notice any other symptoms?
- Now, I am going to ask you more questions to see if you have any other symptoms beside
your cc.
+ By systems: e.g. chest pain
1- Same system
2- Near-by systems
3- Constitutional symptoms [RED FLAGS]
4- Risk factors
Now, I am going to ask you more questions to see if you have any medical conditions
that may explain / cause / predispose your cc.
5- Causes and consequences
6- Review of systems:
+ By differential diagnosis: e.g. headache
+ By causes and consequences: e.g. Macrocytic anemia – Vit B12 deficiency

3- Past medical history


Because it is the first time I see you, I need to ask you some questions about your past medical
history.
 Allergy / Medications:
a. Allergy
b. Medications (OTC, Rx meds, supplements, herbs)
 Past history of diseases for which you see doctor on regular basis (DM, high blood pressure,
heart attack, stroke, cancer)?
 LMP for females
 Events: any history of hospitalization / procedures?

4- Family history
Because it is the first time I see you, I need to ask you some questions about your family medical
history, and by this I mean your parents and siblings.

OSCE-guide-III.doc Page 10 of 255


History taking – Medicine

1- How do you describe their general health?


2- Any long term disease in the family? DM, HTN, heart attack, stroke?
3- Any specific disease runs in the family?

5- Social history
1- How do you support yourself financially?
2- With whom do you live?
a. Alone  are you in any relationship? Are you sexually active?
b. With a family  how is the relation with …? Is she/he supportive?
3- SAD:
a. Do you smoke?
b. Do you drink alcohol?
i. How much?
ii. For how long?
c. Have you ever tried recreational drugs?

OSCE-guide-III.doc Page 11 of 255


History taking – Medicine

Notes
EMPATHY:
If during history taking you noticed the patient is in pain  empathy: I can see you are in pain,
please bear with me for few minutes and I will give you a pain medication as soon as I can
- In the short cases (5 minutes)  use at least 1-2 empathy statement
- In the long cases (10 minutes)  use at least 3 empathy statements
- Patient says “I’m not ok / I’m not so good”  I am sorry to hear that
- Patient says “I fell down”  Oh, did you hurt yourself / “No” – I am glad to hear that
- After suicide  It looks like you have gone through difficult times, can you tell me more
about these difficulties you are facing
- Patient is regaining consciousness in the ER  Mr … you have had … and you are in the
hospital now, you are ok now, I am Dr … and we are here to make sure you’re ok
I have a concern!
Whenever the patient says: “I have a concern”: STOP the interview!
- Can you tell me your concerns!
- Why are you concerned?
Worried / occupied patient!
Whenever the patient shows non-verbal clues of being worried / occupied:
- I can see that you are worried / occupied! Would you like to tell me more about your worries
or concerns?
Question types:
- Types of questions you can use: open-ended, closed-ended, choices
- Types of questions you can NOT use: leading questions, stacking questions

Time usage technique: 1/2/3


If you are stuck during the history taking; and could not find questions to ask;
- First time: Summarize
- Second time: Ask about constitutional symptoms / Review of systems
- Third time: PMH / FH / SH
Medications:
When you ask about the medications and even if the patient says NO, in certain diseases, you
need to confirm special medications, by saying, what about …
- Diarrhea  what about antibiotics
- Asthma  aspirin / β–blockers (HTN, heart failure, thyroid disease, social phobias)
- Migraine  what about OCPs? (any birth control pills)
- Bleeding  what about aspirin / blood thinners
- Torticollis  do you take anti-psychotics? Do you see psychiatrist/ did he give you any
medications

MSD (mood / suicide / drinking): whenever you find one, screen for the others
When the patient comes with a chronic long duration complaint, ask him: and what
happened recently that made you decide to seek medical advice now?
Whenever the patient has something affecting his life / social issue: Refer to social worker
/ services
Do NOT criticize other doctors or the patient

OSCE-guide-III.doc Page 12 of 255


History taking – Medicine

For breaking bad news [SPIKES] approach:


o Setting
o Perception of the patient: what do you know about …
o Invitation: how much details you would like to know about …
o Knowledge: give knowledge in understandable pieces, and make sure the patient
understands this info.
o Empathy /+/ Expectations: what are your expectations from today’s visit
 You will have tremendous support, you are not working alone
 We will try to make you as comfortable as possible
 Oncology group AND palliative group: Referral: do you have any doctor you
are comfortable with?
o Summarize /+/ Strategy

Counselling:
1. Inform the patient
a. The medical condition is called “…”
b. Explain the pathophysiology
c. Consequences / complications of the condition! May happen again, may affect
ability to do certain things, …
d. Investigations that might be needed to conclude the condition OR to look for
complications
2. Preventive measures: e.g. modify the poly-pharmacy …
3. Treatment: life style / medications (side effects / alternatives / consequences of not
receiving treatment)
4. Offer more info: brochures / web sites / support groups
5. Break every 30-60 seconds (check & recheck that your patient understands); ask the
patient: does that make sense? Is this acceptable? Reasonable? Is it clear?
6. General tips for the counselling sessions:
- Make it interactive not lecturing
- At the beginning ask whether your patient has a specific concern
- Do not mislead your patient; if you are not sure about any thing, say that this is a very
good question and you are going to check the answer for him.
Whenever you hear “car accident”:
- I am sorry to hear that!
- Was anyone hurt? I am sorry for that
- Were you driving or a passenger?
If you do not know the answer to a question:
- This is a good question / point, I will check it for you and we will discuss it next visit.
- It is better to refer you to the specialist; there are too many points regarding this issue that it
will be better to discuss it with the specialist.
A good statement to use in different counselling situations: always in medicine, we balance
the benefits and the side effects.
GIT symptoms:
- Nausea / Vomiting - Heart burn / acidic taste in mouth
- Abdominal pain - Distension / bloating / gas
- Change in bowel movements: constipation / diarrhea
- Blood in stools / vomiting blood
- LIVER: yellowish discoloration / itching / dark urine / pale stools

OSCE-guide-III.doc Page 13 of 255


History taking – Medicine

Chest pain – ACUTE


Introduction: Good evening Mr …, I am Dr …, I am the physician on duty now, and I
understand that you are here because you have chest pain for the last … minutes. In the next few
minutes I will ask you some questions about your chest pain, to figure out a working plan that can
help you. If you have any concerns or questions, please fell free to stop me and let me know
Analysis of  OsCfD: Onset / setting: what were you doing?
CC  PQRST:
─ Position: where did it start? Can you point with one finger on it?
─ Quality: how does it feel like? Squeezing, tightness
─ Radiation: does it shoot anywhere? Your jaw, your shoulders, your back?
 What ↑ or ↓:
─ Breathing / position
─ How did you come to the clinic today? Ambulance  did they give you
aspirin / nitrates? Did it help you?
Impact  Atherosclerosis:
─ Hx of stroke? Symptoms of stroke (weakness / numbness / change in vision
/ difficulty finding words)?
─ Any sexual dysfunction?
─ Do you feel abdominal pain after eating?
─ Do you feel cold extremities
─ Do you feel cold feet? Pain after walking?
 CHF:
─ SOB? How many pillows do you use? Do you wake up gasping for air?
─ Any swelling in your LL? How high does it go? Is it related to position?
─ Eye puffiness? Pain on the liver?
Red flags Constitutional  Fever / night sweats / chills
symptoms  How about your appetite? Any weight changes?
 Any lumps or bumps in your body?
Risk factors CAD
Pericarditis
PE
Differential Same system  Nausea / vomiting
diagnosis  Sweating / feeling tired
 SOB  if yes, analyze (OsCfD)
 Do you feel your heart racing?
 Did you feel dizzy / light headedness / LOC? Are you tired?
 Did you notice swelling in your ankles? Legs? Calf muscles?
Near by  CHEST:
systems ─ Any cough or phlegm? Chest tightness? Wheezes?
─ Recent fever / flu like symptoms? Muscles/ joint ache?
 GIT:
─ Difficulty swallowing (esophageal spasm)
─ Heart burn / acidic taste in your mouth?
─ Any hx of PUD? Reflux? GERD?
 Chest wall: any trauma, any blisters / skin rash on your skin
 DVT: any pain / swelling / redness in your legs / calves? Any
recent long travel?

OSCE-guide-III.doc Page 14 of 255


History taking – Medicine

Risk Factors:
─ CAD (Coronary Artery Disease):
MAJOR:
o High blood pressure
o High blood sugar
o High cholesterol: have you got your cholesterol measured before?
o Family hx of heart attack at age < 50 yrs
o SAD: Smoking / Cocaine
MINOR:
o Look for obesity
o Do you exercise
o How about your diet, do you eat a lot of fast food?
o Are you under stress?
─ Pericarditis:
o Recent flu like symptoms
o Medications (Isoniazide / Rifampicin)
o Hx of surgery
o Hx of heart attack
o Hx of kidney disease / puffy face / frothy urine
o Hx of TB
o Hx of autoimmune disease
─ Pulmonary Embolism:
o Recent long flight
o History of malignancy
o Family history of blood clots
o Female: pregnancy / OCPs / HRT

Chest Pain
Acute Chronic
Minutes – hours Hours – days Intermittent Continuous
Cardiac: Cardiac: Cardiac:
- CAD - Pericarditis - Unstable angina
- Aortic dissection - Unstable angina
Non-cardiac: Non-cardiac - Cancer
- Tension - Pneumonia - Herpes zoster
pneumothorax - Pleurisy - Trauma
Panic attack Pulmonary embolism Panic attack
GIT: GIT:
- GERD - GERD
- PUD - PUD
- Esophageal spasm - Esophageal spasm

Questions:
 Investigations: ECG / Cardiac enzymes

OSCE-guide-III.doc Page 15 of 255


History taking – Medicine

Chest pain – CHRONIC


Chest pain for 6 weeks
UNSTABLE ANGINA /+/ GERD – GIT CAUSES OF CHEST PAIN

Intro … But first I would like to ask you, how do you feel now?
Analysis of  OsCfD: Onset / setting: what were you doing?
CC  PQRST:
─ Position: where did it start? Can you point with one finger on it?
─ Quality: how does it feel like? Squeezing, tightness
─ Radiation: does it shoot anywhere? Your jaw, your shoulders, your back?
 What ↑ or ↓:
─ Breathing / position
─ Is it related to activity? How many blocks were you able to walk? And
now?
─ How about rest? And during night?
─ When was the last attack
 Triggers Angina GERD
Exertion
  Golf (leaning forward)
Stress (emotional)
  Coffee / dairy products
Cold air
  Smoking / Alcohol
Heavy meals
  Heavy / late meals
Sexual activity
  Pregnancy (progestin)
Impact Effect  Atherosclerosis  Chronic cough
 CHF  Change in the voice
Red flags Constitutional  Fever / night sweats / chills
symptoms  How about your appetite? Any weight changes?
 Any lumps or bumps in your body?
Risk factors CAD
Differential Same system  Nausea / vomiting
diagnosis  Sweating / feeling tired
 SOB  if yes, analyze (OsCfD)
 Do you feel your heart racing?
 Did you feel dizzy / light headedness / LOC? Are you tired?
 Did you notice swelling in your ankles? Legs? Calf muscles?
Near by  CHEST:
systems ─ Any cough or phlegm?
─ Chest tightness? Wheezes?
─ Recent fever / flu like symptoms? Muscles/ joint ache?
 GIT:
─ Difficulty swallowing (esophageal spasm)
─ Heart burn / acidic taste in your mouth?
─ Any hx of PUD? Reflux? GERD?
 Chest wall: any trauma, any blisters / skin rash on your skin
 DVT: any pain / swelling / redness in your legs / calves? Any
recent long travel?
PMH
FH
SH

OSCE-guide-III.doc Page 16 of 255


History taking – Medicine

Counselling:

Concern: The patient has a concern; is this IHD? Is his heart endangered?
─ This is quite a reasonable concern? What made you think about that?
─ Especially you have many risk factors that may predispose to heart attack. Right now the
physical exam is ok; it is less likely your condition is due to heart problem. But we still
need to check your heart more, we will do some lab works and an electrical tracing for
your heart (ECG), then if we find that we still need, we may send you to have a stress
ECG, in which, we trace your heart while you are exercising. Then we know for sure the
condition of your heart.
─ However, we would like to take measures to try to decrease your risk of developing heart
attack, e.g. exercise / diet / smoking / cholesterol.
─ On the other hand, the most likely diagnosis of what you have is a medical condition
called “GERD”. GERD stands for Gastro-Esophageal Reflux Disease, any idea about
that? Do you know anything about GERD?
─ Explain with a drawing: the esophagus (food pipe) / lower esophageal sphincter /
physiologic mechanism to keep it competent / in GERD  weak sphincter  acid
refluxes / irritates the esophagus / impact (short term and long term)
─ Treatment:
o Avoid triggers
o Life style modifications:
 Raise the head of the bed
 Smaller meals
 Do not eat late
 ↓ smoking
o Medications: proton pump inhibitors (PPIs), e.g. pantoprazole
o Side effects of PPIs:
 In general, proton pump inhibitors are well tolerated, and the incidence of
short-term adverse effects is relatively uncommon
 Common adverse effects include: headache (in 5.5% of users in clinical
trials), nausea, diarrhea, abdominal pain, fatigue, and dizziness. Long-term
use is associated with hypomagnesemia
 Because the body uses gastric acid to release B12 from food particles,
decreased vitamin B12 absorption may occur with long-term use of proton-
pump inhibitors and may lead to Vitamin B12 deficiency
 Infrequent adverse effects include rash, itch, flatulence, constipation, and
anxiety

OSCE-guide-III.doc Page 17 of 255


History taking – Medicine

Headache
HPI:
─ OsCfD: gradual onset / all the time / increasing / for few days
─ PQRST: temporal area / vague deep pain / severe
o Severe:  empathy: this must be difficult, were you able to sleep
o Triggers:
 What brings your headache?
 Is it related to: stress / lack of sleep / flashing lights / smells / diet?
 If female: is it related to your periods? Are you taking any OCPs?
─ What ↑ or ↓? Lying down / coughing / resting in quiet room / …

Associated symptoms / differential diagnosis:


1- Infection:
─ Fever / night sweats / chills / constitutional symptoms
─ Neck pain / stiffness
─ Recent flu like symptoms / skin rash
─ Bothered by light
─ Nausea / vomiting

2- Subdural hematoma:
─ Trauma / fall
─ SAD (Smoking, Alcohol, Drugs)

3- Subarachnoid hemorrhage:
─ Very acute /+/ Very severe headache / the worst headache
─ History of aneurysm or polycystic kidney disease
─ Visual changes (pupil changes)
─ Your heart is beating slow

4- Neurological screening:
If while you are doing the neurological screening, you suspect particular cause, e.g.
temporal arteritis  go to TA block then return to complete the neurological screening.
─ Cranial nerves:
o Any change in smelling perception?
o Any difficulty in vision / vision loss?
o Any difficulty in hearing / buzzing sounds?
o Difficulty finding words? Aphasia?
o Difficulty swallowing?
─ Brain:
o Any dizziness / light headedness / LOC?
o Any tremors / jerky movements / hx of seizures?
─ Personality and cognition:
o Any memory / mood / concentration problems?
o Did anybody tell you that you there is a change in your personality recently?
─ UL/LL:
o Any weakness / numbness / tingling in your arms / legs
o Any difficulty in your balance / any falls?
─ Spine:
o Any difficulty with urination / need to strain to pass urine?
o Any change in bowel movements?

OSCE-guide-III.doc Page 18 of 255


History taking – Medicine

5- Temporal arteritis:
─ Age > 55 years
─ When you touch this part of your head, is it painful? Can you comb your hair?
─ Do you feel cord-like structure?
─ Do you have any visual disturbances / impairment?
─ When you are chewing, is it painful, cramps in your jaws?
─ Any weakness / numbness in your shoulders / hips?
─ Is there any cough? Mild fever?
6- HTN:
─ Were you diagnosed before with high blood pressure?
─ Do you know your blood pressure? Have you had it checked before?
─ Salty food? Family history of HTN / heart disease?
─ Any history of repeated headaches?
7- Extra-cranial causes of headache:
─ Eyes: any hx of glaucoma, red eye, pain in your eyes? Do you usually wear eyeglasses?
Do you see well? Any vision problems? When was last time you saw your optometrist?
─ E – do you have any pain / discharge in your ears?
─ N – nasal discharge / sinusitis / hx of facial pain?
─ T – any teeth pain / difficulty swallowing?
8- Medications:
─ Do you take any nitrates?
─ Do use too much of advil (or other NSAIDs)? For how long?
─ Were you used to take large amounts of coffee and then you stopped abruptly?
─ OCPs?

Temporal Arteritis:
Investigations: Treatment: If suspect GCA (Giant Call Arteritis),
─ TA biopsy immediately start high dose prednisone; 1 mg/kg
─ Doppler OD (to prevent blindness) then maintain dose daily
─ ESR (in divided doses), then taper prednisone dose
─ CT head after symptoms resolve.

Polymyalgia Rheumatica:
─ Constitutional symptoms + Fatigue Treatment: Corticosteroids; 15 mg/day (for long
─ Age > 50 yrs periods of time). Taper after ESR decreases < 50
─ ESR > 50 mm/hr mm/hr and stop if ESR normalizes (< 20 mm/hr)

OSCE-guide-III.doc Page 19 of 255


History taking – Medicine

PRIMARY HEADACHE

Intermittent / episodic
Headache Tension Migraine Cluster
Duration Days Hours Minutes
Quality Pressing / tightening / Mostly unilateral / Comes in series / severe
bilateral pulsating / interferes with pain / hyperaesthesia
daily activities
Place Band around the head Mostly unilateral Around the eyes / nose
Associated Photophobia / phonophobia Red eyes / lacrimation /
symptoms rhinorrhea / sweating
Aggravating Stress Physical activity / motion Smoking / alcohol
factors Physical injury Light / sound Smell / exercise
Others  Family history
 Types:
+ Classical: with aura
+ Non-classical: no aura
Treatment Acute phase:
- Acetaminophen - Acetaminophen - Oxygen
- NSAIDs - NSAIDs (ibuprofen) - NSAIDs
- Physiotherapy / ms - Triptans / ergotamine
massage / heat Prophylactic:
compresses (neck) - Remove precipitant
- Ca channel blockers
- Triptans (somatriptan)

OSCE-guide-III.doc Page 20 of 255


History taking – Medicine

Blood results / Macrocytic Anemia / B12 Deficiency

Cases:
─ Middle age man received blood report showing Macrocytic anemia
─ Elderly (65 years old) man presenting with ataxia, dizziness, macrocytic anemia.
Findings: poor diet. Most likely diagnosis: pernicious anemia

Investigations:
─ CBC / Differential / Peripheral blood film
─ B12 level in the blood / Folic acid level in the blood

Introduction
Good morning Mr …, I am Dr …, I understand that you are here today (OR we called you to
come) to get the results of your blood tests (OR x-ray) that you have done few days ago, I have it
and I am going to discuss it with you. However, because it is the first time that I see you, I need
first to ask you some questions to help me get better understanding and interpretation of these
results. Is it OK with you?

+ If patient anxious about results  tell her/him then continue history

1- First let me ask you few questions about the lab test itself (this applies to any blood work, x-
ray, HIV testing, biopsy, jaundice, anemia):
─ Why have you done this test?
─ Is it the first time to have it?
─ Who ordered this test for you? Why?
─ When did you have it?

2- Give the information:


─ If it is bad news  SPIKES
─ If abnormal blood results or x-ray: explain the results to patient, to decrease the patient
anxiety. The blood tests you had show that you have special kind of anemia that we call
“Macrocytic anemia” in which the size of the red blood cells (which are a component of
your blood) is larger than usual, there is different causes for this.
o If the patient panicked? Is it serious doctor?
 Do you have any concerns?
 There are different causes that may lead to this result, some of them are
serious, could be, we need to do more investigations.

3- I would like to ask you some questions to see how did this (anemia) affect you:
CONSEQUENCES of anemia:
─ Anemia symptoms:
o Did anyone comment that you are pale, recently?
o Did you notice any ↓ in your activity level?
o Heart racing / SOB / chest pain with exercise?
o Any dizziness / light headedness / fainting?
─ Neuro symptoms:
o Any tingling / numbness / in your feet?
o Difficulty in your balance / any falls?
o Any difficulty concentrating / memory problems?

OSCE-guide-III.doc Page 21 of 255


History taking – Medicine

4- I would like also to ask more questions to find out what might be the cause:
CAUSES of Vit B12 deficiency:
─ Diet intake: Are you vegetarian? For how long? Do you take supplements?
─ Gastric causes:
o Did you have any surgeries in your stomach? When?
o History of long standing PUD? Any heaviness / fullness after meals /
indigestion? (Lack of acidity)
o Were you ever yourself or any member of your family diagnosed with what is
called “autoimmune disease”; by this I mean a condition called “pernicious
anemia”, or rheumatoid disease / lupus?
─ Terminal ileum:
o Did you have any bowel surgery before?
o Were you diagnosed with “Crohn’s disease” before? Any repeated attacks of
diarrhea? Any foul smelling bulky stools?
─ Pancreatic and liver failure:
o Any hx of liver / pancreatic disease?
o Yellowish discoloration / itching / dark urine / pale stools?
─ Alcohol:
o Do you drink alcohol? How much? For how long?
─ Meds:
Do you take medications on regular basis? What kind?
o Have you ever been diagnosed with epilepsy? Do you take anti-epileptics?
o Do you see a psychiatrist? Do you take a mood stabilizer?
o Any hx of chemotherapy? Have you ever taken a drug called “methotrexate”?
─ Hematological causes:
o Any recent bleeding (nose / gum / coughing / vomiting blood)? Any bruises /
dark urine / tarry stools?
o Any fever / night sweats / chills? Change in appetite / weight loss? Lumps and
bumps in your body (for LNs)? Bony pains? Any repeated infections?
─ Parasites:
o Have you ever consumed raw fish (chronic intestinal infestation by the fish
tapeworm: Diphyllobothrium)?

5- PMH
6- FH
7- SH

OSCE-guide-III.doc Page 22 of 255


History taking – Medicine

Difficulty swallowing
What do you mean by difficulty swallowing?
─ Do you feel difficulty initiating the swallowing?
─ Do you feel pain when you swallow?
─ Do you feel food is stuck? Can you point where it is usually stuck?
Dysphagia
(esophageal)

Progressive Intermittent
All the time and ↑ On and Off Achalasia:
respiratory
symptoms

Progressive, solids Fluids first Fluids and solids Solids only Scleroderma:
then fluids Then solids (Large bolus) reflux / tight skin of
fingers / change
hand color when
exposed to cold
(Reynaud’s
Mechanical  Achalasia Esophageal spasm Esophageal webs and
disease)
Cancer OR stricture  Scleroderma rings

Mechanical Dysphagia:
Analysis  OsCfD: gradual, ↑ progressively, to solids then fluids / PQRST / What ↑ / ↓
of CC  Associated symptoms:
─ The same system:
o Nausea / vomiting / undigested food
o Change in bowel movements
o Change in the size of the abdomen / abd pain / blood in stools
o Liver: yellowish discoloration / itching / dark urine/ pale stools
─ Near-by systems:
o Any chest pain / tightness
o Any cough / change in your voice / neck swelling (thyroid lump)
Impact Weight loss
Red flags  Constitutional symptoms: fever/ night sweats/ chills / change in appetite / loss
of weight / lumps & bumps
 Risk factors:
─ GERD / PUD:
o Hx of heart burn
o Were you ever diagnosed with a condition called GERD / PUD
o Have you ever checked with a camera or a light (endoscope)
inserted into your stomach
─ Smoking / Alcohol
─ Family history: esophageal cancer
─ Radiation to chest
─ Have you ever swallowed any chemical?
Barium swallow: string sign /or/ apple core sign / graded narrowing of intra-esophageal diameter
extending from T5-T8 level  most likely diagnosis: esophageal cancer
Investigations: endoscopy and biopsy / chest x-ray and CT / liver function tests / abdominal US

OSCE-guide-III.doc Page 23 of 255


History taking – Medicine

Elevated liver enzymes

Introduction
HPI:
1- First let me ask you few questions about the lab test itself (this applies to any blood work, x-
ray, HIV testing, biopsy, jaundice, anemia):
─ Why have you done this test?
─ Is it the first time to have it?
─ Who ordered this test for you? Why?
─ When did you have it?

2- Give the information:


─ Liver enzymes: AST / ALT ≥ 2
─ Explanation: there is increase in one of the markers used to assess / check the liver
functions, it may indicate that there is an injury to your liver, I am happy you came here
today to discuss it so that we can figure this out.

3- I would like to ask you some questions to see how did this affect you:
CONSEQUENCES of liver injury:
─ Acute phase:
o Any yellowish discoloration / itching / dark urine/ pale stools
o Recently, have you noticed any fever / flu-like symptoms / muscle/joint aches
o Constitutional symptoms: sweats / chills / appetite / weight / lumps
─ Chronic manifestations:
o Did you notice any increase in the size of your abdomen? Puffiness in your face?
Swelling in your legs/ ankles?
o Bruises in your body?
o Vomited blood? Blood in stools?
o WITH ALCOHOL: did you notice changes in memory and concentration? Any
weakness / numbness? Balance and falls?

4- I would like to ask you more questions to find what might be the cause:
CAUSES of liver injury:
Now, I would like to ask you some questions to see if you were exposed to liver disease
without being aware of that, some of these questions may be personal, but it is important to
ask (start from least offensive to most offensive)
Including the alcohol, during which  Drinking assessment

5- PMH:
─ Were you ever diagnosed with liver disease before
─ Were you ever checked for liver disease before
─ Were you ever vaccinated for liver disease before
6- FH: suicide / depression / drinking / liver cancer
7- SH

OSCE-guide-III.doc Page 24 of 255


History taking – Medicine

Now, I would like to ask you some questions to see if you were exposed to liver disease without
being aware of that, some of these questions may be personal, but it is important to ask (start
from least offensive to most offensive)

Oral
─ Any recent travel outside Canada?
─ Did you eat any raw shell fish? Did you eat in new place that you are not used to?

Surgical
─ Any history of surgeries / hospitalization?
─ Any history of blood transfusion?
─ Any history of blood donation?

Social
─ How do you support yourself financially? If hazardous occupation?
─ Did you get exposed to blood products / body fluids?

Risky behaviour
─ Any tattooing or piercing?
─ SAD?
o Do you smoke?
o Drink alcohol? How about the past?  Drinking assessment
o Have you ever tried recreational drugs? Any injected drug use? When was the
last time?
─ With whom do you live? For how long have you been together?
─ Before being with your current partner, did you have other partners?
─ When did you start to be sexually active? How many sexual partners did you have from
that time till now?
─ What is your preference, men, women or both?
─ What type of sexual activity do you practice? Did you practice safe sex all the time? And
by that I mean using condoms!
─ Any history of sexually transmitted infections? And screening for STIs?
─ Have you ever had sex with sex worker?
─ Within the last 12 months, have you had any other sexual partners?

When do you need to take extensive Liver investigations


sexual history? Risky behaviour!
 Liver enzymes / Jaundice  AST / ALT / GGT / Alkaline phosphatase
 Fever / Tired  LDH
 LNs  Bilirubin
 Discharge  INR / PTT
 PAP results  Albumin
 HIV test results  Glucose
 Serum ferritin / TIBC / serum ceruloplasmin
 Viral serology: Hep A/B/C antibodies and
Hep B Ag
 Abdominal US
 Liver biopsy

OSCE-guide-III.doc Page 25 of 255


History taking – Medicine

Drinking / Alcohol

Drinking assessment

Use / abuse MOAPS Impact


Medical Social Legal
Drinking hx Mood Liver Home
How much? Organic Memory / conc. Work
CAGE Anxiety B12
Psychosis Heart
Self-care / suicide

Use / Abuse:
─ Do you drink alcohol? How about the past?
─ What do you drink?
o For how long?
o How often?
─ How much?
o 2 bottles of wine a day? 12 beers a day? Have you drunk more than 6 drinks in
one setting? Have you ever exceeded the amount you intended to drink?
o Do you drink alone or with other people?
o Did you ever drink to the extent of black out?
o What do you feel if you do not drink? Any shaking / heart racing / sweating?
Have you ever had seizures before? Were you hospitalized? Did you have
delirium tremens?
o Do you avoid going to places where you do not have access to alcohol?
─ CAGE:
o Did you ever think that you need to cut down on your drinks?
o Do you get annoyed by other people criticizing your drinking?
o Do you feel guilty for your drinking habits?
o Early morning drink?
Problem drinking: 2 of CAGE list for males OR 1 for females

MOAPS:
─ Mood:
o How is your mood? Interest? If ok  do not proceed
o If not ok  MI PASS ECG
─ Anxiety:
o Are you the person who worries too much?
o Do you have excessive fears or worries?
─ Psychosis:
o Do you hear voices or see things that others do not?
o Do you think that someone else would like to hurt you?
─ Self care / suicide
o Any chance you might harm yourself or somebody else?

OSCE-guide-III.doc Page 26 of 255


History taking – Medicine

Impact:
I am going to ask some questions to check what effects does alcohol have on your life?
─ The medical is already done in the consequences of liver injury
─ Social:
o With whom do you live? For how long? How is the relation? Is there any
problems? Is it related to your drinking habits?
o How do you support yourself financially? Where do you work? For how long?
 How is the relation with your coworkers / manager?
 Do you miss working days because of your drinking habits?
 Do you need to drink at work?
─ Legal:
o Did you have any legal issues related to your drinking?
o Fights? Arguments? Were you arrested before because of drinking?
o Were you charged before for DUI (driving under influence)?

Alcoholic beverages:
─ Beer:
o Alcohol percent around 5%
o Pitcher (60 oz) = 3 pints (pint = 20 ounces)
o Ounce (oz) = around 28-30 ml
─ Wine / Champaign:
o Alcohol percent around 12%
o Bottle: 750 ml
o Glass: 150 ml
─ Hard liquor (whisky / gen):
o Alcohol percent around 40%
o 1 glass (shot) = 1 ½ oz (50 ml)

Ethical challenges (patient asks)


Doctor is there any possibility that these test results are wrong? Can we repeat it to make
sure?
 Whenever we do blood work, we take a lot of measures to make sure it is accurate and
usually if it is positive, the labs double check it before they send it. Chances of having a
mistake are very low.
 Based on your history, symptoms and examination, we might not only repeat the test, we
might need to do more tests. It is early to tell now, let us proceed with history and
examination, and at the end I will be able to tell you.

Do you drink yourself, doctor?


 Why do you ask?
 Whether I drink or no does not matter, it is better to discuss your case now.

By the way, are you Canadian graduate, doctor?


 I fulfill all the requirements to practice medicine in Canada!
 I passed all the exams, follow Canadian guidelines, and practice under supervision!

OSCE-guide-III.doc Page 27 of 255


History taking – Medicine

Fever / Tired

Intro … But first I would like to ask you, how do you feel now?
CC  Fever
 Do you have other concerns?
Analysis OsCfD
 Did you measure it? How often? How? What is highest?
 And medications? Did it help?
 Any flu / illness / sickness
 Any diurnal variation? Any special pattern? Is it more every 3rd or 4th day?
(malaria)
Impact Are you able to function?
Red flags Constitutional symptoms
Differential  CNS: headache / neck pain / stiffness / nausea / vomiting / vision changes /
diagnosis bothered by light / weakness / numbness
 ENT:
Extensive ─ Ears: pain / discharge
review of ─ Nose: runny nose / sinusitis (facial pain)
systems ─ Throat: sore throat / teeth pain / difficulty swallowing
 Cardiac: chest pain / heart racing (pericarditis)
 Lung (pneumonia, PE (DVT), TB, cancer): cough / blood / phlegm / wheezes /
chest tightness / contact with TB pt
3
 GIT (except the liver ): abd pain / distension / change in bowel movements /
blood in stools
 Urinary: burning / frequency / flank pain / blood in urine
 Do you have any discharge? Ulcers? Blisters? Warts?
 MSK: joint pain / swelling / ulcers in your body / mouth / skin rash / red eye
 Autoimmune: fm hx / dx before with autoimmune dis
 The LIVER:
─ Local: yellow / itching / dark urine / pale stools
─ Dx before with liver dis? Screened? Vaccinated?
─ Transition to risky behaviour
PMH Cancer / Autoimmune disease
FH Cancer / Autoimmune disease
SH Does your partner have any fever? Discharge? Skin rash?

 During the last 6 weeks, did you seek medical attention?


 What made you choose to come today? Compared to before, any special changes?

3
The liver will be put at the end as a transition to ask about risky behaviour (see liver enzymes case)

OSCE-guide-III.doc Page 28 of 255


History taking – Medicine

Diarrhea – ACUTE

Analysis of OsCfD  How many times?


CC COCA  What bout during night?
± B/Mucous ─ Yes  organic
↑↓ ─ No  irritable bowel syndrome (IBS) – day only
─ How does if affect your sleep?
 Consistency: watery / loose / formed / bulky. Any floating
fat droplets / difficult to flush / undigested food
 Did you notice blood? When did it start?
─ Before you have your bowel move?
─ Mixed (higher source of bleeding)?
─ On the surface?
AS  Pain  OCD / PQRST
─ If pain improves after bowel movement: IBS
 Vomiting
 Alteration with constipation
Impact  Acute dehydration: thirsty / dizziness / light headedness / LOC / weak
Red flags  Constitutional symptoms – for infection / cancer
 Flu like symptoms
 Any body around you have the same diarrhea?
Differential  Other causes:
diagnosis ─ Hyperthyroidisms: heat intolerance
─ Stress? What do you do for life? Any stress? Does the diarrhea ↑ with
stress? How about your mood?
─ Infectious: travel / camping / with whom do you live? Any other person
at home with diarrhea?
─ HIV – if risk factors
─ Diet: Celiac disease / a lot of dairy products / lactose intolerance / lots of
juice / sugars
─ Medications: antibiotics / stool softeners
 Rheumatic diseases: red eyes / mouth ulcers / skin changes/ rash / nail
changes / hx of psoriasis / joint pain / swelling / back pain / stiffness
(especially in morning) / discharge / renal stones
PMH
FH
SH

Bloody diarrhea DD:


─ GE (gastroenteritis)
─ IBD (inflammatory bowel disease)
─ Bleeding peptic ulcer

Investigations for clostridium difficile Treatment for clostridium difficile


 CBC / differential / lytes and chemistry  Stop the antibiotic
 Stool culture for parasites  Metronidazole (500 mg tid x 7 days)
 Stool assay for clostridium toxin  If metronidazole is not effective or severe
 Endoscopy case  vancomycin (125 mg qid x 14 d)
 Blood grouping and cross matching

OSCE-guide-III.doc Page 29 of 255


History taking – Medicine

Diarrhea – CHRONIC

The same as acute diarrhea, except the impact and red flags

Introduction
CC
Analysis of OsCfD  How many times?
CC COCA  What bout during night?
± B/Mucous ─ Yes  organic
↑↓ ─ No  irritable bowel syndrome (IBS) – day only
─ How does if affect your sleep?
 Consistency: watery / loose / formed / bulky. Any floating
fat droplets / difficult to flush / undigested food
 Did you notice blood? When did it start?
─ Before you have your bowel move?
─ Mixed (higher source of bleeding)?
─ On the surface?
AS  Pain  OCD / PQRST
─ If pain improves after bowel movement: IBS
 Vomiting
 Alteration with constipation
Impact  Acute dehydration: thirsty / dizziness / light headedness / LOC / weak
 Chronic  weight loss
Red flags  Constitutional symptoms – for infection / cancer
 For cancer: Age / family hx of Ca colon / change in the calibre of stools /
what kind of diet
Differential  Rheumatic diseases: red eyes / mouth ulcers / skin changes/ rash / nail
diagnosis changes / hx of psoriasis / joint pain / swelling / back pain / stiffness
(especially in morning) / discharge / renal stones
 Other causes:
─ Hyperthyroidisms: heat intolerance
─ Stress? What do you do for life? Any stress? Does the diarrhea ↑ with
stress? How about your mood?
─ Infectious: travel / camping / with whom do you live? Any other person
at home with diarrhea?
─ HIV – if risk factors
─ Diet: Celiac disease / a lot of dairy products / lactose intolerance / lots of
juice / sugars
─ Medications: antibiotics / stool softeners
PMH
FH
SH

Rheumatic diseases: IBS / ankylosing spondylitis / psoriasis / reactive arthritis

OSCE-guide-III.doc Page 30 of 255


History taking – Medicine

Counselling:
─ Explanation:
o From what you have told me, the most likely explanation for your diarrhea is the
medical condition known as “Irritable Bowel Syndrome”, it is like “unhappy colon”
o What do you know about IBS?
o We do not know the exact mechanism behind this disease, and it is a common
condition, a lot of people have it, this is a long term disease, but it is treatable.
o What I need to do is to do physical exam, and do some blood works and stool
analysis to rule out other causes, how do you think about that?
o Is it serious condition doctor?
 It is not serious, as it does not affect life expectancy, and around 80% of
patients improve over time
─ Management:
o Psychotherapy:
 Establish good relationship with the patient
 CBT (cognitive behavioural therapy)
 If mood is low  depression counselling, it might be a mood problem
o Life style modifications:
 Stress management and relief
─ Relaxation techniques such as meditation
─ Physical activities such as yoga or tai chi
─ Regular exercise such as swimming, walking or running
 Diet modification: lactose-free diet or a diet restricting fructose is sometimes
recommended
 If drinks too much alcohol  advise to decrease alcohol
o Medications
 Abdominal pain:
─ Hyoscyamine (antispasmodic): 0.125 to 0.25 mg PO or SL q4h or PRN
/OR/ extended-release tablets: 0.375 to 0.75 mg orally every 12 hours
(do not exceed 1.5mg in 24 hours)
─ Amitriptyline (10 mg qhs)
 Diarrhea:
─ Imodium up to 8 tab / day
─ Lomotil
 Constipation:
─ ↑ fibre content in diet
─ Metamucil (psyllium): bulk-producing laxative and fibre supplement
 SSRIs
o Alternative medicine:
─ Probiotics
─ Herbal remedies, e.g. peppermint oil:
─ Offer more information:
o I will give you some brochures and web sites in case you want to read more
about that
Associated diseases:
- IBS
- Fibromyalgia
- Chronic fatigue syndrome
- Interstitial cystitis

OSCE-guide-III.doc Page 31 of 255


History taking – Medicine

ASTHMA
Mr … comes to your office as post-ER visit follow-up, he had asthmatic attack three days ago.
He went to ER; he was treated and discharged with advice to see his family physician.
Introduction How do you feel now?
EVENT  O S Cf D
 Which medication was used? How many times did you need to puff?
 Symptoms: SOB / Tightness / Wheezes / Sweating / heart racing /
LOC / did you turn blue? Were you able to talk?
 Did you call 911 or someone called for you? Did they give you meds?
What were these meds?
 Were you admitted to hospital? ER? Did they need to put a tube?
What were the discharge meds?
Asthma history  When were you diagnosed? How? Type of buffers?
 Were you controlled? How many times do you puff (excluding
exercise)? Are you using spacer?
 Recently, did you notice a need to ↑ the doses?
 Any attacks during the night?
 Do you use peak flow meter?
 Did you have PFTs (pulmonary function tests) done?
 How many times did you have to go to ER?
Triggers Infection  Recent chest infection? Flu-like symptoms? Fever / chills?
Medications  How do you use puffers? Stored properly? Not expired?
 Did you start new medication? β-blockers? Aspirin? Any recent ↑ in
dose of these medications?
Outdoor  Exercise
 Cold air
 Pollens (is it seasonal?)
 Dust: construction / smug (smoke/ fog/ exhaust)
Indoor  Do you smoke? Anybody around you?
 Do you have pets? People around you?
 Fabrics related: carpets floor? Any change in linen? Pillows?
Blankets? Mattress? Curtains?
 Relation to any type of food?
 Perfumes
 Do you live in a house (basement  mold)?
 Any construction renovation? Exposure to chemicals?
Stress  Any new stressful situations?
PMH and FH  Skin allergies
 Other allergies

Asthma Management
1- Confirm diagnosis:
─ Symptoms:
o Cough (dry / more at night / more with exercise / induced by allergens)
o Wheezes (noisy breathing)
o Chest tightness
─ Examination: wheezes
─ Diagnosis:
o Chest x-ray: R/O pneumonia / infection / cancer
o Pulmonary Function Tests (PFTs):
 FEV1/FVC < 80% of expected  obstructive lung disease
 Give bronchodilators, repeat PFTs after 20 min, if ↑ > 12%  Asthma

OSCE-guide-III.doc Page 32 of 255


History taking – Medicine

2- Management:
─ Environment control: avoidance of irritant and allergic triggers (e.g. avoid smoking /
change β-blocker for treatment of HTN)
─ Patient education: the allergic nature of the disease and triggering factors
─ Written action plan: see the diagram below (next page)

3- Medications:
Type Symptoms Treatment Notes
Mild < 2 times / week Short acting β2-agonist: Does not need daily
intermittent 1-2 puffs (PRN and medication
before exercise)
Mild > 2 times / week Short acting β2-agonist Low dose ICS LTRAs are second-line
persistent but < 1 time / day (Ventolin 100 mcg – 1- (Flovent 125 monotherapy for mild
2 puffs qid) mcg – 1 puff bid) asthma
Moderate Daily LABA Moderate dose 6-11 yrs: ICS should be ↑
(Serevent 50 mcg – 1 ICS to moderate dose
puff bid) (Flovent 250 > 12 yrs: LABA should be
mcg – 1 puff bid) considered first
Severe Continuous / Add LABA or LTRA High dose ICS Oral Omalizumab (anti IgE)
Uncontrolled (Singulair 10 mg PO (Flovent 250 prednisone may be considered in
qhs) mcg – 2 puffs patients > 12 yrs
bid)
ICS : Inhaled Corticosteroids; 1 puff = 100 mcg
LABA : Long-acting beta2-adrenoceptor agonist
LTRA : Leukotriene receptor antagonist

Puffers and LTRA tablets:


Medication Color Active ingredient / puff Class Notes
Ventolin Blue Salbutamol 100 mcg Short acting β2-agonist
Serevent Blue Salmeterol 50 mcg LABA Also available as diskus
Flovent Orange Fluticasone propionate 125 mcg ICS
250 mcg
500 mcg
Advair Purple Fluticasone 250 mcg ICS Also available as diskus
Salmeterol 50 mcg LABA
Pulmicort Brown Budesonide 100 mcg ICS
200 mcg
400 mcg
Symbicort Red Budesonide 400 mcg ICS
Formoterol 12 mcg LABA
Atrovent Green Ipratropium bromide 20 mcg Anti-cholinergic For Asthma and COPD
Spiriva Spiriva Tiotropium bromide 18 mcg Long-acting, 24 hrs, anti- For COPD
Handihaler cholinergic bronchodilator
Singulair Tablets Montelukast 4 mg Leukotriene receptor
10 mg antagonist (LTRA)

4- Emergency treatment of Asthma:


─ Oxygen, aim for SaO2 > 94%
─ Bronchodilators:
o Beta 2 agonists: salbutamol 100 mcg 4 puffs q 15-20 min x 3 AND
o Ipratropium bromide 4 puffs q 15-20 min x 3
─ Corticosteroids: hydrocortisone 200 mg IV or prednisone 40-60 mg PO
─ Rehydration: aggressive IV fluids to liquefy bronchial secretions

OSCE-guide-III.doc Page 33 of 255


History taking – Medicine

Canadian Thoracic Society – Asthma Management Continuum – 2010

Asthma action plan:

OSCE-guide-III.doc Page 34 of 255


History taking – Medicine

COPD management

Prolong survival  Smoking cessation


 Vaccination: influenza virus, pneumococcus (Pneumovax)
 Home oxygen: to prevent cor pulmonale and decrease
mortality if used > 15 hrs/day (indications: PaO2 < 55
mmHg; or < 60 mmHg with cor pulmonale or
polycythemia)
Bronchodilators Short acting Ventolin (q6h PRN) + Atrovent (1-2 puffs q6h)
Long acting  LABA (Serevent 50 mcg/dose) ± Atrovent /or/
 LACA (Spiriva): 18 mcg qAM + must stop Atrovent
Respiratory rehabilitation
Corticosteroids Inhaled Combination of ICS + LABA
 Advair: Fluticasone / Salmeterol (250mcg/50mcg) od or
bid
 Symbicort: Budesonide / Formoterol (400mcg/12mcg) 2
puffs bid
Oral Short course of oral corticosteroids: 50mg/d x 5 days

COPDE (COPD exacerbation):


 Definition: episode of increased dyspnea, coughing, increase in sputum volume or purulence
 Etiology: viral URTI, bacteria, air pollution, CHF, PE, Ml must be considered
 Management:
─ Assess ABC, consider assisted ventilation if decreasing LOC or poor ABGs
─ Supplemental O2 (controlled FiO2: target 88-92% SaO2 for CO2 retainers
─ Bronchodilators by nebulizer
o Short acting beta2-agonists used concurrently with anti-cholinergics
o Salbutamol and Ipratropium bromide via nebulizers x 3 back-to-back
─ Systemic corticosteroids: IV solumedrol or oral prednisone (50mg/d x 5 days)
─ Antibiotics: often used to treat precipitating infection:
o Indications (2 out of 3): increased SOB, increased sputum amount, or increased
sputum purulence (change in colour, e.g. greenish)
o Type of antibiotics: see below
─ Post exacerbation: rehab with general conditioning to improve exercise tolerance
 ICU admission
─ For life threatening exacerbations
─ Ventilator support
o Non-invasive: NIPPY, BiPAP
o Conventional mechanical ventilation

Antibiotics:
─ Outpatient: resp fluoroquinolones: levofloxacin 750 mg PO q24h x 5 days OR
beta-lactam + macrolide (amoxicillin 1000 mg PO tid + clarithromycin 500 mg PO bid)
─ Risk factors (group home / hospital infection / immunocompromised):
ceftriaxone (1 g IV q24h) + azithromycin (500 mg IV q24h x 5 days).
Step-down to oral therapy when tolerated
─ Susceptible for pseudomonas / recent use (within 3 months) of antibiotics or cortisone:
piptazo (piperacillin / tazobactam); 3.375 gm IV q6h)
─ MRSA: Vancomycin 1 gm IV q24h

OSCE-guide-III.doc Page 35 of 255


History taking – Medicine

Ankle swelling – Bilateral

Introduction
CC uni- vs. bi- lateral
Analysis of OsCfD  What ↑? Walking / standing what ↓? Raising legs
CC ↑↓  How high does it go?
AS  Local symptoms:
─ Pain / fullness / heaviness / tightness
─ Skin changes (redness / swelling / do you feel your feet warm?)
─ Nail changes
 Other swellings in your body:
─ How about swelling in your face? Eye puffiness? Do you find it
difficult to open your eyes in the morning?
─ How about your belly? Did you need to ↑ the size of your belt?
─ Hands, did you feel it is tight to wear your ring?
Impact  How does this affect your life?
Red flags  Constitutional symptoms – for infection / cancer
Differential Differential diagnosis of BILATERAL ankle swelling:
diagnosis  Failure Heart
 Failure Liver
 Failure Kidney: history of kidney disease (changes in urine / bruising /
frequency / burning / frothy urine / clear or no)
 Hypoalbuminemia
 Thyroid diseases
Specific cause within this system (e.g. kidney)
 Hx or Dx of DM
 Any medications (penicellamine, gold, NSAIDs, …)
 Recent sore throat
 Any skin infection / rash
 Hx of autoimmune disease
 How about diet? Is it balanced? Any diarrhea?
PMH
FH
SH

Case: patient with face swelling, BP 150/90, protein in urine, ketones, no blood, no glucose, no
WBCs
Diagnosis: nephritic syndrome (minimal changes)
Investigations:
─ Kidney function tests / urinalysis / 24 hrs protein in urine / renal biopsy
─ Lipid profile / blood glucose studies
─ Hepatitis B serology / ANA / C3 and C4
Management:
─ Salt restriction / avoid fats
─ Diuretics / monitor fluids in and out
─ Anti-HTN: ACE inhibitors
─ Prednisolone

OSCE-guide-III.doc Page 36 of 255


History taking – Medicine

Ankle swelling – Unilateral

Introduction
CC uni- vs. bi- lateral
Analysis of OsCfD  What ↑ / ↓?
CC ↑↓  How high does it go?
 If pain  PQRST
AS  Local symptoms:
─ Pain / fullness / heaviness / tightness
─ Skin changes (redness / swelling / do you feel your feet warm?)
─ Nail changes
 Other joints? Toes? Other ankle?
Impact  How does this affect your life?
Red flags  Constitutional symptoms – for infection / cancer
Differential Differential diagnosis of UNILATERAL ankle swelling:
diagnosis  Any trauma, any twist in your ankle?
 Gout; previous attacks, screen kidney  for kidney stones
 Infection, sepsis, cellulitis; fever, pus, discharge, tenderness
 Gonorrhea septic arthritis; Sexual history, penile discharge? Unprotected sex
recently?
 DVT
Specific cause within this system (e.g. gout)
 Tell me more about your diet? Too much protein?
 How about alcohol?
 Medications? Pain meds (aspirin) / diuretics (furosemide, thiazides)?
 Hx of cancer / chemotherapy (cytotoxic drugs) / radiation?
 Family hx of gout / kidney stones?
PMH
FH
SH

DVT: see the physical examination section

OSCE-guide-III.doc Page 37 of 255


History taking – Medicine

Congestive heart failure – CHF

68 years old man comes to ER with 4 weeks of SOB

Analysis of Clarification 1- When do you say SOB; what do you mean? Cardiac or chest?
CC ─ Is it difficult to breathe in and out?  cardiac / anemia
─ Is it difficult to breathe out?  COPD / asthma
2- Do you have any hx of asthma? Lung disease?
─ Any wheezes? Chest tightness? Cough?
3- Do you have any hx of heart disease?
─ No  newly dx
─ Yes  ? acute on top of CHF
─ Any racing heart? Dizziness? LOC? Any hx of HTN?
OSCfD  Is it first time? Or you had it before? When and how were you
PQRST diagnosed? How about treatment?
↑↓  Is it related to activity? How many blocks were you able to walk?
And now?
 How about at rest? And at night?
Impact  Left ventricle:
─ SOB? How many pillows do you use?
─ Do you wake up at night gasping for air?
─ Cough / crackles?
 Right ventricle:
─ Any swelling in your LL? How high does it go? Related to position / standing?
Weight gain?
─ Eye puffiness? Swollen face? Pain on the liver?
 Other cardiac symptoms:
─ Chest pain? Nausea/vomiting? Sweating?
─ Heart racing / dizziness / LOC? Do you feel tired?
Red flags  Constitutional symptoms – for infection / cancer
 Risk factors for ischemic heart diseases – IHD
DD Causes (that precipitated acute on top of CHF):
 Compliance
 Diet
 Medical
PMH DM / Kidney / Liver diseases
FH HTN / heart attacks
SH SAD

DD (Causes that precipitated acute on top of CHF):


 Compliance:
─ Are you receiving treatment? Which medications do you take? How much? For how long? Any
change in medications? Change in dose?
─ Do you take it on regular basis? Any chance that you may skip one or more doses?
─ Do you take it by yourself or do you need help?
─ Did you get your Digoxin level measured before / recently?
─ Did you start new medication? Rx or (OTC) over the counter? e.g. indomethacin
─ How about water pills?
─ Are you under regular F/U? How often? When was the last time? Were you symptoms free at that
time?
 Diet:
─ Do you have special diet? Salt-free diet? Do you monitor that?
─ Any new changes in diet?
─ Any chance of salty food, e.g. pickles, canned food, dried meet and fish

OSCE-guide-III.doc Page 38 of 255


History taking – Medicine

 Medical:
─ Do you take medications on regular basis? Any new medication? Advil?
─ Any hx of thyroid dx, any sweating / diarrhea?
─ Any hx of heart disease / HTN ( A Fib) / heart attack / CAD (ischemia) / did you feel your
heart bouncing (arrhythmias)? Any congenital or valvular disease / Chest pain / tightness /
dizziness / light headedness / LOC?
─ Any chest / lung disease (wheezes, cough, chest tightness)
─ Any kidney disease? Renal failure?
─ Any bleeding? Anemia?

New York Heart Association functional Classification of heart failure:


 Class I: ordinary physical activity does not cause symptoms of HF
 Class II: comfortable at rest, ordinary physical activity results in symptoms
 Class III: marked limitation of ordinary activity; less than ordinary physical activity  symptoms
 Class IV: inability to carry out any physical activity without discomfort; symptoms may be present at rest

Investigations:
 Labs: CBC / lytes / ABG (arterial blood gases) / glucose / INR / PTT / serial cardiac enzymes (q8h x 3)
/ ECG / fluid balance
 Chest x-ray findings of CHF: (1) Enlarged heart, (2) Upper lobe vascular redistribution, (3) Kerley B
lines (thin linear pulmonary opacities caused by fluid or cellular infiltration into the interstitium of the
lungs), (4) Bilateral interstitial infiltrates, (5) Bilateral small effusions

Treatment:
─ Acute heart failure:
o Treat acute precipitating factors (e.g. ischemia. arrhythmias)
o L Lasix (diuretics)  ↓ pre-load (furosemide: 40-500 mg IV)
o M Morphine; 2-4 mg IV – decreases anxiety and preload (venodilation)
o N Nitrates (venous and arterial dilator  ↑ kidney perfusion)
o O Oxygen
o P Positive airway pressure (CPAP/BiPAP) – decreases preload and need for
ventilation / Position (sit patient up with legs hanging down unless hypotensive)
o In ICU or failure of LMNOP: sympathomimetics (dopamine or dobutamine)
─ Chronic heart failure (long term management):
o ACEI (slow progression and improve survival) or ARBs (if ACEI not tolerated)
o Beta blockers: slow progression and improve survival
 Should be used cautiously, titrate slowly because may initially worsen CHF
 Side effects: fatigue / bradycardia
 If pt on β-blockers  exacerbation  stop the β-blockers for 2 days
o Digoxin (if A Fib OR symptomatic on ACEI)
o Diuretics: symptom control, management of fluid overload; furosemide 80 mg OD
(furosemide opposes the hyperkalemia induced by beta-blockers, ACEIs)
• Spironolactone for class Ill-b and IV CHF already on ACEI and loop diuretic
• If still uncompensated: Implantable Cardioverter Defibrillator (ICD)
o Anti-arrhythmic drugs: for use in CHF with arrhythmia can use amiodarone, beta-
blocker, or digoxin
o Anticoagulants: warfarin for prevention of thromboembolic events

Digoxin overdose:
─ Anorexia, nausea, vomiting
─ Bradycardia, dizziness, LOC
─ ECG: PVC, heart block
─ Vision: yellow hallos around objects

OSCE-guide-III.doc Page 39 of 255


History taking – Medicine

Heart racing

For few weeks


Introduction … But first I would like to ask you, at the moment, how do you feel?
CC
Analysis of Clarification When do you say your heart is racing; what do you mean?
CC ─ Do you feel your heart is going fast
─ Or is skipping beats
─ Can you tap it for me please? …
─ It sounds irregular for me!
OSCfD  Is it first time? Or you had it before? When and how were you
PQRST diagnosed? How about treatment?
↑↓  Is it related to activity? How many blocks were you able to
walk? And now?
 How about at rest? And at night?
 When was the last attack? And what is the duration of the
longest attack?
 Is it related to caffeine, chocolate, coke, any other type of
food?
 SAD (cocaine or any other stimulant)
Impact  CVA (any weakness / numbness / difficulty finding words / visual problems)
 Heart failure (SOB / limitation of activity / swelling in your legs / how many
pillows do you use??
 Other cardiac symptoms:
─ Chest pain? Nausea/vomiting? Sweating?
─ Heart racing / dizziness / LOC? Do you feel tired?
Red flags  Constitutional symptoms – for infection / cancer
 Risk factors for ischemic heart diseases – IHD
DD  Do you take medications on regular basis? Any new medication?
 Any hx of thyroid dx, any sweating / diarrhea?
 Any hx of heart disease / HTN ( A Fib) / heart attack / CAD (ischemia) /
did you feel your heart bouncing (arrhythmias)? Any congenital or valvular
disease / Chest pain / tightness / dizziness / light headedness / LOC?
 Any chest / lung disease (wheezes, cough, chest tightness)
 Any kidney disease? Renal failure?
 Any bleeding? Anemia?
PMH DM / Kidney / Liver diseases
FH  Family history of sudden death at a young age? (cardiomyopathy)
 HTN / heart attacks
SH SAD
Physical  Vitals
examination  Cardiac exam (looking for mid-diastolic, mitral stenosis, rumbling character)
 Thyroid exam
 Neurological exam: brief / gross motor and reflexes

OSCE-guide-III.doc Page 40 of 255


History taking – Medicine

The patient daughter has a concern: my mother was diagnosed with AF,
Should I worry about this?
1. This is a reasonable concern?
 AF may lead to embolic event (CVA)
 AF may lead to heart failure
 AF may lead to V. Fib
2. However, this is not uncommon condition, and it is treatable with medications

Causes for AF: Causes for TACHYCARDIA


1- IHD 1- Exercise
2- Hypertension / CHF 2- Pregnancy
3- Hyper-thyroidism 3- Caffeine / stimulants
4- Medications (e.g. digoxin, some anti- 4- Anemia
arrhythmic meds – class I) 5- Hypovolemia
5- Electrolytes imbalance 6- Fever
6- Too much alcohol (holiday heart) 7- Stress
7- Cardiomyopathy 8- Smoking
8- Valvular heart diseases 9- Hyper-thyroidism
9- Congenital heart diseases 10- Pheochromocytoma
10- Loan AF
11- Myocarditis
12- COPD / pneumonia
13- Cardiac surgery
14- Cocaine
15- Any condition that lead to tachycardia in a
susceptible person

Atrial fibrillation
Stable Unstable

< 48 hours > 48 hours  SOB


If in doubt  TEE  Rate control:  BP < 90/60
─ β-blockers  Chest pain
─ Ca ch blockers (diltiazem, verapamil)  Confusion
─ Digoxin (in patients with heart
failure)
 Anti-coagulation (3 weeks prior and 4
weeks after cardioversion)

Cardioversion:
 Electrical: 150 joules for A Fib (50 joules for A Flutter)
 Pharmacological: procainamide; 1 g / 1 hr infusion

Anti-coagulation:
Assess stroke risk: determine CHADS2 score in patients with non-valvular AF
Risk factor Points CHADS2 score Anti-coagulation
CHF 1 0-1 Aspirin 81-325 mg daily
Hypertension 1 ≥ 2 moderate risk Warfarin
Age > 75 yrs 1 factors or any high risk
Diabetes 1 factor (prior stroke,
Stroke / TIA 2 TIA or embolism,
mitral stenosis,
prosthetic valve)

OSCE-guide-III.doc Page 41 of 255


History taking – Medicine

Fall

Orthostatic hypotension
76 years old male patient came to clinic because he fell few days ago. He was getting out of bed,
when he fell to the ground

Introduction  Did you hurt yourself? How do you feel?


HPI: analysis of CC  Was this the first time, or did it happen before?
 Was it related to Emotions? Coughing? Urination?
Associated During  Did you lose conscious? Did you hit your head?
Symptoms  Were you alone or with someone? Did your wife describe it to
you? Is she with you? If it is ok with you, after we finish, I would
like to speak with her to get some info.
 Were you shaking? Certain part of your body or whole?
For any  Were you breathing? Did you turn blue?
Fall,  Did you bite your tongue? Roll your eyes? Wet yourself?
LOC Before  Were you able to take few steps or did you fall immediately
or seizure (orthostatic hypotension)?
 Before you lose consciousness, did you feel:
─ Dizzy, lightheaded, nausea, vomiting, any chest pain, heart
racing (cardiac)
─ Things are spinning around you (vertigo)
─ Weakness, numbness, vision changes (stroke)
─ Any flashing light, strange smells (epilepsy)
─ Sweaty, shaky, hungry (hypoglycemia)
After  How long did it last?
 How did you regain consciousness? By yourself or did you need
intervention?
 After you regain consciousness; were you able to recognize the
surroundings? Able to talk? Able to move?
 Did you feel any weakness, numbness?
Impact  Did you hurt yourself? How do you feel now?
Red flags  Constitutional symptoms
 Risk factor for IHD
Any geriatric  Balance
patient; ASK about:  Vision
 Hearing
 Urination
DD  Diseases: arrhythmia / CAD, CVA, seizure, hypoglycemia
(already analyzed – before the event)
 Environment: is your room well lit? Any chance you tripped?
 Do you take any medications? Do you have a list?
─ Go through it one by one
─ Which one was added / changed recently?
─ Each medication: ask about the disease, when started?
─ Any OTC? Aspirin? Who prescribed it to you?
 Are you getting enough fluids

OSCE-guide-III.doc Page 42 of 255


History taking – Medicine

Counselling:
 Inform the patient
─ The most likely explanation to what happened is a condition called “postural orthostatic
hypotension”. It means drop in the blood pressure with change of posture.
─ Explain the pathophysiology:
o When we change position from lying or sitting to standing, blood tends to pool in
the lower extremities, and this leads to drop in blood pressure. Normally, blood
vessels in our body react by narrowing in order to prevent this and to maintain
normal blood pressure.
o In patients having orthostatic hypotension, and this could be due to age /
medications / DM or combination, their blood vessels fail to react fast enough,
this leads to pooling of blood in lower extremities  ↓ amount of blood reaching
to heart  ↓ blood reaching the brain  they end-up losing their consciousness
temporarily.
─ Consequences: this might happen again
─ Investigations:
o Blood works / CBC / differential / lytes / kidney and liver function tests
o ECG
 Preventive measure:
─ Contact the psychiatrist to check the poly-pharmacy, to discuss with him the possibility
of decreasing the dose or changing medications.
─ Meanwhile, if you are changing positions, do this slowly, on steps, e.g. from lying down,
sit for a couple of minutes on the bed before standing up, and before you stand up, push
your feet against the ground for few seconds.
 I will give you brochures and web sites in case you need to read more.

Notes:
 The patient will have a list of medications:
─ Lipitor
─ Hydrochlorothiazide  ask about fluids intake
─ β-blocker
─ Aspirin  ask about bleeding
─ Lorazepam
─ Oxazepam  I can see that you are taking 2 sleeping pills, who prescribed them to you?
The same doctor or no?
─ Metformin
─ B12 / B complex

 If the patient looks sad / depressed  you look down for me, any chance you are depressed

OSCE-guide-III.doc Page 43 of 255


History taking – Medicine

Peripheral vascular disease:

Calf pain / swelling

Introduction
Analysis of  OsCfD
CC  PQRST
─ P: unilateral or bilateral
─ R: what about other joints, knees? Thighs? Feet?
 What ↑ or ↓: did you notice that your pain ↑ while walking up or down hill?
─ ↑ while walking uphill: peripheral arterial disease
─ ↑ while walking downhill: spinal stenosis
 Is it first time? Or you had it before? When and how were you diagnosed?
How about treatment?
 Is it related to activity? How many blocks were you able to walk? And
now?
 How about at rest? And at night?
 When was the last attack? And what is the duration of the longest attack?
Impact  History of strokes / TIAs / neurological symptoms
 Chest pain / SOB / heart racing
 Pain after eating (intestinal ischemia)
 Effect of pain on daily activities / work?
 Leriche syndrome (aorto-iliac occlusive disease): numbness in buttocks &
thighs / absent or decreased femoral pulses / impotence
Red flags  Constitutional symptoms – for infection / cancer
 Risk factors for ischemic heart diseases – IHD
─ Smoking? How much and for how long?
─ High blood pressure? For how long? Controlled or not?
─ Diabetes mellitus
─ Cholesterol measured? When? What was it?
DD Peripheral Arterial Disease versus Spinal Canal Stenosis
Vascular symptoms Neuro symptoms
 Cold feet / ulcers  Weakness / numbness / tingling
 Swelling / redness  Back trauma / back pain
 Delayed wound healing  Sexual dysfunction / difficulty with
 Nail changes / hair loss erection
PMH Past history of heart disease / stroke / symptoms of stroke / DM / Kidney / Liver
diseases
FH Family history of heart disease / HTN / heart attacks
SH SAD

N.B. the 6 Ps of ischemia:


Pallor / Pain / Parathesia / Paralysis / Pulseless / Polar (cold)

OSCE-guide-III.doc Page 44 of 255


History taking – Medicine

Urinary symptoms:

 Obstructive (prostatic disease in ♂  anuria):


─ Difficulty to initiate urine? Do you need to strain?
─ Any changes in the stream?
─ Any dripping?
─ After you pass urine, do you feel that you emptied your bladder completely or do you
need to go again?
 Irritative (frequency  UB disease):
─ How many times do you go to the washroom?
o How about before? Any change?
o How about during night time? How does this affect your sleep? How about your
concentration and mood?
─ Do you need to rush to washroom? Are you able to make it all the time?
─ Have you ever lost control or wet yourself?
─ Any burning sensation? Any flank pain?
─ Fever / night sweats / other constitutional symptoms
 Urine analysis (changes):
─ COCA + B (content: frothy / cloudy / not clear)

Summary of irritative symptoms: FUND


Frequency / Urgency / Night time / Discomfort

OSCE-guide-III.doc Page 45 of 255


History taking – Medicine

Anuria
Introduction Empathy – how do you feel right now?
Analysis of CC  OsCfD
 PQRST
 What ↑ or ↓
 Is it first time? Or did it happen before? When and how were you diagnosed? How
about treatment?
Associated symptoms:
 Obstructive symptoms
 Irritative symptoms
 Urine analysis (changes): COCA ± Blood
Local symptoms:
 Any problems with passing stools? What? When?
 Any masses in the groin / pelvic mass / pain?
 Abdominal pain? Distension?
Impact Metastasis  Back: pain / weakness / numbness
 Liver: yellow / itchy / urine / stools
 Lungs: cough / phlegm / hemoptysis
 Brain: headache / nausea / vomiting
Renal failure Generalized swelling / face puffiness / itching
Sexual Sexual dysfunction
Red flags  Constitutional symptoms – for infection / cancer
 Risk factors for cancer prostate / bladder
─ Were you ever diagnosed with prostate disease? Screened for prostate
diseases? (DRE or PSA)
─ Family history of prostate disease / cancer?
─ Ca bladder (radiation / exposed to chemicals / aniline dye)
─ Smoking? Alcohol?
DD  Renal stones: Have you ever had a renal stone? Any history of colicky pain in
flanks? Have you ever passed a small crystals or stone during voiding? Hx or
repeated UTIs?
 Medications: glaucoma / anti-psychotic meds / anti-cholinergic drugs; like those
used for incontinence; e.g. Ditropan (Oxybutynin), Detrol (Tolterodine)
 2 Neuro:
─ Back problem: trauma – metastasis – cauda equine (spoiled himself with stools
/ buttocks numbness)
─ Stroke (diagnosed / weakness / numbness / difficulty)
 2 Cancer:
─ Cancer prostate
─ Ca bladder (hematuria)
PMH AMPLE
FH DM / anemia / polycystic kidney disease / renal stones
SH SAD
Most likely diagnosis: BPH
Other possible diagnoses: UTI / prostatitis / Ca prostate
Investigations: urea & creatinine / urinalysis / renal US / DRE & PSA / TRUS
If cancer is suspected: bone scan / CT
Treatment:
─ Watchful waiting: may resolve spontaneously
─ Medical treatment: α-adrenergic antagonists (doxazosin, terazosin) / 5-α-reductase
inhibitors (finasteride)
─ Surgery: open surgery / TURP / minimally invasive (stent / laser ablation / cryosurgery)

OSCE-guide-III.doc Page 46 of 255


History taking – Medicine

Hematuria
Introduction Empathy – how do you feel right now?
Analysis of  OsCfD
CC  Timing:
─ Initial versus terminal or total
─ Diurnal variation
 What ↑ or ↓
 Painful or Painless
 Is it first time? Or did it happen before? When and how were you
diagnosed? How about treatment?
Associated symptoms:
 Obstructive symptoms  prostate disease
 Irritative symptoms  UB disease
 Urine analysis (changes): COCA ± Blood
Local symptoms:
 Any problems with passing stools? What? When?
 Any masses in the groin / pelvic mass / pain?
 Abdominal pain? Distension?
Impact Metastasis
Renal failure Generalized swelling / face puffiness / itching
Sexual Sexual dysfunction
Red flags  Constitutional symptoms – for infection / cancer
 Risk factors for cancer prostate / bladder / RENAL
─ Were you ever diagnosed with prostate disease? Screened for prostate
diseases? (DRE or PSA)
─ Family history of prostate disease / cancer?
─ Family history of cancer bladder or kidney?
─ Ca bladder (radiation / exposed to chemicals / aniline dye)
─ Smoking? Alcohol?
DD  Renal stones: Have you ever had a renal stone? Any history of colicky pain
in flanks? Have you ever passed a small crystals or stone during voiding?
Hx or repeated UTIs?
 Medications: blood thinners / aspirin / bleeding from other sites?
 Pseudo-hematuria:
─ Diet: eating too much beet
─ Medications: Rifampicin
─ Other bleeding: bleeding per rectum / vaginal bleeding
PMH  AMPLE
 History of hemolytic anemia / polycystic kidney
FH DM / anemia / polycystic kidney disease / renal stones
SH SAD
Investigations:
(1) Kidney: urinalysis (casts / crystals / C&S / cytology) / ultrasound (abd/pelvic) / IVP / KFTs
(2) Bladder: cystoscopy
(3) Prostate: PSA / TRUS
(4) Others: CBC / differential / INR
Case: patient on warfarin for A. fib for 2 yrs; went to walk in clinic for sore throat and was prescribed
Biaxin, developed hematuria. Diagnosis: coagulopathy.

OSCE-guide-III.doc Page 47 of 255


History taking – Medicine

Renal stones

Risk Factors
─ Hereditary: RTA, G6PD, cystinuria, xanthinuria, oxaluria, etc.
─ Dietary excess: Vitamin C, oxalate, purines, calcium
─ Dehydration (especially in summer months)
─ Sedentary lifestyle
─ Medications: thiazides
─ UTI (with urea-splitting organisms)
─ Hypercalcemia disorders: hyperparathyroidism, sarcoidosis, histoplasmosis, etc.

Investigations
─ Screening labs
o CBC: elevated WBC in presence of fever suggests infection
o Electrolytes, Cr, BUN  to assess renal function
o Urinalysis: R&M (WBCs, RBCs, crystals), C&S
─ Imaging
o Kidneys, ureters, bladders (KUB) x-ray to differentiate opaque from non-opaque
stones (e.g. uric acid, indinavir) / 90% of stones are radiopaque
o CT scan: no contrast; distinguish radiolucent stone from soft tissue filling defect
o Abdominal ultrasound: may demonstrate stone (difficult for ureters) / may
demonstrate hydronephrosis
o IVP (not usually done): anatomy of urine collecting system, degree of
obstruction, extravasation
─ Cystoscopy for suspected bladder stone
─ Strain all urine  stone analysis
─ If recurrent stone formers, conduct metabolic studies
o Serum electrolytes, Ca, PO4, uric acid, creatinine and urea
o PTH if hypercalcemic

Approach to renal stone:

OSCE-guide-III.doc Page 48 of 255


History taking – Medicine

Treatment – Acute:
─ Medical:
o Analgesics (Tylenol #3)
o NSAIDs help lower intra-ureteral pressure
o ± antibiotics for UTI
o ± (antiemetic + IV fluids) for vomiting
─ Interventional:
o Ureteric stent (cystoscopy)
o Percutaneous nephrostomy (image-guided)
─ Admit if necessary:
o Intractable pain
o Intractable vomiting
o Fever (? infection)
o Compromised renal function
o Single kidney with ureteric obstruction / bilateral obstructing stones

Treatment – Elective:
─ Medical:
o Conservative if stone < 5 mm and no complications
o Fluids to increase urine volume to > 2 L/day (3-4 L if cystine)
o Specific to stone type:
 Calcium oxalate stones: thiazides / potassium citrate (alkalinization of urine)
 Calcium struvite: antibiotics for 6 wks (stone must be removed to treat infection)
 Uric acid: allopurinol / potassium citrate (alkalinization of urine to pH 6.5 to 7) /
shockwave lithotripsy not effective
 Cystine: alkalinize urine (bicarbonate / potassium citrate) / penicellamine / captopril
(forms complex with cystine) / shockwave lithotripsy not effective
─ Interventional:
o Procedural / surgical: If stone is > 5 mm or presence of complication
o Kidney
 Extracorporeal shockwave lithotripsy (ESWL) if stone < 2.5 cm
 Percutaneous nephrolithotomy; indications:
+ Size > 2.5 cm + Staghorn + UPJ obstruction
+ Calyceal diverticulum + Cystine stones
o Ureter
 ESWL is the primary modality of treatment
 Ureteroscopy (extraction or fragmentation) if failed ESWL / Ureteric stricture
o Bladder
 Transurethral cystolitholapaxy
 Remove outflow obstruction (TURP or stricture dilatation}

Management of UTI:
─ Investigations:
o Urine for culture and sensitivity
o Blood: CBC / differential
o Imaging (if suspect complicated pyelonephritis or symptoms do not improve with 72
hours of treatment): Abd/pelvic U/S / IVP / Cystoscopy / CT
─ Pregnant: amoxicillin 500 mg TID x 7 days
─ Non-pregnant:
o Septra (sulfamethoxazole and trimethoprim) DS (800/160): 1 tab bid x 7 days
o /OR/ Ciprofloxacin 500 mg bid x 7 days
─ Pyelonephritis:
o Ceftriaxone (third-generation cephalosporins): 1 g IV q24hrs x 2 days
o Then continue oral ciprofloxacin x 7 days
─ Abscess: + drain

OSCE-guide-III.doc Page 49 of 255


History taking – Medicine

Incontinence
Obstructive / 62 years old female, with hx of 3 years of urinary incontinence
Introduction Empathy – how do you feel right now?
Analysis of  OsCfD
CC  What ↑ or ↓: lifting objects / coughing / straining
 Is it first time? Or did it happen before? When and how were you
diagnosed? How about treatment?
Associated symptoms: If at any time there is a frequency or
 Obstructive symptoms some new symptom  analyze it
 Irritative symptoms first then resume!
 Urine analysis (changes): COCA ±
Blood Frequency in ♀  UTI
Local symptoms:
 Any problems with passing stools? What? When?
 Any masses in the groin / pelvic mass / pain?
 Any perineal skin lesions?
Impact  How does it affect your life? Daily activities?
Red flags  Constitutional symptoms – for infection / cancer
 Risk factors (MGOS):
M  Menopausal symptoms, and HRT
 LMP
G  Gynaecological history
 Previous abdominal or pelvic surgeries
O  Obstetric: How many pregnancies? Route of delivery?
S  Sexual: Repeated infections / dryness / dyspareunia
DD  Overflow incontinence
 Urge incontinence
 Detrusor overactivity: CNS lesion, inflammation / infection (cystitis),
bladder neck obstruction (tumour, stone)
 Stress incontinence
 Urethral hypermobility: childbirth, pelvic surgery, aging
 Intrinsic sphincter deficiency (ISD): pelvic surgery, neurologic
problem, aging and hypoestrogen state
Diagnosis:
─ History
─ Urinalysis + C&S (if infection suspected)
─ Urodynamics
─ Stress test
Treatment of urge incontinence Treatment of stress incontinence
─ Bladder habit training ─ Weight loss
─ Botox (botulinum toxin) injection ─ Kegel’s exercises
─ Medications: anti-cholinergics; ─ Bulking agents
Tolterodine (Detrol), Oxybutynin ─ Surgery (slings, TVT / TOT4, artificial
(Ditropan), TCAs sphincters)
N.B. Causes of reversible urinary incontinence (DIAPERS): Delirium, Inflammation / Infection, Atrophic
vaginitis, Pharmaceuticals / Psychological, Excess urine output, Restricted mobility, Stool impaction

4
TVT: Tension-free Vaginal Tape, TOT: Trans Obturator Tape

OSCE-guide-III.doc Page 50 of 255


History taking – Medicine

Lump – Neck Swelling

Introduction
Analysis of CC:  Can you point to it?
The lump  OSCfD / Anything special at that time? Fever? Rash?
 Is it painful? PQRST
 Can you estimate its size for me? Is it like a lent, olive, lemon, or
larger? Did it change in size? How fast was the change in size?
 Did you try to feel it? Does it feel soft / rubbery / hard?
 Do you feel it is fixed or moving?
 Any skin changes? Redness? Ulcers?
 Any history of trauma?
 Is it the only one? Did you notice other lumps in your body? How about
other side of your neck? Arm pits? Groins?
Associated (local)  Rule out infection: Any recent flu-like symptoms? Do you feel tired/
symptoms fatigue? History of sinusitis/ Pain in your face? Runny nose?
Pain/discharge in ears? Any sore throat/ oral ulcers/ tooth pain?
Difficulty swallowing? Neck stiffness/pain? Headache? Vomiting?
 Thyroid (if central): heat vs. cold intolerance / sweating / hand shaking
/ heart racing / diarrhea vs. constipation
Impact  How does this affect your life?
 Do you feel tired? ? HIV
 Easy bruising? Repeated infections? ? Leukemia
Red flags  Constitutional symptoms
 Bone pains / Tender points
HEAD SSS  risky behaviour:
 A: includes recent travel
 SAD: how about injection drugs? Did you share needles?
 Sexual hx: Detailed (safe sex, last time, how many partners). Did you
notice any vaginal discharge/ bleeding? Any pain/ blisters/ warts?
Discoloration/ itchiness?
Differential HIV / Lymphoma / Leukemia / Infectious mono-nucleosis
Diagnosis
PMH History of cancer
FH History of cancer / lymphadenopathy
Physical exam  Vital signs
 Neck exam / Thyroid exam if the swelling is central
 LNs / Lymphatic system / LNs in groin / pelvic exam
 Liver / Spleen

Notes:
─ Whenever there is IV drugs  screen for liver symptoms / HIV
─ Whenever there is risk for STIs  screen for liver symptoms and PID

OSCE-guide-III.doc Page 51 of 255


History taking – Medicine

Lump – Breast
Introduction
Analysis of CC:  Can you point to it?
The lump  Is it one breast or both? Where did you notice it? You can ask
verbally, is it LT / RT? Upper / Lower? Outer / Inner? How about
the other breast?
DO NOT POINT WITH YOUR HANDS OR FINGERS!
OSCfD / Anything special at that time? Fever? Rash?

Is it painful? PQRST

Can you estimate its size for me? Is it like a lent, olive, lemon, or

larger? Did it change in size? How fast was the change in size?
 Did you try to feel it? Does it feel soft / rubbery / hard?
 Do you feel it is fixed or sliding (moving)?
 Any skin changes? Redness? Ulcers?
 Any history of trauma?
 Is it the only one? Did you notice other lumps in your body? How about
your neck? Arm pits? Groins?
 Is it related to your period? Does it change with the period?
 Any nipple changes? Discharge? Bleeding? Itching?
Associated (local)  Rule out infection: Any recent flu-like symptoms? Do you feel tired/
symptoms fatigue?
Impact  Headache/ vomiting?
(consequences of  Back pain/ weakness/ numbness/ tingling in arms or legs?
cancer:  Chest pain/ cough/ phlegm/ wheezes/ heart racing?
metastasis)  Liver: yellow discoloration/ itching/ urine/ stools?
Red flags  Constitutional symptoms
 Bone pains / Tender points
Risk factors of cancer: MGO
 Menstrual history: first period / last period / regular?
 G: OCPs?
 Obstetric: History of pregnancies? Number of pregnancies? First
pregnancy at what age?
 Breast feeding?
 Diet rich in fat
 PMH or FH of cancer breast / ovarian carcinoma
Differential  Benign disease
Diagnosis  Trauma  fat necrosis
PMH History of cancer breast / ovarian carcinoma
FH History of cancer breast / ovarian carcinoma
DD for Breast Mass:
─ Breast Cancer ─ Sclerosing adenosis
─ Fibrocystic changes ─ Lipoma
─ Fibroadenoma ─ Neurofibroma
─ Fat necrosis ─ Granulomatous mastitis (e.g. TB,
─ Papilloma / papillomatosis sarcoidosis)
─ Galactocele ─ Abscess
─ Duct ectasia ─ Silicon implant
─ Ductal / lobular hyperplasia

OSCE-guide-III.doc Page 52 of 255


History taking – Medicine

 Galactorrhea (prolactinoma): normal prolactin level: 10-20 ng/ml (non-pregnant)


 If < 40 ng/ml: follow-up
 If > 40 ng/ml: CT and bromocriptine
 If progressive ↑ / headache / affecting vision  surgery
 Benign Breast Lesions:
─ Non-proliferative lesions:
o Aka fibrocystic changes, chronic cystic mastitis, mammary dysplasia
 Benign condition characterized by fibrous and cystic changes in the breast. No ↑ risk
of breast cancer / Age 30 to menopause / ↑ pre-menstrual.
 Breast pain, focal areas of nodularity or cysts often in upper outer quadrant
 Treatment: Evaluation of breast mass and reassurance / Analgesia (ibuprofen, ASA) /
If > 40 years old: mammography every 3 years or biopsy
─ Proliferative lesions:
o Fibroadenoma:
 Most common benign breast tumour in women under age 30
 Risk of subsequent breast cancer is increased only if fibroadenoma is complex, there
is adjacent atypia or a strong family history of breast cancer
 Clinical features: nodules: smooth, rubbery, discrete, well-circumscribed, non-
tender, mobile, hormone-dependent. Needle aspiration yields no fluid
 Investigations: Core or excisional biopsy required
 Treatment: Generally conservative; serial observation. Consider excision if size 2-3
cm, rapidly growing on serial US, if symptomatic or pt preference
o Intra-ductal Papilloma
 Solitary intra-ductal benign polyp
 Present as nipple discharge (most common cause of spontaneous, unilateral bloody
nipple discharge), breast mass, nodule on U/S
 Treatment: excision of involved duct to ensure no atypia
─ Other lesions:
o Fat Necrosis
 Uncommon, result of trauma (may be minor – commonly a tight bra, positive history
in only 50%), after breast surgery (i.e. reduction)
 Firm, ill-defined mass with skin or nipple retraction, ± tenderness
 Regress spontaneously, but complete imaging ± biopsy to R/O cancer
o Mammary Duct Ectasia
 Obstruction of a subareolar duct  duct dilation, inflammation, and fibrosis
 May present with nipple discharge, bluish mass under nipple, local pain
 Risk of secondary infection (abscess, mastitis)
 Resolves spontaneously
o Abscess
 Lactational vs. periductal / subareolar
 Unilateral localized pain, tenderness, erythema, subareolar mass, nipple discharge,
nipple inversion
 Rule out inflammatory carcinoma, as indicated
 Treatment: initially broad-spectrum antibiotics and I&D (incision and drainage), if
persistent total duct excision (definitive)
 If mass does not resolve: fine needle aspiration (FNA) to exclude cancer, U/S to
assess for presence of abscess
 Breast Cancer:
─ 1/9 women in Canada will be diagnosed with breast cancer in their lifetime
─ Risk factors:
 Prior history of breast cancer
 1st degree relative with breast cancer (greater risk if relative was premenopausal)
 Increased risk with high breast density, nulliparity, first pregnancy >30 years old,
early menarche (< 12 yrs), late menopause (> 55 yrs), >5 years HRT
 Decreased risk with lactation, early menopause, early childbirth

OSCE-guide-III.doc Page 53 of 255


History taking – Medicine

─ Investigations
o Mammography
 Screening: every 1-2 years for women age 50-69 / If positive family history in 1st
degree relative: every 1-2 years starting 10 years before the youngest age of
presentation
 Diagnostic: investigation of patient complaints (discharge, pain, lump)
 Follow-up after breast cancer surgery
 Findings indicative of malignancy: mass that is poorly defined, spiculated border,
micro-calcifications, architectural distortion, normal mammogram does not rule out
suspicion of cancer based on clinical findings
o Other radiographic studies:
 Ultrasound – differentiates between cystic and solid
 MRI – high sensitivity, low specificity
 Galactogram / ductogram (for nipple discharge): identifies lesions in ducts
 Metastatic workup as indicated (usually after surgery or if clinical suspicion of
metastatic disease) – bone scan, abd U/S, CXR, head CT
─ Diagnostic Procedures
o Needle aspiration: for palpable cystic lesions; send fluid for cytology if blood or cyst
does not completely resolve
o Fine needle aspiration (FNA): for palpable solid masses; need experienced practitioner
for adequate sampling
o U/S or mammography guided core needle biopsy (most common)
o Excisional biopsy: only performed as second choice to core needle biopsy; should not be
done for diagnosis if possible
─ Genetic Screening: consider testing for BRCA 1/2 if:
o Patient diagnosed with breast AND ovarian cancer
o Strong family history of breast / ovarian cancer (e.g. Ashkenazi Jewish)
o Family history of male breast cancer
o Young patient ( <35 years old)
─ Pathology
o Non-invasive: ductal carcinoma in situ (DCIS): completely contained within breast ducts,
often multifocal / 80% non-palpable, detected by screening mammogram.
 Treatment: lumpectomy with wide excision margins + radiation OR mastectomy if
large area of disease, or high grade
o Invasive:
 Invasive ductal carcinoma (most common 80%): hard, infiltrating tentacles
 Invasive lobular carcinoma (8-15%): 20% bilateral. Does not form micro
calcifications, harder to detect mammographically (may benefit from MRI)
 Paget's disease (1-3%): ductal carcinoma that invades nipple with scaling,
eczematoid lesion
 Inflammatory carcinoma (1-4%): ductal carcinoma that invades dermal lymphatics,
most aggressive form of breast cancer.
─ Clinical features: erythema, edema, warm, swollen, tender breast ± lump
─ Peau d'orange indicates advanced disease (III-b – IV)
─ Treatment of breast cancer:
Stage Primary treatment options Adjuvant systemic
therapy
0 (in situ) BCS + radiotherapy None
I BCS (or mastectomy) + axillary node dissection + May not be needed
II radiotherapy Chemotherapy and /
III mastectomy + axillary node dissection + radiotherapy or hormone therapy
Inflammatory
IV Surgery as appropriate for local control
BCS = breast-conserving surgery

OSCE-guide-III.doc Page 54 of 255


History taking – Medicine

Dizziness

Causes Vertigo Non-vertigo

Peripheral Central Cardiac Non-cardiac


─ Benign paroxysmal ─ Stroke ─ Arrhythmias ─ Vaso-vagal
positional vertigo ─ TIAs ─ CAD / MI episode
─ Ménière's disease ─ Brain tumour ─ CHF ─ Panic attack
─ Vestibular neuritis ─ MS ─ Aortic stenosis ─ Somatization
─ Labyrinthitis ─ Cerebellar lesion ─ Postural
─ Acoustic neuroma hypotension
Symptoms EAR Brain / Neuro
─ Imbalance ─ Mild-moderate ─ Severe
─ N&V ─ Severe ─ Variable
─ Auditory ─ Common ─
─ Neurological ─ ─ Common
─ Nystagmus ─ Unidirectional ─ Bidirectional
(horizontal or (horizontal or
rotatory) vertical)

Clarification Do you feel


 You have blackout (syncope)
 OR the room is spinning around you (vertigo)
Analysis of CC  OCD
 Timing: when / frequency
 What ↑ or ↓: certain position
Impact  Did you lose consciousness?
 Did you fall to ground? Did you hit your head?
Red flags Constitutional symptoms
DD Syncope  Vasovagal attack: LOC / while straining or urinating / nausea / do you feel
warning signs before the dizziness?
 Cardiac (tight AS / arrhythmia): heart racing / chest pain / immediate (no
warning signs)
 Hypotension: antihypertensive meds; change of dose or new medication
 Postural hypotension: diabetes / dehydration / parkinsonism
 Neuro (stroke / TIAs): vision changes / loss, speech impairment, weakness
Vertigo Condition Duration Hearing loss Tinnitus Other features
Benign Paroxysmal Seconds to minutes – – ─ Certain
Positional Vertigo positions
─ Nystagmus
Ménière's disease Minutes to hours Fluctuating + Ear fullness
Vestibular neuritis Hours to days Unilateral –
Labyrinthitis Days Unilateral Whistling Recent AOM
Acoustic neuroma Chronic Progressive + ─ Ataxia
─ CN VII palsy
Cerebellar  Risk factors for CAD: HTN / DM / cholesterol / smoking
lesion  Unbalanced gait
 Aspirin / blood thinners
 Alcohol: neuropathy / cerebellar degeneration
PMH / FH / SH

OSCE-guide-III.doc Page 55 of 255


History taking – Medicine

Condition Management
Benign Acute attacks of transient vertigo lasting ─ Reassure patient that process resolves spontaneously
Paroxysmal seconds to minutes initiated by certain ─ Particle repositioning manoeuvres: Epley’s manoeuvre
Positional head positions, accompanied by torsional (performed by MD) OR Brandt-Daroff exercises
Vertigo (rotatory) nystagmus (performed by patient)
Diagnosis: ─ Surgery for refractory cases
(BPPV) Anti-emetics for nausea/vomiting
─ History ─
─ Positive Dix-Hallpike manoeuvre ─ Drugs to suppress vestibular system delay eventual recovery
and are therefore not used
Ménière's Episodic attacks of tinnitus, hearing loss, ─ Acute management may consist of bed rest, anti-emetics,
disease aural fullness (pressure / warmth), and anti-vertiginous drugs (betahistine)
vertigo lasting minutes to hours ─ Long term management may include:
 Medical: (1) Low salt diet, diuretics
(hydrochlorothiazide), (2) Local application of
gentamicin to destroy vestibular end-organ, results in
complete SNHL, (3) Betahistine (Serc) prophylactically
to decrease intensity of attacks
 Surgical: selective vestibular neurectomy or
transtympanic labyrinthectomy
─ Must monitor opposite ear (bilaterality in 35% of cases)
Vestibular Acute onset of disabling vertigo often ─ Acute phase:
neuritis accompanied by nausea, vomiting and  Bed rest, vestibular sedatives (Gravol), diazepam
imbalance without hearing loss that ─ Convalescent phase:
resolves over days leaving a residual  Progressive ambulation especially in the elderly
imbalance that lasts days to weeks  Vestibular exercises: involve eye and head movements,
sitting, standing, and walking
Labyrinthitis ─ Acute infection of the inner ear Investigations:
resulting in vertigo (days) and ─ CT head
hearing loss ─ If meningitis is suspected: lumbar puncture, blood cultures
─ May be serous (viral), or purulent Treatment:
(bacterial) ─ IV antibiotics
─ Occurs as complication of acute and ─ Drainage of middle ear
chronic otitis media, bacterial ─ ± mastoidectomy
meningitis and cholesteatoma
Acoustic Schwannoma of the vestibular portion of Investigations:
neuroma CN VIII ─ MRI with gadolinium contrast is the gold standard
─ Audiogram – SNHL (sensori-neural hearing loss)
─ Vestibular tests: normal or asymmetric caloric weakness (an
early sign)
Treatment
─ Expectant management if tumour is very small or in elderly
─ Definitive management is surgical excision
─ Other options: gamma knife, radiation
Dix-Hallpike Positional Testing: the
patient is rapidly moved from a sitting
position to a supine position with the
head hanging over the end of the table,
turned to one side at 45° holding the
position for 20 seconds. Onset of
vertigo is noted and the eyes are
observed for nystagmus

OSCE-guide-III.doc Page 56 of 255


History taking – Medicine

INR – Counselling

Analysis History / Give the information


Impact DVT relapse / Bleeding
Red flags Female: OCP / pregnancy / LMP / vaginal bleeding

DVT Causes / Complications


Warfarin / Blood thinners
Decision

Conclusion Offer brochures, support. If you have time: SAD / PMH

 Analysis:
─ History:
o Why are you doing this INR?
o When were you diagnosed? How?
o Were you admitted through the ER or outpatient?
o Was there any involvement of your lungs?
o Which medications were you taking?
o Do you measure your INR regularly? When was the last time? What was the result? What
is your target INR?

─ Give the information: Your measurement today shows INR of 1, any idea why?
o Compliance: Are you still taking your warfarin? On regular basis? Did you stop your
medication? Why?
o Forget: Do you take your medications on your own, or does someone else help you? Any
chance that you missed a dose?
o New medications: Did you start a new medication? What? Why? When?
o Diet: Do you eat a lot of spinach? Or dark green vegetables? (rich in vit K)

 Impact:
Now, I would like to ask you some questions to check if you have relapse of your DVT or bleeding,
then we will go from there
─ DVT relapse:
Because you stopped your medication, I would like to make sure that there is no relapse
o DVT: Have you had any pain / swelling / redness in your calf muscles?
o PE: Have you had any SOB, chest pain, heart racing?
o Stroke: Any confusion? Vision changes? Difficulty finding words? Weakness?
─ Bleeding:
o Did you notice any bleeding?
o Did you notice bleeding from your gums / nose / coughing or vomiting blood / bruises in
your body / dark urine / urine in stools?
o Any weakness / numbness / difficulty finding words / vision difficulty?
o Did any one tell you that you look pale? Do you feel fatigued?

Based on what you have told me, there are no obvious serious consequences, if it is ok with you, we
can discuss your situation now!

 Red flags: for FEMALES


─ Are you taking any OCPs? OCPs might increase the risk of developing DVT.
─ Are you pregnant? Warfarin is not to be used during pregnancy; we will use heparin instead of it.
─ Have you had any vaginal bleeding?

OSCE-guide-III.doc Page 57 of 255


History taking – Medicine

 What is your understanding about DVT?


─ Causes:
o It might happen after prolonged sitting without movements (like very long flights)
o Or due to certain medical condition,
The blood tends to form clots in the deep veins of the lower extremities
─ Complication:
o Relapse 8%: without treatment, and that is concerning!
 Whenever we treat the patient, our target is to decrease the relapse rate to 0.8%
which is 1/10 of the risk without treatment
o These clots are not fixed, and sometimes they get dislodged from your leg and travel
along your blood vessels, all the way to the lungs (chances are 3%):
 If large enough  might cause sudden death
 If showers of small clots  you may not feel anything right now, but it later will
cause what we call “pulmonary HTN”, which is a debilitating disease, with serious
consequences and we do not have treatment for it right now
o Always in medicine, we try to balance the benefits and the side effects, and in this
condition, the benefits largely outweigh the risks.

 Now, what do you know about blood thinners?


o It is a medication used to make our blood thin, preventing our body from forming clots
by competing with vitamin K, which is needed for the formation of the elements of blood
clots.
o We take warfarin seriously, and that is why we monitor it closely and regularly, by
assessing the INR which is an indicator of the effect of warfarin. Therefore, as long as
your INR is within your target, the risk of bleeding is less than 1%, and almost near 0%
to have intra-cranial (brain) hemorrhage without having external bleeding first. That is
why you need to keep monitoring yourself, and seek medical attention if you notice any
signs of bleeding.

 Decision:
If the patient decides that he will restart the treatment:
─ We will do it the same way as we did the first time:
o We will start heparin and warfarin together then stop heparin after 3 days
─ We will need to measure the INR daily (till we reach our target) then twice a week, then weekly,
then every 2-4 weeks

NOTES:
─ Numbers to remember:
o Relapse (recurrence) of the DVT: 8% without treatment and 0.8% with treatment.
o Possibility of DVT  clots and PE: 3%
o Chances of having bleeding with warfarin: 1%, and almost near 0% chance of having
intra-cranial bleeding without having an extra-cranial bleeding.
─ The initial DVT counselling should have been done in the first time, when the patient was
diagnosed; which includes:
o General knowledge about DVT
o Causes and risk factors
o INR follow up
─ My best friend was taking warfarin, and he had brain hemorrhage!
o I am sorry to hear that, this must be stressing / worrying, especially that you are taking
the same medication and he is a close friend to you.
o We prescribe warfarin for many reasons, the issue here is that your friend was not my
patient, and I do not know about his condition, so I am not in a position to comment on
this situation.
o I am glad you came here today, so that we can discuss this together.

OSCE-guide-III.doc Page 58 of 255


History taking – Medicine

Patient is receiving blood – counsel for adverse reactions

You were called to assess a patient who is receiving blood, and the nurse has concerns.

Adverse effects of blood transfusion:


 Febrile reaction: most common / not serious
 Anaphylactic reaction: not common / serious
 Haemolytic reaction: not common / serious

Introduction to nurse / what is your concerns / ethical challenge


Introduction to patient / ethical challenge
 ABCD
 History
 Brief physical exam
Adverse reactions of blood transfusion
Plan

 Introduction to nurse / what is your concerns / ethical challenge


─ Good afternoon, I am Dr … May I get your name please?
─ How can I help you? OR What are your concerns?
o There is a mistake!
─ What do you mean by mistake? How is the patient doing?
o This was wrong blood!
─ Did you stop the transfusion?
o Yes
─ That is great, this is the first step in the right direction
─ Now, what do you mean by “wrong blood”? Is it the same bld group or no? Cross matched or not?
Do we have the patient name on the units?
o It is the same group but with other patient name
o Doctor, please do not tell the patient!
─ Why? Do you have any concerns?
o I might be fired!
─ I see you have concerns here, but we need to stabilize the patient first. Then we will speak about
that. However, we need to investigate before making decisions.
─ Can you tell me when did this happen? How much did he receive?

 Introduction to patient / ethical challenge


─ Good afternoon Mr …, I am Dr … The nurse was updating me about your condition. It looks like
there was an unintentional medical error took place, and I need to make sure you are ok.
o Is it serious doctor?
─ Could be! There are different possibilities; I need first to check you.
o Whose mistake is this? Is it the nurse mistake?
─ Usually in the blood transfusion process, there are many steps; any one of those might go wrong.
It is early now to judge. I need first to make sure you are ok and stable, and then I will file an
incidence report. Investigations will be done, and you will be informed with the results.

 ABCD
Let us make sure you are safe and stable first.
AB:
─ Can you please open your mouth? Mouth is clear with no swelling. Do you have any itchiness or
swelling in your mouth?
─ Trachea is central, no engorged jugular veins. Can I listen to your heart please! Normal heart
sounds.

OSCE-guide-III.doc Page 59 of 255


History taking – Medicine

C:
─ Can I know the vitals please? Normal / stable.
─ Can you remove the blood unit please, and send it to the blood bank. We need to re-cross this
patient blood with this unit.
─ Can you put another IV line please! We need to take samples for: CBC / differential / lytes / blood
grouping and re-crossing / haptoglobin / bilirubin level
D:
─ I am going to shine light in your eyes!
─ Can you hold my fingers please? Do not let go.
─ Do you feel me touching you?
─ Patient is grossly neurologically free.

─ If fever: give 2 tablets Tylenol 325 mg


─ Can you please prepare:
o Allergic reaction: Benadryl
o Anaphylactic reaction: Benadryl / Epinephrine / Steroids
o Haemolytic reaction: Diuretics and fluids

 History
Now, I would like to ask you some questions:
─ Why are you taking blood? They have found that I have anemia
─ Did you take blood before? Or is this the first time?
─ Do you feel warm? Shivering? Chills?
─ Do you feel any itching or swelling in your lips / mouth?
─ Any heart racing? SOB? Wheezes? Dizziness?
─ Any flank pain? Back pain? Weakness?

 Brief physical exam


─ No IV line oozing / No hives on skin / No mouth swelling
─ Listen to heart / lungs  clear
─ Press on flanks / spine  no tenderness

 Adverse reactions of blood transfusion


─ Blood transfusion is a commonly used procedure, and it is life saving. A lot of precautions are
taken to make sure it is completed safely. However, like any other medical intervention, it has
some side effects.
─ The most common reaction that might happen is called “febrile reaction”. This is not serious
reaction and it is self limited. It might happen again, so if it happens, next time we give you
Tylenol before the transfusion.
─ Another adverse effect, which is less common but more serious, it is called “anaphylactic
reaction”. This is a form of severe and serious sensitivity reaction, in which the blood pressure
drops suddenly, and there is a swelling of the tongue, lips, and mouth, with difficult breathing. We
do not have a method to predict it. However, based on your symptoms, your physical exam and
vital signs it is less likely you have that.
─ The third adverse effect is called “hemolytic reaction”, and it happens if the patient receives blood
that belongs to another blood group. It causes damage to blood cells which leads to back pain and
flank pain, and could have serious consequences.
Again, based on your symptoms, physical exam and vital signs, your condition does not cope with
this reaction too. And the fact that you received blood from the same blood group makes it less
likely you will have hemolytic reaction.
─ We prepared medications to deal with any reaction and we will keep you for a while to monitor
you, to make sure that will not happen.

 Plan
─ Call the blood bank to withhold the other units (previously cross-matched)
─ File an incident report

OSCE-guide-III.doc Page 60 of 255


History taking – Medicine

Counselling – Ventilator

Mr Johnson is 75 years old gentleman, his life-long wife for 50 years has a terminal COPD, with
severe pneumonia, and she is on ventilator for the last 3 weeks, and it is not possible to wean her
from ventilator, you called him to inform him about the condition.

 What do you know about your wife’s condition?


─ Listen carefully
─ Show understanding and empathy
 Give information about her condition,
─ Give clear simple information
─ Stress on the progressive, irreversible nature of the disease

 Give alternatives:
─ Remain on ventilator, with no evidence that she will be able to breath by own, and with
the possibilities of fatal complications like infections, bed sores, … Some people does not
like to have this quality of life
─ Stop the ventilator and she will pass away in peace
 As regarding her condition now, have you ever discussed this with her? Has she ever
expressed her wishes about what would she like to be done to her if she needs to be
resuscitated or put on ventilator? Does she have any advance directives or living will?
 What do you think about this now?

 Offer time if he needs to discuss it with other close family members, or if he needs to arrange
any thing (e.g. I am just giving you information, and we can arrange a meeting with the
family within 2 days so that I can explain to them).

 What if she does not want to be on ventilator but he would like to leave her on the ventilator?
─ Mr Johnson, I am sorry to tell you that, actually it is not our decision or your decision, it
is her choice. And she expressed her wishes before; she decided that she does not want to
have this poor quality of life. We have to respect her wishes.

OSCE-guide-III.doc Page 61 of 255


History taking – Medicine

Ethical questions

 Patient has the right to access his/her medical file, we can not withhold it
 Patient wants to leave you as family physician  it is his right, and he/she has the right to
take all his/her medical data and file
 If you want to terminate a patient from not seeing you as family physician:
o Give proper notice period
o See him/her for emergency
 Confidentiality; when to break confidentiality? To report for the ministry of transportation for
example:
o Dementia / delirium
o Vision problems
o Seizure disorders
o Schizophrenia (case-based)
o Heart attack  1 month not allowed to drive
o Alcoholic with liver failure (based on Child’s criteria: albumin / ascites / INR /
bilirubin)
 Report for child safety  CAS (Children Aid Society)
o Even if POTENTIAL or SUSPECTED
o Child neglect / abuse
 Patient wants to leave hospital against medical advice; e.g. patient has just had a heart attack,
and still insists to leave the hospital!
o I would like to make sure he is competent, not under influence of alcohol or any
substance, and to rule out suicidal ideation
o I would explain to the patient: diagnosis / treatment / side effects of treatment /
complications of not receiving treatment / alternatives
o I will document this, and I will ask the patient to sign a LAMA (leaving against
medical advice), and I will let him go
 Biological parent wants to know the medical details of his/her son, who is adopted by another
family!
o In order to determine whether I should release any information or no, I would
like first to know who has the legal custody (guardian) of this child. It might be
the adopting father, a social worker (case manager) …
 Any unconscious patient  ask for DNR or advanced directives
 MMS exam score < 24  patient is incompetent;
o You have a case of patient, who had surgery, is taking medications, but he
developed delirium post-operative and now he wants to discontinue his
medications  NO; he is delirious, incompetent to change decisions, he already
consented to take the medications before he entered this delirium.
o What if this patient broke his leg; do you want to operate him without consent?
This is a new condition; we do not know what would be his competent wishes 
look for SDM (substitute decision maker).

OSCE-guide-III.doc Page 62 of 255


History taking – Medicine

HIV post-test counselling

─ Usually you are covering for other physician to give the test result — which means this is a new pt
to you.
─ Be sensitive, empathetic, and flexible

 Introduction:
Your Dr. is away, I am covering for him/her, and I have your file with me, I just need to understand the
situation here,
─ What have you discussed last time?
─ Why did you ask for the test last time?
─ Did you feel sick in any way?
─ Was there anything made you worried about your own health?

 Give the test result:


─ It is positive, sure about the result, because they do two tests before giving the positive result.
─ Check with patient’s reaction: normalize patient’s feeling; all the feelings you are having now
are very normal.
─ What kind of thoughts are going through in your mind? What concerns you the most right
now?
─ AIDS: don’t have AIDS, will not die tomorrow or so. Prognosis is variable, but many people carry
it without feeling it for quite long time, years

 Consequences of HIV:
─ Repeated infections / LNs
─ Tired / fatigue
─ Memory – dementia
─ Depression

 Causes of HIV:
─ SAD – shared needles
─ Sexual:
o Risky behaviour
o Confidentiality – how to inform the partner?
 Get the background info: duration of the relationship, how close to each other,
 Partner has to know: Risk of infection / Needs to be tested
 Will know anyway, either from public health or him. Prefer him to tell, offer help to
tell.

 Before discharge the patient:


─ Arrange follow-up visit in the next couple days after patient digests the info
─ Make sure that patient is safe to go home, safe to drive back, no suicidal or homicidal ideation,
inquire about support system.
─ Education: emphasize the importance of safe sex: advice use barrier contraceptive methods all the
time with all partners in the future to prevent the transmission.

Case: HIV patient with diplopia  cranial nerve examination


─ CT brain: enhanced multiple rings  toxoplasmosis with HIV
─ DD: TB / toxoplasmosis / CMV / CNS lymphoma
─ Management: refer to infectious disease specialist

OSCE-guide-III.doc Page 63 of 255


History taking – Medicine

Lung Nodule
 Introduction:
─ Why X-ray was taken? When?
─ When was last normal X-ray? Do we have it?
 Give the test result:
─ Solitary Lung Nodule. Definition: a round or oval, sharply circumscribed radiographic lesion, size
up to 3-4 cm, which may or may not be calcified, and is surrounded by normal lung. Can be
benign or malignant
─ Any ideas about what could be causing this nodule

 Consequences:
─ Local symptoms: cough, phlegm, haemoptysis, SOB, wheezing
─ Constitutional symptoms: fever, chills, night sweat / change of appetite, weight loss, fatigue /
pumps or lumps in the neck or elsewhere in the body
─ Impact / screen for metastasis:
o Brain: headache/ vomiting?
o Back: back pain/ weakness/ numbness/ tingling in arms or legs?
o Lungs: chest pain/ cough/ phlegm/ wheezes/ heart racing?
o Liver: yellow discoloration/ itching/ urine/ stools?

 Causes:
─ Smoking
─ Exposure to chemicals / smokes at work
─ T.B.: Contact with sick person (T.B.) / Recent travel / T.B. skin test
─ Sarcoidosis: associated symptoms; joint pain, skin rash
─ History of lung disease
─ History of cancer
─ HIV status
─ Family History of T.B. or Lung cancer

 Management:
─ Investigations
o CXR: always compare with previous CXR
o CT densitometry and contrast enhanced CT of the thorax
• Sputum cytology / stains
• TB skin test
o Biopsy: bronchoscopic or percutaneous(CT-guided) or excision (thoracoscopy or
thoracotomy): if clinical and radiographic features do not help distinguish between
benign or malignant lesion
 If at risk for lung cancer, biopsy may be performed regardless of radiographic
features
 If a biopsy is non-diagnostic, whether to observe, re-biopsy or resect will depend on
the level of suspicion
o PET scan not yet routine but can help distinguish benign from malignant nodules
• Watchful waiting: repeat CXR and/or CT scan at 3, 6, 12 months
─ Algorithm: Evaluation of a Solitary Pulmonary Nodule; check previous CXR
o Looks benign or unchanged  repeat CXR q 3-6 months for 2 years
o Significant risk factor on history or looks malignant or changed  CT chest
 Cause (infection or cancer)  stage and treat
 Calcification  observe
 No diagnosis  trans-thoracic needle biopsy
─ Inflammatory  treat the cause
─ Cancer  stage and treat
─ Still NO diagnosis  resect for diagnosis

OSCE-guide-III.doc Page 64 of 255


History taking – Medicine

 Past Medical History


 Family history: of T.B. or Lung cancer
 Social History

Evaluation of a Solitary Pulmonary Nodule:

OSCE-guide-III.doc Page 65 of 255


History taking – Medicine

High Creatinine

 Introduction:
─ Why the test was done?
─ When was the last normal test?
─ Any idea about the meaning of the test

 Give the test result:


 Consequences: manifestations of renal failure


─ Nausea and vomiting / stomach pain
─ Itching
─ Pallor
─ Fatigue
─ S.O.B with exertion
─ Swelling (ankle, around eye)
─ Bone pain

 Causes:
─ Renal:
o Hypertension
o Diabetes
o Repeated kidneys infection
o Poly-cystic kidneys
o Medications: NSAIDs / gold / penicellamine / ACEIs
─ Post-renal:
o Kidney stones
o Bladder cancer
o Prostate problem

 Past Medical History:


o Kidney disease
o Previous hospitalization
o Nephrectomy
o Allergies
 Family history: of renal problems
 Social History
o Smoking
o Alcohol
o Drugs
o Work
o Home
o Support

OSCE-guide-III.doc Page 66 of 255


History taking – Medicine

Impotence / Erectile Dysfunction

 Introduction:
─ ED is a common problem in men, with a broad DD, encompassing organic & psychogenic causes.
This is often a difficult topic for men to discuss with their doctor. Confidentiality.
─ Penile erection is a multi-factorial process dependent on integration of neurologic, hormonal,
vascular and emotional factors.
 Analysis of the CC:
─ Primary vs. Secondary
o Chronology (frequency, onset, duration, course)
 Onset: acute (more likely psychogenic) or gradual (organic)?
 Course: intermittent (more likely psychogenic)? Libido affected?
o Severity or amount? All the time?
o Aggravating / precipitating and alleviating factors
─ Organic vs. Psychogenic
o Do you have early morning erection?
o Do you have night time emissions?
o Do you have desire?
o Are you able to masturbate to an erection or climax?
o Situational dysfunction; does function vary depending on the setting? Partner / Place /
Time?
 Consequences: How does this affect your life? Your relationship?
 Causes:
─ Many endocrine disorders and systemic diseases cause ED by influencing libido, autonomic
pathways and/or blood flow.
─ Organic causes:
o Medical causes: history of DM, HTN, hyperlipidemia, peripheral vascular disease,
intermittent claudication
o Neuro: back trauma / constitutional symptoms (back metastasis) / back pain / weakness,
numbness / history of MS,
o Low testosterone: changes in secondary sex characteristics, e.g. hair pattern changes /
history of gynecomastia / galactorrhea / history of thyroid disease / pituitary disease (
visual defect, headache)
o Medications; e.g. anti-depressants, hormonal treatment, opioids, MAO inhibitors
o SAD: smoking / alcohol / recreational drugs
─ Psychogenic causes:
o Any problems with their partner(s)
o History / screening of depression
o Any recent changes in life (home, work, socially) / anxiety attacks? Any stress? Past life-
background, upbringing, …

Counselling:
─ Normalize patient feelings
─ ED can often be improved with:
o Life style modifications: exercise / weight loss / improved diet / DM control / smoking
cessation / ↓ alcohol / stress management / ↓ anxiety / sleep hygiene
o Improvement of patient relationship with partner: marital counselling / address sexual
boredom / refer to specialist in sexual education and therapy
─ Unfortunately, many organic causes are irreversible, but we have treatment options:
o Testosterone preparations (if low testosterone)
o Viagra or Cialis
o Penile self-injection
o Vacuum – rubber ring device
o Penile prosthesis
─ Follow-up appointment for BOTH partners

OSCE-guide-III.doc Page 67 of 255


History taking – Medicine

Rheumatology – History Taking

 Chief complaint: pain / stiffness / weakness / deformity / limitation of movement / joint


clicking
─ OCD / Acute (< 6 weeks) versus chronic (> 6 weeks)
─ PQRST / ↑↓

 Associated Symptoms:
─ Morning stiffness
o Inflammation: morning stiffness (>30 min), better with use, constitutional
symptoms
o Non-inflammatory: worse with use, worse at end of day, can have some stiffness
but usually not prolonged
─ Joint swelling / redness
─ Other Joints / Pattern of joint involvement:
o Mono-arthritis, oligo-arthritis (4 or less), poly-arthritis (5 or more)
o Symmetric vs. asymmetric
o Peripheral joints versus axial involvement (spine, SI joints)
o Small joints (hands / feet) versus large joints (hips / shoulders)
o Additive joints vs. migratory joints
o Tendon involvement
─ Constitutional symptoms
─ Extra-articular features:
Seropositive (e.g. RA, SLE, Sjogren’s, scleroderma, inflammatory, myositis)
Seronegative (Ankylosing spondylitis, psoriatic arthritis, enteropathic arthropathy,
reactive arthritis)
o Eyes: iritis, scleritis, conjunctivitis, dry eyes
o Oral ulcers
o Respiratory: pleural effusion, pleuritis, pulmonary fibrosis, pulmonary nodules
o Cardiac: pericarditis, pericardial effusion, conduction defects
o GIT: GERD, inflammatory bowel disease, malabsorption, bloody diarrhea
o Dermatology: malar rash, discoid, nodules, telangiectasias, sclerodactyly,
calcinosis, alopecia, periungal erythema, psoriasis, nail pitting, onycholysis,
erythema nodosum, pyoderma gangrenosum
─ Crystal arthropathies
o Mono-arthritis (red, hot), chronically can be poly-arthritis: gout (tophi, alcohol
history, renal failure, drugs)
o CPPD (hyperparathyroidism, hypomagnesemia, hemochromatosis, Wilson’s
disease, hypothyroidism)
─ Septic arthritis: usually mono-arthritis, fever, red, hot. Gonococcal arthritis can be
migratory, with tenosynovitis and skin pustules

 Disability and adaptation


─ How does it affect your life? Ask about daily activity!
─ Effects on ability to work! Support!

OSCE-guide-III.doc Page 68 of 255


History taking – Medicine

Multiple Sclerosis

Middle aged man (or woman) with episodes of numbness in one leg.
─ History: Review of systems
─ Diagnosis: MS
─ Investigations: MRI / CSF

 History of present illness (neurological screening)


─ Ethnic background.
─ History of weakness: MS can mimic ANY neurological disease process.
o Determine which groups of muscles are involved: proximal vs. Distal, symmetric
vs. Asymmetric
o Pattern of weakness: stepwise decline, relapse and remitting
─ Dysphagia, dysarthria
─ Dry eyes, dry mouth
─ Numbness in upper or lower limbs
─ History of previous vertigo, loss of vision (optic neuritis), diplopia,
─ Clumsiness, loss of balance, falls
─ Any associated sensory symptoms
─ Bowel or bladder dysfunction (difficulty fully emptying bladder, urgency, incontinence)
─ Fatigue, arthralgia, depression, behavioural changes
─ Constitutional symptoms
─ Impact on daily functioning

─ Lhermitte's sign: electrical sensation down back on neck flexion


─ Uhthoff's phenomenon: worsening of symptoms in heat (hot bath, summer)

 Past Medical History


─ Previous transient focal neurological deficits: vertigo, sensory deficits, etc.
─ History of headache
─ Sarcoidosis
─ HIV status
─ Systemic inflammatory disease

 Family History: MS, SLE, weakness NYD, visual deficits NYD

OSCE-guide-III.doc Page 69 of 255


History taking – Medicine

Obesity

 Analysis of chief complaint:


─ Weight analysis: now and how about one year ago? Maximum and minimum weights?
─ Weight loss history:
o Attempts, medically vs. surgically
o What weight achieved, what caused failure
o Assistance: dietition, exercises programs
o Explore why patient want to do it again
o Admire and encourage the pt
─ General health now:
o HTN, IHD
o DM, cholesterol
o Sleep apnea, asthma
o OA, functional impairment, disabilities, social activities involvement
o Psychological assessment: embarrassment, low self esteem, depression, anxiety
─ Diet: detailed history; how many meals, how much, bedtime meals,
─ Exercises: how often, type

 Past medical history: AMPLE,


 Social history: SAD, social activities, home and work environments
 Family history: obesity in the family

Counselling:

 Encouragement: admire patient, it is important for your general health, requires a lot of effort;
it is very difficult process, very common multiple tries.
 Methods:
─ Set up a goal first, start slowly
─ Diet: can refer you to a dietition
o Type of food: high fibre, vegetables and fruits, less fat/cholesterol, low
carbohydrate,
o Caloric intake should be calculated /+/ does not exceed 1800 Cal/d
o I will give you tables and graphs to show you the ideal meal composition, but
generally, lunch and supper must be formed of: 50% vegetables and fruits /+/
25% protein /+/ 25% carbohydrates
─ Exercises:
o Program: 3-5 times per week /+/ 30-50 min each time
o Set up personal instructor to guide
─ Medications: locally to absorb fats or centrally working on the satiety centre; do not like
to start with
─ Surgical procedures, in very advanced cases and there is medical impairment, we can
discuss it later.

─ Avoid: smoking / alcohol /+/ Healthful life style.


─ Educational information: group program / booklet and brochures
─ Follow up on regular basis

OSCE-guide-III.doc Page 70 of 255


History taking – Medicine

Epilepsy Counselling

 Young 16 yr old male for driving license counsel

Introduction  Why does the patient want a note from doctor for a driver’s license?
Usually Dr does not give such note unless there is underlying condition!
Analyze  Age of onset? / When was the diagnosis? / What was the diagnosis?
epilepsy history  How frequently do the attacks occur?
 How long does each attack last? ± LOC
 Aura prior to attack?
 When was last attack? Similar to previous ones?
 What happens during an attack? Does the patient shake / all over / partly /
roll up eyes/ bite tongue?
 How do you regain consciousness / how do you feel after the attack
Triggers  Which medication does the patient take? Compliance? When was the drug
level checked?
 Any other medications that might interact with epileptic drugs?
 Sleep deprivation / Long screen time before sleep?
 Alcohol? Stimulants?
 Are you under stress
MOAPS Scan the mood and anxiety
HEAD SSS Home / Education / SAD (do you take stimulants)

 Counselling: What is your understanding of seizures?


─ It is a common condition due to increased electrical activity in the brain, causing abnormal body
movements. Some people lose consciousness, and some do not. Usually, it does not cause learning
disability or brain damage
─ The diagnosis of epilepsy requires the occurrence of at least 2 unprovoked seizures 24 hours apart
─ Most of patients have no clear reason to explain why they are having this, but it tends to run in the
family (idiopathic) or structural brain damage (post-meningitis)
─ Other conditions that should be considered include: Syncope (arrhythmias), Vascular (TIAs),
Metabolic (hypoglycemia), Psychiatric (conversion, panic attacks, malingering)
─ Prognosis: The patient's prognosis for disability and for a recurrence of epileptic seizures depends
on the type of epileptic seizure and the epileptic syndrome in question. Regarding morbidity,
trauma is not uncommon. Regarding mortality, seizures cause death in a small proportion of
individuals. Most deaths are accidental due to impaired consciousness

 Plan:
─ Diagnostic workup
─ Patient education
─ Treatment
─ Pregnancy

─ Diagnostic workup:
o Two imaging studies must be performed after a seizure. They are neuro-imaging
evaluation (MRI or CT) and electroencephalography (EEG).
o Lumbar puncture for CSF examination has a role in the patient with obtundation or in
patients in whom meningitis or encephalitis is suspected.
o Metabolic screen
o Serum studies of anticonvulsant agents (e.g. phenytoin); if therapeutic level but side
effects or poor seizure control  add another drug (carbamazepine / valproic acid)

OSCE-guide-III.doc Page 71 of 255


History taking – Medicine

─ Patient education:
o Dangerous activities: to prevent injury, educate patients about seizure precautions. Most
accidents occur when patients have impaired consciousness. Restrictions apply on:
 Driving (report to ministry of transportation), must be seizures-free for more than 1
year
 Diving, swimming, hiking, mountain climbing
 Taking unsupervised baths, better take shower not bath, with open door
 Working at significant heights, operating machines and the use of fire and power
tools.
o Avoid the triggers for seizure attack:
 Alcohol will exacerbate (chronic alcohol: ↓ blood level of anti-epileptics due to ↑
metabolism / excess alcohol: ↓ seizure threshold)
 Stress; if the patient is having stress / anxiety / alcohol issues: counsel and offer
social support
 Sleep deprivation / long screen time before sleep
 Head trauma,
 Forgetting to take medication on time
 Taking other medications that interact with the treatment
o Life style:
 You have to take the treatment almost for your whole life
 Talk with your physician about any new medication you want to take
 Medications are teratogenic, females to take proper contraceptive measures
 Patient might choose to wear a bracelet indicating he has epilepsy
 If a seizure will happen: go to the ER
 Regular follow-up visits and monitoring of anti-convulsion level in blood
─ Treatment:
o The mainstay of therapy for people with recurrent unprovoked seizures is an
anticonvulsant. If a patient has had more than 1 seizure, administration of an
anticonvulsant is recommended. However, standard of care for a single, unprovoked
seizure is avoidance of typical precipitants (e.g. alcohol, sleep deprivation); no
anticonvulsants are recommended unless the patient has risk factors for recurrence
o Medications will be taken for long term, there are many options, will start with one
medication, if no full control, we may increase the dose and/or add another drug
o Side effects of medications: movement disorders (ataxia, dysarthria), teratogenic, liver,
kidney, drowsiness, poor concentration
o Discontinuation: After a person has been seizure free for typically 2-5 years, physicians
consider discontinuing the medication. About 75% of relapses after discontinuation occur
in the first year, and at least 50% of patients who have another seizure do so in the first 3
months. Therefore, patients to observe strict seizure precautions (including not driving)
during tapering and for at least 3 months after discontinuation. Authors recommend that
anticonvulsants be gradually discontinued over 10 weeks
─ Pregnancy:
 Are you sexually active?
 Do you take use contraception?
o No  Are you planning to get pregnant? Yes! Let us talk about pregnancy and the meds
you will start. Can you postpone the pregnancy for a while? It is better to have good
control of seizures for a while; to get any seizure during pregnancy will pose great risk
for both of you and baby. And the medications can cause serious malformation to the
baby
o Yes is it OCPs? Yes! There might be drug interaction, so for the time being you need
to continue to use your pills and add another method (mechanical) till you contact your
gynecologist

OSCE-guide-III.doc Page 72 of 255


History taking – Medicine

Medical note

 For past date!


 For headache

1. What type of doctor’s note pt wants; reason for the note?

2. Analyze the case  headache case  full history

3. Deal with the patient request:


 Mention that patient is probably suffering from migraine or tension headache.
 Mention that you can’t write note for previous visit; but can give copy of today’s visit and
notes about what happened;
 Encourage the patient to talk to the tutor (or supervisor) frankly about what happened.
Mention you believe the professor would be reasonable; ask about any possibility to make up
the class he missed;
 Mention you are willing to do any thing to help him out; and apologize that you can’t lie or
write a note for the pt you didn’t see at that time;
 Mention you are happy to see this patient again to see how things going after he talks to
supervisor and for follow-up regarding his headache.

OSCE-guide-III.doc Page 73 of 255


History taking – Medicine

Pre-diabetes – Counselling
 What is DM?
o Fasting blood sugar (FBS) > 7 mmol/L
o Random blood sugar (RBS) > 10 mmol/L + symptoms
o Glucose tolerance test (GTT) > 11.1 mmol/L
 What is pre-diabetes? Impaired glucose tolerance
o Fasting blood sugar (FBS) 6.1 – 6..9 mmol/L
o Glucose tolerance test (GTT) 7.8 – 11 mmol/L

Introduction  Pre-diabetes: does not mean that you have diabetes, but it shows that you have an
increased chance of having it, about 1–5% per year. It also shows increased risk of
you having complications in the large blood vessels causing heart diseases, strokes
and peripheral vessel diseases
 Diabetes:
─ Increase of blood sugar in our blood due to deficient or ineffective insulin.
─ Explain the role of insulin in helping cells to utilize glucose, two types of DM,
type I and type II.
─ With one reading we can not say that you are prone or have DM, so let me ask
few questions, to see if you have the symptoms of DM!

Impact Symptoms of  Eat more, drink more, pee more even at night
hyperglycemia  Blurred vision
 Tired / weight loss
 Yeast infections, are there itching / rashes in your groins, in the
toes and finger webs?
 Do your wounds get long time to heal?
Symptoms of N/V, abdominal pain, dehydration, LOC
Ketoacidosis
Symptoms of If patient is on insulin: sweating, shaking, palpitation, fatigue,
hypoglycaemia headache, confusion, seizures
Complications of  Micro-vascular: nephropathy / neuropathy / retinopathy
high blood sugar  Macro-vascular: CAD / peripheral arterial dis / impotence
Red flags  Lifestyle: too much simple sugars, lack of exercises, overweight, family history
 Medications: steroids / beta blockers (β-blockers are contraindicated in DM: it causes
hyperglycemia / and it masks hypoglycemia)

PMH Medications: used long term steroids, thiazides, phenytoin, clozapine or other anti-
psychotics, HTN, Cholesterol, CAD, CVD, kidney, hospitalization
FH DM in first degree relatives
SH  Sexual function: any concerns
 Smoking
 From the conversation we had, it looks like you are likely to get DM. However I am going to examine
you and do blood tests (FBS, Hb A1C – which shows your blood sugar level over the past 3 months,
lipid profile, micro albumin / Cr ratio, ECG).
 I strongly recommend you to work on lowering your chance of having diabetes by half by: watching
your diet (healthy balanced diet, avoid saturated fats and simple sugars, choose low glycemic content
foods), exercising (30 -45 min of moderate exercise for 4-5 days/wk) and life style changes (limit Na,
alcohol, caffeine, stop smoking).
 I can refer you to diabetes educational program if you wish.
 Treatment targets: Hb A1C < 7 FBS 4 – 6
Lipids: LDL < 2, Triglycerides < 1.5 or TC/HDL < 4 BP < 130/80

OSCE-guide-III.doc Page 74 of 255


Emergency Medicine

Emergency Medicine

OSCE-guide-III.doc Page 75 of 255


Emergency Medicine

Emergency Room

Trauma Non-trauma

ATLS: Advanced Trauma Cardiac Non cardiac


Life Support protocol
Primary survey: Chest pain: - Acute abdomen for 24 hrs,
- Any trauma (kicked, car - MONA ± β-blockers ↓BP, ↑HR
accident, thrown from - STEMI: catheterization, - Upper / lower GIT
height, …) thrombolytics bleeding
- Manage over the phone: - No ST elevation: heparin, - Severe headache for 2
trauma or meningitis angiography hours
- Secondary survey: patient Arrhythmias: - Seizure for the last 20
in the ER after car - V Fib minutes
accident, primary survey - V Tachy - 16 years brought
was done, do the ACLS: Advanced Cardiac unconscious to the ER
secondary survey Life Support code - Patient receiving blood,
N.B. if knife: leave it in place, Heart block: old patient counsel him
fix with gauze - DNR
- Advanced directive

Management:

Trauma Medical
I I
A A
B
B C
D
OCD
C PQRST
↑↓
D Associated symptoms
Risk factors
AMPLE PMH
Head to toe Focused physical exam
Management Management

OSCE-guide-III.doc Page 76 of 255


Emergency Medicine

Trauma

I: introduction:
- Because it is a trauma case, I would like to activate the ATLS protocol
- I would like also to get protection for me and my team; gloves, gowns, goggles and
masks

- Mr … I am Dr …, the physician in charge in the emergency room;


o If the patient is wearing a neck collar, proceed
o Pt is not wearing a neck collar: Mr … please do not move your head, nurse can
you please fix his head, we need to put neck collar

- I understand that you are here because you had a car accident
- How are you feeling / doing right now?
o I would like to make sure that you are stable, I will check with the nurse and we
will start the management then I will be asking you more questions.
o I can see that you are in a lot of pain, please bear with me for few minutes, and I
will give pain killer as soon as I can.
o Doctor, where is my wife? How is she doing? Was she with you? I can see that
you are concerned about your wife, I will look for her and I will get back to you
as soon as I can, meanwhile my first concern is to make sure you are stable

We will start the primary survey now:


Airways / Breathing
Mouth / O2 / inspect chest, neck / listen to lungs, heart

Can you please open your mouth? Mouth is clear; no FB, no dentures, no vomitus
Pt is talking to me that means airways are patent
Nurse, what is O2 saturation, plz? Can you give him O2 – 4 L with a nasal canula
Any change in saturation? Can you plz let me know if
any change in saturation happens!
Inspect the chest By inspection, chest is symmetrical, no bruises, no
open wounds, no paradoxical movements of the chest,
no use of accessory muscles for breathing
Open the collar window, or fix pt head Trachea is central, JV not engorged, bilateral air entry,
and remove anterior part: normal heart sounds (HS) S1 and S2
─ Trachea Trachea JV Air HS Diagnosis
─ Jugular veins (JV) entry
shifted Engorged ↓ same normal tension
away side pneumo-
Listen to lungs thorax
shifted depleted ↓ same normal Hemo-
Listen to heart sounds same side thorax
side
central engorged bilatera muffle cardiac
l d temponad
e
Usually no cardiac temponade in the exam

OSCE-guide-III.doc Page 77 of 255


Emergency Medicine

+ If ↓↓ BP and ↑↑ HR / other signs of tension pneumothorax  nurse, I need to put a large needle
(16 / 14 G) in the 2nd intercostal space at MCL (upper border of the 3rd rib);
─ Is there any gush of air?
─ Check the trachea centrality and air entry
─ We need to put a chest tube in the 5th intercostal space
+ If ↓ BP and ↑ HR / other signs of hemothorax  nurse, I need to put chest tube in the 5th
intercostal space at anterior Axillary line;
─ What is the amount of blood?
─ If > 1.5 L  stat surgery
─ Otherwise, monitor; if > 200 ml/hr  surgery

Circulation
Vital signs / fluids / withdraw blood samples / look for source of bleeding
Can I get the vital signs please Comment, patient is … hypo- / hyper- / tension,
comment on HR, pt is stable / unstable
I would like to have two large IV lines, 16 G in both anti-cubital fossae:
─ One to start fluids: bolus 2 L ringer lactate or normal saline
─ The other line is to withdraw samples for: CBC/differential/lytes /+/ blood grouping and
cross matching / and prepare 6 units of blood (4 matched and 2 “O”) /+/ stat glucose /+/
INR/PTT/LFT /+/ Bun/creatinine /+/ toxic screen/alcohol level /+/ continuous cardiac
monitoring/cardiac enzymes and ECG
Can you please inform me with the vitals; after the bolus fluid is
done and every 5-10 minutes or if there is a change in the vitals
Look for the source of bleeding
Abdomen:  Inspect the abdomen  bruises
I am going to look at and feel your  Palpate the abdomen  rigidity and guarding
abdomen  If positive; I am suspecting intra-abdominal
bleeding, I would like:
─ To get stat surgery consult
─ To arrange for FAST (focused abdominal
sonogram for trauma)
─ To do DPL (peritoneal lavage)
I am going to press on your pelvis  Press from the sides
 Press open book
 If positive; I am suspecting pelvic fracture:
─ Cut pt sheet and wrap around the pelvis to
support, and check blood on penile meatus
─ Stat orthopedics consult
Lower extremities By inspection, patient lower extremities are
symmetrical, no abnormal posture or deformity. No
inequality in length, no pain, no deviation
 If positive: I am suspecting femur fracture;
─ Check the pulses
─ Thomas splint
─ Stat orthopedics consult
Log rolling I need more team members to roll the patient on his left
side:
 To check for external source of bleeding
 To press on the spinal processes
 To perform digital rectal exam

OSCE-guide-III.doc Page 78 of 255


Emergency Medicine

I would like to get trauma X-ray series: for neck, chest, LSS and pelvis

D:
D1: Deficits / Disability D2: Detoxification D3: Drugs
Neuro screen /
I am going to shine light in your eyes? Pupils are round, symmetrical and reactive
Can you please squeeze my fingers, do Patient is gross neurologically free
not let them go
Can you wiggle your toes?
Do you feel me touching you here,
here, and here
Glasgow coma scale – eyes Alert 4 Pain 2
AVPU Verbal 3 Unresponsiveness 1

AMPLE
A Do you have any allergies?
M Do you take any medications on regular basis?
P PMH, any history of HTN, heart attack, stroke, DM, any long term disease
L  Last meal
 Last tetanus shot
 LMP
E Event:
 Can you describe to me want happened?
 Car accident! Were you the driver or passenger / front passenger?
 Were you wearing your seat belt?
 Did you hit your head? Did you lose your conscious?
 Do you remember what happened, before and after the accident?

Conclusion:
I am suspecting an intra-abdominal bleeding; we are waiting for (surgeon, orthopedics surgeon)
to intervene
Summary:
Introduction to examiner If you are done  go for secondary survey:
Hello  Expose the patient
Neck collar  Examine him head to toe, looking for fractures,
Introduction to patient more detailed neurological examination
A/B / C / order x-rays / D / AMPLE

OSCE-guide-III.doc Page 79 of 255


Emergency Medicine

NOTES:
FLUIDS:
- Trauma / GIT bleeding: we always start with 2 L bolus
o If the patient is stable for the beginning  do not give anything more
o It the patient was not stable, but becomes stable after the first 2L bolus  give
maintenance fluids
o If patient was not stable, and remains unstable  start bld transfusion: 1 unit of
packed RBCs for every 3 units of fluids, and continue till you find source of
bleeding
Stable 2 L bolus Stable Give nothing
Unstable 2 L bolus Stable Give fluids – for maintenance
Unstable 2 L bolus Unstable Start blood transfusion – 2 RBCs
Then continue 1 (RBCs) : 3 (NS)

- Anaphylactic shock:
o 0.5 L bolus
o Give epinephrine / steroids / anti-histaminics (Benadryl)
- Acute abdomen (pancreatitis / DKA):
o 1-2 L bolus
o Followed by 1 L / hour till the urine output improves
- Heart attack:
o KVO (keep vein open) 100 cc / hour
- If trauma, ↓ BP, ↓ HR with warm extremities  neurogenic shock (spinal cord injury) 
give only 2 L of fluids then give vasopressors

For the exam:


 Tension pneumothorax
 Hemothorax
 Rupture spleen
 Fracture pelvis
 Fracture femur

OSCE-guide-III.doc Page 80 of 255


Emergency Medicine

Unconscious Patient – Neuro


1- Introduction:
- Mr … I am Dr … I am the physician in charge in the emergency room now,
- Mr … Mr … if you hear me open your eyes please. Tap on the shoulder, do you hear me
 I would like to activate the ACLS code please / start primary survey
A  Check the mouth, listen for patent air way  Give 4L O2 via nasal
B  What is the O2 please canula
 Trachea central, chest is moving  Monitor O2 for need
 Listen to lungs, heart to intubation
C I would like to get the vital signs please:  2 large IV lines; for
 Based on vitals, no ↑ in BP and no ↓ in HR (as seen IV fluids and to
in Cushing triad) withdraw samples
 No fever
 No abnormal breathing patterns
D  Mr … I will open your eyes, and shine light in it, pupillary reaction  both
pupils are round, symmetrical and reactive
 Mr … if you hear me can you please move your eyes up and down  NO
locked-in syndrome
 D3: universal antidotes: thiamine 100 mg
2- Glasgow Coma Scale  if < 8  intubate (ask about DNR)
1 2 3 4 5 6
Eyes No Pain Voice Spontaneou N/A N/A
response s
Verbal No sounds Sounds Words Confused Normal N/A
Motor No Extension Flexion Withdrawal Localizes Obeys
movements (decerebrat (decorticat commands
e response) e
response)
3- Cranial nerve exam: excluding the motor and the sensory:
- Inspection:
o Face is symmetrical, no deviation of the angle of the mouth
o Both eyes are symmetrical, no nystagmus
- I would like to do a fundoscopic examination, to look for disc edema, or retinal
hemorrhage
- Reflexes: Pupillary reflex (II, III) / Corneal reflex (V, VII) / Gag reflex: (IX, X)
4- Upper extremities:
- Inspection: no abnormal posture or contractures
- Tone: check the wrist and elbow in both sides  No cog wheel, no lead pipe rigidity, no
clasp knife spasticity
- Reflexes: Biceps / Triceps
- Check the radial pulse
5- Lower extremities:
- Inspection:
- Tone: check by leg rolling  rigidity, elevate the knee rapidly  spasticity
- Reflexes: Knee reflex / Ankle reflex / I would like to do Babinski reflex / Clonus
- Check dorsalis pedis pulse
- If the neck is cleared (by CT), I would like to do nuchal rigidity, Kernig's sign,
Brudzinski's sign
6- To check for brain death: I would like to arrange for: dolls eyes, caloric reflex test

OSCE-guide-III.doc Page 81 of 255


Emergency Medicine

Unconscious Patient – Diabetic

16 years old female found unconscious in her class, next 10 minutes manage and counsel

Introduction:
- Ms … I am Dr … I am the physician in charge in the ER,
- Ms … … if you hear me; can you open your eyes please? Tap on the shoulder, do you
hear me  I would like to activate ACLS code please / start primary survey
A  Check the mouth, listen for patent air way  Give 4L O2 via nasal
B  What is the O2 please canula
 Trachea central, chest is moving  Monitor O2 for need to
 Listen to lungs, heart intubation
C  I would like to get the vital signs please: ↑ BP and ↑ HR
 2 large IV lines; for IV fluids5 and to withdraw samples6
When you ask the nurse for stat glucose by finger prick:
Hypoglycemia Hyperglycemia
 Stat 100 mg thiamine IV  Stat insulin 10 units IV
 Stat 50 ml D50 (Dextrose 50%) IV  Stat 100 mg thiamine IV
 If no IV line  glucagon IM  2 L fluids
At that time, the patient will Orient her; … your blood sugar was low, your class-mates
start to regain her conscious brought you here, you are in the ER in hospital, you are
doing well now, how do you feel right now?
Patient states that she is  Reassure her
worried she will lose her  I can help by giving you a doctor’s note
exam / or other important  This is a very serious condition, you need medical
appointment! attention for some time it is not safe to leave
D D1: Brief neurology  Start D5 (Dextrose 5%): 250 ml / hr
D3: Dextrose  Nurse, I would like to monitor her blood glucose
every 5-10 minutes

+ In case of DKA and physical exam:


History  The same as in hypoglycemia (see below)
 The causes are (5 Is): insulin missed / infection / intoxication / ischemia /
infarction
Physical exam  Brief neuro exam
 Brief DM exam

+ In case of hypoglycemia:
History Are you diabetic?
Analysis Diabetic  When were you diagnosed? And how?
history  Do you take insulin?
 Have you had coma (DKA or hypoglycemia) before?
 When was your last DM follow-up visit? Any reason?
 At that time; were you controlled? Symptoms free?
 When was your last Hb A1C test? What was it?
 How about last few days, were you measuring your glucose?
EVENT  This morning, did you get breakfast, your insulin? Did you check your glucose?
Did you exercise?
 Before you lost conscious, hoe did you feel? Hungry / shaky / dizzy / sweating?

5
If the HR is normal and other VS are normal, you can give only 50 ml/hr to keep vein open (KVO)
But if ↑ HR  give 2 L fluids for follow-up
6
For any female patient: β-HCG with the blood works you will order

OSCE-guide-III.doc Page 82 of 255


Emergency Medicine

Impact  If pt dx < 5 years  no worry about complications


 If pt dx > 5 years screen for complications (urine changes / visual changes /
numbness and tingling in her feet)
Red flags  Constitutional symptoms
DD  How much insulin do you take? Do you make sure you eat after your insulin
dose?
 Any ↑ or ↓ in your weight? Stress?
 Did you start new medication?
PMH HEAD SSS /+/ Pregnancy

Counselling:
- What is your understanding about diabetes mellitus?
Pathophysiology:
- It is a condition related to our blood sugar. Whatever we eat, the food contains different
components, including sugar. The food travels through the food pipe to our stomach, to our
bowels where it is absorbed and goes to all our body. Our organs (brain / muscle) use this sugar as
source of energy. In order for muscles to use this sugar, it needs a key to enter into cells, this key
is the insulin.
- We have two types of DM, type I and type II.
- Patients with DM type I, their body does not produce insulin, so we need to compensate for that
by giving it from external source.
Complications:
- High blood sugar is harmful for our bodies, because it affects all our blood vessels, the small and
big ones, and may give a lot of complications! It might cause kidney, eye, or nerves injury and
harm on the longer term.
- On the other hand, low blood sugar is even more dangerous; do you know why? Because our brain
can not survive without blood sugar for more than 5-7 minutes, it is the only source of energy to
our brains.
Prevention:
- What happened to you is a very serious condition, and it might happen again. The best way to treat
is to prevent this from happening; by:
o Make sure that you always eat after your insulin dose
o Monitor your blood sugar frequently
o If you exercise, adjust your insulin dose based on your blood sugar level
- Now, if this happens again, do you know how to identify it before you totally lose your conscious?
o Whenever you feel hungry / sweating / shaky / dizzy / heart racing
o You need to stop, and immediately eat a candy / chocolate / juice
o So, you need to keep glucose tablets in your bag, to take it in case of emergency
• If you are at home; keep monitoring your blood sugar,
• If you are out; reach to the nearest ER
Emergency measures:
- If you exercise, there is a special type of injections (glucagon emergency kit); if your blood sugar
drops suddenly, use it, or other people can use it to inject you.
- That is why it is important that you have a bracelet that mentions you are diabetic, so if you lose
conscious and some one finds you, they can identify the situation and provide help.
Follow-up:
- You should see your family physician within few days, and he can refer you to “diabetes” clinic,
for more education and assessment.
- I will still give you some brochures and web sites in case you would like to know more.

Notes: If you are the family physician, what referral will you do for a diabetic patient?
- Diabetes clinic / Foot specialist / Dietician
- If DM type I > 5 years, OR type II at any time: Ophthalmologist / Nephrologist / Neurologist

OSCE-guide-III.doc Page 83 of 255


Emergency Medicine

Unconscious Seizing Patient – DT / Epilepsy / Brain tumour / …

Patient arrives to the ER with his wife, on the way he had attack of seizures, and received 1 dose
of diazepam, he is unconscious now. In the next 10 minutes; manage.

Introduction Very brief introduction to wife, I will make sure he is stable then I will ask
you more questions
Mr … ; Patient is unresponsive, I will start my primary survey:
can you hear me  A: can you open your mouth (open and comment) / trachea central / JV
not engorged
STABILIZE  B: listen to lungs and apex / normal air entry on both sides / normal
heart sounds
 C: can I get the vitals please! Normal! 2 large IV lines please
─ One to give IV fluids 50 ml/hr to keep vein open (KVO)
─ The other one is to withdraw samples
 D1; deficits: pupillary reaction
 D3; drugs: universal antidotes  thiamine 100 mg / if O2 is ok, no need
for naloxone, if blood sugar is ok, no need for dextrose
If at any time, the patient starts to seize, give ativan 2 mg IV and reassess ABCD
History Event  First time to seize?
(wife)  Can you describe what happened? Did he fall to the ground?
 Before he seized; did he shout? Starred at the wall? Complained of
strange smell?
 Was all his body seizing or part of it? For how long? Did he bite his
tongue? Rolled up eyes? Did he wet himself? Was breathing?
 Did he regain conscious alone or with intervention?
Cause  History of epilepsy? Medications for epilepsy?
 And mood stabilizers medications?
 RECENTLY, did he complain of: Neurological / Constitutional
symptoms
 Any history of trauma / head injury?
 Recent ear infection?
 SAD: sweating / shaking
 Any medications / blood thinners
PMH  Long term disease; e.g. HTN, DM, kidney, lung, or heart disease
 Previous hospitalization / surgeries?
FH  FH of epilepsy
Examination  Vitals from the examiner
 Glasgow coma scale (if < 8  arrange for intubation)
Neurological examination:
 Cranial nerve examination
 UL and LL: tone and reflexes
Management  Stat neurology consult
 Stat CT brain

OSCE-guide-III.doc Page 84 of 255


Emergency Medicine

N.B. Causes of seizing:


- Epilepsy relapse
- Brain tumour / brain hemorrhage
- Infection / meningitis
- Organ failure
- Electrolytes imbalance
- Delirium Tremens
- Withdrawal from sleeping pills
- Cocaine overdose

How to identify if this is narcotics overdose?


- While doing AB; if O2 saturation is low even after you give O2, and respiratory rate is
low; jump to shine light in the pupils  if pin-point-pupil  give stat naloxone 0.2 – 2.0
mg IV
- Then back to ABCD

Any unconscious patient:


- A; if fluids  suction
- B; if not breathing  mask and bag
- C
- D
- Then assess Glasgow coma scale, if < 8  intubate (ask for DNR here)

Common 5 causes for unconscious patients:


- Hypoglycemia / DKA
- Epilepsy
- Meningitis
- Stroke lateralization
- Heart block

OSCE-guide-III.doc Page 85 of 255


Emergency Medicine

Heart Attack – Chest Pain (MI or Heart Block)


4 cases:
- Chest pain for 45 minutes, with normal ECG
- Chest pain for 45 minutes, with ST elevation
- V. fib: chest pain and the patient is intoxicated
- V. fid: chest pain and the patient is calm

Introduction … How are you feeling right now?


Brief history  Os Cf D
 PQRST
 ↑...↓ – position or deep breathing
ABCD  Patient is talking to me  air ways ok.
 Listen to chest to exclude pneumothorax  breathing is ok
 C and D briefly
MONA Oxygen  4 L O2 via nasal canula
Aspirin  Aspirin 325 mg
 Plavix 300 mg
Nitrates  0.3 mg puff X 3 times; 5 minutes apart
- If no low BP
- If the pt did not consume Viagra recently
Morphine  If patient is in pain after 3 puffs of nitrates and there is NO
hypotension or bradycardia  morphine 2 mg
- If antero-lateral infarction (V2/3/4/6 & aVL): 5 mg morphine
- If (II, III, aVF)  usually involves the right ventricle, and the
morphine increase the pre-load  2 mg morphine only
± β-blockers Given to all patients; EXCEPT:
 History of asthma
 ↓ BP, ↓ HR, heart block, and inferior MI (II, III, aVF)
 Cocaine (unopposed alpha  ↑ blood pressure)  labetalol 20 mg
(2 mg/min for 10 min)
You give metoprolol 2 mg IV
No ST elevation - Stat cardiology consult
- Catheterization lab  angiography
 Heparin (ask about the hospital protocol for heparin – usually 5000
U IV then 1000 U/hr by infusion) or give enoxaparin 1 mg/ kg)
ST elevation - Stat cardiology consult
- Catheterization lab  angioplasty
 Heparin (enoxaparin 1 mg/ kg)
 If the cardiologist is not available  start thrombolytics, but first
rule out the contra-indications for thrombolytics:
- 2 recent (< 2 weeks): Recent surgery / Recent bleeding
- 2 brain: Brain tumour / Bleeding in brain ± stroke
- 1 heart: Aortic dissection

 Chest pain presents with heart racing / SOB / nausea / vomiting / sweating
 History will be: chest pain analysis / cardiac symptoms / risk factors
 If blood pressure is low: we only give oxygen / aspirin / and plavix
 If inferior MI (II, III, aVF)  I need 15 lead ECG / do not give β-blockers
 Risk of bleeding with thrombolytics is 1%, but being serious, this needs consent

OSCE-guide-III.doc Page 86 of 255


Emergency Medicine

Case 1: Chest pain with initial normal ECG

Chest pain for 45 minutes, manage

Introduction … How are you feeling right now?


Pain history  Os Cf D
 PQRST (R: how about your jaws / back / arms)
 ↑...↓ – position or deep breathing
ABCD  Start O2 4 L by nasal canula
 … vitals please: comment and ask for regular VS check
 2 IV lines: KVO / take samples and serial ECG
- ECG normal: order Serial ECG every 5 minutes / if any ↓ VS / any
change in pain
- How did you come here today?
- Did you take any aspirin? aspirin 325 mg / plavix 300 mg
- Have you recently used Viagra? If no  nitroglycerine puffs 0.3 mg X
3; 5 minutes apart
 D1: brief neuro exam  pt is grossly neurological free
 Any results back?
 Order chest x-ray?
Brief history  Cardiac symptoms: heart racing / SOB / nausea / vomiting / sweating /
dizziness / LOC
 Chest symptoms
 Constitutional symptoms
 DD: GIT (difficulty swallowing / acidic taste / GERD / heart burn / hx of
PUD) / Trauma / relation to respiration / calf pain / swelling
 PMH: medications / blood pressure / blood glucose / cholesterol / SAD
(especially for smoking and cocaine) / stress
 FH: heart diseases / heart attacks

Another ECG

Normal ECG Do physical exam:


 Vitals / compare BP in both arms – ? aortic dissection
 General status of the patient
 Eyes / mouth
 Heart and chest examination
 LL edema

OSCE-guide-III.doc Page 87 of 255


Emergency Medicine

ST elevation  Morphine 2 mg (up to 5 mg in antero-lateral MI)


 β-blockers: metoprolol 2 mg IV (if NO contraindications: history of
asthma, ↓ BP, ↓ HR, heart block, inferior MI, cocaine)
Mr …, based on your symptoms and your ECG, it seems that you
have a heart attack. (Am I dying Dr?). This is a serious condition,
however you are stable and doing well, you are in a safe place and
we have experience in dealing with that. Heart attack means one or
more arteries providing blood supply to your heart are blocked by a
clot, we need to reopen this clot.
 Stat cardiology consult
 Do we have catheterization lab?

What we need to do now is to use medication called “thrombolytic”


or “clot buster”. I would like first to make sure that you are a good
candidate; recently have you had any surgery or bleeding? Have you
ever had stroke before? Were you diagnosed or told that you have
brain mass? Lesion? Based on the ECG, I do not see signs consistent
with cardiac aneurysm, and based on the BP, it does not look like
having dissection. Based on this, I can say that you are a good
candidate for thrombolytics, it is a life saving procedure, however
like any other medical intervention, it has side effects; it might cause
bleeding in 1% of cases, this includes stroke, so we need your
consent if you would like to have it!
 Heparin (ask about the hospital protocol for heparin – usually 5000 U IV
then 1000 U/hr by infusion) or give enoxaparin 1 mg/ kg)
 Can you get the thrombolytics kit please

ST elevation:
 Lateral MI
 Inferior MI

The patient wants to leave the hospital:


- I would like to make sure he is competent, not under influence of alcohol or any
substance, and to rule out suicidal ideation
- I would explain to the patient: diagnosis / treatment / side effects of treatment /
complications of not receiving treatment / alternatives
- I will document this, and I will ask the patient to sign a LAMA (leaving against medical
advice), and I will let him go

OSCE-guide-III.doc Page 88 of 255


Emergency Medicine

Case 2: Chest pain with STEMI


Introduction … How are you feeling right now?
Pain history  Os Cf D
 PQRST (R: how about your jaws / back / arms)
 ↑...↓ – position or deep breathing
ABCD  Start O2 4 L by nasal canula
 … vitals please: comment and ask for regular VS check
 2 IV lines: KVO / take samples and serial ECG
- ECG: ST elevation in V 2/3/4, aVL  ant-lat MI
- How did you come here today?
- Did you take any aspirin? aspirin 325 mg / plavix 300 mg
- Have you recently used Viagra? If no  nitroglycerine puffs 0.3 mg X
3; 5 minutes apart
 D1: brief neuro exam  pt is grossly neurological free
 Any results back?
 Order chest x-ray?
Brief history  Cardiac symptoms: heart racing / SOB / nausea / vomiting / sweating /
dizziness / LOC
 Chest symptoms
 Constitutional symptoms
 RISK factors
 PMH: medications / blood pressure / blood glucose / cholesterol / SAD
(especially for cocaine) / stress
 FH: heart diseases / heart attacks
ST elevation  Morphine 2 mg (up to 5 mg in antero-lateral MI)
 β-blockers: metoprolol 2 mg IV (if NO contraindications: history of
asthma, ↓ BP, ↓ HR, heart block, inferior MI, cocaine overdose)
Mr …, based on your symptoms and your ECG, it seems that you
have a heart attack. (Am I dying Dr?). This is a serious condition,
however you are stable and doing well, you are in a safe place and
we have experience in dealing with that. Heart attack means one or
more arteries providing blood supply to your heart are blocked by a
clot, we need to reopen this clot.
 Stat cardiology consult
 Do we have catheterization lab?
What we need to do now is to use medication called “thrombolytic”
or “clot buster”. I would like first to make sure that you are a good
candidate; recently have you had any surgery or bleeding? Have you
ever had stroke before? Were you diagnosed or told that you have
brain mass? Lesion? Based on the ECG, I do not see signs consistent
with cardiac aneurysm, and based on the BP, it does not look like
having dissection. Based on this, I can say that you are a good
candidate for thrombolytics, it is a life saving procedure, however
like any other medical intervention, it has side effects; it might cause
bleeding in 1% of cases, this includes stroke, so we need your
consent if you would like to have it!
 Heparin (hospital protocol or give enoxaparin 1 mg/ kg)
 Can you get the thrombolytics kit please
Have time Physical exam: listen to lung bases / S1 and S2 / compare BP both arms

OSCE-guide-III.doc Page 89 of 255


Emergency Medicine

Case 3: Chest pain – v fibrillation / v tachy

- I would like to initiate code blue – ACLS


- Is this is the patient ECG? Is it the last one? We need to start defibrillation!
o Done.
o Can you give me the last ECG please?  NORMAL ECG
- How do you feel? Do you feel drowsy? Because you have just had cardiac arrest, we
would like to make sure you are stable.

- Manage as the first case  the chest pain with normal ECG

Case 4: Chest pain – v fibrillation – intoxicated patient

- I would like to initiate code blue – ACLS


- Is this is the patient ECG? Is it the last one? We need to start defibrillation!
o Done.
o Can you give me the last ECG please?  NORMAL ECG
- How do you feel? Do you feel drowsy? Because you have just had cardiac arrest, we
would like to make sure you are stable.

- The patient will be rude / angry / aggressive. ? intoxicated


o If the nurse gave you “cocaine” and said it was found with the patient; ask her to
keep it with the hospital security
o Mr … you have just had a cardiac arrest, this is a very serious condition, and
might happen again, you should not leave, it is important to stay; we want to
make sure you are stable.
o If agitated  1 mg lorazepam injection

Cocaine (arrhythmias / HTN / neurological manifestations):


- What were you doing in the party? Any alcohol? Drugs?
- We will give you a medication that interacts with cocaine, it is very important to tell us if
you took any cocaine recently! (if cocaine  Ca channel blockers)
- Any weakness / numbness / tingling / vision problems / hearing problems?
- Do you hear voices / see things? Do you think someone want to hurt you?

- Manage as the first case  the chest pain with normal ECG

OSCE-guide-III.doc Page 90 of 255


Emergency Medicine

Heart Block

2 cases:
- One of them is DNR (must be dated, valid, and signed)
- The other case is: do not intubate / do not defibrillate. You can still pace maker

1- Introduction:
- Is this is the last ECG for this patient? I do not see any signs of V. fib or V. tachy. I
would like to see the patient first to make sure he is stable, and then I will look at the
ECG.
- Mr … I am Dr … working in the ER, do you hear me?
- I would like to activate the ACLS code please / start primary survey
A  Check the mouth, listen for patent air way  Give 4L O2 via
B  What is the O2 please nasal canula
 Trachea central, chest is moving  Monitor O2 for
 Listen to lungs, heart need to intubation
C  I would like to get the vital signs please;
 2 large IV lines; for IV fluids and to withdraw samples

2- Glasgow Coma Scale  [if < 8  intubate (ask about DNR)]


- This patient Glasgow coma scale is …
- What is this patient code status? Any advanced directives?
o Is it signed, dated and valid?
o What does he have?
- We will respect his wishes, we will not … (if DNR  do nothing)
- If no DNR:
D2 Third degree heart block:
 1 mg atropine  any changes?
 Pace maker:
- Rate: 20 more than his base heart rate
- Leads on sternum and apex
D1 Brief neurological scan, pupils, …
Collateral  Do we have his file?
history  Can we contact his family physician? Or family member?
 Does he have a med alert?
 Do we have paramedics report?
 Can we check his belongings? He is taking …, thiazides and digoxin
+
 I need to get his K level / digoxin level and I would like to check the dose for
digi-bind and digi-fib
Physical  Neuro exam
exam  Cardiac exam

Notes:
- For any unconscious patient: ask about advanced directives or DNR! What is this patient
code status?
- Whenever the examiner or the nurse tries to give you an ECG at the room entrance,
assess for V. fib or V. tachy and report: there are no signs of V. fib or V. tachy. I would
like to see the patient first to make sure he is stable.

OSCE-guide-III.doc Page 91 of 255


Emergency Medicine

Headache

Introduction
CC Headache for 2 hours (very acute – very serious)
Analysis of CC Os Cf D
 Is this your first time
 Did you get any trauma?
 Would you describe it as the worst headache in your life? Thunder
clap?
- Can you please lie down?  Put the bed 45°, I would like to make
sure u r stable!
- ABCD: IV lines / D1: Pupils
 PQRST
 ↑...↓ – position or coughing
Associated  Acute neuro: fever / neck pain / stiffness / vision / hearing / gait / falls
symptoms / weakness / numbness
 The patient says: I am diabetic  stat blood glucose (prick)
PMH HTN / blood thinners / kidney diseases
FH Kidney cysts / disease / aneurysm
SH Cocaine

If the patient loses his conscious  manage as unconscious patient


ABCD  If ↑ BP and ↓ HR  I am suspecting Cushing triad, put the bed in 45°
 D1: Pupils, D3: thiamine 100 mg
Secondary  Glasgow coma scale As in the case of the
survey unconscious patient

Physical exam  Orientation (if conscious) ? subarachnoid


 Quick cranial nerves hemorrhage
 Quick neuro (tone/ motor / sensory / reflexes)
 Nuchal rigidity, Kernig's, Brudzinski's signs ? meningitis
(FEVER)
Management  CT scan stat ± LP
 Others: Chest x-ray / Urinalysis
 Blood works (CBC / differential / lytes) Treatment of
 Septic workup (samples / C&S) meningitis:

 Stat neurosurgery consult vancomycin 1 g IV


 Empiric antibiotics q12h + ceftriaxone
 IV mannitol 20% – 1 gm / kg 2 g IV q12h
 Intubate:
- To keep air ways patent ± ampicillin 2 g IV
- To ↓ intra-cranial pressure q4h (if >50 years or
hx of alcohol use or
immunocompromise
)

OSCE-guide-III.doc Page 92 of 255


Emergency Medicine

Acute Abdominal Pain

OsCfD /+/ PQRST  will give you an idea


In abdominal pain cases:
 If you suspect certain diagnosis  scan for the risk factors for that disease
 If no diagnosis  continue GIT symptoms and general differential diagnosis

You suspect obstruction  nausea / vomiting (COCA+B / coffee ground material) AND bowel
movements  if vomiting  screen for dehydration
 If you dx obstruction  check risk factors of obst  then rest of GI symptoms
 If not obst  scan GIT  near-by systems  PMH for systemic disease
If you suspect kidney stones  screen with renal symptoms
 If you dx renal stone  check risk factors (diet, medications, hx of renal stones, uric
acid, bone pains / fractures)  then rest of urinary symptoms

Intestinal obstruction

Intro … But first I would like to ask you, how do you feel now?
Analysis of Analysis: OsCfD: gradual, started colicky, and now continuous dull pain /

CC PQRST / What ↑ or ↓ (position / eating / bowel movements / vomiting)
 Screen for obstruction:
─ Nausea/ vomiting
o Relation to pain, which started first, does it relief pain
o COCA + Blood (coffee ground material)
Impact  Screen dehydration (dizziness / light headedness / thirsty / LOC)
 Bowel movements
─ How about any blood? Any time?
─ Still passing gas?
Red flags Risk factors for intestinal obstruction:
 Previous surgery? What? When?
 Fever/ night sweats/ chills / appetite / loss of weight / lumps & bumps
 PMH or FH of cancer or benign tumour
 Hx of Crohn’s disease (hx of abd pain/ bloody diarrhea) / family hx
 Hx of hernia / groin mass
 Gall bladder stones / right upper quadrant pain
Differential  Gastroenteritis:
diagnosis ─ What did you eat yesterday? Place that you are not used to?
─ Diarrhea / blood in stools?
─ Anybody else ate with you and suffered from the same problem
 Renal: flank pain / burning sensation / going more to washroom / stone
 Liver: yellowish discoloration / itching / dark urine/ pale stools
 Hx of HTN / SOB / cough / phlegm (aortic dissection)
PMH / FH / SH

 X-ray findings of small intestinal obstruction: (1) Multiple air/fluid levels, (2) Dilated loops
of small intestine, (3) No air under the diaphragm.
 Management: (1) NPO / NG tube, (2) Oxygen mask, (3) IV fluids, (4) Stat surgical consult,
(5) Foley’s catheter, (6) Correct electrolytes.

OSCE-guide-III.doc Page 93 of 255


Emergency Medicine

Right / Left lower quadrant abdominal pain – middle aged female


 History: nausea / vomiting / fever / LMP / vag discharge / risky behaviour
 Physical exam: MUST mention pelvic and bimanual exam
 DD: (Left side): ectopic / PID / ovarian cyst / torsion / kidney stone
 DD: (Right side): ectopic / PID / ovarian cyst / torsion / kidney stone / appendicitis / intest
obstruction
 Investigations: β-HCG / CBC / differential / abd-pelvic US / culture of the vag and cervical
secretions
 Long term complications: abscess / ectopic / infertility / intestinal obstruction / peritonitis
 Signs at PV exam: left adnexal mass / cervical motion tenderness

Acute abdomen in a female  missed period (ectopic), bleeding (abortion), discharge (PID)

PID
 Diagnostic plan:
─ Pregnancy test – β-HCG
─ CBC / ESR
─ Cervical culture (for Gonorrhea and Chlamydia)
─ Syphilis serology
 What is the treatment of pelvic inflammatory disease?
─ Cefoxitin 2 g IV every 6 hours X 2 days (covers anaerobic bacteria)
─ Doxycycline 100 mg orally BID X 2 weeks
─ Remove any IUD (if present)
 What are the indications of hospitalizing the patient?
(1) Pregnancy, (2) Pelvic abscess on U/S scanning / high fever (> 38.5 °C), (3) PID at young
age, (4) Recurrent PIDs, (5) Failure to respond to outpatient management, (6)
Immunodeficiency (patients with HIV infection) or severe illness
 Complications of PID: abscess / ectopic / infertility / intestinal obstruction / peritonitis

Left lower quadrant abdominal pain – elderly  Diverticulitis


 Signs:
─ Fever
─ Peritoneal signs: +ve cough tenderness / percussion tenderness / tenderness / rebound
tenderness / DRE: severe pain
 DD: diverticulitis / abscess / cancer
 Investigations:
─ AXR, upright CXR:
o May be normal
o Localized diverticulitis (ileus, thickened wall, SBO, partial colonic obstruction)
o Free air may be seen in 30% with perforation and generalized peritonitis
─ CT scan (optimal method of investigation). 97% sensitive, very useful for assessment of
severity and prognosis. Very helpful in localizing an abscess
─ Hypaque (water soluble) enema – SAFE (under low pressure):
o Saw-tooth pattern (colonic spasm)
o May show site of perforation, abscess cavities or sinus tracts, fistulas
─ Barium enema: contraindicated during an acute attack: risk of chemical peritonitis
(perforation)
─ Sigmoidoscopy/colonoscopy:
o Not during an acute attack, only done on an elective basis
o Take biopsies to rule out other diagnoses (polyps, malignancy)

OSCE-guide-III.doc Page 94 of 255


Emergency Medicine

 Management: (1) NPO / NG tube, (2) Oxygen mask, (3) IV fluids, (4) Stat surgical consult
(5) IV antibiotics (IV ciprofloxacin 500 mg BID / IV Metronidazole 500 mg TID)
 Indications for surgery for diverticulitis:
─ Unstable patient with peritonitis
─ Hinchey stage 2-4 (large abscess / fistula / ruptured abscess / peritonitis)
─ After 1 attack if: (a) immuno-suppressed, (b) abscess needing percutaneous drainage
─ Consider after 2 or more attacks, recent trend is toward conservative management of
recurrent mild/moderate attacks

Diabetic ketoacidosis  acute abdominal pain

Analysis  Pain: OCD / PQRST / ↑↓


 Vomiting: number of episodes + COCA
 AS: fever / malaise / cough / urinary symptoms / diarrhea /
gastroenteritis
 Diabetic history
Impact  Dehydration: feel weak / pee less / dry tongue and skin
Red flags  Hyperglycemia symptoms: thirst, polyuria, polydipsia, and nocturia
DKA  Acidosis: shallow rapid breathing or air hunger (Kussmaul or sighing
respiration), abdominal tenderness, and disturbance of consciousness
 Complications; of diabetes mellitus
DD – causes of  Non-compliance OR wrong dose
DKA  Recent surgery
 Pregnancy
 Trauma
 MI
 Infection

Management of  IV fluids – NS (1 L/hr x 2 hrs then 500 ml/hr x 2 hrs then 250 ml/hr
DKA x 4 hrs)
 Foley’s catheter
 Insulin drip 2 units / hour – check glucose and lytes every 2 hours
 When glucose reaches down to 15  fluids will continue as
maintenance, 2/3 : 1/3 of D5W : NS + 20 mEq KCl/L. 4:2:1 rule: 4
ml/kg/hr for the first 10 kg, then 2 ml/kg/hr for the next 10, then 1
ml/kg/hr for the next whatever
 Serial blood glucose
 ABG / serum ketones
 CBC / lytes
 Septic workup (chest x-ray / blood cultures / urinalysis)
 ECG (for the ↑ in K+)

OSCE-guide-III.doc Page 95 of 255


Emergency Medicine

Acute Abdomen

Patient came to ER with abdominal pain / vomiting / diarrhea / BP 90/60 / HR 120


Acute pancreatitis / dissecting aortic aneurysm (no vomiting) / perforated peptic ulcer

Introduction  I can see that you have a lot of pain, bear with me for few minutes and I will
give you a pain killer as soon as I can.
 In the moment, I would like to make sure you are stable
 What are the vitals pleas?
Stable Unstable
Proceed to I am going to start my primary survey  ABCD
history  When you send blood works: add lipase / amylase
↓ Did you vomit blood? How about coffee ground? (if yes: order
blood)
Analysis Os Cf D / PQRST / ↑↓ / relation to position / breathing / eating
 Vomiting  COCA + Blood
 Change in the bowel movements
Impact  Dehydration
 How do you feel right now? What are the vitals please?
Red flags  Constitutional symptoms
DD Liver / GB  Yellowish discoloration / itching / dark urine / pale stools?
 Recent flu-like illness?
 Do you have hx of gall bladder stones? Repeated attacks abd
pain?
Stomach  Hx of PUD? GERD? Acidic taste / heart burn?
 Alcohol? How much? When was the last time? Did u drink >
usual?
 Gastroenteritis (What did you eat yesterday? Place that you
are not used to? Diarrhea / blood in stools? Anybody else ate
with you and suffered from the same problem?)
Medications  If vomited blood: Do you take steroids / NSAIDs / blood
thinners?
Kidney  Flank pain? Burning sensation? Dark urine? Frequency?
Aorta  Hx of HTN / atherosclerosis / DM / ↑ cholesterol / smoking /
SOB
Trauma  Did you have trauma?
PMH  Medications / allergies / long term disease?

Physical exam: abdominal exam


Face Eyes for jaundice / mouth for dehydration / ulcers
7 8
Inspection  No Cullen’s sign / no Grey Turner sign
 Cough tenderness

7
Cullen’s sign: peri-umbilical ecchymosis. It arises from spread of retroperitoneal blood associated with: pancreatitis / ruptured
ectopic preg / ruptured aortic aneurysm / ruptured spleen / perforated duodenal ulcer
8
Grey-Turner sign: ecchymoses of the skin of the flanks, also with retroperitoneal bleeding

OSCE-guide-III.doc Page 96 of 255


Emergency Medicine

Auscultation  Patient in severe pain:


Percussion ─ I can see that you are in pain, can you bear with me for few minutes
Palpation then I will give you pain killer as soon as I can.
─ In order for me to examine you properly, I need to get good look at your
abdomen, for that reason, can you please lie on your back!
─ Do you feel relieved like that, I understand. It will be only few minutes,
do you want to give it a try? Slowly! Do you want me to help you! It is
crucial to reach a proper diagnosis!
─ If still refusing  offer 2 mg morphine S.C. Finally she will lie down.
 ASK FOR X-RAY (3 view x-ray abdomen)  ? perforation
─ Perforation  severe guarding, will not be able to proceed
─ Obstruction

Patient is obviously in severe pain, I will not be able proceed with examination

Management I am suspecting acute peritonitis I am suspecting pancreatitis


(? perforated peptic ulcer)
 NPO / nasogastric tube  NPO / nasogastric tube
 O2 mask  O2 mask
 IV fluids (250 ml/hr) / Cross  ICU admission
match blood

 Pain killer: Meperidine 1 mg / kg  Pain killer: Meperidine 1 mg / kg


or Fentanyl or Fentanyl
 Stat SURGICAL consult  Stat GIT consult

 3 view abdominal x-ray  3 view abdominal x-ray


 Abd U/S  Abd U/S for gall stones
 CT abdomen  CT abdomen

 Lab: CBC / blood sugar / calcium  Lab: CBC / blood sugar / calcium /
/ amylase / lipase amylase / lipase
 Albumin level / serum Ca

DD:
 Perforated PUD: vomiting coffee ground material
 Aortic dissection: NO vomiting / severe pain shooting to the back
 Acute pancreatitis:
─ NO upper GIT bleeding
─ Fever (due to chemical irritation not infection)
─ Pain improves when leaning forward
─ Paralytic ileus
─ Tetany

Ethical question:
 The patient girl friend is on the phone, she is asking about his condition?!
─ I am still doing my examination,
─ I can assure you that he is well taken care of, and we will do our best to help him,
─ All the details of his medical information is absolutely confidential, and I can not release

OSCE-guide-III.doc Page 97 of 255


Emergency Medicine

Upper GIT bleeding

Patient vomited blood; esophageal: varices / gastric: perforated PUD

Introduction  Because this is a case of GI bleeding, … protection …


 Mr …, … make sure that you are stable, then I will ask you …
 How do you feel right now?
ABCD Be aggressive with fluids – 2 L IV
Analysis  Os Cf D / COCA + dehydration
 Associated with pain (painless: esophagus / pain: stomach)  PQRST
 Nausea / vomiting / diarrhea
 Recently; any blood in stools / dark stools?
Impact  Weak / drowsy / …
Red flags  Constitutional symptoms – cancer stomach
DD Liver  Yellow / itchy / urine / stool
 Alcohol; how much / for how long / more than usual? SAD?
 Long term liver disease? ↑ Abdomen size? Bruises?
Memory? Concentration? Numbness LL?
 Risk factors for hepatitis: piercings, bld transfusion,
unprotected sex
Stomach  History of PUD / pain / GERD, heart burn
 Risk factors for ca stomach: smoking
Medications  Do you take any blood thinners? Aspirin? Steroids? NSAIDs?
PMH Any medications / allergies
FH Family history of gastric cancer
SH

Vitals ↑ or ↓ fluids (if stable: fluids 250 ml/hr) and monitor vitals
General General appearance of the patient: cachectic / distressed / …

I would like to check if there is any postural drop in the blood pressure

Exam Liver exam: extra-hepatic signs of liver cell failure /+/ Bruises

Abdominal exam: epigastric mass / pain / liver / ascites

If painful: manage as acute abdomen case (perforated PUD)

Management STAT GIT consult for UPPER GIT endoscopy

IV pantoprazole (80 mg bolus then 8 mg/hr)

IV octereotide (25 mcg/hr)  portal circ VD  ↓ portal pressure

Abdominal x-ray

Admission to ICU

Longer term management:
 If portal HTN: non-selective β-blockers
 Advice on cutting down the alcohol
 Advice to follow-up with the family doctor

OSCE-guide-III.doc Page 98 of 255


Emergency Medicine

Lower GIT bleeding


Introduction  Because this is a case of GI bleeding, … protection …
 Mr …, … make sure that you are stable, then I will ask you …
 How do you feel right now?
ABCD Be aggressive with fluids – 2 L IV
 D1: you might find some deficits  previous cerebro-vascular accident (may
be history of weakness and numbness)
Analysis  Os Cf D / COCA + dehydration
 Associated with abdominal pain  PQRST / ↑ – ↓ / related to meals
 Vomiting blood? Nausea / vomiting?
Impact  Weak / drowsy / …
Red flags  Constitutional symptoms – cancer colon
DD Liver  Liver: yellow / itchy / urine / stool
Colon  Hx of dx / screening / for Ca colon? When? What was the
result?
 Recent change in bowel movement?
 Recent changes in calibre of stools?
 Risk factors for Ca colon: Fm hx of ca colon / diet (↓ fibres ↑
fat)?
 Risk factors ischemic colitis? Atherosclerosis (similar to
CAD)?
Medications  Do you take any blood thinners? Aspirin? Steroids? NSAIDs?
PMH Any medications / allergies
FH Family history of colon cancer
SH SAD
Vitals ↑ or ↓ fluids (if stable: fluids 250 ml/hr)
General  General appearance of the patient: cachectic / distressed / …
 I would like to check if there is any postural drop in the blood pressure
Exam  Abdominal exam:
─ If soft abdomen, and no findings (PE disproportionate to the pain) 
ischemic colitis
Management  STAT SURGERY consult
 NPO / IV fluids / Foley’s catheter
 Admission to ICU
 Abdominal x-ray
 CT angiography

DD With pain Painless


─ Cancer colon: chronic intermittent ─ Diverticulosis
─ Ischemic colitis: acute / sudden ─ Angiodysplasia
─ Upper GIT bleeding
─ Aspirin
─ Hemorrhoids

Colitis: Radiation / Infectious / Ischemic / IBD (UC > CD)


Abdominal x-ray findings:
─ Intestinal ischemia: thumb print sign
─ Ischemic colitis: pneumatosis intestinalis (coli)
N.B. ischemic bowel  metabolic acidosis

OSCE-guide-III.doc Page 99 of 255


Emergency Medicine

ECG
 Normal
 V fib /+/ V tachy /+/ Torsades du pointes
 A fib /+/ Atrial flutter
 ST elevation:
o Pericarditis: all leads
o MI:
 V 2/3/4 ± V5/6, aVL: antero-lateral MI (left coronary)
 II, III, aVF: inferior MI (right coronary, posterior and inferior surfaces)
 Hear block – third degree /+/ Bundle branch block
 Hyperkalemia /+/ Hypokalemia /+/ Hypercalcemia
 Digitalis toxicity

OSCE-guide-III.doc Page 100 of 255


Emergency Medicine

1. Rate:
 Regular: 300/number of big squares (R-R)
 Irregular: Number of “R”s x 6

2. Rhythm  check for P wave in lead II


Tachyarrhythmias:
 Sinus tachycardia
 Irregular irregularities: A. Fibrillation
 Saw teeth (regular irregularities): A. Flutter
 Rapid SVT
 Ventricular arrhythmias: Premature ventricular beats / V. Tachy / V. Fib / Torsades de
pointes
Bradyarrhythmias: (< 60/min)
 Sinus bradycardia
 Heart block:
- 1st degree: P-R intervals increasing, but every P  QRS.
- 2nd degree:
 Mobitz type I: P-R intervals increasing, with missing QRS
 Mobitz type II: P-R intervals constant, with missing QRS
- 3rd degree: P-P has a rate, and the R-R has another rate

3. Axis
Normally, QRS in leads I, II, III are positive (upwards ↑).
 Right axis deviation: QRS in I is negative (downwards ↓); I and III facing.
 Left axis deviation: QRS in II, III is negative (downwards ↓); I and III opponents.
Diagram showing how the polarity of the QRS complex in leads I, II, and III can be used to
estimate the heart's electrical axis in the frontal plane:

Lead I negative and aVF positive: Rt axis deviation / Lead I positive and aVF negative: Lt axis
deviation.

4. Bundle Branch Block:


 Normally QRS in V1 is downwards ↓, if in V1: QRS is upwards ↑ & wide: RBBB.
 Wide QRS in V6 (M mountain): LBBB.
A mnemonic to remember ECG changes is WiLLiaM MaRRoW, i.e. with LBBB there is W in
V1 and M in V6 and with a RBBB there is M in V1 and W in V6

OSCE-guide-III.doc Page 101 of 255


Emergency Medicine

5. ST segment:
Angina STEMI AND Non-STEMI

6. Others:
Hypokalemia Hyperkalemia
ST segment depression, inverted T waves, 1- Flat P wave
large U waves, and a slightly prolonged PR 2- Wide QRS
interval. 3- Spiked T wave

OSCE-guide-III.doc Page 102 of 255


Emergency Medicine

Phone calls

THE SEIZING CHILD – PHONE CALL

The mother is on the phone, panicked as her child is seizing for 3 minutes

Introduction - Good evening, I am Dr … the physician in charge at the (clinic/ER), may


+ Reassurance I know who is on the line?
- Before I proceed, I would like to know the name of your child, your
phone number and your address please! In case the line is disconnected,
hang on, and do not call me, I will call you back.
- What is your child weight?
- I know this is stressful time for you; however, I need your help here. I
will ask you some questions and give you some instructions.
Stabilize - Is this his first time?
- Did you call the ambulance? I will ask the nurse to call for you
- Is your child alert/conscious? Is he breathing? What is his color?
- Can you support and monitor him, remove any object near him?
- Can you put him on his side?  analyze during
- Do not put anything in his mouth, do not fix him
- Monitor till he stops seizing  assess for focal symptoms  after
Analyze the event - Can you describe the seizure for me please!
- Is it the first time? Or did it happen before?
During - Did you watch that?
- Did he lose conscious? Which happened first, seizing or LOC?
- Did he fall from height? Did he hit his head?
- Was he shaking? Certain part of the body or whole?
- Is he breathing? Did he turn blue?
- Any tongue biting? Did he wet himself? Roll up his eyes?
After - How long did it last? How did it stop? Spontaneously?
- Did he regain consciousness? After he regains consciousness; is he
able to recognize you? Able to talk? To move? Confused?
- Does he feel any weakness, numbness?
Fever - Does he have fever? When did it start? Any medical advice? Any
diagnosis? Treatment? Is he taking the medications? Any reason?
- Constitutional symptoms
Rule out Meningitis / pneumonia
BINDE
PMH Neurological deficits / diabetes mellitus
FH Febrile seizure / epilepsy
Counselling The seizing child counselling

Notes:
 Febrile seizure vs. meningitis: 1st time send the ambulance, 2nd time: send the ambulance if:
the seizure is > 15 minutes or > 2 attacks in 24 hours

OSCE-guide-III.doc Page 103 of 255


Emergency Medicine

THE CHILD SWALLOWED MEDICATIONS / CAUSTIC MATERIAL

The mother is on phone, panicked as her child swallowed medication / caustic material at home

Introduction - Good evening, I am Dr … the physician in charge at the (clinic/ER), may


+ Reassurance I know who is on the line?
- Before I proceed, I would like to know the name of your child, your
phone number and your address please! In case the line is disconnected,
hang on, and do not call me, I will call you back.
- What is your child weight?
- I would like to ask you to take a deep breathe to calm down; I will need
your help here. I will ask you some questions and give you some
instructions, this is important for the sake of your child, ok.
Primary survey - Is your child alert?
+ Stabilize Yes NO
- Good to hear that - Do you know how to do CPR?
- Yes, start it now
- No, guide her:
- If < 8 yrs 5 / 1
- If > 8 yrs 15 / 2
ABC - Is he breathing? Talking to you?
- Can you check the child lips color?
- Does he recognize you?
- Can you check his mouth, and if there is any of the medications can
you take it out?
- If there are any remnants of the chemical on his mouth or face, can
you wash it with some water?
- Did you call the ambulance? I will ask the nurse to call the ambulance
and we will send it to your home now
- You need not to give him/her any thing by mouth
- You do not need to do anything; you can just hold him/her?
Analyze the event - Can you tell me what happened?
- Which medication did he take? If does not know; who is taking this
medication? Why for? Was it recently renewed? Is there vitamins?
Sleeping pills? Aspirin? When ambulance arrives, please give them any
bottle that the child might have taken medications from?
- How much did he take?
- For how long was he/she left alone? Any reason?
Is it the first time? - If yes, analyze the event
Or did this happen - Are there other children at home? Any history of repeated illnesses at
before? home? Repeated visits to ER?
BINDE - B: Was he a full term? Did he need special attention? Any congenital
anomalies?
- I: Are his/her shots up-to-date?
- N: Is he a fussy baby?
- D: Is he a difficult baby?
PMH

OSCE-guide-III.doc Page 104 of 255


Emergency Medicine

EMERGENCY TRAUMA – PHONE CALL

A nurse is calling you from a remote rural medical center; she has a patient of trauma after a car
accident, BP 90/60 and HR 120. Manage over the pho ne.

A/B: Mouth / O2 / inspect chest, neck / listen to lungs, heart


- I am Dr …, the physician in charge in the emergency room; may I know who is on the
line?
- Can you describe the case / situation for me!
- Now, I would like to get your phone number (and address if reasonably far), and patient
name, and if by any chance the line gets disconnected, do not try to call me, just hang the
line and I will call you back, ok.
- Are you a nurse? Do you have someone else with you to help?
- Is the patient alert?
- Does he have a neck collar?
- Open his mouth, check it for any FB or blood, and make sure it is clear. Can you feel air
coming from patient mouth?
- What is his O2 saturation? Can you give him O2 please, 4 L/min via a nasal canula
- Can you open the neck collar window and check his trachea and neck jugular veins, is the
trachea central or not? Are the JV engorged?
- Can you look to his chest; is it bilateral symmetrical movements of the chest wall?
- Can you listen to air entry on both sides?
- I am suspecting tension pneumothorax, I would like you to insert a wide bore needle (16
or 14 G) into the second intercostal space, in the MCL (on the upper margin of the third
rib)
o If she said, I did not do it before, or I am afraid I can not do it, or who is
responsible for this: encourage her and tell her that you are responsible
- Did you get a gush of air?
o Yes, that is great, you have just saved the patient life
- Can you re-check the trachea centrality, jugular veins and HR again for me please
o If stable: can you please fix a chest tube in the 5th intercostal space, anterior
Axillary line
C: Vital signs / fluids / withdraw blood samples / D:
- Can you please insert two wide bore (16 G) IV lines in the patient ante-cubital fossae,
and give fluids (2 L NS bolus) and then withdraw samples for …
- Can you find any source of bleeding?
- D3: Give universal antidotes:
o Give 100 mg thiamine
o If ↓ RR  naloxone
o If ↓ blood sugar dextrose
- Can you tell me his Glasgow coma scale! Check his papillary reaction (D1).
- I would like also to intubate the patient

What are the requirements to transfer patient from a center to another center?
- Accompanied by two trained medical personnel (paramedics, nurses, physicians)
- Intubated and on ventilator
- Secured IV lines and fluids
- pre-arrangement with the place that will receive the patient

OSCE-guide-III.doc Page 105 of 255


Physical Examination

Physical Examination

OSCE-guide-III.doc Page 106 of 255


Physical Examination

Medical Physical Exam

─ Always be nice and smile


─ Use the alcohol anti-septic
─ Introduction
─ Drape / cover the patient properly
─ Warm your hands before touching the patient
─ Warm your stethoscope
─ If you are going to make painful manoeuvres: explain before / apologize for the pain / do
not repeat
─ Make sure you put the patient in the proper position at ease
─ Whenever you notice the patient is in pain, tell him that you will give pain medication as
soon as you can
─ If you ask the patient to walk, make sure he will not fall, surround him
─ Make sure you talk to the patient: I am going to look, feel, press, tap, listen (use
simplified non-medical language)
─ Report to the examiner – every thing – in medical terms, by inspection …, by palpation,
percussion, …
─ Cover the patient after you finish
─ Thank the patient and the examiner

Introduction:
- Good evening Mr …, I am Dr … I am the physician working in the clinic today / I am the
physician in charge in the ER now. I understand that you are here because you have been
having … For the next few minutes I am going to do physical exam for your … and I will
need to ask you questions during my exam. Also, I will be asking you to do some
movements and manoeuvres, if you feel any discomfort or pain, please do not hesitate to
let me know and stop me
- If you have any questions or concerns please feel free to ask me / to bring it up
- If SOB: during my exam, if you feel that you can not continue, please stop me

If there is history taking and then physical exam:


- I will be asking you some questions, then I will do physical exam. Hopefully towards the
end, we reach a working plan

Vital signs:
- If vitals are given: based on the vitals, the patient is stable, I would like to proceed. Or
the patient is unstable! Or comment: with mil fever
- If the vitals are missing one; e.g. the temperature: ask about it specifically
- Vitals are not gives:
o I would like to get the vitals before I start!
o I am going to start my exam by measuring your vital signs that is your blood
pressure, heart rate. And I will start by measuring your heart rate

OSCE-guide-III.doc Page 107 of 255


Physical Examination

Abdominal examination:
 Introduction / Vital signs / General inspection of the patient: pt is sitting comfortably …
- Inspection
- Auscultation: bowel sounds / bruits (aortic / renal / iliac)
- Percussion
- Palpation: superficial / deep / special tests
Respiratory examination:
 Introduction / Vital signs / General inspection of the patient: pt is sitting comfortably …
- Inspection: face / hand / neck / chest / back
- Palpation: tenderness / tactile fremitus / chest expansion
- Percussion: dullness / percussion note / diaphragmatic excursion
- Auscultation: regular / special tests
o Then end with cardiology exam
Cardiac examination:
 Introduction / Vital signs / General inspection of the patient: pt is sitting comfortably …
- Inspection: face / hand / neck / chest / heart (PMI)
- Palpation: apex / left para-sternal areas for heaves / valvular areas for thrills
- Auscultation: in Z format A-P-T-M
o Leg exam for edema
o Lung bases
- If full CVS exam  peripheral vascular assessment: abdominal bruits / legs pulses
palpation / chest exam
Musculoskeletal examination:
 Introduction / Vital signs / General inspection of the patient: pt is sitting comfortably …
- Inspection: SEADS (scars / erythema / atrophy / deformity / swelling) / specific findings
(bulk of muscles / bony symmetry)
- Palpation: (TTC) tenderness / temperature / crepitus / effusion
- ROM: active (if normal, NO need to do the passive) / passive / against resistance
- Special test: mechanical (shoulder / elbow / hip / knee / ankle)
o To complete my exam, I would like to do:
 Check the pulses of the limb (upper or lower)
 Brief neurological examination of the limb
 One joint above and one joint below examination
 The other side joint
Neurological exams:
 Introduction / Vital signs / General inspection of the patient: pt is sitting comfortably …
- Orientation: what is your name sir? Where are you? Time? Place?
- Cranial nerves
- Upper and lower extremities:
o Inspection
o Palpation / bulk
o Tone
o Motor power
o Sensory
o Reflexes
- Gait / Romberg test
- Cerebellar signs / Coordination
- Cortical sensations: two points discrimination

OSCE-guide-III.doc Page 108 of 255


Physical Examination

Abdominal examination:
- Introduction
- Vital signs
- General inspection of the patient:
o By general inspection, pt is lying down comfortably, no obvious distress
o Can I take a look at your eyes, would you please look downwards? No jaundice.
Upwards please? No pallor
o Would you please open your mouth: no signs of dehydration or vomiting
o Can I take a look at your hand?
 Temperature is fine / and skin is moist
 Normal capillary refill (< 3 seconds)
 No obvious clubbing
- I am going to drape you now!
o Bed flat
o Can you please put you hands to your sides
o Allowing the patient to bend his/her knees so that the soles of their feet rest on
the table will also relax the abdomen!

- Inspection; I will take a look at your abdomen:


o By inspection: abdomen is flat, not distended, normal contour, umbilicus is
inverted, abd is moving with respiration, no scars from previous surgeries, no
bruises, no obvious bulge or mass
o Would you please look to the left side, can you please cough
 Check pt face for tenderness  there is cough tenderness
 If no  cough another time  no herniation along the middle line.
Thank you, you can look back if you wish

- Auscultation; I am going to listen to your abdomen, warm my stethoscope:


o McBurney’s point: normal bowel sounds (hyperactive / decreased)
o 2 inches above the umbilicus  no aortic bruits
o 2 inches above / 2 inches bilateral from umbilicus  no renal bruits
o 2 inches below / 2 inches bilateral from umbilicus  no iliac bruits

- Percussion: now, I am going to tap on your abdomen, can you point to your painful are.
I am going to start away from there:
o Percuss in 2 X 2 lines, and percuss to side  for ascites
o No percussion dullness / normal tympanic percussion note / no percussion
tenderness / no ascites

- Palpation:
o I am going to feel your abdomen. Start away from the painful area:
 I am checking (name the 4 quadrants or the 9 areas of the abdomen);
(NO) tenderness, guarding or rigidity
o I am going to apply more pressure now: no obvious masses, no organomegaly
o I am going to feel your kidneys now (bimanual)  no enlargement, no
tenderness of the kidneys
o I am going to do some special tests:
 Murphy’s sign (Rt costal margin)  can you take a deep breath
 Rebound tenderness: I am going to press and release my hand, can you
tell me which causes more pain! (any point except McBurney’s)

OSCE-guide-III.doc Page 109 of 255


Physical Examination

 Press at McBurney’s point  is it painful


 Press left iliac area, tell me if there is pain:
• Not painful
• Pain to the left side: left iliac tenderness / negative Rovsing's
sign
• Pain to the right side  positive Rovsing's sign
/+/+/+/+/+/+/+/ COVER THE PATIENT ABDOMEN /+/+/+/+/+/+/+/
 Psoas sign: Rt LL extended; I am going to press on your thighs down, can
you press up against my hand (actively flex thigh at the hip)
 Obturator sign: Rt hip flexed, knee flexed; can you relax yourself, I will
rotate your leg (int and ext rotation), if pain  +ve sign

- Can you please sit up now; I am going to:


o Tap on your back, check the costo-vertebral angle for renal tenderness
o Listen to the back of your lungs  no crepitation
- Can you please lie down; thank you.
I would like to complete my exam by doing pelvic and vaginal exam / and digital rectal
exam
o Inguinal LNs
o Pelvic, vaginal and bimanual exam; looking for:
 Bleeding and/or discharge
 Adnexal masses / tenderness
 Cervical motion tenderness
 To obtain samples for culture and sensitivity
o Digital rectal exam; looking for:
 Bleeding
 Piles / haemorrhoids
 Fissures

How to differentiate between spleen and kidney by palpation:


- Spleen changes position with respiration (goes down with inspiration)
- You can feel the lower border of the spleen
- Spleen does not ballot

If there is history of Crohn’s disease:


- Inspection:
o General: truncal obesity / moon-like face
• Red eyes
• Mouth: ulcers
o Hands: nail changes / clubbing / skin rash / psoriatic changes / joint tenderness
o Abdomen: stria / scars
o Lower limbs: erythema nodosum
o DRE: fissures / fistulae
- When he sits up  press on the sacro-iliac joints  sacroiliitis

OSCE-guide-III.doc Page 110 of 255


Physical Examination

Liver Examination
Patient vomited blood 20 minutes ago, perform focused examination for liver cell failure
- Because the patient is vomiting blood, I would like to ask for protection for me and my
team please (gloves / gowns / masks / goggles)
- Introduction / Vital signs
- Orientation: I am going to ask you some questions which are part of my physical exam.
Do you know where you are now? What is the time? Do you know why you are here? 
Patient is oriented to people, time and place
- General:
o Patient is lying comfortably no signs of obvious distress
o Eyes  sclera: no jaundice
o Mouth: no fetor hepaticus / no clots / no vomitus
o Face: no parotid gland enlargement
o Hands: no clubbing / nail changes / palmar erythema / Dupuytren’s contracture / atrophy
of thenar and hypothenar muscles / look for IV marks / stretch your hands please
(flapping tremors)
o Can you please lower your gown: no gynecomastia / no spider nevi
o Check lower limbs edema
- Inspection:
o No bruises / normal hair distribution / no caput medusa / no dilatation of collateral veins /
no obvious ascites / no scratch marks
- Auscultation
o Listen to liver: no hepatic rub / hum / or bruits
o Listen to spleen: no rub / no hum
- Percussion:
o 2 X 2 lines tap
o Liver: MCL (from above downwards and from down upwards) liver span
o Spleen: ant axillary line  last intercostal space / ask pt to take deep breathe in / then re-
percuss for the spleen
o Ascites: from midline to the side, no dullness  so there is no need to perform the
shifting dullness (to be clinically palpable: ascites > 500 ml)
- Palpation:
o Liver: start from the right iliac fossa and go upwards, while the patient is breathing in and
out (push during inspiration, do not move your hand from the patient)  margin of liver
is not palpable, not tender, and not nodular.
o Spleen: patient elevates his LEFT side 45°, support from left back. Start from above the
umbilicus towards the spleen  spleen is not palpable
- I would like to complete my exam by doing:
o Digital rectal exam: for hemorrhage / piles
o Check for testicular atrophy

 DD of hepatitis: (alcoholic / viral / drug-induced) / cirrhosis / liver cancer


 Worst prognostic signs: ascites / encephalopathy
 Investigations: AST:ALT > 2:1, GGT, albumin, INR, CBC
 Treatment of encephalopathy: treatment of precipitating causes / Lactulose (15-30 ml tid) /
Rifaximin (550 mg PO bid)

 DD of ascites: liver cirrhosis / renal failure / heart failure / TB / malignancy /


Hypoalbuminemia
 Treatment of ascites: Sodium restriction (20-30 mEq/d) / Diuretic therapy / Therapeutic
paracentesis may be performed in patients who require rapid symptomatic relief for refractory
or tense ascites

OSCE-guide-III.doc Page 111 of 255


Physical Examination

Nasal bleeding  Hematological Examination

- Because the patient is bleeding, I would like to ask for protection for me and my team
please (gloves / gowns / masks / goggles)

- Introduction
- Vital signs

- General:
o Patient is lying comfortably no signs of obvious distress
 Nose: open the speculum antero-posteriorly
 Use the otoscope for ENT (nose / ear / mouth)
o Look for bruises / petichae  if you find them  continue hematological exam

- I ma going to feel your glands:


o Occipital / post-auricular / pre-auricular / sub mandibular / sub mental / anterior /
posterior cervical / supra-clavicular
o I would like to check the axillary LNs (ant / post / med / lat / apical)
o I would like to check the inguinal LNs:
 Superficial (transverse, along the groin crease)
 Deep:
• Transverse: along the groin crease
• Longitudinal: medial to the femoral artery
(Lat  Med: NAVAL – Nerve Artery Vein LNs)
o I would like also to check the popliteal LNs
- Check for bony pains: sternum and 3-5 spots on the vertebral column (I am going to press
on your back)

CAN YOU LIE DOWN PLEASE?


- Listen to heart / lungs
- Abdominal exam:
o Inspection
o Auscultation
o Percussion: liver / spleen
o Palpation: liver / spleen
o I would like to: perform vaginal and pelvic exam

- Finally, I would like to do:


o Fundoscopic retinal exam
o Orientation
o Brief neurological exam:
 Hands: pull/push/ do you feel the same
 Legs: pull/push/ do you feel the same
Patient is grossly neurologically free
N.B. focal neurological deficits to DD between TTP and HUS.

OSCE-guide-III.doc Page 112 of 255


Physical Examination

Questions:
- Diagnosis:
o ITP (Immune Thrombocytopenic Purpura)
 Most common cause of isolated thrombocytopenia
 Diagnosis of exclusion (i.e. isolated thrombocytopenia with no clinically
apparent cause)
- Investigations:
o CBC: thrombocytopenia
o Peripheral blood film: decreased platelets, giant platelets
• Bleeding time: increased / PT and aPTT: normal
• Anti-platelets antibodies
o Bone marrow: increased number of megakaryocytes (critical test to rule out other
causes of thrombocytopenia for age > 60 years; e.g. myelodysplasia)
o Markers of hemolysis: increased unconjugated bilirubin, increased LDH,
decreased haptoglobin
o Kidney function tests (urea / creatinine for HUS)
- Treatment:
o Steroids (methylprednisolone 1 g/d for 3 days, then prednisone 1.5 mg/kg/day)
o Immunoglobulins (if low platelet count): IVIG 1 g/kg/d X 2 days
o Splenectomy
o Vaccination (pneumococcus, meningococcus, HIB)
- DD:
o ITP (Immune Thrombocytopenic Purpura)
o TTP (Thrombotic Thrombocytopenic Purpura)
o HUS (Hemolytic Uremic Syndrome):
ITP TTP HUS
 Remitting / relapsing  Predominantly adults  Predominantly children
course  Thrombocytopenia  Severe thrombocytopenia
 Mild fever  Micro-angiopathic  Micro-angiopathic
 Splenic discomfort hemolytic anemia hemolytic anemia
(mild engorgement) (MAHA) (MAHA)
 Renal failure  Renal failure
 Neurological symptoms
(headache, confusion,
focal deficits, seizures)
 Fever
Investigations  CBC and blood film: decreased platelets and schistocytes
(both TIP, HUS)  PT, aPTT, fibrinogen: normal
 Markers of hemolysis: increased unconjugated bilirubin.
increased LDH, decreased haptoglobin
 Negative Coombs' test
 Creatinine, urea, to follow renal function
 Stool C+S (HUS)
Management  Plasmapheresis ± steroids
(both TIP, HUS)  Platelet transfusion is contraindicated (increased micro-
vascular thrombosis)
 Plasma infusion: if plasmapheresis is not immediately
available
 TTP mortality – 90% if untreated

OSCE-guide-III.doc Page 113 of 255


Physical Examination

Chest Examination
- Introduction
- Vital signs; especially: tachypnea / temperature
- General inspection of the patient; comment on respiratory distress
- Inspection:
o General:
▬ Face: symmetrical, no nasal flaring / laboured breathing
▬ Eyes: jaundice / pallor
▬ Mouth / can you flip your tongue please: no central cyanosis / dehydration / exudates
or secretions
▬ Hands: no nicotine staining / clubbing / peripheral cyanosis / normal capillary refill
o Neck; can you lower your gown please
▬ Trachea is central, no accessory ms used for breathing / LNs
o Chest:
▬ Chest is symmetrical, no accessory ms used for breathing, no intercostal retraction
▬ Look from the side: no increase in the A/P diameter of the chest / no barrel chest
- Palpation; I am going to feel your chest
o Tenderness:
▬ Check the ant chest wall: no tenderness
o Tactile fremitus: can you say 99 for 4-5 times, whenever you feel my hands on
your chest  normal tactile fremitus
▬ Any lung pathology  ↑ tactile fremitus (↑ conduction)
▬ Any pleural pathology  ↓ tactile fremitus (insulation
o Chest expansion: normal chest expansion
/+/ I will continue to examine you from the back, then I will check the front again /+/ Can
you cross your arms please
- Inspection: chest is symmetrical, no scars, swellings, or deformity
- Palpation; I am going to feel your chest
o Tenderness: no tenderness
o Tactile fremitus: can you say 99 for 4-5 times, whenever you feel my hands on
your back
- Percussion; I am going to tap on your chest
o Apex (2) / back (6) / sides (2)  normal percussion note:
- Auscultation; now, I am going to listen to your back
o Whenever you feel the stethoscope touching your back, can you please take a
deep breathe in and out from your mouth
o Normal vesicular breathing sounds bilaterally, no rhonchi, no wheezes
- Now, I am going to demonstrate the special tests that should be done if there is
consolidation, with auscultation:
o Can you say letter “E”  egophony (normal: ee / over affected area: ay)
o Can you say “99”  ↑ vocal fremitus
o Whisper 1/2/3: whispering pectorology (↑ in audibility)

/+/ I will now go to examine you from the front again /+/
- Percussion; I am going to tap on your chest – on both sides
o Apex (2) / MCL (6) / sides (2)  normal percussion note:
- Auscultation; now, I am going to listen to your lungs
o Mid clavicular line – both sides
o Normal vesicular breathing sounds bilaterally, no rhonchi, no wheezes
- Cardiology exam: Auscultation

OSCE-guide-III.doc Page 114 of 255


Physical Examination

Pneumonia

▬ CURB 65 score – Pneumonia Clinical Prediction Tool


Confusion
Urea: > 7 mmol, BUN > 19
Respiratory rate: > 30 breaths / min
Blood pressure: systolic < 90 and diastolic < 60
65: or older in age
Management Mortality
0–1 Can treat as outpatient <5%
2–3 Consider hospitalization <10%
4–5 Consider ICU <30%

▬ Pneumonia severity index: another clinical index (scored on age, resp rate, co-morbidities …),
used to determine whether to admit the patient to the hospital or not.

IDSA / ATS Community Acquired Pneumonia Treatment Guidelines 2007:


(IDSA: Infectious Diseases Society of America, ATS: American Thoracic Society)
Setting Circumstances Treatment Medications
Outpatient Previously well Macrolide  Azithromycin 500 mg PO once,
OR THEN 250 mg qDay for 4 day
Doxycycline (total of 5 days)
 Clarithromycin 500 mg PO bid
for 7-14 days
Comorbidities, or Beta-lactam + Macrolide  Levofloxacin 750 mg PO q24h
use of antibiotics OR for 5 days
in last 3 months Respiratory fluoroquinolones OR
Inpatient Ward  Amoxicillin 1000 mg PO tid
+ Clarithromycin 500 mg PO
bid
ICU9 3rd gen cephalosporin + Ceftriaxone 1 g IV q24h
Macrolide + Azithromycin 500 mg IV q24h
OR for 5 days (step to oral when
Beta-lactam + Macrolide tolerated)

Other medications for the management of pneumonia:


─ Susceptible for pseudomonas / recent use (within 3 months) of antibiotics or cortisone:
piptazo (piperacillin / tazobactam); 3.375 gm IV q6h)
─ MRSA: Vancomycin 1 gm IV q24h

Pneumococcus vaccination:
─ Elderly patients > 65 yrs old
─ COPD
─ CHF
─ Cirrhotic
─ Cancer
─ Immunocompromised: steroids / HIV / DM / splenectomy
─ Leukemia / lymphoma
9
Especially in the presence of risk factors: group home / hospital infection / immunocompromised

OSCE-guide-III.doc Page 115 of 255


Physical Examination

Case: HIV positive man – C/O: SOB for 1 week / cough / fatigue ? Pneumonia (? PCP)

▬ DD of unilateral lobar reticular pattern on CXR: PCP (pneumocystis pneumonia) / Kaposi


sarcoma in the lung / Lymphoma / Atypical pneumonia (mycoplasma / chlamydia)
▬ DD of bilateral hilar infiltrates: PCP / TB / CMV
▬ Signs of HIV: check the entire skin for Kaposi sarcoma / mouth and pharynx for thrush or oral
hairy leukoplakia / LNs / check the abdomen for liver and spleen
▬ Investigations:
o O2 saturation / ABG
o CBC / differential / CD4 count
o LDH (↑ in 95% of PCP cases)
o Blood culture
o Sputum for cytology / culture / gram stain / acid fast stain
o Bronchoscopy: bronchial washing
▬ Treatment of PCP:
o Septra DS (trimethoprim / sulfamethoxazole: 160mg/800mg): 2 tablets q8h x 14 days then
continue prophylactic 1 tab od
o Severe illness (PO2 < 70 mmHg):
 Admit
 O2
 Septra
 Steroids: prednisone 40 mg bid x 5 d then 40 mg od x 5 d then 20 mg od x 5 d
o If patient is allergic to TMP/SMZ:
 For treatment: IV pentamidine or atavaquone
 For prophylaxis: Dapsone or atavaquone

▬ HIV routine prophylaxis:


o Based on CD4 count:
 CD4 < 200  prophylaxis for PCP: TMP/SMZ or Dapsone
 CD4 < 100  prophylaxis for toxoplasma: TMP/SMZ or Dapsone
 CD4 < 50  prophylaxis for mycobacterium avium complex: azithromycin
(1/week) or clarithromycin (2/d)
o Regardless the CD4 count:
 PPD testing for TB, considered positive if > 5 mm induration  INH for 9 months
 Vaccinations for pneumococcus / influenza / hepatitis B

OSCE-guide-III.doc Page 116 of 255


Physical Examination

Cardiac Examination – Essential HTN case


+ 65 years old gentleman, with high blood pressure for 20 years, for follow-up visit
+ 35 years old gentleman with newly diagnosed high blood pressure, please assess
+ If 10 minutes case: vitals + examination + If 5 minutes: only examination

Analysis ▬ For how long? Medications?


▬ How frequent do you check your BP? Is it controlled?
Impact ▬ End organ damage? Proteinuria / visual changes / headaches /
confusion?
Red flags ▬ Other cardiovascular risk factors: smoking / DM / high
cholesterol / history of MI in the family

Introduction Can you please lie down


HR and BP (lying down) Get the systolic by pulse palpation, then increase 30 mmHg, then
auscultate
Sit up, relax for 2 minutes  Orientation
 Mouth for dehydration
 Eyes for pallor
 Lung bases auscultation
 Sacral edema
 Lower limb edema
HR and BP (sitting up) No difference
General  Asses the radial pulse bilaterally
 Hands: capillary refill / clubbing / temperature
 Fundoscopic examination: disc edema / retinal hge
Neck Carotid  Carotids: bruits / palpate
JVP  JVP: measure / Kussmaul sign / hepato-jugular reflux
Heart Inspection  Symmetrical / PMI
Palpation  Apex
 Heaves (Lt para-sternal / with knuckles)
 Thrills (A-P / with palmar aspect of distal phalanges of fingers)
Auscultation Do timing by simultaneously checking the carotid pulse.
 Areas A-P-T-M: normal S1, S2, no murmurs.
 Look for S3, S4 at the apex (with the bell)
 Aortic insufficiency murmur
 Lung bases
Abdominal exam Pulsation / Bruits (aortic / renal / iliac)
Lower limbs exam Inspection / Palpation (temperature / pulsations) / LL BP
- Introduction
- Vital signs; especially: BP / HR
o Ask the patient to lie down, I am going to assess your blood pressure twice, one
while lying down, the other after you sit up
 I will start by measuring HR for 10 seconds; your HR today is … which is
normal
 Do the BP: get the systolic by pulse palpation, then increase 30 mmHg, then
auscultate  your BP today is …
o Can you please sit up on the edge of the bed, (feet dangled)? I need you to relax
for 2 minutes, before I proceed to other measurements, during which I am going
to ask you some questions:

OSCE-guide-III.doc Page 117 of 255


Physical Examination

 Orientation: Do you know where you are? Why? Date?


 Mouth for dehydration
 Eyes for pallor
 Can I listen to the back of your lungs (lung bases)  no crackles
 Press against the sacrum for sacral edema
 Press on the lower limbs, for edema
o Measure the sitting HR and BP  no difference in readings  no orthostatic
hypotension (no changes: no ↓ 20 mmHg in systolic, no ↓ 10 mmHg in diastolic,
no ↑ 20 in HR)
- General exam:
o Comment on the general status / respiratory distress of the patient
o Assess the radial pulse bilaterally: pulse is normal, regular, equal on both sides,
with no delay
o Hands: normal capillary refill / no clubbing / warm moist skin
o I would like to do fundoscopic examination (I am going to shine a light in your
eyes to examine the back of your eyes): looking for disc edema / cotton wool
exudates / retinal hemorrhage / nipping of the veins
- I am going to put the bed in 45° position; examine your neck:
o Carotids:
 Auscultate for bruits  use the bell
 If no bruits  palpate the carotid arteries (ONE AT A TIME)
 Pulse is regular, normal volume and contour
o JVP:
 Measure the JVP
 Kussmaul test: take a deep breathe and hold it, measure the JVP:
• Normally it will ↓  negative Kussmaul test
• If ↑ JVP  positive Kussmaul test (restrictive cardio-myopathy,
constrictive pericarditis)
 Press on the liver for 10 seconds  normally the JVP will ↑ for few seconds
then will ↓  if it does not ↓  hepato-jugular reflux
- Chest inspection:
o Chest is symmetrical, no obvious pulsations
o PMI (point of maximum intensity) is not obvious / or is obvious
- Palpation:
o Palpate the apex and locate it  apex is palpable at … in the MCL, not
hyperdynamic (vol ovrld), not sustained (pres ovrld), not displaced
o Palpate the left para-sternal area by knuckles: no Rt ventricle heave
o Palpate the Aortic / Pulmonary areas by finger tips  no thrills
- Auscultation:
o In Z format A-P-T-M: normal S1, S2, no murmurs
o Flip the bell; check the apex for S3, S4
o While listening to the left para sternal area: can you please take a deep breathe in,
exhale it out and hold, and lean forward  no enhancement of aortic
insufficiency murmurs
o Listen to the lung bases  no basal lung crackles
- I am going to put the bed flat now:
o Abdominal exam: inspect for pulsations / auscultate for bruits
o Lower limb exam:
 Inspection: symmetry / SEADS / nails hypertrophy / skin changes
 Palpation: temperature / pulsations / radio-femoral delay
 I would like to measure the blood pressure in lower limb

OSCE-guide-III.doc Page 118 of 255


Physical Examination

Secondary Hypertension

A 25 years old young man with HTN;

Cardiac Renal Endocrine


 Coarctation of the aorta  Poly-cystic kidney  OCPs
 Cocaine (nasal septum)  Renal artery stenosis  Cushing disease / syndrome
 Pheochromocytoma
 Primary hyperaldosteronism
 Hyper / hypo thyroidism

▬ Patient disrobed to underwear, draped below waist


Introduction Can you please lie down
Vitals  There is systolic and diastolic HTN
 I want to rule out orthostatic hypotension (pheochromocytoma)
 I would like to compare UL and LL (coarctation of aorta)
 No tachycardia (pheochromocytoma / hyperthyroidism)
 No bradycardia (hypothyroidism)
General  Orientation (brain edema)
 Truncal obesity / cervical fat pad / moon face (Cushing)
 Check for proximal ms weakness (Cushing)
 Check the eyes from the side for proptosis, lid lag test
(hyperthyroidism)
 Asses the radial pulse bilaterally
 Hands: capillary refill / temperature / skin dry or wet (thyroid).
Stretch your hands please  fine tremors (hyperthyroidism)
 Fundoscopic examination: disc edema / retinal hemorrhage
Neck Thyroid  No masses
 Palpate for the thyroid
 Percuss for the retro-sternal thyroid dullness
JVP  JVP
Chest  Palpate the apex
 Look for S3, S4 at the apex (with the bell)
 Listen to lung bases
Abdominal exam  Inspection: no obvious masses, no stria, no café au lait spots
 Auscultation: bruits (aortic / renal)
 Palpation: feel for renal / supra-renal mass
Lower limbs exam  Edema
 Rule out femoral / radial delay
Brief neuro exam  Knee or ankle reflex

Notes:
▬ Watch for labile white coat HTN
▬ Any HTN in middle age  secondary HTN  most likely kidney disease

OSCE-guide-III.doc Page 119 of 255


Physical Examination

Hypertension
 Predisposing Factors
─ Family history ─ Sedentary lifestyle ─ Excessive salt intake /
─ Obesity ─ Smoking fatty diet
─ Alcohol consumption ─ Male gender ─ African American
─ Stress ─ Age >30 ─ Dyslipidemia
 Diagnosis:
─ Visit ONE:
o If hypertension urgency or emergency (sBP > 210 or dBP > 120) or end organ damage
(e.g. confusion)  diagnose HTN
o Else (provided 2 more readings during same visit)
 Search for target organ damage: history (cardio-vascular risk factors) / examination
 Investigations:
▬ CBC / Na+, K+ / fasting blood sugar / lipids (total cholesterol, HDL, LDL, TG)
▬ Kidney function tests / Urinalysis / Renal Doppler
▬ ECG / Echocardiogram
▬ For secondary HTN: TSH / Plasma aldosterone / renin levels / 24 hours urine
for metanephrines / VMA
 Life style modifications (↓ salt / ↓ alcohol / ↓ cholesterol / exercise)
 Follow-up visit within 4 weeks
─ Visit TWO; within 4 weeks
o If (target organ damage OR diabetes mellitus OR chronic kidney disease OR blood
pressure > 180/110 mmHg)  diagnose HTN
o Else (BP: 140-179 / 90-109 mmHg)  24 hours BP monitor (diagnose if mean awake
sBP >135 mmHg and/or dBP > 85 mmHg or mean 24 hours sBP >130 mmHg and/or
dBP > 80 mmHg)
 Management:
─ Target BP is < 140/90 mmHg, < 130/80 if DM or chronic kidney disease
─ Life style modifications (initial management):
o Smoking cessation and decrease alcohol consumption
o Diet: ↓ salt / ↓ cholesterol and saturated fats / follow Canada's Guide to Healthy Eating
o Weight: maintain healthy BMI (18.5-24.9)
o Moderate intensity dynamic exercise: 40-60 minutes, 4-6 times/week
─ Pharmacological:
o First line: Diuretics; e.g. hydrochlorothiazide 12.5 – 25 mg PO od Except:
 DM: ACEIs; Ramipril 2.5 – 5 mg PO od
 Gout: Amlodipine (5 mg PO od) OR Candesartan (4 – 8 mg PO od)
 Elderly (especially if IHD):
─ ACEIs
─ β-blockers: metoprolol 25 mg bid
Especially if CHF / EXCEPT: asthma / bradycardia
 Pregnant:
─ Hydralazine: 10 mg PO qid for few days then 25 mg PO qid
─ OR α-methyl dopa: 250 mg PO bid
 If > 3 cardiovascular RF: statins / ASA
o If partial response to standard dose monotherapy, add another first-line drug
 Do NOT give β-blockers and Ca ch blockers  may cause heart block
 Do NOT give ACE and ARBs  both ↑ K+,
 Available combinations: Altace plus (ramipril + diuretic) / Diovan H
o Notes on ACEIs:
 Contraindications of ACEIs: Angio edema / Bilateral renal artery stenosis
 ACEIs are nephroprotective except in acute renal injury  nephrotoxic
 If patient on ACEIs developed cough  switch to ARBs
─ HTN emergency: Hydralazine: 20 – 40 mg IV or IM, repeated as necessary, decrease the dose in
case of renal impairment

OSCE-guide-III.doc Page 120 of 255


Physical Examination

SOB – shortness of breathe

Patient who had a car accident 24-48 hours ago developed SOB.
Complications: 1st day: atelectasis / fat embolism. 3rd day: DVT / PE

Common causes: Cardiac (CHF) / Lungs (asthma / COPD /


pneumonia / PE)
Brief cardio-pulmonary Life threatening conditions: MI / PE / pneumothorax / aortic
hx dissection
Vitals I would also like to rule out orthostatic hypotension (? hypovolemia)
General  General assessment of the patient
 Orientation (SOB  hypoxia  confusion)
 Face: eyes / no signs of respiratory distress / mouth (no signs of
dehydration / central cyanosis)
 Hand: temperature / capillary refill / skin / no peripheral cyanosis
/ no nicotine staining / comment on both pulses
 Sacral edema
Neck Trachea  Trachea centrality
JVP  JVP: not engorged
Chest Inspection  Symmetry
 Accessory muscles breathing / intercostal retraction
 Obvious pulsations / PMI
 If car accident (bruises on chest wall)
Palpation  Tenderness
 Tactile fremitus
 Apex
 Para-sternal heaves
Percussion  Chest back for any dullness
Auscultation  Heart (ask the patient to turn slightly to left)
 Lungs: 8 spots scan for air entry
 Lung bases
Lower limbs exam  Inspection: SEADS
 Palpation: temperature / capillary refill / peripheral edema /
pulses / press for pain in calf muscle area
 Special tests:
- Measure leg circumference 10 cm below tibial tuberosity;
difference should be < 2.5 cm between the two calves
- Homans' sign10: there is pain in the calf or popliteal region
with examiner's abrupt dorsiflexion of the patient's foot at the
ankle while the knee is flexed to 90 degrees

Indications for intubation: ABG showing poor PO2 (60s) / elevated PCO2 (80s) / acidosis / GCS
score < 8

10
A positive Homans' sign does not positively diagnose DVT (poor positive predictive value), and also negative Homans' sign does
not rule out the DVT diagnosis (poor negative predictive value), and there is theoretical possibility of dislodging the DVT.

OSCE-guide-III.doc Page 121 of 255


Physical Examination

DVT

Clots clinical probabilities / risk factors (Virchow's Triad):


─ Vascular injury (endothelial damage); e.g. recent surgery
─ Venous stasis; immobilization (post-MI, CHF, stroke, post-operative, obesity, long
travel or flight, chronic venous insufficiency) inhibits clearance and dilution of
coagulation factors
─ Hyper-coagulability; aging, surgery, trauma, malignancy, antiphospholipid antibody
syndrome, hormone related (pregnancy, OCP, HRT, SERMs)

DD: muscle strain or tear, lymphangitis or lymph obstruction, venous valvular insufficiency,
ruptured popliteal cysts, cellulitis, and arterial occlusive disease

Wells Clinical Score for Deep Venous Thrombosis

Clinical Parameter Score


Active cancer (treatment ongoing, or within 6 mo or palliative) +1
Paralysis or recent plaster immobilization of the lower extremities +1
Recently bedridden for >3 d or major surgery < 4 wk +1
Localized tenderness along the distribution of the deep venous system +1
Swollen unilateral superficial veins (non-varicose) +1
Entire leg swelling +1
Calf swelling >3 cm compared with the asymptomatic leg +1
Pitting edema (greater in the symptomatic leg) +1
Previous DVT documented +1
Alternative diagnosis (as likely or greater than that of DVT) -2
A Wells score can be interpreted in a binary (likely vs. unlikely) or ternary (low, moderate, or
high probability) fashion.
Ternary interpretation Binary interpretation
≥3 High probability score ≥ 2 DVT likely
1 or 2 Moderate probability score < 2 DVT unlikely
≤0 Low probability
In the unlikely group (low-to-moderate risk of DVT)  D-dimer assay (ELISA):
─ Negative  rule-out DVT
─ Positive  duplex ultrasonography
o Negative  rule-out DVT
o Positive  treat for DVT
In the likely group (moderate-to-high risk of DVT)  duplex ultrasonography
─ Negative  repeat clinical evaluation and ultrasonography in 1 week
─ Positive  treat for DVT
When discordance exists between the pretest probability and the duplex ultrasonographic study
result, further evaluation is required.
─ Other non-invasive tests include MRI.
─ Venography is the gold standard, but is expensive, invasive and higher risk

OSCE-guide-III.doc Page 122 of 255


Physical Examination

Initial treatment of DVT:


─ Unfractionated heparin (UFH): requires bolus (7500-10,000 ill), followed by
continuous IV infusion (1000-1500 IU/h). Advantages: rapidly reversible by
protamine. Disadvantages: must monitor aPTT
─ Low molecular weight heparin (LMWH): administered SC. Predictable dose response
& fixed schedule
Long-term treatment:
─ Warfarin: standard treatment;
o Should be initiated with heparin overlap – dual therapy for at least 5 days.
Discontinue heparin after INR >2.0 for two consecutive days.
o Warfarin should be dosed to maintain INR at 2-3. Monitor INR twice weekly for
1-2 weeks, then weekly until INR stable, then every 2-4 weeks
o Duration of anticoagulant treatment (with warfarin unless otherwise noted):
─ First episode DVT with transient risk factor: 3 months
─ First episode DVT with ongoing risk factor (e.g. cancer, antiphospholipid
antibody) or > 1 risk factor: consider indefinite therapy
─ Recurrent DVT (2 or more episodes): indefinite therapy
─ IVC filters: useful in those with contraindications to anticoagulant therapy, recurrent
thromboembolism despite adequate anticoagulation, recurrent embolism with
pulmonary HTN, or those who require emergent surgery without time to initiate
anticoagulation
─ Special considerations:
o Pregnancy: treat with LMWH during pregnancy, then warfarin for 4-6 weeks
post-partum
o Surgery: avoid elective surgery in first month after venous or arterial
thromboembolic event
─ Preoperatively: IV heparin may be used up to 6 hrs pre-operatively;
warfarin should be discontinued for at least 4 days pre-op. Surgery safe
when INR <1.5
─ Postoperatively: IV heparin or LMWH can be started 12 hours after
major surgery

Wells Prediction Rule for Diagnosing Pulmonary Embolism

For Predicting Pretest Probability of PE (N.B. the guideline notes that the Wells rule performs
better in younger patients without comorbidities or a history of venous thromboembolism)
Clinical Characteristic Score
Previous pulmonary embolism or deep vein thrombosis + 1.5
Heart rate >100 beats per minute + 1.5
Recent surgery or immobilization (within the last 30 d) + 1.5
Clinical signs of deep vein thrombosis +3
Alternative diagnosis less likely than pulmonary embolism + 3
Hemoptysis +1
Cancer (treated within the last 6 mo) +1

Score Clinical Probability of PE Simplified Wells


0-1 Low score > 4 PE likely
2-6 Intermediate score ≤ 4 PE unlikely
≥7 High

OSCE-guide-III.doc Page 123 of 255


Physical Examination

Evaluation of suspected PE: Notes:


─ Low clinical probability of PE  D-dimer: 1. Use D-dimers only if low clinical
o Negative: ruled out probability, otherwise, go straight to spiral
o Positive  CT scan with contrast: CT or V/Q
─ Negative: ruled out 2. If using V/Q scan (CT contrast allergy or
─ Positive: ruled in renal failure):
─ Intermediate OR high probability: ─ Negative V/Q scan rules out the diagnosis
o CT scan: ─ Inconclusive V/Q scan requires leg US
─ Negative: ruled out duplex to look for DVT (q2d)
─ Positive: ruled in ─ High probability V/Q scan only rules in the
diagnosis if have high clinical suspicion

CXR of PE: may be normal / wedge-shaped infiltrate / unilateral effusion / raised hemi-
diaphragm

Treatment of PE:
─ Admit for observation (patients with DVT only are often sent home on LMWH)
─ Oxygen: provide supplemental O2 if hypoxemic or short of breath
─ Pain relief: analgesics if chest pain – narcotics or NSAIDs
─ Acute anticoagulation: therapeutic-dose SC LMWH or IV heparin – start ASAP
o Anticoagulation stops clot propagation, prevents new clots and allows
endogenous fibrinolytic system to dissolve existing thromboemboli over months
o Get baseline CBC, INR, aPTT ± renal function ± liver function
o For SC LMWH: dalteparin 200 U/kg once daily or enoxaparin 1 mg/kg bid – no
lab monitoring – avoid or reduce dose in renal dysfunction
o For IV heparin: bolus of 75 U/kg (usually 5,000 U) followed by infusion starting
at 20 U/kg/hr – aim for aPTT 2-3 times control
─ Long term anticoagulation:
o Warfarin – start the same day as LMWH/heparin – start at 5 mg PO od – overlap
warfarin with LMWH/heparin for at least 5 days and until the INR is in target
range of 2-3
o LMWH instead of warfarin for pregnancy; active cancer, high bleeding risk
o Duration of long-term anticoagulation treatment:
─ If reversible cause for PE (surgery, injury, pregnancy, etc.): 3-6 months
─ If PE unprovoked OR ongoing major risk factor (active cancer):
indefinite
─ IV thrombolytic therapy:
o If patient has massive PE (hypotension or clinical right heart failure)
o Hastens resolution of PE but may not improve survival or long-term outcome
─ Interventional thrombolytic therapy (massive PE is preferentially treated with
catheter directed thrombolysis by an interventional radiologist, works better than IV
thrombolytic therapy and fewer contraindications)
─ IVC filter: only if recent proximal DVT + absolute contraindication to
anticoagulation

OSCE-guide-III.doc Page 124 of 255


Physical Examination

Peripheral Arterial Disease Examination

Introduction
Vitals Based on the vitals, the patient is stable, I would like to proceed …
General
Inspection  Drape the patient / expose the lower limbs (triangular)
 I would like to take a look at your feet, can you please remove the socks; do
you want me to help you?!
 SEADS
 No signs of arterial insufficiency: no hair loss / no shiny tight skin / no
hypertrophic nails
Palpation  Temperature
 Capillary refill (< 3seconds)
 Pulses: dorsalis pedis / posterior tibial / popliteal / (to examiner) I would like
to check the femoral arteries
Abdomen  Drape the patient / I would like to examine your abdomen / can you please
uncover your abdomen
 Listen for bruits (aortic / renal / iliac)
Neurology  Check for light touch, here is a piece of cotton, this is how it feels; can you
please close your eyes! Tell me when you feel it touching you! Check both
lower limbs from distal to proximal
 If light touch is ok, do not proceed with more tests
Burger test I would like to raise your legs, for 1-2 minutes, if you feel any pain / numbness /
tingling please let me know, check the color of the foot. Then dangle the feet and
check the color  no pallor on elevation, no rubor on dependence  Burger test
is negative.
Special tests I would like to arrange for ankle / brachial index

Investigations:
▬ CBC
▬ Fasting blood sugar / lipid profile
▬ ECG
▬ Angiography (side effects: nephrotoxic / allergy / aneurysm risk)
▬ Doppler U/S study of the arterial tree both lower limbs

Treatment:
▬ Life style modifications (refer to HTN)
▬ Foot care
▬ Graded exercise
▬ Surgery (if severe disability)

OSCE-guide-III.doc Page 125 of 255


Physical Examination

Diabetic Foot

Diabetic patient with long hx of diabetes, has an ulcer for few days

Introduction
Vitals Based on the vitals, the patient is stable, I would like to proceed …
General
Inspection  Drape the patient / expose the lower limbs (triangular)
 I would like to take a look at your feet, can you please remove the socks; do
you want me to help you?!
 Describe the ulcer: location (in the sole at base of 1st metatarsal), shape
(round, irregular), size (… cm), margins not elevated, no active
bleeding or oozing
 No other ulcers in the same foot / check the other foot / check in
between toes  no evidence of infection in between toes / in nails
 No pigmentation around the medial and lateral malleoli
 SEADS – quadriceps wasting / swollen joints
 No signs of arterial insufficiency: no hair loss / no shiny tight skin / no
hypertrophic nails
Palpation  Temperature
 Capillary refill (< 3seconds)
 Pulses: dorsalis pedis / posterior tibial / popliteal / (to examiner) I would
like to check the femoral arteries
Abdomen  Drape the patient / I would like to examine your abdomen / can you please
uncover your abdomen
 Listen for bruits (aortic / renal / iliac)
Neurology  Check for light touch, here is a piece of cotton, this is how it feels; can you
please close your eyes! Tell me when you feel it touching you! Check both
lower limbs from distal to proximal
 If light touch is ok, do not proceed with more tests
 Light touch sensation is absent distal to the level of … cm above ankle
 Proprioception: I will move your toe, close your eyes please, tell me is
it up or down. Then move to the next joint. Start with head of
metatarsal, medial malleolus, tibial tuberosity, …
 Vibration: tuning fork, here is the sensation you will feel, tell me when
it stops  intact / decreased / absent
 Monofilament test: to distinguish between the light touch and pressure
sensation / 10 points on the foot (9 on the sole, and one on the dorsum
above the big toe meta-tarso-phalangeal joint)
 Ankle reflex; if you have time: knee reflex and Babinski

Burger test
Special tests I would like to arrange for ankle / brachial index

Balance (if  Gait ± ataxia


DM focused  Romberg test: positive (with open eyes: pt can balance himself by vision;
exam case) while with closed eyes: pt loses balance

OSCE-guide-III.doc Page 126 of 255


Physical Examination

Neurological Examination

o Introduction
o Vital signs
o General inspection of the patient: pt is sitting comfortably …

- Orientation: what is your name sir? Where are you? Time? Place?
- Cranial nerves
- Upper and lower extremities:
o Inspection
o Palpation / bulk
o Tone:
 Just relax please, let me do everything for you. I am going to check the
tone in your Rt arm …
 Tone is normal, no hypo or hyper tonia
o Motor power (5  0)
 5 full power
 4 less than full power (like Lt hand in Rt handed person)
 3 can do the movement against gravity
 2 can do the movement with the gravity eliminated
 1 muscle twitches, not able to initiate movements
 0 no power – no movements
o Sensory:
 Light touch:
• Pin prick or piece of cotton
• First check on forearm or sternum
• Can you close your eyes please
• Distal to proximal
• Bilateral  sensation is equal bilaterally
 Posterior column (B12 deficiency / alcohol / syphilis):
• Vibration sense: tuning fork / test on sternum / tell me when it
stops / start distal / if intact move on / if not intact go proximal
on the next joint
• Proprioception: eyes closed / start with the big toe or thumb / is
it moving or not? / is it up or down?
o Reflexes:
 0 absent
 1 weak (hyporeflexia)
 2 normal
 3 hyper reflexia
 N.B. Babinski reflex: I am going to tickle the bottom of your foot:
• Planter flexion: normal response
• Big toe dorsiflexion and toes fanning: UMNL (e.g. stroke)

OSCE-guide-III.doc Page 127 of 255


Physical Examination

- Gait – ATAXIA
o Can you take few steps for me please?
o Protect the patient, surround him with your arms, and walk with him
- Romberg test
o Can you put your legs together!
o Can you close your eyes please!
o Watch (protectively) for few seconds!
 Ataxia due to peripheral neuropathy (B12 deficiency / DM / syphilis): ↑↑
with eyes closed
 Cerebellar ataxia: no ↑ with closed eyes (always on)
- Cerebellar signs (stroke / alcohol / tumours / para-neoplastic / …):
o Nystagmus:
 Can you follow my finger please (move it side to side)
• Physiological: transiently then corrected
• Central: horizontal or vertical
• Peripheral: horizontal only. Conditions: benign positional vertigo
/ acute labyrinthitis / drugs
o Finger to finger:
 Patient hand must be extended
 Move the examiner hand
 Check both upper limbs
o Finger to nose test: lesion in the cerebellum on the same side.
 Intentional tremors
 Loss of coordination
o Heel to shin: lesion in the cerebellum on the same side

- Cortical sensations: two points discrimination


- Mini-mental status exam

OSCE-guide-III.doc Page 128 of 255


Physical Examination

Cranial Nerves Examination

- Vital signs
- Comment on the patient general condition

CN I:
- Do you have problems with smells?
Can you please close your eyes?
- What is that? Coffee / ammonia
- What is that? Ammonia / coffee

CN II:
The optic nerve:
- Visual acuity: Do you wear glasses? reading / color (Snellen chart at 1 foot distance – 35
cm)
- Visual fields: eye by eye / by confrontation (when you see my fingers wiggling)
- Pupillary reflex: I am going to shine light in your eyes, please look straight to the wall,
each eye: direct and consensual (afferent: CN II, efferent: CN III)
- I would like to do fundoscopy examinations, looking for: disc edema, retinal hemorrhage,
neovascularisation, nipping of the veins

CN III, IV, VI:


- By inspection; both eyes are symmetrical, no deviation, no nystagmus, no head tilting, no
ptosis (CN III: opens, CN VII: closes)
- I would like you to follow this pen, without moving your head please, and if you see
things double or blurred at any time, please let me know  move the pen in large H-
shaped manner, then conversion
- Normal extra-ocular muscles movements, no nystagmus or double vision

CN V:
- Motor:
o By inspection: no atrophy of the temporal or masseter area
o Can you please clench, feel the temporalis and masseter
o Can you open your mouth against my hand?
- Sensory:
o This is a piece of cotton, and this is how it feels, I am going to touch your face,
and whenever you feel it, please tell me. Can you close your eyes please?
o Touch the face in symmetrical areas; cover the ophthalmic, maxillary, and
mandibular areas. Does it feel the same?
o Facial sensation of the trigeminal nerve is intact and equal on both sides
- Reflexes:
o Corneal reflex (afferent: CN V, efferent: CN VII)

OSCE-guide-III.doc Page 129 of 255


Physical Examination

CN VII: (mainly motor)


- Motor:
o Face is symmetrical, no deviation of the angle of the mouth, normal naso-labial
folds, no drooling
o Can you please copy me:
 Raise your eye brows, can you frown (wrinkle your forehead) please?
 Can you close your eyes, and do not let me open them?
 Can you blow your cheeks? Whistle?
 Can you show me your teeth?
o If there is a mouth angle deviation, it is deviated to opposite side of lesion
o UMNL vs. LMNL:
 UMNL: intact upper face muscles (the nucleus receives double sided
innervation)
 LMNL (Bell’s palsy)  ptosis; all face muscles are affected
- Sensory:
o To complete the facial nerve examination, I would like to check the sensations in
the outer 2/3 of the tongue
o I would like to check the ext auditory canal and the tympanic membrane to rule
out herpes zoster of the facial nerve
- Reflexes:
o Jaw reflex
o Corneal reflex (afferent: CN V, efferent: CN VII)

CN VIII
- Check by whispering (ABC – CBA), while rubbing fingers in front of the other ear OR
by rubbing your fingers
- Because the hearing is normal, I am going to skip Weber and Rinne tests
o Rinne: place the tuning fork in front of ear, then on the mastoid process
o Weber: place the tuning fork on the forehead

CN IX, X:
- Patient voice is normal, no hoarseness
- Can you swallow a sip of water please? Normal swallowing
- Can you open your mouth please? Soft palate is symmetrical, uvula is central
o Uvula deviates to the opposite side of the lesion
- To check the reflexes: I need to do the gag reflex

CN XI:
- Can you please shrug your shoulders?
- Turn your head to the right, and to the left. I am going to resist you. I feel for the opposite
side sterno-mastoid

CN XII:
- Can you please open your mouth? Can you stick your tongue out?
o Tongue is central, no deviation. No fasciculations or atrophy of tongue.
o If there is a lesion, the tongue deviates towards the lesion side
- Can you please move it to the right and to the left? Can you stick it against your cheeks?
 Normal movements of the tongue

OSCE-guide-III.doc Page 130 of 255


Physical Examination

Tremors

? Parkinson disease

Introduction Shaky right hand


Vital signs I would like to rule out any orthostatic hypotension
General comment Pt is sitting comfortably …, with Rt hand tremors

Inspection Tremors Right hand tremors, not obvious on the left hand
 Count from 10 to 1 please  tremors ↑ with mental activity 
consistent with Parkinson disease, and rules out anxiety
 Stretch your hand plz / no fine tremors  r/o hyperthyroidism
 No flapping tremors  rule out liver failure
 Finger to nose / no intentional tremors  r/o cerebellar dis
Patient tremors consistent with Parkinson disease, resting tremors,
beads rolling, and limited to Rt hand (…). No head nodding
Face (No) limited facial expression, decreased eye blinking, drooling
Palpation Check the wrist and elbows:
Rigidity  (No) cog wheeling Positive with parkinsonism
 (No) lead pipe rigidity
 (No) clasp knife spasticity Positive with stroke
Standing / walking  Would you please stand up! Do you need help  patient finds
Postural instability difficulty in standing up
 Can you walk few steps for me please: comment with + or -
- Stooped posture
- Shuffling (festinating) gait
- Decreased arm swinging
- Patient turns in blocks
Special Tests  Rapid alternating movements (hand supination & pronation /
oppose thumb to fingers)  dysdiadochokinesia
 Can you please repeat “British constitution”  monotonous
 Can you write a sentence for me  micrographia
 Can you draw a spiral parallel to this (draw spiral on paper)
I would like to do the mini-mental status exam

Treatment of Parkinsonism:
 Pharmacologic
▬ Mainstay of treatment: Sinemet (levodopa / carbidopa). Levodopa is a dopamine
precursor, carbidopa decreases peripheral conversion to dopamine
o Levodopa related fluctuation: delayed onset of response (affected by mealtime),
end-of-dose deterioration (i.e. “wearing-off”), random oscillations of on-off
symptoms
o Major complication of levodopa therapy is dyskinesias
▬ Treatment of early PD: DA agonists, amantadine, MAOI
▬ Adjuncts: DA agonists, MAOI, anticholinergics (especially if prominent tremors),
COMT inhibitors
 Surgical: thalamotomy, pallidotomy, deep brain stimulation (thalamic, pallidal, subthalamic),
embryonic dopaminergic stem cell transplantation

OSCE-guide-III.doc Page 131 of 255


Physical Examination

Thyroid Exam

Introduction
Vital signs BP, HR
General  Can you stretch your hands:
- Fine tremors
- Palms for sweating
- Nail changes
- Hair loss (hypothyroidism)
 Examine the eyes:
- Exophthalmos – stand by the patient (stand behind the right shoulder
and look from above)
- Lid lag (can you follow my finger without moving your head – from
above downwards)
 Proximal muscle weakness:
- Can you shrug your shoulders (bilaterally against my hand) please
 Knee reflex: brisk11 reflex
 Peritibial myxedema: indicates hyper-thyroidism
Thyroid Exam Patient is sitting on a chair
Inspection Can you swallow12 please?  no apparent thyroid enlargement
Palpation  Thyroid gland:
- From behind the patient, bi-manually
- Then while swallowing a sip of water  thyroid movement is normal,
I do not feel any masses, nodules, and no tenderness
 Lymph nodes:
- Sub-mandibular and cervical
Percussion  DIRECT percussion on upper part of sternum
 Checking for retro-sternal extension (no retro-sternal dullness)
Auscultation  BOTH lobes
 For thyroid bruits

11
Reflexes grades: 0 absent 1 hypo 2 normal 3 hyper (brisk) 4 hyper with clonus (ankle)
12
Whenever you ask the patient to swallow, give a sip of water, it is difficult to swallow on an empty mouth

OSCE-guide-III.doc Page 132 of 255


Physical Examination

Dermatomes

OSCE-guide-III.doc Page 133 of 255


Physical Examination

Neck Examination

Patient complaining of pain in the neck

Vitals  Can I get the vital signs please


General  Comment on patient general condition: patient is sitting comfortably …
Inspection  I would like to take a look at your neck; can you lower your gown please?
 From the back: SEADS (Swelling / Erythema / Atrophy / Deformity / Scars)
 From the side: normal cervical and thoracic curvatures
 Can you cough please, any pain
- No neck pain with Valsalva manoeuvre
Palpation  I would like to feel you back please
- Temperature is normal
- Tenderness: pressing on the spinal processes and para-vertebral muscles,
trapezius and sternomastoid
 Thyroid: can you swallow for me please?
 Check the LNs: no enlarged LNs
ROM  Can you touch your chest by your chin?
 Can you look to ceiling?
 Can you turn your head to the right? And to the left?
 Can you tilt your head? To the right and to the left?
- Normal flexion, extension, rotation and lateral flexion
Powers  Test again against resistance
- Neck pain is not associated with muscle contractures
Special tests  Spurling test: stretching the nerve  reproduces the pain
 Upper limb examination and neurological screening

Part of my exam is to check your upper extremities, can you roll up your sleeves please!
Inspection Upper extremities are symmetrical, normal bulk, no atrophy / SEADS
Palpation I am going to feel your shoulder; deltoid, biceps, triceps, forearm, thenar,
hypothenar  are symmetrical / no deformity / no atrophy
Motor Power Deltoid C5 Biceps C5/6 Triceps C7/8
Sensory C4: deltoid C5: biceps – lateral aspect Test light
Neurological

C6: thumb C7: middle finger C8: touch


little finger
T1: elbow – medial aspect
screen

Reflexes  Biceps and brachio-radialis C5/6


 Triceps C7/8
Pulse Radial pulse

Post encounter: what is the level of the lesion?


C6 nerve root lesion (C5-C6): weak biceps, weak biceps reflex
C7 nerve root lesion (C6-C7): weak triceps, weak triceps reflex
Diagnosis: Osteoarthritis of the cervical spine at … level
X-ray findings: osteophytes of the cervical vertebrae / narrowing of disc space / subchondral
sclerosis (increased bone formation around the joint), subchondral cyst formation
Management:
▬ Soft neck collar
▬ NSAIDs / acetaminophen
▬ Physiotherapy
▬ If worsening: neurosurgery consult / CT myelography / nerve conduction studies

OSCE-guide-III.doc Page 134 of 255


Physical Examination

Carpal Tunnel Syndrome


Hand pain for 6 months
History
Analysis CC  Os Cf D / PQRST / ↑↓
 Radiation: how about your elbow? Your fingers?
 Triggers: is it related to time? To work? What do you do?
AS  Weakness, numbness, tingling
 Swelling, redness, warm
 How about the other hand?
Impact  How did this affect your life? Your work?
Red flags  Constitutional symptoms
Differential  Hx of neck pain / injury / trauma
diagnosis  Hx of diabetes / symptoms
 Hx of thyroid disease / symptoms of hypothyroidism
 Hx of autoimmune disease / Rh arthritis / symptoms
 Headache / visual disturbances / shoes are getting tight
 Pregnancy / LMP
PMH / FH

Physical examination
Vital signs
Inspection  SEADS (thenar / hypothenar ms)
 No nail changes, no nodules / no deformity
Palpation  Temperature: is normal
 Tenderness: palpate distal radial bone, styloid process, joint line, styloid
process, distal ulnar bone, base of the thumb, carpal bones, metacarpal
bones, digits
ROM  Flexion / Extension /+/ move your hand to the right, to the left
 Can you make a fist / fan your fingers  there is no obvious damage to
the nerves / muscles / and tendons of the hand
 Thumb movements:
- Touch base of your little finger (thumb opposition)
- Move it all the way to opposite direction
- Point to the ceiling (with hand supine, flat)
- Touch the tips of your fingers
Power  Like ROM but against resistance
 Thumb 90°, DIP flexed: do not let me straighten it
 Biceps ROM / against resistance / biceps reflex (C6)
Sensory  Check with cotton tip,
 For the ring finger: check both sides: ulnar / radial
Special tests  Phalen's test,
 Tinel's sign / tap on the carpal tunnel
Investigations: EMG / nerve conduction studies
Treatment:
▬ Modify nature of work
▬ NSAIDs
▬ Wrist splint
▬ Local corticosteroids injection
▬ Surgical decompression

OSCE-guide-III.doc Page 135 of 255


Physical Examination

Hand Laceration / Wrist Laceration


Physical examination
Introduction Gloves / vital signs
Inspection  If big wrist laceration (BIG band aid): DO NOT remove the band aid,
should be removed under anesthesia
 If small laceration, remove the band aid  describe the laceration: linear /
size / position / elevated margins / active bleeding / oozing
 Other ulcers / other hand
 SEADS (Swelling / Erythema / Atrophy / Deformity / Scars)
 No nodules / no deformity
Palpation  Temperature: is normal, is hand pink and warm?
 Tenderness: palpate distal radial bone, styloid process, joint line, styloid
process, distal ulnar bone, base of the thumb, carpal bones, metacarpal
bones, digits
 Capillary refill
 Feel the pulse  radial ± ulnar
 Allen’s test
ROM  Do NOT ask patient to apply force against resistance as this may rupture
a partially severed tendon. Test active ROM only
 Median nerve: thumb abduction (thumb to ceiling)
 Ulnar nerve: finger spread (fanning)
 Radial nerve: wrist extension
 Tendon examination of the hand (flexor tendons):
- DIPs; flexor digitorum profundus: hold the MCP / PIP joints in
extension, and ask the pt to flex DIPs.
- PIPs; flexor digitorum superficialis: hold all fingers – except one you
are testing – in extension and ask pt to flex the remaining finger
- MCP joints: intrinsic (lumbricals)
Power  Do NOT ask patient to apply force against resistance as this may rupture
a partially severed tendon. Test active ROM only
Sensory  Check with cotton tip or pin-prick, for light touch
 Check two point discrimination on either side of each digit
 For the distribution of the three nerves: ulnar / radial / median
Neuro-motor examination of the hand
Median Ulnar Radial
Sensory Radial aspect of index finger Ulnar aspect of little finger pad Dorsal web space of thumb
pad
Motor Flex DIP if index finger (FDP) Flex DIP of little finger (FDP), Extend wrist and thumb,
extrinsic extensor carpi radialis (extensor pollicis longus)
Motor Thumb to ceiling with palm up Abduct index finger (first
intrinsic (abductor pollicis brevis) dorsal interosseous)

Structures lacerated
Diminished ulnar territory sensation Ulnar nerve
Allen test shows (no) refill from the ulnar circulation Ulnar artery
FDS weakness in little finger and ring finger Flexor retinaculum, ulnar two divisions of FDS

Management: clean and explore wound under local anesthesia and sterile conditions. Consult plastic
surgery for micro-vascular repair. If at night, may suture the skin and arrange for pt to be seen by plastic
surgeon next day.

OSCE-guide-III.doc Page 136 of 255


Physical Examination

Back Pain

Acute  Herniated disc Neurological exam


 Muscle spasm
Acute on top
of chronic
Chronic Pain at end of day shoots to legs, increases when leaning
forward:
 Disc herniation
 Degenerative disease (OA)
Pain related to position, increases when extension (or
leaning backwards), walking down the hill (arching the
back)  spinal canal stenosis
Pain wakes patient in morning, improves at end of day, Mechanical exam
stiff  Ankylosing spondylitis  limitation in all
directions

Red flags for back pain:


BACK PAIN
 B bladder or bowel dysfunction
 A anaesthesia (saddle)
 C constitutional symptoms  malignancy
 K chronic disease

 P parathesia
 A age > 50 years old
 I IV drug user
 N neuro-motor deficits

OSCE-guide-III.doc Page 137 of 255


Physical Examination

Acute Back Pain

Acute back pain: pain for 3 days


Acute on top of chronic: pain for 3 hours – old patient

Introduction How do you feel?


Do you prefer to remain standing, sit, or lie down?
That is ok, whenever you want to lie down or rest, feel free to do so
Analysis Os Cf D  What were you doing? What do you do for work?
 Did you hear any crack sound?
 Were you able to stand up and continue? Were you able to
move? Did anyone help you?
 Is this the first time?
 How about at rest? During night?
 Recently did you have any discomfort? Milder back pain?
How did it affect you?
PQRST  Does it shoot to your thighs? Toes?
 Which is bothering you more; your back or your legs?
↑↓  How about lying down? Stretch your back? Coughing?
Moving? Leaning forward or backward?
 Any medication? Did it help?
Impact  How does it affect you?
 Mets to lungs / liver / brain
Red flags  Constitutional symptoms
 Overweight
In addition to your pain, did you notice any other symptoms:
 Weakness, numbness, tingling
 Difficulty with balance, falls
 Any difficulty passing urine? How about bowel Cauda equine
movement? Did you find that you soiled yourself?
 Any numbness in the buttocks area?
 Do you have morning erection? Any sexual dysfunction?
DD
PMH Cancer prostate / bladder
FH Cancer prostate
SH

OSCE-guide-III.doc Page 138 of 255


Physical Examination

Chronic Back Pain


Pain for 3 months

Introduction Do you prefer to remain standing, sit, or lie down?


That is ok, whenever you want to lie down or rest, feel free to do so
Analysis Os Cf D  What were you doing? What do you do for work?
PQRST  Quality: stiffness
 Timing: is it worse in morning? Improves with time? Or is it worse
at the end of the day?
 Does it shoot to your thighs? Toes?
 Which is bothering you more; your back or your legs?
↑↓  How about lying down? Stretch your back? Coughing? Moving?
Leaning forward or backward?
 Any medication? Did it help?
AS  Pain other joints (knees / hips / hands) / distribution? Osteoarthritis
Sero-  Pain other joints (knees / hips / hands) / distribution? Ankylosing
negative  Eye pain, redness / mouth ulcers spondylitis
symptoms  Skin changes / nail changes / hx of psoriasis
 Repeated attacks of abd pain / diarrhea
 Urethral discharge
Impact  How does it affect you?
 Mets to lungs / liver / brain
Red flags  Constitutional symptoms
 Overweight
In addition to your pain, did you notice any other symptoms:
 Weakness, numbness, tingling
 Difficulty with balance, falls
 Any difficulty passing urine? How about bowel movement? Did you Cauda equine
find that you soiled yourself?
 Any numbness in the buttocks area?
 Do you have morning erection? Any sexual dysfunction?
DD
PMH  Trauma
 Injury to back
FH  Osteoarthritis
 Ankylosing spondylitis
 Other rheumatic disease
SH

Osteoarthritis: older patient / worse at evening

Ankylosing spondylitis:
▬ Morning stiffness improves by time
▬ LSS x-ray: sacroiliitis OR fusion of SI joints
▬ ESR: ↑
▬ HLA-B27 tissue antigen: positive
Associated symptoms: inflammatory arthritis / Uveitis / psoriasis / IBD / pericarditis / aortic regurgitation
Management:
▬ No cure
▬ Regular therapeutic exercises to prevent deformity (swimming / back extension exercises)
▬ NSAIDs: Indomethacin (50 mg PO bid) or Naproxen (250 mg PO bid)
▬ In severe cases: total joint replacement

OSCE-guide-III.doc Page 139 of 255


Physical Examination

Back Joint Examination

Physical examination
Introduction Can you stand up please?
Vital signs
Inspection  Gait / balance / stance
 Ask the patient to stand up from sitting position
 Posture: normal cervical, thoracic, lumbo-sacral curvatures
 Adam’s forward bend test (if scoliosis: the scapula will be higher)
- No scoliosis or kyphosis
 SEADS
Palpation  Temperature
 Tenderness: spinal processes, para-vertebral muscles, sacro-iliac joints
(medial to dimples of Venus)
ROM  Can you touch your toes with your fingers? Without bending knees
 Can you arch your back? Without bending knees (stand supported by the
bed foot: will not fall, less possibility of knee bending)
 Slide your arms on both sides (Rt and Lt)? (stand against wall, normally
the tips of finger travel > 10 cm)
 Cross your arms? Turn to the Rt and Lt (pt sitting on bed)
 Modified Schober's test: (midline, between the dimples of Venus) + 5
cm below + 10 cm above  bend forward  N> 6 cm diff.
Special tests  Occiput-to-wall distance (tragus & nose same level): normally zero
 Straight leg raise (irritation of the roots of sciatic n: L4/L5/S1/S2):
elevate the lower extremity straight, when it is painful  where it does
hurt?  straight leg test positive
 Decrease the angle, try to dorsiflex foot  Lasègue sign
 Cross straight leg raise test: elevate the other LL  trigger pain
 Faber’s test (figure 4 test): to check sacro-iliac joint pathology
 Femoral nerve stretch (done for patients c/o pain radiating to the anterior
aspect of the thigh): patient prone, knee flexed,
Motor  Hip flexion (L1/L2/L3) / extension (S1/S2)
 Knee flexion (L5/S1/S2) / extension (L2/L3/L4)
 Ankle dorsiflexion (L4/L5) / plantar flexion (S1/S2)
Neurological screen

 Can you walk on your heals? Normal L4/L5 muscles


 Can you walk on your toes? Normal S1/S2 muscles
Sensory S1: little toe L5: first web L4: medial malleolus
L3: knee – med L2: thigh – ant L1: groin
T10: umbilicus
Reflexes Knee (L2/L3/L4 – mainly L4) / Ankle (S1/S2 mainly S1) / I would like to do
the Babinski reflex (positive in UMNL)
Pulse Dorsalis pedis

Other clinical examinations: DRE; to rule-out cauda equina (sphincter weaknesses, reduced anal
tone)
N.B. dimples of Venus correspond to PSIS

OSCE-guide-III.doc Page 140 of 255


Physical Examination

Ankle Twist

Young man comes with ankle twist; history and physical examination are normal, no fractures,
and no lacerations. In the next 10 minutes counsel him about the treatment

History  Mechanism of trauma / injury


 Noise heard at the time of trauma
 Where you able to walk after the injury?
Inspection Symmetry / SEADS
Palpation  Temperature
 Tenderness: med malleolus / lat malleolus / ant joint line (between malleoli)
/ lat ligaments (slightly ant and inferior to the lat malleolus) / med ligaments
(inferior and slightly ant to the med malleolus) / tendon Achilles / navicular
head (medial bony prominence) / fifth meta-tarsal head (lat) / around the
meta-tarsal heads / origin of the plantar fascia (inf med aspect of calcaneus )
 Crepitus
ROM Patient is sitting on the side of the bed
 Dorsi-flexion (point toes up) and plantar flexion (point toes down)
 Inversion (point the bottoms of your feet towards each other) and eversion
(the opposite direction)
Power ROM against resistance / after stabilizing the ankle with your other hand
Special tests  Talar drawer sign: 1 cm is significant, indicates ant talo-fibular lig rupture
 Talar tilt: lat calcaneo-fibular lig rupture (inversion), med lig rupture
(eversion)
 Squeeze test (squeezing calf  ankle pain); if negative  NO ankle
fracture

Investigations: x-ray
 Ottawa ankle rules; for ankle series:
o Pain in the malleolar zone and any one of the following:
 An inability to bear weight both immediately and in the emergency
department for four steps
 Bone tenderness along the tip of the medial or lateral malleolus
 Ottawa foot rules; for foot series
o If there is any pain in the mid-foot zone and any one of the following:
 An inability to bear weight both immediately and in the emergency
department for four steps
 Bone tenderness at the base of the fifth metatarsal
 Bone tenderness at the navicular bone

Management:
 Complete tear should be evaluated by orthopedics  stat orthopedics consult
 RICE: rest (and crutches) / ice for 20 min QID x 3 days / compression (by tensor bandage) /
elevation
 Pain medication: NSAIDs; e.g. Ibuprofen 400 mg, PO, q6h.
 Show him how to wrap it, remove the wrap, and ask him to wrap it again (to make sure he
knows how to). Remember: from distal to proximal and 1/3 width overlap.
 Show him how to use the crutches.

OSCE-guide-III.doc Page 141 of 255


Physical Examination

Shoulder Joint
History - Trauma to shoulder / neck? X-ray done? What is your occupation?
- Neurological deficits? How does it affect your life?
Vital signs
General Patient condition (restlessness, discomfort, willingness to move)
Inspection - Both shoulders symmetrical / clavicle level / scapula level / deltoid
- SEADS (Swelling / Erythema / Atrophy / Deformity / Scars)
Palpation - Temperature: compare
- Tenderness: sternal notch / sterno-clavicular joint / clavicle / acromio-
clavicular joint / deltoid / long head of the biceps / insertion of the rotator
cuff muscles / spine of the scapula / medial border of scapula / spinal
processes of the cervical spine
- Crepitus
ROM - Active ROM: can you copy me please:
- Abduction  and comment on painful arc test
- Adduction  and comment on drop arm test
- Forward flexion (180°) /+/ Backward extension (60°)
- External rotation /+/ Internal rotation
- Another faster way to check:
- Hands behind your neck (abduction / ext rotation)
- Hands behind back (adduction / int rotation) between shoulder blades;
touch the tip of the contra-lateral scapula.
- Passive ROM: If patient is unable to complete the whole range of
movements actively, complete the ROM passively and comment (in
inflammation: passive ROM is > active ROM)
Power - Like the ROM, but against resistance
Special tests 1 Painful arc (between 60° and 120°) All these tests are done to
2 Drop arm test  complete tear of supra- test for subacromial
spinatous tendon impingement of supra-
3 Neer’s test spinatous
4 Hawkin’s test
5 Jobe’s test (empty can test)
6 Lift-off test: try to push my hand away from Sub-scapularis
your back
Yergason’s test; palm face up test: shake
7 For bicepital tendinitis
hands, try to let your palm face upwards, I
will resist you, and press on your shoulder
8 Speed’s test: supine, semi-flexed, do not let
me push your arm down
9 Stability testing: For joint stability
+ Push ant / post
+ Pull down  sulcus sign
10 Apprehension test (ant and post): for
dislocation
To complete my exam, I would like to do:
- Check the pulses of the upper limb (radial / ulnar / brachial)
- Brief neurological examination of the upper limb
- One joint above and one joint below examination (cervical spine / elbow)
- The other shoulder examination

OSCE-guide-III.doc Page 142 of 255


Physical Examination

Rotator cuff muscles:


Supra-spinatous Abduction
Infra-spinatous External rotation
Teres minor
Sub-scapularis Internal rotation Lift-off test

Impingement syndrome:
- The most common symptoms in impingement syndrome are pain, weakness and a loss of
movement at the affected shoulder
Treatment:
- Mild: RICE / NSAIDs / PT. Rest (cessation of painful activity), ice packs and NSAIDs
may be used for pain relief. Physiotherapy (PT) focused at maintaining range of
movement and avoiding shoulder stiffness.
- Moderate: therapeutic injections of corticosteroid and local anesthetic may be used for
persistent impingement syndrome
- Severe: surgery

Investigations:
- U/S

Possibilities:
- Normal shoulder exam
- Frozen shoulder stiff, with limited active and passive ROM (ttt:
physiotherapy, NSAIDs, steroids)
- Bicepital tendinitis +ve palm face up test / speed test
- Repeated ant dislocation positive apprehension test
- Rotator cuff tear
o Complete tear drop arm test (ttt: surgery)
o Partial tear pain with initiation of movement / +ve empty can test (ttt:
physiotherapy, NSAIDs, steroids, surgery)
- Rotator cuff tendinitis similar to partial tear / +ve impingement test
- Sub-deltoid bursitis

Elbow

ROM: flexion / extension /+/ pronation / supination

Tennis elbow (lateral epicondylitis)


 With the elbow fully extended, there are points of tenderness over the lateral epicondyle
(origin of the extensor carpi radialis brevis muscle).
 Cozen's test: pain with passive wrist flexion and resistive wrist extension.[
 X-rays are used to confirm and distinguish possibilities of existing causes of pain that are not
related to Tennis Elbow, such as fracture or arthritis.

Golfer's elbow (medial epicondylitis)


 The common tendinous sheath is inserted into the medial epicondyle of the humerus
 Treatment:
o NSAIDs: ibuprofen, naproxen or aspirin /+/ Heat or ice
o A counter-force brace or "elbow strap" to reduce strain at the elbow epicondyle, to
limit pain provocation and to protect against further damage.

OSCE-guide-III.doc Page 143 of 255


Physical Examination

Hip Joint

+ Middle-age male with septic arthritis


+ Elderly female with osteoarthritis

Vital signs
General -
Patient condition (restlessness, discomfort, willingness to move)
May I ask for full exposure please?
-
Inspection -
Hip joint is deeply seated joint, I am looking for the surroundings
-
SEADS (Swelling / Erythema / Atrophy / Deformity / Scars)
-
Scoliosis / kyphosis / pelvic tilt (level of both iliac crests)
Gait: no wide stance, shuffling, drop foot, or antalgic gait
-
Balance: Trendelenberg sign; standing on one leg (while the patient is putting his
-
arms on the examiner shoulders), the pelvis drops
Palpation - Temperature: compare
- Tenderness: ASIS, iliac crest, PSIS, sacro-iliac joint, greater trochanter of the
femur.
I would like to check symphysis pubis and inguinal ligament.
- Crepitus: over femoral head (lat to femoral art, below inguinal lig)
ROM Active ROM, each one followed immediately by passively stressing (increasing) the
ROM – while patient is lying
- Forward flexion (120°)
- Internal rotation (30°) /+/ External rotation (45°)
- Create space, stabilize the contra-lateral hip with your left hand: Abduction (45°)
/+/ Adduction (30°)
- Backward extension (while lying prone): stabilize the lower back by your left hand,
can you lift your thigh (20°)
Power - Resisted isometric testing (patient lying supine)
Special tests - Figure 4 test (Patrick or Faber’s test)  the leg of the examined side flexed and
externally rotated with the ankle resting on the patella of the contra-lateral leg. The
examiner applies counter-pressure at the opposite hemi-pelvis, and applies gentle
downward force on the knee. Post hip pain indicates sacro-iliac joint pathology,
while ant lat hip pain may suggest hip joint pathology
- Thomas test  put your hand under pt LSS, and try to max flex the contra-lateral
knee
- True leg length  from ASIS (anterior superior iliac spine) to medial malleolus –
on both sides
- Apparent leg test  from umbilicus to medial malleolus
To complete my exam, I would like to do:
- Check the pulses of the lower limb (dorsalis pedis / posterior tibial / popliteal)
- Brief neurological examination of the lower limb
- One joint above and one joint below examination (LSS / knee)
- The other hip examination

Septic arthritis:
Physical exam: fever / very painful joint / +ve trendlenberg test / restricted movements on all directions
DD: Septic arthritis / Osteoarthritis / Osteomyelitis
One diagnosis: septic arthritis / One diagnostic test: arthrocentesis (joint aspirate)
Management:
▬ IV antibiotics, empiric therapy, (based on age and risk factors; oxacillin [2 g IV q4h for 4 weeks], or
vancomycin [if suspecting MRSA; 20 mg/kg IV q8h, for 8 wks], combined with ceftriaxone for gram
–ve, if suspecting Gonococcal: ceftriaxone; IV for 2 wks then oral for 2 wks), adjust pending C&S
▬ For small joints: needle aspiration, serial if necessary until sterile
▬ For major joints such as knee, hip, or shoulder: urgent decompression and surgical drainage

OSCE-guide-III.doc Page 144 of 255


Physical Examination

Knee Joint
Vital signs, General
Inspection - Gait and stance: normal; no antalgic (painful) gait
- Bilateral joint exposure (quadriceps)
- SEADS (Swelling / Erythema / Atrophy / Deformity / Scars)
- No genu varum (bow legs) and no genu valgum (knock-knee) deformities
Palpation - Temperature: compare
- Extended knee: tenderness over patella /+/ Lateral movement of patella /+/
quadriceps muscle / quadriceps tendon / patellar ligament / Tibial tuberosity
/ popliteal fossa and popliteal artery
- Flexed knee: tibial plateau / bilateral joint lines /+/ Collateral ligaments /+/
Femoral condyles /+/ patellar crepitus
- Popliteal fossa /+/ Cuff muscles [slightly flexed knee]
Knee effusion:
- Fluid wave or bulge sign (or milking test): for small amount of effusion;
from below and med to upward and lat. Then immediately sweep hand
down the lateral aspect pushing the fluid back
- Fluid ballottement test: for moderate amount of effusion
- Patellar tap: for large amount of effusion
ROM Patient lying down:
- Flexion (130°) and extension (180°)
- Internal and external rotations: while knee is flexed 90°, point your toes in &
out please
- Patellar movement: medial and lateral
- Patellar compression test: tight your thigh please  rough or painful
movement: patello-femoral syndrome or osteoarthritis
Power - Flexion and extension, while the knee is flexed 90°
Special tests - Anterior drawer test  for anterior cruciate ligament tear
- Posterior drawer test  for posterior cruciate ligament tear
- Lachman test: hip / knee semi flexed (30°)  ACL tear
- Check for the medial / lateral collateral ligaments (stability of knee); while
flexed at 30°  no laxity nor pain
- McMurray’s test (for medial and lateral meniscus tears)  feel for
crepitus / patient feels pain
- For medial: maximally flexed knees, externally rotated foot  extend while
applying varus force (from inside outwards)
- For lateral: maximally flexed knee, internally rotated foot  extend while
applying valgus force (from outside inwards)
To complete my exam, I would like to do …
- Painful clicking  cruciate, meniscus ACL MCL
- Knee lock  torn meniscus - Knee giving way - Can not descend
- Instability  cruciate  - Inability to continue activity stairs

DD Osgood–Schlatter disease Chondromalacia patellae


Tenderness  Pain on tibial tuberosity (↑ by kneeling)  Pain on lateral movement of patella
Investigations X-ray (AP / LAT / skyline)
Treatment  Benign self-limited condition  Non-impact activities
 Continue activity as tolerated  NSAIDs
 NSAIDs  Physiotherapy
 Physiotherapy  Surgery for refractory cases

OSCE-guide-III.doc Page 145 of 255


Physical Examination

Obstetrics and Gynecology

OSCE-guide-III.doc Page 146 of 255


Physical Examination

History taking – OB-GYN

Introduction
CC
Analysis of CC Os Cf D
COCA ± Blood
↑ ↓
HPI Associated
symptoms
DD
M Menstrual
G Gynecological
O Obstetric
S Sexual
PMH
FH
SH

OB/GYN cases

History taking:
- Vaginal discharge
- Vaginal bleeding
- Amenorrhea
- Infertility

Counselling:
- OCPs
- HRT
- C-section (wants to have c-section or wants to have vag delivery after c-section)
- Abortion
- 22 years old pregnant  anti-natal counselling
- 39 years old found she is pregnant, counsel her
- 30 yrs old pregnant (36 wks), HTN/+++ ptn in urine  counsel for pre-eclampsia
- PAP smear; 16 years old wants to arrange for a PAP smear
- PAP smear: 38 year old had abnormal PAP smear

OSCE-guide-III.doc Page 147 of 255


Physical Examination

MGOS history questions:

Menstrual:
- When was your LMP? First day? Was your LMP similar to the previous ones?
- Are they regular or not? How often do you have periods?
- How long does it last? How many days?
- How about the amount? Is it large / small? How many pads/day? Any blood clots?
- Are your periods painful? [not painful  anovulatory (PCOS/infertility)]
- Any spotting / bleeding between periods?
- When was your first period? Was it regular? For how long it was not regular? Normal to be
irregular for up to 18-24 months.

Gynecological:
- Do you have history or were diagnosed with any gynecological disease (e.g. polyps)?
- Do you have history of pelvic surgery or instrumentation (e.g. D&C)?
- Do you use contraception? What method? Since when? When was the last time?
Screening:
- Have you ever had Pap smear before? When was the last time? Any reason (if long time)? What
was the result?
- (>40 yrs) have you had mammogram done before? When? (Is it painful doctor? Could be; we
need to apply pressure on the breast to get better image)
- (>65 yrs) have you had your bone mineral density (BMD) done? Any reason?

Obstetrical  GTPAL:
- Have you ever been pregnant before? Any abortions (termination)? Or miscarriages (spontaneous
abortion)?
- Number of babies you delivered? Any twins? Any children with congenital abnormalities?
- For each delivery: was it full term or pre-term? Vaginal or CS? Any complications like high blood
pressure / high blood sugar?
- Family history of: repeated abortions / CS / congenital anomalies / twins

Sexual history:
- With whom do you live?
- If (alone / with family): are you in any relationship? Are you sexually active? Have you ever been
sexually active?
- If with partner: how do you describe the relationship? Is it stable? Are you sexually active? Do
you practice safe sex, and by that I mean using condoms every time? For how long you have
been together? (> 6 months  stable). And before that, were you sexually active?
- When did you start sexual activity?
- How many partners have you had for the last 12 months? For the last month?
- What is your sexual preference? Men/ women/ both? What type of sexual activity?
- Have you screened or diagnosed before with STIs? HIV? Vaginal discharge?
- How about your partner? Any fever? Discharge? Burning sensation?
- Do you feel safe in this relation?

What if the male partner does not like condoms? Is it ok to consider it safe sex? Yes, provided
that:
- Scan the partner for STIs first
- Strict monogamy relation (no extra-marital affairs)
- Use alternative reliable contraception (e.g. OCPs)

OSCE-guide-III.doc Page 148 of 255


Physical Examination

History of pregnant lady – third trimester

Are you doing regular ante-natal follow-up visits?

NO Yes

Social issue

Deal with the social issue

Last visit history / pre-eclampsia  When was your last f/u visit?
 What was your BP? Was there any headache?
 Was there leg swelling? Weight gain?
Make sure the mother is stable  Any abdominal pain? Cramps?
 Vaginal bleeding? Discharge?
 Any gush of water?
Make sure the baby is stable  Is your baby kicking like before? > 6 in 2 hrs
U/S  Have you done your U/S? How many times? When
was the last time?
 Number of babies?
 Location of the placenta?
 Amount of fluids?

N.B. to make sure the mother and baby are stable: ABCDE
 Activity of the baby
 Bleeding
 Contractions / pain
 Dripping / Discharge
 EDD (expected date of delivery)

OSCE-guide-III.doc Page 149 of 255


Physical Examination

Vaginal Discharge
Teenager / 5 minutes case
CC How can I help you?!
Analysis of CC Os Cf D
COCA ± Blood / color / fishy odour?
- ↑ ↓
- Related to periods
- Related to sexual intercourse (bact vaginosis: ↑ discharge post-coitus)
M - LMP / regular / how often / similar to previous ones?
HPI AS - Any pain? With intercourse? Same system
DD - Itching? Redness? ? Candida
- Any blisters / warts / ulcers13?
- Inguinal swellings?
- Urine changes? Dysuria, frequency? Nearby systems
- Bowel movements changes? GIT symptoms
- Abdominal pain  OCD / PQRST / ↑↓
- ? PID  Adnexal tenderness / fever
- Dissemination to liver (pain Rt upper abd)
- Constitutional symptoms DD
- Sore throat? Mouth ulcers? Red eyes?
- Joint swelling/pain? Skin rash? Reiter’s
G - IUD
- PAP smear!
- History of STI / PID?
O
S Complete sexual history for both partners
PMH - Any medications? Recent use of antibiotics
- Allergies
- DM
FH / SH - How do you support yourself?
- HEAD SS / SAD
Conclusion: STI because of risky sexual behaviour
 Physical examination including pelvic, speculum exam / PAP smear / swabs for C&S including those
for Chlamydia & Gonorrhea / saline slide microscopy / KOH / Whiff test
 DD: Gonorrhea, Chlamydia, Candidiasis (whitish), Bacterial vaginosis (thin gray, clue cells),
Trichomonas (frothy yellowish / greenish discharge, motile organism).
 Treatment:
o Gonorrhea: Ceftriaxone 250 mg IM single dose
o Chlamydia: Azithromycin 1g orally single dose
o Candidiasis: Miconazole 200 mg vag supp, 1 vag supp od qhs x 3 d
o Bacterial vaginosis: Metronidazole 500 mg PO bid x 7 d
If pregnant: Amoxicillin 500 mg PO tid x 7 d
o Trichomonas: Metronidazole 500 mg PO bid x 7 d
 Follow up with in 4 weeks
 Her partner(s) to be notified and to come for treatment, ask about sexual health (fever, discharge)
 Advice regarding safe sex (condoms, multiple partners, STIs)
 Chlamydia and Gonorrhea are reportable diseases
 HIV testing and other STIs screening if high risk sexual behaviour
 Advise regarding PAP smear regularly, vaccination against HPV

13
Blisters: HSV (Herpes Simplex Virus) / warts: HPV (Human Papilloma Virus) / ulcers: syphilis

OSCE-guide-III.doc Page 150 of 255


Physical Examination

Vaginal Bleeding – Non-Pregnant / Not-Known Pregnant


Analysis of CC Timing: OS Cf D – When did it start to be continuous?
COCA / fresh blood vs. clots  if large amounts  impact  anemia /
dehydration symptoms (pallor, SOB, dizziness, fainting, heart racing)
- ↑ ↓
- Related to sexual intercourse (if yes: cervical*)
M - For the last few weeks, how do you distinguish between your regular
periods and the bleeding? Related to periods
- LMP / regular / how often / how much / similar to previous ones?
- First menstrual period? Regularity?
HPI AS - Any chance you are pregnant? How do you know for sure? Same
DD - Any nausea / vomiting / breast engorgement? Frequency? system
- Any pain? With intercourse?
- Itching? Redness?
- Any blisters / warts / ulcers?
- Grape like tissue (hydatiform mole)
- Urine changes? Urinary symptoms? Nearby
- Bowel movements changes? GIT symptoms systems
- Abdominal pain: OCD/PQRST (? PID / ectopic)
- Pelvic fullness / heaviness?
- Constitutional symptoms DD
- Bleeding disorders/ tendencies? Any bleeding elsewhere
(nose, gums, with stool, easy bruising)?
- Blood thinners? Aspirin?
- Thyroid problems? Symptoms?
G - Polyps, fibroids, endometriosis, gyn cancer?
- Hx pelvic surgeries? Instrumentations?
- Contraception history; OCPs / IUD / HRT / mammogram
- PAP smear! Was it normal?
O - Any previous pregnancies? Abortions? How many? Route
of delivery?
S - Number of partners / safe sex / when did you start activity?
- Hx of STIs
PMH - Breast cancer, mammogram (if > 40 years), HTN
FH - Gynecological cancer / Breast cancer
SH - How do you support yourself?
- SAD / HEAD SSS (if teenager)
Vaginal bleeding cases:
- Middle age / risky behaviour / old abnormal Pap smear  cervical cancer.
- A 48 years patient with vag bleeding and all symptoms will be negative  dysfunctional uterine
bleeding (DUB); intermittent / lose track of periods / no pain with periods.
- A 52 years pt / constipation / OCP/HRT / no pregnancies14  endometrial cancer / constitutional
- A 62 years pt with intermittent bleeding / small amount with secretions  atrophic vaginitis;
(menopausal symptoms / dyspareunia / itchy vulva)  Rule out cancer (endometrial biopsy) 
estrogen vaginal cream
- DD: Fibroid / Cervical polyp / hyper/hypo-thyroidism / Trauma / Coagulopathy / PCOS
Investigations: pregnancy test β-HCG / progesterone challenge test / hysteroscope / PAP / U/S /
endometrial biopsy / TSH

14
Cervical cancer for prostitutes (risky behaviour) and endometrial cancer for nuns (no pregnancies)

OSCE-guide-III.doc Page 151 of 255


Physical Examination

Vaginal Bleeding – Pregnant / Ante-Partum Hemorrhage


 Patient usually sleeping in a left lateral position with oxygen, very anxious, crying
 Reassure her & ask her to bear with you for a while to get to the bottom of her problem
 Ask about her feelings & empathize (impact on her and her partner)
Analysis of CC Timing: OS Cf D first time or happened before
COCA / fresh blood vs. clots  if large amounts  impact  anemia /
dehydration symptoms (pallor, SOB, dizziness, fainting, heart racing)
↑ ↓
M - LMP / regular / how often / how much / similar to previous ones?
HPI AS - How many weeks? date based on LMP Same
DD - How did you know that you are pregnant? Was it planned? system
- Did you have regular antenatal care? F/U visits? U/S?
- What is the result of U/S?
- Any pain / discomfort? (OCD / PQRST)? Contractions
- Water gush / grape like tissue (hydatiform mole)
- Fetal movement
- Urine changes? Urinary symptoms? Nearby
- Bowel movements changes? GIT symptoms systems
- Abdominal pain: OCD/PQRST (? PID / ectopic)
- Pelvic fullness / heaviness?
- Constitutional symptoms DD
- Bleeding disorders/ tendencies? Any bleeding elsewhere?
- Headache, hand or feet swelling
- Trauma, sexual intercourse
G - Polyps, fibroids, endometriosis, gyn cancer?
- Hx pelvic surgeries? Instrumentations?
- PAP smear! Was it normal?
O - GTPAL
- Any previous pregnancies? Abortions? How many? How
many weeks? Route of delivery?
S - Hx of STIs
PMH - Breast cancer, mammogram
- HTN, DM, kidney disease, blood gp & Rh
- Medication / allergies / hospitalization / surgeries / blood transfusion.
FH - Gynecological cancer / abortions
SH - How do you support yourself?
- SAD / HEAD SSS (if teenager)

Threatened abortion Separation of part of the placenta


Will do physical and obstetric examination. Vaginal exam only after U/S excludes placenta previa
- 50/50% chance to keep or loose the baby - Admit to the hospital
- Investigations: continuous vitals monitoring / CBC / INR / PTT / - Arrange for continuous fetal heart monitoring, U/S
fibrinogen / Rh status / blood grouping and cross matching / US / and biophysical profile
fetal monitoring - Management: O2, IV fluid, LLP, if fetus is still in
- Management: distress, arrange for C/S
- If she is improving / the fetus is not distressed: she will go home,
Placenta previa Placenta abruption
resume normal activity, come back if more blood or more pain
- Otherwise: bed rest / steroids / fetal monitoring / Rhogam / platelets - Painless - Painful (contractions)
Incomplete abortion: 3 findings on vag exam would confirm the diagnosis: (1) Cervix dilated, (2)
Ruptured membranes, (3) Product of conception passed

OSCE-guide-III.doc Page 152 of 255


Physical Examination

Abnormal Uterine Bleeding (AUB)

 Dysfunctional Uterine Bleeding (DUB):


─ Abnormal bleeding not attributable to organic (anatomic / systemic) disease. DUB is a
diagnosis of exclusion. Anovulatory AUB often used synonymously with DUB.
─ Causes:
o ↓ progesterone: luteal phase defect (estrogen-dependent DUB)
o PCOS
o Endocrinal (↓ TSH / ↑ prolactin)
o Stress, weight loss, exercise
o Liver and kidney disease
 Investigations for AUB:
• Beta-hCG
o CBC, serum ferritin
o Coagulation profile (esp. adolescent): rule out von Willebrand's disease
• TSH, free T4
• Prolactin if amenorrhea
o FSH, LH
o Day 21 (luteal phase) progesterone to confirm ovulation
o Serum androgens (especially free testosterone)
• Pelvic U/S: detect polyps, fibroids; measure endometrial thickness (postmenopausal)
• Sonohysterogram (SHG): very sensitive for intrauterine pathology (polyps, submucous
fibroids}
• Hysterosalpingography (HSG)
o Pap test
o Endometrial biopsy: women > 40 years are at higher risk of endometrial cancer
 Must do endometrial biopsy in all women presenting with postmenopausal bleeding
to exclude endometrial cancer
o D&C: not for treatment; diagnosis only (usually with hysteroscopy)

OSCE-guide-III.doc Page 153 of 255


Physical Examination

 Treatment of AUB:
─ Treat underlying disorders / if anatomic lesions and systemic disease have been ruled out,
consider dysfunctional uterine bleeding (DUB)
─ Medical:
o Mild DUB
 NSAIDs
 Anti-fibrinolytic (e.g. Cyklokapron) at time of menses
 Combined OCP
 Progestins (Provera) on first 10-14 days of each month if oligomenorrheic
 Mirena IUD
 Danazol (pseudo-menopause)
o Acute, severe DUB
 Replace fluid losses, consider admission
 Medical treatment:
─ (a) estrogen (Premarin) 25 mg IV q4h x 24h with Gravol 50 mg IV/PO q4h or
─ (b) Ovral15 1 tab PO q4h X 24h with Gravol 50 mg IV /PO q4h
Taper Ovral: 1 tab tid X 2d  bid X 2d  OD
 After (a) or (b), maintain patient on monophasic OCP for next several months or
consider alternative medical treatment
o Clomiphene citrate: consider in patients who are anovulatory and who wish to get
pregnant
─ Surgical:
o Endometrial ablation; consider pre-treatment with danazol or GnRH agonists
 If finished childbearing
 Repeat procedure may be required if symptom recurrence
o Hysterectomy: definitive treatment

Post-coital bleeding / middle age:


─ STIs
─ Cervical (Cervicitis / Cervical polyp / Cervical cancer / Ectropion; if OCPs)
─ Bleeding tendencies
─ Trauma
Management:
─ Vaginal and cervical swabs; C&S and gram stain
─ Polyp: Sonohysterogram
─ Biopsy
─ If ectropion: stop OCPs, cautery (silver nitrate)

Cancer uterus:
Risk factors:
─ Early menarche
─ Nulliparity
─ Weight gain
─ HRT / estrogen therapy
─ Unopposed estrogen
Management:
─ Endometrial biopsy; if positive
o Total abdominal hysterectomy and bilateral salpingo-oophorectomy
o Adjuvant chemotherapy

15
Ovral is progestin (levonorgestrel) and estrogen (ethinyl estradiol) combination OCP

OSCE-guide-III.doc Page 154 of 255


Physical Examination

Amenorrhea
CC  Did not have periods for 6 months?!
 Did you seek medical attention? Any recent changes?
Analysis of CC  During these 6 months; any irregular bleeding? Spotting?
M  When was your first period? What age? Was it regular? For how long it was
regular / not regular? How often? How much? LMP?
 When it was regular; was it painful? (painless  anovulatory)
 Did you use any contraception? When did you stop? Why?
HPI AS  Any chance you are pregnant? How do you know for sure?
DD  Any nausea / vomiting? Breast engorgement? Frequency?
 For how long have you been trying to get pregnant?
 Any previous pregnancies? Abortions?
 Constitutional symptoms?
 Are you under stress? Hypothalamus
 Excessive exercise?
 Any concerns about your weight? (anorexia)
 Any headache? Vomiting in morning? Visual changes? Pituitary
Difficulty seeing to sides? Milk secretions from breast?
 History of thyroid disease? Heat/ cold intolerance?
Bowel movements? Moist/ dry skin?
 Do you have excessive hair growth? Acne? Did you Ovarian
notice any weight changes? Hx of DM / thirsty /
frequency? Fm Hx of PCOS?
 Hx of chemotherapy? Radiotherapy? Hot flushes?
Vaginal dryness? Soreness?
 Any change in your voice? ↑ Muscle bulk?
G  Any repeated surgical procedures? D&C? Uterine
 Pelvic surgeries? Instrumentations?
 PAP smear!
O  Any previous pregnancies? Abortions?
S  Hx of STIs
PMH - Any medical conditions? Psychiatric illness?
- Any medications? Recent use of antibiotics
FH - Family hx of PCOS / infertility?
SH - How do you support yourself?
- SAD
 Investigations: β-HCG / progesterone challenge test / hormonal assay (estrogen / progesterone / FSH /
LH / prolactin / thyroid-TSH / serum testosterone; total and free) / US / CBC
 Ovarian causes of amenorrhea: PCOS /+/ Premature ovarian failure /+/ Androgen-producing tumours
 PCOS (poly-cystic ovarian syndrome):
─ History: previous pregnancy / contraception hx
─ To diagnose PCOS: must have 2 of 3 criteria: (1) oligomenorrhea / irregular menses for 6
months, (2) hyper-androgenism (hirsutism or ↑ blood level), (3) PCOS by US
─ Investigations: β-HCG / US / High LH:FSH ratio > 2:1 / Fasting blood sugar
─ Treatment:
o Lifestyle modification (↓ BMI, ↑ exercise) to ↓ peripheral estrone formation
o Metformin 500 mg PO tid
o Clomiphene citrate; if she wants to become pregnant
o Tranexamic add (Cyklokapron); for menorrhagia only
o OCPs; if she does not want to become pregnant
 Dysmenorrhea:
─ DD: PID, fibroid, endometriosis
─ Investigations: U/S to exclude other conditions
─ Treatment: NSAIDs (ibuprofen 400 mg tid), OCPs.

OSCE-guide-III.doc Page 155 of 255


Physical Examination

Infertility

Transitional statement before going in details with the history:


In order for a couple to achieve pregnancy, both partners should be capable of having children
and relatively healthy. For that reason, I am going to ask some questions about your health and
your partner health; some of these questions are personal, but it is important to ask. And I would
like to assure you that all the conversation is strictly confidential and I will not release any
information without your permission!
Introduction
CC
Analysis of CC For how long have you been trying to conceive?
HPI DD - Anovulation (irregular cycles / painless / no pre-menstrual syndrome)
- PCOS: Do you have excessive hair growth? Acne? Did you notice any
weight changes? Hx of DM / thirsty / frequency? Fm Hx of PCOS?
- Tubal occlusion: surgeries / STIs and PID / IUD
- Endometriosis: dysuria / dyspareunia / dyschezia / back pain
- History of chemotherapy and radiotherapy
Gynecology - Do you have hx or were diagnosed with any gyn disease (e.g. polyps)?
Hx - Did you use contraception? What? Since when? When was the last time?
- Have you ever had Pap smear before? When was the last time? Any
reason (if long time)? What was the result?
Obstetric Hx - Have you ever been pregnant before? Any abortions (termination)? Or
miscarriages (spontaneous abortion)?
- Family history of: repeated abortions / CS / congenital anomalies / twins
Coital history - For how long are you in this relation? For how long have you been
trying to achieve pregnancy?
- How frequent? Regularity?
- Are you aware of sexual cycle and ovulation (both you and your partner)
Partner - Was he sexually active before? Did he father kids from another partner
history before?
- Did he receive any chemo or radio therapy?
- Does he complain of any testicular problem; varicose veins,
inflammation?
- Does he complain of any penile discharge?
- Did he go through any investigations; e.g. semen analysis?
PMH - Allergies
- DM
FH Infertility
SH

Investigations:
─ Semen analysis
─ Ovulation documentation (mid-luteal phase progesterone; d 21-22 / US)
─ Tubal patency (HSG / laparoscopy)

When to start investigations:


─ If the woman is < 35 years  after 12 months of trying to conceive
─ If the woman is 35-40 yrs  after 6 months of trying to conceive
─ If the woman is > 40 years  start investigations after 1 months of trying to conceive

OSCE-guide-III.doc Page 156 of 255


Physical Examination

Counselling – pre-eclampsia
36 weeks pregnant lady comes for f/u visit, BP 160/110, +++ protein in urine, Manage.
Introduction Like the B12 results case
I will discuss results with you
Ethical challenge: travel permission
History Last visit history / pre-eclampsia
Make sure the mother and baby are stable
U/S
Obstetric history / Gynecological history
PMH / Social history
Counselling Explain what is pre-eclampsia
Serious concerns with pre-eclampsia
Management Hospitalize
If insisting to leave  sign a LAMA
Introduction
- Good afternoon Ms … I am Dr … I understand that your blood pressure was measured and urine
test was done, I have the results with me and I will discuss it with you. However, because this is
my first time to see you, I need to ask you some questions, to get a better understanding of your
health condition, is that ok with you?
- Is this you first time to have these checks during your pregnancy?
- Are you under regular follow-up?
o Yes  proceed to history
o No  any reason? My husband had a car accident! I am sorry to hear that; was he hurt?
Was anyone else hurt? When was that? It must be difficult, how did this affect your life?
Ethical challenge: travel permission
o Actually I am here to get a note.
- What type of notes?
o Travel note, I really need to travel.
- It looks like it is an important trip for you; usually pregnant ladies do not travel during this time of
pregnancy!
o It is a business trip that would save our financials.
- I see it is important for you, however, before we proceed, let me check your health condition first,
and I will start by asking you some questions:

History
Last visit history / pre-eclampsia
- When was your last f/u visit?
- What was your BP? Was there any headache?
- Was there leg swelling? Weight gain? Did they do urine test?
- How about before being pregnant? Any hx of high blood pressure?
Make sure the mother and baby are stable: ABCDE
- Activity of the baby, is your baby kicking like before?
- Bleeding
- Contractions / pain
- Dripping / Discharge
- EDD (expected date of delivery)
U/S
- Have you done your U/S? How many times? When was the last time?
- Number of babies?
- Location of the placenta?
- Amount of fluids?
Obstetric history: any pregnancy before / any similar conditions? Gynecological history
PMH: high blood pressure Social history: SAD / support / home environment

OSCE-guide-III.doc Page 157 of 255


Physical Examination

Counselling
Explain what is pre-eclampsia
- Your blood pressure is 160/110, which is high, and the urine test shows protein in large amount
(+++) which is not normal, the most likely diagnosis is a medical condition called “pre-eclampsia”
OR “pregnancy-induced hypertension”.
- I would like to ask more questions to see how it affected you!
o My dad had HTN, and lived with it, I am ok.
- These are different conditions; your dad had HTN, but you have “pregnancy-induced” HTN,
which is a serious condition, with very serious and may be fatal consequences.
o Have you had hx of headache?  OCD / PQRST (not detailed)
o Nausea / vomiting
o Change in your vision? Flashing lights? Flying objects?
o Any abdominal pain in your upper right part of your abdomen?
o Any bruises? Yellowish discoloration / itching / dark urine / pale stools?
o Any chest pain / heart racing / SOB?
o Any weakness / numbness?
o Any swelling in your body / face/eyes? Did you feel your shoes tight?
o Did you gain weight?
o Any changes in the urine? Frothy? Burning sensation?
- Based on all this, the most likely explanation for your increased is “pre-eclampsia”; and this is a
very serious condition, we need to admit you to the hospital to monitor you. Then, the obstetrician
will assess you and may consider delivering the baby now.
o But doctor, I need to travel, just 2 days and I will come back.
- I understand your concern about traveling, but we have a serious situation here.
- We do not know exactly why patients have pre-eclampsia. We believe it is imbalance of
hormones, or it might be related to placenta, however the only treatment is delivering the baby.
Serious concerns with pre-eclampsia
- What happens is that there is a narrowing of blood vessels, this leads to ↓ the amount of blood
reaching the baby, subsequently ↓ the amount of oxygen and nutrients. On the long term this will
lead to some injury and even damage to the baby AND the mother.
o This includes your heart and blood vessels, that is why you have ↑ BP,
o This includes your kidney, that is why you have +++ protein in urine,
o This includes your liver, that is why you may have abdominal pain,
o This includes your brain, that is why you have headache, visual changes,
o This includes your baby, that is why he is not kicking like before …
This is not because of your pregnancy; all of these are due to this condition.
- The concerns we have is that we can not predict the outcome, without the proper medical care,
patients having pre-eclampsia will end up going to the next stage which is “eclampsia”; do you
any idea what is “e0clampsia”?
- A condition in which, the patient will start to seize, lose conscious, will not be able to breath and
turn blue. The only resolution for this is delivering the baby.
- Imagine that I give you the note, and they allow you to take the trip, 2 hours later while you are in
the plane, you start to fall down and seize. What will happen? Nobody will be able to help you.
- By this you endanger your life and your baby’s life.
Management
- What we need now is to admit you to the hospital and arrange for obstetrical assessment.
- If insisting to leave  sign a LAMA (leaving against medical advice)
- Suggest solution for her business travel, like giving a doctor note that she needs to be hospitalized.
Treatment Plan
- Assess severity including good history and physical exam focusing on heart, lungs, reflexes, fetus,
urine analysis and BW (important CBC, liver function tests, Uric Acid)
- If all above are stable, consider daily check, urine dips and fetal kick counts as outpatient. If any
of above unstable may need to hospitalize as inpatient for close monitoring
- Measure L/S ratio of the baby, give corticosteroids for lung maturation
- MgSO4 and delivery
- Blood pressure controlled often with labetalol, Ca channel blockers

OSCE-guide-III.doc Page 158 of 255


Physical Examination

Caesarean Section – Counselling – wants to have CS

Young 18-20 years old pregnant lady would like to have CS, counsel her.
Introduction
Any reason you want to have CS? Social issue
History Last visit history / pre-eclampsia
Make sure the mother and baby are stable
U/S
Obstetric history / Gynecological history
PMH / Social history
Counselling Address patient concerns
Why not caesarean section?
Management Refer to obstetrician
Spend some time to think / stabilize

Introduction
Any reason you want to have CS?
- I understand that you are here to discuss the possibility of CS; we will discuss this in details, but
before that I would like to ask you is there any reason you would like to have CS?
o I do not want to have this severe pain!
- How do you know it is painful?
o I had previous abortion OR
o I attended my sister delivery and it was very painful experience
- When was that? Did you attend?
o Congratulations! How is your sister doing? How is the baby?
o I understand that you saw her in pain, but people differ! And within few minutes I will be
explaining different options to control labour pains!
History
- Let me ask you some questions to assess the condition first!
o How do you feel? How is your mood?
o How about this pregnancy, was it planned? How do feel about your pregnancy? How is
the feeling of your partner?
o Are you under regular follow-up? NO! Any reason?
There may be social issue here.
 Empathy: it looks like you are doing through difficult times! How are you coping?
 Offer social support: being pregnant lady without support, you have priority and
there are a lot support and resources in the community. I will make sure to connect
you with social worker who will help you with proper support (housing / financially /
for both of you and the baby)

Last visit history / pre-eclampsia


- When was your last f/u visit?
- What was your BP? Was there any headache?
- Was there leg swelling? Weight gain? Did they do urine test?
- How about before being pregnant? Any hx of high blood pressure?
- Did you have your blood sugar checked? How about before pregnancy?

Make sure the mother and baby are stable: ABCDE


- Activity of the baby, is your baby kicking like before?
- Bleeding
- Contractions / pain
- Dripping / Discharge
- EDD (expected date of delivery)

OSCE-guide-III.doc Page 159 of 255


Physical Examination

U/S
- Have you done your U/S? How many times? When was the last time?
- Number of babies?
- Position of the placenta?
- Amount of fluids?

Obstetric history: GTPAL


- Any pregnancy before? Any abortions or miscarriages?
- What were the circumstances? How many weeks?
- How did you feel about it? How did you cope with that?
Gynecological history: Fibroid, Genital herpes

PMH: Medications / allergy / blood transfusion


Social history: SAD / support / home environment

Counselling

Address patient concerns


- I know that you are here to talk about CS. But first let me explain some facts about delivery. The
natural route for delivery is the vaginal delivery, and if there is no real indication for CS, we
would prefer to go for vaginal delivery.
- However, I appreciate you concerns, if your concern is the pain there is a lot of options to control
it.
o We can start by learning some relaxation techniques
o And then on the delivery day, when you go there, there are a lot of support groups; one of
them is called “doolas”; they attend with you and they provide a lot of emotional support
o Finally, when you start your labour, we have good measures; and I mean what we call
“epidural anesthesia”; this is very effective and safe. Where the anesthesiologist puts a
needle into your back and injects a freezing substance that helps you to go through the
delivery without pain (this is like the dentist freezes your mouth before doing painful
procedures). It might however cause headache, bleeding, and less likely infection.

Why not caesarean section


- I would like to ask you; what is your understanding of CS?
- It is commonly used obstetrical intervention, used when there is a problem or contraindication for
vaginal delivery and if there is an emergency situation that necessitates immediate delivery; and in
these cases it is life saving; for both the mother and the baby!
- However, it is a major surgery, has the risks of bleeding, higher risk of infection, DVT, you stay
longer in the hospital, and it will leave scar in your abdomen.

Management
- After all, I am not the person who makes the decision; this should be decided by the obstetrician.
- I am going to refer you to the obstetrician; who will perform further and detailed assessment then
discuss the results with you.
- Meanwhile, I would recommend you spend some time to think about what I told you, try to
stabilize yourself emotionally.

- I will give you some brochures and web sites so that you can read more about that.
- I will connect you with the social worker.
- And if at any time you have any questions or concerns, you can come to see me.

OSCE-guide-III.doc Page 160 of 255


Physical Examination

Caesarean Section – Counselling – does not want to have CS


Middle aged pregnant lady (36 weeks) is here to have her file. Three years ago, she had urgent CS
for cord prolapse. Now she would like to deliver at home with the midwife.
Introduction
Any reason you want to have your file? Concern
Deal with the patient concern
What was the type of your CS?
History Last visit history / pre-eclampsia
Make sure the mother and baby are stable
U/S
Obstetric history / Gynecological history
PMH / Social history
Counselling What is CS? The two types of CS
Risks of vaginal delivery post CS
Management Prepare a copy of the file
Speak with your midwife
Introduction
Any reason you want to have your file?
- I understand that you are here to have a copy of your file for the urgent CS you had 3 years ago.
This is your right, and I will ask someone to prepare a copy for you.
- But first I would like to ask you some question, is it ok with you!
- Is there any reason you would like to have your file?
o Yes, I would like to have delivery at home this time. A lot of my friends did it at home
with the midwife and they say it is much easier and relax.
- Are you seeing obstetrician? Are you doing any regular follow-up visits? Any reason for that?
o No, I am going to follow up with the midwife.
o I did not like last time when they did CS at the hospital!
- What happened last time?
o They told me the baby had cord prolapse!
- Were there any consequences?
o How is the baby doing? How old is he? What can he do? Walk? Talk?
o How about you? Any complications? Infections? Scars?
Deal with the patient concern:
- What is your understanding of cord prolapse?
- It is a condition where the umbilical cord goes into the birth canal before the head, and then gets
stuck and squeezed by the head.
- This is a very serious condition. The cord delivers blood and nutrients to the baby. If blocked for
long time, the baby will suffer from brain damage.
- That is why they had to do urgent CS, which was a life saving procedure for the baby, and it had
to be done immediately.
- Do you know what the type of your CS was? Classical!
History
- I understand your point of view, but first let me ask you some questions about your health and
your pregnancy!
Last visit history / pre-eclampsia
- When was your last f/u visit?
- What was your BP? Was there any headache?
- Was there leg swelling? Weight gain? Did they do urine test?
- How about before being pregnant? Any hx of high blood pressure?

Make sure the mother and baby are stable: ABCDE


- Activity of the baby, is your baby kicking like before?

OSCE-guide-III.doc Page 161 of 255


Physical Examination

- Bleeding
- Contractions / pain
- Dripping / Discharge
- EDD (expected date of delivery)
U/S
- Have you done your U/S? How many times? When was the last time?
- Number of babies?
- Location of the placenta?
- Amount of fluids?
Obstetric history: GTPAL
- Other than the pregnancy that you had CS 3 years ago; any pregnancy before? Any abortions or
miscarriages?
- What were the circumstances? How many weeks?
- How did you feel about it? How did you cope with that?
Gynecological history
PMH: Medications / allergy / blood transfusion
Social history: SAD / support / home environment

Counselling
What is CS? The two types of CS
- I would like to ask you; what is your understanding of CS? …
- It is commonly used obstetrical intervention, used when there is a problem or contraindication for
vaginal delivery and if there is an emergency situation that necessitates immediate delivery; and in
these cases it is life saving; for both the mother and the baby!
- There are two types of CS:
o The transverse (done at the lower segment of uterus); it is the most common type; its
advantages include: smaller scar and better healing.
o The classical or vertical type; it is done less common; as we cut through the muscle fibers
of the uterus it produces weaker scar; but it is indicated and actually needed in urgent
case, like yours. As it allows quick access and fast delivery, because in some cases (like
cord prolapse) we can not afford even few minutes more.
Risks of vaginal delivery post CS
- Due to the scar formed after the CS procedure; it is always recommended to deliver by CS, to
avoid the tearing pressure of the uterine contractions during vaginal delivery.
- If you decide to go for vaginal delivery, my concern is that the scar might undergo severe tearing
pressure and might rupture, which will lead to massive bleeding. This is an obstetrical emergency
that necessitates immediate intervention. Because you may end up losing your life and/or losing
your baby.
- I do not want to scare you, but the risks of having uterine rupture after classical CS is 12%, of
which 10% of cases end up losing their lives.
- For that reason: once classical CS, it is always CS.
In case of counselling transverse CS:
- Risks of having uterine rupture after transverse CS is 1%.
- Even though, if you want to try vaginal delivery, we can not take the risk to try this at home, we
can try this in the hospital, so that just in case any emergency might happen, we can intervene in
the proper time.

Management
- I will ask someone to prepare a copy of your file
- Speak with your midwife:
- I am sure that your midwife is highly trained and qualified, and we share the same guidelines. I
would recommend that you take your file and speak with your midwife, and I am sure she will
explain the situation to you.
- I will give you some brochures and web sites so that you can read more about that.
- And if at any time you have any questions or concerns, you can come to see me.

OSCE-guide-III.doc Page 162 of 255


Physical Examination

OCPs / Contraception Counselling


XX is a 16 years old girl. She would like to get information about OCPs. For the next 10 minutes
talk to her

Introduction
Concerns ─ Do you have any concerns?
─ Why do you need / think about contraception at this point?
─ Have you used contraception before? What is your previous
experience? Why did you stop it?
History Exclude ─ Any chance you are pregnant? How do you know for sure?
pregnancy ─ Any nausea / vomiting? Breast engorgement? Frequency?
M ─ Painful periods? / Irregular? / Heavy bleeding?
G ─ Last PAP / any abnormal PAP
─ Previous D&C
O ─ GTPAL / IUD is not recommended in nullipara
S ─ DETAILED
─ If risky behaviour: OCPs will not protect against STIs
─ Risky behaviour or previous STI/PID: NO IUD

Available Definition: birth control is an umbrella term for several techniques and methods used to prevent
methods fertilization
─ Hormonal (OCPs / implants / injections)
─ IUD (contragestion: prevents the implant)
─ Barrier methods (condoms / diaphragms) ± spermicidal
─ Behavioural (fertility awareness/timing) / coitus interruptus
─ Post-coital contraception
─ Sterilization (male / female)
OCPs IUD
Mechanism of ─ OCPs are hormones: estrogen and ─ Mechanically prevents the implantation of
action progesterone the fertilized ovum
─ OCPs prevent ovulation, increase thickness ─ Available forms: Copper / hormone-
of cervical secretion coated
─ 21 tab + 7 sugar pills  28 days
Contraindications ─ HTN / CAD / Cerebro-vascular disease / ─ Structural uterine anomalies
DVT ─ History of ectopic pregnancy
─ Breast or gynecological cancer (past
personal and family hx) ─ Undiagnosed vaginal Bleeding
─ Undiagnosed vaginal Bleeding ─ History of PID(s)
─ Active liver disease ─ Risky behaviour
─ Smoker > 35 yrs
─ Migraine
Benefits ─ Regulate periods ─ Longevity
─ Independence to coitus or compliance
ABCD:
─ Improves anemia (↓ bleeding)
─ ↓ benign breast lesions
─ ↓ ovarian cysts and cancer
─ ↓ risk of uterine cancer
─ ↓ dysmenorrhea

OSCE-guide-III.doc Page 163 of 255


Physical Examination

Side effects ─ Does not prevent STIs ─ Heavy periods


─ ↑ risk of DVT / CAD ─ If hormonal coated: prog side effects:
TRANSIENT: headache / wt gain / mastalgia
─ Breast tenderness
─ Weight gain
─ Headache
─ Nausea / vomiting
Failure rate ─ 0.1% – 5% ─ 0.5% – 2%
OCPs IUD
Plan ─ Will do physical and pelvic examination, and document blood pressure, pap smear
─ Brochures and information
─ Follow up within 1 month
─ How we will start:
─ First day of next menstrual cycle ─
─ Put package in obvious place that you see
frequently
─ Take the pill in the same approximate time
every day
─ Use another method of contraception for the
first TWO months
─ If you miss a pill: see below

NOTES

 N.B.: if woman with risky behaviour:


- Counsel for safe sex
- OCPs do not protect against STIs

 Failure rates are very high with behavioural methods:


- Because it depends largely on the knowledge, experience of user and the usage technique;
perfect-use versus typical-use failure rates.

 What happens if I missed 1 or 2 pills (OCPs)?


- 1 missed pill  take 2 pills the next day
- 2 missed pills  take 2 pills a day, for the next 2 days
- If missed more than 2 consecutive pills: use a backup method of contraception
simultaneous to finishing up packet of pills (2 pills a day) until next menses

 Woman comes to request sterilization (tubal ligation)


1- Discuss various other alternatives, make sure she knows everything about all of them
(OCPs, IUDs, diaphragm, condoms, vasectomy)
2- Counsel the patient on the permanent nature of the procedure, the operative risks, and
the chance of failure (1 in 200)
Counsel about the risks of regret for the decision (young age, recent emotional trauma,
family coercion)
3- Bring the patient back after 1 month for the bilateral tubal ligation procedure,
preferably immediately after menstruation to decrease possibility of pregnancy

OSCE-guide-III.doc Page 164 of 255


Physical Examination

Advantages Contraindications Side effects Available Failure rate


OCPs - Regulate periods - Pregnancy - Does not prevent STIs Combined pills (E+P): 0.1% – 5%
- Un-dx vag bleeding - ↑ risk of DVT / CAD low dose estrogen (20,
ABCD: - Cerebro-vascular dis / CAD 35, 50)
- Improves anemia (↓ - Active liver disease TRANSIENT:
bleeding) - Hormone-dep cancer - Breast tenderness
- ↓ benign breast lesions - Smoker > 35 yrs - Weight gain
- ↓ ovarian cysts and cancer - Hx of DVT / PE - Headache
- ↓ risk of uterine cancer - Migraine - Nausea / vomiting
- ↓ dysmenorrhea
IUD - Longevity - Pregnancy - Heavy periods - Copper 0.6% – 2%
- Independence to coitus or - Un-dx vag bleeding - If hormonal coated: prog - Hormone coated
compliance - Structural uterine anomalies side effects: headache /
- Hx of PID(s) wt gain / mastalgia
- Risky behaviour
- Hx of ectopic pregnancy

IM - ↓ Dysmenorrhea - NO absolute contraindications - Irregular bleeding Every 3 months < 1%


Injection - Effective - Amenorrhea after 1 year
- Independence - Delayed post-use
Depo- fertility
provera
SC implants - Longevity - Un-dx vag bleeding - Irregular bleeding Every 5 years < 1%
- Effective - Acute liver disease - Headaches
Nor-plant - Independence to coitus or - Thrombophlebitis - Mood changes
compliance - Thromboembolic disease

Condoms 14%
Diaphragm 20%
Behavioural - High failure rates up to 25%
- To decrease the failure rate, can combine 2 methods

OSCE-guide-III.doc Page 165 of 255


OB-GYN

HRT counselling

Introduction / overview
History General  How do you describe your general health?
 Support
M DETAILED
G D&C / OCPs / PAP / mammogram / BMD
O GTPAL
S Dyspareunia
Menopausal  Irregular menstrual cycles
symptoms  Vasomotor symptoms: sweating / hot flashes (hot flushes) /
palpitations
 Uro-genital symptoms: vaginal dryness / soreness /
superficial dyspareunia / urinary frequency and urgency
 Neurologic symptoms: mood changes / sleep disturbance /
depression / anxiety
Risk factors for Breast Cancer Uterine Cancer Ovarian Cancer
CANCER  Early menarche  Obese
 Late menopause  Diabetic
 OCPs  Nullipara
 No breast feeding  PCOS
 Age
 Past medical history of cancer or biopsy
 Family history
Risk factors for  Screen the first major risk factors for osteoporosis (see
Osteoporosis osteoporosis counselling case)
Counselling What is your understanding about HRT?
Common forms / products  Local preparations: creams / pessaries / rings
of therapy  Systemic formulations: oral drugs / trans-dermal patches and
gels / implants

- Estrogen alone
- Combined estrogen and progestogen
- Selective estrogen receptor modulator (SERM)
Indications for HT  Menopausal symptoms For SHORT term only, 1 – 2 years
 Osteoporosis
Contraindications to HT
Pre-treatment evaluation Hx / PE / baseline investigations
Adverse effects and risks
Any concerns? Media spoke that HRT increases incidence of stroke, heart attacks
and breast cancer, this was done by the (Women's Health
Initiative), on the other hand, smoking, obesity, cholesterol
increases the risk of these dis much more than HRT. In your case,
you do not have the risk factors for cancer, and it will be
beneficial for your hot flashes, vaginal dryness, and will protect
you against osteoporosis … In medicine we always weight risk /
benefits …

OSCE-guide-III.doc Page 166 of 255


OB-GYN

Introduction / overview:
- The reproductive years of a woman’s life are regulated by production of the hormones
estrogen and progesterone by the ovaries. Estrogen regulates a woman's monthly
menstrual cycle and secondary sexual characteristics (e.g. breast development and
function). In addition, it prepares the body for fertilization and reproduction.
Progesterone concentrations rise in a cyclical fashion to prepare the uterus for possible
pregnancy and to prepare the breasts for lactation.
- Toward the end of her reproductive years when a woman reaches menopause, circulating
levels of estrogen and progesterone decrease because of reduced synthesis in the ovary,
which may lead to several symptoms, the severity of which can vary widely.
- Hormone therapy (HT) involves the administration of synthetic estrogen and
progestogen. HT is designed to replace a woman's depleting hormone levels and thus
alleviate her symptoms of menopause. However, HT has been linked to various risks, and
debate regarding its risk-benefit ratio continues

Contra-indications of HRT:
No absolute contraindications of hormone therapy have been established. However, HT is
relatively contraindicated in certain clinical situations (similar to OCPs):
- Breast and/or endometrial cancer
- Undiagnosed vaginal bleeding
- Acute liver disease
- Thromboembolic disorders / DVT
- Endometriosis / Fibroids
- Diabetes, HTN, Heart disease

Required baseline investigations


- CBC
- Urinalysis
- Blood sugar levels
- Fasting lipid profile
- Electrocardiography
- Pap test
- Ultrasonography to measure endometrial thickness and ovarian volume
- Mammography

Possible adverse effects are as follows (similar to OCPs):


- Nausea / bloating
- Fluid retention
- Weight gain (equivocal finding)
- Mood swings (associated with use of relatively androgenic progestogens)
- Breakthrough bleeding
- Breast tenderness
- HT may slightly increase the risk for breast cancer
- There is association between HT and uterine hyperplasia and cancer
- There is increased risk of thromboembolism with HT

OSCE-guide-III.doc Page 167 of 255


OB-GYN

Needle Stick Counselling – HIV

You are about to see Mrs … 33 years old female nurse, upset because she had just had needle
stick after she gave an IV injection to a patient. Counsel her.

Introduction
Concerns HIV infection / fatal disease / will impact her family
History - Can you tell me what happened?
- Complete immunization record, including tetanus and hepatitis B
- Previous occupational exposure to body fluids
- Intravenous drug abuse
- Sexual history
Inform the - What is HIV? Major pathogens of concern!
patient about First of all; let me tell you the transmission rates: (no accurate studies)
HIV - Risk of blood transmission is: 0.3% for percutaneous exposure
- Risk of female to male transmission is: 0.03%
- Are you pregnant? Risk of intrauterine tx is: 3% with treatment and 30%
without treatment
- Advancement of HIV treatment
Address pregnancy concerns:
- Patient should receive ttt (not teratogenic)
- HIV positive mothers should not breastfeed their babies
Plan - I will speak with the patient, explain the whole situation and ask him to
consent for HIV status
- If he agrees; we will know possibility of tx to you. If he is HIV negative,
NO post-exposure prophylaxis is needed
- If he refuses or if he is HIV +ve; we will have to assess what is called
“exposure code” and match it with “HIV status code”; to simplify this,
guidelines state we should assume you were exposed and give
prophylaxis treatment: 4 weeks of 2 anti-virals (the basic regimen)
- Hepatitis B vaccination 3 doses + immunoglobulins (immediately)
Workup - Blood tests for the patient if possible and for the exposed
- I am going to speak with the patient now, and I will come back to you
with his decision.
- Any other questions or concerns?

What is HIV?
- Human immunodeficiency virus (HIV) is a blood-borne, sexually transmissible virus.
The virus is typically transmitted via sexual intercourse, shared intravenous drug
paraphernalia, and mother-to-child transmission (MTCT), which can occur during the
birth process or during breastfeeding.
- The major pathogens of concern in occupational body fluid exposure are HIV, hepatitis
A, hepatitis B, hepatitis C, and hepatitis D. These pathogens are viruses that require
percutaneous or mucosal introduction for infectivity. The major target organs are the
immune system (HIV) and the liver (hepatitis).

OSCE-guide-III.doc Page 168 of 255


OB-GYN

Advancement of HIV treatment:


- It used to be fatal
- Currently, it is not cured, but controllable, may be in the future they can develop a cure to
it
- A lot of anti-virals were developed since 1990s till now, with ↑ efficacy and ↓ side
effects, we started by giving many pills q4h, now it is once or twice a day
- We follow the guidelines with monitoring of what is called “viral load” and “cell counts”
of the patient immunity cells. To decide when to start treatment

Workup:
- Source patient (if available)
o HIV
o Hepatitis B antigen
o Hepatitis C antibody
- Victim/health care worker
o HIV; testing now, at 1 month, and at 3-6 months
o Hepatitis B surface antibody / titre (if vaccinated)
o Hepatitis C antibody; testing now and after 4 and 8 weeks
- Prior to initiating retrovirals:
o Pregnancy test (stat) – if she is not pregnant
o CBC count with differential and platelets
o Serum creatinine/BUN levels
o Urinalysis with microscopic analysis
o AST/ALT levels / Alkaline phosphatase level
o Total bilirubin level

The ethical questions that might arise:


- We can solve all this by calling the lab and adding HIV status check for the list of blood
works of the patient, we had just send his blood to the lab!
o You are right, this will save us the whole prophylaxis plan, however, we need to
speak to the patient first; we can not do this HIV status test with the patient
knowledge and consent.
o What I am going to do –to help you– is that after we finish, I will go to speak
with the patient, explain the whole situation to him, and ask his permission that
we do this HIV statue test.
- Even if the patient refused, we can ask Dr …, his surgeon, may be he knows!
o May be Dr … knows the patient status or may not, but if the patient refuses to let
us know, it is patient confidentiality, we can not ask Dr … about this information
unless the patient consents we can ask the surgeon.

OSCE-guide-III.doc Page 169 of 255


OB-GYN

Counselling – PAP smear

CC I understand you are here because you have some inquiries/worries about your
last PAP test, is this right? How can I help you today?
HPI M
G  Previous Pap test? How many? How frequent? Any abnormal Pap test?
Any previous colposcopy?
 Contraceptive history
O GTPAL
S  RISK factors for cervical dysplasia:
- Early age of sexual activity
- Risky behaviour: unprotected sex / multiple partners
- Smoking
AS Same system - Any pain? With intercourse?
- Discharge? Itching? Redness?
- Any blisters / warts / ulcers?
- Inguinal swellings?
Nearby systems - Urine changes? Dysuria, frequency?
- Bowel movements changes? GIT symptoms
- Abdominal pain  OCD / PQRST / ↑↓
General - Constitutional symptoms
PMH Any allergy / medication / disease
FH Gynecological tumours
SH If teenager: HEAD SSS
COUNSELLING:
- What do you know about (LGSIL)? What would you like to know?
- Have you had any experience with … in the past?
- Have you [read / talked to someone / searched the internet] about this issue?
Worried about PAP results
- PAP smear or test is done to screen for any changes that might happen in the cervix,
before it turns to serious disease (to early detect pre-malignant lesions).
- At the cervix there is transitional zone between two types of cells, it undergoes rapid
growth, if there is irritation due to HPV, it might turn malignant. It takes years from the
moment it begins to grow abnormally to the moment it becomes malignant, that is why
we do frequent PAP tests, to detect it before it turns into malignant tumour.
- The results come back from PAP test either ASCUS (Atypical squamous cells of
Undetermined Significance) / LG-SIL (low grade squamous intra-epithelial lesion) / or
HG-SIL (high grade squamous intra-epithelial lesion)

OSCE-guide-III.doc Page 170 of 255


OB-GYN

- For ASCUS:
o Woman ≥ 30 yrs  HPV DNA testing
 If negative  repeat cytology after 1 year
 If positive  colposcopy
o Woman < 30 yrs  repeat cytology in 6 months
 If negative  repeat after 6 months  still negative  routine screening
 If ≥ ASCUS  colposcopy
- For LG-SIL:
o Colposcopy
o Or repeat cytology after 6 months
 If negative  repeat after 6 months  still negative  routine screening
 If ≥ ASCUS  colposcopy
- For HG-SIL:
o We send you for colposcopy

- For colposcopy, we will refer you to the gynaecologist who will perform special
procedure, during which, the gynaecologist will take a biopsy, and send it for further
investigations;
o If the biopsy is negative, we will repeat the PAP after 6 months
o If the biopsy is positive, we will do more investigations to establish a diagnosis
and may need to do another larger biopsy called cone biopsy

Treatment options:
- Laser
- Cone biopsy
- LEEP (loop electrosurgical excision procedure)

Colposcopy
- Colposcopy is a magnification of the cervix (10-12 times), the procedure may cause some
discomfort but is not painful.
- The gynaecologist will insert a speculum (the same instrument used for Pap test), and
then she/he will use a special magnification device (the colposcopy) to visualize the
cervix.
- The gynaecologist will apply acetic acid (vinegar) that helps make the vascular patterns
more visible, application of this acetic acid may give an itchy sensation.
- Then if the gynaecologist suspects a lesion, she/he will need to take a biopsy, you will
feel a punching sensation, and you might experience a little discomfort and spotting for
few days.
- You need not to have anything inserted into your vagina for 24 hours before and 2 days
after the procedure (no vaginal intercourse, no douching), and you might need to take
some OTC medications (Advil) for few days after the procedure.

OSCE-guide-III.doc Page 171 of 255


OB-GYN

Antenatal Counselling

o Ms XX has missed her period for 2 wks; she did a home preg test which was positive. This is
her first experience. In the next 10 min, please talk to her and give her necessary advices
about her pregnancy.
o A 38 yrs old pregnant lady came to you because she is concerned about problems during
advanced-age pregnancy, counsel

Pregnancy  Was this planned pregnancy?


 What is your feeling about that? And your partner feeling? congratulations ☺
 How do you describe your general health?
 Symptoms suggesting pregnancy: any nausea / vomiting? Breast engorgement
/ heaviness? Frequency?
M  LMP, calculate due date
G  Pap smear?
O 
S 
PMH  HTN, Diabetes, Kidney disease, Heart disease, Epilepsy, exposure to
chickenpox as a child, Rubella vaccine, HBV, HIV, Blood Group & Rh group
 Allergy
 Medications
FH  FH of complicated pregnancies, abortions, MR, genetic & congenital
abnormalities
Social Hx  Home, violence, support
 Environment: work
 Activity: exercise / any pet
 Diet
 SAD
Concerns  Address any problem you find while you’re taking history

OSCE-guide-III.doc Page 172 of 255


OB-GYN

Counselling:

 To ensure healthy outcome of the pregnancy I need to see you on scheduled visits, every 4
weeks till the 28th week, then every 2 weeks till the 36th week, and then every week thereafter
and till delivery
 Today we’ll do physical examination including pelvic exam, Pap smear if more than 6
months, blood work including CBC, Lytes, INR/PTT, Urea, Creatinine, Blood Type, VDRL,
Rubella antibody, Hepatitis, ± HIV, Urine dip and microscopy, ± ECG.
 Anatomy US at 20 weeks. Glucose challenge test at 24 weeks
 Risks of Down syndrome are: 1/400 at 30 yrs of age, 1/200 at 35 yrs of age, and 1/100 above
40 yrs of age  we try to anticipate it by US and integrated prenatal screening then confirm it
by amniocentesis
 U/S for nuchal translucency: at 12 weeks
 IPS I: 11-14 wks /+/ IPS II: 15-18 wks (Maternal serum alpha-fetoprotein, β-hCG, uE3–
Unconjugated estrogen)
 Amniocentesis (U/S-guided trans-abdominal extraction of amniotic fluid / for identification
of genetic anomalies): at 15-16 wks, 0.5% risk of spontaneous abortion and risk of fetal limb
injury
 You need a well balanced diet; Canada's Food Guide to Healthy Eating suggests 3-4 servings
of milk products daily (greater if multiple gestation), a daily caloric increase of -100 cal/d in
the 1st trimester, -300 cal/d in the second and third trimesters and -450 cal/d during lactation.
If you do not consume an adequate diet, you can take daily multi-vitamins (avoid excess
vitamin A)
 Important nutrients during pregnancy: folate; 0.4-5 mg per day / calcium; 1200-1500 mg per
day / iron: 1 mg/d in T1, 4 mg/d in T2 and > 6 mg/d in T3
 Pregnant ladies tend to have constipation, you can take Lactulose for this, avoid raw or
processed meat
 Haemorrhoids, back pain, heartburn and increased vaginal discharge are common
 Will gain weight; 5-10 pounds in 1st half, 1 pound /week in 2nd half, total of 25-35 pounds in
average
 Exercise is OK… walking, swimming, avoid strenuous activities
 Stay away from cats’ litter
 No medication without asking your doctor, no x-rays
 Smoking increase the risk of abortion, LBW, premature delivery
 No safe level of alcohol during pregnancy, better to avoid it totally
 Offer brochures, connect to support groups and classes for pregnant women

OSCE-guide-III.doc Page 173 of 255


OB-GYN

Endometriosis
You are covering for your colleague Dr. Smith. You are about to meet Mrs. XX to discuss the
result of her laparoscopy & inform her that she has endometriosis. For the next 10 minutes, please
talk to her& address all her concerns.
- Introduction: I would like to discuss the result of your laparoscopy but I need to get some
information.
o Why did you have laparoscopy done and what was your doctor’s concern?
o You have endometriosis:
─ This means implantation of the interior lining of the uterus somewhere in
other places outside the uterus including the ovaries, the supporting structures
of the uterus or on the intestine (draw a picture of the uterus and ovaries for
the patient).
─ During periods, this outside tissue also begins to bleed. This explains the
painful periods.
─ This may cause infertility in some people.
─ Sometimes it runs in the family
o I would like to ask about some symptoms (to fit everything together)
─ Pain: Analyze (OCD / PQRST). Relation to the period. Dysmenorrhea,
dysuria, dyschezia, dyspareunia. Do you need pain killer?
─ Infertility: I understand your frustration. For how long have you been trying
to conceive? Are you currently sexually active? How frequent?
─ Irregular vaginal bleeding  analyze
─ Frequency
─ Blood with stool, diarrhea
- Menstrual history – brief
- Gynecological history
- Obstetric history – GTPAL
- Sexual history – brief
- Past medical history: HTN, Diabetes, kidney disease, blood group & Rh. Allergies /
medications / hospitalization / surgeries / blood transfusion
- Family history of abortion
- Social history: smoking, alcohol, drugs / work / home environments / support
- Conclusion: endometriosis
- Plan:
o Will do physical and obstetric examination
o Give her treatment options
─ Medical:
 NSAIDs – e.g. naproxen sodium (250 mg PO bid)
16
 Pseudo-pregnancy: OCPs trial for 6-12 months (Ovral 1 tab PO od)
 Pseudo-menopause (only short-term <6 months): Danazole (weak
androgen / Side effects: weight gain, fluid retention, acne, hirsutism,
voice change)
─ Surgical treatment:
 Conservative laparoscopy: laser ablation / resection of implants
 Definitive: bilateral salpingo-oophorectomy ± hysterectomy
o Brochure & support groups
16
Ovral is progestin (levonorgestrel) and estrogen (ethinyl estradiol) combination OCP

OSCE-guide-III.doc Page 174 of 255


OB-GYN

Woman wanting an abortion

- History

Pregnancy ─ Pregnancy (LMP, symptoms, how did you find out, Rh status) How do
you feel? How is the feeling of your partner about the pregnancy?
─ Gynecological history: contraception history, surgeries, infections, PAP
─ Obstetrical history: hx of previous pregnancies / GTPAL
Social ─ Partner involvement? Abuse, rape?
─ Support; family, friends, spoken to any one else about this?
─ Education, Religious beliefs?
─ SAD
Abortion ─ What are your thoughts regarding abortion?
Depression ─ Exclude depression: MI PASS ECG
PMH / FH / SH

- Counselling

Be supportive and not judgmental


Pregnancy ─ Health while pregnant:
─ Counsel for nutrition, exercise, and activity
Social ─ Social support: it is important to communicate with support groups
Abortion ─ Does not need to rush decision
─ Abortion options are available in Canada; most hospitals will do less
than 14-16 weeks, no problems with some up to 20 weeks. Private clinics
also provide the service
─ No legal gestational age where it can no longer be performed
─ Most commonly done via D&C under mild anesthetic
─ Discussion about contraception especially if not the first time (not
necessarily at this meeting)
─ Alternative choices: adoption / or rethink about keeping the baby with
the help of social support
Depression ─ Normalize the patient depressed mood and feelings in view of
circumstances
─ Close follow-up
─ May refer to psychiatrist

OSCE-guide-III.doc Page 175 of 255


OB-GYN

Osteoporosis – Counselling / OR / Short Case


Age related decrease in bone mass, starts at 35 years old, accelerated post-menopause, leading to bone
fragility and an increased risk of fracture. May manifest later as kyphosis, loss of height and increased
abdominal girth
Risk factors for osteoporosis:
1- Personal history of fragility fracture (above the age of 40 years)
2- Maternal history of hip fracture
3- Malabsorption syndrome (celiac disease / inflammatory bowel syndrome)
4- Prolonged use of corticosteroids (> 7.5 mg/day for > 1-3 months)
5- Primary hyper-parathyroidism

6- Weight < 57 Kg
7- Current smoker
8- Premature ovarian failure (female on Tamoxifen for breast cancer / surgical menopause)
9- Male on androgen-deprivation therapy for prostate cancer
10- Heparin or anti-epileptic use or biologics (anti-cancer treatment)

Investigations:
- BMD:
Age group < 50 years 50 – 65 years > 65 years
When to do BMD If > 2 of the first (5) If > 2 of any from the list Always do BMD, screen
risk factors of risk factors even there is no C/O
- Blood work:
o Serum calcium and phosphate levels
o Alkaline phosphatase
o Creatinine
o SPEP (serum protein electrophoresis)
o PTH (para-thyroid hormone)
o Give vitamin D for 2-3 months, then assess the level, if > 0.75 nanogram  it is normal and
do not repeat it again

Treatment:
- Based on BMD, risk factors, age of patient  Fracture Risk Stratification  low, moderate, or high
LOW MODERATE HIGH
- Life style If fragility fracture (in thoraco-lumbar x-ray) OR prolonged - Life style
modifications use of corticosteroids modifications
- F/U DEXA NO YES - Medical
after 5 yrs - Life style modifications - Life style modifications treatment
- F/U DEXA after 2 yrs - Medical treatment

Life style modifications Medical treatment


- Weight-bearing exercises (walking, jogging) - Bisphosphonates
- Ca 1200 mg/d (including the dietary intake, - SERM (Raloxifene): agonistic effect on bone
avoid ↑↑↑ Ca) but antagonistic effect on breast and uterus
- Vit D 1000 IU/d if < 50 yrs, and 2000 IU/d if > - Parathyroid hormone (PTH)
50 yrs - Calcitonin (if back fragility fracture + pain)
- Smoking cessation - HRT
- ↓ alcohol and caffeine

OSCE-guide-III.doc Page 176 of 255


OB-GYN

Pediatrics

OSCE-guide-III.doc Page 177 of 255


OB-GYN

History taking – Pediatrics

History taking – Pediatrics In any pediatrics case: always


 Introduction: CONSIDER child ABUSE / NEGLECT
 Chief complaint  report to CAS
1- Analyze the CC
2- Impact
3- Rule out infection
4- Differential diagnosis
5- BINDE - BINDE screening
6- Past medical history - Mother attitude!
7- Family history

1- Analyze the CC - TIME: Os Cf D: When did it start? How did it start? Sudden or
gradual?
- At that time, did your baby have any fever, flu-like symptoms?
- Is it continuous or on and off? How often? Day and night?
- Character:
- PQRST
- If vomiting or diarrhea: COCA + BLOOD
- Timing: is it related to feeding / meals?
- ↑↓ Factors: is it related to position? Meals?
2- Impact - Is he drowsy? Floppy?
- Does he cry? Is it high pitched cry?
- Did you notice his suckling is weaker than before?
3- Red flags - Constitutional symptoms!
Rule out infection - R/O infection: Did you notice if your child has fever or skin rash?
Cough / wheezes? Ear pulling or discharge? Runny nose? Foul
smelling urine? Abdominal distension? Diarrhea?
4- Differential - Rule out child neglect - BINDE screening
diagnosis - Mother attitude!
5- BINDE - Scan for risk factors for child abuse / neglect
6- Past medical history - Hospitalizations / surgeries / blood transfusion
- Illnesses (cancer) / infections
- Medications / allergies.
- Travel
7- Family history - Family history of similar disease in the family

0-6 years 6-14 years 14-18 years


BINDE School performance HEAD SSS
Pregnancy / Birth Home
Immunization Education
Nutrition Activity
Developmental Diet
Environmental Suicide
Sexual activity
SAD (smoking / alcohol / drugs)

OSCE-guide-III.doc Page 178 of 255


OB-GYN

BINDE

Pregnancy:
- Was your pregnancy planned? If no  social issues
- Were you having regular follow-up visits? How about U/S? Was it normal?
- During your pregnancy, did you have any illness? How about any fever or skin rash? Have
you ever been in contact with sick kids? Kids with skin rash or fever? Have you ever been in
contacts with pets?
- Did you take medications? Even OTC? Did you smoke? Drink alcohol? Have you ever tried
recreational drugs? What about before pregnancy?
- Were you screened for Hepatitis B virus? HIV? other diseases? There is screening test that
we do a vaginal swab at 36 weeks called GBS, did u have it?
- What is your blood group? What is your baby blood group?

Birth:
- Was your baby full term or not?
- Was it a vaginal delivery or c-section?
o If c-section  why? Was there any complication? Abnormality?
o If vaginal  was it difficult labour? Prolonged labour? How many hours? Was
there any early gush of water? How many hours? Did you need any help to make
it easier; e.g. vacuum?
- Did your baby cry immediately or not? Do you know what his Apgar score was? Did he need
special attention? When did you leave the hospital?
- Were there any bruises or swellings on your baby’s body?
- Were you told that your baby had any special features?
- After delivery, did you have any fever / discharge? Did you take any medications?

Immunization:
- Are your baby’s shots up-to-date?
o Yes  when was the last shot?
o No  any reason for that?
 Our religion prohibits vaccination: ok, that is fine
 We think vaccines cause autism: correct this info, vaccines are safe
 We were busy  neglect concern  what is baby weight?

Nutrition:
- Weight: Weight calculation:
o What is your baby’s weight today?
o What was his weight at birth? At birth : x Kg
o What was his highest weight? 5 months :2x Kg
o Do you have access to growth charts? 1 year :3x Kg
 If below 3rd percentile: underweight 2 years :4x Kg
 If (at any time) he crossed (down)
two major lines: failure to thrive More than 2 years old:
[(age X 2) + 8] Kg
Even low birth weight, catch up weight later, i.e. @ 1 year
they must be around 10 Kg, not only 3 x

OSCE-guide-III.doc Page 179 of 255


OB-GYN

- Height:
o To calculate height:
 At birth X cm 50 cm
 1 year 1 ½ X cm + 25 75 cm
 2 years 1 ¾ X cm + 12.5 87.5 cm
 3 years 1 7/8 X cm + 6.5 94 cm
 4 years 2 X cm 100 cm
o For each year: the baby gains (½) of the previous year increase, so the baby gains
½ X by the first year, ¼ X by the second year, 1/8 X by the third year.
- Diet:
o What do you feed your baby?
 Everything we eat. No restrictions!  that is fine
 Breast feed  if more than 4 months:
• Any iron supplement?
• Any Vit D supplement?
 Formula:
• Since when?  if since birth: any reason that you chose formula
over breast feeding?
• Which formula? Any recent change in formula?
o For any case of chronic diarrhea?
 Do you give him solid food; biscuits / bread / cereal?
 Which started first? The diarrhea or this new food?
 How many diapers do you change per day? (normally 5 – 6)

Developmental:
Now I would like to ask you some questions about the kind of activities that your child can do,
and other questions to assess his development.
Gross motor Fine motor
Sit alone / roll over 6 months Draw line 15 months
Crawling 9 months Draw cross 2 years
Standing / cruising 1 year Draw circle 3 years
Walking 15 months Draw square 4 years
Go upstairs holding 18 months Draw triangle 5 years
Go downstairs 2 feet 2 years
Tricycle 3 years
Social Speech / verbal
Social smile 6 weeks Mama / papa 9 months
Stranger anxiety 6 months 2 words beyond Ma, Pa 1 year
Separation anxiety 9 months 2-3 words phrases 2 years
Says “NO” 2 years Short sentences 3 years
Speaks fluently 5 years

N.B. (autism / Down syndrome / child abuse): there is no stranger or separation anxiety.

OSCE-guide-III.doc Page 180 of 255


OB-GYN

Environment:
- How do you feel being a new mom? How do you feel about your baby?
o How is your mood? You look down for me, any chance you are being depressed?
Did you have depression before?
- With whom do you live? How is the relation between you?
o How is the relation between you and the baby?
o How is the relation between your partner and the baby?
- How do you support yourself financially?
- Do you live in home (basement: mold) or apartment? Is it an old building (lead)?
- Any other children in the house?
- Do you or any body in the home smoke? Drink? Use recreational drugs?
- Is anyone of your family seeing a psychiatrist? Has mental illness?
- In ABUSE cases: tell me more about your childhood …

HEAD SSS
Home:
- With whom do you live?
- How is the relation between you? Are they supportive?
- Any siblings?

Education:
- Do you go to school? Do you like going to school?
- Which grade? Which subjects do you study?
- How about your marks, what marks do you get? What about in the past?

Activity:
- What kind of hobby do you have?
- Have you travelled recently?
- In EPILEPSY case: do you operate machines / drive / go hiking?

Diet:
- How about your diet? What do you eat? Do you follow special diet?
- What is your weight? What was your weight before?

Suicide:
- How is your mood?
- Any chance that you might hurt yourself?

Sexual activity:
- Are you dating? Are you in relationship?
- Are you sexually active? When did you start? When was the last time?
- How many partners do you have? Do you practice safe sex?

Smoking / Alcohol / Drugs:


- Now, I would like to ask you some personal questions, it is important to ask it, and it is
confidential, do you Smoke? Drink Alcohol? Have you ever tried recreational drugs?
- Sometimes people at your age might start to smoke, drink, or use recreational drugs. Do you
know any of your friends doing this? How about you? Have you tried that?
- For IV drugs: When was the last time? Did you share needles?

OSCE-guide-III.doc Page 181 of 255


OB-GYN

Jaundice
A new born 5 days old, with jaundice since day 2
Introduction Differential diagnosis of newborn jaundice
CC - Physiologic (usually days 2-7) 
- Analyze the jaundice (OCD) unconjugated
- Impact / consequences - Breast milk jaundice
- Red flags / rule out infection - Breast feeding jaundice
- DD - Pathologic (anytime)
- BINDE - Hemolysis (unconjugated)
- Birth  pathological - Infection  sepsis (conjugated or
- Nutrition  physiological unconjugated)

FH
Introduction:
Good morning Mrs …, I am Dr …, I am the physician in charge today, I understand that you are
here because your son has jaundice (or is yellow). In the next few minutes I will be asking you
some questions to help me figure out the condition, before I proceed, I would like to know the
name of your child? … This is a nice name.

1- Analyze the CC:


- When did it start? Early in the 2nd day (or before: pathological) or late (pathological or
physiological)?
- Who noticed it? You or someone else? When? Where did you notice it? How about his eyes?
How about his feet? Is it spreading? Is it ↑ or ↓?
- How about his urine, is it darker? And stools, is it pale?
2- Impact / Consequences:
- Is he drowsy? Floppy?
- Does he cry? Is it high pitched cry?
- Did you notice his suckling is weaker than before?
3- Red flags / Rule out infection:
- Did you notice if your child has fever or skin rash? Cough / wheezes? Discharge from his
ears? Runny nose? Foul smelling urine? Abdominal distension?
- Any night sweats / chills? Any lumps or bumps in his body? Tender points?

4- Differential diagnosis:
Physiological Pathological
How do you feed him? Breast milk? Formula? ─ Infection should be ruled out or
─ Breast feeding jaundice: (or “lack of confirmed by now
breastfeeding” jaundice): Not enough milk ─ Hemolysis:
 dehydration  What is your blood group? Your
─ Breast milk jaundice: is more of a baby blood group? Father blood gp?
biochemical problem (inhibition of  Rh incompatibility  IUGR
bilirubin conjugation leads to increased  Were you screened for infections
levels of bilirubin in the blood). during pregnancy?
Treatment: substitute with formula ─ Biliary atresia
─ Hepatitis: neonatal

OSCE-guide-III.doc Page 182 of 255


OB-GYN

5- BINDE
Birth:
Nutritional history:
- How do you feed him? Breast milk? Formula?
- Breast:
o How many times do you feed him?
o Do you use 1 breast or both of them? How long each?
o After feeding him, do you feel your breast engorged?
- Formula:
o Any reason to choose formula feeding?
o Which type of formula? Do you know how to prepare it?
Environment:
- Any other children? Did any of them develop jaundice after birth before?

6- PMH?!
7- FH:
- Jaundice
- Liver disease
- Blood disease
- Disease called cystic fibrosis

Diagnostic - Hemolytic workup: CBC / blood gp (mother and baby) / peripheral blood
workup: smear / Coomb’s test / bilirubin (direct and indirect)
- Septic workup: CBC / differential / blood & urine cultures / TORCH screen
- TSH and G6PD screening
- Liver enzymes / bilirubin / and coagulation profile
When to - If in the first day (or early second day) of life
suspect - Bilirubin rises > 85 µmol/L/day
pathological - Bilirubin level > 220 µmol/L before 4 days of age
jaundice? - Conjugated (direct) bilirubin > 35 µmol/L
- Persistent jaundice lasting beyond 1-2 weeks of age
Treatment - Ensure proper hydration and feeding
- If sepsis: treat the underlying infection
- Phototherapy: if total bilirubin is > 300 µmol/L, and only for unconjugated
hyperbilirubinemia, it is contraindicated in direct hyperbilirubinemia
- Exchange transfusion: if total bilirubin is > 400 µmol/L

The mother has a concern: will he develop mental retardation?


1. This is a reasonable concern, I am glad you came here today to figure out
2. What made you think about that?
3. In order to be able to answer this, I first need to ask you some questions, examine him
and we may need to do some blood tests. However I would like to inform you that
jaundice in newborns is a common condition, and it is usually physiologic jaundice that
does not lead to mental retardation

Complications of Rh incompatibility: (1) kernicterus (brain damage  seizures), (2) Hydrops


fetalis (generalized edema), (3) Hypoglycemia

OSCE-guide-III.doc Page 183 of 255


OB-GYN

IUGR

A newborn 3 hours old with IUGR, counsel the mother

Introduction Good morning … I understand that you just gave birth, my colleagues are
taking care of your baby. And I would like to ask you some questions
regarding your child health, but first tell me;
- How do feel right now?
- Have you seen the baby?
- Did you pick a name?
News Your baby has just been diagnosed with a condition called “intra-uterine
growth retardation” or “low birth weight” … For that reason; I would like
to ask some questions about your pregnancy!
BINDE
Obstetrical history - GTPAL
- Were you pregnant before? How many times? Any abortions?
Miscarriages?
Mother PMH - Any history of chemo therapy or exposure to radiation
- Any family history with congenital anomalies

+ The mother has a concern: the baby is green


 This means he passed meconium (baby stool) which means the baby had a stressful delivery

+ The mother has a concern: is it my mistake?


 This condition is related to multiple factors, some factors are related to pregnancy, and others
are related to the baby or the environment. On the other hand, there are NO safe limits for
smoking / drinking alcohol during pregnancy, so, in the future, if you decide to become pregnant,
it is important NOT to smoke / drink alcohol / use recreational drugs during your pregnancy

Possible causes:
─ Smoking / alcohol / cocaine during pregnancy, (cocaine during pregnancy 
microcephaly, IUGR, MR)
─ TORCH infection,
─ Extreme of age, esp. advanced age pregnancy

Risks for the next 48 hours: asphyxia / hypoglycemia

OSCE-guide-III.doc Page 184 of 255


OB-GYN

Crying Baby

Introduction
CC
Analysis of CC ─ OCD / all the time / day and night?
─ Is he crying > 3 hrs/day for > 3 days/week for > 3 weeks
─ What initiates or increases the crying?
o Any chance the baby is hungry? What do you feed him?
o Any chance that he is too hot / too cold? Do you adjust the
temperature?
o Any chance that he is wet? How often do you change his diapers
daily? Is there any skin or diapers rash?
─ What improves or decreases the crying? When he cries, what do you do?
o Did you try to hug / hold / burp / sooth / play music / give him a
walk?
o Did you try to rock him? Shake him? What happened to him?
─ When he cries, does he pull his legs? Is he passing gases? Is his abdomen
distended? Is it related to feeding? How are you coping with this?
Impact ─ How does this affect your life? And your partner life? Are you able to go
to work?
─ Is he drowsy? Floppy?
Red flags / R/O ─ Did you notice if your child has fever or skin rash? Cough / wheezes?
infection Discharge from his ears? Runny nose? Foul smelling urine? Abdominal
distension? Diarrhea?
DD ─ Any infection (there will be other symptoms)  review of systems
─ Infantile colics (crying > 3 hrs/day for > 3 days/week for > 3 weeks),
between the age of 3 weeks and 3 months, without another explanation
 reassure
─ Child neglect
─ Feeding problems: overfeeding / hungry
BINDE Scan for risk factors for child abuse
Nutritional ─ How do you feed him? Breast milk? Formula?
─ What about his weight?
Environmental ─ With whom do you live? How is the relation?
─ How do you support yourself financially? Do you get
enough support?
─ Any other kids? Any repeated visits to the ER?
FH ─ Mental problem
─ Parent SAD

Investigations (not including those for suspicious child abuse): CBC / urinalysis / stool analysis

OSCE-guide-III.doc Page 185 of 255


OB-GYN

Chronic Cough – Asthma


Child complains of cough for 6 weeks, post-pneumonia, father wants to renew antibiotics

Here, we have two issues:


 The productive cough & fever 6 weeks ago  pneumonia
 The intermittent / dry cough that presented after

CC Cough
HPI Analysis of the  Os Cf D /+/ COCA + B + Phlegm
CC  Certain time of the day? Night?
Cough Acute phase Chronic phase
Continuous / productive / Intermittent / dry cough / on and off /
fever / loss of appetite no fever
 Seen by a doctor? What  SOB, noisy breathing, wheezes,
diagnosis? Treatment? chest tightness, nausea / vomiting
 Anti-biotic history!  Does he cough to the extent of
 Did you renew it? From vomiting or LOC
the same doctor? Was  Pertussis vaccination?
he examined? Any x-
rays were done?
Impact How did this affect his life? Daily activity?
Red flags  Constitutional symptoms
 Triggers of Asthma: any thing that ↑ this cough?
Differential diagnosis  Chronic diarrhea  cystic fibrosis
 Any allergy
BINDE Brief
PMH Other allergic diseases: atopic dermatitis / allergic rhinitis
FH Allergic diseases: asthma / skin allergies

Triggers Infection  Recent chest infection? Flu-like symptoms? Fever / chills?


Medications  How do you use puffers? Stored properly? Not expired?
 Did you start new medication? β-blockers? Aspirin? Any recent ↑
in dose of these medications?
Outdoor  Exercise
 Cold air
 Pollens (is it seasonal?)
 Dust: construction / smug (smoke/ fog/ exhaust)
Indoor  Do you smoke? Anybody around you?
 Do you have pets? People around you?
 Fabrics related: carpets floor? Any change in linen? Pillows?
Blankets? Mattress? Curtains?
 Relation to any type of food? + Perfumes
 Do you live in a house (basement  mold)?
 Any construction renovation? Exposure to chemicals?
Stress  Any new stressful situations?

OSCE-guide-III.doc Page 186 of 255


OB-GYN

Questions:
Diagnosis: hyper-reactive airways disease
Investigations: x-ray
Treatment: steroids puffer for 4 weeks

Counselling:
─ The most likely explanation for that is a condition called: hyper-reactive airways disease.
It is a term used to describe asthma-like symptoms in infants (< 6 years old) that may
later be confirmed to be asthma when they become old enough to participate in asthma
tests (spirometry and bronchodilators).
─ This is a common problem, and is usually triggered by infection (acute bronchitis or
pneumonia), it may last up to 10 weeks after infection.
─ It may be self limited; however, we need to start treatment with puffer (steroids puffer
for 4 weeks).
─ When the child becomes older than 6 years, and if the condition is still persistent for
more than 10 weeks, we send the child for investigations (spirometry and
bronchodilators) to confirm the diagnosis of bronchial asthma.

─ If this condition happens in adults, we treat with puffer for 4 weeks, if no improvement;
we send to investigate for asthma (spirometry and bronchodilators then metacholine
challenge test).

Case: 9 weeks history of cough on Amoxil for 2 weeks


─ DD: asthma / bronchitis / cystic fibrosis / recurrent pneumonia
─ Investigations: CXR / CBC, differential, lytes / sweat chloride test

OSCE-guide-III.doc Page 187 of 255


OB-GYN

Anemia
6-9 months, mother complains he is pale?
1- Analyze the CC - Clarify CC: What do you mean he is pale? Is he yellow?
- Os Cf D
- Who noticed it? You or someone else? Is there any chance that
he had this pallor before and you were not aware of it?
2- Impact - Is he drowsy? Floppy?
- Does he cry? Is it high pitched cry?
- Did you notice his suckling is weaker than before?
Signs of - Is he active / playful like before? What can he do? Is he crawling?
anemia - If he is doing activity, did you notice any SOB? Fainting?
- Is he gaining weight?
3- Red flags: rule out - Constitutional symptoms!
infection - Did you notice if your child has fever or skin rash? Cough /
wheezes? Ear pulling or discharge? Runny nose? Foul smelling
urine? Abdominal distension? Diarrhea?
4- Diff diagnosis: - Rule out child neglect
- Bleeding disorders: nose / gums / coughing / vomiting / bruises
 Iron def. anemia
 Thalassemia on body / blood in urine / stools / joint swelling
 Hemolytic disorders - Leukemia: Constitutional symptoms / Bone pain [if he walks,
 Bleeding disorders does he limp? if you carry him, does he complain of tender
 Chronic diseases points in his body] / cough / repeated infection
 Lead intoxication
 Leukemia
5- BINDE - Scan for risk factors for child abuse / neglect
- N: What are you feeding him? Breast milk? From the beginning?
Do you give him any iron supplements or iron fortified cereals?
- B: was he term or not?
- E: with whom do you live? How do you support yourself
financially?  offer social support
- Where do you live, if old place, have you ever seen him eating
the paint scales?
6- Past medical history - Any heart / lung / kidney / liver disease?
- Hospitalizations / surgeries / illnesses (cancer) / infections
- Medications (Sulpha drugs – G6PD deficiency) / allergies
- Travel
7- Family history - Family history of similar disease in the family
- Any bleeding disorder
- Any repeated surgeries? (cholecystectomy / splenectomy)
- Ethnicity: some blood diseases are more common in certain parts
of the world, that is why I need to ask you about your
background, what about your partner?
- Are you related by blood to your partner?

Investigations: lab works; CBC / differential / lytes / serum iron studies (ferritin, TIBC) /
hemoglobin electrophoresis / KFTs / INR / PTT
Treatment: iron supplement

OSCE-guide-III.doc Page 188 of 255


OB-GYN

Vomiting

The mother of (6 weeks – 3 months) old baby came to the clinic complaining of child’s repeated
vomiting.

Introduction
Chief complaint

1- Analyze - When did it start? How did it start? Sudden or gradual?


the CC - At that time, did your baby have any fever, flu-like symptoms?
- Is it continuous or on and off? How often? Day and night?
- COCA + BLOOD: what do you feed him? Is he vomiting the entire amount? Is
it watery or curdy?
- Character: is the vomiting forcible? Projectile?
- Timing: is it related to feeding / meals?
- After vomiting, do you fell he is still hungry? General condition after vomiting
- ↑↓ Factors: is it related to position? When lying down?
- Other GIT symptoms: diarrhea
2- Impact - Is he drowsy? Floppy?
- Does he cry? Is it high pitched cry?
- Did you notice his suckling is weaker than before?
- Dehydration: do you feel his lips dry / skin dry? Does he tear? How many
diapers
- Failure to thrive: what about his weight, do you know his weight? What was
his weight at birth? Do you have access to his growth charts?
3- Red flags - Constitutional symptoms!
- R/O infection: Did you notice if your child has fever or skin rash? Cough /
wheezes? Ear pulling or discharge? Runny nose? Foul smelling urine?
Abdominal distension? Diarrhea?

4- DD Differential diagnosis of 6 weeks vomiting


- Rule out child neglect - BINDE screening
- Mother attitude!
- Pyloric stenosis - Family history of pyloric stenosis
- Appear at age of 2 – 4 weeks
- Projectile / non-bilious / baby still
hungry after feeds
- Any infection / meningitis - Continuous
- Brain tumour - Other symptoms / neurological:
weakness / neck stiffness / seizures
- GERD - After feeds
- Wrong formula OR not preparing - Not all feeds
it well - No ↓ in weight
- Overfeeding OR NOT burping

OSCE-guide-III.doc Page 189 of 255


OB-GYN

5- BINDE - Scan for risk factors for child abuse / neglect


Nutritional history:
- How do you feed him? Breast milk? Formula? Which type? Do you know how
to prepare it properly? Any solid food?
- Which happened first, the vomiting or switching to the new formula?
- Weight analysis!
Environment:
- How do you feel being a new mom? How do you feel about your baby?
- How is your mood? You look down for me, any chance you are being
depressed? Have you ever been depressed before?
- With whom do you live? How is the relation?
- How do you support yourself financially?
- Do you get enough support?
6- PMH
7- FH

Pyloric stenosis DD: duodenal atresia / tracheo-esophageal fistula

Management plan:
- Investigations: lab works (CBC, lytes, ABG) / US
- If dehydrated: admission
- If suspicious child neglect: contact CAS

Potential risk factors for child abuse:


- SAD parents (smoking / alcohol / drug use)
- Pregnancy not planned
- Preterm baby
- Congenital anomalies
- Baby who needed special attention after delivery
- Separation from the child
- Difficult child
- Young couple
- Parents with history of abuse
- Stress or financial difficulties in the family

Investigations for child neglect CONTACT CAS


- Full blood work / CBC / albumin level
- Fundoscopy
- Skeletal survey

OSCE-guide-III.doc Page 190 of 255


OB-GYN

Diarrhea
Diarrhea
Failure to thrive – FTT NO FTT
 What about his/her appetite? - Toddler’s diarrhea
 What other associated symptoms? (Respiratory / Gluten) - Infections
Cystic fibrosis Celiac disease Milk protein HIV - Lactase Deficiency
allergy (lactose intolerance)
- Good appetite - Poor appetite From cow milk
- Respiratory - Gluten Should not be
given < 1 year

A 50 years old father comes with 9 months child with 6 weeks of diarrhea (CHRONIC)
1- Analyze the - Os Cf D
CC - COCA + BLOOD + others:
- Watery / loose / bulky
- Any undigested food
- Difficult to wipe?
- ↑↓ Factors: Juice (Excess fruit juice)
- Identify FTT – weight: What is weight today? At birth? Last visit? The
highest weight? Not gaining weight?
- Other GIT symptoms: vomiting
- APPETITE
2- Impact - Is he drowsy? Floppy?
- Does he cry? Is it high pitched cry?
- Did you notice his suckling is weaker than before?
- Dehydration: do you feel his lips / skin dry? Does he tear? How many
diapers
- Failure to thrive: what about his weight, do you know his weight? What
was his weight at birth? Do you have access to his growth charts?
- Long period  malabsorption  anemia and rickets
3- Red flags: - Constitutional symptoms!
(R/O infection) - Did you notice if your child has fever or skin rash? Cough / wheezes? Ear
pulling or discharge? Runny nose? Foul smelling urine? Abdominal
distension? Diarrhea?
4- Differential DD for ACUTE diarrhea:
diagnosis - Use of antibiotics
- Infectious:
- Camping / travelling
- Any body else at home with diarrhea?
- Does he go to day care?
DD for CHRONIC diarrhea without failure to thrive:
- Toddler’s diarrhea: does he drink too much juice daily?
- Infectious – parasitic / traveller’s diarrhea
- Lactose intolerance:
- Does he pass a lot gas?
- Does he have any redness / skin rash at his buttocks?

OSCE-guide-III.doc Page 191 of 255


OB-GYN

DD for CHRONIC diarrhea WITH failure to thrive:


- HIV (if susceptible): was he screened for HIV
- Milk protein allergy: cow milk given before 1 year
- Cystic fibrosis
- Celiac disease
Cystic fibrosis Celiac disease
- Good appetite - Poor appetite
- Repeated chest infections? Cough? - bulky foul-smelling stools
- When did he poo the first time? - What do you feed your son? How
- Did he have yellow discoloration many times?
after birth? For how long? - Do you know how to prepare the
- Does he have any bulging through formula? Was it changed
his buttocks? recently?
- Did you start to give him solid
food? What type of food? How
about bread, cereals, biscuits?
Which started first, the diarrhea or
the new food?
- After you feed him, does he have
abdominal distension? Gases?
Pulling his legs?
- Does he have flat buttocks? Thin
legs and arms? Distended
abdomen?
5- BINDE - Scan for risk factors for child abuse / neglect
- Nutritional history
- Weight analysis
6- PMH
7- FH

Questions:
- What is your differential diagnosis:
o Cystic fibrosis
o Celiac disease
- If the biological mother called, want to know about her son, do you tell her or no?
o In order to determine whether I should release any information or no, I would
like first to know who has the legal custody (guardian) of this child. It might be
the adopting father, a social worker (case manager) …

Notes:
- If the child was adopted, and you are speaking with one of the new parents:
o Are you the biological mother/father?
o Is this adoption or foster home?
o When was the child adopted? At which age? From where?
o What were the circumstances?
o Do you have information about the biological parents?
o Was he screened for HIV?

OSCE-guide-III.doc Page 192 of 255


OB-GYN

Mother worried about her child weight


- Not gaining weight
- Not eating well

1- Analyze the CC Weight analysis:


- Weight today, birth, last visit, highest
- When did you start to worry about that? Why?
- Who noticed it?
- Who is the primary care giver? For how long have you been with him?
- Do you have his growth chart? If no: if you do not mind, I need to
contact his family physician to take a look at his charts
Height analysis:
- Height today, birth, last visit

2- Impact - Is he drowsy? Floppy?


- Is he playful? Active like before? Any limitations? Does he turn blue
with activity?
3- Rule out any - Constitutional symptoms!
serious condition: - Any congenital or long term disease?
chronic inf/ dis / - Review of systems: cardiac/ chest/ GIT/ urinary/ MSK/ skin/ allergy/
malignancy pale/ bleeding
4- Differential - Rule out child neglect
diagnosis - Difficulty swallowing (CP, Cleft Palate)
- Chronic loss: chronic Diarrhea (Celiac disease, CF, pancreatic
insufficiency) / chronic vomiting (pyloric stenosis in a younger child)
- Diabetes mellitus (drinks too much water, pees a lot, tired)
- Chromosomal abnormalities / inborn error of metabolism
5- BINDE - Scan for risk factors of potential child abuse / neglect
- Apgar score at birth
- Diet in details: breast feeding/ formula/ cow milk? Any reason? For
how long? Any supplements? Any solid food?
- Developmental milestones
- Environment: with whom do you live? Who takes care of the baby? Is
he/she capable of doing this?
- SAD during pregnancy and now
6- PMH -
7- FH -

Under weight:
- Failure to thrive (FTT): weight decreases first then height will be affected later
- Endocrine causes: fat and short
- Congenital: everything is small / short, thin with small head

Failure to thrive
- Weight < 3rd percentile or falls across 2 majors percentiles
- Most common cause is inadequate intake

OSCE-guide-III.doc Page 193 of 255


OB-GYN

Case: A 2 years old boy does not want to eat. The father carries a bag!
─ History:
o When you ask about the bag, he says it is for the boy lunch, it is full of candy and a coke.
o Details about breakfast, lunch, dinner and snacks
o Review of systems  will be negative

─ Plan; to improve the health and diet education of the father:


o Educate about nutritious food and supplementation
o Give brochures and Canada dietary guidelines
o Refer to dietician specialist or nurse
─ Recommend supplementation:
o Iron
o Multi-vitamins

─ Examine the baby


─ Blood works: CBC / lytes / serum iron studies (ferritin, TIBC)

─ Differential diagnosis:
o Stresses int the family
o Child abuse / neglect
o Failure to thrive

Case: A 6 years old developed severe allergy to peanut, child is now stabilized, counsel the
father.
─ Is it first time to eat peanuts? Any similar reaction before? Any known food allergy?
─ Review of systems  will be negative

─ Past history of asthma, allergic rhinitis, allergic skin reaction?


─ Family history of allergy? Asthma, allergic rhinitis, allergic skin reaction?
─ Other siblings with allergic reactions?

Management:
─ Will send the boy for allergic testing
─ Strict avoidance of allergens
─ Epi-pen

OSCE-guide-III.doc Page 194 of 255


OB-GYN

Fever

Introduction
CC FEVER
1- Analyze the CC - Os Cf D
- Any flu at that time?
- Any diurnal variation? More at morning or night?
- Any special pattern? More every 2nd or 3rd day?
- Do you measure it? How many times daily? How do you measure
it?
- Did you try to give any medications to help? Did it help?
- Is it the first time?
- Other constitutional symptoms
- Other persons at home with the same symptoms?
2- Impact - Is he drowsy? Floppy?
- Does he cry? Is it high pitched cry?
- Did you notice his suckling is weaker than before?
3- Red flags - The fever and constitutional symptoms are already analyzed
- Review of systems: DD
4- Differential - Is he tired?
diagnosis: - Did you notice any skin rash?
Review of systems  OCD / distribution / color / do you feel it elevated?
 Are his shots up-to-date?
- Buttocks / abdomen  henoch schonlein purpura /
SKIN RASH

Investigations: urinalysis Treatment: steroids


- Trunk  vesiculo-papular  chickenpox
- Face: measles / rubella
- Cheek: fifth disease
- Headache / drowsy / neck pain / rigidity / nausea / vomiting? Does
he recognize you? Talk to you?
- ENT
- Chest: cough / phlegm / SOB / wheezes
- Abdomen (pain, distension, diarrhea) / liver (yellow color, itching,
dark urine, pale stools) / urinary (urine changes, crying while
peeing, loin pain)
- Joints: pain / swelling / mouth ulcers
5- BINDE - Scan for risk factors for child abuse / neglect
6- Past medical history

7- Family history

OSCE-guide-III.doc Page 195 of 255


OB-GYN

Runny Nose / Flu / URTI

8-15 years child is coming to see you with his mom, c/o: runny nose / flu / URTI?
Introduction - To BOTH the mother and the child
- During the encounter, distribute the questions and interaction between
both the mother and the child
1- Analyze the CC - Os Cf D
- COCA
- What ↑ or ↓
- Is this the first time? Or did it happen before?
2- Impact - Is he playful? Active like before? Any limitations?
3- Red flags - Constitutional symptoms
Review of systems:
- Rule out infection: Any recent flu-like symptoms? Do you feel tired/
fatigue? History of sinusitis / Pain in your face? Any sneezing? Red
eyes? Pain/discharge in ears? Any sore throat/ oral ulcers/ tooth pain?
Pale / bleeding
- R/O meningitis: Neck stiffness / pain? Headache? N/V?
- Cardiac / chest / GIT / urinary / MSK / allergy
- Skin rash
4- DD - Allergic rhinitis: runny nose related to seasons, recurrent, no fever
- Viral flu: respiratory symptoms / joints & muscles ache
- Viral common cold
5- BINDE - Scan for the risk factors of potential abuse
- Immunization
- School performance
6- PMH - Any congenital or long term disease?
7- FH - Other members in the family with symptoms?
- School contacts?

Physical exam Mouth


ENT
LNs
Chest exam

OSCE-guide-III.doc Page 196 of 255


OB-GYN

Rash

Clinical Presentation When does the kid go back to school?


Mumps Fever, headache, parotitis (bilateral; pushes earlobes up and out), myalgia, Infectivity: 7 days pre-parotitis to 7 days
malaise post-parotitis
Measles Appearance: erythematous maculo-papular rash; Koplik spots No rash, no fever (infectivity: 4 days pre-
Timing: 10-14 days incubation, rash 3 days after start of symptoms rash)
Distribution: starts at hairline, spreading downwards; palms and soles
typically not involved
Rubella Appearance: pink, maculo-papular rash. Infectivity: 7 days pre-rash to 5 days post-
Timing: 14-21 day incubation; rash 1-5 days after start of symptoms. rash
Distribution: starts on face spreading to neck and trunk.
Chickenpox Appearance: macules  papules  vesicles  crusting; all stages apparent Infectivity: 1-2 days pre-rash until vesicles
(varicella) at once (polymorphous rash)  very pruritic have crusted over
Timing:
 10-21 days incubation;
 1-3 days prodrome: (fever and respiratory symptoms),
 Then rash
Distribution: face, trunk, extremities, mucosa, palms and soles.
Erythema Appearance: uniform, erythematous maculo-papular rash Infectivity: prior to onset of rash
Infectiosum Timing: 4-14 days incubation, rash 10-17 days after symptoms
(fifth disease) Distribution: bilateral cheeks with circum-oral sparing, can affect trunk
Roseola Appearance: pink maculo-papular rash (faint).
Timing: 5-15 days incubation; rash 3-5 days after symptoms.
Distribution: starts at neck and trunk spreading to face and extremities
Management: rest / anti-pyretics / fluids / good nutrition

Reye Syndrome:
 Acute hepatic encephalopathy and non-inflammatory fatty infiltration of liver and kidney
 Mitochondrial injury of unknown etiology results in reduction of hepatic mitochondrial enzymes, diagnosis by liver biopsy
 Associated with aspirin ingestion by children with varicella or influenza infection.
 40% mortality

OSCE-guide-III.doc Page 197 of 255


Pediatrics

Delayed Speech

Introduction Clarification: is it not gaining words, losing words OR


not speaking at all?
Verbal assessment
Rule out any serious condition - Hearing loss
- Autism
- Anatomical: tie tongue / cleft palate
BINDE
PMH - History of meningitis / jaundice
FH - Family history of deafness or hearing loss

VERBAL ASSESSMENT
- Would you please tell me more about that!
- When did you start to have concerns? Did you seek medical attention before?
- Is the child able to speak at all? How many words is your child capable of using? When
did he start to say it? Can he use many words in one sentence?
- Was he able to use more words (talk better) and lost them?
- How can he communicate with you? What does he do if he wants something?
I would like to ask you some questions in order to reach to the cause of this condition:
HEARING:
- How do you describe his hearing? Does he have hearing difficulties?
- If you call him, would he respond and reply? What if you are behind him? What if you
are in another room?
- Did you notice that he keep increasing the volume of the TV?
- Did he get repeated ear infections? Fluids in the ears? Discharge?
- Did he take any medications? Any antibiotics (aminoglycosides)?
- Was he ever screened for hearing test, when he was born?
AUTISM:
- Does he maintain eye contact? Does he show emotions?
- Is he aggressive? Does he play with other kids?
- Does he do repeated movements like rocking, or head banging?
- Does he have a favourite toy? How does he play with it? (train / spinning wheels)
- Any family history of autism?
BINDE:
- Start with the development: to rule out MR
Developmental (mile stones):
- What can he do? When did he start to sit? Crawl? Stand? Walk? Climb stairs?
- As a child, did strangers make him nervous?
- Does he control his urine / bowel movements?
Environment:
- Screen for neglect: how many hours you spend with him? Is he a difficult child?
- Family factor: how many languages do parents and other family speak at home?
Pregnancy / Birth:
- Did you have skin rash during pregnancy? TORCH infection? SAD during preg?
- Was it complicated labour? Apgar score?
- Did he have any special features? Congenital malformations? Cleft palate?

OSCE-guide-III.doc Page 198 of 255


Pediatrics

Seizing child counselling

Refer to the seizing child phone call case in the emergency medicine section for analysis of the
event

History Analyze the event During / before / after


Fever Analyze the fever
Rule out Rule out meningitis / pneumonia
BINDE
PMH
FH Febrile seizure / epilepsy
Counselling Introduction
Febrile seizures

Introduction:
─ Based on what you have told me, the most likely explanation of your child seizures is a
medical condition we call “benign febrile seizure”
─ What do you know about “febrile seizures”? Do you want me to clarify some information
about it? In details?

Febrile seizures:
─ This condition usually affects kids 6 months to 6 years, it is not uncommon, and a lot of
children (around 3%) might have attacks.
─ We do not know exactly the reason for it, but it is related to fever and may be because the
children brain is not fully developed at that age, and can not tolerate high fevers.
─ Usually it is self-limited, benign, typical attack is less than 15 minutes, and will not recur in 24
hours. Most children will outgrow their condition after the age of 6 years.
─ Another attack(s):
○ From the studies we know it might happen again; for each 100 child who got 1
febrile seizure attack:
o 65 children will not have it again
o 30 children will have another attack
o 3 children will have many other attacks even without fever
o 2 children will develop seizure disorder
○ The best treatment for it is the prevention that is why it is important to make sure that
whenever he gets a fever, to seek medical attention and to decrease the fever ASAP
using Tylenol or cold foments. Then find the source of fever and treat.
○ In case it happens again:
○ Turn the child on his side / protect him from hitting any nearby object / do not
force objects into his mouth
○ Bring to ER if seizure does not stop within 15 minutes
○ Diazepam 5 mg PR suppository
○ If repeated attacks, we may consider prophylactic anti-convulsion therapy
○ Will do CT, EEG
─ I will give you some brochures and web sites in case you want more information.
─ Any other questions or concerns.

OSCE-guide-III.doc Page 199 of 255


Pediatrics

ADHD counselling

The father comes to you saying that his son was diagnosed with ADHD two days ago and he has
concerns about ADHD and Ritalin. Counsel for 10 minutes.
Introduction To diagnose ADHD:
Address concerns - 2 settings (school / home)
 Diagnosis (symptoms of ADHD) - > 6 months duration
 Impact - < 7 years old child
 Differential diagnosis Differential diagnosis:
BINDE - ODD /+/ Conduct disorder
PMH - Specific learning disability
FH  ADHD / MR / autism / depression - Seizures (petit-mal epilepsy)
Conclusion - Depression
Introduction:
- Who diagnosed it? Usually teachers recognize it first (pick it), but to make a diagnosis a
psychiatrist, paediatrician, or a specialized nurse assessment is needed

Concern – do you give Ritalin (which is amphetamine) to children?


- Actually yes. A lot of children use Ritalin, it is the first line of treatment for ADHD, and it is
effective and has been used for long time.
- It is not exactly amphetamine, it is the same family, it is called “methylphenidate” and it is
approved for this indication.
- In children, it helps them to focus as increases their concentration and channels their energy,
this is crucial for children, as it allows them to do better in schools. Even though we might
not be able to cure all children with ADHD, by we try to help them with education, so that
they can have career and live independently in the future, without problems with the law.
- It is generally a safe medication in children. It is not addictive, and we can stop it at any time.
However, like any other medication, it has its side effects, that include: insomnia (that is why
we give it early), abdominal pain, and not all children improve on it.

Before talking further about ADHD and Ritalin, let me first ask you some questions to see if your
child meets the criteria of ADHD or any other developmental challenge:
Diagnosis (hyperactive / inattentive / impulsive):
- Did the teachers complain that your child is full of energy? Spinning all the time? Refuse to
stand still? Talk all the time? Answers even if he is not asked? Does he stand in-line or does
he break the queues?
- Can he focus on one subject for > 30 minutes? Can he finish his tasks (e.g. the homework)?
Does he jump from one activity to another without finishing it? Does he lose his stuff? Does
he forget his belongings?
- Is this only at school or also at home?
- Did you notice that yourself?
- How much time do you spend with him? How about the mother, is she involved?
- How about before? Did anyone mention that or no?

IMPACT:
- Impact on functioning, school performance, relationship with peers

OSCE-guide-III.doc Page 200 of 255


Pediatrics

Differential diagnosis:
ODD - Does he like not to follow the instructions?
- Does he like to challenge his teachers and other family members?
Conduct - Is he aggressive? Does he fight a lot with other children?
disorder - Does he have a pet? How does he treat his pet / or other pets?
- Did you notice that he takes others’ belongings without telling them?
- Does he tell the truth all the time?
- Does he like to set fires?
Learning - Does he like to go to school?
disability - Does he have specific difficulty in reading / writing / mathematics?
Petit-mal - Does he have a history of seizures?
epilepsy - LOC? Abnormal movements?
Depression - Was he stressed recently? Any loss of a beloved one?
- Is he sad? Crying? Nightmares? Losing weight?
Autism -
MR -

BINDE: detailed developmental history

Conclusion:
- I am really sorry for this loss; it must be difficult for children in his age to go through all
that. How is he/she coping with that?

He has NO symptoms OR does not fulfill the criteria:


- Based on what you have told me, and the fact that it happened > 8 years old, it is less
likely that he has ADHD.
- On the other hand, his symptoms and behavioural changes could be related to the loss of
his mother, which might lead to depression. And in his age group, depression may
manifest as behavioural changes like what he is experiencing right now.
- We can refer your child to a child psychiatrist who can help him deal with that.

He has symptoms:
- Based on what you have told me, your child symptoms meet the criteria for diagnosis
with ADHD. However, this is not uncommon condition, and there is medical treatment
for it, in which the first line is Ritalin.
- Counsel on Ritalin.

Notes:
- Whenever you hear that one of the parents has passed away  show empathy.
I am sorry to hear that, it must be difficult for children in his age to go through all that.
How is he/she coping with that? How are you coping?

OSCE-guide-III.doc Page 201 of 255


Pediatrics

Vaccination counselling
New comer to Canada, comes to you as she has some concerns about vaccinations
Introduction / welcome her / how do you feel? Speak with enthusiasm (to
Identify the language barrier encourage) with three
Identify concerns counselling sessions:
- Deal with concerns one by one - Pap smear
- Pose frequently and ask if she has any questions - Breast feeding
Candidacy for vaccination - Vaccination
Mother vaccination
○ What are vaccines? Otherwise, speak neutrally
○ How do we vaccine?
○ Side effects of vaccines
Introduction / welcome her / how do you feel?
- Good evening Mrs …vich, my name is Dr … I understand that you are a new comer to Canada,
and you came to the clinic because you have some concerns about vaccinations. We will discuss
all you concerns. First of all, welcome to Canada, for how long have you been here? How do you
feel being here?
Identify the language barrier
- Before we proceed, am I clear, or do I need to talk slower? We can arrange for an interpreter or a
family member, if you would like to.
Identify concerns
- Now, can you tell me more about your concerns?
- Do you need general information, or do you have specific concerns?
o I heard that vaccines cause autism!
o I think we do not have these diseases in Canada, why should we give the vaccines for
diseases not common here?
Thanks for coming here to discuss your concerns with me.
Vaccines cause autism!
- What gave you this feeling? Concerns?
- There is misinformation among the general population that there is a connection between vaccines
and autism. And the origin of this misinformation is a study done in England many years ago, the
study found there is a connection between autism and 1 type of vaccines; namely the MMR.
- And because we take vaccines very seriously, a much larger study was done, in large number of
countries, including very large number of children. Now we found for sure that there is no
connection between vaccines and autism. The only relation is a coincidence between the age in
which parents start to notice autism symptoms and the age we start to give MMR.
- When we tried to figure out why the first study found the connection, the explanation of that was a
bias in the selection by the author and the study was conducted to favour this outcome. Another
theory to explain the connection was the preservative used in the vaccine (Thiomersal) and it
contains mercury. However Canadian vaccines do not contain it.
- I can assure you that there is no connection between vaccines and autism. Any questions till now?

Why to vaccine against diseases not preset in Canada?


- Even though we do not have these diseases commonly in Canada, and this is because we have a
successful vaccination program. But because the world is a becoming more and more a small
village, and people travel easily from a place to another, we do not want your child to be
vulnerable to preventable diseases if there is any outbreak anywhere. And I will give you some
examples:
o In England, people stopped to vaccinate their children for MMR, they started to have
measles infections again, and some children ended up losing their lives.
o Diphtheria is also coming back because of lack of vaccinations, even in many European
and developed countries.

OSCE-guide-III.doc Page 202 of 255


Pediatrics

o Tetanus is universal; it is in the soil everywhere, if a child is injured, the injury is


vulnerable to be contaminated with tetanus, which has serious fatal consequences.
- Do you have any questions or any more concerns?

Candidacy for vaccination / history – BINDE:


Before I proceed further, I would like to make sure that your child is a good candidate for vaccination,
and we usually start vaccination at 8 weeks.
- Does your child have any illness, sickness, fever, flu-like symptoms? If high fever, we wait. Did
he get recurrent infections before? (if not very young)
- Was he diagnosed with any neurological diseases?
- Was he scanned for HIV? Were you scanned for HIV? Is there any risk that you might have HIV?
- Does he have any allergy?
Based on what you told me, your child is a candidate for vaccination.
- Is it ok till now?

Mother vaccination:
- As a child, were you vaccinated? How do you feel about that?
- If it is ok with you, we can set up a follow-up meeting to discuss in details your vaccination status
and find what vaccination(s) you might need to take.

What are vaccines?


- Vaccines are one of the most significant preventive interventions in medicine. It helped to save the
lives of large number of children all over the world. As children are vulnerable to be infected with
many infections, even thought it might affect adults, but usually with no serious consequences as
compared to children!
- We have large number of diseases and infections, caused by large number of bugs; some of them
cause serious illnesses. We try to protect against many of it. We take the different bugs (bacteria
and viruses), and we either kill it or we make it so weak so it will not cause any harm, then we
inject it into our bodies by a needle. Usually the body reacts by forming “anti-bodies” which are
protective chemicals against these bugs. Later in life, if the child will get exposed to the real bugs,
these anti-bodies will protect him.
- Was I clear? Any questions?
How do we vaccine?
- Because we have large number of diseases, and each one needs more than one shot, we try to
decrease the number of shots to be given.
- For example, there is a shot called “pentacel” that stimulates the body to produce protective
antibodies against 5 diseases, it protects against diphtheria, tetanus, polio, pertussis and
hemophilus influenza B. It is to be given at 2, 4, 6, 12 and 18 months and another booster dose at
around 5 years.
- You do not need to worry about memorizing them; I will give you a schedule of the required
immunizations to follow with his family doctor. And these vaccines are covered by OHIP.
- Any questions or concerns do you have?
Side effects of vaccines
- Vaccines are generally safe, millions and millions of children were and are vaccinated and it saved
the lives of large number of them.
- However, like any other medical intervention, vaccines have some side effects.
- It may cause minor issues, like pain, redness and swelling, at the site of injection, it may cause
fever, which we deal with Tylenol for pediatrics.
- There are some few rare side effects that include prolonged crying, seizure, and some children
become floppy, again, this is rare, but if this happens, you need to seek medical attention
immediately.
- Lastly, very rarely, children experience a serious allergic reaction, and that is why we keep the
children under medical supervision for 20 minutes after vaccination.
I hope that I answered you concerns, and now you have a better idea about vaccinations. Any other ideas or
concerns you need to talk about?
Thanks for coming and see you next visit for the first vaccination shot.

OSCE-guide-III.doc Page 203 of 255


Pediatrics

Child with DM counselling


Father of 6 years old boy comes to you as he has some concerns about his diabetic son

History - When was the child diagnosed? How?


- What type of insulin?
- Is DM controlled? Regular measure of blood glucose? Hb A1c!
- Symptoms of DM
- Symptoms of DKA
- Symptoms of complications: nephropathy / retinopathy / neuropathy
(usually develops more than 5 years after diagnosis)
Counselling - Do you have special concerns?
- What do you know about diabetes? Did you read? Hear? Anything?
- Type 1 DM
- Insulin
Complications - DKA /+/ Hypoglycemia /+/ Prevention of complications
- Macro and micro vascular complications
Management - Meal plan, education, exercise, psychological support
- Insulin injections with BS monitoring
- Children are more prone to brain damage with hypoglycaemia, therefore
high target range in younger and tight control in older kids

Diabetic ketoacidosis (DKA):


- Diabetic ketoacidosis (DKA) is an acute, major, life-threatening complication of diabetes.
DKA mainly occurs in patients with type 1 diabetes, but it is not uncommon in some
patients with type 2 diabetes.
- DKA is a state of absolute or relative insulin deficiency aggravated by ensuing
hyperglycemia, dehydration, and acidosis The most common causes are underlying
infection, disruption of insulin treatment, and new onset of diabetes.
- DKA is defined clinically as an acute state of severe uncontrolled diabetes associated
with ketoacidosis that requires emergency treatment with insulin and intravenous fluids.
- Symptoms of hyperglycemia associated with diabetic ketoacidosis may include thirst,
polyuria, polydipsia, and nocturia.
- Signs of acidosis may include shallow rapid breathing or air hunger (Kussmaul or
sighing respiration), abdominal tenderness, and disturbance of consciousness.
- Signs of dehydration include a weak and rapid pulse, dry tongue and skin, hypotension,
and increased capillary refill time.
Hypoglycemia:
- Hypoglycemia is a syndrome characterized by a reduction in either plasma glucose
concentration or its tissue utilization to a level that may induce symptoms or signs such
as altered mental status and/or sympathetic nervous system stimulation.
- Symptoms of hypoglycemia may be categorized as neurogenic (adrenergic) or
neuroglycopenic. Sympathoadrenal activation symptoms include sweating, shakiness,
tachycardia, anxiety, and a sensation of hunger. Neuroglycopenic symptoms include
weakness, tiredness, or dizziness; inappropriate behavior (sometimes mistaken for
inebriation), difficulty with concentration; confusion; blurred vision; and, in extreme
cases, coma and death

OSCE-guide-III.doc Page 204 of 255


Pediatrics

Bed wetting counselling / Nocturnal Enuresis

Introduction
1- Analyze the CC - OS CF D:
- When did it start? How did it start? Sudden or gradual?
- Frequency
- Primary or secondary (dry period(s) of time)?
- Is it continuous or on and off? How often? Day and night? Every
day? Every night?
- ↑↓ Factors: stress / drinking too much fluids before bedtime
2- Impact - How does Mom feel about it?
- How does the child feel about it (impact of this on child)?
3- Rule out infection - Constitutional symptoms!
- Did you notice if your child has fever or skin rash? Odd smell or
colour of urine? Pulls his penis? Cries while peeing?
4- DD - Rule out child - BINDE screening
neglect / abuse - Parent attitude!
- Medical conditions - DM (drinking too much water / going more
often to pee / feeling tired / losing weight)
- Diabetes insipidus (history of meningitis /
brain infection / head trauma)
- UTI (detailed in No 3)
- Neurological: trauma or surgery to back /
bowel dysfunction / leg weakness or
numbness
- Seizure disorder
- Stressors - New sibling
- Home / school change
- School performance
5- BINDE - Very briefly because the child is more than 6 years old
- Scan for risk factors for child abuse / neglect
- How is his school performance?
- Who is the primary care giver, who else does live with them at home,
is he the only child, any sisters or brothers?
6- PMH - Kidney disease
7- FH - Kidney disease
- Bed wetting
- DM
- Seizure disorder

OSCE-guide-III.doc Page 205 of 255


Pediatrics

Counselling - The condition is common;


- By the age of 4 years, 25% of children have this problem
- By the age of 7 years, 5-10% of children have this problem
- More common in boys than girls
- May be it is due to regression of his development because of the
current stresses in his life
- Condition is usually self limiting & you need to give the child some
time & he will adapt very well to the changes
Management - Try behavioural modification for 3 months:
- Avoid drinking late before sleep,
- Try to wake him up to go to the washroom,
- Document dry nights and use reward system.
- Alarm therapy
- Pharmacological measures:
- Desmopressin (DDAVP): 1 hour before bedtime. Be careful to
water intake, the only serious side effect is seizures due to water
intoxication
- Oxybutynin: at bed time, it ↓ bladder contraction therefore it leads
to ↑ bladder capacity

OSCE-guide-III.doc Page 206 of 255


Pediatrics

Breast feeding counselling


A lot of enthusiasm
 Congratulation for the news, how do you feel being a (prospective) mother? It is good to hear
that you plan to breast feed. Do you have any concerns?
 I need to ask some questions
─ If still pregnant:
○ How the pregnancy is going so far?
○ Pregnancy questions of BINDE
○ Ante-natal care follow-up (from OB-GYN)
─ If already delivered:
○ Birth questions of BINDE

 PMH: medications (lithium, antibiotics, anti-convulsions) & allergies


 Social history: smoking, alcohol, drugs / work and home environments

1- Rule out contra-indications for breast feeding:


- The mother has active herpes simplex on breast
- The mother has untreated, active T.B.
- The mother has been infected with HIV / is taking antiretroviral medications
- The mother is using or is dependent upon an illicit drug
- The mother is taking prescribed cancer chemotherapy agents, such as antimetabolites
that interfere with DNA replication and cell division OR is undergoing radiotherapy;
however, such nuclear medicine therapies require only a temporary interruption in
breastfeeding
- An infant diagnosed with galactosemia, a rare genetic metabolic disorder
- N.B.: Hepatitis B is NOT a contraindication to breastfeeding

2- Counselling:
 Advantages of breast feeding
- Highly nutritional, providing all elements baby needs (especially colostrum), breast milk
contains: more vitamin C, easily absorbable iron, less protein load on the baby
- Contains antibodies to help your child fight infections
- Ready, worm, clean, economic, sterile
- Less allergic
- Secures bonding between mom and baby, emotional satisfaction for the mother and
creates sense of security for the baby
- Help mom reduces weight, a method of contraception

 Problems with breast feeding that might happen to the mother:


- Sore nipple (clean & apply soothing lotion)
- Engorgement (pump)
- Mastitis (treat & do not stop feeding)

OSCE-guide-III.doc Page 207 of 255


Pediatrics

3- Advice:
- Mother should get enough nutrition, fluids, vitamins and rest.
- Give supplementations of:
o Vitamin D from day 1
o Iron from 4 – 6 months
o Start solid food from 4 – 6 months, I will give you a table with the recommended
time and types to start solid food
- Mother can use OCP but it will reduce amount of milk OR use an IUD
- Avoid using any medication without asking your Doctor
- Avoid smoking & alcohol
- Care of the breast: frequent cleaning with water and proper hygiene, warning signs:
engorgement, tenderness, redness, hotness
- I will give brochures & information about BF classes
- I will give you the immunization schedule so that you remember to bring him for follow-
up and for vaccination
- Do you have any questions or concerns?

4- Frequently asked questions about breast feeding:


 How often you should feed your child?
- On demand at the beginning
- Then the child will adapt to a schedule of every 3 – 4 hours

 How long should the baby stay on each breast? (10 minutes)

 How do you know that your baby is feeding well?


- Gaining weight
- Sleeping well
- Wetting 5 – 6 diapers daily
 What do you do if you do not have enough milk? (You can supplement with formula)

 How can you breast feed & work at the same time?
- Use pump & keep the milk in a bottle for 3 – 6 hours outside and 24 hours in a fridge,
you can keep it in the freezer

OSCE-guide-III.doc Page 208 of 255


Pediatrics

Psychiatry

OSCE-guide-III.doc Page 209 of 255


Pediatrics

Mental status exam – the psychiatry interview


ABS MAP TCIJ ALWAYS clarify! What do you mean by that?
Appearance Dressed Dishevelled: dressed and groomed
Groomed poorly
Given age matches his chronological age
Dressing matches the weather
Behaviour Psychomotor retardation vs. Psychomotor agitation
Eye contact *
Cooperative *
Hostile *
Abnormal movements / jerks / tics / lip smacking (anti-psychotics)
Speech Volume (low / normal / loud)
(process of Tone (parkinsonism: monotonous)
talking) Articulation
Fluent
Pressured speech
Mood How is your mood? How do you feel? Write it in patient “own words”
Affect Quality (by pt observation) Elevated
Euthymic
Depressed
Anxious
Congruency Appropriateness
Others Stable / range / labile / flat
Perception Normal perception
Illusions
Hallucinations
Thought Processing (how does the patient connect Goal directed
ideas); e.g. how did you come here today? Thought block
Flight of ideas
Loose association
Tangentiality
Circumstantiality
Contents Obsessions
Delusions
Suicidal / homicidal ideation
Cognition Mini mental exam
Insight
Judgement

Brief comment: (1) The patient is well dressed, well groomed; and his appearance matches his
chronological age. (2) He has (good / poor) eye contact, cooperative (not), with psychomotor …
(retardation / agitation) (3) His speech is of normal volume, tone, fluent, not slurred, and not
pressured. (4) His mood is … (5) His thoughts are organized (or disorganized). (6) There are no
delusions or hallucinations. (7) There is no suicidal ideation or homicidal thoughts. (8) Judgement
(good / poor), insight (intact / lost).

OSCE-guide-III.doc Page 210 of 255


Pediatrics

Perception
Hallucinations:
- Visual:
o Usually organic (tumour / epilepsy / cocaine and amphetamine)
o Brain tumour /+/ alcohol intoxication / DT /+/ cocaine / hallucinogens
o Do you see objects / things that others do not see?
o Can you describe what do you see?
o Do they give you any messages?
o Are these messages asking you to harm yourself or anyone else?
- Auditory:
o Usually schizophrenia
o Do you hear voices / things that other people do not hear? When you are alone,
do you hear voices coming from your head?
o How many voices
o Are they familiar or not?
o Are they talking to you or about you? What are they telling you?
o Did they ever ask you to harm yourself or somebody else? What is preventing
you from doing this?
o How do you feel about these voices?
- Tactile:
o Cocaine chronic use (most probably) OR delirium tremens
o Do you feel ants / insects crawl on your body / skin?
- Smell: usually epilepsy

Though
Processing:
o How did you come here today?

Content:
+ Obsessions:
- Repeated intrusive thoughts that the patient knows it is wrong, and he can not resist, if he
resists  ↑ anxiety  take actions to try to ↓ anxiety (compulsions)
- Mostly regarding: cleanliness, contamination / order / checking / …
o Do you have any repeated thoughts or images that you find difficult to resist?
About what? What do you do?

+ Suicidal / homicidal ideation:


o Do you have any thoughts or ideas of harming yourself?
o Or harming other people?
o Any access to weapons?
- If the patient is suicidal / homicidal / can not take care of himself  admit, if he/she
refuses  form 1 (for involuntarily admission – for 3 days – for psychiatric assessment
– by another physician). Form 1 has to be filled within 1 week from seeing the patient.
- I want to file form 1 for the patient and call the hospital security to bring the patient back.
I have concerns about safety of the patient and other people.

OSCE-guide-III.doc Page 211 of 255


Pediatrics

+ Delusions:
- False fixed believes, that do not match with the patient cultural and religious background
- You can not convince the patient it is wrong, even with proof
- The ideas
o Believable (could be) – non bizarre
o Unbelievable (could never be) – bizarre
o Do you believe that other people would like to harm you? OR conspire against
you?
o Do you think that others would like to control you?
o Read your mind? Thought broadcasting
o Put thoughts into your head? Thought insertion
o Steal thoughts from your head? Thought withdrawal
o If you are watching the TV or reading the newspaper, do you believe that they
are talking about you? Delusion of reference
o Do you believe that you are a special person? With a special talents? Or special
power? Do you believe that you have a special mission to do in life? Do you
think you deserve to be treated specially? Grandiosity
o Do you feel other people are falling in love with you? Eromantic
o Do you believe any part of your body is rotten?

Cognition:
- Are you becoming forgetful? Are you losing your staff?
- Assess abstract vs. concrete thinking!

Insight:
- Do you think that you are doing well? Or do you need help?

Judgement:
- If there is a fire in the building, what are you going to do?
- If you find a stamped and addressed envelop on the ground, near the mail box, what
would you do?

General screening:
- Depression:
o What is your mood? How do you feel?
o Did you lose interest in things that were interesting to you before (e.g. certain
hobby, playing something)?
- Anxiety:
o Are you the kind of person who worries too much?
o Do you have excessive fears or worries?
- Psychosis:
o Do you hear voices or see things that others do not?
o Do you think that someone else would like to hurt you?

OSCE-guide-III.doc Page 212 of 255


Pediatrics

DSM-IV-TR
Diagnostic and Statistical Manual of Mental Disorders 4th Ed/2000 – Text Revision
Multi-axial system (5 axes)
The DSM-IV organizes each psychiatric diagnosis into five dimensions (axes) relating to
different aspects of disorder or disability:
- Axis I: Clinical disorders, including major mental / psychiatric disorders, and learning
disorders, Substance Use Disorders
- Axis II: Personality disorders and intellectual disabilities (although developmental
disorders, such as Autism, were coded on Axis II in the previous edition, these disorders
are now included on Axis I)
- Axis III: Acute medical conditions and physical disorders
- Axis IV: Recent stressors, i.e. psychosocial and environmental factors contributing to the
disorder
- Axis V: Global Assessment of Functioning or Children's Global Assessment Scale for
children and teens under the age of 18 (a questionnaire)
Example of a full proper psychiatric diagnosis:
- Bipolar I / Anti-social personality / DM+HTN / Divorce / global assessment was not
done because the patient was not cooperative
Diagnosis of diseases based on DSM-IV-TR is based on CRITERIA and TIME.
- Depression:
o MI PASS ECG or MIS GE CAPS
o You need to find at least 5 of the 9 for > 2 weeks, including at least one of the
Mode or Interest.
o If not fulfilling these criteria: non-specified mood disorder
o In teenagers: we do not need M or I, we can replace it with agitation OR drop in
school performance + other 4 criteria.
- Schizophrenia:
o 4 positive symptoms: hallucinations, delusions, disorganized speech,
disorganized behaviour.
o 1 other category; negative symptoms: mood, catatonia …
o At least 1 month of active symptoms (2 of 5) + 6 months of deterioration in
functioning.
o 1 active symptom (not 2) is accepted in the following cases:
 If the hallucinations are > 2 voices (commanding or commenting)
 The delusions are bizarre
- Anxiety:
1- Panic attack vs. panic disorder:
a. In panic disorder, there is at least ONE panic attack with at least ONE month
of worries and fears of having it again
b. Panic attack might be one or more attacks
c. If patient is avoiding going outside  with agoraphobia
2- Phobias specific to certain objects
3- OCD
4- GAD: excessive unrealistic fears for more than 6 months PLUS other manifestations
5- PTSD (acute or chronic): Have you ever encountered a situation in which your
personal or mental safety and wellbeing were endangered? When? Do you have
flashbacks or nightmares?

OSCE-guide-III.doc Page 213 of 255


Pediatrics

History taking – Psychiatry

MOAPS: mood / organic / anxiety / psychosis / serious conditions (self care, suicide, homicide, support) / HEADSSS
Major psychiatric illness Suicide Minor psychiatric illness
Personality disorder / Drinking
S / addiction / Eating / sleeping
Mood Anxiety Psychosis A disorders / Somatic disorders /
D Cognitive (delirium /
dementia) ...
 Low: MI PASS ECG 1- Panic attack vs. panic Criteria (1 month of 2-5 active P Criteria
 High: DIG FAST + disorder symptoms + 6 month of E
MI PASS ECG 2- Specific phobias function deterioration) R
- 1st time or did you have it 3- OCD - 1st time or did you have it S
before? 4- GAD before? O
- What about the opposite? 5- PTSD N
Dx: one of the mood disorders S
Past psychiatric history Past psychiatric history
Organic: MOAPS
1- SAD if IV drug use: check for liver (hepatitis) / constitutional symptoms (HIV)
2- PMH, including constitutional symptoms
3- Rule out medical conditions as DD, e.g. medications and specific diseases
Thyroid disease Mitral valve prolapse Brain tumour / HIV
Anxiety / psychosis Mood / psychosis Mood / anxiety
Serious conditions (red flags):
- Self care (are you eating / sleeping well?)
- Suicidal / homicidal ideation
Social history:
- How do you support yourself financially?
- With whom do you live? Family support?
For teenagers, add: HEADSSS
Family history of psychiatric illness: suicide / depression / SAD / seen by psychiatrist

OSCE-guide-III.doc Page 214 of 255


Psychiatry

Mood disorders:

OSCE-guide-III.doc Page 215 of 255


Psychiatry

Psychosis
Pt comes to the clinic complaining of strange feelings in his right hand

Clarify the CC 1- Can you tell me what is going on!


OSCD 2- What do you mean? Is it pain, numbness, tingling?
3- OS: When did it start? What were you doing?
4- C: Is it all the time, or on and off? Any specific setting?
5- In you opinion, why do you have this?
6- In addition to …, do you have any other strange feelings?
Criteria :  Any hallucinations: What? For how long?
 Hallucinations  Any (tactile or visual) hallucinations  will be mostly organic
 Delusions  cocaine (substance abuse) until proven otherwise
1st time or did you have it
before?
Past psychiatric history Have you seen a psychiatrist before?
Organic Cover the following:
 Head injury / trauma
 Brain infection (fever, vomiting) / Brain tumour (N/V)
 HIV
 Thyrotoxicosis
 Medications: thyroxin, anti-parkinsonism (L-dopa)
 SAD (cocaine, marijuana, amphetamines)
Mood / Anxiety
Serious conditions Constitutional symptoms / self care / suicide
Social history
Family history

Differential diagnosis:
- Schizophrenia (a mental disorder that impairs the way you perceive reality. It could be
very disabling)
- Brief psychotic disorder
- Post-partum psychosis
- Drug-induced
- Brain tumour
- HIV
- Delirium / dementia
- Mood disorder

Investigations:
- CBC / toxicology screen
- HIV / syphilis test
- Septic workup
- CT / MRI brain

Pay attention to patient cues


- Poor hygiene
- Looking at wall or ceiling
- Paranoid
- Talking to some body.

OSCE-guide-III.doc Page 216 of 255


Psychiatry

Management:
─ Will examine and do some tests
o Because you have stopped your medications, it looks like your schizophrenia is
relapsing, that is why we need to admit you and refer you to psychiatrist to
reassess your condition, how do you feel about that?
─ Will start medication which is helpful in reducing the symptoms (Risperidone). Explain
about side effects: weight gain / ↑ blood glucose level / ↑ cholesterol / drowsiness
─ Arrange follow up visit
─ Information e.g.: support groups / brochures

Notes:
─ Whenever you suspect substance abuse: after you ask “have you ever tried recreational
drugs?” ask “what about crack cocaine? Do you sniff? Do you inject? Did you share
needles”
o If shared needles  scan for hepatitis (liver symptoms), HIV (repeated
infections / repeated diarrhea)
─ If the patient came because his parents or roommate have concerns, you can ask the
patient: what kind of concerns does … have?

Difficult situations:
─ If the patient with hallucinations tells you that he sees a radiation and gives you a photo and asks:
do you see it doctor?  For me it does not look like radiation, but I can understand that you see
this as radiation
─ At any time the patient starts to agitate and worries about special hallucinations!
o You are safe here, no body will harm/hurt you
─ If the patient is away:
o Do not chase him/her around the room, stand by your chair
o I would like to assure you that you are safe here, no one will harm you
─ I do not like “Egyptian people”, by the way, are you Egyptian doctor?
o Why are you concerned about that?
o Whether I am Egyptian or not will make no difference in this situation
─ I do not like “gays”, by the way, did you see a gay patient today doctor?
o Why are you concerned about that?
o As a physician, I deal with all patients, regardless their race, religion, sex, sexual
orientation or anything else!
─ Do you think I am crazy doctor?
o There is no medical term called “crazy”. However sometimes some people have
difficulties in the way they handle their thoughts and the way they interact with and
perceive reality, we call that schizophrenia. It is a mental illness like any other illness that
can affect the body, that we can treat with medications

Case: A young man can not move his neck, DD acute dystonia:
─ Trauma
─ Meningitis
─ Subarachnoid hemorrhage
─ Cervical disc
─ Muscle spasm
─ Anti-psychotic medication (e.g. haloperidol), treatment: lorazepam

Case: Patient is in the balcony, wants to fly, mother is calling you?! What do you tell her?
─ Talk to him to attract his attention
─ The doctor should get the phone number and address and ask the nurse to call 911
─ Ask her if you can speak with the patient  psychosis patient

OSCE-guide-III.doc Page 217 of 255


Psychiatry

Schizotypal personality disorder

- Delusions
- Magical believes
- Limited number of friends that share the same believes

Ethical challenges:
- Will you hook me to the cleaning machine that cleans the blood? I am glad you came
here today, I think you need help, but not with the machine.
- Will you admit me doctor? We need further psychiatrist assessment then we may need to
admit you.

OSCE-guide-III.doc Page 218 of 255


Psychiatry

Panic attack

Patient comes to the clinic complaining of dizziness

Clarify the CC When you say dizziness, do you feel:


- light headed
- Spinning
Analysis of CC Os Cf D
Analyze the attack: How did it end? How many attacks? Are they similar?
What were you doing? When was your last one?
HPI Criteria  ↑ cardiac: heart racing17 / chest pain / tightness / excessive sweating
during  ↑ respiratory: SOB / wheezing
attack  ↑ neurology: dizziness / numbness / tingling / weakness / shaky /
buzzing sounds / headache / vision changes / difficulty balance
(AS)  ↑ GIT: nausea / vomiting / difficulty swallowing
 Depersonalization: you feel that you are outside of your body
 Derealisation: feel things around you are strange / not real
 Excessive fears of: losing control / going crazy / dying
Between  Do you have fears of having other attacks?
attacks  How does it affect you? Do you avoid going out? (Relation to
agoraphobia?)
Anxiety  Are you the kind of person who worries a lot? Excessive fear
 Are you under any stress in your life? How can you cope with this?
 Any special fears? High altitudes? Closed places? Talking in public?
Pets?
 Have you ever encountered a situation in which your personal or mental
safety and wellbeing were endangered? When? Do you have flashbacks
or nightmares?
MOAPS  Hypoglycemia
 Thyroid disease
 Pheochromocytoma
 SAD (cocaine / amphetamine / alcohol withdrawal) / Caffeine
 Arrhythmias / MVP
 Anemia (fatigue / light headedness / heavy menses / PMH anemia)
Past psychiatric
history
Serious conditions  Constitutional symptoms
 Self care
 Suicide
PMH AMPLE
Family history Heart diseases / thyroid / abdominal tumours
Social history

17
Any heart racing, ask the patient “can you tap it for me”, then comment to the examiner: “it
looks regular / irregular for me”

OSCE-guide-III.doc Page 219 of 255


Psychiatry

COUNSELLING

- With what I heard from you today, the most likely diagnosis to your symptoms is a
medical condition that we call “panic attack”. We still need to do physical examination,
some investigations like blood works, urine analysis, electrical tracing of your heart
(ECG), to exclude other medical conditions and to confirm our diagnosis.
o Now Mr … what do you know about “panic attacks”?
o Do you want me to explain this in details over the next few minutes?
- Inform the patient:
o Explain the pathophysiology: panic attack or panic disorder is a kind of severe
anxiety, it happens suddenly, in attacks. Usually it is related to stress.
o It is due sympathetic over-activity, imagine you are crossing the road, and a
speedy car is approaching you, normally, our body reacts to this by enhancing the
sympathetic nervous system, which leads to some changes: increase in the heart
rate, rise in blood pressure, and you feel alert. This is normal and useful reaction.
o The same reaction might happen suddenly without any external trigger, and this
would be stressful, and this is what we call a “panic attack”.
o Consequences: this might happen again / may cause significant limitations
- Preventive measure:
o Life style modification (↓caffeine and alcohol / better sleep hygiene)
o Relaxation techniques (e.g. breathing techniques / meditation)
- Treatment:
o Like many other conditions, it could be treated.
o Treatment varieties include:
 Talk therapy
 Medications: 2 types
• Anti-anxiety: Lorazepam 0.5 mg qhs x 2 weeks (it is important
to use it on schedule, not irregularly)
• SSRIs: Paroxetine 10 mg od x 4 weeks – similar to what we
usually use with depression. Like any other medication, they
have their side effects; GIT disturbances, headache, some sexual
dysfunction. And this improves by time.
• Follow-up 2-3 weeks
- Offer more information: brochures / web sites
- Whenever you suspect social problems  involve the social workers

Generalized Anxiety Disorder (GAD)


 Excessive anxiety and worry (apprehensive expectation), occurring more days than not for at
least 6 months, about a number of events or activities (e.g. work / school)
 The person finds it difficult to control the worry
 Treatment:
o Lifestyle: caffeine and alcohol avoidance, sleep hygiene
o Psychological: psychotherapy, relaxation, mindfulness, and CBT
o Pharmacological:
 Benzodiazepines (short term, low dose, regular schedule, long half-life,
no PRN)
o Buspirone (tid dosing)
o Others: SSRIs/SNRI (paroxetine), TCAs (nortriptyline), beta-blockers
o Avoid Bupropion due to stimulating effects

OSCE-guide-III.doc Page 220 of 255


Psychiatry

Tiredness OR weight loss

Introduction
CC Tiredness
Clarify the CC - Is it weakness? Can not do?
- Lack of energy? Tiredness?
- Limitation of activity? How many
blocks are you able to walk?
- Not being refreshed after sleep? Do
you have any special concerns?
Os Cf D Timing:
- Morning or all day: ?depression
- End of the day: organic
Ask about sleep - How many hours? And before?
- Find difficulty falling asleep?
- Do you wake up during night?
- When you wake up, do you feel
refreshed? Do you need naps?

↑ sleep /or/ the same ↓ sleep / insomnia


? organic ? depression
- Constitutional symptoms Criteria : MI PASS ECG
- Tender points in your body
All systems review (head to toe): 1st time or did you have it before?
- Cardiovascular What about the opposite?
- Lung Past psychiatric history
- GIT / liver Organic …/…/…
- Urinary Anxiety / psychosis
- MSK / skin / rheumatology / auto- Serious conditions
immune Social history
- Anemia / bleeding Family history
± LMP / mens. hx / pregnancy
Cancer colon for males
- Endocrine (thyroid / DM) Counselling on depression
MI – mood / interest
PMH of cancer
Social history – SAD
Family history

Diabetes Mellitus:
- Hx of DM - Blood sugar measured - Symptoms:
Fluctuations (acute) Emergencies Complications (chronic) vascular
MICRO MACRO
- Eat more - Blurred vision DKA - Nephropathy - CAD
- Drink more - Tired Hypoglycemia - Neuropathy - CVS screen
- Pee more - Weight loss - Retinopathy - PAD /
impotence
N.B. β-blockers are contraindicated in DM: it causes hyperglycemia / and it masks hypoglycemia

OSCE-guide-III.doc Page 221 of 255


Psychiatry

Sleep / fatigue notes

1- Sleep hygiene questionnaire:


- How many hours? How about before?
- How does this affect your life? Do you work night shifts? empathy
Before - When do you go to bed?
- Find difficulty falling asleep? How long does it take you?
- Do you sleep in dark room? Lit room?
- Do you eat before sleeping? Heavy meals? Late meals?
- Do you drink before sleep? Alcohols? Coffee?
- Do you exercise before sleep?
- Do you read in bed? Watch TV?
During - Do you wake up during night? Any reason? Can you sleep again?
- Do you sleep alone? Or do you have sleep partner?
o Does he notice you are snoring? Do jerky movements?
o Does he snore? Does he do jerky movements?
- Do you have dreams? Nightmares?
After - Do you wake up early?
- When you wake up, do you feel non-refreshed? Do you need naps?
- Do you work on shifts?

2- Depression:
 Psychomotor question: do you think things take more time to do now? Compared
to before?
 Pancreatic cancer  depression
 Whenever you find alcoholic patient  check for complications:
i. Cancer pancreas
ii. Liver damage (↑ liver enzymes) / hepatitis / cirrhosis / carcinoma
iii. GIT: upper GIT bleeding / peptic ulcer perforation
iv. Depression (alcohol / depression / suicide) is common combination
 Treatment for depression (or most of the psychiatric diseases):
i. Life style modification
ii. Talk therapy
iii. Medications
Usually in combination

3- Domestic abuse presentations 4- Fatigue


- Headache - Depression / PTSD
- Abdominal pain - Domestic abuse
- Insomnia / sleeping pills - Hypothyroidism
- Vaginal bleeding - Fibromyalgia
- Fatigue - Anemia:
- Old person: think cancer & occult blood
- Young female: think menorrhagia
- Diabetes Mellitus, polyuria
- Anorexia nervosa

OSCE-guide-III.doc Page 222 of 255


Psychiatry

Insomnia

A lady complaining of insomnia


Common presentation to: domestic abuse / depression / anxiety

Introduction
CC Insomnia / Tiredness
Clarify the CC - Difficulty falling sleep
- Waking up
Analysis CC: Os Cf D - More at certain time of the week?
- Did you try anything to help? Did it work?
Ask about sleep Sleep hygiene questionnaire
Anxiety - Do you have too many worries?
- What comes in your mind before falling asleep?
- Any changes / stresses in your life?
- Do you wake up with nightmares?
Depression - Screen with MI; if positive  screen MI PASS ECG
PMH
Social - With whom do you live? Support?
 Screen for domestic violence or spouse abuse
- Children?
- Financial support?

Notes
 Did you ever think to hurt yourself? NO, my kids need me,
o What about if they are not around? Maybe!
o This means: implicit yes to suicidal ideation

 Lady looking for renewal of benzodiazepines:


o Renew it and tell her that she needs to use it properly
o Tell her that she is in grief, she needs SSRIs, start SSRIs, taper benzodiazepines

OSCE-guide-III.doc Page 223 of 255


Psychiatry

Domestic Violence – Spouse Abuse

Screen for domestic violence or spouse abuse:

- ASSURE confidentiality: I would like to assure you that our conversation is completely
confidential, whatever you will tell me here, I will not release any information, unless
otherwise required by the law!
- With whom do you live? How do you describe this relationship? Supportive?
o How long have you been in this relation?
o Do you feel safe at home? In this relationship?
- Do you or your partner go through stressful times?
o Do you sometimes have conflicts? Arguments?
- Is there any chance that you partner drinks or uses drugs? How often? When he drinks,
does he become angry? Lose control? When was the last time?

Verbal / emotional:
- Does he start to shout at you? Swear at you?
- Does he call you names? How does this affect your self-esteem?
Physical:
- Did he ever get angry to the extent that he became physical?
- Did he try to put you down? Does he try to control you? How did this affect you?
- Did he try to push you? Hit you? How many times?
- Any visits to the ER? When was the last time?
Financial:
- Who is controlling the spending at home?
- Do you have access to financials? Do you take permission?
- Did he ever to try to take you money against your wishes?
Sexual:
- Did he ever force you to do sexual activity against your will? How do you feel?

- Duration of abuse? Severity? Hospitalized!


- How does this affect you? Are you pregnant now? Do you work now? How do you
support yourself financially?

Children involvement:
- Did he ever mistreat / abuse you in front of the children?
- Did he ever mistreat / abuse the children?

Fatality:
- Do you have access to weapons at home?
- Did you ever have thoughts to put an end to this all by ending your life or his life?
- Did you ever talk to anyone about this?

OUTCOME:
- The patient decides to end the relationship and leave  you must provide support and
shelter
- The patient decides to continue: either with OR without police involvement

OSCE-guide-III.doc Page 224 of 255


Psychiatry

Wrap-up:
- Based on what you have told me, what you are experiencing (or have gone through) is
called domestic violence or spouse abuse, and it is common. It is an illegal crime, and it
is against the law.
- It is not your mistake, and you should not feel guilty about that. It is unacceptable, and
nobody deserves to be treated in this way.
- We know from studies that the situation will not improve, on the contrary, it will
deteriorate, and you do not need to accept this. The studies show that the longer you stay
in this relationship, the higher the chance of abuse.
- Consequence to the children (if any): psychological trauma
- It is important that you consider reporting the situation to the police for your safety. It is
difficult decision to leave or stay.
- The husband needs help, check willingness to get counsel.

- If you like to end the relationship and leave:


o You can call the police, they will come and arrest him, then they will investigate
the case, and may put charges against him, and you do not need to worry that he
might hurt you, the police can give a restraining order
o I will connect you with the social services and support groups, you do not need to
go through all of this by yourself, they will be able to help with housing,
financial support for both of you and the kids
- However, if you want to continue the relationship, you can still involve the police. We
know from the studies that if you involve the police, situation will improve, they will
come, speak with him, do some investigations, and then, by law, your husband will be
obliged to attend special training courses:
o Anger control
o Stress management and relaxation techniques
o Drinking problem rehabilitation
o Marital counselling
- My concern is that if you go back without taking any measures, things might deteriorate
and get out of control and one of you might end up losing her/his life, at least:
o You need to prepare an escape plan:
 Where to go when you don’t feel safe!
 A bag with essential belongings (e.g. IDs, bank documents, …)
 Easily accessible (you can pick it in second and leave)
o We will schedule a follow-up visit within few days
- Is it ok with you if I document this?
- Support:
o Police is not the only option, in the city, there are community recourses: shelter,
hotline, legal aid, they are free, and confidential.
o Give social support group numbers.

Clues for domestic abuse:


- She is avoiding eye contact
- Not answering directly for questions about the relationship with partner
- I hate drinking
- He is a great father, likes his daughters, and works very hard.
- I wish I can be a better mom / I am a failure / do you think too I am clumsy.

OSCE-guide-III.doc Page 225 of 255


Psychiatry

Child Abuse18

The child came to the ER with femur fracture, the skeletal survey showed multiple healing
fractures, counsel

Introduction … I assure you that he is ok, and after we finish I would accompany
you to see him, is it ok with you. Before this I would like to ask you
some questions to know more about his condition / fracture
Analyze the event - Can you describe what happened? What he was doing?
- Who witnessed it? Anybody else?
- When did this happen? When did you come to the ER?
Is it the first time? - Other injuries before or visits to ER?
- Analyze each event
- Did you take him to the same hospital?
BINDE  Are you the biological mother?
 Is your current partner the biological father?
- B: screen for the risk factors for child abuse:
 Was this pregnancy planned? Regular f/u visits?
 Was he a term baby? Did he need special attention?
 Has he had congenital anomalies?
 Do you think he is a difficult baby? Fussy baby?
 SAD for both partners!
- I: Are his shots up-to-date? If no, any reason?
- N: What is his weight? Do you know about his growth charts?
Regular f/u visits?
- D: Is he hyperactive baby? Challenges you most of time?
- E:
 How do you support yourself financially? Any support from
the biological father?
 Anybody at home seeing a psychiatrist? Illness?
 Tell me more about your childhood
Other children - Do you have other children?
- Repeated visits to ER?
PMH of the child Chronic illness / bone or metabolic diseases

Screen for domestic violence or spouse abuse

18
Good TWO screening questions: immunization (not up-to-date) / weight (FTT or under
nutrition)

OSCE-guide-III.doc Page 226 of 255


Psychiatry

Wrap-up:

- How do you feel your child has so many fractures? …


- I know that you are concerned about your son, sometimes it is challenging to look after a
child, especially if you do not have enough support.
- In the same time, children at that age have flexible bones, and it is difficult to explain the
nature of his fracture(s) only by jumping from a couch.
- In these situations we usually involve the children aid society (CAS), this is a kind of
social services devoted to the safety and well being of children
o Please do not do this? Why?
o They will take my son! Why r u saying so? Any experience with them?
o Not necessarily that they take your child, this is not their first priority
- Their first concern is the safety of your child, what will happen is that they will:
o Ask you some questions about what happened!
o Come to visit you at home
o Talk with your partner
Then they will take their next step based on the results of these meetings
- I am sure you are sharing my concerns about … (the child name) safety!

If there is spouse abuse / domestic violence:


- Also, based on what you have told me, what you are experiencing (or have gone through)
is called domestic violence or spouse abuse, and it is common. It is an illegal crime, and
it is against the law.
- It is not your mistake, and you should not feel guilty about that. It is unacceptable, and
nobody deserves to be treated in this way.
- We know from studies that the situation will not improve, on the contrary, it will
deteriorate, and you do not need to accept this. The studies show that the longer you stay
in this relationship, the higher the chance of abuse.
- It is important that you consider reporting the situation to the police for your safety.

OSCE-guide-III.doc Page 227 of 255


Psychiatry

Domestic abuser

You are bout to see a 55/60 years old gentleman, whose wife is recovering in the ER, she has
bruises, and he asked to speak with you. In the next 10 minutes counsel him

Introduction
Analysis
SH / Safety
Counsel  Domestic violence

 Anger control
 Stress management and relaxation techniques
 Drinking problem rehabilitation
 Marital counselling

 Offer social support if there is a need

Introduction:
 If the patient asked to see you: I understand that you are here because you are accompanying
your wife, she has bruises and my colleagues are taking care of her right now. How can I help
you today?
 If the patient is inquiring about her status: I can assure you that she is stable and in safe hands
now.
 If the patient asks to see her: After we will finish, I will ask her, if that is ok with her, I can
take you there.

Analysis:
 Do you have any idea how did she end up having all these bruises?
 Was there any argument / disagreement / shouting? Did you lose control? Did it end up that
you physically hurt her?
 Is this the first time or happened before? Any repeated visits to the ER before?

Social history:
 How long have you been together? What is the nature of your relationship? Stable? Was there
and significant conflicts before?
 Was there any recent change or stressor in your life? How do you support yourselves
financially? Do you have enough resources?
 Do you have anybody else at home? Any family support? Do you have children? How is the
relation with them?
 SAD

Safety:
 Criminal record / access to weapons at home
 If you go home now and face the same situation, how would you react?
 Any chance that you might hurt yourself or any other one?

OSCE-guide-III.doc Page 228 of 255


Psychiatry

Counselling:

 I can see that you are going through stressful period of time. It must be difficult for you and
your wife. Sometimes this stress might present by changes in behaviour and/or personality.
 If you do not have enough support at home, things might get out of control.
 What happened is what we call “domestic violence”; it is a kind of “physical abuse”. It is not
acceptable, and it is considered illegal crime. However, this is your wife decision. If she
chooses to report you, that is her right, and nobody can prevent her. She can press charges
against you, and they will take you to the court, in this case you might need legal help, this
might have serious consequences.

 On the other hand, if she decides not to take any measure, may be you should try to improve
the situation by taking steps to decrease the stress in your life, and you can consider reducing
your alcohol drinking. Drinking alcohol might leads to what we call “disinhibition” in which
one might lose control on his reactions and usually this leads to violent and serious
consequences.
 I can help you by referring you to attend:
o Alcohol rehabilitation programs
o Stress management and anger control programs

 I recommend also that you consider attending family marital counselling; they have good
experience in dealing with couples going through difficult times.

 Finally, I can help you to contact the social services. They might be able to help; you can
speak with them and see what they might be able to do! Is that ok with you?

OSCE-guide-III.doc Page 229 of 255


Psychiatry

Depression
Screen: MI PASS ECG
Organic:
 Illness: hypothyroid, anemia / pernicious anemia, M.S, cancer / cancer pancreas
 Medication B Blockers, Anti-parkinsonian
 SAD

Depression management / counselling


7. Inform the patient
a. Based on what you have told me, the most likely explanation for your condition
is a medical condition called “depression”, what do you know about depression?
Did you read anything about it? It is the most common mood disorder, in which
you feel low, upset and lack of energy. It is a common problem, and it is
treatable.
b. Explain the pathophysiology: it is related to imbalance of the chemicals in our
brain, most likely related to decreased serotonin
c. Consequences / complications of the condition: it affects functionality, leads to
decreased concentration and ability to work, and in severe cases in susceptible
persons, it might lead them to suicide
8. Treatment (outpatient):
a. Talk therapy (psychologist / psychiatrist)
b. Medications: SSRIs (which are very effective medications) take effect after few
weeks  Cipralex 10 mg PO od x 3 weeks (side effects include: 1. weight gain,
2. GIT symptoms, 3. sexual dysfunction). These side effects improve by time and
to reduce it, we start increasing the dose gradually (start low, go slow).
 Because depression has serious (fatal) consequences, if you do not take this
medication, you will be compromising your safety.
 DO NOT stop it on your own; we can start to gradually decrease the dose
after the proper period of treatment. You need to continue on it for at least 6
months after symptoms improve.
c. Follow up visit after 2-3 weeks
d. Contract: sometimes when the anti-depressant starts to work, the energy level
improves while the mood is still low, that is why sometimes there is increase in
suicidal ideation. Usually happens 2-3 weeks, you need to promise me that if this
happens with you, call 911 or call me immediately and come to see me
9. Offer more info: brochures / web sites
10. Break every 30-60 seconds and ask the patient: does that make sense? Is this acceptable?
Reasonable? Is it clear?

11. Treatment (hospitalization): for suicidal patients


a. From what you have told me, you are meeting the criteria of what we call “…”
and I have concerns about your safety, because you have more than THREE risk
factors for suicide as per the screening test. Do you mind to stay with us in the
hospital for few days, so we can do the required investigations and start the
medications, until you feel ok, what do you think about that?
─ No doctor, I am not staying in the hospital!!!
─ Actually, Mr … as I told you, I have concerns about your safety, we can not
compromise your safety. And by allowing you to leave today, we will be
compromising your safety.

OSCE-guide-III.doc Page 230 of 255


Psychiatry

Dysthymia
COMMON CASE IN THE EXAM

 Depression presentations:
o Sad (low mode), weight loss, insomnia, tired
 Scale the sadness 0 – 10
o Indecisiveness: difficulty making decisions
o Low self esteem  how do you feel about yourself?
o If good days: ask for periods (check for gaps ≤ 2 months)

 How does it affect your life?


o Then assess functionality; what do you do?!

 Screen MI:
o If positive  MI PASS ECG
 If positive  assess SAD PERSONS

 Any relation to menstrual periods  pre-menstrual dysphoric disorder

 Counselling: similar to depression, but mention that Dysthymia is a milder form of


depression, with longer duration, and does not interfere with life functionality.

OSCE-guide-III.doc Page 231 of 255


Psychiatry

Premenstrual Dysphoric Disorder (PMDD)

DSM-IV-TR Diagnostic Criteria for Premenstrual Dysphoric Disorder


A. In most menstrual cycles during the past year, five (or more) of the following symptoms were
present for most of the time during the last week of the luteal phase, began to remit within a
few days after the onset of the follicular phase, and were absent in the week post-menses,
with at least one of the symptoms being one of the first four listed
1. Markedly depressed mood. Feelings of hopelessness, or self-deprecating thoughts
2. Marked anxiety, tension, feeling of being "keyed up" or "on edge»
3. Marked affective lability
4. Persistent and marked anger, irritability, or increased interpersonal conflicts
5. Decreased interest in usual activities
6. Difficulty concentrating
7. Lethargy, easily fatigued, lack of energy
8. Change in appetite – overeating or specific food cravings
9. Hypersomnia or insomnia
10. A sense of being overwhelmed or out of control
11. Physical symptoms – breast tenderness or swelling, headaches, joint or muscle pain,
sensation of bloating or weight gain

B. The disturbance markedly interferes with work, school, social activities or relationships with
others
C. The disturbance is not merely an exacerbation of the symptoms of another disorder such as
Major Depressive Disorder, Panic Disorder, Dysthymic Disorder or Personality Disorder
D. Criteria A, B and C must be confirmed by prospective daily recordings and/or ratings during
at least two consecutive symptomatic cycles (how to diagnose)

Treatment
 1st line: SSRIs highly effective in treating PMDD
o Fluoxetine (20 mg od) and sertraline (50 mg od) most studied
o Can be used intermittently in luteal phase (mid cycle  onset of menstruation –
pre-menstrual) for 14 days
 2nd line
o Alpraxolam (Xanax) for anxiety symptoms
 3rd line
o ± OCP containing progesterone drospirenone (e.g. Yasmin)
o GnRH agonists (e.g. leuprolide)
o If GnRH agonist completely relieves symptoms, may consider definitive surgery
(i.e. Total abdominal hysterectomy+ bilateral salpingo-oophorectomy)

OSCE-guide-III.doc Page 232 of 255


Psychiatry

Abdominal Pain / Headache


Abdominal pain for few weeks, and was seen by a surgeon last week, comes to your office (± to
have MRI OR to renew medication).
Headache for 7 months, young man, comes to renew Tylenol 3

Chief complain ± a request (investigations OR medication renewal)

HPI Analyze  Os Cf D / PQRST / ↑↓ / 1st time


the CC  When did the headache (pain) start?
 Did you seek medical attention? What was the diagnosis? Did you take
any medication? When did you start Tylenol 3? Why?
 Analyze previous visits: is the pain different from before? How?
AS  Other pains / headache
 GIT / liver
 Genito / urinary
Impact  How does this affect your life? How are you coping?
Red flags  Constitutional symptoms
 Screen red flags for headache:
 Trauma
 Worse at night
 Nausea / vomiting
 Bothered by light /+/ Neck pain / stiffness
 Weakness / numbness / tingling in body / seizures
 Are you under stress?
 Support systems
DD
Medical problem Somatisation
MOAPS screening (screen for depression)
PMH PMH: HEAD SSS
FH FH of psychiatric disease
SH SH
Physical examination Counselling
Notes:
- Somatisation disorder: (4 pains / 2 GIT / 1 neuro / 1 sexual) complains
- If the pain is only during the day, and not nights  mostly non-organic

+ If requesting MRI  NO MRI


+ Actions for Tylenol 3:
- If using it for few weeks  stop it / do not worry about withdrawal symptoms
- If using it for long time  counsel / renew / promise to cut down gradually
- If using it for depression  start SSRIs / taper Tylenol 3 (decrease gradually) / then brief
counselling for BOTH Tylenol 3 & depression
- If drug seeker  DO NOT give any narcotics / rehabilitation
+ Always renew the medication, except for drug seeker (very anxious to renew the narcotic / will
not accept another alternative / making stories to rash you to prescribe it).
+ If not drug seeker: reassure the patient that you will prescribe her pain medication before the
end of the session, but in order to prescribe the proper medication, you need to ask some
questions, can you bear with me for few minutes?

OSCE-guide-III.doc Page 233 of 255


Psychiatry

Somatoform disorders DD

General Characteristics:
 Physical signs and symptoms lacking a known medical basis in the presence of psychological factors
 Cause significant distress or impairment in functioning
 Symptoms are produced unconsciously
 Symptoms are not the result of malingering or factitious disorder which are under conscious control
 Primary gain: somatic symptom represents a symbolic resolution of an unconscious psychological
conflict; serves to reduce anxiety and conflict; no external incentive
 Secondary gain: the sick role; external benefits obtained or unpleasant duties avoided (e.g. work)
Management of Somatoform Disorders:
 Brief frequent visits
 Limit number of physicians involved in care
 Focus on psychosocial not physical symptoms
 Minimize medical investigations; co-ordinate necessary investigations
 Biofeedback
 Psychotherapy: conflict resolution
 Minimize psychotropic drugs: anxiolytics in short term only, antidepressants for depressive symptoms

Somatization disorder ─ Lots of symptoms: ≥ 8 physical symptoms that have no organic


pathology: 4 pain + 2 GIT + 1 sexual + 1 pseudo-neurology
Conversion disorder ─ One or more symptoms or deficits affecting voluntary motor or
sensory function that mimic a neurological or general medical
condition (e.g. impaired co-ordination, local paralysis, double vision,
seizures or convulsions
─ La belle indifference
Pain disorder ─ Pain is primary symptom and is of sufficient severity to warrant
medical attention
─ Post-traumatic / post-surgical
─ Exacerbated by psychic factors
Hypochondriasis ─ Preoccupation with fear of having, or the idea that one has, a serious
disease (e.g. brain tumour) based on a misinterpretation of one or
more bodily signs or symptoms

Fibromyalgia ─ Onset often after car accident


─ Wake from sleep feeling un-refreshed
─ Wide spread pain, above and below waist, both sides of the body
─ Characteristic reproducible tender points: occiput, low cervical C5-
C7, lateral border of the sternum, post neck, lateral epicondyle 2cm
below that point, lat thigh, med knee
─ Treatment:
 Patient education
 Exercise program (walking, aquatic exercises), physical therapy
(good posture, stretching, muscle strengthening, massage)
 Stress reduction, CBT
 Amitriptyline 10 – 25 mg qhs
 Gabapentin 300 mg tid
Chronic fatigue ─ Similar to fibromyalgia but FATIGUE is the predominant feature
syndrome ─ Associated with sleep apnea / irritable bowel syndrome
Factitious disorder /
malingering

OSCE-guide-III.doc Page 234 of 255


Psychiatry

Counselling for somatisation disorder:


- I understand that you are here because of …, and to (renew medication / do MRI / …)
and we will discuss that, but before discussing this, I would like to explain the findings in
your case.
- Based on the symptoms (± and the surgeries you had) the most likely explanation to your
pain (headache) is a medical condition called “somatisation disorder”.
- What do you know about “somatisation disorder”? Would like me to explain?
- It is not uncommon condition, and we do not know the exact explanation for it, but we
believe that because some patients are more sensitive to pain than others, or may be due
to patients difficulty in handling stresses in their lives, these stresses may manifest as
painful experiences (symptoms).
- Do you have a family doctor?
 YES
o I will explain some points for you now, and then you can arrange a meeting with
your family physician and discuss the follow up with him, in these situations, it is
important to have only one physician dealing with all the investigations so that he
can get better understanding of the whole situation.
 NO, I do not have a family physician!
o I can be your family physician, if you would like to. That means we will set a
follow-up visits every 4 weeks, during which we will review underlying
symptoms, to make sure we are not missing any serious condition.
o We will review the stress in your life and see how we can help you with it:
 I can refer you to psychiatrist to help you deal better with any stress /
conflict in your life
 And we can consider some medications (Amitriptyline 25 mg PO qhs),
it belongs to a family of medications called TCA (tri-cyclic anti-
depressants) but we use it for pain control

Counselling if the patient is depressed:


- I understand that you are here because of …, and to renew medication and we will
discuss that, but before that, I would like to explain the findings in your case.
- Based on what you have told me, the most likely explanation to your symptoms is a
medical condition called “depression”.
- We need to treat the depression with [talk therapy, behavioural modifications, and
medications (SSRIs, TCA)]  depression counselling
- For Tylenol 3 we will not stop it suddenly, I will renew it for you, but we will agree that
you will gradually cut it down, over the next few days, till the other medication (SSRIs)
kicks in.

If there was a suicidal attempt:


- However, because of the suicidal attempt 2 days ago, we would like you to stay with us
in the hospital for few days so that we can start the treatment

OSCE-guide-III.doc Page 235 of 255


Psychiatry

Drug seeker

If you find a man searching in the drawers of the hospital, firmly ask him to stop, tell him this is private
property and he is not allowed to go through this medical stuff

I wish it could be that simple, but I need more information and physical exam before I can write any
prescriptions to you, as I am a little bit concerned about the amount you have been taking, which might
have been harmful to you

Introduction  Why are you taking it? What was the diagnosis?
HPI Analyze  Os Cf D / PQRST / ↑↓ / 1st time
the CC  When did the headache (pain) start?
 Did you seek medical attention? What was the diagnosis? Did you take
any medication? When did you start Tylenol 3? Why?
 Analyze previous visits: is the pain different from before? How?
AS  Other pains / headache
 GIT / liver
 Genito / urinary
Impact  How does this headache affect your life? How are you coping?
 Have the medications been impacting your life?
 Relationship with family
 Education, Employment
 Legal problems, police involvement?
Red flags  Constitutional symptoms
 Screen red flags for headache:
 Trauma
 Worse at night
 Nausea / vomiting
 Bothered by light /+/ Neck pain / stiffness
 Weakness / numbness / tingling in body / seizures
 Are you under stress?
 Support systems
Analyze Tylenol 3
Other medications In addition to Tylenol 3, do you take any other meds? Sleeping pills?
MOAPS screening
PMH: HEAD SSS
FH of psychiatric disease
SH
Counselling
Analyze Tylenol 3
- So you told me you are taking it for …
- Who prescribed it to you?
- Who renewed it to you? Why?
- When was the last renewal? Can you show me your last bottle?
- How many tablets do you use now? And before? When did you start to ↑ the use?
- When you take it, beside for the headache relief, how do you feel? What if you do not take it, how do
you feel? Shaking? Heart racing? You feel you are on the edge?
- Do you renew it from the same doctor or different doctors? Why you did not go to him this time? Is it
ok that I contact him?
- Do you renew it from the same pharmacy or different pharmacies? Is it ok that I contact the pharmacy?
- Did you ever obtain the medication from the street?

OSCE-guide-III.doc Page 236 of 255


Psychiatry

Given the benign history with no suspicion of ↑ ICP or focal deficits, and description of headache
consistent with the common tension headache, full neurological examination is not indicated, I
would like to perform a brief neuro screening exam  move on.

Counselling:
- I understand that you are here to renew your Tylenol 3; we will discuss that, but before
that let me ask you: what is your understanding of Tylenol 3?
- Tylenol 3 is a good medication when it is used for particular indication. Do you know
what does it contain? It contains 2 medications:
o One of them is the regular Tylenol as you buy it from the pharmacy
o The other one is codeine
- Tylenol itself is a safe and effective drug, and can be used for long time, however, if there
is no strong indication to use it, it is better to ↓ it as it might cause liver and kidney injury.
- On the other hand, the other medication “codeine” it is a drug belongs to the family we
call “narcotics” which is similar to morphine. It is an excellent pain killer if used for short
term, but, if it is used for long term, this is concerning for us, do you know why?
o First of all, people need to keep increasing the dose in order to obtain the same
effect; we call that “tolerance”.
o Also, if you stop using it suddenly, you will have “withdrawal” symptoms,
similar to that you have now; running nose, tearing, N/V, diarrhoea, drowsiness,
muscle aches, sweats, shaking, and heart racing.
- For these reasons, people get easily hooked on Tylenol 3, and can not stop it. Not only
that, they will need to keep increasing the dose. We call that “a habit forming
medication”.
- If I renew your medication, I will not be helping you, it will be like a vicious circle, and
the more I renew your medication; the more dependent you will be on it; the more you
will need it. For that reason it is not the right step to renew it.
o Can you give me just few pills; I have a very important interview?
o Even if I give you few pills, this is not the solution, this will be temporarily, and
the problem will keep increasing. We must stop the drug
o I can help you with “sick note”
o I can give you another non-narcotic medication that can help you with your pain
- I appreciate your trust to give me all the information, but based on what you described,
you are having “dependence” on narcotics.
- It sounds like you have been going through a lot of stress in your life. I am wondering
that if you would be interested in talking to one of our social works here, who is expert to
find out the community resources for you.
There are also some numbers you can call; they are professionals to help people deal with
medications or drugs. Or if you like, I can refer you to a detoxification center, where they
will help you to quit.

The seeker may be seeking “Fiorinal”


- Fiorinal is a combination preparation of (barbiturate / caffeine / ASA) properly used only for the
relief of occasional tension headaches.
- It is a “habit-forming” medication; that can precipitate withdrawal symptoms: agitation, delirium
and seizures.
- The fact that patient consumes a lot suggests overuse due to dependence.
- The patient may also develop “analgesic headache” syndrome, in which inappropriately used
analgesics actually cause headache.
- Suggest a “drug holiday”, with weaning from caffeine and alcohol, proper sleep hygiene, diet
control, exercise, and stress management.

OSCE-guide-III.doc Page 237 of 255


Psychiatry

Lithium discontinuity

Introduction  Have been diagnosed with bipolar 3 years ago, and would like to
discontinue your medication.
 What is the medication you want to stop?
 Why would you like to stop your medication?
 I am glad you came here to discuss it, any other concerns
Mania History  When were you diagnosed? How?
 Any serious consequences? Were you hospitalized? For how
long?
 Were you seen by a psychiatrist? Regular f/u?
Today Do you feel: DIG FAST (distractibility, impulsiveness, grandiosity,
flight of ideas, activity, sleep, talkative)
Scan for MI PASS ECG
depression
Lithium History  Do you renew your medications on regular basis?
 How much lithium do you take? From the beginning?
 Are you taking it regularly?
 Do you measure lithium level? On regular basis? When was the
last time? What was it? What is your target?
 Are you still taking it? Did you stop?
 How do you feel about lithium?
Side effects  Hypothyroidism: do you have your thyroid hormone measured?
Do you feel cold? Dry skin? Constipation?  give thyroxin
 Diabetes insipidus: do you feel thirsty? Drink more? Pee more?
Got your urine checked? ttt: thiazides
 Abdominal pain? Nausea / vomiting?
 Neuro – shaking/tremors: β-blockers
 Neuro – ataxia/balance/seizure: stop it
MOAPS I know that you have been asked all these questions before, let me ask
it for another time!

Do you feel: DIG FAST (distractibility, impulsiveness (with painful consequences), grandiosity,
flight of ideas, activity, sleep, talkative)
D Do you have a lot of projects? Were you able to finish it to the end? Can you focus on
multiple projects?
I  Are you spending more money than before? Are you borrowing money that you can not
pay back? Are you over-using your credit cards?
 With whom do you live? Many sexual partners?
 SAD: what started 1st; feeling high or talking drugs?
 Have you had problems with the law? Fighting? Arrest? Speeding tickets?
G Do you feel very special? Have special mission?
F Do you feel a lot of thoughts? Ideas?
A How much time do you spend on your projects?
S How many hours do you sleep? Any changes?
T Did anybody mention that you are talking fast?

OSCE-guide-III.doc Page 238 of 255


Psychiatry

Side effects of any medication:


- Liver toxicity
- Nausea, vomiting, diarrhea
- Headache
- Insomnia, irritability
- Agranulocytosis (Carbamazepine): check CBC every week, then every 2 weeks, then
every month

Counselling:
- I understand you are here because you would like to discontinue the lithium, however
before we discuss that; I would like to know your understanding about mania and mood
disorders!
- Mood disorders are common, and the most common of them is depression where people
feel low and do not concentrate and its treatment include the talk therapy and medications
that could be used for 6-12 months and could be stopped if the condition improved and in
some times we need to give the treatment for longer periods of time.
- This is not the case for mania/bipolar. We can treat and control it, but we can not cure it,
may be one day in the future we will be able to do this.
- Your chance of relapse if you stop it is 60% and after the second time this goes up to
80%, and after the third time it goes higher to 90%. You can see it is increasing.
- Based on your lithium level, which is within therapeutic target (0.5 – 1.2), we can
measure it today and we can try to decrease it gradually to check if you are feeling good
and closely monitor you. But you have to promise me that at anytime you feel high mood,
start to spend too much, talk fast or start not to sleep well, you have to come to see me or
go to the nearest ER and inform them.
- Regarding your inability to write, this is not related to lithium, thought block is not a side
effect of lithium. You may try some relaxation techniques to help you concentrate more.

OSCE-guide-III.doc Page 239 of 255


Psychiatry

Manic patient

If the patient is psychotic:


First step is to detect early what is his mood?
- If high mood: manic attack, with psychotic feature
- If no high mood: brief psychotic disorder / schizophreniform

Usually patient brought by police or family member or asked to come by family members
 Patient is talking fast and a lot, laughing, moving around
 Ask whether the patient has been on medication before or not, e.g.: Lithium
 Ask about any side effects of lithium medication N/V / Diarrhea / tremors / polyuria
 Obtain history in the usual format

Introduction
Ask about the Mood
Assure the patient Assure the patient: you are safe here, you are in the hospital and no
one will hurt you
Red flags Fever / headache / nausea & vomiting / head injury
HPI  OCD
 Mania (DIG FAST)
 Depression (MI PASS ECG)
 Suicide (SAD PERSONS)
 If you leave what will happen? What would you like to do?
MOAPS Screen
 SAD: alcohol / substance abuse / amphetamine
 Medical conditions; hyperthyroidism: history of thyroid
problems, symptoms (heart racing, sweating, heat intolerance,
neck swelling, visual field changes)
PMH / FH Psychiatric disease
SH

N.B. if any patient has mood disorder; go through DIG FAST and MI PASS ECG

Management:
 Explain that the patient has recurrence of his mania or bipolar. This is because he stopped
taking the Lithium.
 Will examine and do some tests.
 Will start medication. If Lithium is causing some troubles, we can start another medication.
 Usually you need to admit the patient to control the symptoms of mania (from what you have
told me, you are meeting the criteria of what we call “manic episode” and I have concerns
about your safety).

OSCE-guide-III.doc Page 240 of 255


Psychiatry

Suicidal attempt
LOTS OF EMPATHY
Introduction - … And to see what should be the next step, first, I would like
to know how you feel about being saved.
o If happy, I am glad for that
o No!
Analyze the event - Assure confidentiality
- Can you tell me more about what happened?
- What is the name of the medication? How many tablets? Any
alcohol with it?
- Why did you do that?
- Is it the first time?
- Who saw you and brought you to the hospital?
Before - Assess the plan here, was it organized? Or it was an impulse?
Did you leave a note? Recently, have you been giving your
belongings away?
After - What is going in your mind now?
- If you leave the hospital, what are your plans? Where do you
want to go? What do you want to do?
- If another crisis may happen, are you going to hurt yourself?
Psychiatric assessment - Were you seen by a psychiatrist? Were you given a diagnosis?
Do you see your psychiatrist? Take meds?
Risk - Assess the risk factors: Analyze SAD PERSONS
MOAPS - Screen for anxiety
- Screen for psychosis
- Screen for suicidal / homicidal ideation / self care
- Past medical history / allergy / medications / …
Decision
Conclusion / Counselling
SAD PERSONS
S A D P E R S O N S
Sex Ag Depressio Previou Ethano Rationa Suicid Organize NO Seriou
Mal e n s l l e in d plan suppor s
e > attempts thinkin the t illness
65 g lost family
SAD HEAD PMH
SSS
3-4 Release if enough support
>5 Hospitalize
E - SAD
R - What did you think will achieve by ending your life?
- Sometimes people hear voices asking them to end their life, did you hear this?
N - HEAD SSS
- H: With whom do you live? … Anybody else? … Anybody else? If there is a step-
parent in the image, ask about the relations with him and with other parents. Do you
feel safe at home? Then ask gradually, if there is a chance that this parent might get
angry when he drinks? May shout, may swear at, may push, and may hurt?
S - Past medical history

OSCE-guide-III.doc Page 241 of 255


Psychiatry

Decision:
- If still depressed and/or SAD PERSONS (>3-5)  admit
- It she is ok, regrets the accidents, no SAD PERSONS  release

Conclusion / Counselling:

HOSPITALIZE
- Based on our interview, I have concerns about your safety, because you have more than
THREE risk factors for suicide as per the screening test. Do you mind to stay with us in
the hospital for few days, so we can do the required investigations and start the
medications, until you feel ok, what do you think about that?

RELEASE
- Based on our interview, it is ok if you would like to leave, but you have to arrange a
follow up meeting with your family doctor within 3 days.
- However, I would like you to know that life sometimes could be challenging, and you
may face challenges in the future. It is important that you learn how to deal with
challenges. If you feel over whelmed, talk to somebody, and ask for help
- I can arrange a meeting with a social worker, a psychiatrist!
- I would also like you to promise me that if at any time you want to harm yourself or end
your life, you will seek medical help immediately; you can come to my office or call 911.

If multiple suicidal attempts  borderline personality disorder  do NOT admit

Notes:
- If no eye contact, wasting time, no pt interaction  assure confidentiality
- Whenever you hear “car accident”  show empathy / did you hurt yourself / ask about
who was in the car / was any one injured?
- If the person driving was < 18 and was driving alone  be curious  this must be an
important meeting / person that you really did not want to miss!
- The girl asks you to tell her mother that she crashed mother’s new car! She does not want
to directly (herself) inform the mother!
o I can not do this.
o Why do you think this would help? “She will not be angry”
I see, however, life is full of challenges, it is better that you try to learn how to
deal with challenges yourself.
o We can help you to tell your mother by yourself, we can arrange a meeting with
your mother, I can be present, or we can ask a nurse or a social worker to be
there.
- The girl does not want to inform her parents that she did attempt suicide!
o You assess her and if she is to be released, e.g. she regrets what happened, she is
happy to be saved, no SAD PERSONS risk factors  she is competent 
respect her wishes.

OSCE-guide-III.doc Page 242 of 255


Psychiatry

Eating disorder

Young female, her parents brought her because they have concerns about her weight

Anorexia nervosa Bulimia nervosa


Restrictive Binge-purge
Under weight Average low weight
Distorted self image
Amenorrhea
Disturbed perception  loss of insight  Binge-purge (> 3 times / week) > 3 months
incompetence  inform parents and admit Lose control  over-eat  react (purge)
involuntarily

Introduction Your parents brought you …. How do you feel about that?
I am glad that you came:
- To figure this out (if she is ok)
- To assure your parents (if she is not ok)
Weight analysis
Diet
Exercise
Extra measures
Impact
MOAPS - Mood: scan for depression
- Organic: DM / hyperthyroidism / constitutional symptoms (cancer)
- AP: screen for anxiety / psychosis
- S: HEAD SSS
FH Eating disorder / psychiatric illness / suicide

Weight analysis:
- What is your weight today?
- When did you start to lose weight? What was your weight at that time? How much did
you lose? What was your highest weight? What is your target weight?
- Why are you losing weight?
- Are you losing weight alone? Or someone else is encouraging you?
- When do you look at yourself in the mirror, how do you perceive yourself? How do you
perceive your weight?
- Do you like to dress in baggie?
- It looks like you lost a lot of weight in short period of time; I would like to know how did
you achieve that?

Diet:
Let us talk about your diet;
- How many meals do you eat per day? How about snacks?
o What do you eat in breakfast? How about the amount?
- Do you calculate calories? How much calories do you eat per day?
- Do you eat alone or with other people?
- Do you like to collect recipes? To cook?

OSCE-guide-III.doc Page 243 of 255


Psychiatry

Exercise
- How about exercise? Do you exercise?
- How many times a week?
- Do you dance? Practice any sports?
Extra measures:
- Do you take anything else to help you to lose weight?
- Do you take stool softeners? Do you take water pills?
- Did you try before to induce vomiting?
- Do you sometimes exceed the amount of food you intended to eat? How many times a
week?
- How do you feel after that? How do you compensate?

Impact / consequences:
Because you have lost a lot of weight, I would like to know the impact of this on you!
- Do you have amenorrhea? When was the LMP? Regular?
- Do you feel cold / tired / swelling in your legs?
- Pigmentation on your skin? Fine hair growth? Skin changes?
- Any bony pains? Fractures?
- Muscle cramps? Calf pain?
- Heart racing? Light headedness, dizziness, fainting?
Conclusion:
- I am concerned that you have a condition called “Anorexia Nervosa” (explain)
- It is affecting your body, without treatment it could be fatal
- The treatment is to start eating and to gain weight. It is a tough task but I will refer you to
a multi disciplinary team to start treatment
- Would you like to discuss that with your parents
Management of anorexia nervosa:
- Anorexia patient is to be admitted to hospital if:
o <65% of standard body weight (<85% of standard body weight for adolescents),
o Hypovolemia requiring intravenous fluid,
o Heart rate <40 bpm
o Abnormal serum chemistry or if
o Actively suicidal
- Agree on target weight on admission and reassure this weight will not be surpassed
- Psychotherapy (individual/group/family): addressing food and body perception, coping
mechanisms, health effects
- Monitor for complications of AN
- Monitor for re-feeding syndrome: a potentially life-threatening metabolic response to re-
feeding in severely malnourished patients resulting in severe shifts in fluid and electrolyte
Bulimia nervosa:
- Criteria for admission: significant electrolyte abnormalities
- Treatment: biological (treatment of starvation effects, SSRIs), psychological (cognitive
behavioural therapy, family therapy, recognition of health risks)
Notes: So doctor do you agree with me that I am overweight? Or do you see me like my parents I am not
good?
- I will share your parents concern, it looks like you lost significant weight in short period of time,
and this is concerning.
- If the patient lost interest  slow down  summarize and start again slowly

OSCE-guide-III.doc Page 244 of 255


Psychiatry

Mini-mental status exam:

Introduction: Mr … Now, we will do a mental exercise, in which I am going to ask you some
questions. Some of these questions are easy, and some questions are difficult, please do as much
as you can!
 Prepare this list before you go to the room in cases of delirium / dementia / post-concussion.
Then you can mark the correct or the wrong ones
1 2 3 4 5 5 Orientation to place
6 7 8 9 10 5 Orientation to time

11 12 13 3 3 words recall – immediate


14 d 15 l 16 r 17 o 18 w 5 world – backwards
19 20 21 3 3 words recall – delayed

22 23 24 3 3 steps command
25 26 2 Aphasia (pen / watch)
27 Close your eyes! 1 Read / execute
28 1 Write
29 1 Copy
30 No ifs, ands or buts 1 Repeat

1-5 / Orientation to place: do you know which country we are in? Province? City? Hospital (or
street) name? Which floor (or suit number)?

6-10 / Orientation to time: do you know which year we are in? Season? Month? Day of the
month? Day of the week?

11-13 / 3 words recall – immediate: I am going to tell you 3 objects, and I would like you to
repeat after me and memorize it, and I will ask you about it later! (penny/ tree/ car)

14-18 / Concentration: can you spell the word “world” backwards? He gets -1 for each non-
matching letter (first check if he can spell it correctly forward)

19-21 / 3 words recall – delayed: can you tell me the 3 words that I told you before

22-24 / 3 steps command: give all the instructions at once; are you left or right handed? Can you
please take this paper by the … hand / fold it into halves / give it back to me?

25-26 / Aphasia (pen / watch): what is the name of this? What is this?

27 / Read and execute: can you read this sentence and do what is written in it!
28 / Write: can you write a sentence for me!
29 / Copy: can you copy these two shapes!
30 / Repeat: can you repeat after me; “no ifs, ands, or buts”!

MMS score < 24  incompetent

OSCE-guide-III.doc Page 245 of 255


Psychiatry

Dementia

Difficulty with memory for 6 months

Introduction I would like to ask some questions; then we will do a mental exercise
Analysis of CC Memory assessment
Behavioural  Did anybody tell you that you have changes in your personality? Being short
changes temper? More arguments?
 If there is a fire in this building; what are you going to do?
 How about your sleep? (dementia: fragmented sleep /+/ delirium: reversed
sleep cycle; sleep at day, awake at night)
MMS
Let us take a day of your life; I would like to see how did it affect your life?
DEATH Activities of daily living (ADL)
SHAFT Instrumental Activities of Daily Living (IADL)
MOAPS Organic in details and screen the rest (especially mood for pseudo-dementia)

Memory assessment: Can you tell me more about this difficulty! OCD +
- Any fluctuations in memory level?
- This deterioration is gradual slowly progressive, or is it you feel ok for a while then you
have attack then you are fine then you have another attack? (step ladder)
- Are you having difficulty memorizing numbers?
- Do you have difficulty finding words?
- Do you have difficulty reading? Writing? Calculating?
- Do you lose your stuff?
- Do you make lists to remind you to do things you used to do on regular basis? Do you
have difficulty organizing your schedule?
- Do you have difficulty doing tasks you used to do before; like tying a tie?
- Do you feel difficulty for new events, or old events?
o Recent: What did you have for breakfast? Confirm from partner!
o Remote: Who was the USA president during WWII? (Roosevelt)

ADL – DEATH:
- Dressing: difficulty dressing and undressing yourself?
- Eating: do you remember to get all your meals? Or do you skip meals?
- Ambulatory: do you have difficulty moving around?
- Toileting: how about urination? Have you ever lost control or wet yourself?
- Hygiene: any difficulty having showers?

IADL – SHAFT:
- Shopping: who is responsible for shopping? You or your wife?
- House keeping: how about house keeping, are you able to help your wife?
- Accounting: who is responsible for banking at home?
Did you ever give cheque without balance?
- Food: do you cook? Did you ever forget the stove on?
- Traffic: do you drive? Difficulty driving? Have you ever lost your way?

OSCE-guide-III.doc Page 246 of 255


Psychiatry

MOAPS screening:
Mood:
- Depression – pseudo-dementia?
Organic:
- Do you have nay long term disease? Kidney? Lung? Heart?
- SAD
 History of stroke? Difficult with vision / hearing? Weakness / numbness? Loss of
balance? Urinary incontinence?
 Head trauma? Injury?
 Brain tumour / infection
- Medications? OTC? Sleeping pills?
- Any history of thyroid disease? Symptoms of hypothyroidism?
 Hx of surgeries? In stomach?
 Are you vegetarian? For how long? Do you take supplements?  pernicious anemia

Anxiety
Psychosis
Self care / suicide

Dementia cases:
- 69 years old man comes to your clinic because he is keeping forgetting for the last few
months. In the next 5 minutes; take history and assess (this is too long for 5 minutes, but
during taking history, and if you mention: I would like to do the MMS exam, the
examiner will give you the score)  Alzheimer.
- 55 years old patient comes to your clinic because he has difficulty in memory. His MMS
score is 21. In the next 5 minutes, take history  thyroid.
- 67 years old man, comes to your clinic complaining of difficulty with memory. In the
next 10 minutes take history and assess (make MMS exam)  Dementia.

The cases could be:


- Thyroid disease (especially if pt is younger than 60 years)
- Alzheimer disease
- Dementia
- Depression – pseudo-dementia
- HIV
- Pernicious anemia
- NPH (normal pressure hydrocephalus): if the patient has difficulty in AT of the
“DEATH”; i.e. falls due to ataxia and urinary incontinence

OSCE-guide-III.doc Page 247 of 255


Psychiatry

Delirium
Delirium cases:
- A middle aged gentleman comes to your clinic because his dad is not himself for the last
3 days. Take history by proxy
- A middle aged gentleman comes to your clinic because his mom is in seniors home; they
gave her 15 units of insulin instead of 5 units, and she is not herself. Counsel him!
(insulin induced hypoglycemia  stressful event  decompensate a border line
delirium)
- Patient has surgery 3 days ago, not feeling himself. Patient will be aggressive.
- Patient has surgery 3 days ago, not feeling himself. Patient will keep repeating: “I do not
know”!  mini-mental status exam

Case 1: Dad has not been himself / not sleeping well


Introduction I will ask some questions in order to reach a working plan
Analysis of the CC  How old is he? What are your concerns?
 Tell me more! Any recent stress? OCD
 Did you notice if your dad is angry / aggressive?
 Does it look like your dad is seeing things do not exist? Hearing
voices? Complaining of insects crawling on his skin?
 Does he sleep during night? What a bout during the day?
 Is he eating? Taking care of himself?
 With whom does he live? How is he capable of keeping life? How
does this affect his / their life?
 Is it first time?
Causes  Constitutional symptoms
DD  Any headache / vomiting / neck pain / skin rash / red eyes / any ear
discharge / runny nose / teeth pain / diff swallowing / SOB / cough /
Infection urine changes / abd pain / calf pain / swelling
Trauma  Head trauma? Injury?
Surgery  Recent surgeries? Pain at site of injection? Dressing change?
SAD  SAD
Medications  What about medications, do you have a list with medications? Go one
by one!
- Is he hypertensive? Controlled? Regular measurements?
- I can see that he is diabetic; for how long? Controlled? Regular
f/u and measurements? HbA1c?
- Cholesterol / Water pills / Anti-depressants
- Sleeping pills; if more than 1; ask if it was prescribed by the
same doctor
- Erythromycin!!! Why was he taking it? Pneumonia!
Conclusion It looks like your dad has a medical condition called “delirium” it is a
serious condition. Your dad needs to be seen by a doctor ASAP, can you
bring him to see me. If he is too far, he needs to be taken to the nearest
ER; we will need to decrease or stop some of his medications, and restart
them gradually.

Theophylline (for asthma): stop and take beta 2 agonist instead


Erythromycin (for pneumonia): change the antibiotic
Lorazepam: discontinue

OSCE-guide-III.doc Page 248 of 255


Psychiatry

Case 2: DT
Patient is agitated, delirious and uncooperative
Introduction I can assure you that are safe here, you are in the hospital and no one will
hurt you, we would like to help you
 I can see that you are looking to the wall, do you see anything? Do
you see anything else? Do you hear voices?
 Doctor, do you see the spiders I see? For me, it does not look like
spiders, however, I understand that you can see them at the moment,
but I can assure you that nothing will hurt you!
Analysis of CC  I can see you are scratching; do you feel anything? Do you hear / see
anything?
 Do you think any one would like to hurt you? Assure safety!
 When did that start? OCD?
 How was your sleep?
Full MMS exam
Causes  Constitutional symptoms
DD  Any headache / vomiting / neck pain / skin rash / red eyes / any ear
discharge / runny nose / teeth pain / diff swallowing / SOB / cough /
Infection urine changes / abd pain / calf pain / swelling
Trauma  Head trauma? Injury?
Surgery  Recent surgeries? Pain at site of injection? Dressing change?
SAD  SAD: any shaking / sweating
Medications  What about medications, do you have a list with medications? Any
sleeping pills?
 Do you have nay long term disease? Kidney? Lung? Heart?
Conclusion It looks like you have a medical condition called “delirium” it is a serious
condition. It is reversible, fluctuating, impairment of LOC. It affects 25%
of Hospitalized people.
 Will give medication to help you calm down
 Will have a nurse close by if you need any thing
 Will keep the room quiet and well lit
 Will come back again to see you
Notes:
- It the patient is not cooperative, keeps repeating “I do not know”; start to ask the questions of the
MMS exam, they will go with you. After you finish, you can continue the rest of your exam
- If the patient is starring at the wall; ask him: I can see that you are looking to the wall, do you see
anything there?
- Mental status exam = psychiatric interview
- For delirium; we do the MMS exam daily until he improves
- For dementia; we do the MMS exam every 3-6 months; for follow-up
If confused patient (long case – examination)
 GCS: only if the patient is poorly responsive
 MMS
 Cranial nerves
 Body:
- Pronator drift
- Hoffman’s reflex – thumb flexion  UMNL
- Cerebellar tests: finger to nose, rapid alternating movements
- Power / sensation / reflexes
 Patient standing: gait, Romberg test, planter flexion power
 Patient supine: tone

OSCE-guide-III.doc Page 249 of 255


Psychiatry

Smoking Cessation – counselling

1- Congratulations, … We will speak in details about how we can work together to achieve this
healthy goal, but first let me ask you some questions, I need to have the bigger picture about your
smoking, and this will help us to figure out the best plan to achieve our goal
2- Smoking history:
 When did you start smoking? For how many years?
 How many cigarettes per day?
3- Reasons (motivations): to seek smoking cessation
4- Previous attempts: How many times? Why did you fail? When was the last time?
EMPATHY: “failure” is a normal part of trying to stop
5- Is there any other smoker in your home? Is she/he willing to quit? It will be a great idea if
both of you tried to quit at the same time, this will increase the success rate of your trial.
If she/he would like to know more information or need help, I will be more than happy to
meet her/him, we can arrange a meeting
6- Impact (complications of smoking):
 Cancer (lung – hemoptysis, tongue, nasopharynx, urinary bladder, other cancers)
 Cardio vascular hazards (myocardial ischemia)
7- Red flags:
 Constitutional symptoms
 Risk factors (personal history or family history) of:
 Heart disease / attack / HTN
 Diabetes mellitus / hyper-cholesterolemia
8- Plan:
 STAR:
i. Set a quit date, print papers with this date and stick it under your vision
so that you see it frequently during the day
ii. Tell your family, friends, they will be your support
iii. Anticipate the challenges you will face (nicotine-withdrawal effects:
headache, nausea and a craving for tobacco, insomnia, irritability,
anxiety, and weight gain)
iv. Remove cigarettes and other tobacco products (e.g. ashtrays) from your
home, car, and work
 Nicotine Replacement Therapy:
i. Nicotine patch [21 mg (if smoking > 25 cig/day), 14 mg, 7 mg]
ii. Nicotine gums
iii. Nicotine inhaler
 Psychological support for smoking cessation (to ↓ the craving):
i. Zyban (Bupropion):
+ used with tapering smoking for 2 weeks, then stop smoking
+ 150 mg qAM x 3 days then 150 mg bid x 3 months
+ Contra-indications: epilepsy, seizure disorder, eating disorders, patients
undergoing abrupt discontinuation of ethanol or sedatives
ii. Champix (Varenicline): ↓ urge to smoke and ↓ withdrawal symptoms
+ 0.5 mg qAM x 3 d then 0.5 mg bid x 4 d then 1 mg bid x 3 months
 Investigations:
i. CBC / urinalysis / lipid profile
ii. If there is risk factors for heart diseases: stress ECG test
iii. If patient is worried, or if there is hemoptysis: chest x-ray

OSCE-guide-III.doc Page 250 of 255


Psychiatry

Refusal to treatment – counselling


Mrs … 56 yrs old, was recently diagnosed with lung cancer, counsel her.
Introduction - I understand you were diagnosed recently with lung cancer
- How are you coping?
History Brief history - SAD
- Family history of lung cancers
Support - With whom do you live?
- Any family support?
- How do you support yourself financially?
Lung cancer - What is your understanding of … (lung cancer)?
- Do you know which type you have?
- Available treatment
- Now, I would like to explain the treatment options we have,
- Once diagnosed, usually surgery is late to be done, so we have radio / chemo
- Based on your condition and stage of cancer, the surgeon thinks that “chemo”
and/or “radio” therapy are the best line of treatment for you, this is based on
the many clinical trials and evidence-based medicine.
Treatment - No doctor, I do not want to contaminate my body with chemicals, I am going
refusal for spiritual therapy! OR whatever cause!
Ask for reason: Why? How long have you been thinking that way? Did you talk
about this with your family?
Assess competency; rule out depression:
- When we talk about decision-making, we need to rule out certain conditions
and to know more about your health. Can you please tell me how has your
mood been lately? Do you find yourself cry easily? Interests? Suicidal
ideation? Any major event, death, accident in the family recently? MI PASS
ECG …
Counsel If depressed  assess suicide and psychiatric consult
- You sound depressed to me, I would love to respect your wish, but I would be
more comfortable to follow that after you talk to psychiatrist, do you want to
do arrange that?
- Give it more thoughts; I will arrange the family meeting, social worker, and
psychiatrist. I will come back to talk to you again tomorrow
In not depressed  she is competent, she can refuse treatment
- After all this is your decision; I just want to make sure you know the available
treatments that were proved to be beneficial in treating lung cancer.
- Why not to try both? We start the medical therapy that we are sure it works,
and you go for spiritual therapy!
- Explain the condition, the available treatments
- The side effects of treatments and the complications of not getting treatment
(terminal illness)
- What about arranging a meeting with some one who has had the same medical
condition, and speak with him/her. You will get better insight into the disease
and you will see the results of treatment.
- How about arranging a meeting for you and your family members (if you wish)
with the surgeon, so that he can explain condition in more details?
- A 71 years old with cancer colon, patient concerns: afraid of living with colostomy / concerned about
being a burden to the family / afraid of complications of the surgery (a friend died in similar surgery)
- A 70 years old lady with lung cancer, refusing the surgery because her husband died 30 years ago in a
surgery (she does not know why) / she thinks that people die from the anesthesia

OSCE-guide-III.doc Page 251 of 255


Psychiatry

Truth telling

Usually a son or daughter asking you not to inform the patient (parent / grandparent) about his
terminal illness or advanced condition

Introduction:
- Well, it is not unusual for families to have that request!
- Why you do not what her/him to know? (can’t handle the bad news, fragile personality,
depression, …)
- Does the patient have advanced directive? Will? Have discussed this before?

Explain patient must be told:


- Well, you know your loved one the best, given her/his previous reaction, it is reasonable
to have that concern.
- On the other hand, the patient has the right to know, we can not hold this right, besides,
usually; people will go through different stages when they react to bad news, we have
specialists here who can really help the patient and family to go through these stages.

Explain the reasons to tell:


- Patient has the right to know
- Patient will have suspicion about his own condition
- We need to discuss the treatment options, and it is the decision on the patient
- Patient may need to start some arrangements

Explain the implications not to tell:


- It is difficult to hide, it is a team work, will eventually know or find out
- Patient will lose the trust to doctors in general

Decision:
- Will talk to the patient to see if she/he wants to know all the details or not?!!
o If yes, we have to tell her/him
o If no, we will ask if she/he would like us to inform someone else
- In all cases, if the patient asks, we have to tell her/him

Conclusion:
- I can tell that the patient has a very caring family, it must be very hard on the family as
well, if they need someone to talk to cope, I can arrange that if they want
- I can give the family a little bit more time to think and we will talk again, patient will
eventually need to know the truth.

OSCE-guide-III.doc Page 252 of 255


Psychiatry

Organ Donation

Explain resuscitation effort


- What has been done to the patient “resuscitation”
- When he came in, he was not breathing, we put down a tube, his heart was not beating,
we did the compression
- What is the outcome, patient is not responding to treatment due to the severe trauma to
the head.

Explain the condition: brain death


- Patient is in a state called the brain death, (check with the relatives if they know this
terminology)
o Irreversible brain damage, no functioning at all
o Not responding to light, to pain
o No spontaneous breathing, will never gain consciousness, when we stop the
machine, he does not breathe
o Legal term for death.
- Confirmed by two nerve specialists: neurologist / neurosurgeon:
- Show empathy: sorry for the loss, patient was young / healthy / family needs him / …
Give time to family to recall how active and how nice he was

Bring the issue of organ donation:


- He was healthy, good candidate for organ donation
- If he had advanced will, driver license
- What family’s view about this
- It is family’s decision now, the decision is a “life gift”

Explain how to do it
- We have a team to do that, they will respond very quickly
- Many organs can be used
- There is time limit; decision should be made within the next 24 hrs
- You will be notified which organ used and where to go, but you won’t get the
individual’s name

Explain funeral
- It won’t affect the arrangement for funeral
- Still can have the open casket, won’t affect the face

Address any questions or concerns

OSCE-guide-III.doc Page 253 of 255


Psychiatry

OCD

- Address body language


- Why do you wear gloves and hat? And refuse hand shaking?
- Start with an open ended question; OCD / trigger / …

 Obsessions:
- Type of obsession: dirt and contamination, orderliness, religious, checking and
rechecking?
- Do you feel that these obsessions are not real?
- Do you want to get rid of them?
- What do you do to overcome the stress created by these ideas?
- How many times do you wash your hands? How long do you take in a shower?
- Impact on life, work,

 MOAPS:
- Screen for mood disorders
- Screen for organic causes
- Screen for other types of anxiety disorder,
- Screen for psychosis
- Screen for suicide, homicide, self care

 Past medical history / medication & allergies


 Family history
 Social Hx: smoking, alcohol, drugs, work, home, support

OSCE-guide-III.doc Page 254 of 255


Psychiatry

NOTES

OSCE-guide-III.doc Page 255 of 255


8 CASES FOR THE NAC OSCE
www.crackthenac.com
8 CASES FOR THE NAC OSCE

Thanks for downloading our free ebook to help you prepare for the NAC OSCE. We definitely appreciate
how stressful preparing for this exam can be and how important a top score is to your residency
applications and eventual practice in Canada. We also understand how there is virtually 0 information
on this exam out there and how secretive the MCC is with any of their exams.

We’ve assembled 8 cases to help get you started and begin your prep for the exam. There are no right
answers to these cases; they are simply opening paragraphs, similar to ones you may see in the NAC
OSCE. These should be used as a guide to help you think critically, act, and work your way towards total
preparation. We suggest you read the opening statement and then explore the checklists. Consider what
you would ask, what you would do, and how you would proceed with your exam. Think about common
issues, and the most likely answers to the examiner questions. As well we’ve added some study tips and
suggestions at the end of each case.

One final piece of advice: practice, practice, practice, grab a friend, a sibling, a colleague and rehearse
every physical exam, rehearse every algorithm for questions, and know your public health and medico-
legal guidelines inside and out.

2
www.crackthenac.com
8 CASES FOR THE NAC OSCE

Case 1:

Directions: Rebecca Johnston, A 27-year old woman comes into the office complaining of chest pain.
Obtain a complete history of this complaint. Do not perform a physical exam.

 Introduces self appropriately


 Washes hands
 Clarifies reason for visit
 Obtains history of chest pain
o Onset
o Location
o Precipitating factors
o Alleviating factors
o Associated symptoms
o Quality
o Radiation
o Severity
o Timing/duration
 Identifies risk factors for heart disease
o Past medical history
o Family history of heart disease or risk factors
o Smoking history
o Illicit Drug use (especially cocaine)
o Hypertension
o Lipids/cholesterol
o Recent stressors
o Exercise tolerance
 Focused review of systems
o Heartburn/GERD symptoms
o Pain with movement/palpation
o Medications
o Allergies
o Summarizes history
o Checks for any other concerns or missed information

Possible examiner questions:

 What tests would you like to order?


 What is your most likely diagnosis? Give up to 2 differential diagnoses
 Please provide the patient with advice on managing “panic attacks”

3
www.crackthenac.com
8 CASES FOR THE NAC OSCE

Tips

1. Always rule out the most serious causes first.


2. Use clear open ended questions.
3. Write down the most common Dx on your notepad as you read the case.
4. Think about age appropriate diagnoses – is it likely that a 27 year old woman is having an MI? Or
is it more likely anxiety? How will this influence your choice of tests? What tests would you
order if she were 67? Would these tests change if she had a history of trauma?
5. Prepare to provide counselling to the patient. They are testing to see how you can not only
diagnose, but communicate common and important health issues in a real setting.

4
www.crackthenac.com
8 CASES FOR THE NAC OSCE

Case 2

Directions: Mr. Lee Vasan a 57 year old male reports to outpatients with a 48 hour history of abdominal
discomfort. He reports he has not passed gas or stool for at least a day. He has experienced 1 episode of
vomiting. Please perform a focused physical exam.

 Introduces self appropriately


 Washes hands
 Clarifies reason for visit
 Obtains consent for personal exposure
 Exposes abdomen
 Drapes appropriately
 Requests Vital signs
 Inspection
o Looks for peristaltic waves
o Looks for distension
o Looks for bruising, scars, mass
 Auscultation
o Listens for bowel sounds
o Listens for bruits
 Percussion
o Percusses abdomen
o Percusses liver
o Percusses spleen
o Comments on tenderness
 Palpation
o Light palpation
o Deep palpation
o Requests Digital rectal exam
o Requests external genital exam
o Palpates for
 Appendicitis
 Palpates for AAA
 Hepatomegaly
 Makes appropriate closing remarks
 Examiner question:
o What tests would you like to perform
o Based on attached image (Case courtesy of Dr Bruno Di Muzio, Radiopaedia.org, rID:
17788)

5
www.crackthenac.com
8 CASES FOR THE NAC OSCE

 Provide a diagnosis and up to 2 differential diagnoses.


 What is your next course of action?

TIPS

1. Be prepared to see simple X-rays. It is important to be familiar with classic head, cervical, spinal,
lung, abdomen, and limb fractures. Images will be simple and straight forward and should be
easy to spot by any future resident.
2. Know all signs and symptoms for classic and common abdominal/hepatic/renal presentations.
The exam will not feature anything weird and wonderful, but you will be penalized for missing
murphy’s sign, jaundice, etc.
3. Comment on anything and everything you see.
4. Comment on what you are looking for, what you see, and what that might mean. They want to
understand your thinking process. Are you fit for practice in Canada?

6
www.crackthenac.com
8 CASES FOR THE NAC OSCE

Case 3:

A 16 year old boy with a history of asthma presents to his family doctor’s office complaining of recurrent
shortness of breath since moving in with his father. (His parents are separated). Obtain a detailed
history and explore possible causes.

 Introduces self to patient


 Washes Hands
 Establishes reason for visit
 History
o Character
o Onset
o Duration
o Frequency
o What makes it worse? What makes it better?
o Current medications
 Asks about
o Smoking
o Pain
o Exposure to chemicals/irritants
o Exposure to pets/birds
o Nocturnal cough
o Exercise Tolerance
o Shortness of breath
o Snoring
o Carpeting
o Fireplace
 Previous Medical History
 Allergies
 Social History
o Alcohol
o Tobacco
o Marijuana
o Illicit drugs
 Family History

7
www.crackthenac.com
8 CASES FOR THE NAC OSCE

Examiner Questions:

Please explain to the patient how to use an aerochamber inhaler?


What tests would you like to perform?
What is the initial treatment for asthma? What other medication can be added to this treatment if
symptoms persist?
Please explain to the patient common asthma triggers.

TIPS

1. It is important to be well versed on common patient education issues including but not limited
to: using an aerochamber, checking blood sugar, injecting insulin, various forms of
contraception, etc. Don’t be surprised if you are asked to counsel a patient on these topics.
2. Know a variety of go to tests for common presentations – asthma, chest pain, abdominal pain,
lower back pain, when to order a pregnancy test, urine dip, swab, etc. You may be asked to
refer a patient or to request testing- verbalize all of these during your case.
3. Be aware of basic algorithms for treatment of things like asthma, diabetes, hypertension, it is
unlikely that you will be asked to choose a specific dose or a frequency, but knowing that X is
first line, Y is added to first line, and Z is third in line for treatment is important for both the NAC
OSCE and clinical practice.
4. Practice counselling individuals on a variety of lifestyle treatments – diet, exercise, prevention,
etc. You will be asked to do this during the exam.

8
www.crackthenac.com
8 CASES FOR THE NAC OSCE

Case 4:

A 21 Month old female is brought to the clinic by her parents as they are complaining that she has been
having loose stools. Please obtain a focused history from her mother while the nurse evaluates the
child’s vital signs.

 Introduces self to parents


 Establishes reason for visit
 Washes hands
 Relevant history
 Asks about presence of
o Blood
o Oily
o Pus
o Mucus
o Colour/Consistency
o Fowl smelling
o Frequency
o Constipation
 Duration
 Onset
 Associated Symptoms ex: vomiting
 Aggravating/Mitigating factors
 Fever
 Diet
 Weight loss/gain
 Travel
 Vaccinations
 Previous Medical History
 Family Medical History
 Medications
 Allergies
 Prenatal Care
 Postnatal Care
 Duration and complications related to pregnancy

9
www.crackthenac.com
8 CASES FOR THE NAC OSCE

Examiner questions:

Please list 1 primary diagnosis and 2 secondary diagnoses.


How would you diagnose Cystic fibrosis in this patient?
How would you assess this child for dehydration?
Assuming all of her vaccinations are up to date, what vaccines should she receive at her next visit? When
would you schedule this visit?

TIPS

1. While it is very unlikely you will see a paediatric patient in the NAC OSCE as they are difficult to
standardize for a national exam, it is likely that you will see the parent/caretaker of a child and
be asked to obtain a history.
2. Being able to take a paediatric history for head to toe is vital to obtain a top mark in this exam.
What illnesses are specific to children? What allergies are most common in children? What
social issues are involved?
3. Know the social and legal issues for paediatric patients inside and out.
4. What is the age of consent in Canada? How does this affect your steps in management?
Consider what happens if a pre-teen/teen requests to not involve their parents?
5. What are the current guidelines for vaccines in Canada? Ages? Schedule? What happens with a
missed vaccine? What happens if a child has a fever or is unwell at a scheduled vaccination
appointment?

10
www.crackthenac.com
8 CASES FOR THE NAC OSCE

Case 5

Jennifer Stevens is a 17 year old female that presents to the clinic for personal reasons. Please
determine the reason for her visit and conduct a focused history.

 Introduces self to patient


 Establishes reason for visit – foul smelling vaginal discharge
 Washes hands
 Enquires about
o Onset
o Duration
o Character
o Sexual history
 Rape? Abuse?
o Sexual orientation
o Contraception and barrier protection
o Obstetric history
o Menstrual history
o Abnormal bleeding
o Urinary symptoms
 Previous medical history
o Previous STI/Pregnancy/ectopic
o Surgery
 Medications
o Oral ABX
o Oral Contraception
 Social History
o Smoking
o Alcohol
o Drugs
o Work life
o School life
o Who is she at home with?

11
www.crackthenac.com
8 CASES FOR THE NAC OSCE

Examiner questions:

Please provide 3 possible diagnoses?


Please specify 3 tests you would like to perform?
If a vagina swab is positive for chlamydia, what is your next course of action?
Please counsel the patient on prevention of the spread of sexually transmitted infections.

TIPS

1. Social cases involving youth/young adults are common in nearly all 4th year OSCEs.
2. What diseases must be reported to public health?
3. Does the partner have to be notified?
4. What are the laws surrounding HIV?
5. What if the individual reports sexual abuse? Rape? Sex with a minor/adult?
6. What is the age of consent in Canada?
7. What if they ask you to not inform their parents?
8. What are the laws surrounding abortion?

12
www.crackthenac.com
8 CASES FOR THE NAC OSCE

Case 6

Mr. Samuel Jones is a 63 year old man with a 20 year history of diabetes he presents to your clinic today
for his annual diabetic check-up. Please perform a physical exam. Do not perform funduscopic exam.

 Introduces self to patient


 Washes hands
 Explains physical exam
 Examines Foot
o Comments on
 Colour
 Skin
 Ulcers
 Swelling
 Calluses
 Deformities
o Palpation
 Temperature
 Capillary refill
 Pulses
 Dorsalis pedis
 Posterior tibial
o Sensation
 Correctly uses monofilament
 Asks patient to close eyes
 Hallux
 Metatarsal heads
 Places tuning fork on distal phalanx of great toe
o Completes exam on both feet
o Examines proprioception of great toe
o Ankle jerk reflex
 Closes interview

13
www.crackthenac.com
8 CASES FOR THE NAC OSCE

Examiner questions

Please advise the patient on proper foot care.


List 3 complications of diabetes.
What 3 tests would you like to perform to conclude this annual checkup?

Tips

1. Be prepared for classic presentations in classic patients. These cases are developed by
physicians for future physicians. You will not be asked to examine something weird and unique.
Cases will present like a normal day in a family practice so you should be able to examine and
diagnose classic presentations.
2. There are no tricks, if the instructions ask you to perform a foot exam, that’s all you are required
to do. There will be no secret malignancy lurking in the case. No secret social issues hidden in a
foot exam.
3. Re-read the instructions if you get lost. They are generally very clear and concise. They will
instruct you what to perform, and what not to perform.
4. “I’m looking for, I’m palpating for, I’m listening for…” these phrases are your friends for physical
exam stations, talk, talk, and talk more so you leave no doubt in the examiner’s mind that you
know your stuff and you know what you are doing and why.

14
www.crackthenac.com
8 CASES FOR THE NAC OSCE

Case 7

Mrs. Abigail Johnson is brought into to your clinic by her 30 year old son for a renewal of her narcotics
prescription. Please explore the reasons for her visit and address any concerns she may have. Do not
perform a physical exam.

 Introduces self to patient


 Washes hands
 Establishes reason for visit
 Establishes reason for narcotics prescription
 Assesses Pain
o Location
o Onset
o Duration
o Intensity
o Provocative/Palliative factors
 Assesses narcotic use
o How often does she take the pills
o Does she take any other pain medication
o Sleep quality
o Does anyone else have access to her medication
o When was her last renewal
o How many pills does she take at a time/per day
 Establishes patient has run out of pills early
 Establishes that she feels pressured to get more pills
 Establishes that her pills are going missing
 Establishes that she may feel threatened if she does not get more pills

15
www.crackthenac.com
8 CASES FOR THE NAC OSCE

Examiner questions:

What is your diagnosis?


What is your next course of action?
If this patient’s son is diverting her narcotics how would you manage her pain?

TIPS

1. The NAC will assess more than your clinical knowledge it will test you on social and legal issues.
2. Be prepared for a variety of social and legal situations. What would you do if you suspected
elder abuse? What is the next course of action if her medication is being misused or sold?
3. Does she really need this level of treatment?
4. This is a common clinical scenario and it is important to know how to effectively manage these
tough presentations.

16
www.crackthenac.com
8 CASES FOR THE NAC OSCE

Case 8

Mrs. Gladys Mortimer, a 72 year old female presents to your clinic with recurrent “accidents” resulting
in her needing to change her underwear and pants more than once a day. As a result she is now
reluctant to leave the house. Please explore the reason for her visit and conduct a focused history.

 Introduces self to patient


 Washes hands
 Establishes reason for visit
 History of Present Complaint
o Character
o Onset
o Intensity
o Duration
o Aggravating/Mitigating factors
 Asks about
o Pain
o Fullness
o Difficulty voiding
o Blood in urine
o Fever/chills
o Weight loss
o Kidney stones
 Previous Medical History
o Malignancy
o Obstetric History
o Surgery
o Renal history
o Bladder History
o Diabetes
 MEDS
o Anticholinergic
o Diuretics
 Social History
o Smoking
o Alcohol
o Drugs
o Sex history
o Activities of daily living
o Support at home

17
www.crackthenac.com
8 CASES FOR THE NAC OSCE

Examiner questions:

What is your primary diagnosis, list 2 alternative diagnoses?


What tests would you order for this patient?

TIPS

1. Paediatrics, geriatrics, and social issues are often tested on this exam. Be well versed in common
presentations and course of action.
2. Demonstrate compassion and understanding, this will count for marks on the exam.
3. Use clear concise language; do not use jargon or difficult terms. This exam is equally as much a
test of your communication skills as it is of your knowledge. You can make up for shortcomings
of knowledge with communication, but you cannot make up for poor communication with
knowing everything.

18
www.crackthenac.com
8 CASES FOR THE NAC OSCE

19
www.crackthenac.com

You might also like